You are on page 1of 563

SYLLABUS

Central Board of Secondary Education, New Delhi


MATHEMATICS Class–XII
(2021-22)

Time: 3 hours One Paper Max. marks: 80

S. No. Units No. of Periods Marks

I. RELATIONS AND FUNCTIONS 30 08

II. ALGEBRA 50 10

III. CALCULUS 80 35

IV. VECTORS AND THREE-DIMENSIONAL GEOMETRY 30 14

V. LINEAR PROGRAMMING 20 05

VI. PROBABILITY 30 08

Total 240 80

Internal Assessment 20

UNIT-I: RELATIONS AND FUNCTIONS

1. Relations and Functions (15 Periods)


Types of relations: reflexive, symmetric, transitive and equivalence relations. One to one and onto
functions, composite functions, inverse of a function.

2. Inverse Trigonometric Functions (15 Periods)


Definition, range, domain, principal value branch. Graphs of inverse trigonometric functions.
Elementary properties of inverse trigonometric functions.

UNIT-II: ALGEBRA
1. Matrices (25 Periods)
Concept, notation, order, equality, types of matrices, zero and identity matrix, transpose of
a matrix, symmetric and skew symmetric matrices. Operation on matrices: Addition and
multiplication and multiplication with a scalar. Simple properties of addition, multiplication
and scalar multiplication. Non-commutativity of multiplication of matrices and existence of non-
zero matrices whose product is the zero matrix (restrict to square matrices of order 2). Concept
of elementary row and column operations. Invertible matrices and proof of the uniqueness of
inverse, if it exists; (Here all matrices will have real entries).

@Cbsebookshub - Join Us on Telegram


2. Determinants (25 Periods)
Determinant of a square matrix (up to 3 × 3 matrices), properties of determinants, minors,
co-factors and applications of determinants in finding the area of a triangle. Adjoint and inverse
of a square matrix. Consistency, inconsistency and number of solutions of system of linear
equations by examples, solving system of linear equations in two or three variables (having
unique solution) using inverse of a matrix.

UNIT-III: CALCULUS
1. Continuity and Differentiability (20 Periods)
Continuity and differentiability, derivative of composite functions, chain rule, derivatives of
inverse trigonometric functions, derivative of implicit functions. Concept of exponential and
logarithmic functions.
Derivatives of logarithmic and exponential functions. Logarithmic differentiation, derivative of
functions expressed in parametric forms. Second order derivatives. Rolle’s and Lagrange’s Mean
Value Theorems (without proof) and their geometric interpretation.
2. Applications of Derivatives (10 Periods)
Applications of derivatives: rate of change of bodies, increasing/decreasing functions, tangents
and normals, use of derivatives in approximation, maxima and minima (first derivative test
motivated geometrically and second derivative test given as a provable tool). Simple problems
(that illustrate basic principles and understanding of the subject as well as real-life situations).
3. Integrals (20 Periods)
Integration as inverse process of differentiation. Integration of a variety of functions by
substitution, by partial fractions and by parts. Evaluation of simple integrals of the following
types and problems based on them.
dx dx dx dx dx
y
, y , y , y , y ,
x ! a2
2 2
x !a 2 2
a –x 2 2
ax bx + c
+ 2
ax + bx + c
px + q px + q
y
2
dx, y dx, y a 2 ! x 2 dx, y x 2 – a 2 dx
ax + bx + c ax 2 + bx + c
y ax 2 + bx + c dx, y (px + q) ax 2 + bx + c dx

Definite integrals as a limit of a sum, Fundamental Theorem of Calculus (without proof). Basic
properties of definite integrals and evaluation of definite integrals.

4. Applications of the Integrals (15 Periods)


Applications in finding the area under simple curves, especially lines, circles/parabolas/ellipses
(in standard form only). Area between any of the two above said curves (the region should be
clearly identifiable).

5. Differential Equations (15 Periods)


Definition, order and degree, general and particular solutions of a differential equation. Formation
of differential equation whose general solution is given. Solution of differential equations by
method of separation of variables, solutions of homogeneous differential equations of first order
and first degree. Solutions of linear differential equation of the type:
dy
+ py = q, where p and q are functions of x or constants
dx
dx
+ px = q, where p and q are functions of y or constants
dy

@Cbsebookshub - Join Us on Telegram


UNIT-IV: VECTORS AND THREE-DIMENSIONAL GEOMETRY

1. Vectors (15 Periods)


Vectors and scalars, magnitude and direction of a vector. Direction cosines and direction ratios
of a vector. Types of vectors (equal, unit, zero, parallel and collinear vectors), position vector
of a point, negative of a vector, components of a vector, addition of vectors, multiplication of a
vector by a scalar, position vector of a point dividing a line segment in a given ratio. Definition,
geometrical interpretation, properties and applications of scalar (dot) product of vectors, vector
(cross) product of vectors, scalar triple product of vectors.

2. Three-dimensional Geometry (15 Periods)


Direction cosines and direction ratios of a line joining two points. Cartesian equation and vector
equation of a line, coplanar and skew lines, shortest distance between two lines. Cartesian and
vector equation of a plane. Angle between (i) two lines, (ii) two planes, (iii) a line and a plane.
Distance of a point from a plane.

UNIT-V: LINEAR PROGRAMMING


1. Linear Programming (20 Periods)
Introduction, related terminology such as constraints, objective function, optimization, different
types of linear programming (L.P.) problems, mathematical formulation of L.P. problems,
graphical method of solution for problems in two variables, feasible and infeasible regions
(bounded and unbounded), feasible and infeasible solutions, optimal feasible solutions (up to
three non-trivial constraints).

UNIT-VI: PROBABILITY
1. Probability (30 Periods)
Conditional probability, multiplication theorem on probability, independent events, total
probability, Bayes’ theorem, Random variable and its probability distribution, mean and variance
of random variable. Binomial probability distribution.

The changes for classes XI-XII (2021-22) internal year-end/Board Examination are as under:

Classes XI-XII

Year-end
(2020-21) (2021-22)
Examination/Board
Existing Modified
Examination (Theory)

 Objective type Questions  Competency Based Questions will


including Multiple Choice be 20%
Question-20%   These can be in the form of Multiple-
 Case-based/Source- based Choice Questions, Case- Based
Integrated Questions-10% Questions, Source Based Integrated
Composition  Short Answer/ Long Answer Questions or any other types
Questions- Remaining 70%  Objective Questions will be 20 %
 Remaining 60% Short Answer/
Long Answer Questions- (as per
existing pattern)

@Cbsebookshub - Join Us on Telegram


Design of Question Paper
Mathematics Code No. 041
Class XII (2021–22)
Time: 3 hours Max. Marks: 80
%
S. No. Typology of Questions Total Marks
Weightage

Remembering:
Exhibit memory of previously learned material by
recalling facts, terms, basic concepts, and answers.
1. Understanding: 44 55
Demonstrate understanding of facts and ideas by
organizing, comparing, translating, interpreting, giving
descriptions, and stating main ideas

Applying:
Solve problems to new situations by applying acquired
2. 20 25
knowledge, facts, techniques and rules in a different
way.

Analysing :
Examine and break information into parts by
identifying motives or causes. Make inferences and
find evidence to support generalizations
Evaluating:
Present and defend opinions by making judgments
3. about information, validity of ideas, or quality of work 16 20
based on a set of criteria.
Creating:
Compile information together in a different way by
combining elements in a new pattern or proposing
alternative solutions

TOTAL 80 100

1. No chapter wise weightage. Care to be taken to cover all the chapters.


2. Suitable internal variations may be made for generating various templates keeping the overall weightage to
different form of questions and typology of questions same.

@Cbsebookshub - Join Us on Telegram


Choice (s):
There will be no overall choice in the question paper.
However, 33% internal choices will be given.

INTERNAL ASSESSMENT 20 MARKS

Periodic Tests ( Best 2 Out of 3 tests conducted) 10 Marks

Mathematics Activities 10 Marks


Note: For activities NCERT Lab Manual may be referred

Conduct of Periodic Tests:


Periodic Test is a Pen and Paper assessment which is to be conducted by the respective subject
teacher. The format of periodic test must have questions items with a balance mix, such as, very
short answer (VSA), short answer (SA) and long answer (LA) to effectively assess the knowledge,
understanding, application, skills, analysis, evaluation and synthesis. Depending on the nature
of subject the subject teacher will have the liberty of incorporating any other types of questions
too. The modalities of the PT are as follows:

(a) Mode: The periodic test is to be taken in the form of pen-papertest.

(b) Schedule: In the entire Academic Year, three Periodic Tests in each subject may be conducted
as follows:

Test Pre Mid-term (PT-I) Mid-Term (PT-II) Post Mid-Term (PT-III)

Tentative Month July-August November December-January

This is only a suggestive schedule and schools may conduct periodic tests as per their
convenience. The winter bound schools would develop their own schedule with similar time
gaps between two consecutive tests.
(c) Average of Marks: Once schools complete the conduct of all the three periodic tests, they will
convert the weightage of each of the three tests into ten marks each for identifying best two
tests. The best two will be taken into consideration and the average of the two shall be taken
as the final marks for PT.

(d) The school will ensure simple documentation to keep a record of performance.
(e) Sharing of Feedback/Performance: The students’ achievement in each test must be shared
with the students and their parents to give them an overview of the level of learning that
has taken place during different periods. Feedback will help parents formulate interventions
(conducive ambience, support materials, motivation and morale-boosting) to further enhance
learning. A teacher, while sharing the feedback with student or parent, should be empathetic,
non- judgmental and motivating. It is recommended that the teacher share best examples/
performances with the class to motivate all learners.

@Cbsebookshub - Join Us on Telegram


Assessment of Activity Work:
Throughout the year any 10 activities shall be performed by the student from the activities given
in the NCERT Laboratory Manual for the respective class (XI or XII) which is available on the link
http://www.ncert.nic.in/exemplar/labmanuals.html. A record of the same may be kept by the
student. An year end test on the activity may be conducted at the School Level.

The weightage are as under:
OO The activities performed by the student through out the year and record keeping : 5 marks
OO Assessment of the activity performed during the year end test: 3 marks
OO Viva-voce : 2 marks

@Cbsebookshub - Join Us on Telegram


PART–A
UU Basic Concepts
UU Selected NCERT QUESTIONS
UU Multiple Choice Questions
UU Fill in the Blanks
UU Very Short Answer Questions
UU Short Answer Questions-I
UU Short Answer Questions-II
UU Long Answer Questions
UU Proficiency Exercise
UU Self-Assessment Test

@Cbsebookshub - Join Us on Telegram


@Cbsebookshub - Join Us on Telegram
Relations and 1
Functions

1. Relation: If A and B are two non-empty sets, then any subset R of A × B is called relation from set A
to set B.
i.e., R : A → B ⇔ R ⊆ A × B

For example:
Let A = {1, 2}, B = {3, 4}
Then A × B = {(1, 3), (1, 4), (2, 3), (2, 4)}
A subset R1 = {(1, 3), (2, 4)} ⊆ A × B is called relation from A to B.
Similarly, other subsets of A × B are also relation from A to B.
If (x, y) ∈ R, then we write x R y (read as x is R related to y) and if (x, y) ∉ R, then we write x R y (read
as x is not R related to y).
2. Domain and Range of a Relation: If R is any relation from set A to set B then,
(a) Domain of R is the set of all first coordinates of elements of R and it is denoted by Dom (R).
(b) Range of R is the set of all second coordinates of R and it is denoted by Range (R).
A relation R on set A means, the relation from A to A i.e., R ⊆ A × A.

3. Some Standard Types of Relations:
Let A be a non-empty set. Then, a relation R on set A is said to be
(a) Reflexive: If (x, x) ∈ R for each element x ∈ A, i.e., if xRx for each element x ∈ A.
(b) Symmetric: If (x, y) ∈ R ⇒ (y, x) ∈ R for all x, y ∈ A, i.e., if xRy ⇒ yRx for all x, y ∈ A.
(c) Transitive: If (x, y) ∈ R and (y, z) ∈ R ⇒ (x, z) ∈ R for all x, y, z ∈ A, i.e., if xRy and yRz ⇒ xRz.
4. Equivalence Relation: Any relation R on a set A is said to be an equivalence relation if R is reflexive,
symmetric and transitive.
5. Antisymmetric Relation: A relation R in a set A is antisymmetric
if (a, b) ∈ R, (b, a) ∈ R ⇒ a = b ∀ a, b ∈ R, or aRb and bRa ⇒ a = b, ∀ a, b ∈ R.
For example, the relation “greater than or equal to, “≥” is antisymmetric relation as

a ≥ b, b ≥ a ⇒ a = b ∀ a, b
[Note: “Antisymmetric” is completely different from not symmetric.]
6. Equivalence Class: Let R be an equivalence relation on a non-empty set A. For all a ∈ A, the
equivalence class of ‘a’ is defined as the set of all such elements of A which are related to ‘a’ under
R. It is denoted by [a].
i.e., [a] = equivalence class of ‘a’ = {x ∈ A : (x, a) ∈ R}

Relations and Functions 9


@Cbsebookshub - Join Us on Telegram
For example, Let A = {1, 2, 3} and R be the equivalence relation on A given by

R = {(1, 1), (2, 2), (3, 3), (1, 2), (2, 1)}
The equivalence classes are
[1] = equivalence class of 1 = {x ∈ A : (x, 1) ∈ R} = {1, 2}
Similarly, [2] = {2, 1} and [3] = {3}
7. Function: Let X and Y be two non-empty sets. Then, a rule f which associates to each element x ∈ X, a
unique element, denoted by f(x) of Y, is called a function from X to Y and written as f : X → Y where,
f(x) is called image of x and x is called the pre-image of f(x) and the set Y is called the co-domain of
f and f(X) = {f(x): x ∈ X} is called the range of f.
8. Types of Function:
(i) One-one function (injective function): A function f : X → Y is defined to be one-one if the
image of distinct element of X under rule f are distinct, i.e., for every x1, x2 ∈ X, f(x1) = f(x2)
implies that x1 = x2.
(ii) Onto function (Surjective function): A function f : X → Y is said to be onto function if each
element of Y is the image of some element of x i.e., for every y ∈ Y, there exists some x ∈ X, such
that y = f(x). Thus f is onto if range of f = co-domain of f.
(iii) One-one onto function (Bijective function): A function f : X → Y is said to be one-one onto, if
f is both one-one and onto.
(iv) Many-one function: A function f : X → Y is said to be a many-one function if two or more
elements of set X have the same image in Y. i.e.,
f : X → Y is a many-one function if there exist a, b ∈ X such that a ≠ b but f(a) = f(b).

9. Composition of Functions: Let f : A → B and g : B → C be two functions. Then, the composition of
f and g, denoted by gof, is defined as the function.
f:AB g:BC
A B C

x f (x) g(f (x))

gof : A  C

gof : A → C given by
gof(x) = g(f(x)), ∀ x ∈ A
Clearly, dom(gof ) = dom(f)
Also, gof is defined only when range(f) ⊆ dom(g)
10. Identity Function: Let R be the set of real numbers. A function I : R → R such that
I (x) = x ∀ x ∈ R is called identity function.
Obviously, identity function associates each real number to itself.
11. Invertible Function: For f : A → B, if there exists a function g : B → A such that gof = IA and fog = IB,
where IA and IB are identity functions, then f is called an invertible function, and g is called the
inverse of f and it is written as f –1 = g.
12. Number of Functions: If X and Y are two finite sets having m and n elements respectively then the
number of functions from X to Y is nm.
13. Vertical Line Test: It is used to check whether a relation is a function or not. Under this test, graph of
given relation is drawn assuming elements of domain along x-axis. If a vertical line drawn anywhere

10 Xam idea Mathematics–XII

@Cbsebookshub - Join Us on Telegram


in the graph, intersects the graph at only one point then the relation is a function, otherwise it is not
a function.

f (x) = x2 y-axis y-axis

Vertical line Vertical line

x-axis x-axis

f (x) = x
(y 2= x)
(a) (b)

(a) It is graph of function i.e., f(x) = x2 is a function.


(b) It is not the graph of function i.e., f (x) = ! x is not a function.
14. Horizontal Line Test: It is used to check whether a function is one-one or not. Under this test graph
of given function is drawn assuming elements of domain along x-axis. If a horizontal line (parallel
to x-axis) drawn anywhere in graph, intersects the graph at only one point then the function is one-
one, otherwise it is many-one.
y-axis y-axis
f(x) = 2x + 1 f(x) = x2

Horizontal line
Horizontal line

x-axis x-axis

(a) (b)

(a) f(x) = 2x + 1 is one-one function.


(b) f(x) = x2 is many-one function.

Selected NCERT Questions


1. Show that the relation R on the set R of real numbers, defined as R = {(a, b): a ≤ b2} is neither
reflexive nor symmetric nor transitive. [CBSE (F) 2009]
Sol. We have, R = {(a, b): a ≤ b2}, where a, b ∈ R
1 1 1 2

Reflexivity: Obviously, is a real number and # c m is not true.
2 2 2
Therefore, R is not reflexive.


Symmetry: Consider the real numbers 1 and 2.
Obviously, 1 ≤ 22 ⇒ (1, 2) ∈ R
But, 2 ≤ (1)2 is not true and so, (2, 1) ∉ R.

Relations and Functions 11


@Cbsebookshub - Join Us on Telegram
Thus, (1, 2) ∈ R but (2, 1) ∉ R
Hence, R is not symmetric.
Transitivity: By taking real numbers 2, – 2 and 1,
we have, 2 ≤ (–2)2 and –2 ≤ (1)2 but 2 ≤ (1)2 is not true.
Thus, (2, –2) ∈ R and (–2, 1) ∈ R, but (2, 1) ∉ R.
Hence, R is not transitive.

2. Check whether the relation R in R defined by R = {(a, b) : a ≤ b3} is reflexive, symmetric or
transitive.
Sol. R = {(a, b) : a ≤ b3 ∀ a, b ∈ R}
1
Reflexivity: Here ! R (Real number)
3
1 1 1 1 3 1 1 3
and > or >d n or %d n
3 27 3 3 3 3
1 1
So, d , ngR
3 3
∴ R is not reflexive.

Symmetry: 1, 2 ∈ R (Real number)
and 1 ≤ 8 or 1 ≤ 23
So, (1, 2) ∈ R but (2, 1) ∉ R [ 2 > 1 or 2 > 13]
∴ R is not symmetric.

Transitivity: Here 10, 3, 2 ∈ R (Real number)
and 10 ≤ 27 or 10 ≤ 33
so, (10, 3) ∈ R and
3
3≤8
or 3≤2
so, (3, 2) ∈ R
But 10 ≥ 8 or 10 ≥ 23 or 10 % 23
So, (10, 2) ∉ R
So, here (10, 3) ∈ R and (3, 2) ∈ R but (10, 2) ∉ R
∴ R is not transitive.
3. Show that the relation R in the set A = {1, 2, 3, 4, 5} given by R = {(a, b) : |a – b| is even} is an
equivalence relation. Show that all the elements of {1, 3, 5} are related to each other and all the
elements of {2, 4} are related to each other. But no element of {1, 3, 5} is related to any element
of {2, 4}. [CBSE Delhi 2009; Chennai 2015]
Sol. For the given relation R on A, we have
R = {(1, 1), (1, 3), (1, 5), (2, 2), (2, 4), (3, 1), (3, 3), (3, 5), (4, 2), (4, 4), (5, 1), (5, 3), (5, 5)}
For an equivalence relation, it must be reflexive, symmetric and transitive.
Reflexivity: Given that, A = {1, 2, 3, 4, 5} and R = {(a, b) : |a – b| is even}
Here (a, a) ∈ R as |a – a| = 0 is even for a ∈ A, so it is reflexive.
Symmetry: Let (a, b) ∈ R i.e.,|a – b| is even ⇒ |b – a| is also even
⇒ (b, a) ∈ R
Thus, it is symmetric.
Transitivity: Now, if (a, b) ∈ R i.e., |a – b| is even
⇒ a – b = ± 2m m ∈ N
and (b, c) ∈ R i.e., |b – c| is even

12 Xam idea Mathematics–XII

@Cbsebookshub - Join Us on Telegram


⇒ b – c = ± 2n, n ∈ N
Then, a – c = (a – b) + (b – c)
= (± 2m) + (± 2n) = ± 2(m + n)
∴ |a – c|= 2(m + n)
Thus, |a – c| is even.
Hence, (a, c) ∈ R ⇒ R is transitive.
Hence, it is an equivalence relation.
In set R all the elements corresponding to {1, 3, 5} i.e., (1, 3), (3, 1), (1, 5), (5, 1), (3, 5), (5, 3) are
related to each other because difference of these elements are even.
Again, all elements corresponding to {2, 4} are related to each other.
But no element of {1, 3, 5} is related to elements of {2, 4} because the difference of elements of the
two sets are not even.
4. Show that each of the relation R in the set A = {x ∈ Z : 0 ≤ x ≤ 12}, given by [CBSE (AI) 2010]
(i) R = {(a, b) :|a – b| is a multiple of 4}
(ii) R = {(a, b) : a = b} is an equivalence relation.
Find the set of all elements related to 1 in each case.
Sol. A = {x ∈ Z : 0 ≤ x ≤ 12}
(i) R = {(a, b) : | a – b | is a multiple of 4}
Reflexive: Let x ∈A
⇒ |x – x| = 0, which is a multiple of 4.

(x, x) ∈ R ∀ x ∈A

R is reflexive.
Symmetric: Let x, y ∈A and (x, y) ∈ R


|x – y| is a multiple of 4
or x – y = ± 4p {p is any integer}

y – x =  4p

| y – x | is a multiple of 4. ⇒ (y, x) ∈ R

R is symmetric.
Transitive: Let x, y, z ∈ A, (x, y) ∈ R and (y, z) ∈ R


|x - y| is multiple of 4 and | y – z |is multiple of 4

x – y is multiple of 4 and y – z is multiple of 4

(x – y) + (y – z) is multiple of 4 ⇒ (x – z) is multiple of 4.

|x – z| is multiple of 4.

(x, z) ∈ R
⇒ R is transitive.
So, R is an equivalence relation.
Let B be the set of elements related to 1.
∴ B = {a ∈ A : |a – 1| is multiple of 4}

⇒ B = {1, 5, 9}
{as |1 – 1| = 0, |1 – 5 |= 4, |1 – 9| = 8}
(ii) R = {(a, b) : a = b}
Reflexive: Let x ∈A
as x = x ⇒ (x, x) ∈R
⇒ R is reflexive.
Symmetric: Let x, y ∈ A and (x, y) ∈R


x = y ⇒ y=x

(y, x) ⇒ R

Relations and Functions 13


@Cbsebookshub - Join Us on Telegram
∴ R is symmetric.

Transitive: Let x, y, z ∈ A

and let (x, y) ∈ R and (y, z) ∈R

x = y and y = z ⇒ x=z ⇒ (x, z) ∈R ⇒ R is transitive.
∴ R is an equivalence relation.

Let C be the set of elements related to 1.
∴ C = {a ∈ A; a = 1} = {1}.

5. Prove that the greatest integer function f: R "R given by f(x) = [x], is neither one-one nor onto,
where [x] denotes the greatest integer less than or equal to x.
Sol. f : R " R given by f (x) = [x]
Injectivity: Let x1 = 2.5 and x2 = 2 be two elements of R.

f (x1) = f (2.5) = [ 2.5] = 2
f (x2 ) = f (2) = [2] = 2
∴ f (x1) = f (x2 ) for x1 ≠ x2

⇒ f (x) = [x] is not one-one i.e., not injective.

Surjectivity: Let y = 2.5 ∈ R be any element.

∴ f (x) = 2.5 ⇒ [x] = 2.5

Which is not possible as [x] is always an integer.
⇒ f (x) = [x] is not onto i.e., not surjective.

6. Show that the modulus function f : R " R given by f (x) = |x|, is neither one-one nor onto,
where |x| is x, if x is positive or 0 and |x| is –x, if x is negative.

Sol. f (x) = |x| = )


x, if x $ 0
–x, if x < 0
One-one: Let x1 = 1, x2 = –1 be two elements belongs to R

f(x1) = f(1) = |1| and f(x2) = f(–1) = – (–1) = 1
⇒ f(x1) = f(x2) for x1 ≠ x2
⇒ f(x) is not one-one.
Onto: Let f(x) = –1 ⇒ |x| = –1 ∈ R, which is not possible.

⇒ f(x) is not onto.
Hence, f is neither one-one nor onto function.
7. Let f : N " N be defined by
Z] n + 1
]]
]] 2 , if n is odd
f (x) = []
]] n
]] , if n is even
2
\
For all n ∈ N, state whether the function f is bijective. Justify your answer. [CBSE (AI) 2009]
]]Z n + 1
]] , if n is odd
] 2
Sol. Given, f (x) = []
]] n
]] , if n is even
2
\
Let x1 = 1 and x2 = 2 be two elements of N.
1+1 2
∴ f (x1) = f(1) = = 1 and f(x2) = f(2) = =1
2 2

14 Xam idea Mathematics–XII

@Cbsebookshub - Join Us on Telegram


f(x1) = f(x2) for x1 ≠ x2
f : N " N is not one-one.
⇒ As f is not one-one. f is not a bijective function.
8. Consider f : R+ → [– 5, ∞) given by f(x) = 9x2 + 6x – 5. Show that f is invertible and
y+6 –1
f – 1 (y) = e o.  [CBSE Delhi 2017; (F) 2010]
3
Sol. Given function f : R+ → [– 5, ∞) such that f(x) = 9x2 + 6x – 5
One-one: Let x1, x2 ∈ R+ then

f(x1) = f(x2) ⇒ 9x12 + 6x1 – 5 = 9x 22 + 6x2 – 5
⇒ 9 (x12 – x 22) + 6 (x1 – x2) = 0 ⇒ 3{3(x1 + x2) + 2} {x1 – x2} = 0

⇒ x1 – x2 = 0 [ x1, x2 ∈ R+ ⇒ x1 + x2 ≠ 0 ⇒ 3(x1 + x2) + 2 ≠ 0]

⇒ x1 = x2

So, given function is one-one.
Onto: Let y ∈ [–5, ∞) then y = f(x)

⇒ y = 9x2 + 6x – 5

i.e., 9x2 + 6x – 5 = y

⇒ (3x)2 + 2.3x.1 + (1)2 – 1 – 5 = y

⇒ (3x + 1)2 = y + 6
⇒ 3x + 1 = y + 6
–1+ y+6
⇒ x =
...(i)
3
Clearly, x ∈ R for all y ∈ [– 5, ∞)
Thus, for every y ∈ [– 5, ∞) there exists
–1+ y+6
x = !R
3
So, given function is onto.
Thus, f is both one-one and onto.
Hence, it is invertible.
Inverse:
From (i) we get,
–1+ y+6 y+6 –1
x = i.e., x=
3 3
y+6 –1
⇒ f –1 (y) =
3
x+6 –1
` f –1 (x) =
3
9. Give examples of two functions f : N " N and g : N " N such that gof is onto but f is not onto.
x – 1, if x > 1
Sol. Let f(x) = x + 1 and g(x) = )
1, if x = 1
Let x ∈ N be any element.

x ≥ 1 ⇒ x +1 ≥ 2

f (x) ≥ 2 ∀ x ∈ N
∴ Rf ≠ N

Hence, f is not onto.

Relations and Functions 15


@Cbsebookshub - Join Us on Telegram
Also, gof : N " N is such that
(gof ) (x) = g(f (x)) = g(x +1) = (x + 1) –1 = x
⇒ (gof) (x) = x ∀ x ∈ N


gof is an identity function.
Hence, gof is onto.
10. Let A = {–1, 0, 1, 2}, B = {–4, –2, 0, 2} and f, g : A "B be function defined by f(x) = x2 – x, x ∈A and
1
g(x) = 2 x – – 1, x ∈ A. Are f and g equal? Justify your answer.
2

Sol. Given f : A " B and g : A " B defined as


1
f(x) = x2 – x and g(x) = 2 x – ∀x∈A
2
1 –3
f(–1) = (–1)2 –(–1) = 2; g(–1) = 2 –1 – –1 = 2 –1 = 2
2 2
1
f(0) = 02 – 0 = 0; g(0) = 2 0 – – 1=1 – 1= 0
2
1 1
f(1) = 12 – 1 =1 – 1 = 0; g(1) = 2 1 – –1 = 2 – 1= 1 – 1 = 0
2 2
1 3
f(2) = 22 – 2 = 2; g(2) = 2 2 – –1 = 2 – 1= 2
2 2
Clearly, f (–1) = g(–1); f(0) = g(0); f(1) = g(1) and f(2) = g(2)

f(x) = g(x) ∀ x ∈ A
Z]1, x > 0
]]
]
11. Let f : R" R be the Signum function defined as f (x) = []0, x = 0 and g: R " R be the Greatest
]]
] –1, x < 0
\
Integer Function given by g(x) = [x]. Then, do fog and gof coincide in (0, 1]?
Sol. Given f : R " R and g : R " R defined as
Z] 1, x > 0
]]
]
f (x) = ][ 0, x = 0 and g (x) = [x]
]]
] –1, x < 0
\
fog (x) = f (g(x)) = f ([x])

f (0), if 0 < x < 1


= *
f (1), if x = 1

0 if 0< x<1

fog (x) = )
1 if x=1

and gof (x) = g(f (x))


= g(1) ∀ x ∈ (0, 1]
= [1] = 1

gof (x) = 1∀ x ∈(0, 1]
Clearly, fog (x) and gof (x) do not coincide ∀ x ∈(0, 1].

16 Xam idea Mathematics–XII

@Cbsebookshub - Join Us on Telegram


Multiple Choice Questions [1 mark]
Choose and write the correct option in the following questions.

1. The relation R in the set A = {1, 2, 3, 4} given by R = {(1, 2), (2, 2), (1, 1), (4, 4), (1, 3), (3, 3), (3, 2)}
is
(a) reflexive and symmetric but not transitive
(b) reflexive and transitive but not symmetric
(c) symmetric and transitive but not reflexive
(d) an equivalence relation
2. If A = {a, b, c, d}, then a relation R = {(a, b), (b, a), (a, a)} on A is
(a) symmetric only (b) transitive only
(c) reflexive and transitive (d) symmetric and transitive only
3. For real numbers x and y, define xRy if and only if x – y + 2 is an irrational number. Then the
relation R is [NCERT Exemplar]
(a) reflexive (b) symmetric (c) transitive (d) none of these
4. Consider the non-empty set consisting of children in a family and a relation R defined as aRb
if a is brother of b. Then R is [NCERT Exemplar]
(a) symmetric but not transitive (b) transitive but not symmetric
(c) neither symmetric nor transitive (d) both symmetric and transitive

5. The maximum number of equivalence relation on the set A = {1, 2, 3} are [NCERT Exemplar]
(a) 1 (b) 2 (c) 3 (d) 5
6. Let L denotes the set of all straight lines in a plane. Let a relation R be defined by lRm if and
only if l is perpendicular to m ∀ l, m ∈ L. Then R is [NCERT Exemplar]
(a) reflexive (b) symmetric (c) transitive (d) none of these
7. Let A = {1, 2, 3}. Then number of relations containing (1, 2) and (1, 3) which are reflexive and
symmetric but not transitive is
(a) 1 (b) 2 (c) 3 (d) 4
8. Let A = {1, 2, 3}. Then number of equivalence relations containing (1, 2) is/are
(a) 1 (b) 2 (c) 3 (d) 4
9. Let A and B be finite sets containing m and n elements respectively. The number of relations that
can be defined from A to B is
(a) 2mn (b) 2m+n (c) mn (d) 0
10. Set A has 3 elements and the set B has 4 elements. Then the number of injective mapping that
can be defined from A to B is [NCERT Exemplar]
(a) 144 (b) 12 (c) 24 (d) 64
11. The function f : R → R defined by f(x) = 2x + 2|x| is
(a) One-one and onto (b) Many-one and onto
(c) One-one and into (d) Many-one and into
12. If the set A contains 5 elements and the set B contains 6 elements, then the number of one-one
and onto mapping from A to B is
(a) 720 (b) 120 (c) 0 (d) none of these

Relations and Functions 17


@Cbsebookshub - Join Us on Telegram
13. Which of the following functions from z into z is bijection? [NCERT Exemplar]
3
(a) f(x) = x (b) f(x) = x + 2 (c) f(x) = 2x + 1 (d) f(x) = x2 + 1

14. Let f : [2, ∞) → R be the function defined by f(x) = x2 – 4x + 5, then the range of f is
[NCERT Exemplar]
(a) R (b) [1, ∞) (c) [4, ∞) (d) [5, ∞)

15. Let f : R → R be defined by f(x) = x2 + 1. Then, pre-images of 17 and –3, respectively, are

[NCERT Exemplar]
(a) f, {4, –4} (b) {3, –3}, f (c) {4, –4}, f (d) {4, –4}, {2, –2}
x
16. Let f : R → R be defined by f(x) = 3x2 – 5 and g : R → R by g(x) = 2 . Then gof is
x +1
[NCERT Exemplar]
3x 2 – 5 3x 2 – 5 3x 2 3x 2
(a) 4 2 (b) 4 2 (c) 4 2 (d)
9x – 30x + 26 9x – 6x + 26 x + 2x – 4 9x + 30x 2 – 2
4

17. If f(x) = sin2 x and the composite function g(f(x)) = |sin x|, then g(x) is equal to

(a) x + 1 (b) x – 1 (c) x (d) – x
18. Let f : R → R be the functions defined by f(x) = x3 + 5. Then f–1(x) is [NCERT Exemplar]
1 1 1
(a) (x + 5) 3 (b) (x – 5) 3 (c) (5 – x) 3 (d) 5 – x

19. Let f : R – ' – 1 " R be a function defined as f (x) =


4 4x
. The inverse of f is the map
3 3x + 4
g : Range f → R – ' – 1
4
[NCERT]
3
3y 4y 4y 3y
(a) g (y) = (b) g (y) = (c) g (y) = (d) g (y) =
3 – 4y 4 – 3y 3 – 4y 4 – 3y

20. Let f : R – ' 1 " R be defined by f (x) =


3 3x + 2
. Then [NCERT Exemplar]
5 5x – 3
1
(a) f –1(x) = f(x) (b) f –1(x) = – f(x) (c) fof(x) = – x (d) f –1 (x) = f (x)
19
Answers
1. (b) 2. (d) 3. (a) 4. (b) 5. (d) 6. (b)
7. (a) 8. (b) 9. (a) 10. (c) 11. (c) 12. (c)
13. (b) 14. (b) 15. (c) 16. (a) 17. (c) 18. (b)
19. (b) 20. (a)

Solutions of Selected Multiple Choice Questions


1. Since every element of A is related to itself in the given relation R, therefore R is reflexive and as
(1, 2) ∈ R and (2, 2) ∈ R ⇒ (1, 2) ∈ R also (1, 3) ∈ R and (3, 2) ∈ R ⇒ (1, 2) ∈ R. Again (1, 3) ∈ R
and (3, 3) ∈ R ⇒ (1, 3) ∈ R. Thus R is also transitive. Hence relation R is reflexive and transitive
but not symmetric because, (1, 2) ∈ R but (2, 1) ∉ R, also (1, 3) ∈ R but (3, 1) ∉ R and (3, 2) ∈ R but
(2, 3) ∉ R.
2. On the set A = {a, b, c, d} given relation R = {(a, b), (b, a), (a, a)} is symmetric and transitive only.
Since, (a, b) ∈ R ⇒ (b, a) ∈ R, therefore it is symmetric
Also, (a, b) ∈ R and (b, a) ∈ R ⇒ (a, a) ∈ R, so it is also transitive. As (b, b), (c, c) and (d, d) does
not belong to R hence R is not reflexive.
Hence relation R is symmetric and transitive only.

18 Xam idea Mathematics–XII

@Cbsebookshub - Join Us on Telegram


4. Given, aRb ⇒ a is brother of b
This does not mean that b is also a brother of a because b can be a sister of a.
Hence, R is not symmetric.
Again, aRb ⇒ a is brother of b and bRc ⇒ b is brother of c.
So, a is brother of c.
Hence, R is transitive.
6. For l, m ∈ L
if (l, m) ∈ R ⇒ l ⊥ m
⇒ m ⊥ l ⇒ (m, l) ∈ R
∴ R is symmetric.

10. The total number of injective mappings from the set containing n elements into the set containing
m elements is mPn. So here it is 4P3 = 4! = 24.
14. Given that, f(x) = x2 – 4x + 5
Let y = x2 – 4x + 5
y = x2 – 4x + 4 + 1 = (x – 2)2 + 1
(x – 2)2 = y – 1 ⇒ x–2= y–1
⇒ x = 2 + y – 1

y – 1 $ 0, y $ 1
Range = [1, ∞)
15. Since f –1(17) = x ⇒ f(x) = 17 or x2 + 1 = 17
⇒ x = ± 4 or f –1(17) = {4, – 4}

and f –1(–3) = x ⇒ f(x) = –3 ⇒ x2 + 1 = –3 ⇒ x2 = – 4 and hence f –1(–3) = f.
3x 2 – 5
16. We have, gof(x) = g(f(x)) = g(3x2 – 5) =
(3x 2 – 5) 2 + 1
3x 2 – 5
⇒ gof(x) =

9x 4 – 30x 2 + 26
17. We have,
g(f(x)) = |sin x| = sin 2 x
⇒ g(f(x)) =
f (x)
⇒ g(x) =
x
1
18. Let y = x3 + 5  ⇒  y – 5 = x3  ⇒  x = (y – 5) 3
1
⇒ f –1 (x) = (x – 5) 3

4x
19. Let y =     ⇒ 3xy + 4y = 4x
3x + 4
⇒ 4x – 3xy = 4y  ⇒  x(4 – 3y) = 4y

4y 4y
∴ x =
  ⇒  g (y) =
4 – 3y 4 – 3y
3x + 2
20. Let y =   ⇒ 5xy – 3y = 3x + 2
5x – 3

Relations and Functions 19


@Cbsebookshub - Join Us on Telegram
⇒ x(5y – 3) = 3y + 2

3y + 2
⇒ x =

5y – 3
3x + 2
⇒ f –1 (x) =
= f (x)
5x – 3
⇒ f –1(x) = f(x)

Fill in the Blanks [1 mark]


1. A relation from a set A to a set B is a ___________ of A × B.
2. A relation R from set A to set B is said to be ___________ if R = A × B.
3. If any set A contains n elements. Then, the total number of injective functions from A onto
itself is ___________ .
4. The domain of the function f : R → R defined by f (x) = x 2 – 3x + 2 is ___________ .
5. If f(x) = {4 – (x – 7)3}, then f –1(x) = ___________ . [NCERT Exemplar]

Answers
1. Subset 2. The universal relation 3. n! 4. (– ∞, 1] ∪ [2, ∞)
1
5. 7 + (4 – x) 3

Solutions of Selected Fill in the Blanks


4. For f(x)to be defined
x2 – 3x + 2 ≥ 0
⇒ x2 – 2x – x + 2 ≥ 0 ⇒  x(x – 2) – 1 (x – 2) ≥ 0

⇒ (x – 2) (x – 1) ≥ 0
⇒ (x – 1) (x – 2) ≥ 0
∴ x ≤ 1 or x ≥ 2

Domain of f = (– ∞, 1] ∪ [2, ∞).
5. Let y = f(x)  ⇒  y = {4 – (x – 7)3}
⇒ y – 4 = – (x – 7)3

⇒ (x – 7)3 = 4 – y

1 1
⇒ (x – 7) = (4 – y) 3   ⇒  x = 7 + (4 – y) 3

1
⇒ f –1 (x) = 7 + (4 – x) 3

Very Short Answer Questions [1 mark]


1. Let A = {1, 2, 3, 4}. Let R be the equivalence relation on A × A defined by (a, b) R (c, d) iff
a + d = b + c. Find the equivalence class [(1, 3)]. [CBSE Sample Paper 2018]
Sol. [(1, 3)] = {(x, y) d A × A: x + 3 = y + 1} = {(x, y) d A × A: y – x = 2}
= {(1, 3), (2, 4)}
2. If R = {(x, y): x + 2y = 8} is a relation on N, write the range of R. [CBSE (AI) 2014]
Sol. Given: R = {(x, y): x + 2y = 8}

20 Xam idea Mathematics–XII

@Cbsebookshub - Join Us on Telegram



a x + 2y = 8
8–x
&
y=
2
& when x = 6, y = 1; x = 4, y = 2; x = 2, y = 3.
` Range = {1, 2, 3}

3. State the reason for the relation R in the set {1, 2, 3} given by R = {(1, 2), (2, 1)} not to be
transitive. [CBSE Delhi 2011]
Sol. R is not transitive as (1, 2) ∈ R and (2, 1) d R but (1, 1) ∉ R
[Note: A relation R in a set A is said to be transitive if (a, b) ∈ R and (b, c) ∈ R ⇒ (a, c) ∈ R ∀ a, b, c ∈ R]
4. Let R = {(a, a 3) : a is a prime number less than 5} be a relation. Find the range of R.
[CBSE (F) 2014]
R = {(a, a 3) : a is a prime number less than 5}
Sol. Here
⇒ R = {(2, 8), (3, 27)}

Hence Range of R = {8, 27}
5. Let A = {1, 2, 3}, B = {4, 5, 6, 7} and let f = {(1, 4), (2, 5), (3, 6)} be a function from A to B. State
whether f is one-one or not. [CBSE (AI) 2011]
Sol. f is one-one because f

f (1) = 4 ;
1 4
f (2) = 5 ; 2 5
f (3) = 6 3 6
7
i.e., no two elements of A have same f image.

6. If X and Y are two sets having 2 and 3 elements respectively, then find the number of functions
from X to Y.
Sol. Number of functions from X to Y = 32 = 9.
7. If the mapping f and g are given by f = {(1, 2), (3, 5), (4, 1)} and g = {(2, 3), (5, 1), (1, 3)], then
write fog. [NCERT Exemplar]
Sol. Obviously, domain of “fog” is domain of “g” i.e., {2, 5, 1}.
Now, fog(2) = f(g(2)) = f(3) = 5, fog(5) = f(g(5)) = f(1) = 2
fog(1) = f(g(1)) = f(3) = 5 ⇒ fog = {(2, 5), (5, 2), (1, 5)}
1
8. If f : R → R is given by f (x) = (3 – x 3) 3 , then determine f(f(x)). [CBSE (AI) 2010]

1
Sol. We have, f (x) = (3 – x 3) 3 ,
1

f (f (x)) = f %(3 – x 3) 3 / = ;3 – %(3 – x 3) 3 /


E
1 1 3 3 1 1
∴ = [3 – (3 – x 3)] 3 = (x 3) 3 = x

9. Find fog(x), if f(x) = |x| and g(x) = |5x – 2|. [CBSE (F) 2011]
Sol. fog(x) = f(g(x)) = f (|5x – 2|) = |5x – 2|
10. Write fog, if f : R → R and g : R → R are given by f(x) = 8x3 and g(x) = x1/3. [CBSE (F) 2011]
1/3 1/3 3
Sol. fog(x) = f (g(x)) = f(x ) = 8(x ) = 8x
11. If f : R → R is defined by f(x) = 3x + 2, define f [ f(x)]. [CBSE (F) 2011]
Sol. f(f(x)) = f (3x + 2) = 3 (3x + 2) + 2
= 9x + 6 + 2 = 9x + 8

Relations and Functions 21


@Cbsebookshub - Join Us on Telegram
12. Which one of the following graph represents the function of x? Why?
y-axis y-axis

x-axis x-axis

(a) (b)

Sol. y-axis y-axis Vertical


Vertical Line
Line

x-axis x-axis

(a) (b)

Graph (a) represents the function of x, because vertical line drawn in (a) meets the graph at only
one point i.e., for one x, in domain there exist only one f(x) in codomain.
13. If f : R → R is given by f(x) = x2, find the value of f –1 (25).
Sol. Let y = f(x)  ⇒  y = x2  ⇒  x = y
⇒ f –1 (x) = x

⇒ f –1 (25) = 25 = ! 5

∴ f –1 (25) = {–5, 5}

Short Answer Questions-I [2 marks]


1. Write the inverse relation corresponding to the relation R given by R = {(x, y): x ∈ N, x < 5, y = 3}.
Also write the domain and range of inverse relation.
Sol. Given, R = {(x, y) : x ∈ N, x < 5, y = 3}
⇒ R = {(1, 3), (2, 3), (3, 3), (4, 3)}

Hence, required inverse relation is
R–1 = {(3, 1), (3, 2), (3, 3), (3, 4)}
∴ Domain of R–1 = {3} and

Range of R–1 = {1, 2, 3, 4}
2. Check if the relation R in the set A = {1, 2, 3, 4} defined as R = {(a, b) : a divides b} is (i) symmetric
(ii) transitive.
Sol. In the set A = {1, 2, 3, 4}
Relation is defined as R = {(a, b) : a divides b}
(i) Symmetric: Take a = 2, b = 4, a, b ∈ A
  2 divides 4  ⇒ (a, b) ∈ R
   but 4 does not divide 2  ⇒ (b, a) ∉ R
∴  It is not symmetric

22 Xam idea Mathematics–XII

@Cbsebookshub - Join Us on Telegram


(ii) Transitive: Let a = 1, b = 2 and c = 4
Here 1 divides 2  ⇒  (1, 2) ∈ R
and 2 divides 4  ⇒  (2, 4) ∈ R

1 divides 4 also   ⇒  (1, 4) ∈ R
∴  It is transitive.

3x – 4
3. If f is an invertible function, defined as f (x) = , write f –1(x). [CBSE (F) 2010]
5
Sol. Since f –1 is inverse of f.
∴ fof –1 = I ⇒ fof –1 (x) = I (x)

⇒ fof –1(x) = x ⇒ f(f –1(x)) = (x)

3 (f – 1 (x)) – 4 5x + 4

= x ⇒ f –1 (x) =
5 3
|x – 1 |
4. What is the range of the function f (x) = ? [CBSE Delhi 2010]
( x – 1)
|x – 1 |
Sol. Given f (x) =
(x – 1)

Obviously, x – 1 = )
(x – 1) if x – 1 > 0 or x > 1
– (x – 1) if x – 1 < 0 or x < 1
( x – 1) – ( x – 1)
Now, (i) 6 x 2 1, f (x) = = 1 , (ii) 6 x 1 1, f (x) = = – 1,
( x – 1) ( x – 1)
i.e., f(x) = –1, 1


Range of f(x) = {–1, 1}.
1
5. Let f : R → R be the function defined by f (x) = 2 – cos x , 6 x d R. Then, find the range of f.
[NCERT Exemplar]
1
Sol. Given function, f (x) = , 6 xd R
2 – cos x
1
y = 2 – cos x

2y – y cos x = 1 ⇒ y cos x = 2y – 1
2y –1 1 1
⇒ cos x =
y =2–y ⇒ cos x = 2 – y
1
⇒ –1 ≤ cos x ≤ 1
⇒ –1 < 2 – #1
y
1 1
⇒ –3 # –
# –1 ⇒ 1# #3
y y
1

#y#1
3
1
So, range of y is < , 1F .
3
6. If f : R → R is defined by f(x) = x2 – 3x + 2, write f {f(x)}. [NCERT Exemplar]
2
Sol. Given that, f(x) = x – 3x + 2
f{f(x)} = f(x2 – 3x + 2)
= (x2 – 3x + 2)2 – 3(x2 – 3x + 2) + 2
= x4 + 9x2 + 4 – 6x3 – 12x + 4x2 – 3x2 + 9x – 6 + 2

Relations and Functions 23


@Cbsebookshub - Join Us on Telegram
= x4 – 6x3 + 10x2 – 3x
f{f(x)} = x4 – 6x3 + 10x2 – 3x

Short Answer Questions-II [3 marks]


1. Check whether the relation R defined on the set A = {1, 2, 3, 4, 5, 6} as R = {(a, b) : b = a + 1} is
reflexive, symmetric or transitive. [CBSE 2019 (65/2/1)]
Sol. Given relation R defined on the set A = {1, 2, 3, 4, 5, 6} as R = {(a, b) : b = a + 1}
Now,
Reflexivity: Let a ∈ A

We have, a ≠ a + 1  ⇒ (a, a) ∉ R
∴ It is not reflexive

Symmetric: Let a = 1 and b = 2 i.e. a, b ∈ A

∴ b = a + 1  ⇒  2 = 1 + 1  ⇒ (a, b) ∈ R

but a ≠ b + 1 as 1 ≠ 2 + 1  ⇒ (b, a) ∉ R
∴ It is not symmetric.

Transitive: Let a, b, c ∈ A
Now, if (a, b) ∈ R  ⇒  b = a + 1 ...(i)
and (b, c) ∈ R  ⇒  c = b + 1 ...(ii)
from (i) and (ii), we have
c = (a + 1) + 1 = a + 2
⇒ c = a + 2  ⇒ (a, c) ∉ R

∴ Is is not transitive

Hence, relation R is neither reflexive nor symmetric nor transitive.
2. Show that the relation R on the set Z of all integers, given by R = {(a, b) : 2 divides (a – b)} is an
equivalence relation. [CBSE 2019 (65/3/1)]
Sol. Given relation R = {(a, b) : 2 divides (a – b)} on the set Z of all integers
Reflexive: Let a ∈ Z

Since (a – a) = 0, which is divisible by 2 i.e., (a, a) ∈ R
∴ R is reflexive.

Symmetric: Let a, b ∈ Z

such that (a, b) ∈ R  ⇒  (a – b) is divisible by 2
 ⇒  – (a – b) is also divisible by 2
 ⇒ (b – a) is divisible by 2  ⇒ (b, a) ∈ R
i.e., (a, b) ∈ R  ⇒ (b, a) ∈ R

∴ R is symmetric.

Transitive: Let a, b, c ∈ Z

such that (a, b) ∈ R   ⇒ (a – b) is divisible by 2
Let a – b = 2k1 where k1 is an integer ...(i)
and (b, c) ∈ R ⇒ (b – c) is divisible by 2  ⇒  b – c = 2 k2 where k2 is an integer ...(ii)
Adding (i) and (ii), we have
(a – b) + (b – c) = 2 (k1 + k2)  ⇒  a – c = 2 (k1 + k2)  ⇒ (a – c) is divisible by 2.

24 Xam idea Mathematics–XII

@Cbsebookshub - Join Us on Telegram


⇒ (a, c) ∈ R

∴ R is transitive

Thus, R is reflexive, symmetric and transitive. Hence, given relation R is an equivalence relation.
3. Let A = {1, 2, 3, ..., 9} and R be the relation in A × A defined by (a, b) R(c, d) if a + d = b + c for
(a, b), (c, d) in A × A. Prove that R is an equivalence relation and also obtain the equivalent class
[(2, 5)]. [NCERT Exemplar]
Sol. Given that, A= {1, 2, 3, ..., 9} and (a, b) R(c, d) if a + d = b + c for (a, b) ∈ A × A and (c, d) ∈ A × A.
Since (a, b) R(a, b) as
a + b = b + a, ∀ a, b ∈ A
Hence, R is reflexive.
Let (a, b) R(c, d) then
a + d = b + c

c + b = d + a ⇒ (c, d) R(a, b)
Hence, R is symmetric.
Let (a, b) R(c, d) and (c, d) R(e, f) then
a + d = b + c and c + f = d + e

a + d = b + c and d + e = c + f

(a + d) – (d + e) = (b + c) – (c + f)

(a – e) = b – f

a+f=b+e
(a, b) R(e, f)
So, R is transitive.
Hence, R is an equivalence relation.
Now, equivalence class containing [(2, 5)] is {(1, 4), (2, 5), (3, 6), (4, 7), (5, 8), (6, 9)}.
4. Let A = {x ∈ R: –1 ≤ x ≤ 1} = B. Show that f : A → B given by f (x) = x |x| is a bijection. [HOTS]
Sol. We have,
x 2, if x $ 0
Y
f (x) = x | x | = *
–x 2, if x < 0
y = x2
For x ≥ 0, f(x) = x2 represents a parabola opening upward
and for x < 0, f(x) = –x2 represents a parabola opening
X' X
downward. O
So, the graph of f(x) is as shown in figure. y = – x2
Since any line parallel to x-axis, will cut the graph at only
one point, so f is one-one. Also, any line parallel to y-axis
Y'
will cut the graph, so f is onto.
Thus, it is evident from the graph of f(x) that f is one-one and onto.
5. If f (x) = x (x $ 0) and g(x) = x2 – 1 are two real functions, then find fog and gof and check
whether fog = gof. [HOTS]
Sol. The given functions are f (x) = x , x $ 0 and g (x) = x 2 – 1
We have, domain of f = [0, ∞) and range of f = [0, ∞)
a x 2 $ 0 for all x ! R
domain of g = R and range of g = [–1, ∞) = G
` x 2 – 1 $ – 1 for all x ! R
Computation of gof: We observe that range of f = [0, ∞) ⊆ domain of g

Relations and Functions 25


@Cbsebookshub - Join Us on Telegram

gof exists and gof : [0, ∞) → R
Also, gof (x) = g (f (x)) = g ( x )) = ( x ) 2 – 1 = x – 1
Thus, gof : [0, ∞) → R is defined as gof (x) = x – 1
Computation of fog: We observe that range of g = [–1, ∞) ⊆ domain of f.
∴ Domain of fog = {x : x ∈ domain of g and g(x) ∈ domain of f}

⇒ Domain of fog = {x : x ∈ R and g(x) ∈ [0, ∞)}

⇒ Domain of fog = {x : x ∈ R and x2 – 1 ∈ [0, ∞)}

⇒ Domain of fog = {x : x ∈ R and x2 – 1 ≥ 0}

Domain of fog = {x : x ∈ R and x ≤ –1 or x ≥ 1}
∴ Domain of fog = {x : x ∈ (– ∞, –1] ∪ [1, ∞)}

Also, fog (x) = f (g (x)) = f (x 2 – 1) = x 2 – 1
Thus, fog : (– ∞, –1] ∪ [1, ∞) → R is defined as fog (x) = x 2 – 1 .
We find that fog and gof have distinct domains. Also, their formulae are not same.
Hence, fog ≠ gof
6. Let A = {–1, 0, 1, 2}, B = {–4, –2, 0, 2} and f, g : A → B be functions defined by f(x) = x2 – x, x ∈ A
1
and, g (x) = 2 x – –1 x ∈ A. Are f and g equal? Justify your answer.
2
Sol. For two functions f : A → B and g : A → B to be equal, f (a) = g(a) ∀ a ∈ A and Rf = Rg.
Here, we have f(x) = x2 – x
1
g (x) = 2 x – – 1 [x ∈ A = {–1, 0, 1, 2}]
2
We see that, f(–1) = (–1)2 – (–1) = 2
1 3
g (–1) = 2 (–1) – –1 = 2# –1 = 3–1 = 2
2 2
So, f(–1) = g(–1)
Again, we check that, f(0) = g(0) = 0, f(1) = g(1) = 0 and f(2) = g(2) = 2.
Hence, f and g are equal functions.
]Z] –1, x < 0
]]
7. Let g(x) = 1 + x – [x] and f (x) = [] 0, x = 0 then for all x find fog (x) . [HOTS]
]]
] 1, x > 0
\
Sol. fog(x) = f(g(x)) = f(1 + x – [x]) = f(1 + {x}) = 1
Here, {x} = x – [x]
Obviously, 0 ≤ x – [x] < 1
⇒ 0 ≤ {x} < 1

⇒ 1 + {x} ≥ 1

∴ fog (x) = f(1 + {x}) = 1

Note: Symbol {x} denotes the fractional part or decimal part of x.
For example, {4.25} = 0.25, {4} = 0, {–3.45} = 0.45
In this way {x} = x – [x] ⇒ 0 ≤ {x} < 1
x–2
8. Let A = R – {3}, B = R – {1}. If f : A → B be defined by f (x) = x – 3 , 6 x ! A . Then, show that f is
bijective. [NCERT Exemplar]

26 Xam idea Mathematics–XII

@Cbsebookshub - Join Us on Telegram


A = R – {3}, B = R – {1}.
Sol. Given that,
x–2
f : A → B is defined by f (x) =
,6x!A
x –3
For injectivity
x1 – 2 x2 – 2
Let f(x1) = f(x2) ⇒ =
x1 – 3 x2 – 3
⇒ (x1 – 2)(x2 – 3) = (x2 – 2)(x1 – 3)


x1 x2 – 3x1 – 2x2 + 6 = x1x2 – 3x2 – 2x1 + 6

– 3x1 – 2x2 = –3x2 – 2x1

–x1 = –x2 ⇒ x1 = x2
So, f(x) is an injective function.
For surjectivity
x–2
Let y= x–3 & x – 2 = xy – 3y
2 – 3y

x(1 – y) = 2 – 3y ⇒ x= 1– y

3y – 2
⇒ x = y –1 ! A, 6 y ! B [codomain]

So, f(x) is surjective function.


Hence, f(x) is a bijective function.
9. Show that the relation S in the set A = {x ∈ Z : 0 ≤ x ≤ 12} given by S = {(a, b) : a, b ∈ Z, |a – b|
is divisible by 3} is an equivalence relation. [CBSE 2019 (65/4/1)]
Sol. On the set A = {x ∈ Z : 0 ≤ x ≤ 12} and relation S is given by
S = {(a, b) : a, b ∈ Z, |a – b| is divisible by 3}

Reflexivity:
Let a ∈ A Then
(a, a) ⇒ |a – a| = 0 which is divisible by 3. ⇒ (a, a) ∈ S
∴ It is reflexive.


Symmetric:
Let a, b ∈ A Then
(a, b) ∈ S ⇒ |a – b| is divisible by 3.
⇒ |b – a| is also divisible by 3. ⇒ (b, a) ∈ S
∴ It is symmetric relation.


Transitive:
Let a, b, c ∈ A Then
(a, b) ∈ S ⇒ |a – b| is divisible by 3. ⇒  a – b = ± 3k1, where k1 is an integer ...(i)
(b, c) ∈ S ⇒ |b – c| is divisible by 3. ⇒  b – c = ± 3k2, where k2 is an integer ...(ii)
∴ a – c = a – b + b – c = ± 3k1 ± 3k2 = ± 3(k1 + k2) ⇒ |a – c| is also divisible by 3.  ⇒ (a, c) ∈ S

∴ It is transitive relation.

Hence, the relation S is an equivalence relation.

Relations and Functions 27


@Cbsebookshub - Join Us on Telegram
x–1
10. Let A = R – {2} and B = R – {1}. If f : A → B is a function defined by f(x) = , show that f is
x–2
one-one and onto. Hence, find f –1. [CBSE 2019 (65/4/1)]
Sol. Given sets A = R – {2} and B = R – {1} and function defined such that f : A → B
x−1
f ( x) =
x−2

One-one:
Let x1, x2 ∈ A such that x1 ≠ x2 ⇒ x1 – 1 ≠ x2 – 1 and x1 – 2 ≠ x2 – 2
x1 – 1 x –1

! 2 ⇒ f(x1) ≠ f(x2)
x1 – 2 x2 – 2

f(x) is one-one function.



Onto:
x−1
Let y =
x−2
⇒ xy – 2y = x – 1  ⇒  xy – x = 2y – 1  ⇒  x(y – 1) = 2y – 1

2y − 1
∴ x =
,y ≠ 1
y−1
Clearly, for every value of y there must be some x.
∴ f(x) is onto.

Here, f(x) is both one-one and onto, so it is invertible.
We have,
2y − 1 2y − 1
x = ⇒ f −1 ( y ) =
y−1 y−1

Long Answer Questions [5 marks]


1. Consider f : R+ → [–9, ∞) given by f(x) = 5x2 + 6x – 9. Prove that f is invertible with

f –1 (y) = d n.
54 + 5y – 3
[CBSE Allahabad 2015]
5
Sol. To prove f is invertible, it is sufficient to prove f is one–one onto
Here, f(x) = 5x2 + 6x – 9
One-one: Let x1, x2 ∈ R+, then
f (x1) = f (x2) & 5x12 + 6x1 – 9 = 5x22 + 6x2 – 9

&
5x12 + 6x1 – 5x22 – 6x2 = 0 & 5 (x12 – x22) + 6 (x1 – x2) = 0

&
5 (x1 – x2) (x1 + x2) + 6 (x1 – x2) = 0 & (x1 – x2) (5x1 + 5x2 + 6) = 0

&
x1 – x2 = 0 [a 5x1 + 5x2 + 6 ! 0]

&
x1 = x2
i.e., f is one-one function.

Onto: Let f(x) = y

28 Xam idea Mathematics–XII

@Cbsebookshub - Join Us on Telegram



` y = 5x 2 + 6x – 9 & 5x 2 + 6x – (9 + y) = 0
– 6 ! 36 + 4 # 5 (9 + y) – 6 ! 216 + 20y
&
x=
10
& x=
10
! 54 + 5y – 3 54 + 5y – 3
&
x=
5
& x=
5
[ x ∈ R+]

Obviously, ∀ y ∈ [–9, ∞) the value of x ∈ R+.


⇒ f is onto function.

Hence, f is one-one onto function, i.e., invertible.
54 + 5y – 3
Also, f is invertible with f – 1 (y) = .
5
2. If f, g : R → R be two functions defined as f(x) = |x| + x and g(x) = |x| – x, ∀ x ∈ R. Then find
fog and gof. Hence find fog(–3), fog(5) and gof(–2). [CBSE (F) 2016]
Sol. Here, f(x) = |x| + x can be written as
2x if x $ 0
f (x) = )
0 if x 1 0
And g(x) = |x| – x, can be written as
0 if x $ 0
g (x) = )
– 2x if x 1 0
Therefore, gof is defined as
For x ≥ 0, gof(x) = g(f(x)) ⇒ gof(x) = g(2x) = 0
and for x < 0, gof(x) = g(f(x)) = g(0) = 0
Hence, gof (x) = 0 ∀ x ∈ R.
Again, fog is defined as
For x ≥ 0, fog(x) = f(g(x)) = f(0) = 0
and for x < 0, fog(x) = f(g(x)) = f(–2x) = 2(–2x) = – 4x
0, x $ 0
Hence, fog (x) = )
– 4x, x 1 0

2nd part
fog(5) = 0 [ 5 ≥ 0]
fog(–3) = – 4 × (– 3) = 12 [ –3 < 0]
gof(–2) = 0
3. Let N denote the set of all natural numbers and R be the relation on N × N defined by
(a, b) R (c, d) if ad(b + c) = bc(a + d). Show that R is an equivalence relation. [CBSE Delhi 2015]
Sol. Here R is a relation defined as
R = {(a, b), (c, d)] : ad(b + c) = bc(a + d)}

Reflexivity: By commutative law under addition and multiplication
b + a = a + b ∀ a, b ∈ N
ab = ba ∀ a, b ∈ N
∴ ab(b + a) = ba(a + b)
∀ a, b ∈ N
⇒ (a, b) R (a, b)

Hence, R is reflexive.

Relations and Functions 29


@Cbsebookshub - Join Us on Telegram
Symmetry: Let (a, b) R (c, d)
(a, b) R (c, d) ⇒ ad(b + c) = bc(a + d)
⇒ bc(a + d) = ad(b + c)
⇒ cb(d + a) = da(c + b) [By commutative law under addition and multiplication]
⇒ (c, d) R (a, b)
Hence, R is symmetric.
Transitivity: Let (a, b) R (c, d) and (c, d) R (e, f)
Now, (a, b) R (c, d) and (c, d) R (e, f)

⇒ ad(b+c) = bc (a+d) and cf(d+e) = de(c+f)



+
⇒ b + c = a + d and d + e = c f

bc ad de cf
1 1 1 1 1 1 1 1

+ = + and + = +
c b d a e d f c
Adding both, we get
1 1 1 1 1 1 1 1
⇒ + + + = + + +
c b e d d a f c
1 1 1 1 e+b f+a

+ = +
b e a f
& be
=
af

⇒ af (b + e) = be (a + f)
& (a, b) R (e, f) [c, d ≠ 0]
Hence, R is transitive.
In this way, R is reflexive, symmetric and transitive.
Therefore, R is an equivalence relation.
4. Consider f : R+ → [4, ∞) given by f(x) = x2 + 4. Show that f is invertible with the inverse (f –1) of
f given by f –1 (y) = y – 4 , where R+ is the set of all non-negative real numbers.
[CBSE (AI) 2013; (F) 2011]
Sol. One-one: Let x1, x2 ∈ R+ (Domain)
f(x1) = f(x2) ⇒ x12 + 4 = x22 + 4

x12 = x22

x1 = x2 [∴ x1, x2 are +ve real number]
Hence, f is one-one function.
Onto: Let y ∈ [4, ∞) such that

y = f(x) ∀ x ∈ R+ [set of non-negative reals]
2
⇒ y = x + 4
⇒ x = y – 4 [∴ x is + ve real number]
Obviously, ∀ y ∈ [4, ∞), x is real number ∈ R+ (domain)
i.e., all elements of codomain have pre image in domain.

⇒ f is onto.

Hence, f is invertible being one-one onto.
Inverse function: If f –1 is inverse of f, then

fof –1 = I [Identity function]

30 Xam idea Mathematics–XII

@Cbsebookshub - Join Us on Telegram



fof –1 (y) = y ∀ y ∈ [4, ∞)

f(f –1(y)) = y
⇒ (f –1(y))2 + 4 = y [ f(x) = x2 + 4]


f –1 (y) = y – 4
Therefore, required inverse function is f –1 : [4, ∞) → R defined by
f – 1 (y) = y – 4 6 y ! [4, 3)
5. Determine whether the relation R defined on the set ℝ of all real numbers as R = {(a, b) : a, b ∈ ℝ
and a – b + 3 ∈ S, where S is the set of all irrational numbers}, is reflexive, symmetric and
transitive. [CBSE Ajmer 2015]
Sol. Here, relation R defined on the set R is given as
R = {(a, b) : a, b ∈ R and a – b + 3 ∈ S}
Reflexivity: Let a ∈ R (set of real numbers)

Now, (a, a) ∈ R as a – a + 3= 3∈S
i.e., R is reflexive.

Symmetry: Taking a = 3 and b = 1, we have

(a, b) ! R as a – b + 3 = 3 – 1 + 3 = 2 3 – 1 ! S

But b – a + 3 = 1 – 3 + 3 = 1 ! S & (b, a) ! R
As (a, b) belongs to R but (b, a) does not belong to R
∴ R is not symmetric.

Transitivity: Taking a = 1, b = 2 and c = 3

(a, b) ! R as a – b + 3 = 1 – 2 + 3 ! S & (a, b) ! R

b–c+ 3 = 2 – 3+ 3 = 2 ! S & (b, c) ! R
But a – c + 3 = 1 – 3 + 3 = 1 ! S & (a, c) ! R
As (a, b) and (c, d) belongs to R but (a, c) does not belong to R
∴ R is not transitive.

Hence, R is reflexive but neither symmetric nor transitive.
6. Let f : W → W, be defined as f(x) = x – 1, if x is odd and f(x) = x + 1, if x is even. Show that f is
invertible. Find the inverse of f, where W is the set of all whole numbers. [CBSE (F) 2014]
Sol. One-one:
Case I : When x1, x2 are even number

Now, f(x1) = f(x2) ⇒ x1 + 1 = x2 + 1 ⇒ x1 = x2
i.e., f is one-one.
Case II : When x1, x2 are odd number

Now, f(x1) = f(x2) ⇒ x1 – 1 = x2 – 1 ⇒ x1 = x2
i.e., f is one-one.
Case III : When x1 is odd and x2 is even number

Then, x1 ≠ x2. Also, in this case f(x1) is even and f(x2) is odd and so
f(x1) ≠ f(x2)

Relations and Functions 31


@Cbsebookshub - Join Us on Telegram
i.e. x1 ≠ x2 ⇒ f(x1) ≠ f(x2)
i.e., f is one-one.
Case IV : When x1 is even and x2 is odd number

Similar as Case III, we can prove f is one-one.

Onto:

f (x) = *
x – 1, if x is odd
Given,
x + 1, if x is even
⇒ For every even number ‘y’ of codomain ∃ odd number y + 1 in domain and for every odd

number y of codomain there exists even number y – 1 in domain i.e., f is onto function.
Hence, f is one-one onto i.e., invertible function.
Inverse:
Let f(x) = y
Now, y = x + 1 ⇒ x = y – 1
And, y = x – 1 ⇒ x = y + 1
Therefore, required inverse function is given by

f – 1 (x) = *
x + 1, if x is odd
x – 1, if x is even
7. If the function f : R → R be defined by f(x) = 2x – 3 and g : R → R by g(x) = x3 + 5, then find the
value of (fog)–1 (x). [CBSE Patna 2015]
Sol. Here f : R → R and g : R → R be two functions such that
f(x) = 2x – 3 and g(x) = x3 + 5
 f and g both are bijective (one-one onto) function.

⇒ fog is also bijective function.

⇒ fog is invertible function.

Now, fog(x) = f {(g(x)} ⇒ fog(x) = f(x3 + 5)
⇒ fog(x) = 2(x3 + 5) – 3
⇒ fog(x) = 2x3 + 10 – 3
⇒ fog(x) = 2x3 + 7 ...(i)

For inverse of fog (x)


Let fog (x) = y & x = fog –1 (y)

( i)
& y = 2x 3 + 7 & 2x 3 = y – 7
1
y–7 y–7 3
& 3
x = & x =c m
2 2
1 1
y–7 3 x–7 3
& fog (y) = c
–1
m & fog –1
(x) = c m
2 2
8. Let f : N → R be a function defined as f(x) = 4x2 + 12x + 15.
Show that f : N → S is invertible, where S is the range of f. Hence, find inverse of f.
[CBSE (F) 2015]
Sol. Let y ∈ S, then y = 4x2 + 12x + 15, for some x ∈ N
( y – 6) – 3
⇒ y = (2x + 3) 2 + 6
& x=
2
, as y 2 6

32 Xam idea Mathematics–XII

@Cbsebookshub - Join Us on Telegram


( y – 6) – 3
Let g: S " N is defined by g (y) =

2
(2 x + 3 ) 2 – 3

` gof (x) = g (4x 2 + 12x + 15) = g ((2x + 3) 2 + 6) = =x
2
2

fog (y) = f d n=< + 3F + 6 = y


( y – 6) – 3 2 [( y – 6 ) – 3]

and
2 2

Hence, fog (y) = IS and gof (x) = IN


y – 6 –3
f is invertible and its inverse is given by f –1 (y) = g (y) =
2
9. Let f : R – ' – 1 " R be a function defined as f (x) = . Show that, in f : R – ' – 1 "
4 4x 4

3 3x + 4 3
Range of f, f is one-one and onto. Hence find f –1: Range f → R – ' – 1 .
4
[CBSE 2017(C)]
3
4
Sol. Let x1, x2 ∈ R – ' – 1
3
4x1 4x2
Now f (x1) = f (x2) & =
3x1 4 3x2 + 4
+
⇒ 12 x1 x2 + 16 x1 = 12 x1 x2 + 16 x2   ⇒ 16 x1 = 16 x2  ⇒  x1 = x2

Hence f is one-one function.
Since, co-domain f is range of f
4
So, f : R – ( – 2 → R is one-one onto function.
3
For inverse
Let f (x) = y
4x
⇒ = y    ⇒  3xy + 4y = 4x
3x + 4
4y
⇒ 
4x – 3xy = 4y ⇒  x(4 – 3y) = 4y  ⇒   x =
4 – 3y
4y
Therefore, f –1 : Range of f → R – {– 4/3} is f –1 (y) = .
4 – 3y

PROFICIENCY EXERCISE
QQ Objective Type Questions: [1 mark each]
1. Choose and write the correct option in each of the following questions.
(i) Let R be a relation on the set N of natural numbers defined by nRm if n divides m. Then R is
(a) reflexive and symmetric (b) transitive and symmetric
(c) equivalence (d) reflexive, transitive but not symmetric
(ii) Let A = {1, 2, 3} and consider the relation R = {(1, 1), (2, 2), (3, 3), (1, 2), (2, 3), (1, 3)}. Then R is
(a) reflexive but not symmetric (b) reflexive but not transitive
(c) symmetric and transitive (d) neither symmetric nor transitive
(iii) If a relation R on the set {1, 2, 3} be defined by R = {(1, 2)}, then R is
(a) reflexive (b) transitive (c) symmetric (d) none of these

Relations and Functions 33


@Cbsebookshub - Join Us on Telegram
(iv) Let f : R → R be defined by f(x) = sin x and g : R → R be defined by g(x) = x2, then fog is
sin x
(a) x2 sin x (b) (sin x)2 (c) sin x2 (d)
x2
–1
(v) Let f : R → R be defined by f(x) = 3x – 4. Then f (x) is given by [NCERT Exemplar]
x+4 x
(a) (b) – 4 (c) 3x + 4 (d) none of these
3 3
1
(vi) Let f : R → R be defined by f (x) = 6 x d R. Then f is
x
(a) one-one (b) onto (c) bijective (d) f is not defined
(vii) Let f : R → R be given by f(x) = tan x. Then f –1(1) is

(a) (b) ( nr + : n d Z 2 (c) does not exist


r r
(d) none of these
4 4
(viii) If A = {1, 2, 3}, B = {1, 4, 6, 9} and R is a relation from A to B defined by 'x is greater than y'.
Then range of R is
(a) {1, 4, 6, 9} (b) {4, 6, 9} (c) {1} (d) none of these
2. Fill in the blanks.
(i) A relation R in a set A is called _____________ relation, if (a1, a2) ∈ R implies that (a2, a1) ∈ R,
a1, a2 ∈ A.
(ii) A relation R on set N defined by R = {(x, y) : x + 2y = 8}. The domain of R is _____________ .
(iii) Let A = {1, 2, 3}. Then the number of relations containing (1, 2) and (1, 3) which are reflexive
and symmetric but not transitive, is _____________ .
x
(iv) Let f : R → R be defined by f(x) = . Then (fofof) (x) = _____________ .
1 + x2
 [NCERT Exemplar]
QQ Very Short Answer Questions: [1 mark each]
3. If A = {3, 5, 7} and B = {2, 4, 9} and R is a relation from A to B given by “is less than”, then write R
as a set of ordered pairs.
4. Check whether the relation R in the set {1, 2, 3} given by R = {(1, 2), (2, 1)} is transitive.
5. If f(x) = x + 7 and g(x) = x – 7, x ∈ R, then find fog (7). [CBSE Delhi 2008]
3x – 2
6. If f(x) is an invertible function, then find the inverse of f (x) = . [CBSE (AI) 2008]
5
7. If f(x) = 27x3 and g(x) = x1/3, find gof(x). [CBSE (F) 2010]
8. For the set A = {1, 2, 3}, define a relation R in the set A as follows R = {(1, 1), (2, 2), (3, 3), (1, 3)}.
Write the ordered pair to be added to R to make it the smallest equivalence relation.
9. If f : R → R is defined by f(x) = 3x + 2, define f [f(x)]. [CBSE (F) 2011]
2 –1
10. If f(x) = x + 4, then find f (x).
QQ Short Answer Questions–I: [2 marks each]
2 2
11. Let the relation R be defined on the set A = {1, 2, 3, 4, 5} by R = {(a, b) : |a – b | < 8}. Then write
the set R.
12. Let A = {0, 1, 2, 3} and define a relation R on A as follows:
R {(0, 0), (0, 1), (0, 3), (1, 0), (1, 1), (2, 2), (3, 0), (3, 3)}
Is R reflexive? symmetric? transitive?
13. For real numbers x and y, a relation R is defined as xRy if x – y + 2 is an irrational number. Write
whether R is reflexive, symmetric or transitive.

34 Xam idea Mathematics–XII

@Cbsebookshub - Join Us on Telegram


14. Let the function f : R → R be defined by f(x) = 4x – 1, ∀ x ∈ R. Then show that f is one – one.
2x − 1
15. If the function f : R → R, defined by f ( x) = x ∈ R is one-one and onto function then find the
3
inverse of f.

QQ Short Answer Questions–II: [3 marks each]


2 2 2 2
16. Show that the relation R in the set N × N defined by (a, b)R(c, d) iff a + d = b + c ∀ a, b, c, d ∈ N,
is an equivalence relation. [CBSE Sample Paper 2016]
17. Show that the relation S in the set R of real numbers, defined as S = {(a, b): a, b ∈ R and a ≤ b3} is
neither reflexive, nor symmetric nor transitive. [CBSE Delhi 2010]
18. Prove that the relation R in the set A = {1, 2, 3, ..., 12} given by R = {(a, b) : |a – b| is divisible by 3},
is an equivalence relation. Find all elements related to the element 1. [CBSE (F) 2013]
19. Prove that the relation R on the set A = {1, 2, 3, 4, 5, 6, 7} given by R = {(a, b) : |a – b| is even }, is
an equivalence relation. [CBSE 2019 (65/4/2)]
4x + 3 2 2
20. If f (x) = ,x ! then show that fof (x) = x, 6 x ! . What is the inverse of f ?
6x – 4 3 3
 [CBSE (F) 2012]
x
21. If the function f : R → R be given by f(x) = x2 + 2 and g : R → R be given by g (x) = , x ! 1 , find
x–1
fog and gof and hence find fog(2) and gof(–3). [CBSE (AI) 2014]
22. Show that the relation R on R defined as R = (a, b) : a ≤ b}, is reflexive and transitive but not
symmetric. [CBSE 2019, (65/1/1)]
23. Let Z be the set of all integers and R be relation on Z defined as R = {(a, b) : a, b ∈ Z and (a – b) is
divisible by 5}. Prove that R is an equivalence relation. [CBSE Delhi 2010]
2 4x + 3
24. Show that the function f in A = R −   defined as f ( x ) = is one-one and onto. Hence,
 3 6 x − 4
find f –1.
25. Prove that the function f : N → N, defined by f (x) = x2 + x + 1 is one-one but not onto. Find inverse
of f : N → S, where S is range of f. [CBSE 2019, (65/1/1)]

QQ Long Answer Questions: [5 marks each]


26. Let A = {x ∈ Z : 0 ≤ x ≤ 12}. Show that R = {(a, b) : a, b ∈ A, |a – b|is divisible by 4} is an equivalence
relation. Find the set of all elements related to 1. Also write the equivalence class [2].
[CBSE 2018]
27. Let f : N → N be a function defined as f(x) = 9x2 + 6x – 5. Show that f : N → S, where S is the range
of f, is invertible. Find the inverse of f and hence find f –1 (43) and f –1 (163). [CBSE Delhi 2016]
x
28. Show that the function f : R → R defined by f (x) = 2 , 6 x d R is neither one-one nor onto.
x +1
Also, if g : R → R is defined as g(x) = 2x – 1, find fog(x). [CBSE 2018]
29. Let f : N → R, be a function defined as f(x) = 4x2 + 12x + 15. Show that f : N → S, where S is the
range of f, is invertible. Also find the inverse of f. [CBSE (F) 2015]
30. Show that the relation R defined by (a, b) R (c, d) ⇔ a + d = b + c on the A × A, where A = {1, 2, 3, ...,
10} is an equivalence relation. Hence write the equivalence class of [(3, 4)]; a, b, c, d ∈ A.
 [CBSE (East) 2016]
31. Let f : N → N be a function defined as f(x) = 4x2 + 12x +15. Show that f : N → S is invertible (where
S is range of f). Find the inverse of f and hence find f –1(31) and f –1(87). [CBSE (South) 2016]

Relations and Functions 35


@Cbsebookshub - Join Us on Telegram
32. Let f : W → W be defined as

f (n) = *
n – 1, if n is odd

n + 1, if n is even
Show that f is invertible and find the inverse of f. Here, W is the set of all whole numbers.
 [CBSE (Panchkula) 2015]
33. If the function f : R → R be defined by f(x) = 2x – 3 and g : R → R by g(x) = x3 + 5, then find fog and
show that fog is invertible. Also, find (fog)–1, hence find (fog)–1 (9). [CBSE Sample Paper 2018]

Answers
1. (i) (d) (ii) (a) (iii) (b) (iv) (c) (v) (a) (vi) (d)
(vii) (b) (viii) (c)
x
2. (i) symmetric (ii) {2, 4, 6} (iii) 1 (iv)
3x 2 + 1
5x + 2
3. R = {(3, 4), (3, 9), (5, 9), (7, 9)} 4. No, it is not transitive. 5. 7 6.
3
7. 3x 8. (3, 1) 9. 9x + 8 10. f –1 (x) = x – 4
11. {(1, 1), (1, 2), (2, 1), (2, 2), (2, 3), (3, 2), (3, 3), (3, 4), (4, 3), (4, 4), (5, 5)}
12. Reflexive, Symmetric but not transitive 13. Reflexive but neither symmetric nor transitive.
3y + 1 4x + 3
15. f –1 (y) = 18. 1, 4, 7, 10 20. f –1 (x) =
2 6x – 4

3x 2 – 4x + 2 x2 + 2 11 4x + 3 –1 + 4y – 3
21. , x ! 1; 2 ; 6, 24. f –1 (x) = 25. f –1 (y) =
(x–1) 2
x +1 10 6x – 4 2
–1 + x + 6 –1
26. {1, 5, 9}, {2, 6, 10} 27. f –1 (x) = , f (43) = 2, f –1 (163) = 4
3
2x – 1 x–6 –3
28. 29. f –1 (x) =
4x 2 – 4x + 2 2
30. {(1, 2), (2, 3), (3, 4), (4, 5), (5, 6), (6, 7), (7, 8), (8, 9), (9, 10)}

, f (31) = 1, f –1 (87) = 3 32. f –1 (x) = )


y – 6 – 3 –1 x + 1, if x is odd
31. f –1 (y) =
2 x – 1, if x is even
1

33. fog (x) = 2x 3 + 7, (fog) –1 (x) = c m , (fog) –1 (9) = 1


x–7 3
2

SELF-ASSESSMENT TEST
Time allowed: 1 hour Max. marks: 30
1. Choose and write the correct option in the following questions. (4 × 1 = 4)
(i) In the set Z of all integers, which of the following relation R is not an equivalence relation?
(a) x R y : if x ≤ y (b) x R y : if x = y
(c) x R y : if x – y is an integer (d) x R y : if x ≅ y (Mod 3)
(ii) Let A = {1, 2, 3} and R = {(1, 2), (2, 3), (1, 3)} be a relation on set A. Then R is
(a) neither reflexive nor transitive (b) neither symmetric nor transitive
(c) transitive (d) None of these

36 Xam idea Mathematics–XII

@Cbsebookshub - Join Us on Telegram


(iii) If f : N → S where S is the range of f and given by f(x) = 2x2 – 4x + 7 is an invertible function.
Then f –1(x) is
x+5 x–2 x–5
(a) + 1, x $ 5 (b) + 1 (c) + 1, x $ 5 (d) None of these
2 5 2
7
(iv) If f(x) = [x] and g(x) = |x|, find fog d n .
2
(a) –3 (b) 3 (c) 4 (d) –4
2. Fill in the Blanks. (2 × 1 = 2)
(i) Total number of all onto functions from the set {1, 2, 3, ..., n} to itself is _____________ .
(ii) Let f = {(1, 2), (3, 5), (4, 1)} and g = {(2, 3), (5, 1), (1, 3)}. Then gof = _____________ .
QQ Solve the following questions. (2 × 1 = 2)
3. Check the injectivity of the function f : R → R, defined by f(x) = 2 – 3x.
4. Show that the function f : ℝ → ℝ defined by f(x) = |x|, for x ∈ ℝ is not surjective.
QQ Solve the following questions. (4 × 2 = 8)
3
5. Show that the function f : ℝ → ℝ defined by f(x) = x + 2 for all x is a bijective function.
6. Let R be a relation on the set of all lines in a plane defined by ‘is parallel to’, i.e., (l1, l2) ∈ R ⇔ l1||l2.
Show that R is an equivalence relation.
7. If f(x) = 27x3 and g(x) = x1/3, find gof(x).
4x + 3 2 2
8. If f (x) = ,x ! then show that fof (x) = x, 6 x ! . What is the inverse of f ?
6x – 4 3 3
QQ Solve the following questions. (3 × 3 = 9)
9. Let A = {x ∈ ℝ: –1 ≤ x ≤ 1}. Show that f : A → B given by f(x) = x|x| is a one-one and onto function.
10. Show that the function f : R → R given by f(x) = x3 + x is a bijection.
2 2x – 4
11. Let A = R – {3} and B = R – ' 1 . If f : A " B : f (x) = , then prove that f is a bijective
3 3x – 9
function.
QQ Solve the following question. (1 × 5 = 5)
12. Show that the relation R on the set A of points in a plane, given by
R = {(P, Q) : distance of the point P from the origin = distance of point Q from origin}

is an equivalence relation.
Further, show that the set of all points related to a point P ≠ (0, 0) is the circle passing through P
with origin as centre.

Answers
1. (i) (a) (ii) (c) (iii) (c) (iv) (b)
2. (i) n! (ii) gof = {(1, 3), (3, 1), (4, 3)}
4x + 3
3. Yes 7. 3x 8. f –1 (x) =
6x – 4
zzz

Relations and Functions 37


@Cbsebookshub - Join Us on Telegram
2 Inverse
Trigonometric
Functions

1. Definition: If f : X → Y is one-one onto (bijective) function, then there exists a unique function
f –1 : Y → X which assigns each element y ∈ Y to a unique element x ∈ X such that f(x) = y and is
called inverse function of f.
–1
i.e.,
f (y) = x ⇔ f(x) = y, x ∈ X and y ∈ Y

Principal value branches: Since trigonometric functions being periodic are in general not bijective
(one-one onto) and thus for existence of inverse of trigonometric function we restrict their domain
and co-domain to make it bijective. This restriction of domain and range gives principal value
branch of inverse trigonometric function which are as follows:
Range
Functions Domain
(Principal value branch)
–r r
y = sin–1 x [– 1, 1] : , D
2 2
y = cos–1 x [– 1, 1] [0, p]
–r r
y = cosec–1 x R – (– 1, 1) : , D – { 0}
2 2

[0, r] – % /
r
y = sec–1 x R – (– 1, 1)
2

a– , k
r r
y = tan–1 x R
2 2
y = cot–1 x R (0, p)
The value of an inverse trigonometric function which lies in its principal value branch is called the
principal value of that inverse trigonometric function.
2. Principal and general values:
r r
(a) If sin q = sin a then its principal value is i = a, – < a < and its general value is
n 2 2
q = np + (–1) a, n ∈ Z
(b) If cos q = cos a then its principal value is q = a, 0 < a < p and its general value is
q = 2np ± a, n ∈ Z
r r
(c) If tan q = tan a then its principal value is i = a, – 1 a 1 and its general value is
2 2
q = np + a, n ∈ Z

Properties of Inverse Trigonometric Functions


1. (i) sin–1(sin q) = q, for all q ∈ [– p/2, p/2]

38 Xam idea Mathematics–XII

@Cbsebookshub - Join Us on Telegram


(ii) cos–1(cos q) = q, for all q ∈ [0, p]
(iii) tan–1(tan q) = q, for all q ∈ (– p/2, p/2)
(iv) cosec–1(cosec q) = q, for all q ∈ [– p/2, p/2], q ! 0
(v) sec–1(sec q) = q, for all q ∈ [0, p], q ≠ p/2
(vi) cot–1(cot q) = q, for all q ∈ (0, p)
2. (i) sin (sin–1 x) = x, for all x ∈ [–1, 1]
(ii) cos (cos–1 x) = x, for all x ∈ [–1, 1]
(iii) tan (tan–1 x) = x, for all x ∈ R
(iv) cosec (cosec–1 x) = x, for all x ∈ (– ∞, –1] ∪ [1, ∞)
(v) sec (sec–1 x) = x, for all x ∈ (– ∞, –1] ∪ [1, ∞)
(vi) cot (cot–1 x) = x, for all x ∈ R
3. (i) sin –1 b x l = cosec –1 x, for all x ∈ (– ∞, –1] ∪ [1, ∞)
1

(ii) cos –1 b x l = sec –1 x, for all x ∈ (– ∞, –1] ∪ [1, ∞)


1

1
(iii) tan –1 c m = ) cot x,
–1
for all x > 0

x –1 for all x < 0
–r + cot x,

4. (i) sin–1 (–x) = – sin–1 x, for all x ∈ [–1, 1]


(ii) cos–1 (–x) = p – cos–1 x, for all x ∈ [–1, 1]
(iii) tan–1 (–x) = – tan–1 x, for all x ∈ R
(iv) cosec–1 (–x) = – cosec–1 x, for all x ∈ (– ∞, –1] ∪ [1, ∞)
(v) sec–1 (–x) = p – sec–1 x, for all x ∈ (– ∞, –1] ∪ [1, ∞)
(vi) cot–1 (–x) = p – cot–1 x, for all x ∈ R
r
5. (i) sin – 1 x + cos –1 x =
, for all x ∈ [–1, 1]
2
r
(ii) tan –1 x + cot –1 x = , for all x ∈ R
2
r
(iii) sec –1 x + cosec –1 x = , for all x ∈ (– ∞, –1] ∪ [1, ∞)
2

6. (i) sin –1 x + sin –1 y if –1 ≤ x, y ≤ 1 and x2 + y2 ≤ 1


or
= sin –1 {x 1 – y 2 + y 1 – x 2 }, if xy < 0 and x2 + y2 > 1
–1 –1
(ii) sin x – sin y if –1 ≤ x, y ≤ 1 and x2 + y2 ≤ 1
or
= sin –1 {x 1 – y 2 – y 1 – x 2 }, if xy > 0 and x2 + y2 > 1
–1 –1
7. (i) cos x + cos y
= cos –1 {xy – 1 – x 2 1 – y 2 }, if –1 ≤ x, y ≤ 1 and x + y ≥ 0

–1 –1
(ii) cos x – cos y
–1 2 2
= cos {xy + 1 – x 1 – y }, if –1 ≤ x, y ≤ 1 and x ≤ y

Inverse Trigonometric Functions 39


@Cbsebookshub - Join Us on Telegram
x+y
8. (i) tan –1 x + tan –1 y = tan –1 d n, if xy < 1
1 – xy
x–y
(ii) tan –1 x – tan –1 y = tan –1 d n, if xy > – 1
1 + xy

9. (i) 2 sin –1 x = sin –1 _ 2x 1 – x 2 i,


1 1
if – #x#
2 2

(ii) 2 cos –1 x = cos –1 ^ 2x 2 – 1 h , if 0 ≤ x ≤ 1

2x
(iii) 2 tan –1 x = tan –1 d n , if – 1 < x < 1
1 – x2
1 1
10. (i) 3sin–1 x = sin–1 (3x – 4x3),
if – # x #
2 2
1
(ii) 3cos–1 x = cos–1 (4x3 – 3x),
if #x#1
2
3x – x 3
(iii) 3 tan –1 x = tan –1 e o ,
1 1
if –
2
<x<
1 – 3x 3 3
2x
11. (i) 2 tan –1 x = sin –1 d n , if –1 ≤ x ≤ 1
1 + x2
1 – x2
(ii) 2 tan –1 x = cos –1 e o , if 0 ≤ x < ∞
1 + x2
x
12. (i) sin –1 x = cos –1 _ 1 – x 2 i = tan –1 f p
1 – x2

1 – x2
= cot –1 e o = sec –1 e
1 –1 1
o = cosec c m
x 1– x 2 x

1 – x2
(ii) cos –1 x = sin –1 _ 1 – x 2 i = tan –1 e o
x
x –1 1
n = sec d n = cosec e
1
= cot –1 d o
–1

1– x 2 x 1 – x2

x 1
(iii) tan –1 x = sin –1 d n = cos e o
–1
1 + x2 1 + x2

1 + x2
= cot –1 c m = sec –1 _ 1 + x 2 i = cosec –1 e o
1
x x
Important substitution to simplify trigonometrical expressions involving inverse trigonometrical
functions.
Expression Substitution
2 2
a +x x = a tan q or x = a cot q
2 2
a –x x = a sin q or x = a cos q
2 2
x –a x = a sec q or x = a cosec q
a+x a–x
or x = a cos 2q
a–x a+x

40 Xam idea Mathematics–XII

@Cbsebookshub - Join Us on Telegram


Important Facts:
(i) If no branch of an inverse trigonometric function is mentioned, we mean the principal value branch
of that function.
1
or ^sin x h and same holds true for other trigonometric functions also.
–1
(ii) sin –1 x !
sin x
(iii) If sin–1 x = y then x and y are the elements of domain and range of principal value branch of sin–1
respectively.
y ! :– , D
r r
i.e., x ∈ [–1, 1] and
2 2
Similar fact is also applicable for other inverse trigonometric functions.

Selected NCERT Questions


1. Find the principal values of cos –1 d n.
3

2

Sol. Let cos –1 d n = i so that i ! 60, r@ and cos i =


3 3
.
2 2

, i.e., cos –1 d n=
r r 3 r
&
cos i = cos
6
& i=
6 2 6
2. Find the principal values of tan –1 (– 3 ) .

Sol. Let tan –1 (– 3 ) = i where i ! c , m


–r r
2 2

tan i = – tan c m
r
&
tan i = – 3 & 3

tan i = tan d – n
r r
&

3
& i=–
3
r
&
tan –1 (– 3 ) = – .
3
3. Write the following function in the simplest form:

tan –1 c m, 0 < x < r


1 – cos x

1 + cos x
JK x NO
KK 2 sin 2 c m OO
Sol. tan –1 c m = tan –1 KK
1 – cos x K 2 O
OO
1 + cos x
c
2 x O
m
KK O
K 2 cos
2 O
L P
= tan –1 d tan c m n = tan –1 c tan m = ;Note that 0 < x < r , 0 < < E
x x x x r

2 2 2 2 2
4. Write the following function in the simplest form:

tan –1 c m, – < x <


cos x – sin x r 3r

cos x + sin x 4 4
r 3r
Note that – <x<
Sol. tan –1 c m
cos x – sin x 4 4
cos x + sin x r 3r
, >–x>–
4 4
Inside the bracket divide Nr and Dr by cos x
r r r
, > –x>–
2 4 2
= tan –1 c m = tan –1 ( tan c – x m2 = – x
1 – tan x r r
1 + tan x 4 4

Inverse Trigonometric Functions 41


@Cbsebookshub - Join Us on Telegram
1 – y2
5. Find the value of the tan f sin –1 p, | x | < 1, y > 0 and xy < 1.
1 2x –1
+ cos
2 1 + x2 1 + y2
[CBSE Delhi 2013]
2
Sol. tan * f sin –1 p4 = tan ' 2 (2 tan –1 x + 2 tan –1 y) 1
1 2x 1 – y 1
+ cos –1
2 1+x 2
1+y 2

2
>a sin –1 2x 2 = 2 tan –1 x and cos –1 = 2 tan –1 yH
1– y
= tan ' ×2 (tan x + tan y) 1
1 –1 –1
2 1+x 2
1+y
x+y x+y
= tan * tan –1 f
p4 = .
1 – xy 1 – xy

1 1 1 1 r
6. Prove that: tan –1 + tan –1 + tan –1 + tan –1 = [CBSE Delhi 2008, (AI) 2010, 2016, 2017]
5 7 3 8 4
x+y
Sol. We know that tan –1 x + tan –1 y = tan –1 f p for xy < 1, therefore,
1 – xy

= c tan –1 + tan –1 m + c tan –1 + tan –1 m


1 1 1 1
LHS
5 7 3 8
JK 1 1 NO JK 1 1 NO
KK + OO KK + O
5 7 O 3 8 OO
= tan –1 KKK OO + tan –1 KKK O
KK 1 – 1 × 1 OO KK 1 – 1 × 1 OOO
K 5 7 OP K 3 8 OP
L L
= tan –1 c m + tan –1 c m = tan –1 c m + tan –1 c m
12 11 6 11
34 23 17 23
JK 6 11 NO
KK + O
23 OO
n = tan –1 c m = tan –1 1 = = RHS
6×23 + 11×17
OO = tan –1 d
K 17 325 r
= tan –1 KK
KK 1 – 6 11 OO 17× 23 – 6 × 11 325 4
K × O
L 17 23 P
cos –1 c + m, x d [0, 1]
–1 1 1–x
7. Prove that: tan x =
2 1 x
r
Sol. Let tan –1 x = i then tan i = x and 0 # i <
2
1 – tan 2 i 1 – tan 2 i
cos –1 c + m = cos –1 e o
1– x
>a cos 2i = H
1 1
Now,
2 1 x 2 1 + tan 2 i 1 + tan 2 i
;a 0 # i < , ` 0 # 2i # rE
1 1 r
cos –1 (cos 2i) = (2i) = i = tan –1 x
=
2 2 2
Note that the result is valid for all x $ 0.
In particular, it is valid for all x ! [0, 1] .

o = , x ! a 0, k
1 + sin x + 1 – sin x
8. Prove that: cot –1 e
x r

+
1 sin x – 1 – sin x 2 4
[CBSE Delhi 2011, 2014; (AI) 2009; (F) 2016]
1 + sin x + 1 – sin x
= cot –1 e o, x ! a 0, k   Given
r
Sol. LHS
+
1 sin x – 1 – sin x 4

^cos x/2 + sin x/2 h + ^cos x/2 – sin x/2 h


2 2
= cot –1 f p
^cos x/2 + sin x/2 h – ^cos x/2 – sin x/2 h
2 2

42 Xam idea Mathematics–XII

@Cbsebookshub - Join Us on Telegram


Z] _ Given
]] cos x + sin x + cos x – sin x bbb RS VW
]] 2 2 2 2 bb SS 0 < x < r WW
= cot –1 [] ` SS 4 WW
]] x x x x bbb SS WW
cos + sin – cos – sin
]]
2 2 2 2 b
b SS& 0 < x < r WW
\ a SS 2 8 WW
+ + SS WW
= cot e
–1 (cos x/2 sin x/2) (cos x/2 – sin x/2)
o a k
SS& x r WW
^cos x/2 + sin x/2 h – (cos x/2 – sin x/2) SS ! 0, 1 ( 0 , r ) WW
2 8
SS WW
SS x x WW
cos x/2 x & cos – sin > 0
= cot –1 d n = cot –1 (cot x/2) = = RHS SS
SS
2 2 WW
W
sin x/2 2
SS x x x x WWW
SS& cos 2 – sin 2 = cos 2 – sin 2 WW
T X
1+x – 1– x
9. Prove that: tan –1 e o=
r 1 1
– cos –1 x, – #x#1 [CBSE (AI) 2011, 2014]
1+x+ 1– x 4 2 2
1+x – 1– x
= tan –1 e
Sol. LHS o
1+x+ 1– x
1+x – 1– x 1+x – 1– x
= tan –1 e × o [Rationalize]
1+x+ 1– x 1+x – 1– x
RS VW
2 – 2 1 – x2 1 – 1 – x2 SSa – 1 # x # 1 WW
= tan –1 e o = tan –1 e o SS 2 WW
1+x –1+x x SS W
r WW
a k
SS r
Put x = sin q ⇒ q = sin–1 x SS & sin –
4
# sin i # sin
2 WW
W
SS WW
JK i NO SS r r WW
KK 2 sin 2 OO SS& – 4 # i # 2 WW
= tan –1 d
1 – cos
n = tan –1 KK
i 2 OO
KK SS WW
sin i i i OO SS r i r WW
K 2 sin . cos O SS& – # # WW
L 2 2P 8 2 4
SS WW
= tan –1 c tan m = = sin –1 x = a – cos –1 x k
i i 1 1 r SS WW
! a– , k
i r r
2 2 2 2 2 SS& WW
SS 2 2 2 WW
r 1 SS r WW
= – cos –1 x = RHS SSa cos x + sin x =
– 1 – 1 WW
4 2 S 2 W
T X
9r 9 1 9 2 2
10. Prove that: – sin –1 = sin –1 [CBSE (F) 2011, 2020 (65/4/1)]
8 4 3 4 3
9r 9 1 9 r 1
=
Sol. LHS – sin –1 = c – sin –1 m
8 4 3 4 2 3
9 1 1
= cos –1 c m ... (i) ;a ! [–1, 1]E
4 3 3
Let, cos –1 c m = i
1 1

3
& cos i =
3
1 2
& sin i =
1–c m =
a i ! [0, r ]
G
3 & sin i is positive
8 2 2
& sin i =
9
& sin i =
3

<a ! [–1, 1]F


2 2 2 2
& i = sin –1

3 3
1 2 2
& cos –1 c 3 m = sin –1 3

9 2 2
∴ From equation (i), we have
sin –1 = RHS
4 3

Inverse Trigonometric Functions 43


@Cbsebookshub - Join Us on Telegram
1–x 1
11. Solve: tan –1 = tan –1 x, x > 0 [CBSE (AI) 2008; (F) 2011]
1+x 2
1– x 1 1– x
Sol. Given, tan –1 = tan –1 x
1+x 2
& 2 tan –1
1+x
= tan –1 x

2c m
1– x
a x>0
>& –1 < 1 – x < 1H
–1 1+x = tan –1 x

& tan
1– x 2
1–c m 1+x
1+x
2 (1 – x 2) 2 (1 – x 2)
& tan –1

4x
= tan –1 x & 4x
=x

& 1 – x2 = 2x2
& 1 = 3x2
1 1
i.e.,
x2 = ` x= [a x > 0]
3 3
x–1 x+1 r
12. If tan –1 + tan –1 = , then find the value of x.
x–2 x+2 4
[CBSE Delhi 2008; (AI) 2010; (F) 2013, 2016]
x–1 x+1 r
Sol. Given tan –1 + tan –1 =
x–2 x+2 4
RS x – 1 x + 1 VW
SS + WW
<Using tan –1 x ! tan –1 y = tan –1 F
–1 S x – 2 x + 2 WW = r x!y
& tan S S W
1 " xy
SS x – 1 x + 1 WW 4
SS 1 – # W
x – 2 x+2 W
T X
(x – 1) (x + 2) + (x + 1) (x – 2) r
& (x – 2) (x + 2) – (x – 1) (x + 1)
= tan
4
x2 + x – 2 + x2 – x – 2 2 ( x 2 – 2)
&
=1 & –3
=1 & 2x 2 – 4 = –3
x2 – 4 – x2 + 1
1
&
2x 2 = 1 & x2 =
2
1
` x =!
2

Multiple Choice Questions [1 mark]


Choose and write the correct option in the following questions.

1. tan –1 3 + tan –1 m = tan –1 d n is valid for what values of l ?


3+m
[CBSE 2020 (65/2/1)]
1 – 3m
1 1
(a) m ! d – , n (b) m > (c) m < (d) All real values of l
1 1
3 3 3 3

tan –1 ^ 3 h + cos –1 d – n corresponding to principal branches is


1
2. The value of
2
r r
(a) – (b) 0 (c) p (d)
12 3
3. The value of cot (sin–1 x) is [NCERT Exemplar]
2
1+x x 1 1 – x2
(a) (b) (c) (d)
x 1+x 2 x x

44 Xam idea Mathematics–XII

@Cbsebookshub - Join Us on Telegram


4. The value of sin –1 d cos
n is
r
[NCERT Exemplar]
9
r 5r –5r 7r
(a) (b) (c) (d)
9 9 9 18
3 3
5. tan d sin –1
+ tan –1 n is equal to [CBSE 2020 (65/3/3)]
5 4
7 24 3 3
(a) (b) (c) (d)
24 7 2 4
6. Let q = sin–1(sin (–600°)), then value of q is
r r 2r –2r
(a) (b) (c) (d)
3 2 3 3
r
7. If sin –1 x + sin –1 y = , then value of cos–1 x + cos–1 y is
2
r 2r
(a) (b) p (c) 0 (d)
2 3
8. If 3 tan–1 x + cot–1 x = p, then x equals [NCERT Exemplar]
1
(a) 0 (b) 1 (c) –1 (d)
2
1
9. The value of the expression 2 sec –1 2 + sin –1 d n is [NCERT Exemplar]
2
r 5r 7r
(a) (b) (c) (d) 1
6 6 6

o – tan –1 e o is equal to
x–y
10. tan –1 e
x
[NCERT]
y x+y
r r r – 3r
(a) (b) (c) (d)
2 3 4 4
11. The value of tan2 (sec–1 2) + cot2 (cosec–1 3) is
(a) 5 (b) 11 (c) 13 (d) 15
–1
12. The domain of the function defined by f (x) = sin x – 1 is [NCERT Exemplar]
(a) [1, 2] (b) [–1, 1] (c) [0, 1] (d) none of these
–1 –1
13. If a # 2 sin x + cos x # b , then
–r r –r 3r
(a) a = , b = (b) a = 0, b = r (c) a = ,b = (d) a = 0, b = 2r
2 2 2 2
2
2a –1 1 – a
2x
14. If sin –1 e 2 o cos f
+ p = tan –1 e o, where a, x d (0, 1) , then the value of x is
1+a 1+a 2
1 – x2
a 2a
(a) 0 (b) (c) a (d)
2 1 – a2
2
15. If cos e sin –1
+ cos –1 x o = 0 , then x is equal to [CBSE 2020 (65/2/2)]
5
1 2 2
(a) (b) – (c) (d) 1
5 5 5
7
16. The value of cot =cos –1 d nG is [NCERT Exemplar]
25
25 25 24 7
(a) (b) (c) (d)
24 7 25 24

Inverse Trigonometric Functions 45


@Cbsebookshub - Join Us on Telegram
17. sin (tan–1 x), |x|< 1 is equal to
x 1 1 x
(a) 2
(b) 2
(c) (d)
1– x 1– x 1 + x2 1 + x2
1 2
18. If tan –1 + tan –1 = tan –1 a then a is equal to
2 11
1 1 3
(a) (b) (c) (d) 1
4 2 4
19. If cos –1 a + cos –1 b + cos –1 c = 3r , then a (b + c ) + b (c + a ) + c (a + b ) equals
[NCERT Exemplar]
(a) 0 (b) 1 (c) 6 (d) 12
3 1
20. The value of tan d cos –1
+ tan –1 n is
5 4
19 8 19 3
(a) (b) (c) (d)
8 19 12 4

Answers
1. (c) 2. (c) 3. (d) 4. (d) 5. (b) 6. (a)
7. (a) 8. (b) 9. (b) 10. (c) 11. (b) 12. (a)
13. (b) 14. (d) 15. (c) 16. (d) 17. (d) 18. (c)
19. (c) 20. (a)

Solutions of Selected Multiple Choice Questions


3+m
1. Given, tan –1 3 + tan –1 m = tan –1 d n is valid for 3l < 1
1 – 3m
1
    ⇒ m <
3
2. We have,

tan –1 ^ 3 h + cos –1 d – n
1
2

= tan –1 d tan n + cos –1 d – cos n


r r
3 3
2r
= + cos –1 =cos d r – nG = + cos –1 d cos n
r r r
3 3 3 3
r 2r 3r
= + = =r
3 3 3
3. Let sin–1 x = q, then sin q = x.
1 1
⇒ cosec i =
x
& cosec 2 i =
x2
1 1 – x2

1 + cot 2 i = & cot i =
x
x2

4. sin –1 d cos n = sin –1 d sin d – nn


r r r
9 2 9
7r
= sin –1 d sin n=
7r
18 18
3 3 9 4
5. Let sin –1 = i   ⇒  sin i =   ⇒  cos i = 1– =
5 5 25 5

46 Xam idea Mathematics–XII

@Cbsebookshub - Join Us on Telegram


sin i 3 3
∴ tan i =
=   ⇒  i = tan –1
cos i 4 4
Now,
3 3 3 3
tan d sin –1 + tan –1 n = tan d tan –1 + tan –1 n
5 4 4 4
JK 3 O N
KK 2# OO
3
= tan d 2 tan n = tan Ktan
–1 K
K – 1 4 OO
2O
4 KK 3 O
KK 1 – d n OOO
4
L P
24 24
= tan d tan –1 n=
7 7
8. Given, 3 tan–1 x + cot–1 x = p

2 tan–1 x + tan–1 x + cot–1 x = p
<a tan –1 x + cot –1 x = F
r r

2 tan –1 x = r –
2 2
–1 r

2 tan x =
2
–1 r

tan x =
4
r

x = tan = 1
4

x=1
Hence, only x = 1 satisfies the given equation.
9. We have,
1
2 sec –1 (2) + sin –1 d n
2

= 2 sec –1 d sec d nn + sin –1 d sin d nn


r r
3 6
r r 2r r 5r
= 2 # + = + =
3 6 3 6 6
JK x x – y NO
KK – O
x–y K y x + y OOO x–y
10. tan a y k – tan
–1 x –1 d n –1 K
= tan KK OO <Here x . > –1F
x+y KK + x x – y OO y x+y
K1 y . x + y O
L P
2
+ xy – xy + y 2 y (x + y)
= tan –1 f p
x
#
y ( x + y) xy + y 2 + x 2 – xy
x2 + y2
= tan –1 f p = tan –1 (1) =
r
2
x +y 2 4

12.  f (x) = sin –1 x – 1


⇒ 0 ≤ x – 1 ≤ 1 [
x – 1 $ 0 and –1 # x – 1 # 1 ]
⇒ 1 ≤ x ≤ 2

∴ x ∈ [1, 2]
–r r
13. We have # sin –1 x #
2 2
–r r r r r
⇒ + # sin –1 x + # +
2 2 2 2 2

Inverse Trigonometric Functions 47


@Cbsebookshub - Join Us on Telegram
⇒ 0 ≤ sin–1 x + (sin–1 x + cos–1 x) ≤ p

⇒ 0 ≤ 2 sin–1 x + cos–1 x ≤ p

2
2a –1 1 – a
2x
14. We have, sin –1 e 2 o + cos f p = tan –1 e o
1+a 1+a 2
1 – x2
⇒ 2tan–1 a + 2tan–1 a = 2tan–1 x

⇒ 2tan–1 a = tan–1 x
2a
⇒ tan –1 = tan –1 x
1 – a2
2a
⇒ x =
1 – a2
15. We have,
–1 2
cos e sin + cos –1 x o = 0 = cos
r
5 2
2 r
⇒ sin –1 + cos –1 x =
5 2
2 r
⇒ sin –1 = – cos –1 x = sin –1 x
5 2
2 2
⇒ sin –1 = sin –1 x & x =
5 5
19. We have, cos–1 a + cos–1 b + cos–1 g = 3p
We know that, 0 ≤ cos–1 x ≤ p
cos–1 a + cos–1 b + cos–1 g = 3p
If and only if, cos–1 a = cos–1 b = cos–1 g = p

cos p = a = b = g ⇒ –1 = a = b = g

a = b = g = –1
∴ a(b + g) + b(g + a) + g(a + b) = –1(–1 – 1) – 1(–1 – 1) – 1(–1 – 1)

=2+2+2=6

Fill in the Blanks [1 mark]


3r
1. The value of sin –1 d sin n ____________ .
5
2. The principal value of tan –1 ^– 3 h is ____________ .
3. The value of sin ^tan –1 2 + cot –1 2h is ____________ .
4. If cos _tan –1 x + cot –1 3 i = 0 , then value of x is ____________ .

5. The value of sin –1 =cos )sin –1 d n3G is ____________ .


3

2
1
6. The principal value of cos –1 d – n is ____________ . [CBSE 2020 (65/2/2)]
2
Answers
2r r r 2r
1. 2. – 3. 1 4. 3 5. 6.
5 3 6 3

48 Xam idea Mathematics–XII

@Cbsebookshub - Join Us on Telegram


Solutions of Fill in the Blanks
1. We have,
3r 2r
sin –1 d sin
n = sin –1 =sin d r – nG
5 5
2r
= sin –1 d sin n=
2r
5 5
2. We have,

tan –1 ^– 3 h = tan –1 <– tan F


r
3

= tan –1 d tan d – nn = –
r r
3 3
3. We have,
sin ^tan –1 2 + cot –1 2h = sin
r
=1
2
d a tan –1 x + cot –1 x = n
r
2
4. We have,
cos _tan –1 x + cot –1 3 i = 0 = cos
r
2
r
⇒ tan –1 x + cot –1 3 =

2
r
⇒ cot –1 3 =
– tan –1 x = cot –1 x
2
⇒ cot –1
3 = cot –1 x   ⇒ x= 3

5. We have,

n3G = sin –1 =cos (sin –1 d sin n2G


sin –1 =cos )sin –1 d
3 r
2 3
1
= sin –1 d cos n = sin –1 d n
r

3 2

= sin –1 d sin n =
r r

6 6

n = cos –1 <– cos F = cos –1 d cos d r – nn = cos –1 d cos


1 2r
n=
2r
6. cos –1 d –
r r

2 3 3 3 3

Very Short Answer Questions [1 mark]

1. Find the value of sin –1 <sin c – mF . [CBSE 2020 (65/5/1)]


17r

8

Sol. We have,
mF = sin –1 ;– sin
sin –1 <sin c – E = sin –1 <– sin c 2r + mF
17r 17r r
8 8 8

= sin –1 d – sin n = sin –1 =sin d – nG


r r
8 8
=a – d <– , FF
r r r r
= –
8 8 2 2

Inverse Trigonometric Functions 49


@Cbsebookshub - Join Us on Telegram
2. Write the principal value of tan –1 1 + cos –1 c – m . [CBSE Delhi 2013]
1

2
Sol. tan –1 1 + cos –1 c – m = tan –1 a tan k + cos –1 acos ar – kk
1 r r
2 4 3

= tan –1 a tan k + cos –1 c cos


2r
m
r

4 3

! a – , k and
r 2r 2r
;a ! [0, r]E
r r r
= +
4 3 4 2 2 3
3r + 8r 11r
= =
12 12

3. Write the value of tan c 2 tan –1 m . [CBSE Delhi 2013]


1

5
RS JK 1 NOVWW
SS KK 2× OW
SS –1 K 5 OOWW 2x
Sol. tan c 2 tan –1 1
m = S
tan Stan K K OOW =a 2 tan –1 x = tan –1 d nG
5 KK 1 2 OOWWW 1 – x2
KK 1 – c 5 m OOWW
SS
SS
T L PX
RS J 2 N VW
SS KK OOW
SS –1 KK 5 OOWWW 2 25
= tan SStan KK 24 OOWW = tan =tan –1 d × nG
SS KK OW 5 24
S K 25 OOWW
T L PX
5 5
= tan d tan –1 n=
12 12

4. Write the principal value of cos –1 c cos m.


7r
[CBSE Delhi 2011; (AI) 2009]
6

m = cos –1 c cos c 2r –
Sol. cos –1 c cos mm
7r 5r
6 6
5r
= cos –1 c cos m= ;a ! [0, r]E
5r 5r

6 6 6
4r
5. Find the value of sin –1 c sin m . [CBSE (AI) 2010]
5

Sol. We are given sin –1 c sin m = sin –1 asin ar – kk = sin –1 asin k =


4r r r r
5 5 5 5
1 1
6. Write the principal value of cos –1 c m + 2 sin –1 c m . [CBSE (F) 2014]
2 2

Sol. We have, cos –1 c m = cos –1 a cos k


1 r
2 3

= :a ! [0, r]D
r r

3 3

sin –1 c m = sin –1 a sin k


Also, 1 r

2 6

= :a ! :– , DD
r r r r

6 6 2 2

` cos –1 c m + 2 sin –1 c m = + 2 a k = + =
1 1 r r r r 2 r

2 2 3 6 3 3 3
[Note: Principal value branches of sin x and cos x are :– , D and [0, r] respectively.]
r r

2 2

50 Xam idea Mathematics–XII

@Cbsebookshub - Join Us on Telegram


7. Write the value of cot (tan–1 a + cot–1 a). [CBSE (F) 2012]
r r
Sol. cot (tan –1 a + cot –1 a) = cot = 0 [a tan –1 x + cot –1 x = 6 x ! R]
2 2
3
8. Write the value of sin c 2 sin –1 m . [CBSE (F) 2013]
5

Sol. Let sin c 2 sin –1 m=y


3
5
3
&
2 sin –1
5
= sin –1 y

sin –1 ( 2 # 2 = sin –1 y
3 9
&

5
1–
25
[a 2 sin –1 x = sin –1 {2x 1 – x 2 }]

6 4 24
&
sin –1 ' # 1 = sin –1 y & sin –1 c m = sin –1 y
5 5 25
24 3 24
&
y= & sin c 2 sin –1 m=
25 5 25
7r
9. Write the principal value of tan –1 c tan m . [CBSE (F) 2013]
6

Sol. tan –1 c tan m = tan –1 a tan a r + kk


7r r
6 6

= tan –1 a tan k = :a ! a – , kD
r r r r r
6 6 6 2 2

10. If sin c sin –1 + cos –1 x m = 1, then find the value of x. [CBSE Delhi 2014]
1
5

Sol. Given sin c sin + cos –1 x m = 1


–1 1
5
1 1 r
&
sin –1
5
+ cos –1 x = sin –1 1 & sin –1
5
+ cos –1 x =
2
1 r 1 1
&
sin –1 = – cos –1 x
5 2
& sin –1
5
= sin –1 x & x=
5
.

43r
11. Find the value of sin –1 c cos c mm . [NCERT Exemplar]
5
r 3r
Sol. sin –1 c cos c 8r + mm = sin –1 c cos m = sin –1 c sin c – mm
3r 3r
5 5 2 5

= sin –1 a sin a – kk = – :a – ! :– , DD
r r r r r

10 10 10 2 2
12. Find the principal value of cos–1 [cos (– 680°)]. [NCERT Exemplar]
–1 –1
Sol. cos [cos (– 680°)] = cos [cos (680°)] [ cos (– q) = cos q]
= cos–1 [cos (720° – 40°)] = cos–1 [cos (4p – 40°)] = cos–1 (cos 40°)

;a 40° = ! [0, r]E


2r 2r
= 40° or
9 9

Inverse Trigonometric Functions 51


@Cbsebookshub - Join Us on Telegram
Short Answer Questions-I [2 marks]
1
1. Write cot –1 d 2 n , |x|> 1 in simplest form. [CBSE (F) 2013]
x –1
1
Sol. cot –1 e o
2
x –1
Let x = sec q ⇒ q = sec–1 x
1 1
Now, cot –1 e o = cot e
–1
o
2
x –1 sec 2 i – 1
1
= cot –1 c m = cot –1 (cot i) = i = sec –1 x
tan i
cos x
2. Express tan –1 d n, –
3r r
<x< in the simplest forms. [CBSE 2020 (65/3/1)]
1 – sin x 2 2
Sol. We have,

tan –1 c m, –
cos x 3r r
<x<
1 – sin x 2 2
JK x x N
KK cos 2 – sin 2 OOO
K 2 2 OO 3r x r
= tan –1 KK 2 O, – < <
c m
KK x x OO 4 2 4
K cos – sin
2 2 O
L P
KJK cos x + sin x ONO
K 2 2 OO 3r x r
= tan –1 KKK O, – < <
KK cos x – sin x OOO 4 2 4
L 2 2 P
JK x NO
KK 1 + tan OO
O, ;Dividing N and D by cos in the bracketE
2O x
= tan –1 KKK
r r
KK 1 – tan x OOO 2
L 2P

= tan –1 d tan c + mn, – < + <


r x r r x r
4 2 2 4 2 2
r x
= +
4 2
r
3. If 2 tan–1(cos q) = tan–1(2 cosec q), then show that i = . [NCERT Exemplar]
4
–1 –1
Sol. We have, 2 tan (cos q) = tan (2 cosec q)
2 cos i 2x
⇒ tan –1 e
2 o = tan –1 (2 cosec i) >a 2 tan –1 x = tan –1 e oG
1 – cos i 1 – x2
2 cos i
⇒ e
o = 2 cosec i
sin 2 i
⇒ cot i . 2 cosec i = 2 cosec i
& cot i = 1
r r
⇒ cot i = cot

4
& i= 4

4. Write the value of tan –1 =2 sin d 2 cos –1 nG . [CBSE (AI) 2013]


3

2

52 Xam idea Mathematics–XII

@Cbsebookshub - Join Us on Telegram


Sol. tan –1 =2 sin d 2 cos –1 nG = tan –1 a 2 sin a 2 # kk
3
<a cos –1 = F
r 3 r
2 6 2 6

k = tan –1 d 2 # n
3
= tan –1 a 2 sin
r
3 2
r
= tan –1 ( 3 ) =
3

5. What is the principal value of cos –1 c cos m + sin –1 c sin m ? [CBSE (AI) 2011]
2r 2r

3 3

Sol. cos –1 c cos m + sin –1 c sin m = cos –1 c cos m + sin –1 d sin c r – m n <a g c – , mF
2r 2r 2r r 2r r r
3 3 3 3 3 2 2

= cos –1 c cos m + sin –1 c sin m =


2r r 2r r
   +
3 3 3 3
RS r r VWW
3r
SSa sin –1 (sin x) = x
SS if x ! ;– , EW
   = =r 2 2 WW
3 SS W
Sand cos –1 (cos x) = x if x ! [0, r] WW
T X
1 1
6. Find the value of 4 tan –1– tan –1 . [NCERT Exemplar]
5 239
1 1 1 1
Sol. We have, 4 tan –1 – tan –1 = 2 . 2 tan –1 – tan –1
5 239 5 239
RS 2 VW
SS WW
SS –1 WW 2x
oG
5 1
= 2. SStan 2W W – tan –1 >a 2 tan –1 x = tan –1 e
1 x2
d
239
n
SS WW 1 –
SS 1 –
5 WW
T X
RS J V
SS K
KK
2 ONOWW
W
S 5 OOWW 1
= 2 . SStan –1 KKK OOW – tan –1
SS KK 1 – 1 OOWW 239
SS K OWW
L 25 PX
T
= 2. =tan –1 d nG – tan –1
2 / 5 1

24/25 239
5 1
– tan –1 = 2 tan –1
12 239
JK 5 NO
KK 2× O
–1 K K 12 OOO –1 1
= tan K 2 O – tan
KK 5 O
KK 1– d 12 n OOO
239
L P
120 1
= tan –1 – tan –1
119 239
JK 120 1 NO
KK – O
119 239 OO 28561
= tan –1 KKK O = tan –1 d n
KK 1 + 120 # 1 OOO 28561
K 119 239 OP
L
r
    = tan –1 (1) =
4

Inverse Trigonometric Functions 53


@Cbsebookshub - Join Us on Telegram
3 – 3x 2
≤ x ≤ 1 then cos –1 x + cos –1 = + G=
1 x r
7. Prove that if .[CBSE Sample Paper 2018]
2 2 2 3
Sol. Let cos–1 x = θ ⇒ x = cos θ
3 – 3x 2
Now, cos –1 x + cos –1 = + G = i + cos –1 < . cos i + 1 – cos 2 iF
x 1 3
2 2 2 2

= i + cos –1 :cos . cos i + sin . sin iD


r r
3 3
RS
= i + cos % cos a – i k/ –1 r SSa 1/2 # x # 1
3 SScos –r/3 # cos i # cos 0
SS
r SS& –r/3 # i # 0
= i + – i SS
3
SS& r/3 $ i $ 0
=
r SS& i ! 60, r/3@]
3 T
r
8. If tan –1 x + tan –1 y = , xy < 1, then write the value of x + y + xy. [CBSE (AI) 2014]
4
x+y
tan –1 > H=
r r
Sol. Given tan –1 x + tan –1 y =
4
& 4
[a xy < 1]
1 – xy
x+y x+y
&
tan –1 > H = tan –1 1 & =1 & x + y = 1 – xy
1 – xy 1 – xy
&
x + y + xy = 1

Short Answer Questions-II [3 marks]


1 3 4– 7
1. Show that: tan c sin –1 m = [CBSE (AI) 2013]
2 4 3

:i ! a – , kD
3
Sol. Let sin –1 = i
4
& sin i = 34 r r
2 2
i
2 tan 2 tan x
2 = 3
& <a sin 2x = F
i 4 1 + tan 2 x
1 + tan 2
2

& 3 + 3 tan2 i2 = 8 tan i2 & 3 tan2 i2 – 8 tan i2 + 3 = 0


i 8 ! 64 – 36 i 8 ! 28
&
tan
2
=
6
& tan
2
=
6
i 8!2 7 i 4! 7
&
tan
2
=
6
& tan
2
=
3
1 3 4– 7 3
&
tan c sin –1 m = ;a i = sin –1 E
2 4 3 4

2. Evaluate: tan '2 tan –1 c m + 1


1 r
[CBSE Ajmer 2015]
5 4
Z] 1 _b
]] 2 × bb
]] –1 5 b
Sol. tan ( 2 tan c m + 2 = tan []tan
–1 1 r –1 b
2
+ tan 1 `b
5 4
1–c m
]] 1 bb
]] bb
5
\ a

54 Xam idea Mathematics–XII

@Cbsebookshub - Join Us on Telegram


  = tan ( tan –1 c × m + tan –1 1 2 = tan ' tan + tan –1 1 1
2 25 –1 5
5 24 12
Z] 5 _b
]] + b
]] –1 12 1 bbb
`b = tan 'tan –1 c m1 = tan 'tan –1 c m1 =
17 12 17 17
  = tan []tan #
]] 5 bb 12 7 7 7
]] 1– # 1 bb
12
\ a
Prove that: cot–1 7 + cot–1 8 + cot–1 18 = cot–1 3
3. [CBSE (F) 2014]
Sol. We have,
LHS = cot–1 7 + cot–1 8 + cot–1 18

= c tan –1 + tan –1 m + tan –1


1 1 1

7 8 18
JK 1 1 NO
KK + OO 1 1 1
= tan –1 KK 7 8 OO + tan –1 ;a # < 1E
KK 1 1 OO 18 7 8
K1 – 7 # 8 O
L P
KJK 3 + 1 ONO
K OO
=a tan –1 x + tan –1 y = tan –1 e oG
–1 3 –1 1 –1 K 11 18 x+y
= tan + tan = tan K OO
11 18 KK 3 1 OO 1 – xy
K1 – #
L 11 18 P
JK 65 NO
KK OO 65 1 3 1
= tan –1 KK 198 OO = tan –1 c m = tan –1 ;a # < 1E
KK 195 OO 195 3 11 18
K O
L 198 P
= cot–1 3 = RHS
63 5 3
Prove that: sin –1 c
4. m = sin –1 c m + cos –1 c m [CBSE (F) 2012]
65 13 5
Sol. Let sin –1 c m = a, cos –1 c m = b
5 3
13 5
5 3
& sin a = 13 , cos b = 5
5 2 3 2
& cos a =
1–c
m , sin b = 1–c m
13 5
& cos a = 12

13
, sin b =
4
5
Now, sin (a + b) = sin a. cos b + cos a. sin b

5 3 12 4 15 48 63 63
= . + . = + = & a + b = sin –1 c m
13 5 13 5 65 65 65 65
Putting the value of a and b, we get

+ cos –1 = sin –1 c m
5 3 63
sin –1
13 5 65
5. Prove the following:
1 + x2
cos [tan –1 {sin (cot –1 x)}] = [CBSE (AI) 2010] [HOTS]
2 + x2
Sol. LHS = cos [tan–1 {sin (cot–1 x)}]
Let cot–1 x = q ⇒ x = cot q

Inverse Trigonometric Functions 55


@Cbsebookshub - Join Us on Telegram
= cos [tan –1 {sin i}] = cos =tan –1 ) 3G
1
cosec i

= cos >tan –1 H = cos >tan –1 H


1 1
2
1 + cot i 1 + x2
1
Let tan –1
=a
1 + x2
1 1
&
= tan a & = tan 2 a
1 + x2 1 + x2
1 sin 2 a 1 sin 2 a
&
2
= & +1 = +1
1+x cos 2 a 1+x 2
cos 2 a

2 + x2 1 1 + x2
&
2
= & cos a =
1+x cos 2 a 2 + x2
1 + x2
& a = cos –1 e
o
2 + x2

1 + x2 1 + x2
& cos a = cos e cos –1
o= = RHS
2 + x2 2 + x2

6. Solve for x:
r
sin –1 (1 – x) – 2 sin –1 (x) = [CBSE Panchkula 2015, CBSE 2020 (65/5/1)]
2
Sol. We have,
r
sin –1 (1 – x) = + 2 sin –1 x
2

⇒ (1 – x) = sin c + 2 sin x m
r –1

2
⇒ (1 – x) = cos (2 sin –1 x) = 1 – 2 sin 2 (sin –1 x)

⇒ (1 – x) = 1 – 2 sin 2 (sin –1 x) = 1 – 2 7sin ^sin –1 xhA


2

⇒ (1 – x) = 1 – 2 x2   ⇒ 2x2 – x = 0

  ⇒ x (2x – 1) = 0
1
  ⇒ x = 0, x =
2
1
But x = does not satisfy the given equation
2
∴ x = 0 is the required solution.

7. Prove that: 2 tan –1 c m + sec –1 d n + 2 tan –1 c m =


1 5 2 1 r
[CBSE Delhi 2014]
5 7 8 4

Sol. LHS = 2 tan –1 c 1 m + sec –1 d 5 2 n + 2 tan –1 c 1 m


5 7 8

= 2 ' tan –1 c m + tan –1 c m1 + sec –1 d n


1 1 5 2
5 8 7
]Z] 1 1 b_b
] + b 2
–1 ] d n –1
] 5 8 bb –1 5 2
= 2 tan [] ` + tan [a sec –1 x = tan –1 x 2 – 1]
]] 1 1 bb 7
]] 1 – 5 . 8 bbb
\ a

56 Xam idea Mathematics–XII

@Cbsebookshub - Join Us on Telegram


13
–1 40 + tan –1 50 13 40 1
= 2 tan – 1 = 2 tan –1 # + tan –1
39 49 40 39 49
40
JK 1 NO
KK 2 # O
3 OO 2x
= 2 tan –1 c m + tan –1 c m = tan –1 KKK O + tan –1 c m <a 2 tan –1 x = tan –1
1 1 1
3 7 2O 7 F
1 – x2
K1 – c 3 m O
KK 1 OO
L P
KKJ 2 ONO
K O 1 2 9 1
= tan –1 KK 3 OO + tan –1 c m = tan –1 c # m + tan –1 c m
KK 8 OO 7 3 8 7
K O
9
L P
JK 3 1 NO
KK + O
4 7 OO
= tan c m + tan c m = tan KK OO = tan –1 c m = tan –1 (1) = = RHS
–1 3 –1 1 –1 K 25 28 r
#
4 7 K1 – # O 3 1 28 25 4
K 4 7 O
L P
8. If y = cot –1 ( cos x ) – tan –1 ( cos x ) , then prove that sin y = tan 2 a k . [CBSE (F) 2013]
x
2
Sol. Given y = cot –1 ( cos x) – tan –1 ( cos x)
r
& y = 2 – tan –1 ( cos x) – tan –1 ( cos x)
1 – cos x
& y = 2 – 2 tan –1 ( cos x) & y = 2 – cos –1 d + n
r r

1 cos x
1 – cos x 1 – cos x
&
y = sin –1 d n & sin y =
1 + cos x 1 + cos x
x
2 sin 2 RS r VW
2 x SSNote: tan –1 x + cot –1 x = , x ! R WW
&
sin y = x & sin y = tan 2
2 SS 2 WW
2 cos 2 SS r WW
2
SS sin –1 x + cos –1 x = , x ! [ –1, 1]WW
SS 2 WW
SS 2 WW
SSand 2 tan –1 x = cos –1 1 – x WW
S , x $ 0 W
1 + x2
T X
8 4 36
9. Prove that: sin –1 c m + cos –1 c m = cot –1 [CBSE 2019 (65/4/2)]
17 5 77
8 2 15
cos a = 1– d n =
8 8
Sol. Let sin –1 c m = a & sin a = ⇒
17 17 17 17

cos –1 c m = b
4
5
& cos b = 54 ⇒ 4 2 3
sin b = 1 – d n =
5 5
15 4
⇒ cot a = , cot b =
8 3
cot a cot b – 1
Now cot (a + b) =
cot a + cot b
15 4 60
× –1 –1 60 – 24 36
= 8 3 = 24 = =
15 4 45 + 32 45 + 32 77
+
8 3 24
` a + b = cot –1 c m
36
77

⇒ sin –1 c m + cos –1 c m = cot –1 c m


8 4 36
Hence proved.
17 5 77

Inverse Trigonometric Functions 57


@Cbsebookshub - Join Us on Telegram
4 12 33
10. Prove that: cos –1 c m + cos –1 c m = cos –1 c m [CBSE (AI) 2012]
5 13 65
4 12
Sol. LHS = Let cos –1 = x, cos –1 =y [x, y ! [0, r]]
5 13
4 12
& cos x = 5 , cos y = 13
4 2 12 2

` sin x = 1 – c m , sin y = 1 – c m [a x, y ! [0, r] & sin x and sin y are + ve]
5 13

& sin x = 35 , sin y = 135

Now, cos (x + y) = cos x . cos y – sin x . sin y


4 12 3 5 33
= #
5 13 5 13
– # & cos (x + y) = 65
x + y = cos –1 c m ;a ! [–1, 1]E
33 33
& 65 65
Putting the value of x and y, we get

= cos –1 c m = RHS
4 12 33
cos –1 + cos –1
5 13 65
r 1 a r 1 a 2b
11. Prove that: tan c
+ cos –1 m + tan c – cos –1 m = [CBSE Delhi 2017; (AI) 2008; (F) 2010]
4 2 b 4 2 b a

Sol. LHS = tan c + cos –1 m + tan c – cos –1 m


r 1 a r 1 a
4 2 b 4 2 b
= tan a + x k + tan a – x k, where x = cos –1
r r 1 a
4 4 2 b
r r
tan + tan x tan – tan x 1 + tan x 1 – tan x
= 4 + 4 = +
r r 1 – tan x 1 + tan x
1 – tan . tan x 1 + tan . tan x
4 4
(1 + tan x) 2 + (1 – tan x) 2 1 + tan 2 x + 2 tan x + 1 + tan 2 x – 2 tan x 2 (1 + tan 2 x)
= = =
1 – tan 2 x 1 – tan 2 x 1 – tan 2 x
RSa cos (cos –1 x) = x if x ! 6–1, 1@WV
2 2 2 SS WW
= = = S a WW
cos a cos –1 k
cos 2x 1 a a SSHere
cos 2 c cos –1 m S b
! [–1, 1] WW
2 b b T X
2 2b
= a = a = RHS
b

12. Find the value of sin d cos –1 c m + tan –1 c m n


4 2
[CBSE 2019 (65/2/2)]
5 3
4 4 3
Sol. Let cos –1 c m = a & cos a = , sin a =
5 5 5

and tan –1 c m = b
2 2 2 3
3 & tan b =
3
, sin b = , cos b =
13 13
sin (a + b) = sin a cos b + cos a sin b

3 3 4 2 9 8 17
=
× + × = + =
5 13 5 13 5 13 5 13 5 13

58 Xam idea Mathematics–XII

@Cbsebookshub - Join Us on Telegram


x y x2 xy y2
13. If cos –1 + cos –1 = a prove that 2 – 2 cos a + 2 = sin 2 a . [CBSE (Central) 2016] [HOTS]
a b a ab b
x y
Sol. Given, cos –1 a + cos –1 = a
b
xy y2
cos –1 * 4=a
x2
&

ab
– 1– 1– [a cos –1 x + cos –1 y = cos –1 {xy – 1 – x 2 1 – y 2 }
a2 b 2

2
xy x2 y xy y2 2 2
x2 x y
&

ab
– 1– 1– = cos a & ab
– 1– – + = cos a
a2 b2 b2 a2 a2 b2
2
x2 y2 x2 y2 xy 2
x2 y x2 y2
c – cos a m = 1 –
xy
&

ab
– cos a = 1– – + & ab
– +
a2 b2 a2 b2 a2 b2 a2 b2
2 2 2 2 2 2
x y xy x y x y
&
+ cos 2 a – 2
ab
. cos a = 1 – 2 – 2 + 2 2
a2 b2 a b a b
x2 xy y2
&
–2
ab
cos a + 2 = 1 – cos 2 a
a2 b
x2 xy y2
&
–2
ab
cos a + 2 = sin 2 a. Hence proved
a2 b

– , x ! a – , k [CBSE Delhi 2012]


cos x r x
14. Prove that: tan –1 c m=
r r

1 + sin x 4 2 2 2
JK x x NO
cos x K cos 2 – sin 2 OO
Sol. Now, tan c
–1
m = tan KK
–1 K 2 2 O
1 + sin x KK 2 x 2 x x x OOO
K cos + sin + 2 cos . sin O
L 2 2 2 2P
RS x VW
SS a cos – sin ka cos + sin k WW
x x x
–1 S 2 2 2 2 W
= tan SS WW
a k
SS x x 2
WW
S cos + sin W
T 2 2 X
JK x x N
KK cos 2 sin OO
KK – 2 OO
JK x x NO x x OO R
K cos 2 – sin 2 OO K
K cos 2 cos O SSDivide each VWW
= tan –1 KKK OO = tan –1 KKK 2 OO S WW
S
KK cos x + sin x OO KK cos x sin x OOO SSterm by cos x WW
L 2 2P KK 2 + 2 OO S 2W
KK x x OO T X
K cos cos O
L 2 2 P
JK x NO JK r x NO
KK 1 – tan 2 OO K tan 4 – tan 2 OO
= tan –1 KK OO = tan –1 KKK O
KK 1 + tan x OO KK 1 + tan r tan x OOO
L 2P L 4 2P
= tan –1 :tan a – kD
r x
4 2

:a x ! a – , kD
r x r r
= –
4 2 2 2

& – r2 1 x 1 r2 & – r4 1 2x 1 r4
& r4 > – 2x > – r4 & r4 + r4 > r4 – 2x > – r4 + r4
& r2 > r4 – 2x 2 0 & a r4 – 2x k ! a 0, r2 k 1 a – r2 , r2 k

Inverse Trigonometric Functions 59


@Cbsebookshub - Join Us on Telegram
15. Solve: tan–1 (x – 1) + tan–1 x + tan–1 (x + 1) = tan–1 3x [CBSE (North) 2016]
Sol. Given: tan–1 (x – 1) + tan–1 x + tan–1 (x + 1) = tan–1 3x
⇒ tan–1 (x – 1) + tan–1 (x + 1) = tan–1 3x – tan–1 x

(x – 1) + (x + 1)
& tan –1 =

1 – ( x – 1) ( x + 1)
G = tan –1 < 3x – x F
1 + 3x 2
2x 2x 2x 2x
& tan –1
= tan –1 & =
1 – (x 2 – 1) 1 + 3x 2 2 – x 2 1 + 3x 2
Either x = 0 or 2 – x2 = 1 + 3x2 ⇒ 4x2 = 1

& x2 = 14 1
` x =! , 0
2
16. If 0 < x < 1, then solve the following for x : [CBSE 2013; Allahabad 2015]
8
tan –1 (x + 1) + tan –1 (x – 1) = tan –1 c m
31
–1 –1
Sol. Given tan (x + 1) + tan (x – 1) = tan –1 8
6a 0 < x < 1 & (x + 1) (x – 1) < 1@
31
x+1+x–1 8
& tan –1
= tan –1
1 – (x + 1) (x – 1) 31
2x 8 2x 8
& tan –1 = tan –1
31
& tan –1 = tan –1
31
1 – x2 + 1 2 – x2
2x 8
& 2
=
31
& 16 – 8x2 = 62x & 4x2 + 31x – 8 = 0
2–x
& 4x2 + 32x – x – 8 = 0 & 4x (x + 8) – 1 (x + 8) = 0
1
& (x + 8) (4x – 1) = 0 & x = – 8 or x = 4
& x = 14 [x = – 8 is not acceptable]

3
17. Solve: cos (tan –1 x) = sin c cot –1 m [CBSE Delhi 2017; (AI) 2013; (F) 2014; (South) 2016]
4

Sol. Given cos (tan –1 x) = sin c cot –1 m


3
4 RS V
SSa sin i = cos a r – i k WWW
& cos (tan –1 x) = cos c 2 – cot –1 4 m & tan –1 x = 2 – cot –1 4 SSS
r 3 r 3 2 WW
WW
SS –1 –1 r
Sand tan x + cot x = 2 WW
T X
3 3
& x = 34
r r
& 2 – cot –1 x = 2 – cot –1 4 & cot –1 x = cot –1 4
1 1 1 
18. Prove that: tan 1    tan 1    tan 1    [CBSE Delhi 2013; (AI) 2011]
2
  5 8 4
1 1 1
Sol. LHS = tan –1 c m + tan –1 c m + tan –1 c m
2 5 8
1 1
+
+ tan –1 c m ;a # = < 1E
2 5 1 1 1 1
= tan –1
1 1 8 2 5 10
1– #
2 5
JK 7 1 NO
KK + O
9 8 OO
= tan –1 c m + tan –1 c m = tan –1 KKK OO = tan –1 c m = tan –1 (1) = = RHS
7 1 65 72 r
#
9 8 7
K1 – # O 1 72 65 4
K 9 8 O
L P
60 Xam idea Mathematics–XII

@Cbsebookshub - Join Us on Telegram


4 5 63
19. Prove that sin –1 c m + tan –1 c m + cos –1 c m =
r
[CBSE 2019 (65/5/3)]
5 12 65 2
Sol. We have to prove that

sin –1 c m + tan –1 c m + cos –1 c m =


4 5 63 r
5 12 65 2

i.e. sin –1 c m + tan –1 c m = – cos –1 c m = sin –1 c m


4 5 r 63 63

5 12 2 65 65

LHS = sin –1 c m + tan –1 c m


4 5
5 12
KJK 5 ONO
K OO x
=a tan –1 x = sin –1 e oG
12
= sin –1 c m + sin –1 KKK
4
2O O
5 KK 1 + c m OO5 1 + x2
K 12 O
L P
JK 5 NO
KK O
= sin –1 c m + sin –1 KK 13 OO = sin –1 c m + sin –1 c m
4 12 O 4 5
5 KK OO 5 13
K 12 O
L P
5 2 5 4 2
= sin –1 e 1 – c 13 m + 1–c5m o
4
5 13

= sin –1 d
12 2 5 3 2n
c m + c m
4
×
5 13 13 5

= sin –1 c × × m = sin –1 c m = sin –1 c m = RHS


4 12 5 3 48 15 63
+ + Hence proved.
5 13 13 5 65 65 65
20. Solve the following for x: [CBSE Patna 2015]

tan –1 c m + tan –1 c m = , x < 1.


x–2 x+2 r

x–3 x+3 4

Sol. Given: tan –1 c m + tan –1 d


x–2 x+2
n= , x <1
r
x–3 x+3 4
JK x – 2 x + 2 NO
KK + O
x – 3 x + 3 OOO r x – 2 x + 2 x2 – 4
&
–1 K
tan K K = >a . = < 1 for| x | < 1H
x – 2 x + 2 OOO 4 x – 3 x + 3 x2 – 9
c md n
KK
K 1 –
x – 3 x+3 O
L P
( x – 2) ( x + 3) + ( x + 2) (x – 3)
tan –1 * 4=
r
&

(x – 3) (x + 3) – (x – 2) (x + 2) 4
x 2 + 3x – 2x – 6 + x 2 – 3x + 2x – 6
tan –1 * 4=
r
&

4
x2 – 9 – x2 + 4
2x 2 – 12 2x 2 – 12 2x 2 – 12
tan –1 ) 3=
r r
&

–5 4
& –5
= tan
4
& –5
=1

7
&
2x 2 – 12 = – 5 & 2x 2 – 7 = 0 & x2 =
2
7
&
x =!
2
, not acceptable as x < 1.

Hence, there is no solution.

Inverse Trigonometric Functions 61


@Cbsebookshub - Join Us on Telegram
2 5r 2
21. If (tan –1 x) + (cot –1 x) 2 = , then find x. [CBSE Delhi 2015]
8
5r 2
Sol. Here, (tan –1 x) 2 + (cot –1 x) 2 =
8
2
5r 2
(tan –1 x) 2 + c 2 – tan –1 x m =
r
&

8
2
r 5r 2
& (tan –1 x) 2 + (tan –1 x) 2 +
4
– r tan –1 x =
8
2 2
r 5 r
& 2 (tan –1 x) 2 – r tan –1 x +
4

8
=0

3r 2
& 2 (tan –1 x) 2 – r tan –1 x –
8
=0 ... (i)

Let tan–1 x = y, then (i) becomes


3r 2
2y 2 – ry –
8
=0 & 16y2 – 8ry – 3r2 = 0
& 16y2 – 12ry + 4ry – 3r2 = 0 & 4y (4y – 3r) + r (4y – 3r) = 0
r 3r
& (4y – 3r) (4y + r) = 0 & y = – 4 or y = 4
does not belongs to domain of tan –1 x i.e., a – , kE
3r
;a
r r r
& tan –1 x = – 4 4 2 2
& x = tan a – 4 k = –1
r

r
22. If tan –1 x + tan –1 y + tan –1 z = , x, y, z > 0 , then find the value of xy + yz + zx. [CBSE Chennai 2015]
2
r r
Sol. Given tan –1 x + tan –1 y + tan –1 z =
2
& tan –1 x + tan –1 y = – tan –1 z
2
+
& tan –1 d 1 – xy n = tan –1 z
x y 1
& tan –1 x + tan –1 y = cot –1 z

x+y 1
&
=
1 – xy z
& xz + yz = 1 – xy & xy + yz + zx = 1

23. Solve the equation for x: sin–1x + sin–1 (1 – x) = cos–1x [CBSE (Central) 2016]
–1 –1 –1
Sol. sin x + sin (1 – x) = cos x

& sin –1 {x 1 – (1 – x) 2 + (1 – x) 1 – x2 } = sin –1 1 – x2


[a sin –1 x + sin –1 y = sin –1 {x 1 – y 2 + y 1 – x 2 } and cos –1 x = sin –1 1 – x 2 ]

& x 1 – 1 + 2x – x2 + 1 – x2 – x 1 – x2 = 1 – x2

& x 2x – x 2 – x 1 – x 2 = 0
& x { 2x – x2 – 1 – x2 } = 0
& x = 0 or 2x – x2 –
1 – x 2 = 0 & x = 0 or 2x – x 2 = 1 – x 2


Now, 2x – x 2 = 1 – x 2
Squaring both sides, we get
2x – x 2 = 1 – x 2 & 2x – x 2 – 1 + x 2 = 0
1
&
2x – 1 = 0 & x=
2
1
Hence, x = 0 and x =
.
2

62 Xam idea Mathematics–XII

@Cbsebookshub - Join Us on Telegram


1 – y2
>cos –1 2x 2 + sin –1 H,| x |< 1, y > 0 and xy < 1. [CBSE (F) 2017]
1
24. Find the value of cot
2 1+x 1 + y2

1 – y2 1 – y2
Sol. cot >cos –1 H = cot > – sin –1 d + – cos –1 f pH
2x
n
1 2x –1 1 r r
+ sin 2
2 1 + x2 1 + y2 2 2 1+x 2 1 + y2
1 – y2
= cot > – sin –1 d – cos –1 f pH
2x
2 n+
1 r r
2 2 1+x 2 1 + y2
1
= cot [r – 2 tan –1 x – 2 tan –1 y]
2

= cot ; – (tan x + tan yE = tan (tan –1 x + tan –1 y)


r –1 –1
2
x+y
= tan f tan –1 e op =
x+y
[a xy < 1]
1 – xy 1 – xy
25. Does the following trigonometric equation have any solutions? If yes, obtain the solution(s):

tan –1 c m tan –1 c m = – tan –1 7


x+1 + x–1
[CBSE Sample Paper 2017]
x–1 x

Sol. tan –1 c m + tan –1 c m = – tan –1 7


x+1 x–1
x–1 x
RS V
SS c x + 1 m + c x – 1 m WWW
= – tan –1 7, if c mc m 1 1
S x–1 x WW x+1 x –1
⇒ tan –1 SS
...(i)
x + 1 x – 1 WWW
SS 1– c x – 1 mc x m WW
SS x–1 x
T X
x (x + 1) + (x – 1) 2
⇒ tan >
–1
H = – tan –1 7
(x – 1) x – (x + 1) (x – 1)
(x 2 + x) + (x 2 + 1 – 2x)

= tan [– tan –1 7] = tan (tan–1 (– 7)) = – 7
(x 2 – x) – (x 2 – 1)
2x 2 – x + 1


–x + 1
= –7 & 2x 2 – 8x + 8 = 0 & 2 (x 2 – 4x + 4) = 0

⇒ (x – 2)2 = 0
⇒ x=2
Let us now verify whether x = 2 satisfies the equation (i)

For x = 2, c mc m = 3× = , which is not less than 1


x+1 x –1 1 3
x–1 x 2 2
Hence, this value does not satisfy the equation (i)
i.e., there is no solution of the given trigonometric equation.

r
26. Find the real solution of tan –1 x (x + 1) + sin –1 x 2 + x + 1 = . [NCERT Exemplar]
2
r
Sol. We have, tan –1 x (x + 1) + sin –1 x 2 + x + 1 = ...(i)
2
Let sin –1 x 2 + x + 1 = i
x2 + x + 1 sin i

sin i = x 2 + x + 1 ⇒ tan i = <a tan i = F
2
–x – x cos i
2
x +x+1
 i = tan –1
= sin –1 x 2 + x + 1
–x 2 – x

Inverse Trigonometric Functions 63


@Cbsebookshub - Join Us on Telegram
On putting the value of q in equation (i), we get
x2 + x + 1 r
tan –1 x (x + 1) + tan –1 =
–x – x 2 2

We know that, tan –1 x + tan –1 y = tan –1 e o, xy < 1


x+y
1 – xy
RS V
SS x 2 + x + 1 WW
S x (x 1) + + WW
–1 S S –x 2 – x WW r
∴ tan S W=
SS x 2 + x + 1 WW 2
SS 1 – x (x + 1) . W
S –x 2 – x WW
TR VX
SS x 2 + x + 1 WWW
SS x 2 + x + W
SS –1 (x 2 + x) WW r
⇒ tan –1 SS W=
SS 2 (x 2 + x + 1) WWW 2
+
SS 1 – (x x) . W
S –1 (x 2 + x) WW
T X
x 2 + x + – (x 2 + x + 1) r 1
⇒ = tan =
2
[1 – – (x + x + 1)] x + x 2 2 0


[1 – – (x 2 + x + 1)] x 2 + x = 0

⇒ – ^x 2 + x + 1h = 1 or x 2 + x = 0

–x2 – x – 1 = 1 or
⇒ x (x + 1) = 0
2

x + x + 2 = 0 or x(x + 1) = 0
–1 ! 1 – 4 × 2 –1 ! –7
∴ x =
= , which is not real or x = 0 or x = –1
2 2
For real solution, we have x = 0, –1.
27. If sin–1 x + sin–1 y + sin–1 z = p, then prove that: x 1 – x 2 + y 1 – y 2 + z 1 – z 2 = 2xyz [HOTS]
Sol. Let sin–1 x = A ⇒ sin A = x
sin–1 y = B ⇒ sin B = y
sin–1 z = C ⇒ sin C = z
Given, sin–1 x + sin–1 y + sin–1 z = p
⇒ A + B + C = p ⇒ 2A + 2B + 2C = 2p

∴ sin 2A + sin 2B + sin 2C = 4 sin A sin B sin C
[Using trigonometric property]
⇒ 2 sin A cos A + 2 sin B cos B + 2 sin C cos C = 4 sin A sin B sin C

⇒ 2 sin A. 1 – sin 2 A + 2 sin B. 1 – sin 2 B + 2 sin C. 1 – sin 2 C = 4 sin A sin B sin C
⇒ 2x 1 – x 2 + 2y 1 – y 2 + 2z 1 – z 2 = 4xyz

⇒ x 1 – x 2 + y 1 – y 2 + z 1 – z 2 = 2xyz
Hence proved.

28. If tan–1 a + tan–1 b + tan–1 c = p, then prove that a + b + c = abc. [HOTS]


Sol. Firstly, let us assume
tan–1 a = a ⇒ tan a = a
tan–1 b = b ⇒ tan b = b
tan–1 c = g ⇒ tan g = c

64 Xam idea Mathematics–XII

@Cbsebookshub - Join Us on Telegram


Now, given that
tan–1 a + tan–1 b + tan–1 c = p ⇒ a+b+g=p
∴ a + b = p – g

Taking tangent on both sides, we have
tan (a + b) = tan (p – g)

tan a + tan b
& 1 – tan a. tan b = – tan c

& tan a + tan b = – tan c (1 – tan a. tan b)


& tan a + tan b = – tan c + tan a. tan b. tan c


& tan a + tan b + tan c = tan a. tan b. tan c



Thus, a + b + c = abc Hence proved.

29. Show that: 2tan –1 ( tan tan c – m2 = tan –1


a r b sin a . cos b
[HOTS]
2 4 2 cos a . sin b

= 2 tan –1 ( tan tan d – n2


a r b
Sol. LHS
2 4 2

. tan c – m
a r b
2 tan 2x
<a 2 tan –1 x = tan –1 F
= tan –1 2 4 2

2 a
c
2 r b
m 1 – x2
1 – tan . tan –
2 4 2
b
1 – tan
a 2
2 tan
2 b
1 + tan
;a tan (a – b) = E
–1 2 tan a – tan b
= tan
JK b NO
2 1 + tan a tan b
K 1 – tan OO
a K 2O
1 – tan 2 . KKK O
2 K bO
KK 1 + tan OOO
L 2P
KJK ON
d1 – tan nd1 + tan n OO
b b
KK 2 2 OO
KK 2 tan a . OO
KK 2 2 R VW
KK d1 + tan n
b OO
O SSS 2 tan
a
. d1 – tan 2 n
b WW
–1 K
K 2 OO –1 S
S 2 2 WW
= tan KK 2 2 O
O = tan SS 2 2W
W
KK b OO SS b WW
KK d1 + tan n – tan . d1 – tan n OO SS d1 + tan n – tan . d1 – tan n WW
b 2 a b 2 a
KK 2 2 2 O 2 2 2
KK O
OO T X
2
d1 + tan n
KK b OO
K 2 O
L P
RS VW
d 2 b
n
SS a WW
SS 2 tan . 1 – tan WW
2 2
= tan –1 SS WW
SS W
S d1 + tan na1 – tan k + 2 tan . a1 + tan k W
2 b 2 a b 2 a W
S 2 2 2 2 W
T X

Inverse Trigonometric Functions 65


@Cbsebookshub - Join Us on Telegram
JK b NO
KK 2 tan a 1 – tan 2 OO
KK 2 2 OO
KK . OO
2 a b
KK 1 + tan 2 OO
2 1 + tan 2
–1 K
<Dividing N r and D r by a1 + tan 2 kd1 + tan 2 nF
K OO a b
= tan K OO
KK a b OO 2 2
KK 1 – tan 2 2 tan OO
KK 2 2
KK + OO
2 a b OO
KK 1 + tan 2 1 + tan 2 O
L 2 P
sin a. cos b
= tan –1 e o = RHS
cos a + sin b
30. If a1, a2, a3, ...., an is an arithmetic progression with common difference d, then evaluate the
following expression. [NCERT Exemplar, HOTS]
d d d d
tan >tan –1 f + p + tan –1 f
+ p + tan –1 f
+ p + .... + tan –1 f
+ pH
1 a1 a2 1 a2 a3 1 a3 a4 1 an–1 an

Sol. We have, a1 = a, a2 = a + d, a3 = a + 2d
and d = a2 – a1 = a3 – a2 = a4 – a3 = .... = an – an – 1
d d d d
Given that, tan >tan –1 f + p + tan –1 f p + tan –1 f p + ... + tan –1 f pH
1 a1 a2 1 + a2 a3 1 + a3 a4 1 + an–1 an

a2 – a1 a – a2 a –a
= tan >tan + tan –1 3 + ... + tan –1 n n–1 H
–1
1 + a2 .a1 1 + a3 .a2 1 + an .an–1

= tan [(tan –1 a2 – tan –1 a1) + (tan –1 a3 – tan –1 a2) + ... + (tan –1 an – tan –1 an–1)]

= tan [tan –1 an – tan –1 a1]


an – a1 x–y
= tan >tan H >a tan –1 x – tan –1 y = tan –1 e oH
–1
1 + an . a1 1 + xy
an – a1
= [a tan (tan –1 x) = x]
1 + an .a1

PROFICIENCY EXERCISE
QQ Objective Type Questions: [1 mark each]
1. Choose and write the correct option in each of the following questions.
(i) Which of the following corresponds to the principal value branch of tan–1 x?

(a) d – , n (b) <– , F (c) d – , n – {0} (d) (0, p)


r r r r r r
2 2 2 2 2 2

(ii) The value of tan –1 (1) + cos –1 c – m + sin –1 c – m corresponding to principal branches is
1 1
2 2
3r r r 3r
(a) (b) (c) – (d) –
4 4 4 4

(iii) The principal value of sin –1 d


n is
– 3
2
2r r 4r 5r
(a) – (b) – (c) (d)
3 3 3 3

66 Xam idea Mathematics–XII

@Cbsebookshub - Join Us on Telegram


(iv) The value of tan (sin–1 x) is
x x 1 – x2 1 + x2
(a) (b) (c) x (d) x
2
+
1 x 1 – x2
1 1
(v) If tan –1 x = tan –1
+ tan –1 , then x is equal to
2 3
r r r
(a) (b) (c) (d) p
4 3 2
(vi) If tan–1 x + 2 cot–1 x = p, then x equals
1
(a) 0 (b) 1 (c) –1 (d)
2
(vii) Which of the following is the principal value branch of cosec–1 x?

(a) d , n (b) [0, r] – ( 2 (c) ; , E (d) ; , E – {0}


–r r r –r r –r r
2 2 2 2 2 2 2
r
(viii) If cos –1 x + cos –1 y = , then the value of sin–1x + sin–1y is
2
r 2r
(a) 0 (b) (c) p (d)
2 3
2. Fill in the blanks.

(i) The principal value of cos –1 d – n is _____________ .


3
2

(ii) The set of values of sec –1 c m is _____________ .


1
[NCERT Exemplar]
2
(iii) The value of cos (sin –1 x + cos –1 x), x # 1 is _____________ . [NCERT Exemplar]

(iv) The value of sin –1 d cos c mn is _____________ .


43r
5
QQ Very Short Answer Questions: [1 mark each]
–1
3. What is the domain of the function sin x? [CBSE (F) 2010]
–1
4. Write the principal value of cot (– 3 ) . [CBSE (AI) 2010]
–1 –1
5. If 4 cos x + sin x = p, then find the value of x.
6. Evaluate: tan (tan–1 (–4)) [NCERT Exemplar]

7. Write the principal value of cos –1 c m – 2 sin –1 c – m


1 1
[CBSE Delhi 2012]
2 2
8. Write the value of sin (cot–1 x)
QQ Short Answer Questions-I: [2 marks each]
–1
9. Find the value of sin (2 sin (0.6))

10. Show that sin –1 _ 2x 1 – x 2 i = 2 sin –1 x, –


1 1
#x#
2 2
1
11. Write the simplest form of tan –1 , x > 1.
2
x –1
–1 1
12. Prove that: 3 sin–1 x = sin–1 (3x – 4x3), xe < , F [CBSE 2018]
2 2
x
13. Write the simplest form of tan –1 f p, x < a
a – x2
2

14. Write the principal value of tan –1 3 – cot –1 ^ – 3 h . [CBSE (AI) 2013, CBSE (Delhi) 2018]

Inverse Trigonometric Functions 67


@Cbsebookshub - Join Us on Telegram
QQ Short Answer Questions-II: [3 marks each]
3 3 6
15. Prove that: cos c sin –1 + cot –1 m = [CBSE (AI) 2012]
5 2 5 13

16. Solve: tan –1 ^x + 1h + tan –1 (x – 1) = tan –1


8
[CBSE (AI) 2008]
31
17. If sin[cot–1 (x + 1)] = cos(tan–1 x), then find x. [CBSE Delhi 2015]

18. Evaluate: tan ' 2 tan –1 c m + 1


1 r
[CBSE Ajmer 2015]
5 4
19. Prove the following:
xy + 1 yz + 1
cot –1 d x – y n + cot –1 d y – z n + cot –1 a z – x k = 0 (0 < xy, yz, zx < 1) [CBSE Allahabad 2015]
zx + 1

20. Prove the following:


1 – x2 1 – x2
sin =tan –1 d n + cos –1 e oG = 1, 0 < x < 1 [CBSE Guwahati 2015]
2x 1 + x2
a cos x + b
21. Prove that: 2 tan –1 d tan n = cos –1 c m
a–b x
[CBSE Patna 2015]
a+b 2 a + b cos x

22. If tan –1 c m + tan –1 c m + ... + tan –1 d n = tan –1 i , then find the value of q.
1 1 1

1 + 1.2 1 + 2.3 1 n. (n + 1)
+
 [CBSE (F) 2015]
1 + cos x + 1 – cos x
23. Prove that: tan –1 f p=
r x 3r
– , where r < x < [CBSE Sample Paper 2016]
1 + cos x – 1 – cos x 4 2 2

24. Solve for x: tan –1 c m = tan –1 , x > 0


2–x 1 x
[CBSE (East) 2016]
2+x 2 2

25. Prove that: 2 sin –1 c m – tan –1 c m =


3 17 r
[CBSE (East) 2016]
5 31 4
6x – 8x 3
26. Prove that tan –1 e o – tan –1 d
4x 1
2n
2
= tan –1 2x;| 2x | < . [CBSE (North) 2016]
1 – 12x 1 – 4x 3

27. Solve for x: tan –1 ; E + tan –1 <


x+3
F=
x–3 r
[CBSE (AI) 2017]
x–4 x+4 4
1
28. If tan –1 x – cot –1 x = tan e o , x > 0, find the value of x and hence find the value of sec –1 b l .
2
x
3
 [CBSE 2019 (65/3/1)]

b l + sin –1 b x l = , then find the value of x.


3 4 r
29. If sin–1 x [CBSE 2019 (65/3/2)]
2

30. Find the value of x, if tan ;sec –1 b x lE = sin ^tan –1 2h , x > 0.


1
[CBSE 2019 (65/3/3)]

+ tan –1 = sin –1 c m 
1 2 1 4
31. Prove that tan –1 [CBSE 2020 (65/2/1)]
4 9 2 5

Answers
1. (i) (a) (ii) (a) (iii) (b) (iv) (b) (v) (a) (vi) (a)
(vii) (d) (viii) (b)
5r –r
2. (i) (ii) z (iii) 0 (iv)
6 10

68 Xam idea Mathematics–XII

@Cbsebookshub - Join Us on Telegram


5r 3 2r 1
3. –1 # x # 1 4. 5. 6. –4 7. 8.
6 2 3 1 + x2
r x r 1 1
9. 0.96 11. – sec –1 x 13. sin –1 a 14. – x=–
16. 17.
2 2 4 2
17 n 2 17 r
18. i=
22. 24. x = 27. ! 28. 3; 29. 5
7 n+2 3 2 6
5
30.
3

SELF-ASSESSMENT TEST
Time allowed: 1 hour Max. marks: 30
1. Choose and write the correct option in the following questions. (4 × 1 = 4)

(i) sin d – sin –1 c – mn is equal to


r 1
3 2
1 1 1
(a) (b) (c) (d) 1
2 3 4
(ii) If cos csin + cos –1 x m = 0 , then x is equal to
–1 3
5
1 3
(a) (b) (c) 0 (d) 1
5 5
r
(iii) If tan –1 a + tan –1 b = , ab < 1 then the value of a + b + ab is
4
(a) 0 (b) –1 (c) 1 (d) 2
2
(iv) Simplify the expression tan * sin –1 –1 1 – y
4
1 2x 1
+ cos
2 1 + x2 2 1 + y2
x+y x+y
(a) (b) (c) xy (d) x 1–x2 + y 1–x2
1 + xy 1 – xy
2.
Fill in the blanks. (2 × 1 = 2)
r
(i) If cot –1 x + cot –1 y = then the value of x + y – xy is _____________ .
4
(ii) The domain of cos–1 (2x – 1) is _____________ .
QQ Solve the following questions. (2 × 1 = 2)

3. Write the principal value of cos –1 c m – 2 sin –1 d – n.


1 3
2 2
4. Write the principal value of cot –1 (– 3 ) .
QQ Solve the following questions. (5 × 2 = 10)

5. Prove that: 3 cos –1 x = cos –1 (4x 3 – 3x), x ! ; , 1E


1

2
3l 2 x – x 3
6. Write the simplest form of tan –1 e o, l > 0;
–l l
3 2
#x# .
l – 3lx 3 3

7. Prove that: cot –1 e o + cot –1 e o + cot –1 d


xy + 1 yz + 1 zx + 1
n = 0 (0 < xy, yz, zx < 1)
x–y y–z z–x

Inverse Trigonometric Functions 69


@Cbsebookshub - Join Us on Telegram
8. Write the principal value of cos –1 c m – 2 sin –1 c – m
1 1

2 2
x
9. Write the simplest form of tan –1 f p, x < a
a – x2
2

QQ Solve the following questions. (4 × 3 = 12)

10. Prove that: cot c– 2 cot –1 3 m = 7


r

4

11. Prove that: 2 sin –1 c m – tan –1 c m =


3 17 r
5 31 4
6x – 8x 3
12. Prove that tan –1 e o – tan –1 d
4x 1
2n
2
= tan –1 2x;| 2x | < .
1 – 12x 1 – 4x 3
13. Prove the following:
1 – x2 1 – x2
sin =tan –1 d n + cos –1 e oG = 1, 0 < x < 1
2x 1 + x2

Answers
1. (i) (d) (ii) (b) (iii) (c) (iv) (b)
2. (i) –1 (ii) [0, 1]

6. 3 tan –1 c m 8.
5r x 2r x
3. π 4. x = 9. sin –1 a
6 l 3
zzz

70 Xam idea Mathematics–XII

@Cbsebookshub - Join Us on Telegram


Algebra of
Matrices 3
1. Matrix: A matrix is a rectangular arrangement of numbers or functions arranged into a fixed number
of rows and columns.
A matrix is written inside brackets [ ]. Each entry in a matrix is called an element of the matrix.
Order of Matrix: The dimension or order of matrix is defined by the number of rows and columns
of that matrix. By conversion the dimension or order of a matrix is given by
No. of rows × No. of columns
If a matrix have m rows and n columns then its order (dimension) is written as m × n and read as
m by n.
2. Row Matrix: A matrix having one row and any number of column is called a row matrix. In other
words, matrix of order 1 × n is always a row matrix.
e.g., [a, b, c, d]1×4 is a row matrix.

3. Column Matrix: A matrix having any number of rows but only one column is called column matrix.
In other words, a matrix of order m × 1 is always a column matrix.
RS VW
SSaWW
SSbWW
e.g., SS WW is a column matrix.
SSc WW
SSdWW
T X4 # 1
4. Square Matrix: A matrix in which the number of rows is equal to the number of columns, say n, is
called a square matrix of order n.
5. Diagonal Elements: The elements aij of a square matrix A = [aij]n × n for which i = j, i.e., the elements
a11, a22, ..., ann are called the diagonal elements and the line along which the diagonal elements lie, is
called the principal diagonal or leading diagonal.
6. Diagonal Matrix: A square matrix [aij] is said to be a diagonal matrix if aij = 0 for i ≠ j.
In other words, a square matrix is said to be a diagonal matrix, if its element not on principal
diagonal are zero.
7. Scalar Matrix: A square matrix A = [aij]n × n is called a scalar matrix, if
(i) aij = 0 ∀ i ≠ j and (ii) aii = c ∀ i, where c ≠ 0.
In other words, a square matrix is said to be scalar, if it is a diagonal matrix and entries on its
principal diagonal are equal.

Algebra of Matrices 71
@Cbsebookshub - Join Us on Telegram
8. Identity Matrix: A square matrix in which all non diagonal elements are zero and all diagonal
elements are equal to 1 is called identity matrix.
i.e., I = [aij]m×n is an identity matrix if
aij = 0 ∀ i ≠ j and aij = 1 ∀ i = j
RS V
SS1 0 0WWW
For example, SS0 1 0WW is an identity matrix.
SS W
S0 0 1WW3 # 3
T X
9. Null or Zero Matrix: A matrix whose all elements are zero is called a null matrix or a zero matrix
i.e., A = [aij]m×n is null matrix if aij = 0, ∀ i, j.
10. Upper and Lower Triangular Matrices: A square matrix A = [aij] is called
(i) an upper triangular matrix, if aij = 0 ∀ i > j, i.e., all entries below principal diagonal are zero.
(ii) a lower triangular matrix, if aij = 0 ∀ i < j, i.e., all entries above principal diagonal are zero.
11. Two matrices A = [aij]m × n and B = [bij]m × n of the same order are equal, if
aij = bij ∀ i = 1, 2, ..., m and j = 1, 2, ..., n.
12. If A = [aij]m × n and B = [bij]m × n are two matrices of the same order m × n, then their sum A + B is an
m × n matrix such that
(A + B)ij = aij + bij ∀ i = 1, 2, ..., m and j = 1, 2, 3, ..., n
Following are the properties of matrix addition:
(i) Commutativity : If A and B are two matrices of the same order, then
A + B = B + A
(ii) Associativity : If A, B and C are three matrices of the same order, then

(A + B) + C = A + (B + C)
(iii) Existence of Identity : The null matrix is the identity element for matrix addition i.e.,

A+O=A+O=A
(iv) Existence of Inverse : For every matrix A = [aij]m × n there exists a matrix – A = [– aij]m × n such

that
A + (– A) = O = (– A) + A
(v) Cancellation Laws : If A, B and C are three matrices of the same order, then
A + B = A + C ⇒ B = C and B + A = C + A ⇒ B = C
13. Let A = [aij] be an m × n matrix and k be any number called a scalar. Then, the matrix obtained by
multiplying every element of A by k is called the scalar multiple of A by k and is denoted by kA.
Thus, kA = [kaij]m × n.
Following are the properties of scalar multiplication :
If A and B are two matrices of the same order and k, l are scalars, then
(i) k(A + B) = kA + kB (ii) (k + l) A = kA + lA (iii) (kl) A = k(lA) = l(kA)
(iv) (–k) A = – (kA) = k(– A) (v) 1 A = A (vi) (–1) A = – A
Note that a scalar matrix can be obtained by multiplying an identity matrix by a scalar.
14. If A and B are two matrices of the same order, then A – B = A + (– B).
15. Multiplication of Matrices : Two matrices A and B are said to be defined for multiplication, if the
number of columns of A (pre multiplier) is equal to the number of rows of B (post-multiplier).
For example, if the order of A (pre-multiplier) is m × n and the order of B (post-multiplier) is n × p

then A and B is defined for multiplication and order of product of A and B denoted by AB is m × p.
i.e., Am × n × Bn × p = ABm × p

72 Xam idea Mathematics–XII

@Cbsebookshub - Join Us on Telegram


Definition of Product : Let A = [aij]m × n and B = [bjk]n × p be two matrices then product of A and B
denoted by AB is given as
AB = [cij]m × p
n
where, Cij = ai1b1j + ai2b2j + ... + ainbnj = / air brj [1 ≤ i ≤ m and 1 ≤ j ≤ p]
r=1
Here, A is pre-multiplier or pre-factor B is post-multiplier or post-factor.
The diagram given below may help the students to understand the process of finding the product
of two matrix:

Matrix multiplication has the following properties:


(i) Matrix multiplication is not commutative.
(ii) Matrix multiplication is associative i.e., (AB) C = A (BC) wherever both sides of the equality are
defined.
(iii) Matrix multiplication is distributive over matrix addition i.e., A (B + C) = AB + AC and
(B + C) A = BA + CA wherever both sides of the equality are defined.
(iv) If A is an m×n matrix, then Im A = A = A In
(v) If A is an m×n matrix and O is a null matrix, then Am × n On × p = Om × p and Op × m × Am × n = Op × n
i.e., the product of a matrix with a null matrix is a null matrix.

(vi) In matrix multiplication the product of two non-zero matrices may be a ‘zero-matrix’ i.e.,
AB = 0, does not imply that at least one of the A or B should be zero.
16. If A is a square matrix, then we define A' = A and An+1 = An. A.
17. If A is a square matrix and a0, a1, ..., an are constants, then
a0An + a1An – 1 + a2An – 2 + ... + an – 1A + an is called a matrix polynomial.

Algebra of Matrices 73
@Cbsebookshub - Join Us on Telegram
18. Transpose of a Matrix: Let A = [aij] be an m × n matrix. Then, the transpose of A, denoted by AT, is
an n × m matrix such that
(AT)ij = aij ∀ i = 1, 2, ..., m; j = 1, 2, ..., n
i.e., the matrix obtained by interchanging rows into columns, of a given matrix A is called the

transpose of A and is denoted by AT or A'.
Following are the properties of transpose of a matrix:
(i) (AT)T = A (ii) (A + B)T = AT + BT (iii) (kA)T = kAT
(iv) (AB)T = BT AT (v) (ABC)T = CTBTAT
19. A square matrix A = [aij] is called a symmetric matrix, if
aij = aji ∀ i, j i.e., A = AT
20. A square matrix A = [aij] is called a skew symmetric matrix, if
aij = – aji ∀ i, j i.e., AT = – A
21. All main diagonal elements of a skew-symmetric matrix are zero.
22. Every square matrix can be uniquely expressed as the sum of a symmetric and a skew-symmetric
matrix.
23. All positive integral powers of a symmetric matrix are symmetric.
24. All odd positive integral powers of a skew-symmetric matrix are skew-symmetric.

Selected NCERT Questions


RS V R V
SSx + y + zWWW SS9WW
S W
1. Find the values of x, y and z if SSS x + z WWW = SSS5WWW . [CBSE (F) 2011]
SS W S W
S y + z WW S7W
T X T X
Sol. We have, x + y + z = 9, x + z = 5 and y + z = 7
On solving these equations, we get
x = 2, y = 4, z = 3.
a – b 2a + c
2. Find the values of a, b, c and d if > H== G.
–1 5
[CBSE Delhi 2013]
2a – b 3c + d 0 13

Sol. Given: = G== G


a – b 2a + c –1 5
2a – b 3c + d 0 13
⇒ a – b = –1
…(i)
2a + c = 5 …(ii)
2a – b = 0 …(iii)
3c + d = 13 …(iv)
b
From (iii) 2a = b
& a=
2
b b
Putting in (i), we get
2
–b=–1 & 2
=1 & b=2

(i) ⇒ a = 1
(ii) ⇒ c=5–2×1=5–2=3
(iv) ⇒ d = 13 – 3 × (3) = 13 – 9 = 4
i.e., a = 1, b = 2, c = 3, d = 4

74 Xam idea Mathematics–XII

@Cbsebookshub - Join Us on Telegram


3. If x = G + y = G = = G , find the value of x and y.
2 –1 10

3 1 5

Sol. Given, x = G + y = G = = G
2 –1 10
3 1 5

= G+= G== G&= G== G


2x –y 10 2x – y 10

3x y 5 3x + y 5
On comparing corresponding elements, we get
2x – y = 10 ...(i)
and 3x + y = 5 ...(ii)
Adding (i) and (ii), we get
5x = 15 ⇒x=3
On putting x = 3 in (ii), we get
9 + y = 5 ⇒y=5–9

y = – 4.
4. Two farmers Ramkishan and Gurcharan Singh cultivates only three varieties of rice namely
Basmati, Permal and Naura. The sale (in `) of these varieties of rice by both the farmers in the
month of September and October are given by the following matrices A and B.
  September Sales (in `)
Basmati Permal Naura
10, 000
A== G
20, 000 30, 000 Ramkishan
50, 000 30, 000 10, 000 Gurchanran Singh
  October Sales (in `)
Basmati Permal Naura
5000
B== G
10, 000 6000 Ramkishan
20, 000 10, 000 10, 000 Gurchanran Singh
(i) Find the combined sales in September and October for each farmer in each variety.
(ii) Find the decrease in sales from September to October.
(iii) If both farmers receive 2% profit on gross sales, compute the profit for each farmer and for
each variety sold in October.
Sol. (i) Combined sales in September and October for each farmer in each variety is given by
Basmati Permal Naura

A+B == G
15, 000 30, 000 36, 000 Ramkishan
70, 000 40, 000 20, 000 Gurchanran Singh
(ii) Change in sales from September to October is given by
Basmati Permal Naura

A–B== G
5000 10, 000 24, 000 Ramkishan
30, 000 20, 000 0 Gurchanran Singh
2
(iii) 2% of B =# B = 0.02 # B
100
Basmati Permal Naura

= 0.02 = G
5000 10, 000 6000 Ramkishan
20, 000 10, 000 10, 000 Gurchanran Singh
Basmati Permal Naura

== G
100 200 120 Ramkishan
400 200 200 Gurchanran Singh

Algebra of Matrices 75
@Cbsebookshub - Join Us on Telegram
Thus, in October Ramkishan receives ` 100, ` 200 and ` 120 as profit in the sale of each variety
of rice, respectively, and Gurcharan Singh receives profit of ` 400, ` 200 and ` 200 in the sale
of each variety of rice, respectively.
RS V
SS1 0 2WWW
5. If A = SSS0 2 1WWW , prove that A3 – 6A2 + 7A + 2I = 0.
SS2 0 3WW
T X
Sol. A2 = A.A
RS VR V R V R V
SS1 0 2WWW SSS1 0 2WWW SSS1 + 0 + 4 0 + 0 + 0 2 + 0 + 6WWW SS5 0 8 WW
S W
= SS0 2 1WW SS0 2 1WW = SS0 + 0 + 2 0 + 4 + 0 0 + 2 + 3WW = SS2 4 5 WW
SS WW SS WW SS WW S W
S2 0 3W S2 0 3W S2 + 0 + 6 0 + 0 + 0 4 + 0 + 9W SS8 0 13WW
T XT X T X T X
A3 = A2.A
RS VR V R V RS VW
SS5 0 8 WWW SSS1 0 2WWW SSS5 + 0 + 16 0 + 0 + 0 10 + 0 + 24WWW SS21 0 34WW
= SS2 4 5 WW SS0 2 1WW = SS2 + 0 + 10 0 + 8 + 0 4 + 4 + 15 WW = SS12 8 23WW
SS WS W S W S W
S8 0 13WW SS2 0 3WW SS8 + 0 + 26 0 + 0 + 0 16 + 0 + 39WW SS34 0 55WW
T XT X T X T X
LHS = A3 – 6A2 + 7A +2I
RS V RS V RS V R V
SS21 0 34WWW SS5 0 8 WWW SS1 0 2WWW SSS1 0 0WWW
= SS12 8 23WW – 6 SS2 4 5 WW + 7 SS0 2 1WW + 2 SS0 1 0WW
SS W SS W SS W S W
S34 0 55WW S8 0 13WW S2 0 3WW SS0 0 1WW
T X T X T X T X
RS V R V R V R V
SS21 0 34WWW SSS–30 0 –48WW SS7 0 14WW SS2 0 0WW
W S W S W
= SS12 8 23WW + SS–12 –24 –30WW + SS0 14 7 WW + SS0 2 0WW
SS W S W S W S W
S34 0 55WW SS–48 0 –78WW SS14 0 21WW SS0 0 2WW
TR X T X T X T VX R V
SS21 –30 + 7 + 2 0+0+0+0 34 –48 + 14 + 0 WW SS0 0 0WW
SS WW SS WW
= SS12 – 12 + 0 + 0 8 – 24 + 14 + 2 23 – 30 + 7 + 0 WW = SS0 0 0WW = 0
SS W S W
34 – 48 + 14 + 0 0 + 0 + 0 + 0 55 – 78 + 21 + 2W S0 0 0W
T X T X
∴ A3 – 6A2 + 7A + 2I = 0 = RHS Hence proved.

6. If A = = G and I = = G , then find k so that A2 = kA – 2I.


3 –2 1 0

4 –2 0 1

A== G
3 –2
Sol. Given:
4 –2
3 – 2 3 –2 9–8 –6+4
A2 = = G#= G== G== G
1 –2


4 – 2 4 –2 12 – 8 – 8 + 4 4 –4
Now, A2 = kA – 2I

& =4 – 4G = k =4 – 2G– 2 =0 1G = G== G–= G== G


1 –2 3 –2 1 0 1 –2 3k – 2k 2 0 3k – 2 – 2k
& 4 –4 4k – 2k 0 2 4k – 2k – 2

= G== G
1 –2 3k – 2 – 2k

`
4 –4 4k – 2k – 2

Equating corresponding elements, we get


1 = 3k – 2 ⇒ 3k = 3 ⇒ k=1
7. Express the matrix as the sum of a symmetric and a skew symmetric matrix:
R V
SSS 3 3 –1WW
SS–2 –2 1 WWW
SS W
S–4 –5 2 WW
T X
76 Xam idea Mathematics–XII

@Cbsebookshub - Join Us on Telegram


R V R V
SSS3 3 –1WWW SSS3 –2 –4WWW
Sol. Let A = SSS–2 –2 1 WWW & A' = SSS3 –2 –5WWW
SS–4 –5 2 WW SS–1 1 2 WW
T X T X
1
Let P=
(A + Al )
2
JKRS3 3 –1VW RS3 –2 –4VWNO
S WW SS WWO
1 KKSS
= KKS–2 –2 1 WW + SS3 –2 –5WWOOO
2 KKSS W S W
S–4 –5 2 WW SS–1 1 2 WWOO
LT X T XP
RS V R V
SS 6 1 –5WWW SSS 3 1/2 –5/2WWW
1
= SS 1 –4 –4WW = SS 1/2 –2 –2 WW
2 SS W S W
S–5 –4 4 WW SS–5/2 –2 2 WW
T X T X
SRS 3 1/2 –5/2WVW
S W
∴ Pl = SS 1/2 –2 –2 WW = P

SS W
S–5/2 –2 2 WW
T X
1

P = (A + Al ) is a symmetric matrix.
2
1
Also, let Q=
(A – Al )
2
JKRS 3 3 –1VW RS 3 –2 –4VWNO RS V R V
S WW SS WWO SS 0 5 3WWW SSS 0 5/2 3/2WWW
1 KKSS O 1
= KKS–2 –2 1 WW – SS 3 –2 –5WWOO = SS–5 0 6WW = SS–5/2 0 3 WW
2 KKSS WW SS WWO 2 SS WW SS W
S–4 –5 2 W S–1 1 2 WO S–3 –6 0W S–3/2 –3 0 WW
LT X T XP T X T X
RS VWl RS VW RS VW
SS 0 5 / 2 3 / 2 WW SS 0 – 5 / 2 – 3 / 2 WW SS 0 5 / 2 3 /2 WW
Q' = SSS–5/2 0 3 WW = SS5/2
W SS
0 –3 WW = – SS–5/2
W SS
0 3WW = – Q
W
SS–3/2 –3 0 WW S3/2 3 0 WW S–3/2 –3 0WW
T X T X T X
1
∴ Q = (A – A') is a skew-symmetric matrix.
2
RS V R V R V
SS 3 1/2 –5/2WWW SSS 0 5/2 3/2WWW SSS 3 3 –1WWW
Now, P + Q = SSS 1/2 –2 –2 WW + SS–5/2
WW SS
0 3 WW = SS–2 –2 1 WW = A
W S W
SS–5/2 –2 2 W S–3/2 –3 0 WW SS–4 –5 2 WW
T X T X T X
Thus, A is represented as P + Q the sum of a symmetric matrix P and a skew-symmetric matrix Q.

8. For matrix A = = G , verify that


1 5

6 7
(i) (A + A' ) is a symmetric matrix.
(ii) (A – A' ) is a skew symmetric matrix.
1 5 1 6
Sol. A = > H & A' = > H
6 7 5 7
1 5 1 6 2 11
(i) A + A' = > H+> H=> H
6 7 5 7 11 14
2 11 l 2 11
Now, (A + Al ) ' = > H=> H = A + A'
11 14 11 14
∴ (A + A')' = (A + A')

⇒ (A + A') is a symmetric matrix.

Algebra of Matrices 77
@Cbsebookshub - Join Us on Telegram
1 5 1 6 1 5 –1 –6 0 –1
(ii) A – Al = > H–> H=> H+> H=> H
6 7 5 7 6 7 –5 –7 1 0
0 –1 l 0 1 0 –1
Now, (A – Al )l = > H => H = –> H = – (A – A')
1 0 –1 0 1 0
∴ (A – A')' = – (A – A')


(A – A') is a skew symmetric matrix.
9. Find the matrix ‘X’ so that

X= G == G [NCERT Exemplar, HOTS]


1 2 3 –7 –8 –9

4 5 6 2 4 6
Sol. Let required matrix X have order m × n

X= G
1 2 3

a is defined ⇒ n = 2
4 5 6 2#3

Also, X = G== G
1 2 3 –7 –8 – 9
4 5 6 2 4 6 2#3
& m=2

i.e., X is a matrix of order 2 × 2.


X== G
x y
Let
z w 2#2
x y 1 2 3
Now, X = G== G⇒= G= G== G
1 2 3 –7 –8 – 9 –7 –8 –9
4 5 6 2 4 6 z w 4 5 6 2 4 6

x + 4y 2x + 5y 3x + 6y
> H== G
–7 –8 –9


z + 4w 2z + 5w 3z + 6w 2 4 6
Equating the corresponding elements, we get
x + 4y = – 7 …(i)
2x + 5y = – 8 …(ii)
3x + 6y = – 9 …(iii)
z + 4w = 2 …(iv)
2z + 5w = 4 …(v)
3z + 6w = 6 …(vi)
On solving (i) and (ii), we get x = 1, y = – 2
On solving (iv) and (v), we get w = 0, z = 2
These values satisfy equations (iii) and (vi) also.
Hence, required matrix X is given by

X== G
1 –2

2 0
RS V
SS a WW
SS 0 – tan W
2 WW
10. If A =SS WW and I is the identity matrix of order 2, then show that:
SStan a 0 WWW
SS 2 W
T X
I + A = (I – A). = G
cos a – sin a

sin a cos a

78 Xam idea Mathematics–XII

@Cbsebookshub - Join Us on Telegram


RS a VW
SS 0 – tan WW
S 2 WW
Sol. Given: A = SS W
SStan a 0 WWW
S 2
T X
RS a VWW RSS a VWW
SS 0 – tan W S 1 – tan W
= I+A == G+S
1 0 SS 2 WW = SS 2 WW
LHS W S WW
0 1 SStan a 0 WW SStan a 1 WW
S 2 W S 2
T X T X
RHS = (I – A) = G
cos a – sin a
sin a cos a
JK RS a VWWNOO RS a VWW
KK SS 0 – tan W SS 1 tan W
O
= KKK= G–S 2 WWOO =cos a – sin aG = SS 2 WW=cos a – sin aG
1 0 SS
W S W
KK 0 1 SStan a 0 WWOO sin a cos a SS– tan a 1 WWW sin a cos a
K S 2 W O
O S 2
L T XP T X
RS VW
a
SS cos a + tan . sin a – sin a + tan . cos aWW a
= SSS 2 2 WW
SS– tan a . cos a + sin a tan a . sin a + cos a WWW
S 2 2 W
T X
RS a a VW
SS sin sin WW
SS 2 2 WW
SS cos a + +
a . sin a – sin a a . cos aWW
SS cos cos WW
= SS 2 2 WW
SS a a WW
SS – sin 2 . cos a sin . sin a
2 W
SS + sin a + cos a WWW
SS a a WW
cos cos
S 2 2 W
T X
RS V
SS cos a.cos a + sin a. sin a cosa.sin a – sin a. cos a WWW
SS 2 2 2 2 WW
SS a a WW
SS cos cos WW
= SS 2 2 W
SS a a a a WWW
SS sin a. cos – cos a. sin cos a. cos + sin a. sin
SS 2 2 2 2 WWW
SS a a WW
cos cos WW
S 2 2
T X
R V
SSS cos a – sin a WWW
SS 2 2 WW
SS a a WW SRS a WV
SS cos cos WW SS 1 – tan WWW
= SS 2 2 W=S 2W
SS a a WWW SS a W
SS sin 2 cos 2 WW SStan 2 1 WWW
SS W T X
SS cos a cos a WWW
S 2 2 W
T X
Hence, LHS = RHS
RS V
SScos x – sin x 0WWW
11. If F ^ x h = SSSsin x cos x 0WWW , then show that F(x).F(y) = F(x + y).
SS W
0 0 1W
T X
Sol. LHS = F(x) . F(y)

Algebra of Matrices 79
@Cbsebookshub - Join Us on Telegram
RS VR V
SScos x – sin x 0WWW SSScos y – sin y 0WWW
= SSsin x cos x 0WW SSsin y cos y 0WW
SS WS W
S 0 0 1WW SS 0 0 1WW
TR XT X
SScos x cos y – sin x sin y + 0 – cos x sin y – sin x cos y + 0 0 + 0 + 0WVW
S W
= SSsin x cos y + cos x sin y + 0 – sin x sin y + cos x cos y + 0 0 + 0 + 0WW
SS W
S 0+0+0 0+0+0 0 + 0 + 1WW
TR X
SScos (x + y) – sin (x + y) 0VWW
S W
= SSsin (x + y) cos (x + y) 0WW
SS W
S 0 0 1WW
T X
= F(x + y) = RHS
12. Show that the matrix BT AB is symmetric or skew-symmetric according as A is symmetric or
skew-symmetric. [HOTS]
Sol. Case I : Let A be a symmetric matrix. Then AT = A.
Now, (BTAB)T = BTAT(BT)T [By reversal law]
= B A B [ a (BT)T = B] T T

⇒ (BTAB)T = BTAB [ a AT = A]

∴ BTAB is a symmetric matrix.

Case II: Let A be a skew-symmetric matrix. Then, AT = – A

Now, (BTAB)T = BT AT(BT)T [By reversal law]
⇒ (B AB) = B A B [ a (BT)T = B]
T T T T

⇒ (BTAB)T = BT(–A)B [ a AT = – A]

⇒ (BTAB)T = – BTAB

∴ BTAB is a skew-symmetric matrix.

SRS1 1 1WVW RS n – 1 V
SS3 3 n – 1 3 n – 1WW
S W W
13. If A = SSS1 1 1WWW , then prove that A n = SSS3 n – 1 3 n – 1 3 n – 1WWW , then n ∈ N.  [HOTS]
SS W SS n – 1 W
1 1 1W 3 3 n – 1 3 n – 1W
T X T X
Sol. We shall prove the result by using principle of mathematical induction.

SRS3 n – 1 3 n – 1 3 n – 1WVW
S W
Let P (n) : A n = SSS3 n – 1 3 n – 1 3 n – 1WWW
S3 n – 1 3 n – 1 3 n – 1WW
S
T X
RS 0 0 0VW R V
SS3 3 3 WW S1 1 1WWS
S W
Now, P (1) : A1 = SSS3 0 3 0 3 0WWW = SS1 1 1WW
SS3 0 3 0 3 0WW SSS1 1 1WWW
T X T X
The result is true for n = 1.
Let the result be true for n = k. So,
RS k – 1 k – 1 k – 1VW
SS3 3 3 WW
A k = SSS3 k – 1 3 k – 1 3 k – 1WWW
SS3 k – 1 3 k – 1 3 k – 1WW
T X
Now, we prove that P (k + 1) is true.
RS V RS k – 1 k – 1 k – 1VW
SS1 1 1WWW SS3 3 3 WW
A k 1 = A. A k = SS1 1 1WW . SSS3 k – 1 3 k – 1 3 k – 1WWW
+
Now,
SS W
S1 1 1WW SS3 k – 1 3 k – 1 3 k – 1WW
T XT X
80 Xam idea Mathematics–XII

@Cbsebookshub - Join Us on Telegram


RS k – 1 V R V
SS3.3 3.3 k – 1 3.3 k – 1WWW SSS3 k 3 k 3 kWWW
= SSS3.3 k – 1 3.3 k – 1 3.3 k – 1WWW = SSS3 k 3 k 3 kWWW = A k + 1
SS3.3 k – 1 3.3 k – 1 3.3 k – 1WW SS3 k 3 k 3 kWW
T X T X
Here, it is true for n = k + 1.
Hence, by principle of mathematical induction P(n) is true for all n ∈ N.

Multiple Choice Questions [1 mark]


Choose and write the correct option in the following questions.

1. If A = = G , then A2 is equal to
0 1
[NCERT Exemplar]
1 0
0 1 1 0 0 1 1 0
(a) > H (b) > H (c) > H (d) > H
1 0 1 0 0 1 0 1

2. If A = G , then A + Al = I, if the value of a is


cos a – sin a
[NCERT]
sin a cos a
r r 3r
(a) (b) (c) r (d)
6 3 2
3. If A and B are square matrices of the same order, then (A + B)(A – B) is equal to
(a) A2 – B2 (b) A2 – BA – AB – B2
(c) A2 – B2 + BA – AB (d) A2 – BA + B2 + AB
4. Total number of possible matrices of order 3 × 3 with each entry 2 or 0 is
(a) 9 (b) 27 (c) 81 (d) 512
RS V
SS 0 –5 8 WW
W
5. The matrix SSS 5 0 12WW is a [NCERT Exemplar]
SS–8 –12 0 WWW
T X
(a) diagonal matrix (b) symmetric matrix
(c) skew symmetric matrix (d) scalar matrix
Assume X, Y, Z, W and P are matrices of order 2 × n, 3 × k, 2 × p, n × 3 and p × k respectively.
Choose the correct option in (6) and (7)
6. The restriction on n, k and p so that PY + WY will be defined are
(a) k = 3, p = n (b) k is arbitrary, p = 2 (c) p is arbitrary, k = 3 (d) k = 2, p = 3
7. If n = p, then the order of the matrix 7X – 5Z is
(a) p × 2 (b) 2 × n (c) n × 3 (d) p × n
2x + y 4x 7 7y–13
8. If > H=> H , then the value of x and y is
5x – 7 4x y x+6
(a) x = 3, y = 1 (b) x = 2, y = 3 (c) x = 2, y = 4 (d) x = 3, y = 3
[NCERT Exemplar]
9. If A is matrix of order m × n and B is a matrix such that AB' and B'A are both defined, the order
of matrix B is
(a) m × m (b) n × n (c) n × m (d) m × n
10. If A and B are matrices of same order, then (AB' – BA') is a [NCERT Exemplar]
(a) skew-symmetric matrix (b) null matrix
(c) symmetric matrix (d) unit matrix

Algebra of Matrices 81
@Cbsebookshub - Join Us on Telegram
11. If A = = G is such that A2 = I, then
a b
c –a
(a) 1 + a 2 + bc = 0 (b) 1 – a 2 + bc = 0 (c) 1– a 2 – bc = 0 (d) 1 + a 2 – bc = 0

12. On using elementary column operations C2 $ C2 – 2C1 in the following matrix equation

= G== G= G , we have:
1 –3 1 –1 3 1

2 4 0 1 2 4
1 –5 1 –1 3 –5 1 –5 1 –1 3 –5
(a) > H=> H> H (b) > H=> H> H
0 4 –2 2 2 0 0 4 0 1 0 2
1 –5 1 –3 3 1 1 –5 1 –1 3 –5
(c) > H=> H> H (d) > H=> H> H
2 0 0 1 –2 4 2 0 0 1 2 0

13. If A = = G , then An (where n∈N) equals


1 a

0 1
1 na 1 n2 a 1 na n na
(a) > H (b) > H (c) > H (d) > H
0 1 0 1 0 0 0 n

14. If A is square matrix such that A2 = I, then (A – I)3 + (A + I)3 –7A is equal to [NCERT Exemplar]
(a) A (b) I – A (c) I + A (d) 3A
15. If the matrix AB is zero, then
(a) It is not necessary that either A = O or B = O (b) A = O or B = O
(c) A = O and B = O (d) All the statements are wrong
SRS2 3 WVW
S W
16. If A = = G and B = SSS4 –2WWW , then
2 –1 3
[NCERT Exemplar]
–4 5 1 SS1 5 WW
T X
(a) only AB is defined (b) only BA is defined
(c) AB and BA both are defined (d) AB and BA both are not defined.
17. If A and B are symmetric matrices of the same order, then (AB' – BA') is a
(a) Skew symmetric matrix (b) Null matrix
(c) Symmetric matrix (d) None of these
RS VW
SS 0 5 –7 WW
18. The matrix SSS–5 0 11WWW is [NCERT Exemplar]
SS 7 –11 0 WW
T X
(a) a skew-symmetric matrix (b) a symmetric matrix
(c) a diagonal matrix (d) an upper triangular matrix.
19. If A and B are two matrices of the order 3 × m and 3 × n respectively and m = n, then the order
of matrix (5A – 2B) is
(a) m × 3 (b) 3 × 3 (c) m × n (d) 3 × n
5 x
20. If A = > H and A = Al , then [NCERT Exemplar]
y 0
(a) x = 0, y = 5 (b) x + y = 5 (c) x = y (d) none of these

Answers
1. (d) 2. (b) 3. (c) 4. (d) 5. (c) 6. (a)
7. (b) 8. (b) 9. (d) 10. (a) 11. (c) 12. (d)

82 Xam idea Mathematics–XII

@Cbsebookshub - Join Us on Telegram


13. (a) 14. (a) 15. (a) 16. (c) 17. (a) 18. (a)
19. (d) 20. (c)

Solutions of Selected Multiple Choice Questions


1. We have,
0 1 0 1 1 0
A2 = A × A = >
H> H=> H
1 0 1 0 0 1
cos a – sin a cos a sin a
2. We have, A + Al = I ⇒ > H+> H=I
sin a cos a – sin a cos a
2 cos a 0 1 0
⇒ > H=> H
0 2 cos a 0 1
1
⇒ 2 cos a = 1 ⇒ cos a =
2
r r
⇒ cos a = cos ⇒ a=
3 3
3. (A + B) (A – B) = A (A – B) + B (A – B) = A2 – AB + BA – B2 = A2 – B2 + BA – AB.
4. Total number of possible matrices of order 3 × 3 with each entry 2 or 0 is 29 i.e., 512.
2x + y 4x 7 7y – 13
8. We have, > H=> H
5x – 7 4x y x+6
⇒ 2x + y = 7 ...(1)
and 4x = x + 6 ⇒ 4x – x = 6 ⇒ 3x = 6 ⇒ x=2
putting x = 2 in eg. (1), we have
2 × 2 + y = 7 ⇒ y = 7 – 4 = 3 ⇒ y=3
∴ x = 2, y = 3
1 –3 1 –1 3 1
12. Give that, > H=> H> H
2 4 0 1 2 4
1 –5 1 –1 3 –5
> H=> H> H [Applying C2 " C2 – 2C1 ]
2 0 0 1 2 0
Since, on using elementary column operation on X = AB, we apply these operations simultaneously
on X and on the second matrix B of the product AB on RHS.
14. We have, A2 = I
` A – I) 3 + (A + I) 3 – 7A = {(A – I) + (A + I)} {(A – I) 2 + (A + I) 2 – (A – I) (A + I)}] – 7A
(
[a a 3 + b 3 = (a + b) (a 2 + b 2 – ab)]
= [(2A) {A 2 + I 2 – 2AI + A 2 + I 2 + 2AI – (A 2 – I 2)}] – 7A
= 2A [I + I 2 + I + I 2 – I + I 2] – 7A [ a A 2 = I]
= 2A [5I – I] – 7A [ a I 2 = I]
= 8AI – 7AI [a A = AI]
= AI = A
19. A3×m and B3×n are two matrices. If m = n, then A and B have same order as 3 × n each, so the order
of (5A – 2B) should be same as 3 × n.
20. We have A = Al
5 x 5 y
⇒ > H=> H ⇒ x=y
y 0 x 0

Algebra of Matrices 83
@Cbsebookshub - Join Us on Telegram
Fill in the Blanks [1 mark]
1. A matrix which is not a square matrix is called a ____________ matrix.

2. If A and B are square matrix of the same order then (AB)l = ____________ .
3. If A is a skew-symmetric matrix and n∈N such that (A n) T = mA n then l= ____________ .
SRS 0 a 1WVW
S W
4. Given a skew- symmetric matrix A = SSS–1 b 1WWW, then value of (a + b + c)2 is ____________ .
SS–1 c 0WW
T X [CBSE 2020 (65/2/1)]
1 0 –1 1
5. If A + B = > H and A–2B = > H, then A = ____________ . [CBSE 2020 (65/5/1)]
1 1 0 –1
Answers
1/3 1/3
1. rectangular 2. Bl Al 3. (–1)n 4. 0 5. > H
2/3 1/3
Solutions of Selected Fill in the Blanks
4. We have A = –Al (Since Matrix A is skew symmetric)
SRS 0 a 1WVW SRS0 –1 –1WVW
SS WW S W
⇒ SS–1 b 1WW = – SSSa b c WWW
SS–1 c 0WW SS1 1 0WW
T X T X
RS V R V
SS 0 a 1WWW SSS 0 1 1 WWW
⇒ SS–1 b 1WW = SS–a –b –cWW
SS W S W
S–1 c 0WW SS–1 –1 0 WW
T X T X
⇒ a = 1, b = –b ⇒ 2b = 0 ⇒ b = 0 and c = –1

∴ (a + b + c)2 = (1+ 0 – 1)2 = 0


5. We have,
1 0 –1 1 2 0 –1 0
2 (A + B) + (A–2B) = 2 > H+> H=> H+> H
1 1 0 –1 2 2 0 –1
1 1 1/3 1/3

3A = > H ⇒ A => H
2 1 2/3 1/3

Very Short Answer Questions [1 mark]


x–y z
1. If = G== G , find the value of x + y. [CBSE (AI) 2014]
–1 4

2x–y w 0 5
x–y z –1 4
Sol. Given > H=> H
2x – y w 0 5
Equating, we get
x – y = – 1 ...(i)
2x – y = 0 ...(ii)
z = 4, w = 5

(ii) – (i) ⇒ 2x – y – x + y = 0 + 1 ⇒ x = 1 and putting it in eq. (i), we get y= 2

x+y=2+1=3

84 Xam idea Mathematics–XII

@Cbsebookshub - Join Us on Telegram


2. If = G =A + = G , then find the matrix A. [CBSE Delhi 2013]
9 –1 4 1 2 –1

–2 1 3 0 4 9

Sol. Given = G = A += G
9 –1 4 1 2 –1
–2 1 3 0 4 9

A== G–= G== G


9 –1 4 1 2 –1 8 –3 5
&

–2 1 3 0 4 9 –2 – 3 – 6
|i – j |
Write the element a23 of a 3 × 3 matrix A = (aij) whose elements aij are given by aij =
3. .
2
[CBSE Delhi 2015]
| 2 – 3 | | –1 | 1
Sol. a23 = = =
2 2 2
RS VW
SS0 a 3 WW
4. If matrix SSS2 b –1WWW is a skew-symmetric matrix, then find the values of a, b and c.
SSc 1 0 WW
[NCERT exemplar]
T X
RS VW
SS0 a 3 WW
Sol. Let A = SSS2 b –1WWW
SSc 1 0 WW
T X
Since A is skew-symmetric matrix
∴ A' = –A
RS V RS V
SS0 2 c WW SS0 a 3 WWW
W
⇒ SSSa b 1WW = – SS2 b –1WW
W SS W
SS3 –1 0WW Sc 1 0 WW
TR X T X
SS0 2 cVWW RSS 0 –a –3 WVW
SS WW SS W
⇒ SSa b 1WW = SS–2 –b + 1WWW
SS3 –1 0WW SS–c –1 0 WW
T X T X
By equating corresponding elements, we get
a = –2, c = –3 and b = –b & b=0

a = –2, b = 0 and c = –3

5. If A = = G , then for what value of a, A is an identity matrix.


cos a – sin a
[CBSE Delhi 2010]
sin a cosa

Sol. If A is identity matrix, then A = I2

= G== G
cos a – sin a 1 0

sin a cos a 0 1
On equating corresponding elements, we get
⇒ cos a = 1, sin a = 0
⇒ a=0

6. If = G= G== G , then find the value of k.


1 2 3 1 7 11
[CBSE Delhi 2010]
3 4 2 5 k 23

Sol. Given: = G= G = = G
1 2 3 1 7 11
3 4 2 5 k 23

= G== G
(1) (3) + (2) (2) (1) (1) + (2) (5) 7 11
&

(3) (3) + (4) (2) (3) (1) + (4) (5) k 23

Algebra of Matrices 85
@Cbsebookshub - Join Us on Telegram
= G== G
7 11 7 11
&

17 23 k 23
Equating the corresponding elements, we get
k = 17
SRS0 a
V
–3 WW
SS W
7. If the matrix A = SS2 0 –1WW is skew symmetric, find the values of ‘a’ and ‘b’.
SSb W
1 0 WW [CBSE Examination Paper 2018]
RS T VW X
SS0 a –3WW
Sol. Given A = SSS2 0 –1WWW
SSb 1 0WW
T X
For skew symmetric matrix.
A’ = –A
RS V R V
SS 0 2 bWWW SSS 0 –a 3WWW
SS a 0 1WW = SS–2 0 1WW
SS W S W
S–3 –1 0WW SS–b –1 0WW
T X T X
On comparing both sides, we get
a = – 2 and – b = – 3 ⇒ b=3

8. Simplify: cos i = G + sin i = G


cos i sin i sin i – cos i
[CBSE Delhi 2012]
– sin i cos i cos i sin i

Sol. Given: cos i = G + sin i = G


cosi sini sin i – cos i
– sin i cosi cos i sin i

=> H+> H
cos 2 i sin i. cos i sin 2 i – sin i. cos i
2
– sin i. cos i cos i sin i cos i sin 2 i

=> H== G
sin 2 i + cos 2 i 0 1 0

0 sin 2 i + cos 2 i 0 1

1
9. If A is a 3 × 3 matrix, whose elements are given by aij = |– 3i + j |, then write the value
3
of a23. [CBSE (F) 2013]

1 1 1
Sol. a23 = | –3 # 2 + 3 | = | –6 + 3 | = # 3 = 1
3 3 3
R V
SSS 3 4WWW
10. If AT = SSS–1 2WWW and B = = G , then find AT – BT.
–1 2 1
[CBSE (AI) 2012]
SS WW 1 2 3
0 1
T X RS V
SS–1 1WWW
Sol. Given: B = = G ⇒ B = SS 2 2WW
–1 2 1 T
1 2 3 SS W
S 1 3WW
T X
R V R V R V
SSS 3 4WWW SSS–1 1WWW SSS 4 3WWW
Now A – B = SS–1 2WW – SS 2 2WW = SS–3 0WW
T T
SS W S W S W
S 0 1WW SS 1 3WW SS–1 –2WW
T X T X T X

86 Xam idea Mathematics–XII

@Cbsebookshub - Join Us on Telegram


RS V
SS 0 1 –2WWW
11. For what value of x, is the matrix A = SS–1 0 3WW a skew-symmetric matrix?[CBSE (AI) 2013]
SS W
S x –3 0WW
T X
Sol. A will be skew symmetric matrix if A = – A'.
R V RS V R V
SSS 0 1 –2WWW SS 0 –1 xWWW SSS 0 1 –xWWW
& SS–1 0 3WW = – SS 1 0 –3WW = SS–1 0 3WW
SS WW SS WW SS WW
S x –3 0W S–2 3 0W S 2 –3 0W
T X T X T X
Equating the corresponding elements, we get x = 2.

12. If A = = G , find AAT.


cos i – sin i
sin i cos i
Sol. We have,
T
cos i – sin i cos i – sin i
AA T
= > H> H
sin i cos i sin i cos i
cos i – sin i cos i sin i
=> H> H
sin i cos i – sin i cos i

cos 2 i + sin 2 i
=> H
cos i sin i – sin i cos i

sin i cos i – cos i sin i sin 2 i + cos 2 i
1 0
=> H = I2
0 1
13. Let A and B are matrices of order 3 × 2 and 2 × 4 respectively. Write the order of matrix (AB).
[CBSE Delhi (C) 2017]
Sol. Order of AB = [aij]3×2 [bij]2×4= [cij]3×4 i.e., order of AB is 3 × 4.

14. Construct a 2 × 2 matrix A = [aij] whole elements are given by aij = |(i)2 – j|. [CBSE 2020, 65/3/1]
Sol. We have, elements of the Matrix are given by aij = (i) 2 –j
a11 = (1) 2 –1 = 0 , a21 = (2) 2 –1 = 3


a12 = (1) 2 –2 = 1 , a22 = (2) 2 –2 = 2
0 1
∴ Matrix A = >
H
3 2

Short Answer Questions-I [2 marks]


1. Find the value of x + y from the following equation:

2= G+= G== G [CBSE (AI) 2012, Bhubneshwar 2015, AI (C) 2017]


x 5 3 –4 7 6

7 y–3 1 2 15 14

Sol. Given, 2 = G+= G== G


x 5 3 –4 7 6
7 y–3 1 2 15 14

= G+= G== G = G== G


2x 10 3 –4 7 6 2x + 3 6 7 6
&

14 2y – 6 1 2 15 14
& 15 2y – 4 15 14
Equating the corresponding elements, we get
7–3 14 + 4
2x + 3 = 7 and 2y – 4 = 14 & x=
2
and y =
2
⇒ x = 2
and y = 9 ∴ x + y = 2 + 9 = 11

Algebra of Matrices 87
@Cbsebookshub - Join Us on Telegram
2. If matrix A = = G and A2 = kA, then write the value of k. [CBSE (AI) 2013]
1 –1

–1 1

Sol. Given: A2 = kA

= G= G= k= G = G= k= G
1 –1 1 –1 1 –1 2 –2 1 –1
&

–1 1 –1 1 –1 1
& –2 2 –1 1

2= G= k= G
1 –1 1 –1
&

–1 1 –1 1
⇒ k=2

3. If A = = G and I = = G , find scalar k so that A2 + I = kA. [CBSE2 2020, (65/2/1)]


–3 2 1 0
1 –1 0 1
Sol. We have,
–3 2 –3 2 11 –8
A 2 = A× A = > H> H=> H
1 –1 1 –1 –4 3
–3 2 –3k 2k
and kA = k > H=> H
1 –1 k –k
11 –8 1 0 –3k 2k
Given, A 2 + I = kA ⇒ > H+> H=> H
–4 3 0 1 k –k
12 –8 –3k 2k

> H=> H
–4 4 k –k

k=–4
4. Show that A' A and A A' are both symmetric matrices for any matrix A. [NCERT Exemplar]
P = Al A
Sol. Let
P' = (Al A)l
= A'(A')' = A'A = P [a (AB)l = Bl Al ]
So, A'A is symmetric matrix for any matrix A.
Similarly, let Q = A A'
Q' = (AA')' = (A')'(A)'
= A(A')' = Q
So, AA' is symmetric matrix for any matrix A.
RS V
SS 0 2b –2WWW
5. Matrix A = SSS 3 1 3WW is given to be symmetric, find values of a and b. [CBSE Delhi 2016]
SS3a 3 –1WWW
T R X
SS 0 2b –2VWW
S W
Sol. We have A = SSS 3 1 3 WWW
SS3a 3 –1WW
T X SRS 0 3 3aWVW SRS 0 2b –2WVW
a A is symmetric matrix.
SS W S W
⇒ T
A = A ⇒ SS2b 1 3WW = SS 3 1 3WW
SS–2 3 –1WWW SSS3a 3 –1WWW
Equating the corresponding elements, we get T X T X
3 2
2b = 3 and 3a = –2 ⇒ b= and a=–
2 3
6. Show that all the diagonal elements of a skew symmetric matrix are zero. [CBSE Delhi 2017]
Sol. Since, A = [aij] is skew symmetric matrix

AT = – A
[aij]T = – [aij] ⇒ [aji] = [– aij]

88 Xam idea Mathematics–XII

@Cbsebookshub - Join Us on Telegram


For diagonal elements i = j

aii = – aii ⇒ 2aii = 0 ⇒ aii = 0 ∀ i
Hence, diagonal elements of skew symmetric matrix are zero.
JK2 3 NO
KK OO
7. If A = e o and B = KK4 5 OO and BA = (bij), find b21 + b32. [CBSE East 2016]
1 –2 3
–4 2 5 KK O
2 1O
L P
RS V
1 –2 3 SS2 3WWW
Sol. We have, A = > H and B = SS4 5WW
SS W
–4 2 5
S2 1WW
RS V T X
SS2 3WWW 1 –2 3
∴ S
BA = SS4 5WW W > H
SS2 1WW –4 2 5 2 ×3
TR X3 × 2
SS2 – 12 –4 + 6 6 + 15 WVW RS
SS–10 2 21WW
V
SS WW W
[bij] = SS4 – 20 –8 + 10 12 + 25WW ⇒ [bij] = SSS–16 2 37WW
SS W SS –2 –2 11WWW
2–4 –4 + 2 6 + 5 W3×3
T X T X3×3
Now, b21 = – 16; b32 = –2

b21 + b32 = –16 – 2 = –18
8. Find the value of (x – y) from the matrix equation.

H+= G== G [CBSE 2019 (65/5/3)]


x 5 –3 –4 7 6
2>

7 y–3 1 2 15 14
2x 10 –3 –4 7 6 2x – 3 10 – 4 7 6
Sol. > H+> H=> H ⇒ > H=> H
14 2y – 6 1 2 15 14 14 + 1 2y – 6 + 2 15 14
2x–3 6 7 6

> H=> H
15 2y – 4 15 14
We know that two matrices of same order are equal if the corresponding entries are equal.
i.e., 2x – 3 = 7 & 2x = 10 & x = 5
and 2y – 4 = 14 & 2y = 18 & y=9

` x – y = 5 – 9 = –4
9. If A and B are symmetric matrices, such that AB and BA are both defined, then prove that
AB – BA is a skew-symmetric matrix. [CBSE 2019 (65/4/2)]
Sol. We have AT = A and BT = B and AB and BA are both defined.
Now (AB – BA) T = (AB) T – (BA) T = BT AT – AT BT (a (AB) T = BT AT)
= BA – AB = – (AB – BA)
⇒ AB – BA is a skew-symmetric matrix.
Hence proved.

10. For the matrix A = = G , find A + AT and verify it is a symmetric matrix.


2 3

5 7
[CBSE 2019 (65/4/2)]
2 3 2 5
Sol. We have A => H, AT = > H
5 7 3 7
2 3 2 5 4 8
⇒ A + AT = > H+> H=> H
5 7 3 7 8 14
T
4 8 4 8
⇒ (A + AT) T = >
H => H = A + AT
8 14 8 14
Hence, A + AT is a symmetric matrix.

Algebra of Matrices 89
@Cbsebookshub - Join Us on Telegram
Short Answer Questions-II [3 marks]
1. For the following matrices A and B, verify that (AB)' = B'A'. [CBSE (AI) 2010]
R V
SSS 1WWW
A = SSS–4WWW , B = [–1, 2, 1]
SS WW
3
RS VW T X
SS 1WW
Sol. Given: A SSS–4WWW , B = [–1, 2, 1]
=
S 3W
S W
T X
RS VW RS V
SS 1WW SS–1 2 1WW
W
AB = SSS–4WWW [ –1 2 1 ] = SSS 4 –8 –4WWW
SS WW SS W
3 –3 6 3W
T X T X
RS VWl RS VW
SS–1 2 1 WW SS–1 4 –3WW
(AB) ' = SSS4 –8 –4WWW = SSS 2 –8 6 WWW
SS W S W
–3 6 3 W S 1 –4 3 W
T X T X
R V R V RS V
SSS 1WWWl SSS–1WWW SS–1 4 –3WW
W
Bl Al = ( –1 2 1 )l SSS–4WWW = SSS 2WWW [ 1 –4 3 ] = SSS 2 –8 6WWW
SS WW SS WW SS W
3 1 1 –4 3W
T X T X T X
∴ (AB)' = B'A'.

2. If A = = G and B = = G and (A + B)2 = A2 + B2, then find the values of a and b.


1 –1 a 1

2 –1 b –1
[CBSE (F) 2015]
1 –1 a 1
Sol. Here, A = > H and B = > H
2 –1 b –1

1 –1 a 1 1+a 0

A+B => H+> H=> H
2 –1 b –1 2 + b –2
1+a 1+a 1 + a 2 + 2a a 2 + 2a + 1
(A + B) 2 = > H.> H=> H => H
0 0 0 0


2+b –2 2 + b –2 2 + 2a + b + ab – 4 – 2b –2 +
2a – b ab – 2 4
1 –1 1 –1 a 1 a 1
Again A2 + B2 = > H.> H+> H.> H
2 –1 2 –1 b –1 b –1

a2 + b a – 1 a2 + b – 1 a – 1
H+> H=> H
–1 0
=>
0 –1 ab – b b + 1 ab – b b
Given, (A + B)2 = A2 + B2
a 2 + 2a + 1 a2 + b – 1 a – 1
> H=> H
0

2a – b + ab – 2 4 ab – b b
Equating the corresponding elements, we get
a2 + 2a + 1 = a2 + b – 1 ⇒ 2a – b = –2 ...(i)
a – 1 = 0 ⇒ a = 1 ...(ii)
2a – b + ab – 2 = ab – b ⇒ 2a – 2 = 0 ...(iii)
b = 4 ...(iv)
a = 1, b = 4 satisfy all four equations (i), (ii), (iii) and (iv)
Hence, a = 1, b = 4.

90 Xam idea Mathematics–XII

@Cbsebookshub - Join Us on Telegram


3. Let A = = G ,B = = G and C = = G . Find a matrix D such that CD – AB = O.
2 –1 5 2 2 5

3 4 7 4 3 8
[CBSE Delhi 2017]
Sol. Since A, B, C are all square matrices of order 2, and CD – AB is well defined, D must be a square
matrix of order 2.

Let D = = G . Then CD – AB = 0 gives


a b
c d

= G= G – = G= G = O
2 5 a b 2 –1 5 2

3 8 c d 3 4 7 4

= G–= G = = G or = G== G
2a + 5c 2b + 5d 3 0 0 0 2a + 5c – 3 2b + 5d 0 0

or
3a + 8c 3b + 8d 43 22 0 0 3a + 8c – 43 3b + 8d – 22 0 0
By equating the corresponding elements of matrices, we get
2a + 5c – 3 = 0 ...(i)
3a + 8c – 43 = 0 ...(ii)
2b + 5d = 0 ...(iii)
and 3b + 8d – 22 = 0 ...(iv)
Solving (i) and (ii), we get a = – 191, c = 77 and solving (iii) and (iv), we get b = – 110, d = 44.

D == G== G
a b –191 –110
Therefore
c d 77 44
4. Express the following matrix as the sum of a symmetric and skew symmetric matrix, and verify
your result. [CBSE (AI) 2010]
RS VW
SS 3 – 2 – 4WW
SS 3 – 2 – 5WW
SS WW
S– 1 1 2W
T X
SRS 3 – 2 – 4WVW
SS WW
Sol. Let A = S 3 – 2 – 5W
SS W
S– 1 1 2 WW
T X
A can be expressed as
A = (A + Al ) + (A – Al ), ... (i) ;a (A + Al ) + (A – A') = = AE
1 1 1 1 2A

2 2 2 2 2
where, A + A' and A – A' are symmetric and skew symmetric matrices respectively.
RS V R V
SS 3 – 2 – 4WWW SSS 3 – 2 – 4WWWl
Now, A + Al = SS 3 – 2 – 5WW + SS 3 – 2 – 5WW
SS W S W
S–1 1 2WW SS– 1 1 2 WW
TR XV TR XV R V
SSS 3 – 2 – 4WWW SSS 3 3 – 1WWW SSS 6 1 – 5WWW
= SS 3 – 2 – 5WW + SS– 2 – 2 1WW = SS 1 – 4 – 4WW
SS W S W S W
S– 1 1 2WW SS– 4 – 5 2WW SS– 5 – 4 4WW
T X T X T X
SRS 3 – 2 – 4WVW SRS 3 3 – 1WVW SSR0 – 5 – 3WVW
S W S W S W
A – Al = SS 3 – 2 – 5WW – SS– 2 – 2 1WW = SS5 0 – 6WW
SS WW SS WW SS W
S– 1 1 2W S– 4 – 5 2W S3 6 0WW
T X T X T X
Putting these values in (i), we get
SR 6 1 – 5WWV SR0 – 5 – 3WVW
1 SSS WW 1 SSS W
A = S 1 – 4 – 4W + S5 0 – 6WW
2 SS WW 2 SS W
S– 5 – 4 4W S3 6 0WW
T X T X

Algebra of Matrices 91
@Cbsebookshub - Join Us on Telegram
RS V R V
SS 3 1/2 – 5/2WWW SSS 0 – 5/2 – 3/2WWW
=SS 1/2 – 2 – 2 WW + SS5/2 0 – 3 WW
SS WW SS W
S–5/2 – 2 2 W S3/2 3 0 WW
T X T X
Verification: RS V
S 1 5 5 3 WW
RS VW RS VW SSS 3 + 0 2 – 2 – 2 – 2 WWW RS V
SS 3 1/2 – 5/2WW SS 0 – 5/2 – 3/2WW S 1 5 W SS 3 – 2 – 4WWW
SS 1/2 – 2 – 2 WW + SS5/2 0 – 3 WW = SSS + – 2 + 0 – 2 – 3 WWW = SS 3 – 2 – 5WW = A
SS WW SS WW S 2 2 WW SS WW
S–5/2 – 2 2 W S3/2 3 0 W SS 5 3 W S– 1 1 2 WX
T X T X SS– + – 2 + 3 2 + 0 WW T
S 2 2 W
T X
5. Find the matrix A satisfying the matrix equation = G A= G== G . [NCERT Exemplar]
2 1 –3 2 1 0

3 2 5 –3 0 1

2 1 –3 2 1 0
Sol. We have, > H A. > H => H
3 2 2×2 5 –3 2×2 0 1 2×2

a b
Let A = >
H
c d 2×2

2 1 a b –3 2 1 0

` > H> H> H=> H
3 2 c d 5 –3 0 1
2a + c 2b + d –3 2 1 0

> H> H=> H
3a + 2c 3b + 2d 5 –3 0 1
–6a – 3c + 10b + 5d 4a + 2c – 6b – 3d 1 0

> H=> H
–9a – 6c + 15b + 10d 6a + 4c – 9b – 6d 0 1

⇒ –6a – 3c + 10b + 5d = 1 ...(i)



4a + 2c – 6b – 3d = 0 ...(ii)

–9a – 6c + 15b + 10d = 0 ...(iii)

6a + 4c – 9b – 6d = 1 ...(iv)
On adding equations (i) and (iv), we get
c + b – d = 2 & d = c + b – 2 ...(v)
On adding equations (ii) and (iii), we get
–5a – 4c + 9b + 7d = 0 ...(vi)
On adding equations (vi) and (iv), we get
a + 0 + 0 + d = 1 & d = 1 – a ...(vii)
From equations (v) and (vii),
c+b – 2=1– a & a + b + c = 3 ...(viii)
⇒ a = 3 – b – c

Now, using the values of a and d in equation (iii), we get
–9 (3 – b – c) – 6c + 15b + 10 (–2 + b + c) = 0

–27 + 9b + 9c – 6c + 15b – 20 + 10b + 10c = 0

34b + 13c = 47 ...(ix)
Now, using the values of a and d in equation (ii), we get
4 (3 – b – c) + 2c – 6b – 3 (b + c – 2) = 0

12 – 4b – 4c + 2c – 6b – 3b – 3c + 6 = 0

–13b – 5c = –18 ...(x)

92 Xam idea Mathematics–XII

@Cbsebookshub - Join Us on Telegram


On multiplying equation (ix) by 5 and equation (x) by 13, then adding, we get

170b + 65c = 235


–169b – 65c = –234
b=1

–13 × 1 – 5c = –18 [From equation (x)]
⇒ –5c = –18 + 13 = –5 & c = 1


` a = 3 – 1 – 1 = 1 and d = 1 – 1 = 0
1 1

` A => H
1 0

6. If A = = G and I is the identity matrix of order 2, then show that A2 = 4A – 3I. Hence
2 –1

–1 2
find A–1. [CBSE (F) 2015]
2 –1
Sol. Here, A = > H
–1 2
2 –1 2 –1 4 + 1 –2 – 2 5 –4

A 2 = A.A = > H.> H=> H=> H ...(i)
–1 2 –1 2 +
–2 – 2 1 4 –4 5
2 –1 1 0 8 –4 3 0 5 –4
Also, 4A – 3I = 4 > H – 3> H=> H–> H=> H ...(ii)
–1 2 0 1 –4 8 0 3 –4 5

From (i) and (ii), we get A2 = 4A – 3I


Pre-multiplying both sides by A–1
A–1 . A2 = A–1 . (4A – 3I) ⇒ (A–1 . A) . A = 4 A–1 . A – 3 A–1 . I

IA = 4I – 3A–1 ⇒ A = 4I – 3A–1 [ AA–1 = I, A–1 I = A–1]
–1
⇒ 3A = 4I – A

e 4> H o = e> Ho = >


1 1 0 2 –1 1 4 0 2 –1 1 2 1 2/3 1/3

A –1 = H–> H–> H=> H
3 0 1 –1 2 3 0 4 –1 2 3 1 2 1/3 2/3

7. Let A == G Then show that A2 – 4 A + 7 I = 0. Using this result calculate A5. [NCERT Exemplar]
2 3

–1 2

A== G
2 3
Sol. Here,
–1 2

⇒ A 2 = A × A = = G= G== G
2 3 2 3 1 12

–1 2 –1 2 –4 1
Now, A 2 – 4A + 7I == G –4 = G+7 = G
1 12 2 3 1 0
–4 1 –1 2 0 1

== G–= G + = G = = G = O (zero matrix)


1 12 8 12 7 0 0 0

–4 1 –4 8 0 7 0 0
⇒ A2 – 4A + 7I = 0 ⇒ A2 = 4A – 7I

A.A2 = 4A.A – 7A.I [Pre multiplying by A]
3 2

A = 4A – 7A [AI = A]

A3 = 4(4A – 7I) – 7A [Putting the value of A2]

A3 = 16A – 28I – 7A ⇒ A3 = 9A – 28I

A.A3 = 9A.A – 28A.I [Pre multiplying by A]

Algebra of Matrices 93
@Cbsebookshub - Join Us on Telegram

A4 = 9A2 – 28A

A4 = 9(4A – 7I) – 28A [Putting the value of A2]

A4 = 8A – 63I

A.A4 = 8A2 – 63A [Pre multiplying by A]
5

A = 8(4A – 7I) – 63A = – 31A – 56I

= –31 = G –56 = G = = G
2 3 1 0 –118 –93

–1 2 0 1 31 –118
8. Prove that every square matrix can be uniquely expressed as the sum of a symmetric and skew-
symmetric matrix. [HOTS]
Sol. Let A be any square matrix. Then,
1 1
A = ( A + A T ) + (A – A T ) = P + Q, (say),
2 2
1 1
where, P = (A + A ), Q = (A – AT)
T
2 2
T
Now, PT = c (A + AT) m
1
[a (KT) T = K.AT]
2
1
= [A T + (A T ) T ] [a (A + B) T = AT + BT]
2
1
= (AT + A) [ a ( A T ) T = A]
2
1
= (A + A T ) = P
2
∴ P is symmetric matrix.
1 1 1 1
Also, QT = (A – AT) T = [AT – (AT) T] = [AT – A] = – [A – AT] = – Q
2 2 2 2
∴ Q is skew-symmetric matrix.
Thus, A = P + Q, where P is a symmetric matrix and Q is a skew-symmetric matrix.
Hence, A is expressible as the sum of a symmetric and a skew-symmetric matrix.
Uniqueness: If possible, let A = R + S, where R is symmetric and S is skew-symmetric, then,
AT = (R + S)T = RT + ST

AT = R – S [ a RT = R and ST = – S]
Now, A = R + S and AT = R – S
1 1
⇒ R = [A + AT] = P, S = (A – AT) = Q
2 2
Hence, A is uniquely expressible as the sum of a symmetric and a skew-symmetric matrix.

PROFICIENCY EXERCISE
QQ Objective Type Questions: [1 mark each]
1. Choose and write the correct option in each of the following questions.
SRSa 0 0WVW
S W
(i) If A = SSS0 a 0WWW , then An is equal to
SS0 0 aWW
RS nT XV RS n V RS n V SRSna 0 0 WWV
SSa 0 0 WWW SSa 0 0WWW SSa 0 0WWW
SS W
(a) SSS 0 a n 0 WWW (b) SSS 0 a 0WWW (c) SSS 0 a n 0WWW (d) SS 0 na 0 WWW
SS W SS W SS W SS 0 0 naWW
S 0 0 a nWW S 0 0 aWW S 0 0 aWW
T X T X T X T X

94 Xam idea Mathematics–XII

@Cbsebookshub - Join Us on Telegram


H== G , then value of y is
x–y 2 2 2
(ii) If >
x 5 3 5
(a) 1 (b) 3 (c) 2 (d) 5
(iii) The number of all possible matrices of order 3 × 3 with each entry 0 or 1 is [NCERT Exemplar]
(a) 27 (b) 18 (c) 81 (d) 512
RS VW RS VW
3
SS WW 2
SS WW
(iv) The A = 72 –3 4A , B = SSS2WWW, x = 71 2 3A and y = SSS3WWW, then AB + XY equals
SS2WW SS4WW
[CBSE 2020, (64/4/1)]
T X T X
(a) 528? (b) [24] (c) 28 (d) 24
RS VW
SS0 0 5WW
(v) The matrix A = SSS0 5 0WWW  [NCERT Exemplar]
SS5 0 0WW
T X
(a) scalar matrix (b) diagonal matrix (c) unit matrix (d) square matrix

i 0
(vi) If A = > H, n∈N, then A4n equals [NCERT Exemplar]
0 i

H (a) = G
0 0 0 i 1 0 0 0
(a) > H (b) > H (a) >
0 0 i 0 0 1 i i
(vii) For any two matrices A and B, we have

(a) AB = BA (b) AB ≠ BA (c) AB = O (d) None of these
(viii) On using elementary row operation R1 " R1 –3R2 in the following matrix equation:

4 2 1 2 2 0
>
H=> H> H, we have:
3 3 0 3 1 1

–5 –7 1 –7 2 0 –5 –7 1 2 –1 –3
(a) > H=> H> H (b) > H=> H> H
3 3 0 3 1 1 3 3 0 3 1 1
–5 –7 1 2 2 0 4 2 1 2 2 0
(c) > H=> H> H (d) > H=> H> H
3 3 1 –7 1 1 –5 –7 –3 –3 1 1
2. Fill in the blanks.

(i) If A and B are symmetric matrices of same order then AB is symmetric if and only if
_____________ .
x+y 2 7
(ii) If > H=>
7
H, then x.y = _____________ . [CBSE 2020, (65/4/2)]
9 x–y 9 4
1
(iii) If 7x 2A> H = 0 , then x = _____________ .
4
(iv) If A is symmetric matrix, then Bl AB is _____________ .

QQ Very Short Answer Questions: [1 mark each]


i
3. For a 2 × 2 matrix, A = [aij], whose elements are given by aij = , write the value of a12.
j
[CBSE Delhi 2011]
4. Write the order of the product matrix. [CBSE (F) 2011]
RS VW
SS1WW
SS2WW [ 2 3 4 ]
SS WW
S3W
T X

Algebra of Matrices 95
@Cbsebookshub - Join Us on Telegram
5. From the following matrix equation, find the value of x :

= G= = G 
x+y 4 3 4
[CBSE (F) 2010]
–5 3y –5 6

6. If = G== G , then find the value of y.


3x – 2y 5 3 5
[CBSE (F) 2009]
x –2 –3 –2
7. Write a square matrix of order 2, which is both symmetric and skew symmetric. [CBSE (F) 2010]

8. If matrix A = [1 2 3], then write AA', where A' is the transpose of matrix A. [CBSE Delhi 2009]
RS V
SS0 a –3 WW
W
9. If the matrix A = SSS2 0 –1WW is skew symmetric, find the values of ‘a’ and ‘b’. [CBSE 2018]
W
SSb 1 0 WW
T X
10. If A is a square matrix such that A2 = A, then write the value of (I + A)3 – 7A. [CBSE (AI) 2014]
11. If a matrix has 5 elements, write all possible orders it can have. [CBSE (AI) 2011]
|i – j |
12. Write the element a23 of a 3 × 3 matrix A = (aij) whose elements aij are given by aij = .
2
[CBSE Delhi 2015]

13. In the matrix equation e o e o=e o . Use elementary operation R2 " R2 + R1 and
2 3 1 0 8 –3

1 4 2 –1 9 –4
write the equation thus obtained. [CBSE Delhi (C) 2017]
14. Write the number of all possible matrices of order 2 × 2 with each entry 1, 2 or 3.
[CBSE Central 2016]
QQ Short Answer Questions-I: [2 marks each]

15. Find a matrix A such that 2A – 3B + 5C = O, where B = = G and C = = G


–2 2 0 2 0 –2

3 1 4 7 1 6
 [CBSE 2019 (65/1/1)]

16. If A = = G and kA = = G , then find the value of k, a and b.


0 2 0 3a
[CBSE 2019 (65/3/1)]
3 –4 2b 24

17. Express A = = G as a sum of a symmetric and a skew-symmetric matrix.[CBSE 2019 (65/3/1)]


4 –3

2 –1
1 0
18. Solve the following matrix equation for x: [x 1] > H = O . [CBSE Delhi 2014]
–2 0
3 4 1 y 7 0
19. If 2 > H+> H=> H , find (x – y). [CBSE Delhi 2014]
5 x 0 1 10 5
2 4 –2 5
20. If A = > H and B = > H , then find (3A – B). [CBSE Guwahati 2015]
3 2 3 4

21. If = G.= G== G , then write the value of x.


2 3 1 –3 –4 6
[CBSE Delhi 2012]
5 7 –2 4 –9 x

22. If matrix A = = G and A2 = lA, then write the value of l.


3 –3
[CBSE (AI) 2013]
–3 3
2 –2
23. If matrix A = > H and A 2 = pA , then write the value of p. [CBSE (AI) 2013]
–2 2

96 Xam idea Mathematics–XII

@Cbsebookshub - Join Us on Telegram


QQ Short Answer Questions-II: [3 marks each]

24. Given matrix A = = G , find f(A), if f(x) = 2x2 – 3x + 5.


1 2

3 4
SRS 2 –1WVW SRS–1 –8 –10WVW
SS WW S W
25. Find the matrix X such that S 0 1 W X = SS 3 4 0 WW .
SS WW SS W
S–2 4 W S10 20 10 WW
RS V T X T X
SS2 3 1WWW
26. Express the matrix SS1 –1 2WW as the sum of a symmetric and a skew symmetric matrix.
SS W
S4 1 2WW
T X
RS V
SS2 0 1WWW
27. If � = SSS2 1 3WWW , then find the value of A2 – 3A + 2I. [CBSE (AI) 2010]
SS1 –1 0WW
T X
28. Show that the elements along the main diagonal of a skew symmetric matrix are all zero.
 [CBSE Sample Paper 2017]
RS VW RS VW RS VW
SS 0 6 7WW SS0 1 1WW SS 2 WW
29. If A S– 6 0 8W, B S1 0 2W, C SS– 2WW , then calculate AC, BC and (A + B) C. Also verify that
= S W = S W =
SS W SS W SS WW
S 7 – 8 0WW S1 2 0WW S3W
T X T X T X
(A + B) C = AC + BC. [CBSE Ajmer 2015]
30. A manufacturer produces three products x, y, z which he sells in two markets. Annual sales are
indicated in the table:
Market Products
x y z
I 10,000 2,000 18,000
II 6,000 20,000 8,000
If unit sale price of x, y and z are `2.50, `1.50 and `1.00 respectively, then find the total revenue in
each market, using matrices.

Answers
1. (i) (a) (ii) (a) (iii) (d) (iv) (a) (v) (d) (vi) (c)
(vii) (d) (viii) (a)
2. (i) AB = BA (ii) – 3 (iii) – 8 (iv) Symmetric Matrix
0 0
7. > H
1
3. a12 = 2 4. 3 × 3 5. x = 1 6. –6 8. [14]
0 0
1
9. a = –2, b = 3 10. I 11. 1×5 and 5 ×1 12.
2
2 3 1 0 8 –3
15. = G 16. k = – 6, a = – 4, b = – 9
–8 3 5
13. > H> H=> H 14. 81
3 7 3 –1 17 –7 –13 –1 –9
RS V R V
SS 4 – 1 WWW SSS 0 – 5 WWW
20. = G
S 2 WW + SS 2 WW 8 7
17. SS 1 WW SS 5 WW 18. x = 2 19. 10 21. x = 13
SS– – 1 W S 0 WW 6 2
SS 2 WW SS 2 W
T X T X
24. = G 25. X = = G
16 14 1 –2 –5
22. l = 6 23. p = 4
21 37 3 4 0

Algebra of Matrices 97
@Cbsebookshub - Join Us on Telegram
RS V R V
SS 2 2 5 WWW SSS 0 1 – 3 WWW
SS 2 WW SS 2 WW RS V RS VW R V RS VW
SS 3 WW SS 1 WW SS 1 –1 –1WWW SS 9 WW SSS 1 WWW SS10WW
26. SS 2 –1 WW + SS–1 0 W 27. S S 3 –3 – 4W
WW 29. AC SS WW, BC SS WW, (A B) C SSS20WWW
= S 12 W = S 8 W + =
SS 2W S 2 WW SS
SS 5 3 WW SS 3 1 WW S–3 2 0 WW SS30WW SS–2WW SS28WW
SS 2 2 2 WWW SSS 2 – 2 0 WWW T X T X T X T X
T X T X
30. I : `46,000 ; II : `53,000

SELF-ASSESSMENT TEST
Time allowed: 1 hour Max. marks: 30
1. Choose and write the correct option in the following questions. (4 × 1 = 4)

(i) If = G== G then


1 2 1 4x
3 4 6y 4
1 1 1 1
(a) x = 2, y = 2 (b) x = , y = (c) x = , y = 2 (d) x = 2, y =
2 2 2 2
i 0 0 i
(ii) If A = > H, B = > H , where i = –1 , then the correct relation is
0 –i i 0
(a) A +B = 0 (b) A2 = B2 (c) A – B = 0 (d) A2 + B2 = 0
(iii) A square matrix A = [aij] in which aij = 0 for i ≠ j and aij = k (Constant) for i = j is called a
(a) Unit matrix (b) Scalar matrix (c) Null matrix (d) Diagonal matrix
3 1
(iv) For the matrix A = > H , find x and y so that A2 + xI = yA.
7 5
(a) (8, 8) (b) (–8, 0) (c) (–8, –8) (d) None of these
2. Fill in the blanks. (2 × 1 = 2)

(i) If = G== G then x = _____________ and y = _____________ .


4 3 4 y
x 5 1 5
(ii) If A and B are square matrices of the same order, then [k (A – B)]′ = _____________ , where k
is any scalar.
QQ Solve the following questions. (2 × 1 = 2)

3. If A = = G , then write An.


cos i sin i

– sin i cos i
RS Vk
SS 2r 2r WW
SS cos – sin W
7 WW
WW = = G , then find the least positive integral value of k.
7 1 0
4. If SS
SSsin 2 r 2 r WW 0 1
SS cos
7 7 WW
T X
QQ Solve the following questions. (5 × 2 = 10)
RS VR V
SS–1 0 –1WWW SSS 1WWW
5. If [2 1 3] SSS–1 1 0WWW SSS 0WWW = A, then find the value of A.
SS 0 1 1WW SS–1WW
T XT X
6. Solve for x, 71 xA = G > H = 60@ .
2 –1 1

1 2 3

98 Xam idea Mathematics–XII

@Cbsebookshub - Join Us on Telegram


7. Find the value of x and y which makes the following pair of matrices equal:
3x + 7 0 y–2
> H=>
5
H
y + 1 2 – 3x 8 4

3 4 1 y 7 0
8. If 2 > H+> H=> H , find (x – y).
5 x 0 1 10 5
2 –2
9. If matrix A = > H and A 2 = pA , then write the value of p.
–2 2

QQ Solve the following questions. (4 × 3 = 12)


RS VW SR1WV
SS 1 3 2WW SS WW
10. Find the value of x, if [1 x 1] SSS 2 5 1WWW SSS2WWW = [0] .
SS15 3 2WW SSxWW
T XT X
11. Show that A = = G satisfies the equation x2 – 6x + 17 = 0. Hence, find A–1.
2 –3

3 4
RS V
SS3 2 5WWW
12. Let A = SSS4 1 3WWW , express A as a sum of two matrices such that one is symmetric and other is
SS0 6 7WW
T X
skew symmetric.
13. (i) Prove that the sum of two skew-symmetric matrices is a skew-symmetric matrix.
(ii) Express the following matrix as the sum of a symmetric and a skew-symmetric matrix.
RS V
SS 1 3 5WWW
SS– 6 8 3WW
SS W
S– 4 6 5WW
T X

Answers
1. (i) (b) (ii) (b) (iii) (b) (iv) (a)
2. (i) 1, 3 (ii) k (A′ – B′)

3. = G
cos ni sin ni 1
4. k = 7 5. [– 4] 6. 7. not possible
– sin ni cos ni 7
8. 10 9. p = 4
RS V R V
SS 3 5 WW SS 5 WW
SS 3 W S 0 –1 W
2 WW SS 2 WW
SS 9 WW SS 3W
= G
1 4 3
10. –14, –2 11. 12. SS 3 1 W+S 1 0 – WWW
17 –3 2 SS 2 WW SS 2W
SS 5 9 WW SS 5 3 WW
SS 2 7 WW SS– 0 WW
2 W S 2 2 W
T X T X
RS V RS V
SS 2 –3 1 WWW S0 9 9 WW
13. (ii)
1 SS–3 16 9 WW + SSS–9 0
1 W
–3WW
2 SS W 2 SS W
S–1 9 10WW S–9 3 0 WW
T X T X zzz

Algebra of Matrices 99
@Cbsebookshub - Join Us on Telegram
4 Determinants

1. Determinant: Every square matrix can be associated to an expression or a number which is known
as its determinant.
a a
Determinant of square matrix A = >a11 a12H is given by
21 22

a a
| A | = a11 a12 = a11 a22 – a12 a21
21 22
RS V
SSa1 b1 c1WWW
and determinant of a matrix A = SSa2 b2 c2WW is given by
SS W
Sa3 b3 c3WW
T X
a1 b1 c1 b c a c a b
| A | = a2 b2 c2 = a1 b2 c2 – b1 a2 c2 + c1 a2 b2
3 3 3 3 3 3
a3 b3 c3
This is known as the expansion of |A| along first row.
In fact, |A| can be expanded along any of its rows or columns.
2. Singular and Non-singular Matrix: A square matrix is a singular matrix if its determinant is zero.
Otherwise, it is a non-singular matrix.
3. (i) Minor: Let A = [aij] be a square matrix of order n. Then the minor Mij of aij in A is the determinant
of the sub-matrix of order (n – 1) obtained by leaving ith row and jth column of A.
RS V
SS 1 2 3 WWW
For example, if A = SS–3 2 –1WW , then
SS W
S 2 –4 3 WW
T X
2 –1 –3 –1
M11 = = 2, M12 = = –7 and so on.
–4 3 2 3
(ii) Cofactor: The cofactor Cij of aij in A = [aij]n×n is equal to (–1)i+j times Mij.
1 2 3
=
For example, if A –3 2 –1 , then

2 –4 3
C11 = (–1)1+1 M11 = M11 = 2 and C12 = (–1)1+2 M12 = – M12 = 7 and so on

100 Xam idea Mathematics–XII

@Cbsebookshub - Join Us on Telegram


4. Some Important Properties of Determinants:
(i) Let A = [aij] be a square matrix of order n, then the sum of the product of elements of any row
(column) with their cofactors is always equal to |A| or, det (A), i.e.,
n n
/ aij Cij = | A | and / aij Cij = | A |
j=1 i=1

(ii) Let A = [aij] be a square matrix of order n, then the sum of the product of elements of any row
(column) with cofactors of the corresponding elements of some other row (column) is zero, i.e.,
n n
/ aij Ckj = 0 and / aij Cik = 0, i ! k or j ! k
j=1 i=1
(iii) Let A = [aij] be a square matrix of order n, then |A| = |AT|.
In other words, we say that the value of a determinant remains unchanged, if its rows and
columns are interchanged.
(iv) Let A = [aij] be a square matrix of order n(≥ 2) and B be a matrix obtained from A by interchanging
any two rows (columns) of A, then |B| = – |A|.
(v) If any two rows (columns) of a square matrix A = [aij] of order n (≥ 2) are identical, then value
of its determinant is zero i.e., |A| = 0.
(vi) Let A = [aij] be a square matrix of order n, and let B be the matrix obtained from A by multiplying
each element of a row (column) of A by a scalar k, then | B | = k | A |.
(vii) Let A be a square matrix such that each element of a row (column) of A is expressed as the
sum of two or more terms. Then the determinant of A can be expressed as the sum of the
determinants of two or more matrices of the same order.
(viii) Let A be a square matrix and B be a matrix obtained from A by adding to a row (column) of A
a scalar multiple of another row (column) of A, then |B| = |A|.
(ix) Let A be a square matrix of order n (≥ 2) such that each element in a row (column) of A is zero,
then |A| = 0.
(x) If A = [aij] is a diagonal matrix of order n (≥ 2), then
|A| = a11 . a22 . a33 .... ann i.e., |A| is the product of its diagonal elements.
(xi) If A and B are square matrices of the same order, then
|AB| = |A||B|
(xii) If A = [aij] is a triangular matrix of order n, then
|A| = a11 . a22 . a33 .... ann i.e., |A| is the product of its diagonal elements.
(xiii) If A = [aij] is a square matrix of order n, then |kA| = kn|A|, because k is common from each row
(or column) of kA.
(xiv) We can take out any common factor from any one row or any one column of a given determinant.
5. Area of a triangle with vertices (x1, y1), (x2, y2) and (x3, y3) is given by
x y 1
1 1 1
D = Numerical value of x y 1
2 2 2
x3 y3 1
Note: Since area is positive quantity therefore we take absolute value of D.
6. (i) If A is a skew-symmetric matrix of odd order, then|A| = 0.
(ii) The determinant of a skew-symmetric matrix of even order is a perfect square.
7. Some Important Facts:
(i) Only square matrices have determinants.

Determinants 101
@Cbsebookshub - Join Us on Telegram
(ii) We cannot equate the corresponding elements of equal determinants like matrices
x y l m x = l, y = m
i.e.,

z w
=
n p
&
Y
z = n, w = p
(iii) In the case of matrices. We take out any common factor from each elements of matrix, while in
the case of determinants we can take out common factor from any one row or any one column
of the determinant.
(iv) If the value of determinant ‘D’ becomes zero by substituting x = a then (x – a) is factor of the
determinant ‘D’.
(v) If area is given then both positive and negative values of the determinant is taken for calculation.
(vi) To prove three points collinear, we show area of the triangle formed by these three points is
zero.
Selected NCERT Questions

1. If A = = G , then show that |2 A| = 4|A|.


1 2

4 2
Sol. We have,

A = = G ⇒ 2A = = G
1 2 2 4
4 2 8 4
2 4
∴ LHS = 2A =
= 8 – 32 = – 24
8 4
1 2
RHS = 4 A = 4 = 4 (2 – 8) = 4 # (–6) = –24
4 2
∴ LHS = RHS

Hence Proved
By using properties of determinant in problems 2 to 5 prove that:
–a 2 ab ac
2. ba –b 2 bc = 4a 2 b 2 c 2 . [CBSE (Delhi) 2011]
ca cb –c 2
–a 2 ab ac
2
Sol. LHS = ∆ = ba –b bc
ca cb –c 2
Taking a, b and c common from R1, R2 and R3 respectively, we get
–a b c
∆ = abc a –b c
a b –c
Taking a, b and c common from C1, C2 and C3 respectively, we get
–1 1 1
2 2 2
∆ = a b c 1 –1 1
1 1 –1
Operating R2 → R2 + R1 and R3 → R3 + R1, we get
–1 1 1
2 2 2
∆ = a b c 0 0 2
0 2 0

102 Xam idea Mathematics–XII

@Cbsebookshub - Join Us on Telegram


Interchanging C2 and C3, we get
–1 1 1
∆ = (–1) a2b2c2 0 2 0
0 0 2
Since the determinant of a triangular matrix is product of its diagonal elements.
= (–1) a2b2c2 (–1) × (2) × (2) = 4 a2b2c2 = RHS
1 1 1
3. a b c = (a – b) (b – c) (c – a) (a + b + c) . [CBSE (Delhi) 2012]
3 3
a b c3
1 1 1
= a
Sol. LHS b c
3 3
a b c3
Operating C2 → C2 – C1 and C3 → C3 – C1, we get
1 0 0
= a b–a c–a
a3 b3 – a3 c3 – a3
Taking (b – a) and (c – a) common from C2 and C3, we get
1 0 0
= (b – a) (c – a) a 1 1
3 2 2
a +
b ba a + c ca + a 2
2
+
Operating C3 → C3 – C2, we get
1 0 0
= (b – a) (c – a) a 1 0
a3 b + ba + a 2
2
c 2 + ca – b 2 – ba
Expanding along R1, we get
= (b – a) (c – a) (c2 + ca – b2 – ba) = (b – a) (c – a) [c2 – b2 + a (c – b)]
= (b – a) (c – a) [(c – b) (c + b) + a (c – b)] = (b – a) (c – a) (c – b) [c + b + a]
= (a – b) (b – c) (c – a) (a + b + c) = RHS.
x x2 yz
2
4. y y zx = (x – y) (y – z) (z – x) (xy + yz + zx) .
2
z z xy
x x2 yz
2
Sol. LHS = y y zx
2
z z xy
Operating R1 → R1 – R3 and R2 → R2 – R3 , we get

x–z x2 – z2 yz – xy (x – z ) (x – z ) (x + z ) – y ( x – z)
2 2
= y–z y –z zx – xy = (y – z ) (y – z ) (y + z ) –x (y – z)
z z 2
xy z z2 xy
Taking (x – z) and (y – z) common from R1 and R2 , we get
1 x+z – y
= (x – z) (y – z) 1 y + z – x
z z2 xy

Determinants 103
@Cbsebookshub - Join Us on Telegram
Operating R2 → R2 – R1, and R3 → R3 – zR1 , we get

1 x+z –y
= (x – z) (y – z) 0 y – x y – x
0 – xz xy + yz
Expanding
along R1 , we get
= (x – z) (y – z) [(y – x) (xy + yz) + xz (y – x)]
= (x – y) (y – z) (z – x) (xy + yz + zx) = RHS
a–b–c 2a 2a
5. 2b b–c–a 2b = (a + b + c) 3
2c 2c c–a–b
a–b–c 2a 2a
Sol. LHS = 2b b–c–a 2b
2c 2c c–a–b
Operating R1 → R1 + R2 + R3 , we get
a+b+c a+b+c a+b+c
= 2b b–c–a 2b
2c 2c c–a–b
Taking (a + b + c) common from first row, we get
1 1 1
= (a + b + c) 2b b–c–a 2b
2c 2c c–a–b
Operating C2 → C2 – C1 and C3 → C3 – C1 , we get
1 0 0
= (a + b + c) b
2 –c–a–b 0
2c 0 –a–b–c
Since determinant of a triangular matrix is equal to product of its diagonal elements

= (a + b + c) (a + b + c) (a + b + c) = (a + b + c)3 = RHS
6. By using properties of determinant, show that:
1 + a2 – b2 2ab –2b
= ^1 + a 2 + b 2 h
3
2ab 1–a 2 + b 2 2a
2b –2a 1 – a2 – b2
[CBSE Delhi 2008, 2009, (F) 2013; Guwahati 2015]
2 2
1+a –b 2ab – 2b
Sol. LHS = 2ab 1 – a2 + b2 2a
2b –2 a 1 – a2 – b2
Applying C1 → C1 – bC3, and C2 → C2 + aC3, we get

( 1 + a 2 + b 2) 0 – 2b
2 2
= 0 +
(1 a b )+ 2a
b (1 + a 2 + b 2) –a (1 + a 2 + b 2) 1 – a 2 – b 2

104 Xam idea Mathematics–XII

@Cbsebookshub - Join Us on Telegram


Taking out (1 + a2 + b2) from C1 and C2 column, we get
1 0 – 2b
= (1 + a 2 + b 2) 2 0 1 2a
b – a 1 – a2 – b2
Applying R3 → R3 – bR1, we get
1 0 – 2b
= (1 + a 2 + b 2) 2 0 1 2a
0 –a 1 – a 2 + b 2
Expanding along first column, we get
= (1 + a2 + b2)2 [1 – a2 + b2 + 2a2]
= (1 + a2 + b2)2 (1 + a2 + b2 ) = (1 + a2 + b2)3 = RHS
7. By using properties of determinant, show that:
a 2 + 1 ab ac
ab b + 1 bc = 1 + a 2 + b 2 + c 2 [CBSE Delhi 2014; (F) 2009, 2013]
2

ca cb c2 + 1
a 2 + 1 ab ac
Sol. LHS = ab b 2 + 1 bc
ca cb c 2 + 1
a 2 + 1 ab ac
abc
= ab b 2 + 1 bc [Multiplying and dividing by abc]
abc
ca cb c 2 + 1
Multiplying a in C1, b in C2 and c in C3, we get
a 3 + a ab 2 ac 2
1
= a 2 b b 3 + b bc 2
abc 2
a c b2 c c3 + c
Taking a, b and c common from R1, R2 and R3 respectively, we get
1 + a2 b2 c2
1 2 2
= ×abc a 1+b c2
abc
a2 b2 c2 + 1
Applying C1 → C1 + C2 + C3 , we get

1 + a2 + b2 + c2 b2 c2
2 2 2 2
= 1+a +b +c 1+b c2
2 2 2 2 2
1+a +b +c b c +1

Taking (1 + a 2 + b 2 + c 2) common from C1, we get


1 b2 c2
= (1 + a 2 + b 2 + c 2) 1 1 + b 2 c 2
1 b2 c2 + 1

Applying R1 → R1 – R3 and R2 → R2 – R3, we get


0 0 –1
= (1 + a 2 + b 2 + c 2) 0 1 –1
1 b2 c2 + 1

Determinants 105
@Cbsebookshub - Join Us on Telegram
Expanding along R1, we get

(1 + a 2 + b 2 + c 2) [–1 (–1)] = (1 + a 2 + b 2 + c 2) = RHS

a2 bc ac + c 2
8. Prove that: a 2 + ab b2 ac = 4a 2 b 2 c 2 [CBSE (F) 2014; Allahabad 2015, 2019 (65/5/3)]
ab b 2 + bc c2
a2 bc ac + c 2
Sol. LHS = a 2 + ab b2 ac
2
ab b + bc c2
a c a+c
= abc a + b b a [Taking out a, b, c from C1, C2 and C3]
b b+c c
0 c a+c
= abc 2b b a [Applying C1 → C1 + C2 – C3]
2b b + c c
0 c a+c
= 2ab 2 c 1 b a [Taking out 2b from C1]
1 b+c c
0 c a+c
2
= 2ab c 0 – c a – c [Applying R2 → R2 – R3]
1 b+c c
c a+c
= 2ab 2 c.1. = 2ab 2 c (ac – c 2 + ac + c 2) [Expanding by I column]
–c a – c
= 2ab2c(2ac) = 4a2b2c2 = RHS

x x2 1 + px 3
9. Prove: y y2 1 + py 3 = ^1 + pxyz h^ x – y h^ y – z h ^z – x h [CBSE (AI) 2010]
z z2 1 + pz 3

x x 2 1 + px 3
Sol. LHS D = y y 2 1 + py 3
z z 2 1 + pz 3

x x2 1 x x 2 px 3 x x2 1 1 x px 2
= y y 2 1 + y y 2 py 3 = y y 2 1 + xyz 1 y py 2 [ Taking common x, y, z from
z z2 1 z z 2 pz 3 z z2 1 1 z pz 2 R1, R2, R3 respectively]

x x2 1 1 x x2
= y y 2 1 + (xyz) p 1 y y 2 [Taking p common from C3 ]
z z2 1 1 z z2
By changing (transforming) column to column in first determinant, we get

1 x x2 1 x x2 1 x x2
= 1 y y 2 + pxyz 1 y y 2 = (1 + pxyz) 1 y y 2
1 z z2 1 z z2 1 z z2

106 Xam idea Mathematics–XII

@Cbsebookshub - Join Us on Telegram


Applying R1 → R1 – R3 and R2 → R2 – R3 , we get

0 x – z x2 – z2
= (1 + pxyz) 0 y – z y 2 – z 2
1 z z2
Taking out (x – z), (y – z) from R1 and R2 respectively, we get
0 1 x+z
= (1 + pxyz) (x – z) (y – z) 0 1 y + z
1 z z2
Expanding along C1, we get
= (1 + pxyz) (x – z) (y – z) [y + z – x – z]
= (1 + pxyz) (x – y) (y – z) (z – x) = RHS.

b+c c+a a+b


10. If a, b and c are real numbers and D = c + a a + b b + c = 0 then show that either a + b + c = 0
or a = b = c. a+b b+c c+a [CBSE (AI) 2007C; (F) 2009]
b+c c+a a+b
Sol. Given D = c + a a + b b + c
a+b b+c c+a

2 (a + b + c) 2 (a + b + c) 2 (a + b + c)
= c+a a+b b+c [Applying R1 → R1 + R2 + R3]
a+b b+c c+a
1 1 1
= 2 (a + b + c) c + a a + b b + c [Taking common 2(a + b + c) from R1]
a+b b+c c+a
Applying C1 → C1 – C3 and C2 → C2 – C3 , we get
0 0 1
= + +
2 (a b c) a – b a – c b +c
b–c b–a c+a
Expanding along R1, we get
= 2 (a + b + c) [(a – b) (b – a) – (b – c) (a – c)]
= 2 (a + b + c) [ab – a 2 – b 2 + ab – {ab – bc – ac + c 2}]
= 2 (a + b + c) [ab – a 2 – b 2 + ab – ab + bc + ac – c 2]
= 2 (a + b + c) [–a 2 –b 2 –c 2 + ab + bc + ca]
= –2 (a + b + c) [a 2 + b 2 + c 2 – ab – bc – ca]
= – (a + b + c) [2a 2 + 2b 2 + 2c 2 – 2ab – 2bc – 2ca]
= – (a + b + c) [(a – b) 2 + (b – c) 2 + (c – a) 2]
Now, given that D = 0
⇒ D = (a + b + c) [(a – b)2 + (b – c)2 + (c – a)2] = 0

So, either (a + b + c) = 0 or (a – b)2 + (b – c)2 + (c – a)2 = 0 i.e., a = b = c.

Determinants 107
@Cbsebookshub - Join Us on Telegram
11. Show that points A(a, b + c), B(b, c + a), C(c, a + b) are collinear.

Sol. We have,
a b+c 1
1
Area of DABC = b c+a 1
2
c a+b 1
a a+b+c 1
1
= b b + c + a 1 (Applying C2 → C2 + C1)
2
c a+b+c 1
a 1 1
1
= (a + b + c) b 1 1 (Taking (a + b + c) common from C2)
2
c 1 1
1
= # (a + b + c) # 0 ( C2 = C3)
2
⇒ ar(DABC) = 0

Since area of DABC is zero, therefore points A, B and C are collinear.
Hence proved.

Multiple Choice Questions [1 mark]


Choose and write the correct option in the following questions.

x 2 6 2
1. If = , then x is equal to
18 x 18 6
(a) 6 (b) ± 6 (c) – 6 (d) 0

a–b b+c a
2. The value of determinant b – c c + a b [NCERT Exemplar]
c –a a+b c
(a) a3 + b3 + c3 (b) 3bc (c) a3 + b3 + c3 – 3abc (d) None of these
3. The area of a triangle with vertices (–3, 0),(3, 0) and (0, k) is 9 sq. units. The value of k will be
(a) 9 (b) 3 (c) –9 (d) 6
–1 cos C cos B
4. If A, B and C are angles of a triangle, then the determinant cos C –1 cos A is equal to
cos B cos A –1
(a) 0 (b) –1 (c) 1 (d) None of these
0 x–a x–b
+
5. If f(x) = x a 0 x – c , then [NCERT Exemplar]
x+b x+c 0
(a) f(a) = 0 (b) f(b) = 0 (c) f(0) = 0 (d) f(1) = 0

1+x 1 1
6. If x, y, z are all different from zero and 1 1+y 1 = 0, then value of x–1 + y–1 + z–1 is
1 1 1+z
(a) xyz (b) x–1 y–1 z–1 (c) –x –y –z (d) –1

108 Xam idea Mathematics–XII

@Cbsebookshub - Join Us on Telegram


1 –2 5
7. There are two values of a which makes determinant D = 2 a –1 = 86 , then sum of these
numbers is 0 4 2a
(a) 4 (b) 5 (c) – 4 (d) 9
8. If A is a non-singular square matrix of order 3 such that A2 = 3A, then value of |A| is
 [CBSE 2020 (65/2/1)]
(a) –3 (b) 3 (c) 9 (d) 27
2 3 2
9. If x x x + 3 = 0 , then the value of x is [CBSE 2020 (65/4/1)]
4 9 1
(a) 3 (b) 0 (c) –1 (d) 1
x x + y x + 2y
10. The value of the determinant x + 2y x x + y is [NCERT Exemplar]
+ +
x y x 2y x
(a) 9x2(x + y) (b) 9y2(x + y) (c) 3y2(x + y) (d) 7x2(x + y)
x + 2 x + 3 x + 2a
11. If a, b, c are in AP, then the value of determinant D = x + 3 x + 4 x + 2b is
x + 4 x + 5 x + 2c
(a) 0 (b) 1 (c) x (d) 2x
2
1 a a 1 1 1
2 =
12. The value of 1 b b a b c is
1 c c 2
a2 b2 c2
(a) (a – b) (b – c) (c – a) (b) (b – a) (c – b) (c – a)
(c) a (b – c) (c – a) (d) None of these
0 a–b a–c
13. The value of b – a 0 b – c is
c–a c–b 0
(a) a (b) b (c) 0 (d) None of these
1 ~ ~2
14. The value of ~ ~ 2 1 is
~2 ~ 1
(a) 1 (b) –1 (c) 0 (d) ω

15. If area of triangle is 35 sq units with vertices (2, –6) (5, 4) and (k, 4), then k is
(a) 12 (b) –2 (c) –12, –2 (d) 12, –2
16. Let A be a square matrix of order 3 × 3, then |KA| is equal to
(a) K|A| (b) K2|A| (c) K3|A| (d) 3K|A|
265 240 219
17. The value of 240 225 198 is
219 198 181
(a) 0 (b) 1 (c) –1 (d) None

Determinants 109
@Cbsebookshub - Join Us on Telegram
1 a b+c
18. The value of 1 b c + a is
1 c a+b
(a) 1 (b) 0 (c) a + b (d) a – b
a11 a12 a13
19. If T = a21
a22 a23 and Aij is cofactors of aij, then value of D is given by
a31 a32 a33
(a) a11 A31 + a12 A32 + a13 A33 (b) a11 A11 + a12 A21 + a13 A31
(c) a21 A11 + a22 A12 + a23 A13 (d) a11 A11 + a21 A21 + a31 A31
20. If A is a 3 × 3 matrix such that |A| = 8, then |3A| equals [CBSE 2020 (65/5/1)]
(a) 8 (b) 24 (c) 72 (d) 216

Answers
1. (b) 2. (c) 3. (b) 4. (a) 5. (c) 6. (d)
7. (c) 8. (d) 9. (c) 10. (b) 11. (a) 12. (a)
13. (c) 14. (c) 15. (d) 16. (c) 17. (a) 18. (b)
19. (d) 20. (d)

Solutions of Selected Multiple Choice Questions


1. We have,
x 2 6 2
=
18 x 18 6
⇒ x2 – 36 = 36 – 36

⇒ x2 – 36 = 0

⇒ x2 = 36

⇒ x = ± 6

2. We have,
a–b b+c a a+c b+c+a a
b – c c + a b = b + a c + a + b b [Applying C1 → C1 + C2 and C2 → C2 + C3]
c– a a+b c c+b a+b+c c
a+c 1 a
= (a + b + c) b + a 1 b [Taking (a + b + c) common from C2]
c+b 1 c
a–b 0 a–c
= (a + b + c) a – c 0 b – c [Applying R2 → R2 – R3 and R1 → R1 – R3]
c+b 1 c
= (a + b + c) [– (a – b) (b – c) + (a – c)2] [Expanding along R2]
= (a + b + c) (a2 + b2 + c2 – ab – bc – ca) = (a3 + b3 + c3 – 3abc)
3. We know that, area of a triangle with vertices (x1, y1), (x2, y2) and (x3, y3) is given by
x1 y1 1 –3 0 1
1 1
x
D = ; 2 2y 1 ;= ; 3 0 1 ;
2 2
x3 y3 1 0 k 1

110 Xam idea Mathematics–XII

@Cbsebookshub - Join Us on Telegram


Expanding along R1, we get
1
9 = ; [–3 (– k) – 0 + 1 (3k)] ; ⇒ 18 = |3k + 3k| = |6k|
2
18
∴ k = ! = ! 3 = 3, – 3
6
0 x–a x–b
+
5. We have, f (x) = x a 0 x – c
x+b x+c 0
0 0 a–b
⇒ f (a) = 2a
0 a–c = [(a – b){2a. (a + c)}] ≠ 0
a+b a+c 0
0 b–a 0
and f (b) = b + a 0 b – c = – (b – a)[–2b(b – c)] = 2b(b – a)(b – c) ≠ 0
2b b + c 0
0 –a –b
and f (0) = a 0 – c = a(bc) – b(ac) = abc – abc = 0
b c 0
1 –2 5
7. We have, D = 2 a –1 = 86
0 4 2a
⇒ 1(2a2 + 4) – 2(– 4a – 20) + 0 = 86
[Expanding along first column]
2

2a + 4 + 8a + 40 = 86

2a2 + 8a + 44 – 86 = 0

a2 + 4a – 21 = 0

a2 + 7a – 3a – 21 = 0

(a + 7)(a – 3) = 0 ⇒ a = – 7 and 3

Required sum = – 7 + 3 = – 4
8. We have,
A2 = 3A  ⇒ |A2| = |3A|
⇒ |A|.|A| = 33 |A|
( order of matrix A is 3 and |A| is not equal to zero)
3
⇒ |A| = 3 = 27  ⇒ |A| = 27

9. We have,
2 3 2
x x x + 3 = 0
4 9 1
2 3 2
⇒ x 1 1 1 + 3 = 0
(Taking out x from R2)
4 9 1
⇒ x {2 (1 – 9) – 3 (1 – 4) + 2 (9 – 4)} + 3 = 0

⇒ x (– 16 + 9 + 10) + 3
⇒ 3x + 3 = 0
⇒ 3x = – 3  ⇒  x = – 1
20. We have,
|3A| = 33 |A| = 27 × 8 = 216

Determinants 111
@Cbsebookshub - Join Us on Telegram
Fill in the Blanks [1 mark]
sin 2 23° sin 2 67° cos 180°
1. The value of the determinant – sin 2 67° – sin 2 23° cos 2 180° is _____________ .
cos 180° sin 2 23° sin 2 67°
RS V
SS2 –3 5 WWW
2. The cofactor of element a12 in the matrix SS6 0 4 WW is _____________ .
SS W
S1 5 –7WW
T X
3. If A is a skew-symmetric matrix of order 3, then the value of |A| = _____________ .
2
0 cos i sin i
4. If cos 2q = 0, then cos i sin i 0 = _____________ . [NCERT Exemplar]
sin i 0 cos i
x 3 7
5. If x = – 9 is a root of 2 x 2 = 0 , then other two roots are _____________ . [NCERT Exemplar]
7 6 x
Answers
1
1. 0 2. 46 3. 0 4. 5. x = 2, x = 7
2
Solutions of Selected Fill in the Blanks
1. Applying C1 → C1 + C2 + C3, we have
sin 2 23° + sin 2 67° + cos 180° sin 2 67° cos 180°
– sin 2 67° – sin 2 23° + cos 2 180° – sin 2 23° cos 2 180°
cos 180° + sin 2 23° + sin 2 67° sin 2 23° sin 2 67°
JKa sin 2 67° = cos 2 23°NO
1 + (–1) sin 2 67° –1 KK OO
= –1 + 1 – sin 2 23° 1 KK cos 180° = –1 OO
KK O
–1 + 1 sin 2 23° sin 2 67° and sin 2
i + cos 2 i = 1 O
L P
0 sin 2 67° –1
= 0 – sin 2 23° 1 =0
2 2
0 sin 23° sin 67°
3. Since matrix A is a skew-symmetric of odd order i.e. 3
∴ |A|= 0

Very Short Answer Questions [1 mark]


1. If A and B are square matrices of the same order 3, such that |A|= 2 and AB = 2I, write the
value of |B|. [CBSE Delhi 2019]
Sol. |A| = 2 and AB = 2I
⇒ |AB| = |2I| = 8

⇒ |A||B| = 8
⇒ 2 |B| = 8 ⇒ |B| = 4

2. Find |AB|, if A = = G and B = = G [CBSE 2019 (65/2/2)]


0 –1 3 5

0 2 0 0

Sol. We have, AB = = G= G== G


0 –1 3 5 0 0
0 2 0 0 0 0

112 Xam idea Mathematics–XII

@Cbsebookshub - Join Us on Telegram


0 0

AB = =0
0 0
3. Let A be a square matric of order 3 × 3. Write the value of 2A , where A = 4.
[CBSE Delhi 2012]
n
Sol. a 2 A =2 A , where n is order of matrix A.

Here A = 4 and n = 3

2A = 23 × 4 = 32

x+1 x –1 4 –1
4. If = , then write the value of x. [CBSE Delhi 2013]
x –3 x+2 1 3
x+1 x–1 4 –1
Sol. Given =
x–3 x+2 1 3
⇒ (x + 1) (x + 2) – (x – 1) (x – 3) = 12 + 1


x2 + 2x + x + 2 – x2 + 3x + x – 3 = 13
⇒ 7x – 1 = 13

⇒ 7x = 14


x = 2
5. If A = [aij] is a matrix of order 2 × 2 , such that |A| = –15 and Cij represents the cofactor of aij,
then find a21C21 + a22C22. [CBSE Sample Paper 2018]
a11 a12 a11 a12
Sol. Given, A=> H     \ A =
a21 a22 a21 a22
Expanding along R2
⇒ –15 = a21 . C21 + a22 . C22 [Cij = Cofactor of aij]

a21. C21 + a22 . C22 = – 15
6. Write the value of the following determinant:
a–b b–c c–a
b – c c – a a – b [CBSE (AI) 2009; (East) 2016]
c–a a–b b–c
Sol. Applying C1 → C1 + C2 + C3, we get
0 b–c c–a
= 0 c – a a – b = 0 [ All elements of Cl are zero]
0 a–b b–c
7. Show that the points (1, 0), (6, 0), (0, 0) are collinear. [CBSE (AI) 2008]
1 0 1
Sol. Since 6 0 1 = 0
0 0 1
Hence, (1, 0), (6, 0) and (0, 0) are collinear.
8. What positive value of x makes the following pair of determinants equal?
2x 3 16 3
,
5 x 5 2 [CBSE (AI) 2010]
2x 3 16 3
Sol.  =
5 x 5 2

Determinants 113
@Cbsebookshub - Join Us on Telegram

2x2 – 15 = 32 – 15 ⇒ 2x2 = 32

x2 = 16 ⇒ x=±4

x = 4 (+ve value).
cos 15° sin 15°
9. Evaluate: [CBSE (AI) 2011]
sin 75° cos 75°
Sol. Expanding the determinant, we get
cos 15°. cos 75° – sin 15°. sin 75° = cos (15° + 75°) = cos 90° = 0
[Note : cos (A + B) = cos A. cos B – sin A. sin B]
10. Write the value of the following determinant:
102 18 36
1 3 4 [CBSE (F) 2012]
17 3 6
102 18 36
Sol. Let D = 1 3 4
17 3 6
Applying R1 " R1 – 6R3 , we get
0 0 0
D = 1 3 4 = 0 [ Each element of R1 is zero]
17 3 6
11. If A is a square matrix and A = 2, then write the value of AAl , where Al is the transpose of
matrix A. [CBSE (F) 2013]
AAl = A . Al = A . A = A 2 = 2 2 = 4
Sol.
[Note: AB = A . B and A = AT , where A and B are square matrices.]

12. If A is a 3 × 3 invertible matrix, then what will be the value of k if det (A–1) = (det A)k.
[CBSE Delhi 2017]
Sol. Given, det (A–1) = (det A)k

⇒ |A–1| = |A|k ⇒
k = –1

Short Answer Questions-I and II [2, 3 marks]


1. Find the equations of line joining (1,2) and (3,6) using determinants.
Sol. Let given points are A(1, 2) and B(3, 6) and P(x, y) lies on the line joining points A and B.
∴ Points A,P and B are collinear

∴ Area of ∆ APB = 0

1 2 1 1 2 1
1

x y 1 = 0 ⇒ x y 1 =0
2
3 6 1 3 6 1
⇒ 1 (y – 6) – 2 (x – 3) + 1 (6x – 3y) = 0

⇒ y – 6 – 2x + 6 + 6x – 3y = 0 ⇒ 4x – 2y = 0
⇒ 2(2x – y) = 0
⇒ 2x – y = 0
∴ Equation of line be 2x – y = 0

114 Xam idea Mathematics–XII

@Cbsebookshub - Join Us on Telegram


x2 – x + 1 x – 1
2. Evaluate the determinant: [NCERT Exemplar]
x+1 x+1
x2 – x + 1 x – 1
Sol. Let D =
x+1 x+1
x2 – x + 1 x – 1
= (x + 1) [Taking common (x + 1) from R2]
1 1
= (x + 1){x2 – x + 1 – x + 1} = (x + 1) (x2 – 2x + 2)
= x3 – 2x2 + 2x + x2 – 2x + 2 = x3 – x2 + 2
3. What is the value of the following determinant: [CBSE (F) 2010]
4 a b+c
4 b c+a
4 c a+b
4 a b+c 4 a a+b+c
Sol. Here, T = 4 b c + a = 4 b a + b + c [Applying C3 → C3 + C2]
4 c a+b 4 c a+b+c
1 a 1
= + +
T 4 (a b c) 1 b 1 [Taking out common 4 from C1 and a + b + c from C3]
1 c 1
a C = C3
D = 4 (a + b + c) . 0 = 0 = 1 G
`T=0
x+y y+z z+x
4. Write the value of T = z x y . [CBSE Allahabad 2015]
–3 –3 –3
x+y y+z z+x
Sol. Here, T = z x y
–3 –3 –3
Applying R1 → R1 + R2, we get
x+y+z x+y+z x+y+z
= z x y
–3 –3 –3
Taking (x + y + z) common from R1 , we get
1 1 1
= (x + y + z) z x y
–3 –3 –3
Applying R3 → R3 + 3R1, we get
1 1 1
= (x + y + z) z x y = 0 [ Each element of R3 is zero]
0 0 0
5. Without expanding evaluate the determinant: [HOTS]
x –x 2 x –x 2
( a + a ) (a – a ) 1
(a y + a –y) 2 (a y – a –y) 2 1 , where a > 0 and x, y, z ∈ R
(a z + a –z) 2 (a z + a –z) 2 1

Determinants 115
@Cbsebookshub - Join Us on Telegram
(a x + a –x) 2 (a x – a –x) 2 1
D = (a y + a –y) 2 (a y – a –y) 2 1
Sol. Here
(a z + a –z) 2 (a z + a –z) 2 1
Applying C1 → C1 – C2, we get
4 (a x – a –x) 2 1
T = 4 (a y – a –y) 2 1 [Using (a + b)2 – (a – b)2 = 4ab]
4 (a z – a –z) 2 1
Taking out 4 from C1, we get
1 (a x – a –x) 2 1
T = 4 1 (a y – a –y) 2 1 ⇒ D = 4 × 0 = 0. [ C1 and C3 are identical]
1 (a z – a –z) 2 1

Long Answer Questions [5 marks]


1. If a, b, c are pth, qth and rth terms respectively of a G.P, then prove that
log a p 1
log b q 1 = 0 [CBSE 2020 (65/5/1)]
log c r 1
Sol. Let A be the first term and R be the common ratio of the G.P respectively.
∴ a = AR p–1, b = AR q–1, c = AR r–1

Now, we have,
log a p 1
L.H.S. T = log b q 1
log c r 1

log AR p–1 p 1

T = log AR q–1 q 1
log AR r–1 r 1

log A + (p–1) log R p 1


⇒ T = log A + (q–1) log R q 1
log A + (r–1) log R r 1
 (log(ab) = log a + log b and logam = m log a)

log A p 1 p–1 p 1
T = log A q 1 + log R q–1 q 1


log A r 1 r–1 r 1
apply C2 " C2 – C1
1 p 1 p–1 1 1
= 1 q 1 + log R q–1 1 1
⇒ T log A

1 r 1 r–1 1 1
(C1 = C3) (C2 = C3)
T = log A × 0 + log × 0
` T = 0 + 0 = 0 = R.H.S

` T=0

116 Xam idea Mathematics–XII

@Cbsebookshub - Join Us on Telegram


4 –x 4 + x 4 + x
2. Using properties of determinants, find the value of x for which 4 + x 4 – x 4 + x = 0 .
4+x 4+x 4– x

 4 –x 4 + x 4 + x [CBSE 2019(65/4/3)]
Sol. We have, 4+x 4– x 4+x =0
4+x 4+x 4– x
Applying C1 → C1 + C2 + C3, we get
12 + x 4 + x 4 + x
12 + x 4 – x 4 + x = 0
12 + x 4 + x 4 – x
1 4+x 4+x

(12 + x) 1 4 – x 4 + x = 0 [ Taking (12 + x) common from C1 ]
1 4+x 4– x
Applying R2 → R2 – R1, R3 → R3 – R1, we get
1 4+x 4+x
(12 + x) 0 –2x 0 =0
0 0 –2x
⇒ (x + 12)(4x2) = 0 ⇒
x = 0, –12
3. Using properties of determinants, prove that [CBSE Delhi 2014]
2y y–z–x 2y
z – x – y = _x + y + zi .
3
2z 2z
x–y–z 2x 2x
2y y–z–x 2y
Sol. LHS = 2z 2z z–x–y
x–y–z 2x 2x

Applying R2 ↔ R3, then R1 ↔ R2, we get


x–y–z 2x 2x
= 2y y – z – x 2y
2z 2z z – x – y
Applying R1 → R1 + R2 + R3, we get
x+y+z y+z+x z+x+y
= 2y y–z–x 2y
2z 2z z–x–y
Taking out (x + y + z) from first row, we get
1 1 1
= (x + y + z) 2y y – z – x 2y
2z 2z z – x – y
Applying C1 → C1 – C3 and C2 → C2 – C3, we get
0 0 1
= (x + y + z) 0 ( y + z + x) 2y
(x y + z)
+ z+x+y z–x–y

Determinants 117
@Cbsebookshub - Join Us on Telegram
Expanding along first row, we get
= (x + y + z) (x + y + z)2 = (x + y + z)3 = RHS

x x 2 x 3 –1
2 3
4. If x, y, z are different and D = y y y –1 = 0, then using properties of determinants, show
z z 2 z 3 –1
that xyz = 1. [CBSE 2019 (65/5/1)]

x x 2 x 3 –1 x x2 x3 x x 2 –1
2 3
Sol. We have y y y –1 = 0 ⇒ y y y + y y 2 –1 = 0
2 3

z z 2 z 3 –1 z z2 z3 z z 2 –1
1 x x2 –1 x 2 x SRSIn det 1 taking x, y, z common WVW
2 2
S
SSfrom each row and in det 2 WW
⇒ xyz 1 y y + –1 y y = 0
SS WW
S WW
1 z y 2 2
–1 z z Susing C1 * C3
and applying C2 * C3
in det 2 W
T X
2 2
1 x x –1 x x
2 2
⇒ xyz 1 y y + –1 y y = 0

1 z z2 –1 z z 2
1 x x2 1 x x2
2 2
⇒ xyz 1 y y + (–1) 1 y y = 0 [Taking (-1) common from C1]

1 z z2 1 z z2
1 x x2
2
⇒ 1 y y (xyz – 1) = 0
1 z z2
RSApplying VW
1 x x2 1 x x2 SS W
2 SSR " R – R WWW
If 1 y y = 0 ⇒ 0 y–x y2 – x2 = 0 SS 2 2 1W

1 z z2 0 z–x z2 – x2 SSR " R – R WWW


3 3 1
T X
y – x y2 – x2
^ y – x h]z – xg
1 y+x

= 0 ⇒ =0
z–x 2
z –x 2 1 z+x

⇒ (y – x)(z – x)(z + x – y – x) = 0

⇒ (y – x)(z – x)(z – y) = 0

⇒ x = y or z = x or y = z, which is a contradiction.

Hence, (xyz – 1) = 0 ⇒ xyz = 1.

a + b + 2c a b
= 2 ^a + b + c h [CBSE Delhi 2014; (F) 2010, 2011]
3
5. Prove that: c b + c + 2a b
c a c + a + 2b
a + b + 2c a b
Sol. LHS = c b + c + 2a b
c a c + a + 2b
Applying C1 → C1 + C2 + C3, we get
2 (a + b + c) a b
= 2 (a + b + c) b + c + 2a b
2 (a + b + c) a c + a + 2b

118 Xam idea Mathematics–XII

@Cbsebookshub - Join Us on Telegram


Taking 2(a + b + c) common from C1, we get
1 a b
= 2 (a + b + c) 1 b + c + 2a b
1 a c + a + 2b
Applying R2 → R2 – R1 and R3 → R3 – R1, we get
1 a b
+
= 2 (a + b + c) 0 a b c+ 0
0 0 a+b+c
Taking (a + b + c) common from R2 and R3, we get
1 a b
3
= 2 (a + b + c) 0 1 0
0 0 1
Expanding along C1, we get
= 2 (a + b + c) 3 [1 – 0] = 2 (a + b + c) 3 = RHS

1 a a2 a3 – 1 0 a – a4
6. If D = a a 2 1 = –4 then find the value of 0 a – a 4 a 3 – 1 . [CBSE Sample Paper 2018]
2
a 1 a a – a a3 – 1
4
0

1 a a2
Sol. We have D = a a2 1
a2 1 a

C11 = a 3 – 1; C12 = 0; C13 = a – a 4


C21 = 0; C22 = a – a 4; C23 = a 3 – 1
C31 = a – a 4; C32 = a 3 – 1; C33 = 0

Where Cij = Co-factor of aij (i, j)th element of determinant D. Let D1 be the determinant made by
corresponding co-factor of each element of determinant D.
C11 C12 C13
i.e., D1 = C21 C22 C23

C31 C32 C33
[ D1 = Dn–1 where each element of D1 is cofactor

We know that D1 = D2

of corresponding element of D and n is order of the
\ D1 = (– 4)2 = 16
determinant]

a3 – 1 0 a – a4

0 a – a a 3 – 1 = 16
4

a – a4 a3 – 1 0

7. Using property of determinant, prove the following:


a a + b a + 2b
+
a 2b a a + b = 9b2 (a + b) [CBSE Delhi 2017; (AI) 2008, 2013]
a + b a + 2b a
a a + b a + 2b
= +
Sol. LHS a 2b a a+b
a + b a + 2b a

Determinants 119
@Cbsebookshub - Join Us on Telegram
3 (a + b) 3 (a + b) 3 (a + b)
= a + 2b a a + b [Applying R1 = R1 + R2 + R3]
a+b a + 2b a
1 1 1
= 3 (a + b) a + 2b a a + b [Taking 3(a + b) common from R1]
a + b a + 2b a
0 0 1
= + +
3 (a b) b – b a b [Applying C1 → C1 – C3, C2 → C2 – C3]
b 2b a
Expanding along R1, we get
= 3(a + b) {1 (2b2 + b2)} = 9b2(a + b) = RHS
x y x+y
8. Using properties of determinants, find the value of k if y x + y x = k (x3 + y3) .
x+y x y
[CBSE x y x+y 2019 (65/4/2)]
3 3
Sol. We have k (x + y ) = y x+y x
x+y x y
2x + 2y y x + y
= 2x + 2y x + y x [Using C1 → C1 + C2 + C3]
2x + 2y x y
1 y x+y
` = ( 2x 2y ) 1 x + y x
+ [Taking (2x + 2y) common from C1]
1 x y
1 y x+y
= 2 (x + y) 0 x –y [Applying R2 → R2 – R1, R3 → R3 – R1]
0 x – y –x
x –y
= 2 (x + y)
x – y –x
= 2(x + y)(–x2 + xy – y2) = –2(x + y)(x2 – xy + y2)

k(x3 + y3) = –2(x3 + y3)
Comparing the coefficient of (x3 + y3) on both the sides, we get
k = –2
9. Using properties of determinants show that
1 1 1+x
+
1 1 y 1 = – (xyz + yz + zx + xy) [CBSE (F) 2017]
1+z 1 1

1 1 1+x
Sol. LHS = 1 1+y 1
1+z 1 1
Apply R2 → R2 – R1 and R3 → R3 – R1, we get

1 1 1+x
= 0 y –x
z 0 –x

120 Xam idea Mathematics–XII

@Cbsebookshub - Join Us on Telegram


Expanding by R1, we get
= 1 [–yx – 0] – 1[+ zx] + (1 + x) (–zy) = – yx – zx – zy – xyz
= – (xy + xz + zy + xyz) = RHS

10. Using properties of determinant, prove that:


a+x y z
x a + y z = a2 ^a + x + y + z h [CBSE (F) 2014]
x y a+z
a+x y z
=
Sol. LHS x a+y z
x y a+z
a+x+y+z y z
= a + x + y + z a + y z [Applying C1 → C1 + C2 + C3]
a+x+y+z y a+z
1 y z
= (a + x + y + z) 1 a + y z [Taking out (a + x + y + z) common from C1]
1 y a+z
0 –a 0
= + + + +
(a x y z) 1 a y z [Apply R1 → R1 – R2]
1 y a+z
Expanding along R1, we get
= (a + x + y + z) {0 + a (a + z – z)}
= a2(a + x + y + z) = RHS
11. Using properties of determinant, prove the following:
x+y x x
5x + 4y 4x 2x = x 3 [CBSE (AI) 2014, 2009]
10x + 8y 8x 3x
x+y x x
= 5x + 4y 4x 2x
Sol. LHS
10x + 8y 8x 3x
x+y 1 1
= x 2 5x + 4y 4 2 [Taking out x from C2 and C3]
10x + 8y 8 3
x+y 1 1
= x 2 3x + 2y 2 0 [Applying R2 → R2 – 2R1 and R3 → R3 – 3R1]
7x + 5y 5 0
Expanding along C3, we get
= x2 [1 {(3x + 2y) 5 – 2 (7x + 5y)} – 0 + 0] = x2 (15x + 10y – 14x – 10y)
= x2 (x) = x3 = RHS

cosec 2 i cot 2 i 1
12. Without expanding, show that: cot 2 i cosec 2 i –1 = 0 [NCERT Exemplar]
42 40 2

Determinants 121
@Cbsebookshub - Join Us on Telegram
cosec 2 i cot 2 i 1
Sol. Given, D = cot 2 i cosec 2 i –1
42 40 2

cosec 2 i – cot 2 i – 1 cot 2 i 1


= cot 2 i – cosec 2 i + 1 cosec 2 i – 1 [Applying C1 → C1 – C2 – C3]
0 40 2

1 – 1 cot 2 i 1
= – 1 + 1 cosec 2 i – 1 [ a cosec2 q – cot2 q = 1]
0 40 2

0 cot 2 i 1
= 0 cosec 2 i – 1 = 0 [ a All elements of C1 are 0]
0 40 2
13. Prove the following using properties of determinant:
b+c c+a a+b
c + a a + b b + c = 2 ^3abc – a3 – b3 – c3 h [CBSE (F) 2010]
a+b b+c c+a
b+c c+a a+b
= c+a a+b b+c
Sol. LHS
a+b b+c c+a

2 (a + b + c) 2 (a + b + c) 2 (a + b + c)
= c+a a+b b+c [Applying R1 → R1 + R2 + R3]
a+b b+c c+a

1 1 1
= 2 (a + b + c) c + a a + b b + c [Taking 2(a + b + c) common from R1]
a+b b+c c+a

1 0 0
= + + +
2 (a b c) c a b – c b – a [Applying C2 → C2 – C1; C3 → C3 – C1]
a+b c– a c–b
= 2(a + b + c) [1(bc – b2 – c2 + bc – bc + ac + ab – a2)] [Expanding along R1]
2 2 2
= 2(a + b + c) (bc + ac + ab – a – b – c )
= –2(a + b + c) (a2 + b2 + c2 – ab – bc – ca) = –2(a3 + b3 + c3 – 3abc)
= 2(3abc – a3 – b3 – c3) = RHS

a b c
14. If a + b + c ! 0 and b c a = 0 , then using properties of determinants, prove that a = b = c.
c a b
[NCERT Exemplar, CBSE Bhubaneswar 2015]
a b c
Sol. We have b c a = 0
c a b
Applying C1 → C1 + C2 + C3 , we get

122 Xam idea Mathematics–XII

@Cbsebookshub - Join Us on Telegram


(a + b + c) b c
(a + b + c) c a = 0
(a + b + c) a b
Taking (a + b + c) common from C1 , we get
1 b c 1 b c
(a + b + c) 1 c a = 0 & 1 c a = 0 [ a a + b + c ! 0]
1 a b 1 a b
Applying R2 → R2 – R1 and R3 → R3 – R1 , we get
1 b c
0 c – b a – c = 0
0 a–b b–c
Expanding along C1 , we get
1{(c – b) (b – c) – (a – c) (a – b)} – 0 + 0 = 0 ⇒ – (b – c)2 – (a – c) (a – b) = 0
⇒ – b2 – c2+ 2bc – a2 + ab + ac – bc = 0
⇒ a2 + b2 + c2 – bc – ab – ac = 0
1
⇒ 62a 2 + 2b 2 + 2c 2 – 2bc – 2ab – 2ac@ = 0 ⇒ (a – b)2 + (b – c)2 + (c – a)2 = 0

2
⇒ (a – b)2 = 0; (b – c)2 = 0; (c – a)2 = 0
⇒ a – b = 0; b – c = 0; c – a = 0;
⇒ a = b = c

15. Using properties of determinants, show that DABC is an isosceles if:
1 1 1
1 cos A + +
1 cos B +
1 cos C = 0 [CBSE (Central) 2016, NCERT Exemplar, HOTS]
2 2 2
cos A + cos A cos B + cos B cos C + cos C
1 1 1
1 + cos A 1 + cos B 1 + cosC =0
Sol. We have 2 2 2
+ + +
cos A cos A cos B cos B cos C cosC
Applying C1 → C1 – C3 and C2 → C2 – C3 , we get
0 0 1

& cos A – cos C cos B – cos C +
1 cos C =0
2 2 2 2 2
cos A + cos A – cos C – cos C cos B + cos B – cos C – cos C cos C + cos C
0 0 1
&
cos A – cos C cos B – cos C 1 + cos C =0
(cos A – cos C) (cos A + cos C + 1) (cos B – cos C) (cos B + cos C + 1) cos 2 C + cos C

Taking common (cos A – cos C) from C1 and (cos B – cos C) from C2, we get
0 0 1
& (cos A – cos C) (cos B – cos C) 1 1 1 + cos C =0
2
cos A + cos C + 1 cos B + cos C + 1 cos C + cos C
Applying C1 → C1 – C2, we get
0 0 1
⇒ (cos A – cos C) (cos B – cos C)
0 1 1 + cos C =0
cos A – cos B cos B + cos C + 1 cos 2 C + cos C
Expanding along R1, we get
⇒ (cos A – cos C) (cos B – cos C) (cos B – cos A) = 0

⇒ cos A – cos C = 0 i.e., cos A = cos C

Determinants 123
@Cbsebookshub - Join Us on Telegram
or, cos B – cos C = 0 i.e., cos B = cos C
or, cos B – cos A = 0 i.e., cos B = cos A
⇒ A = C or B = C or B = A
Hence, DABC is an isosceles triangle.
16. Using properties of determinants, prove the following:
x + 4 2x 2x
2x x 4 2x = ^5x + 4 h^ 4 – x h [CBSE Delhi 2011]
2
+
2x 2x x + 4
OR
x + m 2x 2x
x + m 2x = ^5x + m h^m – x h [CBSE (F) 2014]
2
2x
2x 2x x+m

x + 4 2x 2x
= 2x x + 4 2x
Sol. LHS
2x 2x x + 4
5x + 4 5x + 4 5x + 4
= 2x x+4 2x [Applying R1 → R1 + R2 + R3]
2x 2x x+4
1 1 1
= (5x + 4) 2x x + 4 2x [Taking (5x + 4) common from R1]
2x 2x x + 4
1 0 0
= (5x + 4) 2x 4 – x 0 [Applying C2 → C2 – C1; C3 → C3 – C1]
2x 0 4 – x
= (5x + 4) [1 {(4 – x)2 – 0} + 0 + 0] [Expanding along R1]
2
= (5x + 4) (4 – x) = RHS

OR
Solve as above by putting l instead of 4.
17. Using properties of determinants, prove that [CBSE Delhi 2012]
b+c q+r y+z a p x
c+a r+p z+x = 2 b q y
a+b p+q x+y c r z
OR
b+c c+a a+b a b c
+ + + =
q r r p p q 2 p q r [CBSE (AI) 2014]
y+z z+x x+y x y z
b+c q+r y+z
= c+a r+p z+x
Sol. LHS
a+b p+q x+y
a+b p+q x+y
= b + c q + r y + z [Applying R1 ↔ R3 and R3 ↔ R2]
c+a r+p z+x

124 Xam idea Mathematics–XII

@Cbsebookshub - Join Us on Telegram


Applying R1 → R1 + R2 + R3, we get

2 (a + b + c) 2 (p + q + r) 2 (x + y + z) a+b+c p+q+r x+y+z


= +
b c +
q r +
y z = 2 b+c q+r y+z
c+a r+p z+x c+a r+p z+x
a p x
= 2 b + c q + r y + z [Applying R1 → R1 – R2]
c+a r+p z+x
a p x
= + + +
2b c q r y z [Applying R3 → R3 – R1]
c r z

Again applying R2 → R2 – R3 , we get


a p x
= 2 b q y = RHS
c r z
18. Using properties of determinant, prove the following: [CBSE Delhi 2012; (AI) 2014]
1+a 1 1
+
1 1 b 1 = ab + bc + ca + abc
1 1 1+c
OR
1+a 1 1 1 1 1
If a, b and c are all non-zero and 1 1 + b 1 = 0 , then prove that + + + 1 = 0
a b c
1 1 1+c
[CBSE (F) 2016]
1+a 1 1
= +
Sol. LHS 1 1 b 1
1 1 1+c

1 1 1
+
a 1 a a
1 1 1
= abc +1 [Taking out a, b, c common from R1, R2 and R3]
b b b
1 1 1
+
c c c 1

1 1 1 1 1 1 1 1 1
+ + + + + + + + +
a b c 1 a b c 1 a b c 1
1 1 1
= abc +1 [Applying R1 → R1 + R2 + R3]
b b b
1 1 1
+
c c c 1
1 1 1
= abc c a + + c + 1 m
1 1 1 1 1 1
+1
b b b b
1 1 1
+
c c c 1
Applying C2 → C2 – C1; C3 → C3 – C1, we get

Determinants 125
@Cbsebookshub - Join Us on Telegram
1 0 0
1 1 1 1 1 1 1
= abc c + + + 1 m 1 0 = abc c + + + 1 m # {1 (1 – 0) – 0 + 0)}
a b c b a b c
1
0 1
c

= abc c a + + c + 1 m = ab + bc + ca + abc = RHS


1 1 1
b
OR
1+a 1 1
abc c a + + c + 1 m = 0
1 1 1 1 1 1

a 1 1+b 1 = 0 & b
& + + + =
a b c 1 0 [a, b, c are non-zero]
1 1 1+c
19. Using properties of determinants, show the following:
(b + c) 2 ab ca
ab (a + c) 2 bc = 2abc (a + b + c) 3 [CBSE Delhi 2010]
ac bc ( a + b) 2
(b + c) 2 ab ca
Sol. LHS = ab (a c) 2
+ bc
ac bc (a + b) 2
Multiplying R1, R2 and R3 by a, b and c respectively, we get
a (b + c) 2 ba 2 a2 c
1
= ab 2 b (a + c) 2 b2 c
abc
ac 2 bc 2 c (a + b) 2
(b + c) 2 a2 a2
1
= abc b2 ( c + a) 2 b 2 [Taking common a, b and c from C1, C2 and C3 respectively]
abc
c2 c2 (a + b) 2
Applying C1 → C1 – C3 and C2 → C2 – C3, we get
(b + c) 2 – a 2 0 a2
= 0 ( c + a) 2 – b 2 b2
2 2
c – (a + b) c 2 – (a + b) 2 (a + b) 2
(b + c + a) (b + c – a) 0 a2
= 0 (c + a + b) (c + a – b) b2
(c + a + b) (c – a – b) (c + a + b) (c – a – b) (a + b) 2
b+c– a 0 a2
[Taking common (a + b + c)
= (a + b + c)2 0 c+a–b b2
2 from C1 and C2]
c–a–b c–a–b (a + b)
b+c– a 0 a2
2
= (a + b + c) 0 c+a–b b2 [R3 → R3 – (R1 + R2)]
–2b – 2a 2ab
ab + ac – a 2 0 a2
(a + b + c) 2
= 0 bc + ba – b 2 b 2 [Multiplying a in C1 and b in C2]
ab
–2ab –2ab 2ab

126 Xam idea Mathematics–XII

@Cbsebookshub - Join Us on Telegram


ab + ac a2 a2
(a + b + c) 2 2
= b bc + ba b 2 [C1 → C1 + C3 and C2 → C2 + C3]
ab
0 0 2ab
b+c a a
(a + b + c) 2 Taking a, b and 2ab common
=
ab
. ab . 2ab b +
c a b = G
from R1, R2 and R3 respectively
0 0 1
b+c a
= 2ab (a + b + c)2
b +
c a
= 2ab (a + b + c)2 {(b + c) (c + a) – ab}
= 2abc (a + b + c)3 = RHS
20. Using properties of determinant, show that: [CBSE (AI) 2012]
b+c a a
b c + a b = 4 abc
c c a+b
b+c a a
=
Sol. LHS b c+a b
c c a+b
2 (b + c) 2 (c + a) 2 (a + b)
= b c+a b [Applying R1 → R1 + R2 + R3]
c c +
a b
(b + c) (c + a) (a + b)
= 2 b c+a b [Taking 2 common from R1]
c c a+b
(b + c) (c + a) (a + b)
= 2 –c 0 – a [Applying R2 → R2 – R1 and R3 → R3 – R1]
–b –a 0
0 c b
= 2 – c 0 –a [Applying R1 → R1 + R2 + R3]
– b –a 0
Expanding along R1, we get
= 2 [0 – c (0 – ab) + b (ac – 0)] = 2 [abc + abc] = 4abc = RHS

21. Using properties of determinants, prove that [CBSE Ajmer 2015]


3
a 2 a
b3 2 b = 2(a – b) (b – c) (c – a) (a + b + c).
3
c 2 c
3
a 2 a
Sol. LHS = b 3 2 b
3
c 2 c
3
a 2 a
3 3
= b – a 0 b – a [Applying R2 → R2 – R1 and R3 → R3 – R1]
3 3
c –a 0 c–a

Determinants 127
@Cbsebookshub - Join Us on Telegram
a3 2 a
Taking common (b – a)
= (b – a) (c – a) b + a 2 + ab 2
0 1 > H
from R2 and (c – a) from R3
c 2 + a 2 + ac 0 1
a3 2 a
= (b – a) (c – a) a + b 2 + ab
2
0 1 [Applying R3 → R3 – R2]
c 2 – b 2 + ac – ab 0 0
Expanding along R3, we get
= (b – a)(c – a)(c2 – b2 + ac – ab)2 = 2 (b – a) (c – a) (c – b) (c + b + a)
= 2 (a – b) (b – c) (c – a) (a + b + c) = RHS
22. Using properties of determinant, prove that: [CBSE (F) 2012]
a a+b a+b+c
2a 3a + 2b 4a + 3b + 2c = a 3
3a 6a + 3b 10a + 6b + 3c
a a+b a+b+c
Sol. LHS = 2a 3a 2b 4a + 3b + 2c
+
3a 6a + 3b 10a + 6b + 3c
Applying R2 → R2 – 2R1 and R3 → R3 – 3R1, we get
a a+b a+b+c
= 0 a 2a + b
0 3a 7a + 3b
Expanding along C1, we get
= a[7a2 + 3ab – 6a2 – 3ab]
= a × a2 = a3 = RHS
23. Using properties of determinant, prove the following: [CBSE (F) 2015]
1 a a2
a2 1 a = (1 – a 3) 2
a a2 1
1 a a2
Sol. LHS = a 2 1 a
2
a a 1
2
1+ a + a a + 1 + a2 a2 + a + 1
= a2 1 a [Applying R1 → R1 + R2 + R3]
2
a a 1
1 1 1
= (1 + a + a2) a 2 1 a [Taking out (1 + a + a2) from first row]
a a2 1
0 1 1
2 2
= (1 + a + a ) a – 1 1 a [Applying C1 → C1 – C2]
a – a2 a 2
1
0 0 1
2 2
= (1 + a + a ) a – 1 1– a a [Applying C2 → C2 – C3 ]
a – a2 a –12
1

128 Xam idea Mathematics–XII

@Cbsebookshub - Join Us on Telegram


Expanding along R1, we have
= (1 + a + a2) [(a2 – 1)2 – a (1 – a)2] = (1 + a + a2) [(a + 1)2 (a – 1)2 – a (a – 1)2]
= (1 + a + a2) (a – 1)2 [a2 + 1 + a] = (1 + a + a2) (a – 1)2 [a2 + 1 + a]
= (a – 1)2 (1 + a + a2)2 = (1 – a)2 (1 + a + a2)2
= [(1 – a) (1 + a + a2)]2 = (1 – a3)2 = RHS
yz – x 2 zx – y 2 xy – z 2
24. Prove that zx – y 2 xy – z 2 yz – x 2 is divisible by (x + y + z), and hence find the quotient.
xy – z 2 yz – z 2 zx – y 2 [CBSE Delhi 2016]

yz – x 2 zx – y 2 xy – z 2
Sol. We have D = zx – y 2 xy – z 2 yz – x 2
xy – z 2 yz – x 2 zx – y 2
Applying C1 → C1 + C2 + C3 , we get

xy + yz + zx – x 2 – y 2 – z 2 zx – y 2 xy – z 2
= xy + yz + zx – x 2 – y 2 – z 2 xy – z 2 yz – x 2
xy + yz + zx – x 2 – y 2 – z 2 yz – x 2 zx – y 2
Taking (xy + yz + zx – x2 – y2 – z2) common from C1, we get

1 zx – y 2 xy – z 2
= (xy + yz + zx – x – y – z ) 1 xy – z 2 yz – x 2
2 2 2

1 yz – x 2 zx – y 2
Applying R2 → R2 – R1 and R3 → R3 – R1, we get

1 zx – y 2 xy – z 2
= (xy + yz + zx – x 2 – y 2 – z 2) 0 xy – z 2 – zx + y 2 yz – x 2 – xy + z 2
0 yz – x 2 – zx + y 2 zx – y 2 – xy + z 2

1 zx – y 2 xy – z 2
= (xy + yz + zx – x 2 – y 2 – z 2) 0 x (y – z) + (y 2 – z 2) y (z – x) + (z 2 – x 2)
0 z (y – x) + (y 2 – x 2) x (z – y) + (z 2 – y 2)

1 zx – y 2 xy – z 2
2 2 2
= (xy + yz + zx – x – y – z ) 0 (y – z) $ (x + y + z) (z – x) $ (x + y + z)
0 (y – x) $ (x + y + z) (z – y) $ (x + y + z)
Taking (x + y + z) common from R2 and R3, we get
1 zx – y 2 xy – z 2
2 2 2 2
= (xy + yz + zx – x – y – z ) (x + y + z) 0 y – z z – x
0 y–x z–y
= (xy + yz + zx – x2 – y2 – z2) (x + y + z)2 {1. (yz – y2 – z2 + zy – yz + xy + xz – x2)}
= (xy + yz + zx – x2 – y2 – z2)2 (x + y + z)2

yz – x 2 zx – y 2 xy – z 2
Hence, zx – y 2 xy – z 2 yz – x 2 is divisible by (x + y + z)
xy – z 2 yz – z 2 zx – y 2
and quotient is (xy + yz + zx – x2 – y2 – z2)2 (x + y + z).

Determinants 129
@Cbsebookshub - Join Us on Telegram
25. If a, b, c are real numbers, then prove that
a b c
b c a = – ^a + b + c h^a + b~ + c~2 h^a + b~2 + c~ h
c a b
where w is a complex number and cube root of unity. [HOTS]
a b c
Sol. LHS = b c a
c a b
a+b+c b c
= b + c + a c a [Applying C1 → C1 + C2 + C3]
c+a+b a b
1 b c
= + +
(a b c) 1 c a [Taking out (a + b + c) from C1]
1 a b
1 b c
= + +
(a b c) 0 c – b a – c [Applying R2 → R2 – R1 and R3 → R3 – R1]
0 a–b b–c
c–b a–c
= (a + b + c) [Expanding along C1]
a–b b–c
= (a + b + c) {– (b – c)2 – (a – c) (a – b)}

= – (a + b + c) (a2 + b2 + c2 – ab – bc – ca) and

RHS = – (a + b + c) (a + bw + cw2) (a + bw2 + cw)

= – (a + b + c) (a2 + abw2 + acw + abw + b2w3 + bcw2 + acw2 + bcw4 + c2w3)


= – (a + b + c) [(a2 + b2 + c2 + ab(w2 + w) + bc(w2 + w4) + ca (w + w2)] [ w3 = 1]

= – (a + b + c) (a2 + b2 + c2 – ab – bc – ca) = LHS [ w2 + w + 1 = 0 and w4 = w3 .w = w]

cos t t 1
f (t)
=
26. Let f (t) 2 sin t t 2t , then find lim 2 . [NCERT Exemplar, HOTS]
t"0 t
sin t t t
cos t t 1 cos t t 1
Sol. Given, f (t) = 2 sin t t 2t = 0 –t 0 [Applying R2 → R2 – 2R3]
sin t t t sin t t t
cos t 1 1
=t 0 –1 0
sin t 1 t
Expanding along R2, we get
t [(–1) (t cos t – sin t)] = – t2 cos t + t sin t
f ( t) –t 2 cos t + t sin t –t 2 cos t t sin t

` lim 2
= lim 2
= lim e + 2 o
t"0 t t"0 t t"0 t2 t

= lim c – cos t + m = –1 + lim


sin t sin t
= –1 + 1 = 0
t"0 t t"0 t

130 Xam idea Mathematics–XII

@Cbsebookshub - Join Us on Telegram


PROFICIENCY EXERCISE
QQ Objective Type Questions [1 mark each]
1. Choose and write the correct option in each of the following questions.
RS V
SS 1 1 1WW
W
(i) The maximum value of T = SSS 2 1 + sin i 1WW is (θ is real number)
W
SS1 + cos i 1 1WW
T X
1 3 2 3
(a) (b) (c) 2 (d)
2 2 4
52 53 54
(ii) The value of 5 3 5 4 5 5 is
5 4 5 5 56
(a) 0 (b) 52 (c) 59 (d) 513
RS V
SS 1 sin i 1 WW
W
(iii) Let A = SSS– sin i 1 sin iWW , when 0 # i # 2r. Then
W
SS –1 – sin i 1 WW
T X
(a) Det (A) = 0 (b) Det (A) ∈ (2, 3 ) (c) Det (A) ∈ (2, 4) (d) Det (A) ∈ [2, 4]

x 4
(iv) If = 0 , then the value x is
2 2x
(a) 0 (b) ! 2 (c) 2 (d) – 2
0 a–x x–b
(v) If f(x) = x + a 0 b–x , then
x+b x+c 0
(a) f (a) = 0 (b) f (b) = 0 (c) f (0) = 0 (d) f (1) = 0
sin (A + B + C) sin (A + C) cos C
(vi) If A + B + C= π, then the value of – sin B 0 tan A is equal to
cos (A + B) tan (B + C) 0
(a) 0 (b) 1 (c) 2 tan A sin B cos C (d) none of these
b 2 –ab b–c bc–ac
(vii) The determinant ab–a 2 a–b b 2 –ab equals

bc–ac c–a ab–a 2
(a) abc (b – c) (c – b) (a – b) (b) (b – c) (c – b) (a – b)
(c) (a + b + c) (b – c) (c – a) (a – b) (d) None of these
2. Fill in the blanks.
2x –9 –4 8
(i) If = , then value of x is _____________ . [CBSE (2020) 65/2/2]
–2 x 1 –2
0 xyz x–z
(ii) –x 0 y–z = _____________ .
y
z–x z–y 0
(iii) If A and B are square matrices of order 3 and |A|= 5, |B| = 3, then the value of |3AB|is
_____________ . [CBSE (2020) 65/5/3]
a + ib c + id
(iv) The value of = _____________ .
–c + id a–ib

Determinants 131
@Cbsebookshub - Join Us on Telegram
QQ Very Short Answer Questions: [1 mark each]
3. For what value of x, the following matrix is singular? [CBSE Delhi 2011]
5– x x+1
=0
2 4 2 3 4
4. Write the value of the following determinant: 5 6 8 [CBSE Delhi 2009]
6x 9x 12x
5. If Aij is the cofactor of the element aij of the determinant
2 –3 5
6 0 4 , then write the value of a32 . A32. [CBSE (AI) 2013]
1 5 –7
3x
7 8 7
6. If = , then find the value of x. [CBSE (AI) 2014]
–2
4 6 4
cos i sin i
7. If A = = G , then for any natural number n, find the value of det (An).
– sin i cos i
[CBSE Ajmer 2015]
If A is a square matrix of order 3 and|3A|= k|A|, then write the value of k. [CBSE Delhi 2010]
8.
9. If A = [aij] is a matrix of order 2 × 2 , such that |A| = –15 and Cij represents the cofactor of aij, then
find a21C21 + a22C22. [CBSE Sample Paper 2018]
1 –2
10. Find the cofactors of all the elements of . [CBSE 2020, (65/5/3)]
4 3
QQ Short Answer Questions–I and II: [2, 3 marks each]
a+x y z
11. Using the properties of determinant, evaluate x a+y z  [NCERT Exemplar]
x y a+z
a b c
12. Show that a + 2x b + 2y c + 2z = 0 , using properties of determinant.
x y z
13. Find the equation of line Joining (3, 1) and (9, 3) using determinant.
14. Using co-factors of elements of third column,
1 x yz
evaluate T = 1 y zx
1 z xy
Long Answer Questions:
QQ [5 marks each]
15. Using properties of determinant, solve for x:
a+x a– x a– x
a – x a + x a – x = 0 [CBSE (AI) 2011; (East) 2016]
a– x a– x a+x
16. In a triangle ABC, if
1 1 1
+
1 sin A +
1 sin B 1 + sin C =0
2 2 2
sin A + sin A sin B + sin B sin C + sin C
then prove that DABC is an isosceles triangle. [NCERT Exemplar, HOTS]
17. Using properties of determinant, prove the following:
x y z
2 2
x y z 2 = xyz (x – y) (y – z) (z – x) [CBSE Delhi 2011]
3 3 3
x y z

132 Xam idea Mathematics–XII

@Cbsebookshub - Join Us on Telegram


bc – a 2 ca – b 2 ab – c 2
18. Prove that ca – b 2 ab – c 2 bc – a 2 is divisible by (a + b + c) and find the quotient.
ab – c 2 bc – a 2 ca – b 2
a b c
19. If a ≠ b ≠ c and b c a = 0 , then using properties of determinants, prove that a + b + c = 0.
c a b [CBSE Chennai 2015]
20. Find the equation of the line joining A (1, 3) and B (0, 0) using determinants and find k if
D (k, 0) is a point such that the area of DABD is 3 sq units.
21. Using properties of determinants, prove the following:
1+x 1 1
1 1 + y 1 = xyz + xy + yz + zx [CBSE (AI) 2009]
1 1 1+ z
3x –x + y –x+z
22. Show that: x – y 3y z – y = 3 (x + y + z) (xy + yz + xz) [CBSE (AI) 2013]
x–z y–z 3z
23. Using properties of determinants, prove that :
1 1 1 + 3x
1 + 3y 1 1 = 9 (3xyz + xy + yz + zx) [CBSE Examination Paper 2018]
1 +
1 3z 1
a 2 + 2a 2a + 1 1
24. Using properties of determinant, prove that: 2a + 1 a + 2 1 = (a – 1) 3 [CBSE (AI) 2017]
a –1 0 3 3 1
25. If f (x) = ax a –1 , using properties of determinants, find the value of f(2x) – f(x).
ax 2 ax a [CBSE Delhi 2015]
x+2 x+6 x –1
26. Using the properties of determinants, solve the following for x : x + 6 x – 1 x + 2 = 0
x –1 x+2 x+6
[CBSE Panchkula 2015]
27. Using the properties of determinants, prove the following:

1 x x+1
2x x (x – 1) x (x + 1) = 6x 2 (1 – x 2) [CBSE Patna 2015]
3x (1 – x) x (x – 1) (x – 2) x (x + 1) (x – 1)
28. If x, y, z are in GP, then using properties of determinants, show that

px + y x y
py + z y z = 0, where x ≠ y ≠ z and p is any real number. [CBSE Sample Paper 2015]
0 px + y py + z
1 1+p 1+p+q
29. Using properties of determinants, prove that 3 4 + 3p 2 + 4p + 3q = 1 .[CBSE Sample Paper 2016]
4 7 + 4p 2 + 7p + 4q
0 2 –3
30. Without expanding the determinant at any stage, prove that –2 0 4 = 0 .
3 –4 0
[CBSE Sample Paper 2016]

Determinants 133
@Cbsebookshub - Join Us on Telegram
31. Using properties of determinants, prove that:
(b + c) 2 a 2 bc
(c + a) 2 b 2 ca = (a – b) (b – c) (c – a) (a + b + c) (a 2 + b 2 + c 2) [CBSE 2020 (65/1/1), (South) 2016]
(a + b) 2 c 2 ab
32. Using properties of determinants, prove the following:
x x+y x + 2y
x + 2y x x + y = 9y 2 (x + y) [CBSE (AI) 2013]
x+y x + 2y x
33. Using properties of determinants, prove that
b+c a a
b c+a b = 4abc [CBSE 2019 (65/3/3)]
c c a+b

Answers
1. (i) (a) (ii) (a) (iii) (d) (iv) (b) (v) (c) (vi) (a) (vii) (d)
2. (i) ! 3 (ii) (y–z) (z–x) (y–x + xyz) (iii) 405 (iv) a 2 + b 2 + c 2 + d 2

3. x = 3 4. 0 5. 110 6. x = –2 7. |An|= 1 8. k = 27
9. –15 10. Cofactors of all the elements of given matrix are as follows:
C11 = 3, C21 = 2
C12 = –4, C22 = 1 where Cij is the co-factors of ith row and jth co-factors
11. a 2 (a + x + y + z) 13. x – 3y = 0 14. (x - y) (y – z) (z – x) 15. x = 0, 3a
7
18. (3abc – a3 – b3 – c3) 20. 3x – y = 0; k = ± 2 25. ax (2a + 3x) 26. x = –
3

SELF-ASSESSMENT TEST
Time allowed: 1 hour Max. marks: 30
1. Choose and write the correct option in the following questions. (4 × 1 = 4)
cos x – sin x 1
(i) If x, y ∈R, then the determinant T = sin x cos x 1 lies in the interval
cos (x + y) – sin (x + y) 0

(a) 6– 2 , 2 @ (b) 6–1, 1@ (c) 6– 2 , 1@ (d) 6–1, – 2 @

sin 10° – cos 10°


(ii) The value of is
sin 80° cos 80°
1
(a) 0 (b) 1 (c) –1 (d)
2
(iii) The value(s) of k if area of triangle with vertices (–2, 0), (0, 4) and (0, k) is 4 sq. units is
(a) 0, 4 (b) – 8 (c) 0, 8 (d) 0 only
5–x x + 1
(iv) The value of x for which the matrix > H is singular, is
2 4
(a) 0 (b) 1 (c) 2 (d) 3

134 Xam idea Mathematics–XII

@Cbsebookshub - Join Us on Telegram


2. Fill in the blanks. (2 × 1 = 2)
(i) If A = = G and B = = G then the value of |A|+|B| is _____________ .
0 i 0 1
i 1 1 0
(ii) If A be a matrix of order 3 × 3 and |A|= 10, then the value of |4A|= _____________ .
Solve the following questions.
QQ (2 × 1 = 2)
3. If A = = G , then find the value of k if |2A| = k|A|.
1 2
4 2
x x 3 4
4. If = , then write the positive value of x.
1 x 1 2
QQ Solve the following questions. (4 × 2 = 8)
5. Using co-factors of elements of second row, evaluate
5 3 8
T= 2 0 1
1 2 3
6. Find the area of the triangle whose vertices area (3, 8), (–4, 2) and (5, 1).
1 –2
7. Find the cofactors of all the elements of .
4 3
8. Using co-factors of elements of third column, evaluate
1 x yz
T = 1 y zx
1 z xy
QQ Solve the following questions. (3 × 3 = 9)
a b c
9. Prove that: a 2
b 2
c 2 = ( a – b ) ( b – c) ^ c – a h ( a + b + c )
b+c c+a a+b
10. By using properties of determinant, prove the following:
x + m 2x 2x
= ^ 5x + m h^m – x h
2
2x +
x m 2x
2x 2x x+m
11. Using properties of determinants, prove the following:
1 1 1
a b c = (a – b) (b – c) (c – a) (a + b + c)
a3 b3 c3
QQ Solve the following question. (1 × 5 = 5)
12. If x, y, z are in GP, then using properties of determinants, show that
px + y x y
py + z y z = 0, where x ≠ y ≠ z and p is any real number. [CBSE Sample Paper 2015]
0 px + y py + z

Answers
1. (i) (a) (ii) (b) (iii) (c) (iv) (d) 2. (i) 0 (ii) 640
61
3. 4 4. –1, 2 5. 7 6. sq. units
2
7. Cofactors of all the elements of given matrix are as follows:
C11 = 3, C21 = 2, C12 = –4, C22 = 1 where Cij is the co-factors of ith row and jth co-factors
8. (x - y) (y – z) (z – x)
zzz

Determinants 135
@Cbsebookshub - Join Us on Telegram
5 Adjoint and
Inverse of a Matrix

1. Adjoint of a Matrix: If A = [aij] is a square matrix of order n and Cij denote the cofactor of aij in A, then
the transpose of the matrix of cofactors of elements of A is called the adjoint of A and is denoted by
adj A.
i.e., adj A = [Cij]T
R V RS V
SSSa11 a12 a13WWW SSC11 C21 C31WWW
If A = SSa21 a22 a23WW, then adj A = SSC12 C22 C32WW
SS W SS W
Sa31 a32 a33WW SC13 C23 C33WW
T X T X
2. The adjoint of a square matrix of order 2 can be obtained by interchanging the diagonal elements and
changing the signs of off-diagonal elements.

If A == G , then adj A == G.
a b d –b
c d –c a
3. If A is a square matrix of order n, then A (adj A) = |A|In = (adj A) A.
4. Following are some properties of adjoint of a square matrix:
If A and B are square matrices of the same order n, then
(i) adj (AB) = (adj B) (adj A) (ii) adj AT = (adj A)T
(iii) adj (adj A) =|A|n – 2A (iv) |adj A|= |A|n – 1
5. Invertible Matrix: A square matrix A of order n is invertible, if there exists a square matrix B of the
same order such that AB = In = BA.
In such a case, we say that the inverse of matrix A is B and we write A–1 = B.
Following are some properties of inverse of a matrix:
(i) Every invertible matrix possesses a unique inverse.
(ii) If A is an invertible matrix, then (A–1)–1 = A.
(iii) A square matrix is invertible, if it is non-singular.
1
(iv) If A is a non-singular matrix, then A–1 = (adj A).
|A |
(v) If A and B are two invertible matrices of the same order, then (AB)–1 = B–1 A–1.
(vi) If A is an invertible matrix, then (AT)–1 = (A–1)T.
(vii) The inverse of an invertible symmetric matrix is a symmetric matrix.
1
(viii) If A is a non-singular matrix, then |A–1| = .
|A |

136 Xam idea Mathematics–XII

@Cbsebookshub - Join Us on Telegram


6. Elementary Operation of Matrix: The following are three operations applied on the rows (columns)
of a matrix.
(i) Interchange of any two rows (columns) Ri ↔ Rj (Ci ↔ Cj).
(ii) Multiplying all elements of a row (column) of a matrix by a non-zero scalar Ri → kRi (Ci → kCj).
(iii) Adding to the elements of a row (column), the corresponding elements of another row (column)
multiplied by any scalar Ri → Ri + kRj (Ci → Ci + kCj).
7. A matrix obtained from an identity matrix by a single elementary operation is called an elementary
matrix.
8. Elementary Row Operations: To find the inverse of an invertible matrix ‘A’ by elementary row
operations, at first we make an equation as
A = IA …(i)
On this equation (i) we apply sequence of row operations successively on ‘A’ on the LHS and on ‘I’
on the RHS till we get I = BA like as
A = IA

Sequence of elementary row operations gives

I = BA
B is called inverse of A i.e., A = B
–1

Elementary Column Operations: To find the inverse of A by using elementary column operations

we write an equation as
A = AI

Sequence of elementary column operations gives

I = AB
B is called inverse of A i.e., A = B
–1

9. System of Linear Equations: A system of n simultaneous linear equations in n unknowns


x1, x2, x3, ... , xn is
a11x1 + a12x2 + ... + a1nxn = b1
a21x1 + a22x2 + ... + a2nxn = b2
... ... ... ... ...
... ... ... ... ...
an1x1 + an2x2 + ... + annxn = bn
This system of equations can be written in matrix form as
RS VR V R V
SSa11 a12 g a1nWWWSSSx1WWW SSSb1WWW
SSa a g a WWSSx WW SSb WW
SS 21 22 2nWS 2W S 2W
=
SS h h h WWSS h WW SS h WW
W
SSa a g a WWSSx WW SSSb WWW
S W
n1 n2 nn n n
T XT X T X
or, AX = B

Adjoint and Inverse of a Matrix 137


@Cbsebookshub - Join Us on Telegram
R V RS x VW RSb VW
SSSa11 a12 ... a1nWWW SS 1 WW SS 1WW
SSa a ... a WW SSx2 WW SSb WW
2nW
where A = SS h21 h22 W, X = SS WW and B = SS 2WW
SS h W SS h WW SS h WW
SSa a ... a WWW SSx WW SSb WW
n1 n2 nn n n
T X T X T X
10. A set of values of the variable x1, x2, ..., xn satisfying all the equations simultaneously is called a
solution of the system.
11. If a system of equations has one or more solutions, then it is said to be a consistent system of
equations, otherwise it is an inconsistent system of equations.
12. A system of equations AX = B is called a homogeneous system, if B = O. Otherwise, it is called a
non-homogeneous system of equations.
13. System of linear equations may or may not be consistent, if consistent may or may not have unique
solution.
It can be, decided by following flow chart:
At first system of linear equations is written in matrix form as AX = B. Then |A|.adj A is obtained
|A|

|A| 0 |A|= 0

System is consistent and


has unique solution given by
X = A–1B
(adj A).B = 0 (adj A).B 0
System of equations have System of equations
is inconsistent.

14. A homogeneous system of n linear equations in n unknowns is expressible in the form AX = O.


If |A|≠ 0, then AX = O has unique solution X = 0 i.e., x1 = x2 = ... = xn = 0. This solution is called the
trivial solution.
If |A| = 0, then AX = O has infinitely many solutions.
Thus, a homogeneous system of equation is always consistent.
15. If A is a square matrix of order n, then |adj A| = |A|n–1.

Selected NCERT Questions

1. Let A = = G and B = = G , verify that (AB)–1 = B–1A–1.


3 7 6 8

2 5 7 9
3 7 6 8 67 87
Sol. AB = > H> H=> H
2 5 7 9 47 61
|AB| = 67 × 61 – 47 × 87 = 4087 – 4089 = –2
∴ AB is invertible.

Cofactors of elements of determinant AB are,
AB11 = 61, AB12 = – 47, AB21 = – 87, AB22 = 67
T
61 –47 61 –87
Adj (AB) = > H => H
–87 67 –47 67
1 1 61 –87 1 –61 87
(AB) –1 = Adj (AB) = > H= > H
AB –2 –47 67 2 47 –67

138 Xam idea Mathematics–XII

@Cbsebookshub - Join Us on Telegram


3 7
| A | = = 15 – 14 = 1
2 5
Cofactors of elements of A are
A11 = 5, A12 = –2, A21 = –7, A22 = 3
T
5 –2 5 –7
Adj A = > H => H
–7 3 –2 3
1 1 5 –7 5 –7
A –1 = Adj A = > H=> H
|A | 1 –2 3 –2 3
6 8
| B | = = 54 – 56 = –2
7 9
Cofactors of elements of B are
B11 = 9, B12 = –7, B21 = – 8, B22 = 6
T
9 –7 9 –8
Adj B = > H => H
–8 6 –7 6
1 1 9 –8 1 –9 8
B –1 = Adj A = > H= > H
|B | –2 –7 6 2 7 –6

1 –9 8 5 –7 1 –45 – 16 63 + 24 1 –61 87
B –1 A –1 = > H> H = > H= > H
2 7 –6 –2 3 2 35 + 12 –49 – 18 2 47 –67

Thus, (AB)–1 = B–1A–1.


RS V
SS 2 –1 1 WWW
2. If A = SS–1 2 –1WW , then verify that A3 – 6A2 + 9A – 4I = 0 and hence find A–1 . [HOTS]
SS W
S 1 –1 2 WW
T X
RS VW RS V
SS 2 – 1 1 WW SS 2 –1 1 WWW
Sol. A = SSS–1 2 –1WWW . SSS–1 2 –1WWW
2
[a A 2 = A.A]
SS 1 –1 2 WW SS 1 –1 2 WW
T XT X
RS V R V
SS 4 + 1 + 1 – 2 – 2 – 1 2 + 1 + 2 WW SS 6 –5 5 WW
W S W
= SS–2 –2 – 1 1 + 4 + 1 –1 – 2 – 2WW = SS–5 6 –5WW
SS WW SS W
S 2 + 1 + 2 –1 – 2 – 2 1 + 1 + 4 W S 5 –5 6 WW
T X T X
RS VW RS VW
SS 2 –1 1 WW SS 6 –5 5 WW
A 3 = SSS–1 2 –1WWW . SSS–5 6 –5WWW [a A 3 = A.A 2]
SS 1 –1 2 WW SS 5 –5 6 WW
T XT X
SRS 12 + 5 + 5 –10 – 6 – 5 10 + 5 + 6 WVW RSS 22 – 21 21 VWW
S W S W
= SS– 6 –10 – 5 5 + 12 + 5 – 5 –10 – 6WW =SS–21 22 – 21WW
SS WW SS W
S 6 + 5 + 10 – 5– 6 –10 5 + 5 + 12 W S 21 –21 22 WW
T X T X
RS VW RS VW RS V RS V
SS 22 – 21 21 WW SS 6 – 5 5 WW SS 2 –1 1 WWW SS1 0 0WWW
Now, A – 6A + 9A – 4I = SS–21 22 –21WW – 6 SS–5 6 –5WW + 9 SS–1 2 –1WW – 4 SS0 1 0WW
3 2
SS W SS W S W SS W
S 21 –21 22 WW S 5 –5 6 WW SS 1 –1 2 WW S0 0 1WW
T X T X T X T X
R V
SSS0 0 0WWW
=SS0 0 0WW = 0 (zero matrix)
SS W
S0 0 0WW
T X
∴ A3 – 6A2 + 9A – 4 I = 0

Adjoint and Inverse of a Matrix 139


@Cbsebookshub - Join Us on Telegram
Pre-multiplying by A–1, we get
A–1A.A2 – 6A–1.A.A + 9A–1.A – 4A–1I = 0
⇒ A2 – 6A + 9I – 4A–1 = 0 ⇒ 4A–1 = A2 – 6A + 9I
R V SRS 2 –1 1 WVW SSR1 0 0WVW
SSS 6 –5 5 WWW S W S W
4A = SS–5 6 –5WW – 6 SS–1 2 –1WW + 9 SS0 1 0WW
–1
SS WW SS WW SS W
S 5 –5 6 W S 1 –1 2 W S0 0 1WW
TR X V TR X T V X
S 3 1 –1WW SS 3/4 1/4 –1/4WW
1 SSS W S W
A = SS 1 3 1 WWW = SSS 1/4 3/4 1/4 WWW
–1
4S
S–1 1 3 WW SS–1/4 1/4 3/4 WW
T X T X
SRS2 –3 5WVW
S W
3. If A = SSS3 2 –4WW , find A–1. Using A–1 solve the system of equations.
W
SS1 1 –2WW
T X
2x – 3y + 5z = 11; 3x + 2y – 4z = –5; x + y – 2z = –3.
2 –3 5
Sol. | A | = 3 2 –4 = 2(–4 + 4) + 3(–6 + 4) + 5(3 – 2) = 0 – 6 + 5 = –1 ≠ 0
1 1 –2
Therefore A is non-singular matrix and A–1 exists.
C11 = 0; C12 = 2; C13 = 1; C21 = –1; C22 = –9;
C23 = –5; C31 = 2; C32 = 23; C33 = 13
SSR 0 2 1WVWT SRS0 –1 2WVW
S W S W
adj A = SSS–1 –9 –5WWW = SSS2 –9 23WWW
SS 2 23 13WW SS1 –5 13WW
T X TR X
SS0 –1 2VWW RSS 0 1 –2VWW
1 1 SS W S W
∴ A –1 = adj A = S2 –9 23WWW = SSS–2 9 –23WWW
|A | –1 SS
S1 –5 13WW SS–1 5 –13WW
T X T X
The given system can be expressed as AX = B, where
RS V RS VW R V
SS2 –3 5WWW SSxWW SSS11WWW
A = SSS3 2 –4WWW , X = SSSyWWW , B = SSS–5WWW
SS1 1 –2WW SSzWW SS–3WW
T X T X T X
Now, AX = B ⇒ X = A–1B
RS VW RS VR V RS VW RS V R V
SSxWW SS 0 1 –2WWW SSS11WWW SSxWW SS 0 – 5 + 6 WWW SSS1WWW
⇒ SSSyWWW = SSS–2 9 –23WWW SSS–5WWW ⇒ SSyWW = SS–22 – 45 + 69WW = SS2WW
SS WW SS W S W
SSzWW SS–1 5 –13WW SS–3WW SzW S–11 – 25 + 39WW SS3WW
T X T XT X T X T X T X
On equating, we get x = 1, y = 2 and z = 3.

Multiple Choice Questions [1 mark]


Choose and write the correct option in the following questions.
RS V
SS–2 0 0 WWW
1. If A = SS 0 –2 0 WW , then the value of |adj A| is [CBSE 2020 (65/3/1)]
SS W
S 0 0 –2WW
T X
(a) 64 (b) 16 (c) 0 (d) – 8

140 Xam idea Mathematics–XII

@Cbsebookshub - Join Us on Telegram


p q
2. The adjoint of matrix A = [aij] = > H is
r s
s –q s q 0 0
(a) > H (b) > H (c) > H (d) None of these
–r p r –p 0 q
3. If A is square matrix of order 3 × 3 such that |A| = 2, then write the value of |adj (adj A)|.
(a) –16 (b) 16 (c) 0 (d) 2
4. If A is a square matrix of order 3, such that A(adj A) = 10 I, then |adj A| is equal to
[CBSE 2020 (65/5/1)]
(a) 1 (b) 10 (c) 100 (d) 101

5. If B. = G== G then matrix B is


1 –2 6 0
1 4 0 6
4 2 4 2 0 0
(a) > H (b) > H (c) > H (d) I
1 1 –1 1 0 0

6. Let A be a 3 × 3 square matrix such that A (adj A) = 2I, where I is the identity matrix. The value
of |adj A| is
(a) 4 (b) –4 (c) 0 (d) None of these
RS V
SS 5 10 3WWW
7. The matrix SSS–2 –4 6WWW is singular matrix, if the value of b is
SS–1 –2 bWW
T X
(a) –3 (b) 3 (c) 0 (d) Any real number
RS V
SSx 0 0WWW
8. If x, y, z are non-zero real numbers, then the inverse of matrix A = SS0 y 0WW is
SS W
S0 0 zWW
RS –1 V RS –1 V RS V T X R
SSx 0 0 WWW SSx 0 0 WWW SSx 0 0WWW SS1 0 0WVW
1 1 S W
(a) SSS 0 y –1 0 WWW (b) xyz SSS 0 y –1 0 WWW (c) xyz SS0 y 0WW (d) xyz SS0 1 0WW
SS W SS W S W S W
0 0 z –1W 0 0 z –1W SS0 0 zWW SS0 0 1WW
T X T X T X T X
RS VW
SS 2 m – 3 WW
9. If A = SS0 2 5 WW , then A–1 exists if [NCERT Exemplar]
SS W
S1 1 3 WW
T X
(a) l = 2 (b) l ≠ 2 (c) l ≠ – 2 (d) None of these
10. If A2 – A + I = 0, then the inverse of A is
(a) A–2 (b) I – A (c) 0 (d) A

11. If A = = G then the value of AT A–1 is


1 tan x
– tan x 1
cos 2x – sin 2x cos x sin x
(a) > H (b) > H (c) A′ (d) Zero matrix
sin 2x cos 2x 1 0
12. The value of (A–1)T is
(a) (AT)–1 (b) A–1 (c) I (d) AT
13. If A is an invertible matrix of order 2, then det(A–1) is equal to
1
(a) det A (b) (c) 1 (d) 0
det A

Adjoint and Inverse of a Matrix 141


@Cbsebookshub - Join Us on Telegram
14. If A is a singular matrix, then A(adj A) is
(a) Null matrix (b) Scalar matrix (c) Identity matrix (d) None of these
15. If A is a matrix of order 3 × 3, then the value of |3A| is
(a) 27|A| (b) –27|A| (c) 9|A| (d) None of these
16. The sum of the products of elements of any row with the co-factors of corresponding elements
is equal to
(a) cofactors sum (b) value of the determinant
(c) 0 (d) Adjoint of matrix
17. Using matrix method to solve the following system of equations: 5x – 7y = 2, 7x – 5y = 3, value
of x, y is
11 1 –11 1 10 1
(a) x = ,y= (b) x = ,y= (c) x = 1, y = 1 (d) x = ,y=
24 24 24 24 24 24
18. If the value of a third order determinant is 12, then the value of the determinant formed by
replacing each element by its cofactor will be
(a) 12 (b) 144 (c) –12 (d) 13
RS V
SS2 m –3WW
WW
19. If A = SSS0 2 5W , then A–1 exist if
W
SS1 1 3WW
T X
(a) l = 2 (b) l ≠ 2 (c) l ≠ –2 (d) None of these
20. If A and B are invertible matrices, then which of the following is not correct.
(a) adj A = |A|. A–1 (b) det (A–1) = [det (A)]–1
(c) (AB)–1 = B–1A–1 (d) (A + B)–1 = B–1 + A–1

Answers
1. (a) 2. (a) 3. (b) 4. (c) 5. (b) 6. (a)
7. (d) 8. (a) 9. (d) 10. (b) 11. (a) 12. (a)
13. (b) 14. (a) 15. (a) 16. (b) 17. (a) 18. (b)
19. (d) 20. (d)

Solutions of Selected Multiple Choice Questions


1. We have a matrix A of order 3 × 3 given by
RS V
SS–2 0 0 WWW
A = SS 0 –2 0 WW ⇒ |A| = – 2 ((–2) × (–2) – 0)
SS W
S 0 0 –2WW
T X
⇒ |A| = – 8
∴ |adj A| = |A|n – 1 = |A|3 – 1 = (–8)2 = 64

3. We have,
(n – 1) 2 2
|adj (adj A)| = A = 2(3 – 1) (where n is order of matrix A)
4
= 2 = 16
4. We have,
A (adj A) = |A| I
⇒ 10 I = |A| I  ⇒ |A| = 10

142 Xam idea Mathematics–XII

@Cbsebookshub - Join Us on Telegram


∴ |adj A| = |A|n – 1 = |A|3 – 1 ( n = 3)

= |A|2 = (10)2 = 100

5. If B = = G then
a b
c d
a + b –2a + 4b
= G>
a b 1 –2 6 0 6 0
H=> H ⇒  > H=> H
c d 1 4 0 6 c + d –2c + 4d 0 6

∴ On comparing , we get

a + b = 6 ⇒ a = 6 –b ...(i)
– 2a + 4b = 0
Using (i), we get
– 2(6 – b) + 4b = 0
⇒ – 12 + 2b + 4b = 0 ⇒ 6b = 12
⇒ b=2
Using in (i), we get a = 4
c + d = 0 ⇒ c = – d ...(ii)
And –2c + 4d = 6
–2(– d) + 4d = 6 ⇒ 6d = 6 ⇒ d = 1
Put in (ii), we get
c = –1
4 2
∴ B = > H
–1 1

13. AA–1 = I
⇒ |AA–1| = |I| ⇒ |A|.|A–1| = 1
1
⇒ A –1 = [ a | A | ! 0]
A
1
=
|A |
14. A (adj A) = |A| I (Using properties of adjoint of matrix)
As it is singular
∴ |A| = 0
⇒ 0×I=0
So, it is null matrix.

Fill in the Blanks [1 mark]


2 –1
1. If A is a matrix of order 3 × 3, then (A ) = _____________ .
2. The value of k for which the system of linear equations:
x+y+z=2
2x + y – z = 3
3x + 2y + kz = 4 has a unique solution, is _____________ .

3. If A is a non-singular matrix such that A –1 = = G , then (A′)–1 = _____________ .


2 3

1 2
RS V
SS1 0 3WWW
4. If A = SS2 1 1WW , then |adj (adj A)|= _____________ .
SS W
S0 0 2WW
T X
Adjoint and Inverse of a Matrix 143
@Cbsebookshub - Join Us on Telegram
Answers
3. = G
2 1
1. (A–1)2 2. k ≠ 0 4. 16
3 2

Solutions of Selected Fill in the Blanks


4. We have,
1 0 3
A = 2 1 1 = 2
0 0 2
(n – 1) 2 ( 3 – 1) 2 4
∴ adj (adj A) = A
= A = A
  = (2)4 = 16

Very Short Answer Questions [1 mark]


1. Find adj A, if A = = G [CBSE 2020 (65/4/1)]
2 –1

4 3
Sol. We have,

A = = G
2 –1
4 3
∴ Co-factors of matrix A are

C11 = 3, C12 = – 4
C21 = 1, C22 = 2

∴ adj(A) = = G
3 1

–4 2
2. If A is a square matrix of order 3, with |A| = 9, then write the value of |2 . adj A|.
[CBSE 2019 (65/4/1)]
Sol.  |2 . adj A| = 23|adj A| [ For any matrix A of order n,
|kA| = kn|A|, where k is some constant]
= 8 |adj A|= 8 |A|3 – 1 [ |adj A|=|A|n–1, for any square matrix of order n]
= 8 × (9)2 = 8 × 81 = 648
3. If A is a square matrix of order 2 and |A| = 4, then find the value of |2AAT|, where AT is
transpose of the matrix A. [CBSE 2019 (65/5/1)]
Sol. We have |A|= 4 and A is a matrix of order 2.
∴ |2AAT| = 22|AAT|
[ A is a matrix of order 2]
T T
= 4|AA | = 4|A||A |
For any square matrix A, |AT| = |A|
∴ |2AAT| = 4 × 4 × 4 = 64

A is a square matrix with |A|= 4. Then find the value of |A.(adj A)| [CBSE 2019 (65/4/3)]
4.

Sol.  A. (adj A) = A In = A = 4 [ A. adj A = |A|In]

5. If A = = G then find |adj A|. [CBSE 2019 (65/5/3)]


8 2

3 2
8 2
Sol. We have A => H
3 2

144 Xam idea Mathematics–XII

@Cbsebookshub - Join Us on Telegram



|A| = 16 – 6 = 10
Now, |adj A| = |A|2 – 1 = ]10g = 10
2–1

6. If A is a matrix of order 3 × 3, then find (A2)–1. [NCERT Exemplar]


Sol. (A2)–1 = (A.A)–1
= A–1 . A–1 = (A–1)2 [ (AB)–1 = B–1 A–1]

Short Answer Questions-I [2 marks]

1. Write A–1 for A = = G . [CBSE Delhi 2010, 2011]


2 5

1 3
Sol. For elementary row operations, we write

= G = = G.A
2 5 1 0
A = IA & 1 3 0 1

= G = = G . A [Applying R1 ↔ R2]
1 3 0 1


2 5 1 0

= G== G A [Applying R2 → R2 – 2R1]


1 3 0 1


0 –1 1 –2

= G== G A [Applying R1 → R1 + 3R2]


1 0 3 –5


0 –1 1 –2

= G== G A [Applying R2 → (–1)R2]


1 0 3 –5


0 1 –1 2

I== GA A –1 == G
3 –5 3 –5


–1 2
& –1 2
[Note : B is called inverse of A, if AB = BA = I]

2. If A = = G , write A–1 in terms of A. [CBSE (AI) 2011]


2 3

5 –2
2 3
Sol. | A | = = – 4 – 15 = – 19 ≠ 0
5 –2
Now, C11 = – 2, C12 = – 5, C21 = – 3 and C22 = 2
T
adj A = = G == G
–2 –5 –2 –3
–3 2 –5 2

= G= = G=
1 1 –2 –3 1 2 3 1
A –1 = adj A = A
|A | –19 –5 2 19 5 –2 19

3. If A is invertible matrix of 3 × 3 and |A| = 7, then find |A–1|. [NCERT Exemplar]


1
Sol.  A –1 = . adj A
|A |
1 3
adj A = d
1
| A –1 | = n . | adj A | [ |KA|= Kn.|A|, where n is order of A]
|A | |A |
1 1 1
= .|A|3–1 = = .
|A | 3 |A | 7

Adjoint and Inverse of a Matrix 145


@Cbsebookshub - Join Us on Telegram
RS V
SS2 m –3WWW
4. If A = SSS0 2 5 WWW , then find the value of l for which A–1 exists. [NCERT Exemplar]
SS W
1 1 3W
T X
Sol. For existence of A–1
RS V
SS2 m –3WWW
| A | ! 0 & SSS0 2 5 WWW ! 0
S1 1 3 W
S W
T X
⇒ 2 (6 – 5) – l(0 – 5) + (–3)(0 – 2) ≠ 0 [Expanding along R1]
8
⇒ 2 + 5l + 6 ≠ 0 ⇒ 5l ≠ – 8 ⇒ m ! –
5
8
Hence, l can have any value other than – .
5

Short Answer Questions-II [3 marks]


1. Using elementary transformations, find the inverse of the matrix
RS V
SS 1 3 –2WWW
SS–3 0 –1WW [CBSE Delhi 2011]
SS W
S 2 1 0WW
RS T V X
SS 1 3 –2WWW
Sol. Let A = SSS–3 0 –1WWW
SS 2 1 0WW
T X
For finding the inverse, using elementary row operation we write
SRS 1 3 –2WVW SRS1 0 0WVW
SS W S W
A = IA ⇒ SS–3 0 –1WWW = SSS0 1 0WWW A
SS 2 1 0WW SS0 0 1WW
T X T X
Applying R2 → R2 + 3R1 and R3 → R3 – 2R1, we get
RS V R V
SS1 3 –2WW SS 1 0 0WW
SS0 W S W
⇒ 9 –7WW = SS 3 1 0WW A
SS WW SS W
S0 –5 4W S–2 0 1WW
RS T XV TR X V
SS1 0 1/3WWW SSS 0 –1/3 0WWW
SS0 9 1
⇒ –7 WW = SS 3 1 0WW A [Applying R1 " R1 – R2 ]
SS WW SS WW 3
S0 –5 4 W S–2 0 1W
T XV TR XV
SRS1 0 1/3WW SS 0 –1/3 0WW
S W S W 1
⇒ SSS0 1 –7/9WW = SS1/3 1/9 0WW A [Applying R2 " R2 ]
W S W
SS0 –5 9
4WW SS –2 0 1WW
TR VX RT XV
SS1 0 1/3WW SS 0 –1/3 0WW
SS W S W
⇒ SS0 1 –7/9WWW = SSS 1/3 1/9 0WW A [Applying R3 → R3 + 5R2]
W
SS0 0 1/9WW SS–1/3 5/9 1WW
TR VX RT VX
SS1 0 0 WW SS 1 –2 –3WW
SS W S W
⇒ SS0 1 0 WW = SS –2 4 7 WW A [Applying R1 → R1 – 3R3, R2 → R2 + 7R3]
SS0 0 1/9WW SS–1/3 5/9 1 WWW
W S
T R X T X
SS1 0 0VWW RSS 1 –2 –3VWW
SS W S W
⇒ SS0 1 0WWW = SSS–2 4 7WW A
W [Applying R3 → 9R3 ]
SS0 0 1WW SS–3 5 9WW
T X T X

146 Xam idea Mathematics–XII

@Cbsebookshub - Join Us on Telegram


RS V
SS 1 –2 –3WWW
⇒ I = SSS–2 4 7WW A
W
SS–3 5 9WW
TR X
SS 1 –2 –3VWW
S W
Hence, A –1 = SSS–2 4 7WW
W
SS–3 5 9WW
T X
SRS 1 –2 3WVW
S W
2. If A = SSS 0 –1 4WWW , find (A′)–1. [CBSE Delhi 2015]
SS–2 2 1WW
T X
RS V RS V
SS 1 –2 3WWW SS 1 0 –2WW
W
Sol. Given A = SSS 0 –1 4WWW and Al = SSS–2 –1 2WW
W
SS–2 2 1WW SS 3 4 1WW
T X T X
|A′| = 1(–1 – 8) – 0 – 2(–8 + 3) = –9 + 10 = 1 ≠ 0
Hence, (A′)–1 will exist.
–1 2 –2 2
A11 = = –1 – 8 = –9; A12 = – = – (–2 – 6) = 8
4 1 3 1
–2 –1 0 –2
A13 = = – 8 + 3 = – 5; A21 = – = – (0 + 8) = –8
3 4 4 1
1 –2 1 0
A22 = = 1 + 6 = 7; A23 = – = – (4 – 0) = –4
3 1 3 4
0 –2 1 –2
A31 = = 0 – 2 = –2; A32 = – = – (2 – 4 ) = 2
–1 2 –2 2
1 0
A33 = = –1 – 0 = –1
–2 –1
RS VT R V
SS–9 8 –5WWW SSS–9 –8 –2WWW
adj (Al ) = SSS–8 7 –4WWW = SSS 8 7 2 WW
SS–2 2 –1WW SS–5 –4 –1WWW
T R X T X
SS–9 –8 –2VWW RSS–9 –8 –2VWW
1S W S W
(Al ) –1 = SSS 8 7 2 WW = SS 8 7
W S
2 WW
W
1S
S–5 –4 –1WW SS–5 –4 –1WW
T X T X
SRS0 2y zWWV
S W
3. Find x, y and z if A = SSSx y –zWWW satisfies A' = A–1. [NCERT Exemplar]
SS W
x –y zW
T X
R V RS V
SSS0 2y zWWW SS 0 x xWWW
Sol. We have, A = SSSx y –zWWW and A' = SSS2y y –yWWW
SS W SS z –z zWW
x –y zW
T X T X
Also, A' = A–1

AA' = AA–1 [ AA–1 = I]
RS VR V R V
SS0 2y zWWW SSS 0 x xWWW SSS1 0 0WWW

SSx y –zWW SS2y y –yWW = SS0 1 0WW
SS WS W S W
Sx –y zWW SS z –z zWW SS0 0 1WW
T XT X T X

Adjoint and Inverse of a Matrix 147


@Cbsebookshub - Join Us on Telegram
RS 2 V
SS 4y + z 2 2y 2 – z 2 –2y 2 + z 2 WWW SSR1 0 0VWW
S W S W
⇒ SS 2y 2 – z 2 x 2 + y 2 + z 2 x 2 – y 2 – z 2WW = SSS0 1 0WWW
SS WW S W
SS–2y 2 + z 2 x 2 – y 2 – z 2 x 2 + y 2 + z 2 WW S0 0 1W
T X T X
By comparing the corresponding elements, we get
⇒ 2y2 – z2 = 0
⇒ 2y2 = z2
⇒ 4y2 + z2 = 1

1
⇒ 2.z2 + z2 = 1
⇒ z =!
3
z2 1

y2 = ⇒ y =!
2 6
Also, x2 + y2 + z2 = 1
1 1 3 1 1
⇒ x2 = 1 – y2 – z2 = 1 –
– = 1– = ⇒ x =!
6 3 6 2 2
1 1 1

x =! , y =! and z = !
2 6 3
4. On her birthday Seema decided to donate some money to children of an orphanage home. If there
were 8 children less, every one would have got ` 10 more. However, if there were 16 children more,
every one would have got ` 10 less. Using matrix method, find the number of children and the
amount distributed by Seema. [CBSE (East) 2016]
Sol. Let the number of children be x and the amount donated by Seema to each child be ` y.
∴ According to question,

(x – 8)(y + 10) = xy and (x + 16)(y – 10) = xy
⇒ xy + 10x – 8y – 80 = xy and xy – 10x + 16y – 160 = xy

⇒ 5x – 4y = 40
...(i)
and 5x – 8y = – 80 ...(ii)
Equation (i) and (ii) may be written in matrix form as
AX = B ⇒ X = A–1B ...(iii)

G, B = > H , X = = G
x
Where A = =
5 –4 40
5 –8 –80 y

Now, |A| = – 40 + 20 = – 20
T
Adj A = = G == G
–8 –5 –8 4
4 5 –5 5

= G
1 –8 4

A –1 = –
20 –5 5
Putting value of A–1 in (iii), we get
x x
= G = – = G= G = G=– = G
1 –8 4 40 1 –320 –320
y 20 –5 5 –80
& y 20 –200 –400
x x
= G=– = G = G== G
1 –640 32


y 20 –600
& y 30
⇒ x = 32, y = 30

⇒ Number of children = 32 and amount donated to each child = ` 30.

148 Xam idea Mathematics–XII

@Cbsebookshub - Join Us on Telegram


5. A trust invested some money in two type of bonds. The first bond pays 10% interest and
second bond pays 12% interest. The trust received ` 2,800 as interest. However, if trust had
interchanged money in bonds, they would have got ` 100 less as interest. Using matrix method,
find the amount invested by the trust. [CBSE (Central) 2016]
Sol. Let ` x and ` y be the amount of money invested in 1st and 2nd bond respectively.
According to information given in question
10 12
x × +y× = 2800
100 100
12 10
x × +y× = 2700
100 100
⇒ 10x + 12y = 280000
…(i)
and 12x + 10y = 270000 …(ii)
Above system of equation can be written in matrix form as
AX = B, where

G, X = = G, B = >
x
A = =
10 12 280000
H
12 10 y 270000
AX = B ⇒ X = A–1B …(iii)
10 12
Now, A = = 100 – 144 = –44
12 10
T
.= G = = G
1 1 10 –12 1 10 –12
A –1 = . adj A =
A A –12 10 –44 –12 10
Putting the value of X, A–1 and B in (iii), we get

= G = = G = – < 2800000 – 3240000 F


x x
= G>
1 10 –12 280000 1
H ⇒
y –44 –12 10 270000 y 44 –3360000 + 2700000

= G=– > = G=>


x 1 –440000 x 10000

H & H
y 44 –660000 y 15000
⇒ x = 10000, y = 15000.
Hence, invested amount in 1st and 2nd bonds are `10000 and `15000.

Long Answer Questions [5 marks]


1. Using elementary row transformations, find the inverse of the matrix
RS V
SS2 –3 5 WWW
SS3 2 –4WW [CBSE 2019 (65/4/1)]
SS W
S1 1 –2WW
TR X
SS2 –3 5 WVW
S W
Sol. A = SS3 2 –4WW
SS W
S1 1 –2WW
T X
We know that A = IA
RS V R V
SS2 –3 5 WWW SSS1 0 0WWW
i.e., SS3 2 –4WW = SS0 1 0WW A
SS W S W
S1 1 –2WW SS0 0 1WW
T X T X
Applying R1 → R1 – R3, we have

Adjoint and Inverse of a Matrix 149


@Cbsebookshub - Join Us on Telegram
RS V R V
SS1 – 4 7 WWW SSS0 0 – 1WWW
⇒ SS3 2 – 4WW = SS0 1 0 WW A
SS W S W
S1 1 – 2WW SS0 0 1 WW
T X T X
Apply R2 → R2 – 3R1 and R3 → R3 – R1, we have
SRS1 – 4 V R
7 WW SS 1 0 – 1WW
V
SS WW SS W
⇒ S0 14 – 25W = S– 3 1 3 WW A
SS WW SS W
S0 5 – 9 W S– 1 0 2 WW
T X T X
Apply R2 → – (R2 – 3R2), we have
SRS1 – 4 7 WVW SRS 1 0 –1WVW
S W S W
⇒ SS0 1 –2WW = SS 0 – 1 3 WW A
SS WW SS W
S0 5 –9W S– 1 0 2 WW
T X T X
Apply R1 → R1 + 4R2 and R3 → R3 – 5R2, we have
RS V R V
SS1 0 –1WWW SSS 1 –4 11 WWW
⇒ SS0 1 –2WW = SS 0 –1 3 WW A
SS W S W
S0 0 1 WW SS–1 5 –13WW
T X T X
Apply R1 → R1 + R3 and R2 → R2 + 2R3, we have
SRS1 0 0WVW SRS 0 1 – 2 WVW
S W S W
⇒ SS0 1 0WW = SS–2 9 – 23WW A
SS WW SS W
S0 0 1W S–1 5 – 13WW
T X T X
RS V
SS 0 1 – 2 WW
W
∴ A = SS– 2 9 – 23WW –1
SS W
S– 1 5 – 13WW
R T V X R
SSS 3 –1 1WWW SS 1 2 –2WVW
S W
2. If A = SSS–15 6 –5WWW and B = SSS–1 3 0WWW , find (AB)–1. [CBSE (F) 2012]
–1
SS W SS W
5 –2 2W 0 –2 1W
T X T X
Sol. For B–1
1 2 –2
| B | = –1 3 0 = 1(3 – 0) – 2(–1 – 0) – 2(2 – 0) = 3 + 2 – 4 = 1 ≠ 0
0 –2 1
i.e., B is invertible matrix
⇒ B–1 exists and have unique solution.
+1 3 0 –1 0
Now, C11 = (–1) 1 = 3 – 0 = 3; C12 = (–1)1 + 2 = – (–1 – 0) = 1
–2 1 0 1

+3 –1 3 2 –2
C13 = (–1) 1 = 2 – 0 = 2; C21 = (–1) 2 + 1 = – (2 – 4 ) = 2
0 –2 –2 1
+2 1 –2 1 2
C22 = (–1) 2 = 1 – 0 = 1; C23 = (–1) 2 + 3 = – (–2 – 0) = 2
0 1 0 –2
+1 2 –2 1 –2
C31 = (–1) 3 = 0 + 6 = 6; C32 = (–1) 3 + 2 = – (0 – 2) = 2
3 0 –1 0

+3 1 2
C33 = (–1) 3 = 3+2 = 5
–1 3
SSR3 1 2WWVT SSR3 2 6WW
V
S W S WW

` adj B = SSS2 1 2WWW = SSS1 1 2WW
SS6 2 5WW SS2 2 5WW
T X T X
150 Xam idea Mathematics–XII

@Cbsebookshub - Join Us on Telegram


RS V R V
1 1 SS3 2 6WWW SSS3 2 6WWW
⇒ B –1 = ^adj B h = SS1 1 2WW = SS1 1 2WW
|B | 1 SS W S W
S2 2 5WW SS2 2 5WW
RS T V RX T XV
SS3 2 6WWW SSS 3 –1 1WWW
Now, ^ AB h = B –1 .A –1 = SSS1 1 2WWW . SSS–15 6 –5WWW
–1

SS2 2 5WW SS 5 –2 2WW


RS T XT X V R V
SS9 – 30 + 30 – 3 + 12 – 12 3 – 10 + 12WWW SSS 9 –3 5WWW
= SSS3 – 15 + 10 –1 + 6 – 4 1 – 5 + 4 WWW = SSS–2 1 0WWW
SS W
6 – 30 + 25 – 2 + 12 – 10 2 – 10 + 10W SS 1 0 2WW
T X T X
RS V
SScos a – sin a 0WWW
3. If A = SSSsin a cos a 0WWW , find adj A and verify that A(adj A) = (adj A)A =|A|I3. [CBSE (F) 2016]
SS 0 0 1WW
T R X
SScos a – sin a 0WVW
S W
Sol. Given, A = SSSsin a cos a 0WWW
SS 0 0 1WW
T X
cos a 0 sin a 0
a A11 = = cos a; A12 = – = – sin a; A13 = 0
0 1 0 1
– sin a 0 cos a 0 cos a sin a
A21 = – = sin a; A22 = = cos a; A23 = – =0
0 1 0 1 0 0
– sin a 0 cos a 0 cos a – sin a
A31 = = 0; A32 = – = 0; A33 = =1
cos a 0 sin a 0 sin a cos a
SSRcos a – sin a 0VWWT RSS cos a sin a 0VWW
S W S W
` adjA = SSSsin a cos a 0WWW = SSS– sin a cos a 0WWW
SS 0 0 1WW SS 0 0 1WW
T X T X
RS VR V
SScos a – sin a 0WWW SSS cos a sin a 0WWW
Now A. adj A = SSSsin a cos a 0WWW SSS– sin a cos a 0WWW
SS 0 0 1WW SS 0 0 1WW
TR XT X V R V
SSS cos 2 a + sin 2 a cos a. sin a – sin a. cos a 0WWW SS1 0 0WW
S 2 2 W SS WW
= SSsin a. cos a – sin a. cos a sin a + cos a 0WW = SS0 1 0WW
SS W
S 0 0 1WW SS0 0 1WW
T X T X
RS VW
SS cos a – sin a 0 W
A. adj A = A I3 ... (i) SSa A = sin a cos a 0 = cos 2 a + sin 2 a = 1WWW
SS WW
S 0 0 1 W
RS VW RS T VW X
SS cos a sin a 0WW SScos a – sin a 0WW
Again, adj A.A = SSS– sin a cos a 0WWW SSSsin a cos a 0WWW
SS 0 0 1WW SS 0 0 1WW
TSR X T X V R V
SS cos 2 a + sin 2 a – sin a. cos a + sin a. cos a 0WWW SS1 0 0WW
S W S W
= SS– sin a. cos a + sin a. cos a sin 2 a + cos 2 a 0WW = SSS0 1 0WWW
SS W
S 0 0 1WW SS0 0 1WW
T X T X
⇒ adj A.A = |A| I3 …(ii)
From (i) and (ii), we get
A . adj A = adj A . A = |A|I3.

Adjoint and Inverse of a Matrix 151


@Cbsebookshub - Join Us on Telegram
RS VR V
SS1 –1 2WWW SSS–2 0 1WWW
4. Use product SSS0 2 –3WWW SSS 9 2 –3WWW to solve the system of equations:
SS WS W
3 –2 4W S 6 1 –2W
T XT X
x – y + 2z = 1, 2y – 3z = 1, 3x – 2y + 4z = 2 [CBSE (F) 2011]
OR
RS VW RS VW
SS 1 – 1 2 WW SS– 2 0 1 WW
Use product SS0 2 –3WW SS 9 2 –3WW to solve the system of equations
SS W S W
S3 –2 4 WW SS 6 1 –2WW
T X T X
x + 3z = 9, –x + 2y –2z = 4,  2x – 3y + 4z = –3. [CBSE Delhi 2017]
Sol. Given system of equations are
x – y + 2z = 1, 2y – 3z = 1, 3x – 2y + 4z = 2
Above system of equations can be written in matrix form
as AX = B ⇒ X = A–1 B
RS V RSxVW RS1VW
SS1 –1 2WWW SS WW SS WW
where, A = SSS0 2 –3WWW, X = SSyWW, B = SS1WW
S3 –2 4W SS WW SS WW
S W z S2W
T V X T X T X
RS
SS–2 0 1WWW
Let C = SSS 9 2 –3WWW
SS 6 1 –2WW
T RS X VR V R V R V
SS1 –1 2WWW SSS–2 0 1WWW SSS –2 – 9 + 12 0 – 2 + 2 1 + 3 – 4WWW SSS1 0 0WWW
Now, AC = SSS0 2 –3WWW SSS 9 2 –3WWW = SSS 0 + 18 – 18 0 + 4 – 3 0 – 6 + 6WWW = SSS0 1 0WWW
SS3 –2 4WW SS 6 1 –2WW SS–6 – 18 + 24 0 – 4 + 4 3 + 6 – 8WW SS0 0 1WW
T XT X T X T X
⇒ AC = I ⇒ A–1 (AC) = A–1 I [Pre-multiply by A–1]
⇒ (A–1 A)C = A–1
[By Associativity]
–1 –1
⇒ IC = A ⇒ A = C
SRS–2 0 1 WVW
S W
⇒ A = SSS 9 2 –3WWW
–1
SS 6 1 –2WW
T X
Putting X, A–1 and B in X = A–1B, we get
RS VW RS VR V RS VW RS + + VW
SSxWW SS–2 0 1 WWW SSS1WWW SSxWW SS–2 0 2WW
SSSyWWW = SSS 9 2 –3WWW SSS1WWW & SSSyWWW = SSS 9 + 2 – 6 WWW
SS WW SS 6 1 –2WW SS2WW SS WW SS 6 + 1 – 4 WW
z z
T X T XT X T X T X
RS VW RS VW
SSxWW SS0WW
⇒ SSSyWWW = SSS5WWW & x = 0, y = 5 and z = 3
SS WW SS3WW
z
T X T X
OR
Solution is similar as above. Ans.: x = 0, y = 5 and z = 3
5. Using matrices, solve the following system of equations:
4x + 3y + 2z = 60; x + 2y + 3z = 45; 6x + 2y + 3z = 70 [CBSE (AI) 2011]
Sol. The system can be written as AX = B ⇒ X = A–1B …(i)
RS VW RS VW RS VW
SS4 3 2WW SSxWW SS60WW
where A = SSS1 2 3WWW , X = SSSyWWW and B = SSS45WWW
SS6 2 3WW SSzWW SS70WW
T X T X T X
152 Xam idea Mathematics–XII

@Cbsebookshub - Join Us on Telegram


|A| = 4(6 – 6) – 3(3 – 18) + 2(2 – 12) = 0 + 45 – 20 ≠ 0
For adj A
A11 = 6 – 6 = 0 A21 = –(9 – 4) = –5 A31 = (9 – 4) = 5
A12 = –(3 – 18) = 15 A22 = (12 – 12) = 0 A32 = –(12 – 2) = –10
A13 = (2 – 12) = –10 A23 = –(8 – 18) = 10 A33 = (8 – 3) = 5
RS VWT RS VW
SS 0 15 –10W S 0 –5
WW SS
5W
W
∴ adj A = SS–5 S 0 10W = S 15 0 –10WW
SS 5 –10 W S W
5WW SS–10 10 5WW
TR XV T R X V RS V
SS 0 –5 5WW SS 0 –1 1WW SS 0 –1 1WW
1 S
SS 15 W 5 S W 1 W
∴ A –1 = 0 –10WW = SS 3 0 –2WW = SS 3 0 –2WW
25 S S W 25 S S W 5S S W
S–10 10 5WW S–2 2 1WW S–2 2 1WW
T X T X T X
Now putting values in (i), we get
RS VW RS VR V
SSxWW SS 0 –1 1WW SS60WW
1 WS W
∴ SSSyWWW = SSS 3 0 –2WW SS45WW
WS W
SSzWW 5 SS–2 2 1WW SS70WW
TR XV TR XT X
SSxWW SS 0 – 45 + 70 WVW SRSxWWV SR25WV SR5WV
SS WW 1 S S WW SS WW 1 SSS WWW SSS WWW
⇒ SSyWW = SS 180 + 0 – 140 WW ⇒ SSyWW = SS40WW = SS8WW
SSzWW 5 SS W SSzWW 5 SS40WW SS8WW
–120 + 90 + 70W
T X T X T X T X T X
Hence, x = 5, y = 8, z = 8.
R VR V
SSS–4 4 4 WWW SSS1 –1 1 WWW
6. Determine the product SS–7 1 3 WW SS1 –2 –2WW and use it to solve the system of equations
SS WS W
S 5 –3 –1WW SS2 1 3 WW
T XT X
x – y + z = 4, x – 2y – 2z = 9, 2x + y + 3z = 1. [CBSE (AI) 2017]
RS VWRS VW RS + + VW
SS–4 4 4 WWSS1 –1 1 WW SS–4 4 8 4 – 8 + 4 4– 8 + 12 WW
Sol. SS–7 1 3 WWSS1 –2 –2WW = SS–7 + 1 + 6 7 – 2 + 3 –7 – 2 + 9WW
SS WS W S W
S 5 –3 –1WWSS2 1 3 WW SS 5 – 3 – 2 –5 + 6 – 1 5 + 6 – 3 WW
T XT X T X
RS V RS V
SS8 0 0WWW SS1 0 0WWW
= SS0 8 0WW = 8 SS0 1 0WW = 8I
SS W SS W
S0 0 8WW S0 0 1WW
T X T X
RS VR V RS V– 1 RS V
SS–4 4 4 WWW SSS1 –1 1 WWW SS1 –1 1 WWW SS–4 4 4 WWW
1S 1
⇒ S–7 1 3 WW SS1 –2 –2WW = I ⇒ SS1 –2 –2WW = SS–7 1 3 WW
8 SS WW SS WW SS W 8 SS W
S 5 –3 –1W S2 1 3 W S2 1 3 WW S 5 –3 –1WW
T XT X T X T X
Given system of equation can be written in matrix form as
AX = B ⇒ X = A–1 B, ...(i)
RS V RS VW RS VW
SS1 –1 1 WWW SSxWW SS4WW
where, A = SS1 –2 –2WW, X = SSyWW, B = SS9WW
SS W SS WW SS WW
S2 1 3 WW SzW S1W
T R X V T X T X
SSS–4 4 4 WWW
1
We have A–1 = SS–7 1 3 WW
8 SS W
S 5 –3 –1WW
T X
Now from (i)
RS VW RS VR V RS V RS VW
SSxWW SS–4 4 4 WWWSSS4WWW SS–16 + 36 + 4WWW SS 24 WW
1 1 1
SSyWW = SS–7 1 3 WWSS9WW = SS –28 + 9 + 3 WW = SS–16WW
SS WW 8 SS WS W 8 SS W 8 SS WW
SzW S 5 –3 –1WWSS1WW S 20 – 27 – 1 WW S –8 W
T X T XT X T X T X
Adjoint and Inverse of a Matrix 153
@Cbsebookshub - Join Us on Telegram
RS VW RS VW
SSxWW SS 3 WW
⇒ SSyWW = SS–2WW ⇒ x = 3, y = – 2, z = – 1
SS WW SS WW
SzW S–1W
T X T X
7. A shopkeeper has 3 varieties of pens ‘A’, ‘B’ and ‘C’. Meenu purchased 1 pen of each variety for a
total of ` 21. Jeevan purchased 4 pens of ‘A’ variety, 3 pens of ‘B’ variety and 2 pens of ‘C’ variety
for ` 60. While Shikha purchased 6 pens of ‘A’ variety, 2 pens of ‘B’ variety and 3 pens of ‘C’
variety for ` 70. Using matrix method, find cost of each variety of pen. [CBSE (Central) 2016]
Sol. Let the cost of varieties of pens A, B and C be ` x, ` y, and ` z respectively.
From question
x + y + z = 21, 4x + 3y + 2z = 60, 6x + 2y + 3z = 70
The given system of linear equation in matrix equation is as follows
RS V RS21VW
SS1 1 1WWW SSRxVWW SS WW
S W
AX = B, where A = SSS4 3 2WWW, X = SSyWW and B = SS60WW
SS6 2 3WW SS WW SS WW
z S70W
–1T X T X T X
 AX = B ⇒ X = A B …(i)
1 1 1
Now A = 4 3 2 = 1 ^ 9 – 4 h – 1 ^12 – 12 h + 1 ^8 – 18 h) = 5 – 0 – 10 = –5 ! 0
6 2 3
A11 = (9 – 4) = 5 A21 = – (3 – 2) = –1 A31 = (2 – 3) = – 1
A12 = – (12 – 12) = 0 A22 = (3 – 6) = – 3 A32 = – (2 – 4) = 2
A13 = (8 – 18) = –10 A23 = – (2 – 6) = 4 A33 = (3 – 4) = –1
RS VWT RS VW
SS 5 0 –10WW SS 5 –1 –1WW
Adj ` A = SSS–1 –3 4WW = SS 0 –3 2WW
SS–1 2 –1WWW SSS–10 4 –1WWW
T XR T X
SS 5 –1 –1VWW
1 1 S W
` A –1 = . adj A = – SSS 0 –3 2WWW
|A | 5S
S–10 4 –1WW
T X
Now from (i) X = A–1B
RSxVW RS VR V RSxVW RS–25VW RSxVW RS5VW
S 5 –1 –1WW SS21WW
SS WW 1SS WW SS WW SS WW 1 SSS WWW SS WW SS WW
SSyWW = – SS 0 –3 2 WW SS60WW & SSyWW = – S–40W & SSyWW = SS8WW & x = 5, y = 8, z = 8
SS WW 5S SS WW 5 SS WW SS WW SS WW
S–10 4 –1WW SS70WW S–40W S W
TzX T XT X T zX
T X TzX T8X
⇒ Cost of pen A = ` 5; cost of pen B = ` 8 and cost of pen C = ` 8
RS V
SS3 2 1WWW
8. If A = SSS4 –1 2WWW , then find A–1 and hence solve the following system of equations:
SS7 3 –3WW
T X
3x + 4y + 7z = 14, 2x – y + 3z = 4, x + 2y – 3z = 0
RS V
SS3 2 1WWW
Sol. Given A = SSS4 –1 2WWW
SS7 3 –3WW
T X
|A| = 3 (3 – 6) + (– 2) (– 12 – 14) + 1 (12 + 7) = 62 ≠ 0
Hence, A–1 exists. Let Cij be the cofactor of aij (ijth element of A)
Now, C11 = (3 – 6) = – 3 C21 = – (– 6 – 3) = 9 C31 = (4 + 1) = 5
C12 = – (– 12 – 14) = 26 C22 = – 9 – 7 = – 16 C32 = – (6 – 4) = – 2

154 Xam idea Mathematics–XII

@Cbsebookshub - Join Us on Telegram


C13 = (12 + 7) = 19 C23 = – (9 – 14) = 5 C33 = (– 3 – 8) = – 11
SRS–3 26 19WWVl SRS–3 9
V
W
5W
S W S W
\ adj A = SSS 9 –16 5WWW = SSS26 –16 –2WWW
SS 5 –2 –11WW SS19 5 –11WW
T R X VT X
SS–3 9 5WW
1 SS W
\ A –1 = S26 –16 –2WWW
62 SS
S19 5 –11WW
T X
The given system of equations can be written in matrix form as
RS V RSxVW RS14VW
SS3 2 1WWW SS WW SS WW
S
Al X = B, where A = SS4 –1 2WW, X = SSyWW, B = SS 4 WW W
SS7 3 –3WW SS WW SS WW
zX S0W
T X T T X
⇒ X = (Al ) –1 B ⇒ X = (A –1)l B [a (Al ) –1 = (A –1)l ]
RS V RS VW RS14VW
RSxVW
SS WW 1 SS–3 9 5WWl SRS14VWW RSxVW
SS WW 1 SS–3 26 19WW SS WW
SS26 –16 –2WW S 4W W S W SS 9 –16
⇒ SSyWW = WW SS WW ⇒ SSyWW = 5WWW SS 4WW
SS WW 62 SS SS WW 62 SS S W
z S19 5 –11W S 0W S W z S 5 –2 –11WW SS 0WW
T X T X T X T X T XT X
RS VW RS–42 + 104 + 0VW RS VW RS62VW R x V RS1VW
x x S
SS WW
1 SS WW SS WW
1 SS WW SS WWW SS WW
⇒ SSSyWWW = SS 126 – 64 + 0 WW ⇒ SSyWW =
SS WW 62 SS62WW & SSyWW SS1WW
S W S W=S W
SSzWW 62 SSS WW SS62WW SzW SS1WW
70 – 8 + 0 W SzW
T X T X
T X T X T X T X
⇒ x = 1, y = 1, z = 1
9. A mixture is to be made of three foods A, B, C. The three foods A, B, C contain nutrients P, Q,
R as shown below: [HOTS]
Food Grams per kg of nutrient
P Q R
A 1 2 5
B 3 1 1
C 4 2 1

How to form a mixture which will have 8 grams of P, 5 grams of Q and 7 grams of R?
Sol. Let food needed be x kg of A, y kg of B and z kg of C. Therefore x kg of A contains 1 gram of
nutrient P. So, x kg of A will contain x grams of nutrient P. Similarly, the amount of nutrient P in
y kg of food B and z kg of food C are 3y and 4z grams respectively. So, total quantity of nutrient
P in x kg of food A, y kg of food B and z kg of food C is x + 3y + 4z grams.
x + 3y + 4z = 8
Similarly, 2x + y + 2z = 5 [For Q]
and 5x + y + z = 7 [For R]
The above system of simultaneous linear equations can be written in matrix form as AX = B.
RS VR V R V
SS1 3 4WWW SSSxWWW SSS8WWW
or, SSS2 1 2WWW SSSyWWW = SSS5WWW
SS5 1 1WW SSzWW SS7WW
T XT X T X
1 3 4
Now, A = 2 1 2 = 1 ^1 – 2 h – 3 ^ 2 – 10 h + 4 ^ 2 – 5 h) = –1 + 24 – 12 = 11 ! 0
5 1 1
So, A–1 exists and system have unique solution.

Adjoint and Inverse of a Matrix 155


@Cbsebookshub - Join Us on Telegram
Let Cij be the cofactor of aij in A = [aij]. Then,
C11 = –1 ; C12 = 8; C13 = –3
C21 = 1 ; C22 = –19; C23 = 14
C31 = 2 ; C32 = 6; C33 = –5
RS VWT RS V
SS–1 8 –3W SS–1 1 2WW
S WW W
` adj A = SS 1 –19 14WW = SS 8 –19 6WW
S 2 S
S–3 14 –5WWW
S 6 –5WW S
T XR T V X
SS 1 1 2WW
1 1 S W
& A –1 = A adj A = 11 SSS 8 –19 6WWW
SS–3 14 –5WW
T X
Putting value of X, A–1 and B in X = A–1B, we get
RS VW SRS–1 1 VR V RS VW RS –8 + 5 + 14 VW R V R V
SSxWW 2 WW SS8WW SSxWW WW 1 SSS11WWW SSS1WWW
S W 1 SS S
WW S W W S W 1 SSS
SSyWW = S 8 –19 6 WW SS5WW & SSyWW = S 64 – 95 + 42 WWW = 11 SSS11WWW = SSS1WWW
SSzWW 11 SS W SS7WW SSzWW 11 SSS W SS11WW SS1WW
S–3 14 –5W –24 + 70 – 35W
T X T XT X T X T X T X T X
⇒ x = 1, y = 1 and z = 1.
Thus, the mixture is formed by mixing 1 kg of each of the food A, B and C.

PROFICIENCY EXERCISE
QQ Objective Type Questions: [1 mark each]
1.
Choose and write the correct option in each of the following questions.

(i) If A = = G be such that A–1 = kA, then k equals


2 3
5 –2
1 –1
(a) 19 (b) (c) –19 (d)
19 19
–1
(ii) If = G= G ==
a –b
G , then
1 – tan i 1 tan i
tan i 1 – tan i 1 b a
(a) a = 1, b = 1 (b) a = cos 2 q, b = sin 2 q
(c) a = sin 2 q, b = cos 2 q (d) none of these
(iii) If A is an invertible matrix of order 3 and |A| = 5, then |adj A| =
(a) 5 (b) 125 (c) 25 (d) none of these
3 2 –1
(iv) If A satisfies the equation x – 5x + 4x + l = 0, then A exists if
(a) l ≠ 1 (b) l ≠ 2 (c) l ≠ –1 (d) all of them

(v) For any 2 × 2 matrix, if A (adj A) = = G , then |A| is equal to


10 0
0 10
(a) 20 (b) 10 (c) 0 (d) 100
2.
Fill in the blanks.

(i) If adj.A = = G and adj B = = G , then adj (AB) = _____________ .


2 3 1 –2
4 –1 –3 1

(ii) If A is a non-singular sequence matrix such that A –1 = = G , than ^ AT h = ___________ .


5 3 –1
–2 –1

(iii) If A = = G then adj A = _____________ .


1 –3
2 0

156 Xam idea Mathematics–XII

@Cbsebookshub - Join Us on Telegram


QQ Very Short Answer Questions: [1 mark each]

3. If A = = G and A (adj A) = = G , then find the value of k.


cos x sin x k 0

– sin x cos x 0 k
4. A is a square matrix of order 3 and |A| = 7. Write the value of |adj A|. [CBSE (AI) 2010]
5. If A is a square matrix of order 3 such that |adj A|= 64, find |A|.

6. If A = = G and B = = G then find adj (AB).


a b 1 0

c d 0 1
QQ Short Answer Questions–I: [2 marks each]

7. If A = = G , show that A –1 =
2 3 1
A
5 –2 19
RS V
SS 1 a WW
S tan W
2 WW
8. Find the adjoint of the matrix SS W
SS– tan a 1 WWW
SS 2 W
T X
9. Solve the following system of equations by matrix method
3x + y = 19
3x – y = 23
10. Using elementary transformations, find the inverse of the following matrix

= G
2 1
7 4
QQ Short Answer Questions–II: [3 marks each]
11. A typist charges ` 145 for typing 10 English and 3 Hindi pages, while charges for typing 3 English
and 10 Hindi pages are ` 180. Using matrices, find the charges of typing one English and one
Hindi page separately. [CBSE (North) 2016]
12. If A, B are square matrices of the same order, then prove that adj (AB) = (adj B) (adj A).
RS V
SS2 0 –1WWW
13. Using elementary transformations, find the inverse of the matrix SSS5 1 0 WWW . [CBSE (F) 2011]
S0 1 3 W
S W
T X
14. Let A = G, B = G . Then compute AB. Hence, solve the following system of equations :
2 3 4 –6
= =
1 2 –2 4
2x + y = 4, 3x + 2y = 1 [CBSE Sample Paper 2016]

QQLong Answer Questions: [5 marks each]


RS V
SS2 –3 5WWW
15. If A = SS3 2 –4WW find A–1. Use it to solve the system of equations
SS W
S1 1 –2WW
T X
2x – 3y + 5z = 11
3x + 2y – 4z = –5
x + y – 2z = – 3 [CBSE 2018]
16. Using elementary row transformations, find the inverse of the matrix
RS V
SS 1 2 3WWW
A = SS 2 5 7WW [CBSE 2018]
SS W
S–2 –4 –5WW
T X

Adjoint and Inverse of a Matrix 157


@Cbsebookshub - Join Us on Telegram
RS V
SS1 1 1WWW
17. If A = SS1 0 2WW . Find A–1. Hence, solve the system of equations
SS W
S3 1 1WW
T X
x + y + z = 6, x + 2z = 7, 3x + y + z = 12 . [CBSE 2019 (65/1/1)]
18. Using matrix, solve the following system of equations:
2x – 3y + 5z = 11, 3x + 2y – 4z = –5, x + y – 2z = –3 [CBSE Delhi 2009]
19. Obtain the inverse of the following matrix, using elementary operations:
RS V
SS3 0 –1WWW
A = SSS2 3 0WWW [CBSE (AI) 2009]
S0 4 1W
S W
T X
20. Using elementary operations, find the inverse of the following matrix:
RS V
SS–1 1 2WWW
SS 1 2 3WW [CBSE Delhi 2011]
SS W
S 3 1 1WW
T X
RS V
SS1 2 2WWW
21. If A = SSS2 1 2WWW , verify that A2 – 4A – 5I = 0. [CBSE Delhi 2008]
SS2 2 1WW
T X
22. Using elementary transformations, find the inverse of the following matrix:
RS V
SS2 5 3WWW
SSS3 4 1WWW [CBSE (AI) 2008]
SS1 6 2WW
T X
23. Using matrices, solve the following system of linear equations:
3x – 2y + 3z = 8, 2x + y – z = 1, 4x – 3y + 2z = 4 [CBSE (F) 2009]
24. Using matrices, solve the following system of equations:
x + 2y + z = 7; x + 3z = 11 and 2x – 3y = 1 [CBSE (AI) 2011]
RS VW
SS2 0 1WW
25. If A = SSS2 1 3WWW , find A2 – 5A + 4I and hence find a matrix X such that A2 – 5A + 4I + X = 0.
SS1 –1 0WW
 T X [CBSE Delhi 2015]
26. Using elementary row operations (transformations), find the inverse of the following matrix:
RS V
SS0 1 2WWW
SSS1 2 3WWW [CBSE Ajmer 2015]
SS3 1 0WW
T X RS V
SS–1 –2 –2WWW
27. Find the adjoint of the matrix A = SS 2 1 –2WW and hence, show that A.(adj A) = |A|I3.
SS W
S 2 –2 1WW
 T X [CBSE Allahabad 2015]
RS VW
SS 3 –1 1WW
28. Find the inverse of matrix A = SS–15 6 –5WW and hence show that A–1. A = I. [CBSE Chennai 2015]
SS W
S 5 –2 2WW
T X SRS8 4 3WVW
S W
29. Using elementary transformations, find the inverse of the matrix A = SS2 1 1WW and use it to solve
SS W
S1 2 2WW
the following system of linear equations: T X
8x + 4y + 3z = 19; 2x + y + z = 5 and x + 2y + 2z = 7 [CBSE Delhi 2016]

158 Xam idea Mathematics–XII

@Cbsebookshub - Join Us on Telegram


30. Using matrices, solve the following system of equations:
x – y + z = 4; 2x + y – 3z = 0; x + y + z = 2 [CBSE (F) 2012]
31. Using elementary row operations, find the inverse of the following matrix:
SRS 2 –1 3WVW
S W
A = SS–5 3 1WW [CBSE (South) 2016]
SS W
S–3 2 3WW
T X
32. Using matrices, solve the following system of equations:
2x + 3y + 3z = 5, x – 2y + z = – 4, 3x – y – 2z = 3 [CBSE (AI) 2012]
33. Using matrices, solve the following system of equations:
x + y – z = 3; 2x + 3y + z = 10; 3x – y – 7z = 1 [CBSE (AI) 2012]
34. Solve the following system of equations by matrix method:
x – y + 2z = 7
2x – y + 3z = 12
3x + 2y – z = 5 [CBSE 2020 (65/3/1)]

Answers
1. (i) (b) (ii) (b) (iii) (c) (iv) (d) (v) (b)

2. (i) = G (ii) = G (iii) = G


–6 5 5 –2 0 3
–2 –10 3 –1 –2 1
RS V
SS 1 a WW
– tan W
6. = G
d –b S 2 WW
3. 1 4. 49 5. ± 8 8. SS W
–c a SStan a 1 WWW
SS 2 W
T X

9. x = 7, y = –2 10. = G
4 –1
11. ` 10 and ` 15 respectively
–7 2
RS V
SS 3 –1 1 WWW
14. AB = = G, x = 7, y = –10
2 0
13. A –1 = SS–15 6 –5WW
SS WW 0 2
S 5 –2 2 W
T X
RS VW RS V
SS 0 1 – 2 WW SS 3 –2 –1WWW
15. A = SS–2 9 –23WW ; x = 1, y = 2, z = 3
–1
16. SS–4 1 –1WW
SS WW SS W
S–1 5 –13W S 2 0 1 WW
T R X V T X
SSS–2 0 2 WWW
1
17. A –1 = SS 5 –2 –1WW ; x = 3, y = 1, z = 2 18. x = 1, y = 2, z = 3
4 SS W
S 1 2 –1WW
RS T V X RS V RS V
SS 3 –4 3 WWW SS 1 –1 1 WWW SS 2 8 –7WWW
1
19. SS–2 3 –2WW 20. A = SS–8 7 –5WW
–1
22. A = –1 S–5 1 7 WW
SS WW SS WW 21 SSS W
S 8 –12 9 W S 5 –4 3 W S14 –7 –7WW
T X T X RS T V RS X V
SS–1 –1 –3 WWW SS1 1 3 WWW
23. x = 1, y = 2, z = 3 24. x = 2, y = 1, z = 3 25. SS–1 –3 –10WW, X = SS1 3 10WW
SS W SS W
S–5 4 2 WW S5 –4 –2WW
T X T X

Adjoint and Inverse of a Matrix 159


@Cbsebookshub - Join Us on Telegram
RS V RS V RS V
SS 3 –2 1 WWW SS–3 6 6 WWW SS2 0 –1WWW
26. A –1 = SS–9 6 –2WW 27. adj A = SS–6 3 –6WW 28. A –1 = SS5 1 0 WW
SS WW SS WW SS W
S 5 –3 1 W S–6 –6 3 W S0 1 3 WW
T X T X T X
SRS 2
V
1 WW
SS 0 – WW
SS 3 3W
–1
29. A = SS 13 2 WW ; x = 1, y = 2 and z = 1 30. x = 2, y = – 1, z = 1
SS 1 – 3 3 WWW
SS–1 4 0 WW
TR X
SS –7 –9 10WVW
S W
31. A –1 = SS–12 –15 17WW 32. x = 1, y = 2, z = – 1 33. x = 3, y = 1, z = 1
SS W
S 1 1 –1WW
T X
34. x = 2, y = 1 and z = 3

SELF-ASSESSMENT TEST
Time allowed: 1 hour Max. marks: 30
1. Choose and write the correct option in the following questions. (4 × 1 = 4)
(i) The sum of three numbers is 6. If we multiply the third number by 2 and add the first number
to the result, we get 7. By adding second and third numbers to three times the first number,
we get 12. Using matrices, then the numbers are
(a) x = 3, y = 1, z = 2 (b) x = – 3, y = 1, z = 2
(c) x = 3, y = – 1, z = 1 (d) x = 3, y = 2, z = 5

(ii) The matrix A satisfying the equation = G A= G = = G is


2 1 –3 2 1 0
3 2 5 –3 0 1
4 1 1 4 1 1
(a) > H (b) > H (c) > H (d) zero matrix
1 0 1 0 1 0

(iii) If A = = G , then |A . adj A| is equal to


2 1
7 5
(a) 3 (b) 9 (c) 27 (d) 81
RS VW
SS 1 2 –1WW
(iv) If A SSS–1 1 2 WWW , then |adj (adjA)| is equal to
=
SS 2 –1 1 WW
T X
(a) 14 (b) 142 (c) 143 (d) 144
2. Fill in the blanks. (2 × 1 = 2)
(i) If A is an invertible matrix of order 3 and |A| = 4, then |adj A| = _____________ .
(ii) If A is an invertible matrix of order n then |adj (adj A)| = _____________ .
QQ Solve the following questions. (2 × 1 = 2)

3. Write the adjoint of the following matrix: = G


2 –1
[CBSE (AI) 2010]
4 3
RS V
SSa 0 0WWW
4. If A = SS0 a 0WW , find the value of |adj A|.
SS W
S0 0 aWW
T X

160 Xam idea Mathematics–XII

@Cbsebookshub - Join Us on Telegram


QQ Solve the following questions. (4 × 2 = 8)

5. If A = = G , then write A–1.


3 10

2 7

6. Find the inverse of the matrix = G , using elementary transformations.


5 2

2 1

7. If A = = G , show that A –1 =
2 3 1
A
5 –2 19
8. Solve the following system of equations by matrix method
3x + y = 19
3x – y = 23
QQ Solve the following questions. (3 × 3 = 9)
9. In a survey of 20 richest persons of three residential society A, B, C it is found that in society A,
5 believe in honesty, 10 in hard work and 5 in unfair means while in B, 5 believe in honesty, 8 in
hard work and 7 in unfair means and in C, 6 believe in honesty, 8 in hard work and 6 in unfair
means. If the per day income of 20 richest persons of society A, B, C are ` 32,500, ` 30,500, ` 31,000
respectively, then find the per day income of each type of people by matrix method.
10. A coaching institute of English (subject) conducts classes in two batches I and II and fees for rich
and poor children are different. In batch I, it has 20 poor and 5 rich children and total monthly
collection is ` 9,000, whereas in batch II, it has 5 poor and 25 rich children and total monthly
collection is ` 26,000. Using matrix method, find monthly fees paid by each child of two types.
RS V
SS8 4 3WWW
11. Using elementary transformations, find the inverse of the matrix A = SS2 1 1WW and use it to solve
SS W
the following system of linear equations: S1 2 2WW
T X
8x + 4y + 3z = 19; 2x + y + z = 5; x + 2y + 2z = 7
QQ Solve the following question. (1 × 5 = 5)
12. Two Trusts A and B receive ` 70000 and ` 55000 respectively from central government to award
prize to persons of a district in three fields agriculture, education and social service. Trust A
awarded 10, 5 and 15 persons in the field of agriculture, education and social service respectively
while trust B awarded 15, 10 and 5 persons respectively. If all three prizes together amount to
`6000, then find the amount of each prize by matrix method.

Answers
(n – 1) 2
1. (i) (a) (ii) (c) (iii) (b) (iv) (d) 2. (i) 16 (ii) A

3. adj A = = G 5. = G 6. = G
3 1 7 –10 1 –2
4. a6 8. x = 7, y = –2
–4 2 –2 3 –2 5
9. Per day income who believe in honesty, hard work, and unfair means are `1500, `2000, ` 1000
respectively.
10. Fee for rich children and poor children are ` 1000 and ` 200.
RS V
SS 0 2/3 –1/3WWW
11. A –1 = SS 1 –13/3 2/3 WW, x = 1, y = 2 and z = 1
SS W
S–1 4 0 WW
T X
12. Prizes in the field of agriculture, education and social service are ` 2000, ` 1000 and ` 3000 respectively.
zzz

Adjoint and Inverse of a Matrix 161


@Cbsebookshub - Join Us on Telegram
6 Continuity and
Differentiability

1. Continuity and Discontinuity of Function: A function y = f(x) is said to be continuous in an interval


if for every value of x in that interval y exist. If we plot the points, the graph is drawn without lifting
the pencil.
If we have to lift the pencil on drawing the curve, then the function is said to be a discontinuous
function.
2. Continuity and Discontinuity of a Function at a Point: A function f(x) is said to be continuous at a
point a of its domain if
lim f (x), lim f (x), f (a) exist and lim f (x) = lim f (x) = f (a)
- + - +
x"a x"a x"a x"a

A function f(x) is said to be discontinuous at x = a if it is not continuous at x = a.


There are three cases of discontinuity of a function which can be illustrated by fig. (diagram) as.

162 Xam idea Mathematics–XII

@Cbsebookshub - Join Us on Telegram


3. Properties of Continuous Function:
If f and g are two continuous functions at a point a, then
(i) f + g is continuous at a. (ii) f – g is continuous at a.
f
(iii) f . g is continuous at a. (iv) is continuous at a, provided g(a) ≠ 0.
g
(v) c.f is continuous at a, where c is a constant.
(vi) |f| is continuous function at a.
OO Every constant function is continuous function.

OO Every polynomial function is continuous function.

OO Identity function is continuous function.

OO Every logarithmic and exponential function is a continuous function.

4. Important Series which are Frequently Used in Limits:


n (n – 1) x 2 n (n – 1) (n – 2) x 3
(i) (1 + x) n = 1 + nx + + + ...
2! 3!
x x2 x3 x x2 x3
(ii) e x = 1 + + + + ...3 and e –x = 1 – + – + ...
1! 2! 3! 1! 2! 3!
x2 x2 x3
(iii) a x = 1 + x log e a + (log e a) 2 + ... and log | 1 + x | = x – + ...
2! 2 3
x2 x3 x 3 x 5 x7
(iv) log | 1 – x | = –x – – ... and sin x = x – + – + ...
2 3 3! 5! 7 !
x 2 x 4 x6 x3 x5
(v) cos x = 1 – + – + ... and tan x = x + + + ...
2! 4! 6! 3! 5!
5. Differentiation from First Principle or Ab-initio Method or by Delta Method:
Given a function f(x) and if there is a small increment h in x, let their corresponding increment
is f(x + h) in f(x) i.e.,
f (x + h) – f (x)
f(x) → f(x + h), then, lim is called the differential coefficient of f(x) with respect to x.
x"0 h
List of Useful Formulae:
d n d
6. (i) (x ) = nx n – 1 (ii) (ax + b) n = n (ax + b) n – 1 . a
dx dx
d x d ax
(iii) (e ) = e x (iv) e = a.e ax
dx dx
d x d bx
(v) a = a x .log e a (vi) a = ba bx log e a
dx dx
d 1 d 1
(vii) log e x = x and log e ax = x
dx dx
d 1 d 1
(viii) log a x = and log a bx =
dx x. log e a dx x. log e a
d d
7. (i) sin x = cos x and sin ax = a cos ax
dx dx
d d
(ii) cos x = – sin x and cos ax = –a sin ax
dx dx
d d
(iii) tan x = sec 2 x and tan ax = a sec 2 ax
dx dx
d d
(iv) cot x = – cosec 2 x and cot ax = –a cosec 2 ax
dx dx

Continuity and Differentiability 163


@Cbsebookshub - Join Us on Telegram
d d
(v) sec x = sec x tan x and sec ax = a sec ax. tan ax
dx dx
d d
(vi) cosec x = – cosec x cot x and cosec ax = – a cosec ax. cot ax
dx dx
d 1 d a
8. (i) sin –1 x = and sin –1 ax =
dx 1 – x2 dx 1 – a2 x2
d –1 d –a
(ii) cos –1 x = and cos –1 ax =
dx 1– x 2 dx 1 – a2 x2
d 1 d a
(iii) tan –1 x = and tan –1 ax =
dx 1 + x2 dx 1 + a2 x2
d –1 d –a
(iv) cot –1 x = and cot –1 ax =
dx 1 + x2 dx 1 + a2 x2
d 1 d 1
(v) sec –1 x = and sec –1 ax =
dx x x2 – 1 dx 2 2
x a x –1
d –1 d –1
(vi) cosec –1 x = and cosec –1 ax =
dx x x2 – 1 dx x a x2 – 1
2

d dv du
9. Product Rule: Let u and v be two functions of x, then (u.v) = u. + v. .
dx dx dx
d
i.e.,
(Product of two functions)
dx
d d
= First function (Second function) + Second function (First function)
dx dx
10. Quotient Rule: If u and v are functions of x then,
du dv d (N r) d (D r)
v –u Dr – Nr
d Nr
d n= f p=
d u dx dx dx dx
i.e.,
dx v v 2 dx D r r 2
(D )
11. Chain Rule: Chain rule is applied when the given function is the function of function i.e.,
dy dy du dy dy du dv
if y is a function of x, then = . or = . .
dx du dx dx du dv dx
12. Logarithmic Differentiation: Logarithmic differentiations are used for differentiation of functions which
consists of the product or quotients of a number of functions and/or the given function is of type [f(x)]g(x),
where f(x) and g(x) both are differentiable functions of x.
Therefore, in this method, we take the logarithm on both the sides of the function and then differentiate it with

respect to ‘x’. So, this process is called logarithmic differentiation.
General method: If y = [f(x)]g(x) then

dy
= y =log f (x) g' (x) + g (x) . . f l (x)G
1

dx f ( x)
13. Parametric Form: Sometimes we come across the function when both x and y are expressed in terms of
another variable say t i.e., x = f(t) and y = ψ(t). This form of a function is called parametric form and t is
called the parameter.
dy
To obtain in parametric type of functions we follow any one of the following two steps:
dx
(i) Try to obtain a relationship between x and y by eliminating the parameter and then proceed to
dy
get which is already discussed.
dx

164 Xam idea Mathematics–XII

@Cbsebookshub - Join Us on Telegram


(ii) If it is not convenient to obtain such a relation between x and y, then differentiate x and y both

dx dy
with respect to parameter t to get
and (treating t as independent variable and x and
dt dt
dy dx dy dy dy dx
y as dependent variables). Finally, divide by to get i.e., = '
dt dt dx dx dt dt
dy dy dx
or sometimes = ' , where q is an independent variable.
dx di di
14. Rolle’s Theorem: If f(x) be a real valued function, defined in a closed interval [a, b] such that:
(i) it is continuous in closed interval [a, b].
(ii) it is differentiable in open interval (a, b).
(iii) f(a) = f(b). Then there exists at least one value c ∈ (a, b) such that f'(c) = 0.
It is illustrated by diagram as
[Note: f'(c) = 0 means tangent at c is parallel to x-axis.]
y-axis y-axis
Tangent
(c, f(c)) (c, f(c)) Tangent

f(a) = f(b) f(a) = f(b)

a c c c b x-axis a c b x-axis

15. Lagrange’s Mean Value Theorem:


If f(x) is a real valued function defined in the closed interval [a, b] such that:
(i) it is continuous in the closed interval [a, b].
(ii) it is differentiable in the open interval (a, b).
f (b) – f (a)
Then there exists at least one real value c ∈ (a, b) such that f l (c) = . It is illustrated by
b–a
diagram.

f(b) B

Tangent

A
f(a)

a b x-axis

f (b) – f (a)
[Note: f l (c) = means tangent at c is parallel to chord AB.
b–a
i.e.,
Slope of tangent at c = Slope of chord AB
16. Limits: Let f(x) be a function of x. Let a and l be two constants such that as x → a, we have
f(x) → l, i.e., the numerical difference between f(x) and l can be made as small as we wish by taking x
sufficiently close to a. In such a case, we say that the limit of function f(x) as x approaches a is l. We
write this as lim f (x) = l .
x"a

Continuity and Differentiability 165


@Cbsebookshub - Join Us on Telegram
17. Procedure to Find lim f (x) :
x"a

(i) Putting x = a in the given function. If f(a) is a finite value, then lim f (x) = f (a) .
x"a
(ii) To find LHL of f(x) at x = a we put x = a – h, h → 0 and find lim f (a – h) after simplification.
h"0
(iii) To find RHL of f(x) at x = a we put x = a + h, h → 0 and find lim f (a + h) after simplification.
h"0

(iv) If LHL = RHL = k (say), then lim f (x) = k .


x"a

18. Fundamental Theorems on Limits: Some important theorems are given below which are frequently
used in limits:
(i) lim c = c , i.e., the limit of a constant quantity is constant itself.
x"a

(ii) lim [f (x) + g (x)] = lim f (x) + lim g (x)


x"a x"a x"a

i.e., the limit of sum of two functions is equal to the sum of their limits.

(iii) lim [f (x) – g (x)] = lim f (x) – lim g (x)
x"a x"a x"a

i.e., the limit of difference of two functions is equal to the difference of their limits.

(iv) lim [f (x) . g (x)] = lim f (x) . lim g (x)
x"a x"a x"a

i.e., the limit of the product of two functions is equal to the product of their limits.

f (x)
(v) lim > H = lim f (x) / lim g (x)
x"a g (x) x"a x"a

i.e., the limit of quotient of two functions is equal to quotient of their limits.
provided limx"a
g (x) finite value not equal to zero.
(vi) lim [cf (x)] = c lim f (x) , where c is a constant.
x"a x"a

i.e., the limit of the product of a constant and the function is equal to the product of the constant

and the limit of the function.
(vii) lim f (x) = lim f (x)
x"a x"a

(viii) lim f (–x) = lim f (x)


x"0 x " 0–

19. Evaluation of Limits:


(i) Direct substituting method: We substitute the value of the point in the given expression and if
we get a finite number, then this number is the limit of the given function.
0 3
(ii) Factorisation method: On substituting x = a in the given expression, if we get , , …, etc.
0 3
form, then we factorize the numerator and denominator and take (x – a) as a common factor
from numerator and denominator. After cancelling out (x – a), we put x = a. If we get a finite
number, then it is the required value otherwise repeat the step till we get a finite number.
(iii) Rationalisation method: Rationalisation method is applicable when
(a) numerator, denominator or both in square root or
(b) after substituting the value of limit if we get the negative number in square root. Hence,
after simplifying in both the cases, we get the required value.
f (x)

(iv) L’ HOSPITAL Rule: With the help of this rule, if we have to evaluate lim such that it
x " c g (x)

166 Xam idea Mathematics–XII

@Cbsebookshub - Join Us on Telegram


0 3
takes indeterminate form, i.e., or 3 , then we differentiate numerator and denominator to
0
f (x)
get lim , if it is determinate form then it is required value, otherwise repeat the step till we
x " c (x)
g
get a determinate form and thus required value.
[Note: According to L’ HOSPITAL rule
f (x) f l ( x)
lim = lim . where g'(x) ≠ 0 ∀ x ∈ Z with x ≠ c]
x " c g (x) x " c g l ( x)

20. Some Standard Results:


xn – an xm – am m m – n
(i) (a) lim x – a = na n – 1, a > 0, n ! Q (b) lim n n = n a , m, n ! Q
x"a x"a x – a
sin x tan x
(c) lim x = 1 (d) lim cos x = 1 (e) lim x = 1
x"0 x"0 x"0

(ii) Evaluation of limits of inverse trigonometric functions:


sin –1 x tan –1 x
(a) lim =1 (b) lim =1
x"0 x x"0 x
(iii) Evaluation of limits of exponential and logarithmic functions:
ex – 1
(a) lim e x = 1 (b) lim x = 1
x"0 x"0
log | 1 + x | (1 + x) n – 1
(c) lim = 1 (d) lim =n
x"0 x x"0 x
ax – 1
(e) lim = log e a
x"0 x
(iv) Limits at infinity: This method is applied when x → ∞.
Procedure to solve the infinite limits:
(a) Write the given expression in the form of rational function.
(b) Divide the numerator and denominator by highest power of x.
1
(c) Use the result lim n = 0, where n > 0.
x"3 x
(d) Simplify and get the required result.

Selected NCERT Questions


1. Show that the function f defined by
f(x) = |1 – x + |x||,
where x is any real number, is a continuous function.
Sol. Define g by g(x) = 1 – x +|x| and h by h(x) = |x| for all real x. Then
(hog)(x) = h (g(x)) = h (1 – x +|x|)
= |1 – x +|x|| = f(x)
We know that h is a continuous function. Hence g being a sum of a polynomial function and the
modulus function is continuous. But then f being a composite of two continuous functions is
continuous.
2. Find all points of discontinuity of f, where f is defined by the following function:
Z]
]]| x |+ 3, if x # –3
]
f (x) = [] –2x, if –3 < x < 3 [CBSE Delhi 2010]
]]
] 6x + 2, if x$3
\
Sol. We know that |x| is a continuous function, so f is continuous ∀ x < –3

Continuity and Differentiability 167


@Cbsebookshub - Join Us on Telegram
– 2x is continuous everywhere, so f is continuous ∀ x, – 3 < x < 3
and 6x + 2 is continuous everywhere, so f is continuous ∀ x > 3
Now we have to check the continuity of the function at x = –3 and x = 3
At x = –3, f(x) = |x|+ 3
f(–3) = |– 3|+ 3 = 3 + 3 = 6
LHL = lim f (x) = lim | x |+ 3 = | –3 |+ 3 = 3 + 3 = 6
x " –3 – x " –3 –
RHL = lim f (x) = lim – 2x = –2 # – 3 = 6
+ +
x " –3 x " –3

LHL = RHL = f(–3), so it is continuous at x = – 3.


Again at x = 3, f(x) = 6x + 2
Here f(3) = 6 × 3 + 2 = 18 + 2 = 20
LHL = lim f (x) = lim – 2x = –2 # 3 = – 6
x " 3– x " 3–

RHL = lim f (x) = lim 6x + 2 = 6 # 3 + 2 = 20


+ +
x"3 x"3

LHL ≠ RHL
So, it is discontinuous at x = 3.
3. Find all points of discontinuity of f where f is defined by the following function:
Z] x
]] , if x < 0
f (x) = ][| x |
]]
–1 , if x $ 0
\
x
Sol. For x < 0, f (x) = is continuous and for x > 0, f(x) is a constant function so it is continuous.
|x |
At x = 0,
x
LHL = lim f (x) = lim e o = lim
x
= –1, ( a |x|= –x, when x < 0)
– – |x | – –x
x"0 x"0 x"0
RHL = lim f (x) = lim (–1) = –1
+ +
x"0 x"0
and f(0) = –1
∴ lim f (x) = lim f (x) = f (0) & f (x) is continuous at x = 0.

x " 0– x"0
+

Hence, f(x) has no points of discontinuity.


For what value of λ is the function defined by
4.
m (x 2 –2x), if x # 0
f (x) = *
4x + 1 if x > 0
continuous at x = 0? What about continuity at x = 1? [CBSE (F) 2011]
Sol.  f(x) to be continuous at x = 0

LHL = RHL = f(0)
i.e.,
lim f (x) = lim f (x) = f (0) ...(i)
x " 0– x"0
+

Now LHL = lim m (x 2 – 2x) = m (0 – 0) = 0 ...(ii)


x " 0–
RHL = lim f (x) = lim (4x + 1) = 4 # 0 + 1 = 1 ...(iii)
+ +
x"0 x"0
and f(0) = 0 ...(iv)
from (ii) and (iii), LHL ≠ RHL

168 Xam idea Mathematics–XII

@Cbsebookshub - Join Us on Telegram


So, f(x) is not continuous for any value of l.
For continuity at x = 1,
f(1) = 4 × 1 + 1 = 4 + 1 = 5
and lim f (x) = lim (4x + 1) = 4 # 1 + 1 = 5
x"1 x"1

As lim f (x) = f (1)


x"1
So, it is continuous at x = 1.
5. Show that the function defined by g(x) = x – [x] is discontinuous at all integral points. Here [x]
denotes the greatest integer less than or equal to x.
Sol. Let g(x) = x – [x] be the greatest integer function. Let n be any integer. Then
lim g (x) = lim g (n – h) = lim (n – h) – [n – h] = lim (n – h) – (n – 1) = n – (n – 1) = 1

x " n– h"0 h"0 h"0

lim g (x) = lim g (n + h) = lim (n + h) – [n + h] = lim (n + h) – n = n – n = 0



x"n
+ h"0 h"0 h"0

∴ lim g (x) ! lim g (x)



x " n– x"n
+

LHL ≠ RHL so, f is discontinuous at all integral points.


6. Find the value of a and b such that the function f(x) defined by:
Z] 5; if x # 2
]]
] +
f (x) = ][ax b; if 2 < x < 10 is a continuous function [CBSE Delhi 2011]
]]
] 21; if x $ 10
\
Sol. Since, f(x) is continuous.
⇒ f(x) is continuous at x = 2 and x = 10
⇒ (LHL of f(x) at x = 2) = (RHL of f(x) at x = 2) = f(x)
⇒ lim f (x) = lim f (x) = f (2)
…(i)
x " 2– x"2
+

Similarly, lim f (x) = lim f (x) = f (10) …(ii)


x " 10 – x " 10
+

lim f (x) = lim 5 = 5 [ a f(x) = 5 if x ≤ 2]


x " 2– x"2

lim f (x) = lim (ax + b) = 2a + b [ a f(x) = ax + b if x > 2]


x"2
+ x"2

f(2) = 5
Putting these values in (i), we get
2a + b = 5 …(iii)
Again lim f (x) = lim (ax + b) = 10a + b [ a f(x) = ax + b if x < 10]
x " 10 – x " 10 –1

lim f (x) = lim 21 = 21 [ a f(x) = 21 if x > 10]


+ +
x " 10 x " 10

f(10) = 21
Putting these values in (ii), we get
10a + b = 21 …(iv)
Subtracting (iii) from (iv), we get
10a + b = 21
– 2a ! b = – 5

8a = 16 ⇒ a=2
∴ b=5–2×2=1
Hence, the value of a = 2 and b = 1.

Continuity and Differentiability 169


@Cbsebookshub - Join Us on Telegram
7. Find the derivative of the function f(x) = sin(x2).
Sol.  f(x) = sin (x2)
∴ f'(x) = cos x2 . 2x = 2x cos x2
8. Prove that the function f given by f(x) = |x – 1|, x ∈ R is not differentiable at x = 1.

Sol. Here f(x) = |x – 1|= )


x–1 if x $ 1
1– x if x < 1
LHD at x =1
f (x) – f (1) f (1 – h) – f (1) 1 – (1 – h) – (1 – 1) h
lim = lim = lim = lim =–1
x"1 – x – 1 h"0 ( 1 – h – 1) h"0 –h h " 0 –h

RHD at x = 1
f (x) – f (1) f (1 + h) – f (1) (1 + h) –1– (1 –1) h
lim = lim = lim = lim = 1
x"1
+ x–1 h " 0 (1 + h – 1) h"0 h h"0 h

∴ LHD ≠ RHD

Thus, f is not differentiable at x = 1.
dy
9. Find , if x – y = p.
dx
dy
Sol. a x – y = p ⇒ y = x – p ⇒ =1
dx
dy 3x – x 3
if y = tan –1 f p, –
1 1
10. Find <x< .
dx 1 – 3x 2
3 3
3x – x 3
Sol. y = tan–1 f p , putting x = tan θ ⇒ θ =tan–1 x
1 – 3x 2
3 tan i – tan 3 i
y = tan–1 >
H = tan–1 [tan 3θ] = 3θ = 3 tan–1 x
1 – 3 tan 2 i
Differentiating both sides w.r.t x, we get
dy 1 3
= 3 × =
dx +
1 x 2
1 x2
+
dy 1 1 .
11. Find if y = sin –1 _ 2x 1 – x 2 i; – <x<
dx 2 2

Sol. Here y = sin–1 (2x 1 –x 2 ), putting x = sin θ ⇒ θ= sin–1 x.

y = sin–1 [2 sin i 1 – sin 2 i] = sin –1 [2 sinθ cosθ]


⇒ y = sin–1 (sin 2θ) = 2θ = 2 sin–1 x

Differentiating both sides w.r.t. x, we get
dy 1 2
= 2. =
dx 1– x 2
1 – x2
12. Differentiate the following with respect to x:
e x, x > 0
x
Sol. Let y = e
dy d d d 1 1

= ( e x)× e x× x = × e x×
dx de x d x dx 2 e x 2 x
x
dy e
=
dx 4 xe x

170 Xam idea Mathematics–XII

@Cbsebookshub - Join Us on Telegram


Aliter : Let y = e
x
put u = x and v = eu
dy 1 1
y= v & dv = =
2 v 2 e x

dv
v = eu & du
= eu = e x

du 1
u= x & =
dx 2 x
dy dy dv du 1 1 1 e x
e x
= . . = .e x . = . =
dx dv du dx x 2 x 4 x x x
2 e e 4 xe
1
1 x dx + n
13. Differentiate with respect to x: y = d x + n +x x .
x
1
1 x dx + n
Sol. Let u = dx +
n and v = x x
x
dy du dv
then y = u + v & = + ...(i)
dx dx dx
1 x
Now, u = d x + n
x
1 x 1
log u = log d x + n = x log d x + n [Taking logarithm on both sides]
x x
Differentiating both sides, w.r.t x, we get
1 du 1 1
u dx = 1. log d x + x n + x.
1
.e1 – o
1 x2
x+
x
x 2
1 x x2 – 1 = d + 1 n > d + 1 n + x – 1 H
= u >log d x + n + x. 2
du
. H x
x
log x
x 2 ...(ii)
dx x x +1 x2 x +1
1
+
Now, v = (x) x x

1
= dx +
n log x
1
+
log v = log (x) x x [Taking logarithm on both sides]
x
Differentiating both sides w.r.t. x, we get
1
dx + n x – 1
2
1 dv 1 1 1 dv x2 + 1
= e 1 – 2 o log x + d x + n . ⇒ = x x > 2 log x + H ...(iii)
v dx x x x dx x x2
From (i), (ii) and (iii), we get
1
1 x x 2 –1 dx + n x – 1
2
dy 1 x2 + 1
= d x + n >log d x + n + 2 H + x x > 2 log x + H
dx x x x +1 x x2
dy
14. If y = xsin x + (sin x)cos x, find . [CBSE Delhi 2009, (F) 2013]
dx
OR
dy
If y = xcos x + (cos x)sin x, find . [CBSE 2019 (65/4/1)]
dx
Sol. In y = xsin x + (sin x)cos x, let u = xsin x, v = (sin x)cos x
Now, y = u + v
dy du dv
and = + …(i)
dx dx dx

Continuity and Differentiability 171


@Cbsebookshub - Join Us on Telegram
We have, u = xsin x
Taking log on both sides, we get
log u = sin x log x
Differentiating both sides with respect to x, we get
1 du 1
u = sin x # + log x. cos x
dx x

= ud + log x. cos x n = x sinx < + log x. cos xF


du sin x sin x
x x ...(ii)
dx
Again, v = (sin x)cos x
Taking log on both sides with respect to x, we get
log v = cos x log sin x
Differentiating both sides with respect to x, we get
1 dv 1
v = cos x # # cos x + log sin x (– sin x)
dx sin x
= cos x . cot x + log sin x . (– sin x) = cos x . cot x – sin x . log sin x
dv
∴ = v [cos x. cot x – sin x. log sin x]
dx
dv
= (sin x) cosx [cos x. cot x – sin x log sin x] ...(iii)
dx
From (i), (ii) and (iii), we get
dy
= x sin x < + log x. cos xF + (sin x) cos x [cos x cot x – sin x log sin x]
sin x
x
dx
OR
Solution is similar only values change.
cos x
Ans. x cos x d – sin x log x n + (cos x) sin x [– sin x tan x + cos x log cos x]
x
15. Differentiate w.r.t x : y = (x cos x)x + (x sin x)1/x
Sol. We have y = (x cos x)x + (x sin x)1/x
1 1
x log (x sin x)
y = e loge(x cos x) + e loge(x sin x) y = e x log (x cos x) + e x
x

1
dy d log x sin x d 1

= e x log (x cos x) . [x log (x cos x)] + e x . d log (x sin x) n
dx dx dx x
dy 1
⇒ = (x cos x) x <1. log (x cos x) + x. (1. cos x – x sin x)F
dx x cos x
–1 1 1
+ (x sin x ) = 2 log (x sin x) + x . x sin x . (1. sin x + x cos x)G
1/x
x
x cot x + 1 – log (x sin x)
= (x cos x) x [log (x cos x) + 1 – x tan x] + (x sin x) 1/x = G
dy

dx x2
dy sin 3 t cos 3 t
16. Find if x = ,y = .
dx cos 2t cos 2t
sin 3 t
Sol. Given x =
cos 2t
Differentiating both sides w.r.t. t, we have
–2 sin 2t
3 cos 2t (3 sin 2 t cos t) – sin 3 t e o
dx d sin t
= f p=
2 cos 2t


dt dt cos 2t cos 2t

172 Xam idea Mathematics–XII

@Cbsebookshub - Join Us on Telegram


2 3
= 3 (cos 2t) sin t cos t + sin 2t sin t
(cos 2t) 3/2
3 (1 – 2 sin 2 t) sin 2 t cos t + 2 sin t cos t. sin 3 t 3 sin 2 t cos t – 4 sin 4 t cos t
= 3/2
=
(cos 2t) (cos 2t) 3/2
sin t cos t (3 sin t – 4 sin 3 t) sin t cos t (sin 3t) sin t cos t sin 3t
= = =
(cos 2t) 3/2 (cos 2t) 3/2 (cos 2t) 3/2
–2 sin 2t
cos 2t (–3 cos 2 t sin t) – cos 3 t e o
dy 3 2 cos 2t
f p=
d cos t
Now, =
dt dt cos 2t cos 2t
–3 (cos 2t) cos 2 t sin t + cos 3 t sin 2t
=
(cos 2t) 3/2
–3 (2 cos 2 t – 1) cos 2 t sin t + cos 3 t (2 sin t cos t)
=
(cos 2t) 3/2
–6 cos 4 t sin t + 3 cos 2 t sin t + 2 cos 4 t sin t 3 cos 2 t sin t – 4 cos 4 t sin t
= =
(cos 2t) 3/2 (cos 2t) 3/2
sin t cos t (3 cos t – 4 cos 3 t) sin t cos t (– cos 3t)
= 3/2
=
(cos 2t) (cos 2t) 3/2
dy dy dt sin t cos t (cos 3t) (cos 2t) 3/2

= × =– × = – cot 3t
dx dt dx (cos 2t) 3/2 sin t cos t sin 3t
–1
t –1 dy y
17. If x = asin and y = acos t , show that =–
x . [CBSE (AI) 2012]
dx
sin –1 t
Sol. x = a ...(i)
–1
t
y = a cos
...(ii)
Multiplying, (i) and (ii) we get
–1 –1 –1
t t t + cos –1 t
x.y = a sin # a cos ⇒ x.y = a sin

x.y = a r/2 [sin–1t + cos–1t = π/2]
On differentiating both sides, we get
dy dy –y
x + y # 1 = 0 ⇒ =
dx dx x
2
d y dy
18. If y = Aemx + Benx, then show that – ( m + n) + mny = 0 [CBSE (AI) 2007; (F) 2013]
dx 2 dx
Sol. Given, y = Aemx + Benx
On differentiating with respect to x, we have
dy
= Ame mx + Bne nx
dx
Again, differentiating with respect to x, we have
d2 y
2
= Am 2 e mx + Bn 2 e nx
dx
d2 y dy
Now, LHS = 2 – (m + n) + mny
dx dx
= Am2emx + Bn2enx – (m + n)(Amemx + Bnenx) + mn(Aemx + Benx)
= Am2emx + Bn2enx – Am2emx – Amnemx – Bmnenx – Bn2enx + Amnemx + Bmnenx
= 0 = RHS

Continuity and Differentiability 173


@Cbsebookshub - Join Us on Telegram
19. If y = (tan–1 x)2, show that (x 2 + 1) 2 y2 + 2x (x 2 + 1) y1 = 2. [CBSE (AI) 2012; Delhi 2012]
Sol. We have, y = (tan–1 x)2 ...(i)
Differentiating with respect to x, we get
dy 1
= 2 tan –1 x. ...(ii)
dx 1 + x2
dy
or (1 + x2) y1 = 2tan–1 x >where y1 = H
dx
Again differentiating with respect to x, we get
dy1 d 1
(1 + x 2) . + y1 (1 + x 2) = 2.
dx dx 1 x2
+
2
⇒ (1 + x 2) .y2 + y1 .2x =
1 + x2
d2 y
>where y2 = H
dy
or (1 + x2)2 y2 + 2x (1 + x2) y1 = 2 and y1 =
dx 2 dx
20. Verify mean value theorem if f(x) = x2 – 4x – 3 in the interval [a, b], where a = 1 and b = 4.
Sol. Given, f(x) = x2 – 4x – 3, x ∈ [a, b] i.e., [1, 4]
Since f(x) is a polynomial so it is differentiable everywhere and so it is also continuous.
So, all conditions of mean value theorem are satisfied.
Now, f(a) = f(1) = (1)2 – 4 (1) – 3 = – 6
f(b) = f(4) = (4)2 – 4 × 4 – 3 = – 3
Now, for any c ∈ (a, b), we have
f (b) – f (a) f ( 4) – f ( 1)
f l (c) =
b–a
& f l (c) =
4–1
–3 – (–6) –3 + 6
⇒ 2c – 4 =
3
& 2c – 4 =
3
=1

5
⇒ 2c = 5 & c = ! (1, 4)
2
1 + sin x + 1 – sin x dy
21. If y = cot –1 > H, 0 < x <
r
then find the value of .
1 + sin x – 1 – sin x 2 dx
[CBSE Delhi 2008; (F) 2013]

1 + sin x + 1 – sin x r
Sol. Consider ,0<x<
1 + sin x – 1 – sin x 2

1 + sin x + 1– sin x 1 + sin x + 1– sin x


= ×
1 + sin x – 1 + sin x 1 + sin x + 1– sin x
^ 1 + sin x + 1– sin x h
2
1 + sin x + 1– sin x + 2 1– sin 2 x
= =
1 + sin x– (1– sin x) 1 + sin x – 1 + sin x
2 x
2 cos
x cot c 2 m
2 + 2 cos x 1 + cos x 2 x
= = = =
2 sin x sin x x
2 sin cos
2 2

1 + sin x + 1 – sin x dy
∴ y = cot –1 > H = cot –1 d cot n =
x x x 1

2 2
` y=
2
& dx
=
2
1 + sin x – 1 – sin x

174 Xam idea Mathematics–XII

@Cbsebookshub - Join Us on Telegram


Z] 1
] x sin d n, if x ! 0
]] 2
22. Determine if f defined by f(x) = ][ x is a continuous function. [NCERT Exemplar]
]]
] 0 , if x = 0
\
Sol. We note that domain of f is R. Let c be any real number then two cases arise.
Case I: If c ≠ 0 then

∴ lim f (x) = lim x 2 sin x = c 2 sin b c l = f (c)
1 1

x"c x"c

⇒ f is continuous for c ∈R, where c ≠ 0.


Case II : If c = 0, then f (c) = f (0) = 0

and lim x 2 sin b x l = 0 × (a number oscillating between –1 and 1)


1

x"0
= 0 = f(0)
⇒ f is continuous at x = 0.
Thus, f is a continuous function at every point of its domain.
d n
23. Using mathematical induction, prove that (x ) = nx n – 1 , for all positive integer n. [HOTS]
dx
Sol. Let P(n) be statement such that
d n
P (n) : (x ) = nx n – 1
dx
For P(1): Putting n = 1, we get
d 1
(x ) = 1.x1 – 1 = 1 ⇒ P(1) is true.
dx
d m
Let P(m) be true ⇒ (x ) = mx m – 1
dx
Now for P(m + 1):
d m+1 d m d
(x )= (x . x) = x m . 1 + x. (x m)
dx dx dx
= xm + x . m . xm – 1 = xm + m . xm = (m + 1) . xm = (m + 1) . x(m + 1) – 1
⇒ P (m + 1) is true

Here, by PMI P(n) is true.
d n
i.e., ( x ) = n. x n – 1
dx
f (x) g (x) h (x) dy f l (x) g l (x) hl (x)
24. If y = l m n , then prove that = l m n . [HOTS]
dx
a b c a b c
f (x) g (x) h (x)
Sol. Given, y = l m n
a b c
y = (mc – nb) f(x) – (lc – na) g(x) + (lb – ma) h(x)
dy
⇒ = (mc – nb) f l (x) – (lc – na) gl (x) + (lb – ma) hl (x)
dx
f l (x) gl (x) hl (x)
= l m n
a b c

Continuity and Differentiability 175


@Cbsebookshub - Join Us on Telegram
Multiple Choice Questions [1 mark]
Choose and write the correct option in the following questions.
1. The function f : R → R given by f(x) = – |x – 1| is [CBSE 2020 (65/2/1)]
(a) continuous as well as differentiable at x = 1
(b) not continuous but differentiable at x = 1
(c) continuous but not differentiable at x = 1
(d) neither continuous nor differentiable at x = 1
2. The function f (x) = e x is [NCERT Exemplar]
(a) continuous everywhere but not differentiable at x = 0
(b) continuous and differentiable everywhere
(c) not continuous at x = 0
(d) none of these
3. The function f(x) = [x], where [x] denotes the greatest integer function, is continuous at
(a) 4 (b) –2 (c) 1 (d) 1.5
1
4. The number of points at which the function f (x) = is not continuous is
x – [x]
[NCERT Exemplar]
(a) 1 (b) 2 (c) 3 (d) none of these
Z] sin x
]] + cos x, if x ! 0
5. The function f (x) = ][ x is continuous at x = 0, then the value of k is
]]
k , if x = 0
\ [NCERT Exemplar]
(a) 3 (b) 2 (c) 1 (d) 1.5
]Z] 1
]sin , if x ! 0
6. The value of k which makes the function defined by f (x) = ][ x , continuous at
]]
k , if x =0
x = 0 is \
(a) 8 (b) 1 (c) –1 (d) none of these
7. The function f(x) = cot x is discontinuous on the set [NCERT Exemplar]
(a) {x = n π : n ∈ Z} (b) {x = 2 nπ : n ∈ Z}

(c) ' x = (2n + 1) ; n d Z 1 (d) ' x = ; n d Z1


r nr
2 2
8. Let f (x) = sin x . Then [NCERT Exemplar]
(a) f is everywhere differentiable
(b) f is everywhere continuos but not differentiable at x = nπ : n ∈ Z
r
(c) f is everywhere continuous but not differentiable at x = (2n + 1) ,n∈Z
2
(d) none of these
x–1
9. The function f (x) = is discontinuous at [CBSE 2020 (65/2/2)]
x ( x 2 – 1)
(a) exactly one point (b) exactly two points
(c) exactly three points (d) no point
1
10. If f (x) = x 2 sin , where x ! 0 , then the value of the function f at x = 0, so that the function is
x
continuous at x = 0, is [NCERT Exemplar]
(a) 0 (b) –1 (c) 1 (d) None of these

176 Xam idea Mathematics–XII

@Cbsebookshub - Join Us on Telegram


11. The function f(x) = |x| + |x – 1| is
(a) continuous at x = 0 as well as at x = 1. (b) continuous at x = 1 but not at x = 0.
(c) discontinuous at x = 0 as well as at x = 1. (d) continuous at x = 0 but not at x = 1.
4 – x2
12. The function f (x) = is
4x – x 3
(a) discontinuous at only one point (b) discontinuous at exactly two points
(c) discontinuous at exactly three points (d) none of these
13. The value of c in Rolle’s Theorem for the function f(x) = ex sin x, in [0, π] is [NCERT Exemplar]
r r r 3r
(a) (b) (c) (d)
6 4 2 4
14. The value of c in Mean Value Theorem for the function f(x) = x(x – 2), x ∈ [1, 2] is
3 2 1 7
(a) (b) (c) (d)
2 3 2 4
15. The value of c in Rolle’s theorem for the function f(x) = x3 – 3x in the interval [0, 3 ] is
3 1
(a) 1 (b) –1 (c) (d)
2 3
16. The set of points where the functions f given by f (x) = |x – 3| cos x is differentiable is
(a) R (b) R – {3} (c) (0, ∞) (d) none of these

17. Differential coefficient of sec (tan–1x) w.r.t. x is


[NCERT Exemplar]
x x 2 1
(a) (b) 2 (c) x 1 + x (d)
+
1 x 2 1 + x 1 + x2
2x 2x
18. If u = sin –1 d n and v = tan –1 d n , then
du
is
1 + x2 1 – x2 dv
1 1 – x2
(a) (b) x (c) {4, –4}, f (d) 1
2 1 + x2
dy r
19. If y = log tan x , then the value of at x = is
dx 4
1
(a) 0 (b) 1 (c) (d) ∞
2
dy
20. If y = sin x + y , then is equal to
dx
cos x cos x sin x sin x
(a) (b) (c) (d)
2y – 1 1 – 2y 1 – 2y 2y – 1

Answers
1. (c) 2. (a) 3. (d) 4. (d) 5. (b) 6. (d)
7. (a) 8. (b) 9. (c) 10. (a) 11. (a) 12. (c)
13. (d) 14. (a) 15. (a) 16. (b) 17. (a) 18. (d)
19. (b) 20. (a)

Solutions of Selected Multiple Choice Questions


1. We have,

f (x) = – x – 1 = )
x – 1, if x # 1
– (x – 1), if x > 1
At x = 1
LHL = lim f (1 – h) = lim (1 – h) – 1 = 0
h"0 h"0

Continuity and Differentiability 177


@Cbsebookshub - Join Us on Telegram
RHL = lim f (1 + h) = lim – (1 + h – 1) = 0
h"0 h"0
f(1) = 1 – 1 = 0
∴ LHL = RHL = f(0)  ⇒  f(x) is continuous every where.

Now, at x = 1
d d
LHD = (x – 1) = 1 ; RHD = {– (x – 1)} = – 1
dx dx
LHD ≠ RHD
∴ f(x) is not differentiable of x = 1.
1
6. Indeed lim sin does not exist.
x"0 x
7. We know that, f(x) = cot x is continuous in R – {n π : n ∈ Z}.
cos x
Since, f (x) = cot x = [since, sin x = 0 at {nπ, n ∈ Z}]
sin x
Hence, f(x) = cot x is discontinuous on the set {x = nπ : n ∈ Z}.
9. We have,
x–1
f (x) =
x (x 2 – 1)
∴ f(x) is discontinuous when x(x2 – 1) = 0

⇒ x = 0, x = ± 1

∴ f(x) is discontinuous at x = 0, –1, 1

i.e., exactly at three points.

10. a f (x) = x 2 sin c m , where x ≠ 0; a


1
lim f (x) = 0
x x"0

Hence, value of the function f at x = 0, so that it is continuous at x = 0 is 0.


15.  f l (c) = 0 [ f l (x) = 3x2 – 3]
3

3c2 – 3 = 0 ⇒ c2 = = 1
3

c = ! 1, where 1 d (0, 3 ) ⇒ c = 1
19. We have, y = log tan x
dy 1 1

= # # sec 2 x
dx tan x 2 tan x
dy sec 2 x dy ( 2) 2 2

=   ⇒  = = =1
dx 2 tan x dx at x = r 2#1 2
4
1/2
20.  y = (sin x + y)

.c m
dy 1 d 1 1 dy
= (sin x + y) –1/2 . (sin x + y) = . +
dx 2 dx 2 (sin x + y) 1/2 cos x dx

c m
dy 1 dy
⇒ = + [ (sin x + y)1/2 = y]
dx 2y cos x dx
dy dy cos x 2y
d1 – n=
1 cos x cos x
⇒ ⇒ = . =
dx 2y 2y dx 2y 2y – 1 2y – 1

178 Xam idea Mathematics–XII

@Cbsebookshub - Join Us on Telegram


Fill in the Blanks [1 mark]
dy
1. If y = tan–1 x + cot–1 x, x ∈ R, then is equal to _____________ . [CBSE 2020 (65/3/1)]
dx
dy
2. If cos (xy) = k, where k is a constant and xy ≠ np, n ∈ Z, then is equal to _____________ .
dx
 [CBSE 2020 (65/3/1)]
3. The number of points of discontinuity of f defined by f(x) = |x| – |x + 1| is _____________ .
 [CBSE 2020 (65/4/1)]
2 – x2
4. The function f (x) = is discontinuous exactly at _____________ points.
9x – x 3
Answers
–y
1. 0 2. x 3. 0 4. three

Solutions of Selected Fill in the Blanks


1. We have, y = tan–1 x + cot–1 x, x ∈ R
r
⇒ y =
2

dy d c 2 m
r
∴ = =0
dx dx
2. We have, cos (xy) = k
Diff. w.r.t x, we get
dy
– sin (xy) # ) x + y3 = 0
dx
dy a xy ! nr
⇒ x
+ y = 0 f p
dx ` sin (xy) ! 0
dy –y

=
dx x

Very Short Answer Questions [1 mark]


Z] 2
]] x – 9
, x!3
1. If the function f defined as f ^ x h = [] x – 3
]
is continuous at x = 3, find the value of k.
]]
] k, x=3
[CBSE \ 2020 (65/5/1)]
Sol. It is given that f(x) is continuous at x = 3
∴ lim f (x) = f (3)
x"3

x2 – 9
⇒ lim
=k
x"3 x – 3
(x – 3) (x + 3)
⇒ lim
=k
x"3 (x – 3)
⇒ lim (x + 3) = k ⇒  3 + 3 = k

x"3

⇒  k = 6

Continuity and Differentiability 179


@Cbsebookshub - Join Us on Telegram
Z] sin 5x
]] + cos x, if x ! 0
2. For what value of ‘k’ is the function f ^ x h = ][ 3x
]
continuous at x = 0?
]]
] k, if x = 0
\ [CBSE (F) 2017]
Sol. lim f (x) = lim f (0 + h)
x "0
+ h"0

= lim f (h) = lim c + cos h m


sin 5h

h"0 h"0 3h
sin 5h 5 5
= lim × + lim cos h = 1× + 1 [ a h → 0 ⇒ 5h → 0]
h"0 5 h 3 h"0 3
8

lim f (x) =
x"0
+ 3
Also, f (0) = k
Since, f (x) is continuous at x = 0.
8

lim f (x) = f (0) ⇒ =k
x"0
+ 3
Z] kx
]]
, if x < 0
3. Determine value of the constant ‘k’ so that the function f(x) = [] x is continuous
]]
3 , if x $ 0
\
at x = 0. [CBSE Delhi 2017]
Sol.  f (x) is continuous at x = 0

lim f (x) = lim f (x) = f (0)
+
x "0 x " 0–

Now, lim f (x) = lim f (0 – h)


x " 0– h"0

k (–h) –kh
= lim f (–h) = lim = lim = –k
h"0 h"0 – h h"0 h

Also, f(0) = 3
 lim f (x) = f (0) ⇒ – k = 3
⇒ k=–3
x " 0–

dy
4. If y = 2 sec (e 2x) ; then find . [CBSE 2019 (65/5/3)]
dx
Sol. Given, y = 2 sec (e 2x)

_ 2 sec (e 2x) i = 2×
dy d 1

= × sec (e 2x) tan (e 2x) × 2e 2x
dx dx 2 sec (e 2x)
= 2 sec (e 2x) tan (e 2x) . e 2x = 2e 2x sec (e 2x) tan (e 2x)
dy
5. If y = cosec (cot x ), then find . [CBSE 2019 (65/4/2)]
dx
Sol. y = cosec (cot x )
dy d

= (cosec (cot x ))
dx dx
1
= – cosec (cot x ) cot (cot x ) × e – cosec ( x ) × o
2
2 x
cosec (cot x ) cot (cot x ) × cosec 2 ( x )
=
2 x

180 Xam idea Mathematics–XII

@Cbsebookshub - Join Us on Telegram


6. Find the derivative of log10 x with respect to x. [NCERT Exemplar]

Sol. Let y = log10 x = log10 e . loge x


dy 1 log10 e
=a log e x = x G
d 1

= log10 e × =
dx x x dx
d2 y
7. If y = 5e7x + 6e –7x , show that = 49y . [CBSE 2019 (65/5/1)]
dx 2
dy
Sol.  y = 5e7x + 6e–7x ⇒ = 35e7x – 42e –7x
dx
d2 y

= 245e7x + 294e –7x = 49 (5e7x + 6e –7x) = 49y
dx 2
dy
8. If y = log (cos ex), then find . [CBSE 2019 (65/4/1)]
dx
Sol.  y = log (cos ex)
dy d 1 –e x sin (e x)

= (log (cos e x)) = × (– sin ( e x
)) × e x
= = – e x tan (e x)
dx dx cos (e x) cos (e x)
dy

= – e x tan (e x)
dx

Short Answer Questions-I [2 marks]


dy 2r
1. Find at t = when x = 10(t – sin t) and y = 12 (1 – cos t). [CBSE (F) 2017]
dx 3
Sol. Given, x = 10 (t – sin t) and y = 12 (1 – cos t)
dx
 x = 10 (t – sin t)
⇒ = 10 (1 – cos t) (Differentiating w.r.t. t)
dt
dy
Again y = 12 (1 – cos t) ⇒ = 12 (0 + sin t) = 12 sin t (Differentiating w.r.t. t)
dt
dy
dy 12 sin t
Now, = dt =
dx dx 10 (1 – cos t)
dt

sin c r– m
2r r
12 sin
E
dy 3 6 3

a = = ×
10 c 1– cos m c 1 – cos c r – m m
dx t = 2r 2r 5 r
3
3 3

r 3
6 sin 6 6 3 2 3
= × 3 = × 2 = =
5 r 5 1 5×3 5
1 + cos 1+
3 2
dy r
2. Find at x = 1, y = if sin2 y + cos xy = K . [CBSE Delhi 2017]
dx 4
Sol. sin2 y + cos xy = K
Differentiating w.r.t. x, we get
dy dy dy dy
2 sin y. cos y + (– sin xy) (x. + y) = 0 ⇒ sin 2y. – x sin xy. – y sin xy = 0
dx dx dx dx

Continuity and Differentiability 181


@Cbsebookshub - Join Us on Telegram
dy y sin xy

=
dx (sin 2y – x sin xy)
r r r 1
. sin .
dy 4
F 4 4 2 r

= = =
dx x = 1, y = r r r 1 4 ( 2 – 1)
4 sin – sin 1–
2 4 2
3. If (1 + x)n = C0 + C1x + C2x2 + .... + Cnxn, then prove that [HOTS]
(i) C1 + 2C2 + .... + nCn = n . 2n – 1 (ii) C1 – 2C2 + 3C3 – .... + (–1)n nCn = 0
Sol. We have, (1 + x)n = C0 + C1x + C2x2 + .... + Cnxn
Differentiating both sides with respect to x, we have
n(1 + x)n – 1 = C1 + 2C2x + 3C3x2 + .... + nCnxn – 1
Putting x = 1 and x = –1 successively, we have
(i) C1 + 2C2 + 3C3 + .... + nCn = n . 2n – 1 and (ii) C1 – 2C2 + 3C3 – .... + (–1)n nCn = 0
(cosx)...3 dy y 2 tan x
4. If y = (cos x) (cosx) , then show that = . [NCERT Exemplar]
dx y log cos x – 1
(cosx)...3
Sol. We have, y = (cos x) (cosx)

y = (cos x)y

log y = log (cos x)y ⇒ log y = ylog cos x
On differentiating w.r.t. x, we get
1 dy d dy 1 dy y d dy
$ = y$ log cos x + log cos x $ ⇒ $ = $ cos x + log cos x $
y dx dx dx y dx cos x dx dx
dy 1 –y sin x

< – log cos xF = = –y tan x
dx y cos x
dy –y 2 tan x y 2 tan x

= =
dx (1 – y log cos x) y log cos x – 1
d2 y
5. If x = a cos q ; y = b sin q, then find . [CBSE 2020 (65/5/1)]
dx 2
Sol. We have,
dx
x = a cos q ⇒  = – a sin i
di
dy
and y = b sin q ⇒  = b cos i
di
dy dy dx b cos i b

= = = – cot i
dx di di – a sin i a

Again diff. w.r.t x, we have


d2 y d dy
e o=–
b d b di
=
a (cot i) = – a # (– cosec 2 i)
dx 2 dx dx dx dx
1
= – a # (– cosec 2 i) # d – n
b
a sin i
d2 y b

=– cosec 3 i
dx 2 a2

182 Xam idea Mathematics–XII

@Cbsebookshub - Join Us on Telegram


Short Answer Questions-II [3 marks]
XX CONTINUITY AND DIFFERENTIABILITY
1. Find the values of p and q, for which [CBSE Delhi 2016]
Z]
]] 1 – sin 3 x r
]] , if x <
]] 3 cos 2 x 2
]] r r
f (x) = [] p , if x = is continuous at x = .
]] 2 2
]] q (1 – sin x) r
]] , if x >
]] 2
( r – 2x) 2
\
]]Z – 3
]] 1 sin x , if x < r
]] 3 cos 2 x 2
]]
] r r
Sol. We have, f (x) = ][ p , if x = is continuous at x = .
]] 2 2
]] q (1 – sin x) r
]]
]] ( r – 2x) 2 , if x > 2
\
+
Now, lim f (x) = lim f e + h o >Let x = + h, x " & h " 0H
r r r
r
+ h"0 2 2 2
x"
2

q )1 – sin e + h o3
r
2 q {1 – cos h} q (1 – cos h)
= lim 2
= lim 2
= lim
h"0 h"0 {r – r – 2h} h"0 4h 2
)r – 2 e + h o3
r
2
JK N2
q.2 sin 2
h KK sin h OOO
q. sin 2
h
2 K 2 OO 1 q
2
= lim = lim = q. lim KK OO # 8 = 8
h"0 4h 2 h"0 2h 2 h " 0K K h
K 2 OO
L P
r–
Again lim f (x) = lim f e – h o >Let x = – h, x " & h " 0H
r r
r– h"0 2 2 2
x"
2

1 – sin 3 e – ho
r
2 1 – cos 3 h (1 – cos h) (1 + cos h + cos 2 h)
= lim = lim = lim
h " 0 3 sin 2 h 3 sin 2 h
3 cos 2 e – ho
h"0 r h"0
2
h h h
2 sin 2
. (1 + 1 + 1) 2 sin 2 .3 2. sin 2
2 2 2
= lim = lim = lim
h"0 3 sin 2 h h " 0 3 sin 2 h h " 0 sin 2 h

Dividing Nr and Dr by h2, we get


2
KJK h ONO h
KK sin sin 2
h KK lim 2 OOO 2
sin 2 2. 2
2 h KK h h OO
2. 2 #4 K 2 " 0 2 OO
h 4 1 P =1
= lim = lim = L
h " 0 sin h2 h"0 2
sin h 2 2 2
e o
sin h
2 2 lim
h h h"0 h

Continuity and Differentiability 183


@Cbsebookshub - Join Us on Telegram
Also f e o=p
r
2

lim f (x) = lim f (x) = f e o


r r
a f(x) is continuous at x =

2 r
+
r– 2
h" h"
2 2
q 1 1

= = p ⇒ p = and q = 4
8 2 2
2. Show that the function f(x) = 2x – |x|is continuous but not differentiable at x = 0. [CBSE (F) 2013]
Sol. Here f(x) = 2x – |x|
For continuity at x = 0
lim f (x) = lim f (0 + h) = lim f (h)
lim f (x) = lim f (0 – h) = lim f (– h)
x"0
+ h"0 h"0 x " 0– h"0 h"0

= lim " 2h –| h | , = lim (2h – h) = lim " 2 (– h) –| – h | , = lim " – 2h – h ,


h"0 h"0 h"0 h"0
= lim (– 3h)
= lim h h"0
h"0
= 0 ...(i) = 0 ...(ii)
Also, f(0) = 2 × 0 – |0| = 0 ...(iii)
(i), (ii) and (iii)
& lim f (x) = lim f (x) = f (0)
+
x"0 x " 0–

Hence, f(x) is continuous at x = 0.


For differentiability at x = 0
f (0 – h) – f (0) f (– h) – f (0)
LHD = lim = lim
h"0 –h h"0 –h
(2 (– h) –| – h |) – {2 # 0 –| 0 |} – 2h – h – 0
= lim = lim
h"0 –h h"0 –h
– 3h
= lim = lim 3
h"0 – h h"0

LHD = 3 ...(iv)
f (0 + h) – f (0)
Again RHD = lim
h"0 h
f (h) – f (0) 2h – | h | – 2 # 0 – | 0 | 2h – h h
= lim = lim = lim = lim
h"0 h h"0 h h"0 h h"0 h

= lim 1
h"0

RHD = 1 ...(v)
From (iv) and (v), we get
LHD ≠ RHD i.e., function f(x) = 2x – |x| is not differentiable at x = 0.
Hence, f(x) is continuous but not differentiable at x = 0.
3. Find the value of ‘a’ for which the function f defined as
]Z] r
]]a sin (x + 1), x # 0
] 2
f (x) = ][ tan x – sin x is continuous at x = 0. [CBSE Delhi 2011; (South) 2016]
]] , x>0
]] x 3
\
Sol. a f(x) is continuous at x = 0.
⇒ (LHL of f(x) at x = 0) = (RHL of f(x) at x = 0) = f(0)

184 Xam idea Mathematics–XII

@Cbsebookshub - Join Us on Telegram


⇒ lim f (x) = lim f (x) = f (0) …(i)

x " 0– x"0
+

=a f (x) = a sin (x + 1), if x # 0G


r r
Now, lim f (x) = lim a sin (x + 1)
x " 0– x"0
+ 2 2

= lim a sin d + x n = lim a cos x = a. cos 0 = a


r r r
…(ii)
x"0
+ 2 2 x"0
+ 2

>a f (x) = , if x > 0H


tan x – sin x tan x – sin x
Again, lim f (x) = lim 3

x"0
+ x"0 x x3
sin x
cos x – sin x sin x – sin x . cos x sin x (1 – cos x)
= lim 3
= lim 3
= lim
x"0 x x"0 cos x.x x"0 cos x.x 3
x
2 sin 2
=a 1 – cos x = 2 sin 2 G
1 sin x 2 x
= lim cos x . lim x . 2
x"0 x"0 x 2
#4
4
JK N2 JK x NO
KK sin x OOO KK sin OO
1 1 K 2O 1 K 2O 1 1
= .1. lim KK x OO = . KK lim x OO = # 1 = ...(iii)
1 2 x " 0K O 2 Kx O 2 2
KK 2 OO KK 2 " 0 2 OO
L P L P
r r
Also, f (0) = a sin (0 + 1) = a sin = a ...(iv)
2 2
 f is continuous at x = 0.
1
∴ (i), (ii), (iii) and (iv) ⇒ a=
2
Z]
]] sin(a + 1) x + 2 sin x
]] x , x<0
]
4. If f (x) ]= [ 2 , x = 0 is continuous at x = 0, then find the values of a and b.
]]
]] +
1 bx – 1
] x , x>0
[CBSE (North) 2016]
\
Z]
]] sin (a + 1) x + 2 sin x
]] x , x<0
]
Sol. We have, f (x) = [] 2 , x = 0 is continuous at x = 0
]]
]] 1 + bx – 1
] x , x>0
\
Since, f(x) is continuous at x = 0

lim f (x) = lim f (x) = f (0) … (i)
+
x"0 x " 0–

Now, lim f (x) = lim f (0 + h) [Let x = 0 + h, h is +ve small quantity x → 0+ ⇒ h → 0]


x"0
+ h"0

1 + bh – 1 1 + bh – 1 1 + bh + 1
= lim f (h) = lim
= lim #
h"0 h"0 h h"0 h 1 + bh + 1
1 + bh – 1 bh b b
= lim = lim = lim =
h " 0 h ( 1 + bh + 1) h " 0 h ( 1 + bh + 1) h " 0 1 + bh + 1 2
Again lim f (x) = lim f (0 – h) [Let x = 0 – h, h is +ve small quantity x → 0– ⇒ h → 0]
x " 0– h"0

sin (a + 1) (– h) + 2 sin (– h)
= lim f (– h) = lim
h"0 h"0 –h
+ sin (a + 1) h 2 sin h
H = lim > H
– sin (a 1) h – 2 sin h
= lim > +
h"0 –h h"0 h h

Continuity and Differentiability 185


@Cbsebookshub - Join Us on Telegram
sin (a + 1) h sin h sin (a + 1) h sin h
= lim + 2 lim = lim # (a + 1) + 2 lim
h"0 h h"0 h h"0 ( a + 1) h h"0 h

= 1 × (a + 1) + 2 = a + 3
Also f(0) = 2
b
Now from (i) = a+3 = 2 ⇒ b = 4, a = – 1
2
5. Show that the function f(x) = |x – 3|, x ∈ , is continuous but not differentiable at x = 3.
[CBSE Delhi 2013]
]]Z – (x – 3) , x < 3
]]
Sol. Here, f(x) = |x – 3| & f (x) = [] 0 ,x = 3
]]
] (x – 3) , x > 3
\
For Continuity:
Now, lim f (x) = lim f (3 + h) [Let x = 3 + h and x → 3+ ⇒ h → 0]
x"3
+ h"0

= lim (3 + h – 3) = lim h = 0
h"0 h"0
lim f (x) = 0 ...(i)
+
x"3
lim f (x) = lim f (3 – h) [Let x = 3 – h and x → 3– ⇒ h → 0]
x"3 – h"0

= lim – (3 – h – 3) = lim h = 0
h"0 h"0
lim f (x) = 0 ...(ii)
+
x"3
Also, f(3) = 0 ...(iii)
From equation (i), (ii) and (iii), we get
lim f (x) = lim f (x) = f (3)
+
x"3 x " 3–

Hence, f(x) is continuous at x = 3.


For Differentiability:
f (3 + h) – f (3) (3 + h – 3) – 0 h
RHD = lim = lim = lim = lim 1 = 1 ...(iv)
h"0 h h"0 h h"0 h h"0
f (3 – h) – f (3) – (3 – h – 3) – 0 h
LHD = lim = lim = lim = lim (– 1) = –1 ...(v)
h"0 –h h"0 –h h"0 – h h"0

Equation (iv) and (v)



RHD ≠ LHD at x = 3.
Hence, f(x) is not differentiable at x = 3.
Therefore, f(x) = |x – 3|, x ∈ is continuous but not differentiable at x = 3.
6. Discuss the continuity and differentiability of the function
f(x) = |x| + |x – 1| in the interval (–1, 2). [CBSE Ajmer 2015]
Sol. Given function is f(x) = |x| + |x – 1|
Function is also written as
]Z] – x – (x – 1), if – 1 < x < 0 Z]
]] ]] – 2x + 1, if x < 0
]
f (x) =[] 1, if 0 # x < 1 ⇒ f (x) =[] 1, if 0 # x < 1
]] ]]
] x + (x – 1), if x $ 1 ] 2x– 1, if x $ 1
\ \
Obviously, in given function we need to discuss the continuity and differentiability of the function
f(x) at x = 0 or 1 only.

186 Xam idea Mathematics–XII

@Cbsebookshub - Join Us on Telegram


For continuity at x = 0
lim f (x) = lim f (0 + h) [Let x = 0 + h and x → 0+ ⇒ h → 0]
x"0
+ h"0

= lim f (h) = lim 1 [ a h is very small positive quantity]


h"0 h"0

= 1 …(i)

lim f (x) = lim f (0 – h) [Let x = 0 – h and x → 0 ⇒ h → 0]
x " 0– h"0

= lim f (– h) = lim {– 2 (– h) + 1} = lim (2h + 1)


h"0 h"0 h"0

lim f (x) = 1 ...(ii)


x " 0–
Also, f(0) = 1 …(iii)
(i), (ii) and (iii) & lim f (x) = lim f (x) = f (0)
+
x"0 x " 0–

Hence, f(x) is continuous at x = 0.


For differentiability at x = 0
f (0 + h) – f (0)
RHD = lim
h"0 h
f (h) – f (0)
= lim [ a h is very small positive quantity]
h"0 h
1–1 0
= lim = lim = lim 0 [ a |h|= h, |0|= 0]
h"0 h h"0 h h"0

RHD = 0 …(iv)
f (0 – h) – f (0) f (– h) – f (0)
LHD = lim = lim
h"0 –h h"0 –h
–2 (– h) + 1 – 1 2h
= lim = lim = lim (– 2)
h"0 –h h " 0 –h h"0

LHD = – 2 …(v)
(iv) and (v) ⇒ RHD ≠ LHD at x = 0.
Hence, f(x) is not differentiable at x = 0 but continuous at x = 0.
Similarly, we can prove f(x) is not differentiable at x = 1 but continuous at x = 1. (Do yourself)
ax 2 + b, if x < 1
7. Find ‘a’ and ‘b’, if the function given by f (x) = * is differentiable at x = 1.
2x + 1, if x $ 1
[CBSE Sample Paper 2018]
Sol. Since, f is differentiable at 1 ⇒ f is also continuous at 1.
Now lim f ^ x h = lim f ^1 + h h [Here h is +ve and very small quantity]
x"1
+ h"0

= lim 2 ^1 + hh + 1 = 2 + 1 = 3
h"0

lim f ^ x h = lim f ^1 – h h = lim # a ^1 – h h + b - = a + b


2

x"1 – h"0 h"0


Since f (x) is continuous at x = 1

a + b = 3 …(i)
Again, since f is differentiable
f ^1 – h h – f ^1 h f ^1 + h h – f ^1 h
⇒ LHD (at x = 1) = RHD (at x = 1)
⇒ lim = lim
h"0 –h h"0 h

Continuity and Differentiability 187


@Cbsebookshub - Join Us on Telegram
a ^1 – h h + b – 3 2 ^1 + h h + 1 – 3
2
a – 2ah + ah 2 + b – 3 2 + 2h + 1 – 3
⇒ lim
= lim ⇒ lim = lim
h"0 –h h"0 h h"0 –h h"0 h
–2ah + ah 2 + ^ a + b h – 3 2h –2ah + ah 2 + 3 – 3
⇒ lim
= lim ⇒ lim =2
h"0 –h h"0 h h"0 –h
ah ^ 2 – h h
⇒ lim
= 2 ⇒ 2a = 2 ⇒ a = 1 ⇒ b = 2 [From equation (i)]
h"0 h

XX DERIVATIVES
dy x + y
1. If tan –1 cm = log x 2 + y 2 , prove that
y
= . [CBSE 2020 (65/2/1)]
x dx x – y
y
Sol. Given, tan –1 d n = log x 2 + y 2
x
y
⇒ tan –1 d n = log (x 2 + y 2)
1

x 2
Differentiating w.r.t x, we have
Z] dy _b
]] b
– y # 1 bb 1 dy
# e 2x + 2y o
1 ]x 1
⇒ # [
]] dx `b = # 2
bbb 2 x + y
2 2 dx
y
1 +d n ]
] x2
x \ a
dy
ex – yo
2 dx dy
e + o
x 1
⇒ 2
# = 2 x y dx
x + y2 x 2 2
x +y
dy dy
⇒ x
– y=x+y
dx dx
dy dy x+y
⇒ (x – y)
= x + y   ⇒ =
dx dx x–y
3
3x – x 1 x
2. Differentiate tan –1 2
, x < w.r.t. tan –1 . [CBSE 2019 (65/5/1)]
1 – 3x 3 1 – x2
3
3x – x 3 3 tan i – tan i
Sol. Let y = tan –1 = tan –1 f p put x = tan q
1 – 3x 2
1 – 3 tan 2 i
y = tan –1 (tan 3i) = 3i = 3 tan –1 x


y = 3 tan –1 x

sin i
= tan –1 f p put x = sin i
x
and let t = tan –1
1 – x2 1 – sin 2 i
sin i
= tan –1 d n = tan –1 (tan i) = i
cos i
t = sin–1 x

3d tan –1 x 3#1
dy d 3 tan x –1
dx 1 + x2 1 – x2

= = = = 3
dt d sin –1 x d sin –1 x 1 1 + x2
dx 1 – x2

188 Xam idea Mathematics–XII

@Cbsebookshub - Join Us on Telegram


x y–x
dy (1 + log y) 2
3. If y = e , then prove that = . [CBSE (AI) 2013]
dx log y
Sol. Given, yx = ey – x
RS V
Taking logarithm both sides, we get log yx = log ey – x SSa (i) log e mn = log e m + log e nWWW
SS WW
⇒ x.log y = (y – x) . log e
⇒ x . log y = (y – x) SS m W
( ii ) log e n
= log m – log n
e W
SS e WW
y SS n WW
⇒ x (1 + log y) = y
⇒ x= SS (iii) log e m = n log e m WW
+
1 log y SS WW
S (iv) log e = 1 W
Differentiating both sides with respect to y, we get T X

(1 + log y) .1 – y. d 0 + y n 1 + log y – 1
1
dx log y
= = =
dy (1 + log y) 2 (1 + log y) 2 (1 + log y) 2

dy (1 + log y) 2

=
dx log y
dy
4. If (cos x)y = (cos y)x, then find . [CBSE Delhi 2012]
dx
Sol. Given, (cos x)y = (cos y)x
Taking logarithm both sides, we get log (cos x)y = log (cos y)x
⇒ y . log (cos x) = x . log (cos y)
[ a log mn = n log m]
Differentiating both sides with respect to x, we get
1 dy 1 dy
y. cos x (– sin x) + log (cos x) . = x.
cos y . (– sin y) . + log (cos y)
dx dx
y sin x dy x sin y dy
⇒ – cos x + log (cos x) .
=– +
cos y . dx log (cos y)
dx
dy x sin y dy y sin x

log (cos x) . + . = log (cos y) +
dx cos y dx cos x
dy x sin y y sin x

=log (cos x) + cos y G = log (cos y) + cos x
dx
y sin x
dy log (cos y) + cos x log (cos y) + y tan x

= =
dx x sin y log (cos x) + x tan y
log (cos x) + cos y

dy r
5. Find the value of at i = , if x = aeq(sin q – cos q) and y = aeq(sin q + cos q)
dx 4
[CBSE (AI) 2008, 2014]
Sol. Given, x = aeq(sin q – cos q) and y = aeq(sin q + cos q)
Taking x = aeq(sin q – cos q)
Differentiating with respect to q, we get
dx
= aei (cos i + sin i) + a (sin i – cos i) .ei = aei (cos i + sin i + sin i – cos i)
di
= 2 aeqsin q … (i)
Again, y = aeq(sin q + cos q)

Continuity and Differentiability 189


@Cbsebookshub - Join Us on Telegram
Differentiating with respect to q, we get
dy
= aei (cos i – sin i) + a (sin i + cos i) .ei = aei (cos i – sin i + sin i + cos i)
di
= 2 aeq. cos q … (ii)
dy
dy di 2aei . cos i
∴ = = [From (i) and (ii)]
dx dx 2aei . sin i
di
dy dy
H
r
⇒ = cot i ⇒ = cot = 1
dx dx i = r 4
4
6. Differentiate the following with respect to x: (sin x)x + (cos x)sin x [CBSE (F) 2013]
x sin x
Sol. Let u = (sin x) and v = (cos x)
∴ Given differential equation becomes y = u + v
dy du dv
⇒ = + ....(i)
dx dx dx
Now, u = (sin x)x
Taking log on both sides, we get
log u = x log sin x
Differentiating with respect to x, we get
1 du 1 du
u .
dx
=x
sin x
. cos x + log sin x & dx
= u (x cot x + log sin x)

du

= (sin x) x {x cot x + log sin x} ....(ii)
dx
Again v = (cos x)sin x
Taking log on both sides, we get
log v = sin x . log cos x
Differentiating both sides with respect to x, we get
1 dv 1
= sin x. +
v .
dx cos x (– sin x) log (cos x) . cos x

sin 2 x sin 2 x
= v )– + cos x. log cos x 3 = (cos x) sinx )cos x. log (cos x) –
cos x 3
dv

cos x
dx
+ sinx
= (cos x) 1 {log (cos x) – tan 2 x} ....(iii)

From (i), (ii) and (iii), we get


dy
= (sin x) x {x cot x + log sin x} + (cos x) 1 + sin x {log (cos x) – tan 2 x}
dx
dy cos 2 (a + y)
7. If cos y = x cos (a + y), with cos a ≠ ±1, then prove that = . Hence show that
dx sin a
d2 y dy
sin a + sin 2 (a + y) = 0 . [CBSE (F) 2014; (North) 2016]
dx 2 dx
Sol. Given, cos y = x cos (a + y)
cos y
∴ x=
cos (a + y)

190 Xam idea Mathematics–XII

@Cbsebookshub - Join Us on Telegram


Differentiating with respect to y on both sides, we get

dx cos (a + y) # (– sin y) – cos y # [– sin (a + y)]


=
dy cos 2 (a + y)

dx cos y sin (a + y) – sin y cos (a + y) dx sin (a + y – y)




dy
= & dy
=
cos 2 (a + y) cos 2 (a + y)

dx sin a dy cos 2 (a + y)

= ∴ =
dy cos 2 (a + y) dx sin a

Differentiating both sides w.r.t. x, we get


d2 y dy
* –2 cos (a + y) . sin (a + y) . 4
1
=
dx 2 sin a dx

d2 y dy d2 y dy

sin a = – sin 2 (a + y) . ⇒ sin a + sin 2 (a + y) . =0
dx 2 dx dx 2 dx
dy b
8. If x = a sin 2t (1 + cos 2t) and y = b cos 2t (1 – cos 2t), then show that =
dx at t = r a . Also find
4
dy
the value of f p at t = . [CBSE Delhi 2016; (AI) 2014; Panchkula 2015; (Central) 2016]
r
dx 3
Sol. Given, x = a sin 2t (1 + cos 2t) and y = b cos 2t (1 – cos 2t)
dx

= a [sin 2t # (–2 sin 2t) + (1 + cos 2t) # 2 cos 2t] = a[– 2sin2 2t + 2 cos 2t + 2 cos2 2t]
dt
= a(2 cos 4t + 2 cos 2t) = 2a (cos 4t + cos 2t)
dy
Again, = b [cos 2t # 2 sin 2t + (1 – cos 2t) ] – 2 sin 2tg]
dt
= b[sin 4t – 2 sin 2t + sin 4t] = b[2 sin 4t – 2 sin 2t] = 2b(sin 4t – sin 2t)
dy dy/dt 2b (sin 4t – sin 2t)
= = G
b sin 4t – sin 2t

= =
dx dx/dt 2a (cos 4t cos 2t) a cos 4t + cos 2t
+
JK N
dy KK sin r – sin r OOO
So, f p O = #c m = a and
b 2O b –1 b
= KK
dx at t = r a KK cos r + cos r OO a –1
KK O
4 2O
L P
JK 4 r 2 r NOO KJK r r ON
KK sin – sin – sin – sin OO
dy 3 OOO b KK
f p
b KK 3 3 3 O
= K O = KK O
dx at t = r a KK 4 r
+
2 r OO a KK – cos – cos r OOO
r
K cos cos
3 3 3 O K
L
3 3O
P
L P
JK r N
KK – 2 sin OOO 3b
b 3O b r
= a # KKK r OO = a tan = a
KK – 2 cos OO 3
L 3 P
2 2
dy 1 – y2
9. If 1 – x + 1 – y = a (x – y), then show that = . [CBSE (F) 2009, 2019 (65/5/3)]
dx 1 – x2
Sol. Given, 1 – x 2 + 1 – y 2 = a (x – y)

Putting x = sin a ⇒ a = sin–1 x and y = sin b ⇒ b = sin–1 y, we get

1 – sin 2 a + 1 – sin 2 b = a (sin a – sin b) ⇒ cos a + cos b = a(sin a – sin b)

Continuity and Differentiability 191


@Cbsebookshub - Join Us on Telegram
(a + b) a+b
cos e o = a.2 cos f p sin e o
a–b a–b

2 cos
2 2 2 2

cot e o= a
a–b a–b


2
& 2
= cot –1 a & a – b = 2 cot –1 a

⇒ sin–1 x – sin–1 y = 2 cot–1 a



Differentiating both sides with respect to x, we get

1 1 dy dy 1 – y2
– =0 & =
1 – x2 1 – y 2 dx dx 1 – x2
dy r
10. If x = cos t(3 – 2cos2 t) and y = sin t (3 – 2sin2 t), then find the value of at t = . [CBSE (AI) 2014]
dx 4
Sol. Given, x = cos t(3 – 2cos2 t)
Differentiating both sides with respect to t, we get
dx
= cos t {0 + 4 cos t. sin t} + (3 – 2 cos 2 t) . (– sin t)
dt
= 4 sin t . cos2 t – 3 sin t + 2cos2 t. sin t
= 6 sin t cos2 t – 3 sin t = 3 sin t (2 cos2 t – 1) = 3 sin t . cos 2t
Again, a y = sin t (3 – 2 sin2 t)
Differentiating both sides with respect to t, we get
dy
= sin t. {0 – 4 sin t cos t} + (3 – 2 sin 2 t} . cos t
dt
= – 4 sin2 t . cos t + 3cos t – 2 sin2 t . cos t = 3 cos t – 6sin2 t . cos t
= 3 cos t (1 – 2 sin2 t) = 3cos t . cos 2t
dy
dy dt 3 cos t. cos 2t dy
Now, = = ⇒ = cot t
dx dx 3 sin t. cos 2t dx
dt
dy
H
r

= cot =1
dx t=
r 4
4

1 – x2
11. Differentiate tan –1 e x o with respect to cos –1 (2x 1 – x 2) , when x ≠ 0. [CBSE Delhi 2014]

1 – x2
Sol. Let u = tan –1 e x o and v = cos –1 (2x 1 – x 2)

du
We have to determine
dv
Put x = sin q ⇒ q = sin–1 x
1 – sin 2 i
Now, u = tan –1 f p ⇒ u = tan –1 e o
cos i
sin i sin i

u = tan –1 =tan e – i oG
r
⇒ u = tan–1 (cot q) ⇒
2
r r

u= – i ⇒ u= – sin –1 x
2 2

192 Xam idea Mathematics–XII

@Cbsebookshub - Join Us on Telegram


du 1 du 1

=0– ⇒ =–
dx 1 – x2 dx
RS 1 – x2 VW
SSa 1 1 WW
–1 2 – <x<
Again, v = cos (2x 1 – x ) SS 2 2 WW
SS W
a x = sin q r WWW
sin e – o < sin i < sin e oW
SS r
SS& 4 4 WW
∴ v = cos (2 sin i 1 – sin i) ⇒ v = cos (2 sin q . cos q) SSS
–1 2 –1
r r
WW
SS& – <i< WW
⇒ v = cos –1 e cos e – 2i oo SS
r S 4 4 WW
⇒ v = cos–1 (sin 2q) WW
2 SS r r
SS& – < 2i < WW
r r WW
⇒ v = – 2i
2
& v = – 2 sin x
2
–1
SS 2 2
WW
SS r r WW
dv 2 dv 2 SS& > –2i > – WW
⇒ =0– & =– SS 2 2
dx 2 dx 2 SS WW
r > d – 2i n > 0
1– x 1– x r WW
SS& WW
1 SS 2
du – SS WW
d – 2i n ! (0, r) 1 [0, r] WWW
du dx 1– x 2
1 SS& r
∴ = = = S 2 W
dv dv 2 2 T X

dx 1 – x2
1 1
[Note: Here the range of x is taken as – <x< ]
2 2
12. Differentiate the following function with respect to x:
y = (sin x) x + sin –1 x [CBSE Delhi 2009, 2013, 2017]
x –1
Sol. Given, y = (sin x) + sin x
y = u + v, where u = (sin x)x, v = sin –1 x
du dvdy

= + …(i)
dx dx dx
Now, u = (sin x)x
Taking log both sides, we get
log u = log (sin x)x ⇒ log u = x . log (sin x)
Differentiating both sides with respect to x, we get
1 du 1 du
u . = x. cos x + log sin x ⇒ = u {x cot x + log sin x}
dx sin x dx
du
⇒ = (sin x) x {x cot x + log sin x} …(ii)
dx
Also, v = sin –1 x
dv 1 1 1
= # = …(iii)
dx 1 – ( x) 2 2 x 2 x (1 – x)
From (i), (ii) and (iii), we get
dy 1

= (sin x) x {x cot x + log sin x} +
dx 2 x ( 1 – x)
dy y
13. If x13y7 = (x + y)20, then prove that =
x . [CBSE (F) 2012]
dx
OR
dy y
If xmyn = (x + y)m+n, then prove that =
x . [CBSE (F) 2014]
dx

Continuity and Differentiability 193


@Cbsebookshub - Join Us on Telegram
Sol. Given x13y7 = (x + y)20
Taking logarithm on both sides, we get
log (x13y7) = log (x + y)20
⇒ log x13 + log y7 = 20 log (x + y)
& 13 log x + 7 log y = 20 log (x + y)
Differentiating both sides with respect to x, we get
13 7 dy dy 7 dy
.f1 + p x – x + y d x + y – y n dx
20 13 20 20
x + y .
dx
=
x + y dx
& =

13x + 13y – 20x 20y – 7x – 7y dy 13y – 7x 13y – 7x dy



=f p & =f p.
+
x(x y) (x + y) .y dx +
x ( x y) y (x + y) dx
dy 13y – 7x y (x + y) dy y


dx
= #
x (x + y) 13y – 7x
& dx
=
x
OR
Do yourself (similar question)
14. Differentiate with respect to x :
+
2 x 1 .3x
sin –1 f p [CBSE (AI) 2013]
1 + (36) x
+

x p = sin f p = sin –1 f p
2 x 1 .3x 2.2 x .3 x 2.6 x
Sol. Let y = sin –1 f –1
1 + (36) 1 + (6 )2 x
1 + (6 x) 2
Let 6x = tan q ⇒ q = tan–1 (6x)

∴ y = sin –1 f p
2 tan i
& y = sin –1 (sin 2i)
1 + tan 2 i
⇒ y = 2q
⇒ y = 2 . tan–1 (6x)
dy 2 dy 2.6 x . log e 6

= . log e 6.6 x & =
dx 1 + (6 x) 2 dx 1 + 36 x
dy sin 2 (a + y)
15. If x sin (a + y) + sin a cos (a + y) = 0, then prove that = . [CBSE (AI) 2013]
dx sin a
Sol. Given x sin (a + y) + sin a cos (a + y) = 0
sin a. cos (a + y)

x=– ⇒ x = – sin a . cot (a + y)
sin (a + y)
Differentiating with respect to y, we get
dx sin a
=+ sin a . cosec 2 (a + y) =
dy sin 2 (a + y)
dy sin 2 (a + y)

=
dx sin a
dy
16. If ex + ey = ex + y, then prove that + e y – x = 0 . [CBSE (F) 2014]
dx
Sol. Given, ex + ey = ex + y
Differentiating both sides with respect to x, we get
dy dy
ex + e y . = e x + y *1 + 4
dx dx

194 Xam idea Mathematics–XII

@Cbsebookshub - Join Us on Telegram


dy dy dy

ex + e y . = ex + y + ex + y . ⇒ (e x
+y
– e y) = ex – ex + y
dx dx dx
dy +y
⇒ (e x + e y – e y)
= ex – ex – e y [a e x + e y = e x (given)]
dx
dy dy ey

ex . = – e y ⇒ =– x
dx dx e
dy dy

= – e y – x ⇒ + ey – x = 0
dx dx
dy y log x
17. If x = ecos 2t and y = esin 2t, prove that =– . [CBSE (East) 2016]
dx x log y
Sol. We have x = ecos 2t
dx
= e cos 2t (– 2 sin 2t) = – 2x sin 2t [Differentiating w.r.t. t]
dt
Again y = esin 2t
dy
= e sin 2t .2 cos 2t = 2y cos 2t [Differentiating w.r.t. t]
dt
dy
dy dt 2y cos 2t dy – y cos 2t
Now, = = ⇒ =
dx dx –2x sin 2t dx x sin 2t
dt
dy y log x
⇒ =– [ a x = ecos 2t ⇒ log x = cos 2t; y = esin 2t ⇒ log y = sin 2t]
dx x log y
Hence proved.
1
18. If x ∈ R – [–1, 1] then prove that the derivative of sec–1x with respect to x is . [HOTS]
| x | x2 – 1
Sol. Let y = sec–1 x
RS VW
Then, sec y = sec (sec–1 x) = x SSIf x > 1, then y ! d 0, r n WW
SS 2 WW
Differentiating both sides with respect to x, we have SS WW
SS` sec y > 0, tan y > 0 WW
d d d dy SS W
⇒ sec y = (x) ⇒ (sec y) =1 SS& | sec y | . | tan y | sec y tan yWWW
=
dx dx dy dx SSIf x < –1, then WW
dy SS WW
⇒ sec y tan y = 1 [Using chain rule] SSy ! d r , r n ` sec y < 0, tan y < 0 WW
dx SS 2 WW
dy SS WW
⇒ =
1
=
1 SS& | sec y || tan y | WW
dx sec y tan y | sec y | . | tan y | SS W
S& (– sec y) (– tan y) = sec y tan y WW
dy 1 1 1 T X
⇒ = = =
dx | sec y | tan 2 y | sec y | sec 2 y – 1 | x | x 2 – 1

XX SECOND ORDER DERIVATIVES


d2 y dy
1. If x = a cos q + b sin q and y = a sin q – b cos q, then show that y 2 –x + y = 0 .
dx 2 dx
[CBSE (F) 2014; Delhi 2015]
dx
Sol. Given, x = a cos θ + b sin θ ⇒ = –a sin i + b cos i …(i)
di
dy
Also, y = a sin θ – b cos θ ⇒ = a cos i + b sin i …(ii)
di

Continuity and Differentiability 195


@Cbsebookshub - Join Us on Telegram
dy
di dy
a cos i + b sin i

= = [From (i) and (ii)]
dx –a sin i + b cos i
dx
di
dy a cos i + b sin i dy x
⇒ =
dx b cos i – a sin i
& dx
= – ...(iii)
y
Differentiating again with respect to x, we get
dy
d2 y y – x.
dx
=–
dx 2 y2
d2 y dy d2 y dy

y2 = –y + x ⇒ y2 –x +y = 0
dx 2 dx dx 2 dx

ax bx
d2 y dy
2. If y = Pe + Qe , then show that – ( a + b) + aby = 0 [CBSE (AI) 2014]
dx 2 dx
Sol. Given, y = Peax + Qebx
On differentiating with respect to x, we have
dy
= Pae ax + Qbe bx
dx
Again, differentiating with respect to x, we have
d2 y
= Pa 2 e ax + Qb 2 e bx
dx 2
d2 y dy
Now, LHS = – (a + b) + aby
dx 2 dx

= Pa2eax + Qb2ebx – (a + b)(Paeax + Qbebx) + ab (Peax + Qebx)


= Pa2eax + Qb2ebx – Pa2eax – Pabeax – Qabebx – Qb2ebx + Pabeax + Qabebx
= 0 = RHS
–1 d2 y dy
3. If y = ea cos x
, –1 < x < 1 , then show that (1 – x 2) –x – a 2 y = 0 . [CBSE 2020 (65/2/1)]
dx 2 dx
–1
Sol. Given, y = e a cos x
, –1 < x < 1
Differentiating w.r.t x, we have
dy –1 1 ay

= e a cos x
#a#– =–
dx 1 – x2 1 – x2
dy

1 – x2
= – ay
dx
Squaring both sides, we have
2
dy
(1 – x 2) e o = a2 y2
dx
Again differentiating w.r.t. x, we have

dy d2 y dy 2
dy
(1 – x 2) # 2 # +e o # (–2x) = a 2 # 2y
dx dx 2 dx dx

196 Xam idea Mathematics–XII

@Cbsebookshub - Join Us on Telegram


d2 y dy
⇒ (1 – x 2)
–x = a2 y
dx 2 dx
d2 y dy
⇒ (1 – x 2)
–x – a 2 y = 0 Proved
dx 2 dx

2
d2 y dy
4. If y = sin (log x), then prove that x +x + y = 0 . [CBSE (F) 2013]
dx 2 dx
Sol. Given, y = sin (log x)
dy 1 cos (log x)

= cos (log x) # =
dx x x

x <– sin (log x) # x F – cos (log x)


1
d2 y – cos (log x) – sin (log x)
Again, 2
= 2
=
dx x x2
d2 y dy
Now, LHS = x2 +x +y
dx 2 dx
x 2 {– cos (log x) – sin (log x)} x cos (log x)
= 2
+
x
+ sin (log x)
x
= – cos (log x) – sin (log x) + cos (log x) + sin (log x) = 0 = RHS
d2 y dy
5. If y = cosec–1 x, x > 1, then show that x (x 2 –1) + (2x 2 –1) = 0 [CBSE (AI) 2010]
dx 2 dx
Sol.  y = cosec–1 x
Differentiating with respect to x, we get
dy –1

=
dx x x 2 – 1
Again differentiating with respect to x, we get
2x
2 x x 2 – 1 .0 + 1. ) x. + x2 – 1 3
d y 2 x2 – 1
=
dx 2 x2 ^x2 – 1h
d2 y x2 + x2 – 1 2x 2 – 1

= =
dx 2
x x – 1h . x – 1
2^ 2 2
x – 1 .x 2 ^x 2 – 1h
2

d2 y 2x 2 – 1 dy

x (x 2 – 1) = = (2x 2 – 1) c – m
dx 2 x x2 – 1 dx
d2 y dy

x (x 2 – 1) + (2x 2 – 1) =0
dx 2 dx
6. If y = 3 cos (log x) + 4 sin (log x), show that

2
d2 y dy
x +x + y = 0. [CBSE Delhi 2009, 2012]
dx 2 dx
Sol. Given, y = 3 cos (log x) + 4 sin (log x)
Differentiating with respect to x, we get
dy 3 sin ^ log x h 4 cos ^ log x h 1
=– + ⇒ y1 = [–3 sin (log x) + 4 cos (log x)]
dx x x x

Continuity and Differentiability 197


@Cbsebookshub - Join Us on Telegram
Again differentiating with respect to x, we get
–3 cos ^ log x h 4 sin ^ log x h
2 x < – F – 6–3 sin ^ log x h + 4 cos ^ log x h]
d y x x
=
dx 2 x2
–3 cos ^log xh – 4 sin ^log xh + 3 sin ^log xh – 4 cos ^log xh
=
x2
d2 y – sin ^ log x h – 7 cos ^ log x h – sin ^ log x h – 7 cos ^ log x h
= ⇒ y2 =
dx 2 x2 x2
Now, LHS = x2y2 + xy1 + y
– sin ^log xh – 7 cos ^log xh
= x2 d n + x × 6–3 sin ^log xh + 4 cos ^log xh] + 3 cos ^log xh + 4 sin ^log xh
1

x2 x
= – sin (log x) – 7 cos (log x) – 3 sin (log x) + 4 cos (log x) + 3 cos (log x) + 4 sin (log x)
= 0 = RHS
r d2 x d2 y d2 y
7. If x = a (cos t + t sin t) and y = a (sin t – t cos t), 0 < t < , find , and .
2 dt 2 dt 2 dx 2
[CBSE (AI) 2012]
Sol. Given, x = a (cos t + t sin t)
Differentiating both sides with respect to t, we get
dx dx
= a (– sin t + t cos t + sin t) ⇒ = at cos t ...(i)
dt dt
Differentiating again with respect to t, we get
d2 x
= a ^ –t sin t + cos t h = a ^cos t – t sin t h
dt 2
Again, y = a (sin t – t cos t)
Differentiating with respect to t, we get
dy dy
= a (cos t + t sin t – cos t) ⇒ = at sin t ...(ii)
dt dt
Differentiating again with respect to t we get
d2 y
= a ^t cos t + sin t h
dt 2
dy dy dt
Now, = [From (i) and (ii)]
dx dx dt
dy at sin t dy

=
dx at cos t
& dx = tan t
Differentiating again with respect to x, we get
d2 y dt 1 sec 2 t sec 3 t
= sec 2 t. = sec 2 t. = = [From (i)]
dx 2 dx dx dt at cos t at
d2 x d2 y d2 y sec 3 t
Hence, = a ^cos t – t sin t h , = a ^t cos t + sin th and = .
dt 2 dt 2 dx 2 at

198 Xam idea Mathematics–XII

@Cbsebookshub - Join Us on Telegram


d2 y dy
8. If y = log 7x + x + a A, show that (x + a )
2 2 2 2
+x = 0 . [CBSE Delhi 2013]
dx 2 dx

Sol. Given y = log 7x + x 2 + a 2 A


dy 1 2x x + x2 + a2

= . =1 + G =
dx x + x 2 + a 2 2 x2 + a2 _ x + x 2 + a 2 i_ x 2 + a 2 i
dy 1
⇒ = …(i)
dx x + a2
2

Differentiating again with respect to x, we get


3
1 2 d2 y –
–x
^ 2h
2
= – x + a .2x =
dx 2 2 3

^x + a 2h
2
2

2
d y –x d2 y x

= & ( x 2 + a 2) =–
dx 2 ^x + a . x + a
2 2h 2 2
dx 2
x + a2
2

2
d y dy

( x 2 + a 2) + x. = 0 [From (i)]
dx 2 dx
2
1 dy y d2 y
x
9. If y = x , then prove that – f p – =
x 0 . [CBSE Delhi 2014, 2016]
dx 2 y dx
Sol. Given, y = xx
Taking logarithm on both sides, we get
log y = x . log x
Differentiating both sides, we get
1 dy 1 dy
⇒ . = x. + log x ⇒ = y ^1 + log x h …(i)
y dx x dx
Again differentiating both sides, we get
d2 y 1 dy d2 y y 1 dy dy
= y. + (1 + log x) . ⇒ = + . . [From (i)]
dx 2 x dx dx 2 x y dx dx
d2 y y 1 dy 2 d2 y 1 dy 2 y

= + c m ⇒ – c m – =0
dx 2 x y dx dx 2 y dx x
d2 y dy
10. If y = _ x + 1 + x 2 i , then show that (1 + x 2)
n
+x = n 2 y . [CBSE (F) 2015]
dx 2 dx

Sol. Given y = _ x + 1 + x 2i
n

Differentiating with respect to x, we get


2
n–1 x+ 1+x
= n_ x + 1 + x2 i .f p
dy 2x dy
= n _ x + 1 + x 2 i . )1 + 3
n–1
&

dx
& dx
2 1 + x2 1 + x2
dy n _ x + 1 + x 2 i
n
dy ny
&

dx
= & dx
=
1 + x2 1 + x2
dy
&

dx
1 + x2 .
= ny

Again differentiating with respect to x, we get


d2 y 2x dy dy d2 y dy dy
1 + x2 . + . =n & (1 + x 2 ) + x. = n . 1 + x2 .
dx 2
2 1 + x 2 dx dx dx 2 dx dx

Continuity and Differentiability 199


@Cbsebookshub - Join Us on Telegram
d2 y dy ny d2 y dy
&
(1 + x 2 ) 2
+x
dx
= n . 1 + x2 .
2
& (1 + x 2 ) 2
+x
dx
= n2 y
dx 1+x dx
2
1
11. If y = log e x + o , then prove that x(x + 1)2y2 + (x + 1)2y1 = 2. [CBSE Sample Paper 2018]
x
1 2 x+1
n = 2 log d n
1
Sol.  y = log d x + n ⇒ y = 2 log d x +
x x x
y = 2 log ^ x + 1 h) – 2 log x ⇒ y = 2 log (x + 1) – log x
2 1 2x – x – 1 x–1

y1 = – = ⇒ y1 =
x 1 x
+ x ( x + 1) x^x + 1h

x ^ x + 1 h – ^ x – 1 h^ 2x + 1 h x 2 + x – 2x 2 – x + 2x + 1

y2 = ⇒ y2 =
x2 ^x + 1h x2 ^x + 1h
2 2

–x 2 + 2x + 1
⇒ y2 =
x2 ^x + 1h
2

–x 2 + 2x + 1 ^x – 1h
x ^ x + 1 h y2 + ^ x + 1 h y1 = x ^ x + 1 h . + ^x + 1h .
2 2 2 2
Now,
x ^x + 1h
2 2 x^ x + 1h

–x + 2x + 1 ^ x + 1 h^ x – 1 h
2
= +
x x
2 2
–x + 2x + 1 + x – 1 2x
= = = 2 Hence proved.
x x
d2 y dy
12. If y = sin (sin x), prove that + tan x + y cos 2 x = 0. [CBSE 2018]
dx 2 dx
Sol. y = sin (sin x)
dy d dy

= cos (sin x) (sin x) ⇒ = cos (sin x) cos x
dx dx dx I II

Again differentiating w.r.t x on both sides, we get


d2 y d d
= cos (sin x) (cos x) + (cos x) cos (sin x)
dx 2 dx dx
d2 y

= cos(sin x) (– sin x) – (cos x) {(sin (sin x)} (cos x)
dx 2
d2 y

= – sin x cos (sin x) – cos2x sin (sin x)
dx 2
Putting these values in LHS, we get
d2 y dy
+ tan x + y cos 2 x
dx 2 dx
= {– sin x cos (sin x) – cos2 x sin(sin x)} + tan x {cos x cos(sin x)} + y cos2 x
= – sinx cos (sinx) – cos2x sin(sinx) + tan x cos x cos(sin x) + y cos2 x
sin x
= – sin x cos (sin x) – cos2x sin(sin x) + cos x cos (sin x) + sin(sin x) cos2x = 0
cos x
Hence proved.

200 Xam idea Mathematics–XII

@Cbsebookshub - Join Us on Telegram


d2 y dy
13. If x = sint and y = sin pt, then prove that (1 – x 2) –x + p 2 y = 0 . [NCERT Exemplar]
dx 2 dx
Sol. We have, x = sint and y = sin pt
dx dy

= cos t and = cos pt.p
dt dt
dy dy/dt p. cos pt

= = ...(i)
dx dx/dt cos t
Again, differentiating both sides w.r.t. x, we get
d dt d dt
d 2 y cos t. dt (p. cos pt) dx – p cos pt. dt cos t. dx
=
dx 2 cos 2 t
dt 1
[cos t.p. (– sin pt) .p – p cos pt. (– sin t)] [– p 2 sin pt. cos t + p sin t. cos pt]
= dx = cos t
cos 2 t cos 2 t
d 2 y – p 2 sin pt. cos t + p cos pt. sin t
=  ...(ii)
dx 2 cos 3 t
Since, we have to prove
d2 y dy
(1 – x 2) –x + p2 y = 0
dx 2 dx
[– p 2 sin pt. cos t + p cos pt. sin t] p cos pt

LHS = (1 – sin 2 t) – sin t. + p 2 sin pt
cos 3 t cos t
2 2
> H
1 (1 – sin t) (– p sin pt. cos t + p cos pt. sin t)
=
cos 3 t – p cos pt. sin t. cos 2 t + p 2 sin pt. cos 3 t
2 3 2
> H[a 1 – sin 2 t = cos 2 t]
1 – p sin pt. cos t + p cos pt. sin t. cos t
=
cos 3 t – p cos pt. sin t. cos 2 t + p 2 sin pt. cos 3 t
1
= .0 = 0 Hence proved.
cos 3 t

Rolle’s and Mean Value Theorem


1. Verify Rolle’s theorem for the function f(x) = ex cos x in ;– , E . [CBSE 2020 (65/4/1)]
r r

2 2

Sol. Given function, f(x) = ex cos x in ;– , E


r r
2 2

∴ f ′(x) = ex (cos x – sin x), in c – , m


r r

2 2

Clearly, f(x) is differentiable in c – , m


r r
2 2

Thus, f(x) is differentiable in c – , m


r r
2 2

∴ f(x) must be continuous in ;– , E .


r r

2 2
Because every differentiable function must be continuous.

Now, f c – m = e – 2 cos c – m = 0
r r r
2 2

f c m = e 2 cos c m = 0
r r r
and,
2 2

Continuity and Differentiability 201


@Cbsebookshub - Join Us on Telegram
∴ f c – m=f c m
r r
2 2
Therefore, Rolle’s theorem is applicable.

So, there exists a real number c such that


f ′(c) = 0 ⇒  ec (cos c – sin c) = 0
⇒ cos c – sin c = 0 ( ec ≠ 0)
⇒ cos c = sin c
r r
⇒ tan c = 1 = tan   ⇒  c =
4 4
! c– , m
r r r
∴ c =
4 2 2
Hence, Rolle’s theorem is verified.
2. Verify Mean Value theorem for the function f(x) = 2 sin x + sin 2x on [0, p]. [CBSE (North) 2016]
Sol. We have, f(x) = 2 sin x + sin 2x
f(x) is continuous in [0, p] being trigonometric function and it is differentiable on (0, p).

Hence, condition of Mean Value Theorem is satisfied.
Therefore, Mean Value Theorem is applicable.
So, there exist a real number c such that
f ^r h – f ^ 0 h
f l^ c h = …(i)
r–0
Now f(0) = 2 sin 0 + sin 0 = 0; f(p) = 2 sin p + sin 2p = 0 and f ' (x) = 2 cos x + 2 cos 2x
∴ f'(c) = 2 cos c + 2 cos 2c

From (i)
0–0
2 cos c + 2 cos 2c =
r

2 cos c + 2 cos 2c = 0 ⇒ 2 cos c + 2(2 cos2 c – 1) = 0
⇒ cos c + 2 cos2 c – 1 = 0
⇒ 2 cos2 c + cos c – 1 = 0

2 cos2 c + 2 cos c – cos c – 1 = 0 ⇒ 2 cos c (cos c + 1) – 1 (cos c + 1) = 0
1

^cos c + 1 h^ 2 cos c – 1 h = 0 & cos c = –1 and cos c =
2
r r
⇒ c = p and c =
! ^0, r h ⇒ c = ! ^0, r h
3 3
Hence, Mean Value Theorem is verified.
3. Discuss the applicability of Rolle’s theorem on the function given by [NCERT Exemplar]
2
x + 1, if 0 # x # 1
f (x) = *
3 – x, if 1 < x # 2

x 2 + 1, if 0 # x # 1
Sol. We have, f (x) = *
3 – x, if 1 # x # 2
We know that, polynomial function is everywhere continuous and differentiable.
So, f(x) is continuous and differentiable at all points except possibly at x = 1.
Now, check the differentiability at x = 1,
At x = 1,

202 Xam idea Mathematics–XII

@Cbsebookshub - Join Us on Telegram


f (x) – f (1)
LHD = lim
x " 1– x–1

(x 2 + 1) – (1 + 1)
= lim  [ f(x) = x2 + 1, 0 ≤ ∀ x ≤ 1]
x"1 x–1
x2 – 1 ( x + 1) ( x – 1)
= lim = lim =2
x"1 x – 1 x"1 x–1
f (x) – f (1) (3 – x) – (1 + 1)
and RHD = lim = lim
x"1
+ x–1 x"1 (x – 1)

3–x–2 – ( x – 1)
= lim = lim = –1
x"1 x–1 x"1 x–1

LHD ≠ RHD
So, f(x) is not differentiable at x = 1.
Hence, Rolle’s theorem is not applicable on the interval [0, 2].

4. Find a point on the curve y = (x – 3)2, where the tangent is parallel to the chord joining the
points (3, 0) and (4, 1). [NCERT Exemplar]
Sol. We have, y = (x – 3)2, which is continuous in x1 = 3 and x2 = 4 i.e., [3, 4].
Also, y’ = 2(x – 3) × 1 = 2(x – 3) which exists in (3, 4).
Hence, by mean value theorem there exists a point on the curve at which tangent drawn is parallel
to the chord joining the points (3, 0) and (4, 1).
f (4) – f (3)
Thus, f l ( c) =
4–3
( 4 – 3 ) 2 – (3 – 3 ) 2

2 (c – 3) =
4–3
1–0 7

2c – 6 =
1
& c=
2
2
1 2 1
y = c – 3m = c m =
7 7
For x = ,
2 2 2 4

So, c , m is the point on the curve at which tangent drawn is parallel to the chord joining the
7 1
2 4
points (3, 0) and (4, 1).

PROFICIENCY EXERCISE
QQ Objective Type Questions: [1 mark each]
1. Choose and write the correct option in each of the following questions.
x2
+ 1 , then which of the following can be a discontinuous function?
(i) If f(x) = 2x and g(x) =
2
g (x)
(a) f(x) + g(x) (b) f(x) – g(x) (c) f(x) . g(x) (d)
f (x)
(ii) The set of points where the function f given by f (x) = 2x – 1 sin x is differentiable is
 [NCERT Exemplar]

(b) R – ' 1 (c) (0, ∞) (d) none of these


1
(a) R
2

Continuity and Differentiability 203


@Cbsebookshub - Join Us on Telegram
(iii) Let f(x)= |cos x|. Then,
(a) f is everywhere differentiable.
(b) f is everywhere continuous but not differentiable at x = nπ, n ∈ Z.
r
(c) f is everywhere continuous but not differentiable at x = (2n + 1) , n ∈ Z.
2
(d) none of these.

5x –5x
d2 y
(iv) If y = A e + B e , then is equal to [CBSE 2020 (65/5/1)]
dx 2
(a) 25 y (b) 5 y (c) –25 y (d) 15 y
dy
at c , m is
1 1
(v) For the curve x + y = 1,
dx 4 4
1
(a) (b) 1 (c) –1 (d) 2
2
dy
(vi) If f ′(1) = 2 and y = f (log ex), then at x = e is
dx
2
(a) 0 (b) 1 (c) e (d) e
2. Fill in the blanks.

(i) If f (x) = cos x , then f lc m = _____________ .


r
4
d
(ii) If f(x) = (x + 1), then fof (x) = _____________ .
dx
d
(iii) sec (tan –1 x) = _____________ .
dx
1
(iv) The number of points at which the function f (x) = is discontinuous is _________ .
log x
QQVery Short Answer Questions: [1 mark each]
Z]
]] sin –1 x
3. If f (x) = [] x , x ! 0 , is continuous at x = 0, then write the value of k.
]] k, x=0
\
4. Determine the value of ‘k’ for which the following function is continuous at x = 3:
]Z] (x + 3) 2 – 36
]]
f (x) = [] , x!3 [CBSE (AI) 2017]
]] x–3
] k, x=3
\
x 2 dy
5. If y = ;sin + cos E , find
x r
at x = .
2 2 dx 6
dy
F, x ! ;– , E
1 + tan x
, if y = tan –1 <
r r
6. Find
dx 1 – tan x 4 4
dy
7. If y = log (ex), then find . [CBSE 2019 (65/4/3)]
dx
QQShort Answer Questions–I: [2 marks each]
8. Examine the continuity at the indicated points.
f(x) = |x|+|x – 1| at x = 1
9. Find k if f(x) is continuous at x = 0.
sin x
f (x) = * x
+ cos x, if x ! 0
k, if x = 0

204 Xam idea Mathematics–XII

@Cbsebookshub - Join Us on Telegram


10. Find the value of c in Rolle’s theorem for the function f(x) = x3 – 3x in [– 3 , 0]. [CBSE (AI) 2017]
11. If f(x) = |cos x – sin x|, find f ′(p/6).
d2 y
12. If y = 5 cos x – 3 sin x, prove that +y = 0.
dx 2
QQ Short Answer Questions–II: [3 marks each]
Z]
]] 3x – 2, 0 < x # 1
]
13. Show that the function ‘f ’ defined by f (x) = [] 2x 2 – x, 1 < x # 2 is continuous at x = 2, but not
]]
] 5x – 4, x>2
differentiable. \ [CBSE Delhi 2010]
14. Show that the function f(x) = |x – 1| + |x + 1|, for all x ∈ R, but is not differentiable at the points
x = –1 and x = 1. [CBSE Allahabad 2015]
dy
15. If y = x3 (cos x)x + sin –1 x , find . [CBSE 2020 (65/3/1)]
dx

16. Differentiate tan –1 f p with respect to cos–1 x2.


1 + x2 – 1 – x2

1 + x2 + 1 – x2
 [CBSE (South) 2016, 2019 (65/4/1)]
1 + x 2 –1
17. Differentiate tan –1 > x H with respect to x. [CBSE (AI) 2012]

x
18. Differentiate tan –1 with respect to sin –1 (2x 1 – x 2) . [CBSE Delhi 2014]
2
1– x
+
2x 1 dy
19. If y = cos –1 f x p, then find . [CBSE (F) 2010]
+
1 4 dx
d2 y dy
20. If y = ex (sin x + cos x), then show that –2 + 2y = 0 [CBSE (AI) 2009]
dx 2 dx
21. Verify Lagrange’s Mean Value Theorem for the following function:
f(x) = x2 + 2x + 3, for [4, 6]. [CBSE (AI) 2008]
dy
22. If y = x 2 + 1 – log f x + p, then find
1 1
1+ . [CBSE Delhi 2008]
x 2 dx
]]Z 1
]] 2x – 1, x 1
2 1
23. Discuss the differentiability of the function f (x) = ][ =
1 at x 2 .
]] 3 – 6x, x $
] 2
 \ [CBSE Sample Paper 2017]
r
24. For what value of k is the following function continuous at x = – ? [CBSE Sample Paper 2017]
6
Z] +
]] 3 sin x cos x r
]] , x!–
] r 6
x +
f (x) = [] 6
]] r
]] k, x=–
] 6
\
dy 1
25. If x 1 + y + y 1 + x = 0, –1 < x < 1, x ! y, then prove that =– . [CBSE (F) 2012]
dx (1 + x) 2
26. Differentiate the following function with respect to x : (x)cos x + (sin x)tan x [CBSE Delhi 2009]
1 1 dy
27. If y = sin –1 (6x 1 – 9x 2), – 1x1 then find . [CBSE Delhi 2017]
3 2 3 2 dx

Continuity and Differentiability 205


@Cbsebookshub - Join Us on Telegram
–1 d2 y dy
28. If y = e a cos x
, –1 # x # 1, show that (1 – x 2) –x – a 2 y = 0. [CBSE (F) 2012]
dx 2 dx
d2 y
29. If xmyn = (x + y)m + n, prove that = 0 . [CBSE Delhi 2017]
dx 2
dy d2 y
If y = log tan c + m, show that
r x r
30. = sec x. Also find the value of at x = .
4 2 dx dx 2 4
 [CBSE (F) 2010]

31. If x = tan b a log y l , show that:


1
[CBSE (AI) 2011]

d2 y dy
(1 + x 2) + (2x – a) =0
dx 2 dx
dy y (x – 1)
32. If xy = e(x – y), then show that = . [CBSE (F) 2017]
dx x (y + 1)

If y = tan –1 f p, x 2 # 1, then find


1 + x2 + 1 – x2 dy
33. . [CBSE Delhi 2015]
2
1+x – 1– x 2 dx
2
d2 y
34. If (ax + b)ey/x = x, then show that x 3 f p = fx
dy
– y p . [CBSE Ajmer 2015]
dx 2 dx
x+1
35. If f (x) = x 2 + 1; g (x) = and h(x) = 2x – 3, then find f’[h’{g’(x)}]. [CBSE Allahabad 2015]
x2 + 1
36. Let f(x) = x –|x – x2|, x ∈ [–1, 1]. Find the point of discontinuity, (if any), of this function on [–1, 1].
[CBSE Bhubaneshwar 2015]

x a dy x
37. If x – y = log x – y , then prove that = 2 – .
y [CBSE Guwahati 2015]
dx
dy
38. Let y = (log x) x + x x cos x, then find . [CBSE Sample Paper 2016]
dx
d2 y dy 2
39. If e y (x + 1) = 1 , then show that =d n . [CBSE (AI) 2017]
dx 2 dx
d2 y
40. If x = a (i – sin i), y = a (1 + cos i), then find . [CBSE Delhi 2011]
dx 2

2
d2 y dy
41. If y = 2cos (log x) + 3sin (log x), prove that x +x + y = 0 . [CBSE (Central) 2016]
dx 2 dx
42. Show that the function f given by [CBSE (East) 2016]
Z] 1
]] e x – 1
] , if x ! 0
f (x) = [] 1 x +
]] e 1
] –1 if x = 0
\
is discontinuous at x = 0.
dy 6x – 4 1 – 4x 2
43. Find if y = sin –1 > H. [CBSE (North) 2016]
dx 5

44. Differentiate (sin 2x) x + sin –1 3x with respect to x. [CBSE (South) 2016]

206 Xam idea Mathematics–XII

@Cbsebookshub - Join Us on Telegram


1 + x2 – 1
45. Differentiate tan –1 e o w.r.t. sin –1
2x
x , if x ! (– 1, 1) . [CBSE (F) 2016]
1 + x2
d2 y
46. If x = a(cos 2t + 2t sin 2t) and y = a(sin 2t – 2t cos 2t), then find . [CBSE (Ajmer) 2015]
dx 2
47. Find the values of a and b, if the function f is defined by [CBSE (F) 2016]
2
f (x) = *
x + 3x + a , x # 1
bx + 2 , x>1
is differentiable at x = 1.

Answers
1. (i) (d) (ii) (b) (iii) (c) (iv) (a) (v) (c) (vi) (d)
1 x
2. (i) – (ii) 1 (iii) (iv) three
2 1 + x2
3
3. k = 1 4. k = 12 5. 6. 1 7. 1 8. Discontinuous
2
11. – ^1 + 3 h 15. x 3 (cos x) x ; x – x tan x + log (cos x)E +
1 3 1
9. k = 2 10. c = –1
2 2 x – x2
+1
1 1 1 – 2x . log e 2 x2 + 1
16. – 17. 18. 19. 22. x
2 2 ( 1 + x 2) 2 1 + 4x
23. Not differentiable 24. k = 2

26. x cos x d x – sin x log x n + (sin x) tan x [1 + sec 2 x log sin x]


cos x 6
27. 30. 2
1 – 9x 2
dy 1 + 1 – x2
; [Hint: At first simplify as y = tan –1 f p by multiplying with 1 + x 2 + 1 – x 2 .
x
33. =–
dx 1 – x4 x2
Then let x2 = sin q and then solve.]
x
y
x log e ax + b o y = log e o
x x x
34. Hint: e y/x =
ax + b
& = & ax + b

; Hint: At first find f ‘(x), g’(x) and h(x) and then find f ‘[h’{g’(x)}] = f l>hl* 4H
2 5 –x 2 – 2x + 1
35.
5 x2 + 1
36. No point of discontinuity.
]]Z 2x – x 2, –1 # x < 0
]]
Hint: f (x) = ][ 0, x=0
]] 2
] x, 0<x#1
\
x
37. Hint: x – y = log a – log (x – y) then differentiate.

38. (log x) x ) + log (log x) 3 + x x cos x {cos x + cos x (log x) – x sin x log x}
1
log x
i 2 3
40. – cot 43. 44. (sin 2x) x {2x cot 2x + log (sin 2x)} +
2 1 – 4x 2
2 3x – 9x 2
1 sec 3 2t
45. 46. 47. a = 3, b = 5
4 2at

Continuity and Differentiability 207


@Cbsebookshub - Join Us on Telegram
SELF-ASSESSMENT TEST
Time allowed: 1 hour Max. marks: 30
1. Choose and write the correct option in the following questions. (4 × 1 = 4)
]Z] log (1 + ax) – log (1– bx) , if x ! 0
]]
(i) If the function f(x) defined by f (x) = [] x is continuous at
]]
] k , if x = 0
\
x = 0, then value of k is
(a) a (b) b (c) a + b (d) 0
dy r
(ii) If x = a sec3 θ and y = a tan3 θ, then at i = is
dx 3
2 1 3
(a) (b) (c) 3 (d)
3 3 2
(iii) Derivatives of x2 w.r.t. x3 is
5 7x 2
(a) (b) (c) (d) 0
3x 3 3x
dy
(iv) If y = (sin x + cos x) 2 , then equals to
dx at x = r
4
(a) 0 (b) 1 (c) –1 (d) 2
2.
Fill in the blanks. (2 × 1 = 2)
(i) Derivative of sin x w.r.t cos x is _____________ .

(ii) If f (x) = )
ax + 1, if x $ 1
is continuous, then a should be equal to _____________ .
x + 2, if x < 1
QQ Solve the following questions. (2 × 1 = 2)
x
e
3. Find the derivative of .
sin x
Z] kx
]]
] , if x < 0
4. Find the value of the constant ‘k’ so that the function f (x) = ][ x is continuous at x = 0.
]]
] 3 , if x $ 0
\
QQ Solve the following questions. (4 × 2 = 8)
5. Write the derivative of f(x) = |x3| at x = 0.
6. Differentiate sin (cos (xy)) with respect to x.
7. Find k if f(x) is continuous at x = 0.
sin x
f (x) = * x
+ cos x, if x ! 0
k, if x = 0

8. If f(x) = |cos x – sin x|, find f ′(p/6).

QQ Solve the following questions. (3 × 3 = 9)


9. For what value of k, the following function is continuous at x = 0?
Z] 1 – cos 4x
]]
] , x!0
f (x) = [] 8x 2
]]
] k , x=0
\

208 Xam idea Mathematics–XII

@Cbsebookshub - Join Us on Telegram


10. Examine the continuity of the following function:
Z] x
]]
]] 2 | x | , x ! 0
]
f (x) = [] at x = 0
]] 1
]] ,x=0
] 2
\
11. If the function f, as defined below is continuous at x = 0, find the values of a, b and c.
]Z]
]] sin (a + 1) x + sin x
]] , x<0
]] x
f (x) = [] c , x=0
]] 2
]] x + bx – x
]] , x>0
] bx 3/2
\
QQ Solve the following question. (1 × 5 = 5)
12. Differentiate the following function with respect to x: (log x)x + xlog x.

Answers
1. (i) (c) (ii) (d) (iii) (c) (iv) (a)

2. (i) – cot x (ii) a = 2

e x (sin x – cos x) –y sin (xy) cos (cos xy)


3. 4. –3 5. 0 6.
2
sin x 1 + x sin (xy) cos (cos xy)

8. – ^1 + 3 h
1
7. k = 2
2
3 1
9. 1 10. f is discontinuous 11. a = – , c = and b is any real number
2 2
2 log x.x log x
12. (log x) x ) + log (log x) 3 +
1
log x x
zzz

Continuity and Differentiability 209


@Cbsebookshub - Join Us on Telegram
7 Application
of Derivatives

1. Rate of Change: If a quantity y varies with another quantity x, satisfying some rule y = f(x), then
dy
(or f'(x0)) represents the rate of change of y with respect to x at x = x0.
dx x = x0
2. Differentials: Let y = f(x) be any function of x which is differentiable in (a, b). The derivative of this
function at some point x of (a, b) is given by the relation
dy Dy f (x + Dx) – f (x)
= lim = lim = f l (x)
dx Dx " 0 Dx Dx " 0 Dx
dy
⇒ = f l (x)
dx
⇒ dy = f'(x) dx, where dy is called the differential of the function.
Note: Formula dy = f'(x) dx or Dy = f'(x) Dx is very useful in measuring the errors in the dependent

variable for given error in independent variable.
(i) Absolute Error: The error Dx in x is called the absolute error.
Dx
(ii) Relative Error: If Dx is error in x then ratio x is called the relative error.

Dx Dx
(iii) Percentage Error: If x is relative error, then x × 100 is called percentage error in x.

y-axis
y = f(x)
3. Tangents and Normals:

}
f ^ a + Dx h – f ^ a h f ^a + Dxh – f ^ a h B f(a + x)
(i) Slope of chord AB = +
=
a Dx – a Dx
f(a + x) f(a)
(a, f(a))
Obviously, if Dx → 0 comes very close to A and then chord A
AB becomes tangent at A i.e., x = a. a+x a

f ^a + Dxh – f ^ a h
i.e., slope of tangent at (a, f(a)) = lim

Dx " 0 Dx a
O a+x x-axis
dy
= [f'(x)](a, f(a)) =c m
dx ^ a, f ^ a h)

Hence, equation of tangent to the curve y = f(x) at the point (x1, y1) is given by
dy
^ y – y1 h = c m ^ x – x1 h [ Equation of line is y – y1 = m(x – x1), where m is slope]
dx ^ x1, y1h

210 Xam idea Mathematics–XII

@Cbsebookshub - Join Us on Telegram


dy
(ii) If dx = ∞ at the point P(x1, y1), then the tangent at P is parallel to y-axis and its equation is
given by
y–y
3 = x – x1 ⇒ x – x1 = 0 ⇒ x = x1
1
dx
dy at P ^x1, y1h 0
[Note : In this case, = ]

dy
(iii) If dx = 0 , at the point P(x1, y1), then the tangent at P is parallel to x-axis and equation is given as
y–y
0 = x – x1 ⇒ y – y1 = 0 ⇒ y = y1
1
(iv) Obviously, normal to the curve y = f(x) at P(x1, y1) is perpendicular to the tangent at P(x1, y1).
–1
∴ Slope of normal = slope of tangent Y-axis

Normal y = f(x)
= –1
c m
dy
Tangent
dx ^ x1, y1h
P = (x1, y1)
Hence, equation of normal to the curve y = f(x) at P(x1, y1) is
X-axis
–1
^ y – y1h = dy ^x – x1h
O

; E
dx ^ x1, y1h
dy
(v) If dx at the point P(x1, y1) is zero, then the equation of normal is x = x1.

(vi) If c m at the point (x1, y1) does not exist, then the equation of normal is y = y1.
dy
dx

(vii) The angle q between two given curves y1 = f1(x) and y2 = f2(x) at a point (x1, y1) is given by
m –m
tan i = 1 +1 m m2 where (x1, y1) is the point of intersection and m1, m2 are slopes of their
1 2

tangents at point (x1, y1).



Note: The gradient of a curve at a point is defined as the slope of tangent to the curve at that point.
4. Nature of Function: To know the behaviour of a function in an interval, we study the properties of
increasing or decreasing functions, maximum and minimum of the functions.
5. Monotonic Function: A function is said to be monotonic in an interval, if it is either increasing or
decreasing in the given interval.
6. Increasing Function: A function f(x) is said to be an increasing function in (a, b) if
x1 < x2 ⇒ f(x1) ≤ f(x2) ∀ x1, x2 ∈ (a, b)
In this way, we can say
f(x) is increasing in (a, b) if ∀ x∈ (a, b), f '(x)> 0
y-axis
Obviously, the angle q made by tangent with +ve direction of x-axis
in interval (a, b) is acute.
f(x2) Tangent
⇒ tan q is +ve ⇒ slope is +ve
dy f(x1)
⇒ =
dx f ' (x) 2 0 θ
x2 b x-axis
O a x1
7. Decreasing Function: A function f(x) is said to be decreasing in the
interval (a, b) if
x1 < x2 ⇒ f(x1) ≥ f(x2) ∀ x1, x2 ∈ (a, b)

Application of Derivatives 211


@Cbsebookshub - Join Us on Telegram
In other way,
f(x) is decreasing in interval (a, b) if ∀ x∈ (a, b), f '(x) < 0 y-axis

Tangent
Obviously, the angle q made by tangent with +ve direction of x-axis in
interval (a, b) is obtuse. f(x1)
f(x2)

tan q is –ve ⇒ slope is –ve
θ
dy
⇒ = ^ h x-axis
dx f ' x < 0
a x1 x2 b

[Note: A function f(x) is said to be:


Strictly increasing if x1 < x2 ⇒ f(x1) < f(x2) ∀ x1, x2 ∈ (a, b)
Strictly decreasing if x1 < x2 ⇒ f(x1) > f(x2) ∀ x1, x2 ∈ (a, b)
8. Maximum and Minimum Value of a Function
(or Absolute Maximum or Minimum Value)
A function f is said to attain maximum value at a point a ∈ Df , if f(a) ≥ f(x) ∀ x ∈ Df then
f(a) is called absolute maximum value of f.

A function f attains minimum value at x = b ∈ Df, if f(b) ≤ f(x) ∀ x ∈ Df then
f(b) is called absolute minimum value of f.

Note that a function ‘f ’ may have maximum (or minimum) values in some parts (intervals) of the
domain. Such values may occur at more than one point. These are therefore, called local (or relative)
maxima (or minima).
9. Local Maxima and Local Minima (or Relative Extrema)
Local Maxima: A function f(x) is said to attain a local maxima at x = a, if there exists a neighbourhood

(a – d, a + d) of ‘a’ such that f(x) < f(a) ∀ x ∈ (a – d, a + d), x ≠ a, then f(a) is the local maximum value of
f(x) at x = a.
Local Minima: A function f(x) is said to attain a local minima at x = a, if there exists a neighbourhood

(a – d, a + d) of ‘a’ such that f(x) > f(a) ∀ x ∈ (a – d, a + d), x ≠ a, then f(a) is called the local minimum
value at x = a.
Caution:
(i) A function defined in an interval can reach maximum or minimum values only for those values of x
which lie within the given interval.
(ii) One should not think that the maximum and minimum of a function are its respective largest and smallest
values over a given interval.
10. Test for Identifying Relative (Local) Maxima or Minima
(i) First Derivative Test
Step I: Find f'(x)
Step II: The equation f'(x) = 0 is solved to get critical points x = c1, c2, ........., cn.
Step III: The sign of f'(x) is studied in the neighbourhood of each critical points.

Let one critical point be x = c.

If the sign of f' (x) changes If the sign of f'(x) changes


from +ve to –ve as x increases from –ve to +ve as x increases
through c (from left to right of c). through c (from left to right of c).

x = c is relative maxima and x = c is relative minima and


f (c) is relative maximum value. f (c) is relative minimum value.

212 Xam idea Mathematics–XII

@Cbsebookshub - Join Us on Telegram



(ii) Second order derivative test
Step I: Find f'(x) = 0

Step II: The equation f'(x) = 0 is solved to get critical points x = c1, c2, ........., cn.

Step III: f"(x) is obtained and the sign of f"(x) is studied for all critical points x = c1, c2, ........., cn.

Let x = c be one critical point.

If f ′′(c) > 0 If f ′′(c) < 0 If f ′′(c) = 0

x = c is relative minima x = c is relative maxima Second derivative


and f (c) is relative and f (c) is relative test fail and first
minimum value. maximum value. derivative test is used.
11. Critical point: A point x = c is called critical point of the function f(x), if f(c) exists and either f'(c) = 0
or f'(c) = ∞ (does not exist).
12. Point of Inflexion: If f(x) is a function and x = c is critical point, then x = c is called point of inflexion if
(i) f'(c) = 0 (ii) f"(c) = 0 (iii) f′″(c) ≠ 0

XX SOME USEFUL RESULTS


™™ Area of a square = x2 and perimeter = 4x, where x is side of the square.
™™ Area of a rectangle = x . y, as x and y are length and breadth of rectangle and perimeter = 2(x + y).
1
™™ Area of a trapezium = 2 (sum of parallel sides) × perpendicular distance between them.

™™ Area of a circle = pr2, Circumference of a circle = 2pr, where r is the radius.


4
™™ Volume of sphere = 3 rr3 ; Surface area = 4pr2, where r is the radius.
™™ Total surface area of a right circular cylinder = 2prh + 2pr2; Curved surface area = 2πrh.
Volume = pr2h, where r is the radius and h is the height of the cylinder.
1
™™ Volume of a right circular cone = 3 rr2 h ; Curved surface area = prl, Total surface area = pr2 + prl,

where r is the radius, h is the height and l is the slant height of the cone.
™™ Volume of a parallelopiped = xyz and surface area = 2(xy + yz + zx), where x, y and z are the
dimensions of parallelopiped.
™™ Volume of a cube = x3 and surface area = 6x2, where x is the side of the cube.
3
™™ Area of an equilateral triangle = 4 (side)2.

Selected NCERT Questions


1. A stone is dropped into a quiet lake and waves move in circles at the speed of 5 cm/s. At the
instant when the radius of the circular wave is 8 cm, how fast is the enclosed area increasing?
Sol. Let x be the radius and A be the area of the given circle at any time t.
dx
Then = 5 cm/s and x = 8 cm
dt
dA dx dA
Now, A = rx 2 & = 2rx & dt = 2r # 8 # 5 = 80r cm2 /s
dt dt
Thus, rate of change of area is 80 π cm2/s.

Application of Derivatives 213


@Cbsebookshub - Join Us on Telegram
2.
A ladder 5 m long is leaning against a wall. The bottom of the ladder is pulled along the
ground, away from the wall, at the rate of 2 cm/s. How fast is its height on the wall decreasing
when the foot of the ladder is 4 m away from the wall? [CBSE (AI) 2012]
Sol. Let x, y be the distance of the bottom and top of the ladder respectively from the edge of the wall.
dx =
Here, dt 2 cm/s
From figure, x2 + y2 = 25
5m
When x = 4 m, y

(4)2 + y2 = 25 ⇒ y2 = 25 – 16 = 9 ⇒ y=3m
2 2
Now, x + y = 25 x

Differentiating with respect to t, we have


dx dy dy
⇒ 2x
dt
+ 2y
dt
=0 & x dx dt
+y
dt
=0
dy dy 8
⇒ 4#2+3#
dt
=0 & dt
=–
3
8
Hence, the rate of decrease of its height = cm / s
3
3. Sand is pouring from a pipe at the rate of 12 cm3/s. The falling sand forms a cone on the ground
in such a way that the height of the cone is always one-sixth of the radius of the base. How fast
is the height of the sand cone increasing when the height is 4 cm? [CBSE Delhi 2011]
1 2
Sol. Let r be the radius and h be the height of the cone so that V = rr h
3

e rr h o = 12
dV d 1 2
We have, = 12cm 3 /s ⇒ …(i)
dt dt 3
1
As h = r ⇒ r = 6h
6
Putting in (i), we get

e r (6h) 2 # h o = 12 ⇒
d 1 d
(12rh 3 ) = 12
dt 3 dt
dh dh 1
⇒ 12r # 3h 2 = 12 ⇒ =
dt dt 3rh 2
dh 1 1
when h = 4 cm, = = cm/s
dt 3r (4) 2 48 r
4. The total cost C(x) in rupees associated with the production of x units of an item is given by
C(x) = 0.007 x3 – 0.003 x2 + 15x + 4000
Find the marginal cost when 17 units are produced.
Sol. Given, C(x) = 0.007x3 – 0.003x2 + 15x + 4000
∴ Marginal cost = dx C ^xh = 0.021x2 – 0.006x + 15
d

When x = 17
Marginal cost = 0.021 × (17)2 – 0.006 × 17 + 15 = ` 20.967
5. Find the values of x for which y = [x(x – 2)]2 is an increasing function. [CBSE (AI) 2014]
2
Sol. Given, y = [x (x – 2)]
dy
dx 2 [x (x – 2)] × (2x – 2) 4x (x – 1) (x – 2)

` = =

214 Xam idea Mathematics–XII

@Cbsebookshub - Join Us on Telegram


dy –ve +ve –ve +ve
For increasing function, dx > 0
0 1 2
Sign rule
4x(x – 1)(x – 2) > 0 ⇒ x(x – 1)(x – 2) > 0
From sign rule,
dy
dx > 0 value of x = 0 < x < 1 and x > 2
For

Therefore, y is increasing ∀ x ∈ (0, 1) ∪ (2, ∞).

– i is an increasing function of i in =0, G .


4 sin i r
6. Prove that y =
(2 + cos i) 2
[CBSE (AI) 2011; (North) 2016]
4 sin i
Sol. Given, y= –i
2 + cos i
dy ^2 + cos ih .4 cos i – 4 sin i. ^0 – sin ih
= –1
dx ^2 + cos ih
2

8 cos i + 4 cos2 i + 4 sin2 i – ^2 + cos ih


2
8 cos i + 4 – 4 – cos2 i – 4 cos i
= =
^2 + cos ih ^2 + cos ih
2 2

dy 4 cos i – cos2 i cos i ^ 4 – cos ih



= =
dx ^2 + cos ih ^2 + cos ih
2 2

dy + ve × ]+ veg
= G
a i ! [0, r/2] & cos i > 0

=
dx + ve 4 – cos i is + ve as – 1 # cos i # 1
dy

dx > 0

– i is increasing function in :0, 2 D .


4 sin i r
i.e.,
y=
2 + cos i

7. Prove that the function f given by f (x) = log sin x is strictly increasing on c 0, m and strictly
r

2

decreasing on c ,r m .
r
2
1
Sol. Here, f (x) = log sin x & f l (x) = (cos x) = cot x
sin x
when x ! d 0, n then f l (x) > 0 & f (x) is strictly increasing on d 0, n .
r r
2 2

when x ! d , r n then f l (x) < 0 & f (x) is strictly decreasing on d , r n .


r r
2 2
x–7
8. Find the equation of tangent to the curve y = at the point, where it cuts the
(x – 2)(x – 3)
x-axis. [CBSE Delhi 2010; Guwahati 2015]
x–7
y=
^x – 2h^x – 3h
Sol. We have …(i)

Let (i) cuts the x-axis at (x, 0) i.e., y = 0


x–7
then
^ x – 2 h^ x – 3 h
=0 & x=7

∴ The required point is (7, 0).


Application of Derivatives 215


@Cbsebookshub - Join Us on Telegram
Differentiating equation (i) with respect to x, we get
dy ^ x – 2 h^ x – 3 h 1 – ^ x – 7 h [^ x – 2 h + ^ x – 3 h]
=
dx [^ x – 2 h^ x – 3 h]
2

x 2 – 5x + 6 – 2x 2 + 19x – 35 – x 2 + 14x – 29
= =
^ x 2 – 5x + 6 h ^ x 2 + 6 – 5x h
2 2

dy – 49 + 98 – 29
F
20 1
= = =
dx ^7, 0h ^ 49 – 35 + 6 h 2 400 20
1
∴ Equation of tangent is y – y1 =
(x – x1)
20
1

y–0= (x – 7) or x – 20y – 7 = 0
20
r
Find the equations of the tangent and normal to the curve x = a sin3 q and y = a cos3q at q =
9. .
4
[CBSE Delhi 2014]
Sol. Given, x = a sin3 q and y = a cos3 q
dx dy
&
di
= 3a sin 2 i. cos i and
di
= – 3a cos 2 i sin i

dy
dy di –3a cos 2 i. sin i
& dx = dx = = – cot i
3a sin 2 i. cos i
di
⇒ Slope of tangent to the given curve at i = 4 is ; E r = – cot 4 = –1 .
r dy r

dx i =
4
r 3 r 3 r
Since for i = , x = a sin and y = a cos
4 4 4
1 3 1 3 a a

x = ad n and y = a d n ⇒ x=
2 2
and y =
2 2
2 2
i.e., co-ordinates of the point of contact = a k
a a
,
2 2 2 2
∴ Equation of tangent is

ay – k (–1) . a x – k
a = a a = a
⇒ y– – x+
2 2 2 2 2 2 2 2
a
⇒ x+y =
2
Also slope of normal ` at i = 4 j = –
r 1 = 1 =
slope of tangent – –1 1
∴ Equation of normal is a y – k ^1 h . a x – k
a = a

2 2 2 2
a = a
⇒ y– x– ⇒ y–x=0
2 2 2 2
r
10. Find the slope of the normal to the curve x =1 – a sin θ, y = b cos2 θ at θ = .
2
Sol. Here x =1 – a sin θ and y = b cos2 θ
Differentiating both sides w.r.t. θ, we have
dx dy
= – a cos i, = – 2b sin i cos i
di di

216 Xam idea Mathematics–XII

@Cbsebookshub - Join Us on Telegram


dy dy / di – 2b sin i cos i 2b sin i
Now, = = =
dx dx / di – a cos i a
r –1 –1
Slope of normal at θ = is =
2 dy 2b
< F r < sin iF r
dx i = a i=
2 2
–a –a
   = =
r 2b
2b sin
2
r a
Thus, slope of normal at θ = is – .
2 2b
11. Find the point on the curve y = x3 – 11x + 5 at which the equation of tangent is y = x – 11.
[CBSE Delhi 2012]
Sol. Let the required point of contact be (x1, y1).
Given curve is y = x3 – 11x + 5 ...(i)
dy dy

= 3x 2 – 11 ⇒ ; E = 3x12 – 11
dx dx ^ x1, y1h

i.e., slope of tangent at (x1, y1) to give curve (i) = 3�12 – 11



From question
3 x12 – 11 = slope of line y = x – 11, which is also tangent.

i.e., 3 x12 – 11 = 1
⇒ x12 = 4 ⇒ x1 = ± 2
i.e., since (x1, y1) lie on curve (i)


y1 = x13 – 11x1 + 5
When x1 = 2, y1 = 23 – 11 × 2 + 5 = – 9
x1 = –2, y1 = (–2)3 – 11 × (–2) + 5 = 19
But (–2, 19) does not satisfy the line y = x – 11.
Therefore (2, – 9) is the required point of curve at which tangent is y = x – 11.
12. Find the equation of the tangent line of the curve y = x2 – 2x + 7 which is (a) parallel to the line
2x – y + 9 = 0, (b) perpendicular to the line 5y – 15x = 13.
Sol. Here, y = x 2 – 2x + 7
dy
= 2x – 2 = 2 (x – 1)
dx
(a) Slope of the line 2x – y + 9 = 0 is
Coefficient of x –2
– = =2
Coefficient of y –1
It is given that tangent is parallel to the line.

\
2 (x – 1) = 2 ⇒ x – 1 = 1 ⇒ x=2
2
when x = 2 then y = (2) – 2 × 2 + 7 = 4 – 4 + 7 = 7
\ Equation of tangent at (2, 7) is
y – 7 = 2 (x – 2) ⇒ y – 7 = 2x – 4 ⇒ 2x – y + 3 = 0
(b) Slope of line 5y – 15x = 13 is
Coefficient of x (–15)
– =– =3
Coefficient of y 5

Application of Derivatives 217


@Cbsebookshub - Join Us on Telegram
It is given that tangent is perpendicular to the line 5y – 15x =13
5
\ 2 (x – 1) × 3 = –1 ⇒ 6x – 6 = –1 ⇒ x =
6
5 5 2 5 25 5
when x = then y = d n – 2 # + 7 = – +7
6 6 6 36 3
25 – 60 + 252 217
= =
36 36
5 217
\ Equation of tangent at d , n is
6 36
217 1 5 36y – 217 –6x + 5
y– = – d x – n ⇒ =
36 3 6 36 18
⇒ 36y – 217 = – 12x + 10 ⇒ 12x + 36y – 227 = 0
13. Prove that the curves x = y2 and xy = k cut at right angles if 8k2 = 1. [CBSE (AI) 2008]
Sol. As we know, two curves intersect at right angles if the tangents to the curves at the point of
intersection are perpendicular to each other, i.e., product of slope of these two curves is –1.
We have x = y2
Differentiating with respect to x, we have
dy dy 1
1 = 2y dx & dx = 2y = m1 , (let) …(i)

xy = k
Also, differentiating with respect to x, we have
dy dy y
x + y.1 = 0 & = – = m2 (let) …(ii)
dx dx x
On solving the equations of the two curves
xy = k and x = y2
We get x = k2/3 and y = k1/3
Putting these values in (i) and (ii), we have
1 – k1/3
m1 = = – k –1/3
and m2 =
2k1/3 k 2/ 3
For the curves to intersect at right angles
× ^ –k –1/3 h = –1
1 1
m1 × m2 = – 1 ⇒ & =1
2k1/3 2k 2/3
1 3 1
c m = ^ k 2/3 h &
3
= k 2 & 8k 2 = 1
2 8
Hence, the result is proved.
x2 y2
14. Find the equation of the tangent and normal to the hyperbola – = 1 at the point
(x0 , y0). a2 b2
Sol. Here, equation of the given curve is
x2 y2
=1 –
a2 b2
Differentiating both sides w.r.t. x, we have
2x 2y dy dy 2x b2 b2 x
– = 0 & = – # =
a 2 b 2 dx dx a 2 –2y a 2 y
dy b2 x

at (x0, y0) = 2 0
dx a y0

218 Xam idea Mathematics–XII

@Cbsebookshub - Join Us on Telegram


\ Equation of tangent at (x0 , y0 ) is

b 2 x0 yy0 y 02 xx0 x 02
y – y0 = (x – x0) & – = –
a 2 y0 b2 b2 a2 a2

xx0 yy0 x 02 y 02 xx0 yy0 x 02 y 02



– = – & – =1 >a – = 1H
a2 b2 a2 b2 a2 b2 a2 b2
Equation of normal at (x 0 , y0 ) is
1
y – y0 = – 2 (x – x0)
b x0 /a 2 y0
a 2 y0 – a 2 y0

y – y0 = – (x – x0) & y – y0 = (x – x0)
b 2 x0 b 2 x0
y – y0 – (x – x0) y – y0 x – x0

2
= 2
& 2
+ =0
a y0 b x0 a y0 b 2 x0
15. An Apache helicopter of enemy is flying along the curve given by y = x2 + 7. A soldier, placed at
(3, 7), wants to shoot down the helicopter when it is nearest to him. Find the nearest distance.
[HOTS]
Sol. Let (x1, y1) be the position of helicopter on the curve y = x2 + 7, when the distance D from soldier
placed at (3, 7) is minimum. 2 y=x +7
^ x1 – 3 h + ^ y1 – 7 h
2 2
Now, D = ⇒ D2 = (x1 – 3)2 + (y1 – 7)2
(x1, y1 )
 (x1, y1) lie on curve y = x2 + 7


y1 = x12 + 7 D

S (3, 7)

D2 = (x1 – 3)2 + ( x12 + 7 – 7)2 ⇒ D2 = x12 – 6x1 + 9 + x14
Differentiate D2 w.r.t. x1, we get
d ^D 2 h
= 2x1 – 6 + 4x13 = 4x13 + 2x1 – 6
dx1
d ^D 2 h
Now, for maximum or minimum distance = 0
dx1


4 x13 + 2x1 – 6 = 0 ⇒ 4 x12 (x1 – 1) + 4x1 (x1 – 1) + 6 (x1 – 1) = 0
⇒ (x1 – 1)(4 x12 + 4x1 + 6) = 0
⇒ x1 – 1 = 0 or 4 x12 + 4x1 + 6 = 0

x1 = 1 [4 x12 + 4x1 + 6 have no real roots]

d 2 ^D 2 h d 2 ^D 2 h
Again = 12x12 + 2 ⇒ H =+ ve
dx12 dx12 x1 = 3

Hence, for x1 = 1, D2 is minimum, i.e., D is minimum.


Also, for x1 = 1, y1 = 12 + 7 = 8
Minimum distance, D = ^1 – 3 h + ^8 – 7 h = 5 unit.
2 2


Thus, when helicopter is at (1, 8) then it is at nearest distance 5 unit from soldier.
16. Find the local maximum or minimum if any of the function f (x) = sin x – cos x, 0 < x < 2π. Also,
find the local extrema values.
Sol. f ’(x ) = cos x + sin x = 0 ⇒ cos x + sin x = 0

Application of Derivatives 219


@Cbsebookshub - Join Us on Telegram
3r 7r

tan x = – 1 ⇒ x= ,
4 4
3r

At x = , f ’(x) = cos x + sin x,
4
3r
when x < , f ’(x) = + ve
4
3r 3r
when x > , f ’(x) = – ve ⇒ x= is a point of local maxima.
4 4
7r
At x = , f ’(x)= cos x + sin x
4
7r
when x < , f ’(x)= – ve
4
7r 7r
when x > , f ’(x)= + ve ⇒ x= is a point of local minima.
4 4
3r 1
– e– o = 2 is the local maximum value.
3r 3r 1
fd n = sin – cos =
4 4 4 2 2
7r 7r 7r 1 1
f d n = sin – cos =– – = – 2 is the local minimum value.
4 4 4 2 2
17. Find the absolute maximum value and the absolute minimum value of the following function
in the given interval.
9
f (x) = 4x – x 2, x d <– 2, F
1

2 2
1 2 9
Sol. Here, f (x) = 4x –x , x d <–2, F & f ’(x ) = 4 – x
2 2
For maximum or minimum f ’(x ) = 0 ⇒ 4–x=0⇒x=4
1
f (–2) = 4 (–2) – (– 2)2 = – 8 – 2 = – 10
2
1
f (4) = 4 (4) – (4)2 = 16 – 8 = 8
2
9 9 1 9 2 81 63
f d n = 4 d n – d n = 18 – =
2 2 2 2 8 8
Thus, absolute maximum value is 8 at x = 4 and absolute minimum value is –10 at x = –2.
18. A rectangular sheet of tin 45 cm by 24 cm is to be made into a box without top, by cutting off
square from each corner and folding up the flaps. What should be the side of the square to be
cut off so that the volume of the box is maximum?
Sol. Let x cm be the length of square, then volume of the box,
V = (45 – 2x) (24 – 2x). x
dV
= (– 2) (24 – 2x) x + (45 – 2x) (– 2) x + (45 – 2 x) (24 – 2x) . 1
dx x x
dV 45 – 2x
= – 48x + 4x 2 – 90x + 4x 2 + 1080 – 138x + 4x 2 x x
dx
= 12x 2 – 276x +1080
24 –2x

= 12 (x 2 – 23x + 90) = 12 (x – 18) (x – 5)


dV
For maximum or minimum, =0 x x
dx
& 12 (x – 18) (x – 5) = 0
⇒ x = 5,18 x x

x =18 is not possible as breadth is 24 cm.


220 Xam idea Mathematics–XII

@Cbsebookshub - Join Us on Telegram


Now, d 2 V = 12 (1) (x – 5) + 12 (x – 18). 1
dx 2
d2 V
f p = 0 + 12 (5 – 18) = – 156 < 0
dx 2 x=5

x = 5 is the point of maximum.
Thus, 5 cm is the side of the square to be cut off from rectangle for box of maximum volume.
19. Prove that the volume of the largest cone that can be inscribed in a sphere of radius a is 8
27
[CBSE Delhi 2016; (AI) 2014; (F) 2013]
of the volume of the sphere.  
OR
Show that the altitude of the right circular cone of maximum volume that can be inscribed in
4R
a sphere of radius R is . [CBSE (F) 2012]
3
[Hint: replace a by R and you can get the result]
Sol. Consider a sphere of radius a with centre at O such that OD = x and DC = r.
Let h be the height of the cone.
A
Then h = AD = AO + OD = a + x ...(i)
(OA = OC = radius)
In the right angle DODC, O
a
x
2 2 2
a = r + x (By Pythagoras theorem) ...(ii)
B r C
D
1
Let V be the volume the cone, then V = rr 2 h
3
1
⇒ V ^ x h = r ^ a – x h^ a + x h
2 2
[From (i) and (ii)]
3
1 d d
V' (x) = r ;^ a 2 – x 2 h ^ a + x h + ^ a + x h ^ a 2 – x 2 hE

3 dx dx
1 1 1
= r [(a 2 – x 2) (1) + (a + x) (– 2x)] = r [(a + x) (a – x – 2x)] = r (a + x) (a – 3x)
3 3 3

Also, V" (x) = r =(a + x) (a – 3x) + (a – 3x) (a + x)G


1 d d
3 dx dx
1

V" (x) =
r [(a + x) (– 3) + (a – 3x) (1)]
3
For maximum or minimum value, we have V'(x) = 0
1 a
r ^ a + x h^ a – 3x h = 0 & x = – a or x = 3
3
Neglecting x = – a [ x > 0]

V" a k = r :a a + k^ – 3 h + a a – 3 a kkD =
a 1 a a – 4r a
<0
3 3 3 3 3
a
∴ Volume is maximum when x = .
3
a a 4a a 2 8a 2
Putting x = in equation (i) and (ii), we get h = a + = and r 2 = a 2 – =
3 3 3 9 9
1 8a 2 4a
rr h = r d nc m =
1 2 8 4 3
Now, volume of cone = c ra m
3 3 9 3 27 3
8
Thus, volume of the cone = (volume of the sphere).
27

Application of Derivatives 221


@Cbsebookshub - Join Us on Telegram
20. Show that the right-circular cone of least curved surface and given volume has an altitude
equal to 2 times the radius of the base. [CBSE (AI) 2011] [HOTS]
Sol. Let ABC be right-circular cone having radius ‘r’ and height ‘h’. If V and S are its volume and
surface area (curved) respectively, then
S = prl
S = rr h 2 + r 2 …(i) A
1 3V
Also, V = rr 2 h ⇒ h=
3 rr 2 l
h
Putting the value of h in (i), we get
B r C
9V 2 2
D
S = rr +r
r2 r4
9V 2 + r 2 r 6

S2 = r2 r2 e o [Maxima or Minima is same for S or S2]
r2 r4
9V 2

S2 = 2
+ r2 r4
r
Differentiating with respect to ‘r’, we get
– 18V 2

^S 2 h ' = + 4r 2 r 3 …(ii)
r3
V2
Now, for max. or min. (S2)' = 0 ⇒ – 18 + 4r 2 r 3 = 0 ⇒ 4p2r6 = 18V2
r3
Putting value of V
1 2 4 2 h

4r 2 r6 = 18 × r r h ⇒ 2r2 = h2 ⇒ r=
9 2
Differentiating (ii) with respect to ‘r’, again
54V 2
^S 2 h " = + 12r 2 r 2
r4

^S 2 h "] h > 0 (For any value of r)
r=
2

h
Hence, S2 i.e., S is minimum for r = or h = 2 r
2
i.e., for least curved surface, altitude is equal to
2 times the radius of the base.
21. Show that the semi-vertical angle of the cone of the maximum volume and of given slant
height is tan–1 2 .
Sol. Let a be the semi-vertical angle of a cone and slant height is l.
then AO = l cos a and BO = l sin a
1 1
⇒ V = r (l sin a) 2 (l cos a) = rl 3 sin 2 a cos a
3 3
= l 6– sin 3 a + 2 sin a cos 2 a@
dV r 3

da 3
For maximum or minimum volume V, we have
dV r
= 0
da
& 3 l3 sin a (– sin2 a + 2 cos2 a) = 0 & 2 cos2 a – sin2 a = 0
1
⇒ 2 cos2 a = sin 2 a ⇒ tan a = 2 ⇒ cos a =
3

222 Xam idea Mathematics–XII

@Cbsebookshub - Join Us on Telegram


d2 V
rl 62 cos 3 a – 7 sin 2 a cos a@ = rl 3 cos 3 a (2 – 7 tan 2 a)
1 3 1
Now, =
da 2 3 3
3
1 3 1
rl e o ( 2 – 7 # 2) 1 0
=
3 3
Thus, V is maximum when tan α = 2 , α = tan –1 2.
22. Show that the semi-vertical angle of the right circular cone of given total surface area and
1
maximum volume is sin –1 3 . [CBSE (AI) 2008]
Sol. Let r, h, l and a be the radius, height, slant height and semi-vertical angle of cone respectively.
If V be the volume of cone then
1 1
V = rr 2 h ⇒ V = rr 2 l 2 – r 2
3 3 
2 2 2
d n – r 2 , where Area S = rrl + rr 2
1 2 S – rr S – rr
⇒ V =

3
rr
rr
& l=
rr h
l

rr 2 S 2 – 2Srr 2 + r 2 r 4 – r 2 r 4 r
⇒ V =
× = × S 2 – 2Srr 2
3 rr 3 r

r2 2 ^S 2 r 2 – 2Srr 4h

V2 = ^S – 2Srr 2h =
9 9
Differentiating with respect to r, we get
d ^V 2h 1
= ^2S 2 r – 8Srr 3h
dr 9
For maxima or minima
d ^V 2 h 1
= 0 ⇒ ^2S 2 r – 8Srr 3h = 0
dr 9
⇒ 2S2r – 8Spr3 = 0
⇒ 2Sr (S – 4pr2) = 0
S

S – 4pr2 = 0 ⇒ 4rr 2 = S & r=
4r
d 2 ^V 2h 1
Again = ^2S 2 – 24Srr 2h
dr 2 9
d V2h
2^
G
1 S 1

= c 2S 2 – 24Sr. m = ^ 2S 2 – 6S 2 h < 0
dr 2
r=
S 9 4r 9
4r

S
, V 2 is maximum i.e., V is maximum.
Hence, for r =
4r
⇒ For S = 4pr2, V is maximum.

Now, since S = prl + pr2 ⇒ 4pr2 = prl + pr2 [For maximum volume S = 4pr2]
rr 2 1
⇒ 3pr2 = prl
⇒ =
rrl 3

=a sin a = G
r 1 1 r
&
=
l 3
& sin a =
3 l

& a = sin –1 e 3 o
1

Hence, for maximum volume a (semi-vertical angle) = sin –1 b 3 l


1

Application of Derivatives 223


@Cbsebookshub - Join Us on Telegram
x2 y2
23. Find the maximum area of the isosceles triangle inscribed in the ellipse +
= 1 with its
a2 b2
vertex at one end of major axis. [CBSE Bhubaneshwar 2015, (AI) 2008]
Sol. Let DABC be an isosceles triangle inscribed in y-axis
the ellipse.
x2 y2 A (a cos , b sin )
+ =1
a2 b2
Such that ‘C’ lies on end of major axis and

AC = BC. D C
x-axis
O
Let coordinates of A and B be (a cos q, b sin q)
and (a cos q, –b sin q) respectively.
B (a cos , – b sin )
If ‘A’ be the area of inscribed triangle then
A = 2 × AB × CD = 2 × 2b sin i× ^a – a cos ih
1 1

= ab sin q (1 – cos q)
Differentiating with respect to q, we get
dA = 6
ab sin i. sin i + ^1 – cos ih .cos i@ = ab(sin2 q + cos q – cos2 q)
di
dA =
For maxima and minima 0
di

ab (sin2 q + cos q – cos2 q) = 0
⇒ cos q – cos 2q = 0


cos 2q = cos q
⇒ 2q = 2np ± q [ cos q = cos a; q = 2np ± a]

i i

i = nr + 2 or nr – 2 , where n = 0, ± 1, ± 2, ....

i = 3 ! ^0, rh
2r

d2 A

= ab (2 sin i. cos i – sin i + 2 cos i. sin i) = ab (2 sin 2i – sin i)
di 2
d2 A
= G <0
di 2 i=
2r
3
2r
Hence, for i = , A is maximum.
3
2r 2r 3 1 3 3
Hence, maximum area of triangle A = ab sin . c 1 – cos m = ab c1 + m = ab sq units.
3 3 2 2 4
24. A window is in the form of rectangle surmounted by a semi-circular opening. Total perimeter
of the window is 10 m. What will be the dimensions of the whole opening to admit maximum
light and air through the whole opening? [CBSE 2006; (AI) 2017; (F) 2011, 2014]

Sol. Let ABCED be required window having length and width y. If A is the area of window. Then
RS VW
SS 1 W

1
A = 2xy + rr 2 SSGiven perimeter = 10 & 2x + y + y + 2rx = 10WWW
2 SS 2 WW
S & 2 y = 10 – 2x – rx W
T X

224 Xam idea Mathematics–XII

@Cbsebookshub - Join Us on Telegram


1 2 1
= x ^10 – 2x – rx h +
rx = 10x – 2x 2 – rx 2 + rx 2
2 2
1 1
= 10x – 2x 2 – rx 2 = 10x – c 2 + r m x 2
2 2
Obviously, window will admit maximum light and air if its area A is maximum.
dA 1
Now, = 10 – 2x c 2 + r m E
dx 2
dA
For maxima or minima of A, dx = 0 .
1
10 – 2x c 2 + r m = 0
D C

2
& 10 – x (4 + r) = 0
y
10 d2 A

x= and = – ( 4 + r) < 0
4+r dx 2 A B
2x
10 10
⇒ For maximum value of A, x = and thus y =
4+r 4+r
Therefore, for maximum area, i.e., for admitting maximum light and air,
20 10
Length = 2x = and width = of rectangular part of window.
4+r 4+r

Multiple Choice Questions [1 mark]


Choose and write the correct option in the following questions.
1. The interval in which the function f given by f(x) = x2 e–x is strictly increasing, is
 [CBSE 2020 (65/2/1)]
(a) (– ∞, ∞) (b) (– ∞, 0) (c) (2, ∞) (d) (0, 2)
2
2. y = x (x – 3) decreases for the values of x given by
3
(a) 1 < x < 3 (b) x < 0 (c) x > 0 (d) 0 < x <
2
3. The abscissa of the point on the curve 3y = 6x – 5x3, the normal at which passes through origin is
1 1
(a) 1 (b) (c) 2 (d)
3 2
1/5
4. The curve y = x has at (0, 0)
(a) a vertical tangent (parallel to y-axis) (b) a horizontal tangent (parallel to x-axis)
(c) an oblique tangent (d) no tangent
5. The equation of normal to the curve 3x2 – y2 = 8 which is parallel to the line x + 3y = 8 is
(a) 3x – y = 8 (b) 3x + y + 8 = 0 (c) x + 3y ± 8 =0 (d) x + 3y = 0
6. The tangent to the curve y = e2x at the point (0, 1) meets x-axis at
(b) b – 2 , 0 l (c) (2, 0)
1
(a) (0, 1) (d) (0, 2)

7. f(x) = xx has a stationary point at


1
(a) x = e (b) x = e (c) x = 1 (d) x = e

1 x
8. The maximum value of d n is
x

(a) e (b) ee (c) e1/e (d) b e l


1 1/e

Application of Derivatives 225


@Cbsebookshub - Join Us on Telegram
9. The two curves x3 – 3xy2 + 2 = 0 and 3x2y – y3 = 2 [NCERT Exemplar]
r r
(a) touch each other (b) cut at right angle (c) cut at an angle 3 (d) cut at an angle 4
r
10. The tangent to the curve given by x = et.cos t, y = et. sint at t = 4 makes with x-axis an angle
r r r
(a) 0 (b) 4 (c) 3 (d) 2
11. The slope of normal to the curve y = 2x2 + 3 sin x at x = 0 is
1 1
(a) 3 (b) 3 (c) –3 (d) – 3
12. The equation of the normal to the curve y = sinx at (0, 0) is
(a) x = 0 (b) y = 0 (c) x + y = 0 (d) x – y = 0
2 r
13. The point on the curve y = x, where the tangent makes an angle of 4 with x-axis is
[NCERT Exemplar]
1 1 1 1
(a) d , n (b) d , n (c) (4, 2) (d) (1, 1)
2 4 4 2
14. The point on the curve x2 = 2y which is nearest to the point (0, 5) is
(a) ^2 2 , 4 h (b) ^2 2 , 0 h (c) (0, 0) (d) (2, 2)
15. The maximum value of [x (x – 1) + 1]1/3, 0 ≤ x ≤ 1 is
1 1/3
(a) d n (b) 2 (c) 1
1
(d) 0
3
16. The line y = x +1 is a tangent to the curve y2 = 4x at the point
(a) (1, 2) (b) (2, 1) (c) (1, – 2) (d) (–1, 2)
17. A ladder, 5 meter long, standing on a horizontal floor, leans against a vertical wall. If the top of
the ladder slides downwards at the rate of 10 cm/sec, then the rate at which the angle between
the floor and the ladder is decreasing when lower end of ladder is 2 metres from the wall is
[NCERT Exemplar]
1 1
(a) radian/sec (b) radian/sec (c) 20 radian/sec (d) 10 radian/sec
10 20
18. The rate of change of the area of a circle with respect to its radius r at r = 6 cm is
(a) 10p (b) 12p (c) 8p (d) 11p
19. The total revenue in rupees received from the sale of x units of a product is given by
R(x) = 3x2 + 36x + 5. The marginal revenue, when x = 15 is
(a) 116 (b) 96 (c) 90 (d) 126
2
20. If x is real, the minimum value of x – 8x + 17 is [NCERT Exemplar]
(a) – 1 (b) 0 (c) 1 (d) 2

Answers
1. (d) 2. (a) 3. (a) 4. (a) 5. (c) 6. (b)
7. (b) 8. (c) 9. (b) 10. (d) 11. (d) 12. (c)
13. (b) 14. (a) 15. (c) 16. (a) 17. (b) 18. (b)
19. (d) 20. (c)

Solutions of Selected Multiple Choice


1. We have,  f(x) = x2 e–x

f ′(x) = –x2 e–x + 2x e–x = x e–x (2 – x)

226 Xam idea Mathematics–XII

@Cbsebookshub - Join Us on Telegram


for f(x) to be strictly increasing, f ′(x) > 0
⇒  x e–x (2 – x) > 0
⇒  x (2 – x) > 0
⇒  x (x – 2) < 0
⇒  0 < x < 2
∴  x ∈ (0, 2)
2. We have, y = x(x – 3)2
dy
= x.2(x – 3).1 + (x – 3)2.1
dx
= 2x2 – 6x + x2 + 9 – 6x = 3x2 – 12x + 9
= 3(x2 – 3x – x + 3) = 3(x – 3)(x – 1)
+ – +
1 3
So, y = x(x – 3)2 decreases for (1, 3).
[Since, y' < 0 for all x ∈ (1, 3), hence y is decreasing on (1, 3)]
3. Let (x1, y1) be the point on the given curve 3y = 6x – 5x3 at which the normal passes through the
dy
origin. Then we have d n = 2 – 5x12 . Again the equation of the normal at (x1, y1) passing
dx (x1, y1)
–x –3
through the origin gives 2 – 5x12 = 1 = . Since x1 = 1 satisfies the equation, therefore,
y1 6 – 5x 2
1
correct answer is (a).
y = x1/5
4. We have,
dy 1 1 – 1 1 – 4/5

= x5 = x
dx 5 5
dy
d n = × ]0 g–4/5 = 3
1


dx (0, 0) 5
So, the curve y = x1/5 has a vertical tangent at (0, 0), which is parallel to y-axis.
7. We have,    f(x) = xx

Let y = xx
and log y = xlog x
1 dy = 1 +

y . dx x. x log x.1 [Differentiate both sides]

dy

= _1 + log x i .x x
dx
dy
    dx = 0 ⇒ (1 + logx). xx = 0
⇒ logx = – 1 ⇒ logx = loge–1
1
⇒ x = e–1 ⇒ x= e
1
Hence, f(x) has a stationary point at x = e .

1 x
8. Let y =d n
x
1

logy = x.log
x

Application of Derivatives 227


@Cbsebookshub - Join Us on Telegram
1 dy 1 1 1 1

. = x. . e – 2 o + log . 1 = –1 + log [Differentiate both sides]
y dx 1 x x x
x
dy 1 1 x
⇒   = d log – 1 n . d n

dx x x
dy 1
Now,   =0 ⇒ log = 1 = loge
dx x
1

=e
x
1

x=
e
1
Hence, the maximum value of f d n = ] e g .
1/e
e
dy
9. From first equation of the curve, we have 3x 2 – 3y 2 – 6xy =0
dx
dy x 2 – y 2

= = (m1) say and second equation of the curve gives
dx 2xy
dy dy dy – 2xy
6xy + 3x 2
– 3y 2 =0 ⇒ = 2 = _ m2 i say
dx dx dx x – y 2
Since m1 . m2 = – 1. Therefore, correct answer is (b).
dy –1
12. dx = cos x. Therefore, slope of normal = d n = –1 .
cos x x = 0
Hence, the equation of normal is y – 0 = –1 (x – 0) or x + y = 0.
Therefore, correct answer is (c).

Fill in the Blanks [1 mark]


1. If the radius of the circle is increasing at the rate of 0.5 cm/s, then the rate of increase of its
circumference is _____________ . [CBSE 2020 (65/2/1)]
2. The values of a for which the function f(x) = sin x – ax + b increases on R are _____________ .
3. The rate of change of volume of a sphere with respect to its surface area, when radius is 2 cm,
is _____________ .
4. The equation of the tangent to the curve y = sec x at the point (0, 1) is _____________ .
 [CBSE 2020 (65/3/1)]
b
5. The least value of the function f(x) = ax + x , (a > 0, b > 0, x > 0) is _____________ .

Answers
1. p cm/sec. 2. (– ∞, –1) 3. 1 cm3/cm2 4. y = 1 5. 2 ab

Solutions of Selected Fill in the Blanks


dr
1. Given, = 0.5 cm/s
dt
dC d2rr dr

= = 2r = 2p × 0.5 cm/s
dt dt dt
= p cm/sec.

228 Xam idea Mathematics–XII

@Cbsebookshub - Join Us on Telegram


4. We have, slope of the tangent to the curve y = sec x is given by
dy
= sec x . tan x
dx
∴ Slope of tangent at (0, 1) = sec 0 × tan 0 = 0
∴ Equation of tangent at the point (0, 1) be
y–1
= 0   ⇒  y – 1 = 0  ⇒  y = 1
x–0

Very Short Answer Questions [1 mark]


r
1. Find the slope of the tangent to the curve y = 2 sin2 (3x) at x = . [CBSE 2020 (65/5/1)]
6
Sol. We have,
y = 2 sin2 (3x)
dy

= 2 × 2 sin (3x) × cos (3x) × 3
dx
dy

= 6 sin (6x)
dx

∴  Slope of tangent at x = is 6 sin c 6 # m


r r

6 6
   = 6 sin p = 6 × 0 = 0
2. Find the rate of change of the area of a circle with respect to its radius ‘r’ when r = 4 cm.
[NCERT Exemplar]
Sol. If A is area and r is the radius of a circle, then
A = pr2 dA
& dr
= 2rr

dA

` = G = 8r cm 2 /cm
dr r = 4
3. An edge of a variable cube is increasing at the rate of 5 cm per second. How fast is the volume
increasing when the side is 15 cm?
Sol. Let x be the edge of the cube and V be the volume of the cube at any time t.
dx
Given, = 5 cm/s, x = 15 cm
dt
Since we know the volume of cube = (side)3 i.e., V = x3.
dV dx
&

dt
= 3x 2 $
dt
& dV dt
= 3 $ (15) 2 # 5 = 3375 cm 3 / sec

4. Find the slope of the tangent to the curve x = t2 + 3t – 8, y = 2t2 – 2t – 5 at t = 2 .


dy
dy 4t – 2
Sol. Slope of the tangent = = dt =
dx dx 2t + 3
dt
dy 4t – 2 4^2h – 2 6
⇒ =d n = =
dx at t = 2 2t 3 at t = 2 2 ^ 2 h + 3 7
+
5. If y = loge x, then find Dy when x = 3 and Dx = 0.03.
Sol. We have, y = loge x
dy 1 0.03
∴ Dy = $ Dx = $ Dx = = 0.01
dx x 3

Application of Derivatives 229


@Cbsebookshub - Join Us on Telegram
Short Answer Questions-I [2 marks]
1. The money to be spent for the welfare of the employees of a firm is proportional to the rate of
change of its total revenue (marginal revenue). If the total revenue (in rupees) received from
the sale of x units of a product is given by R(x) = 3x2 + 36x + 5, find the marginal revenue when
x = 5. [CBSE (AI) 2013]
Sol. Given: R(x) = 3x2 + 36x + 5

R'(x) = 6x + 36
∴ Marginal revenue (when x = 5) = R'(x)]x = 5

= 6 × 5 + 36 = ` 66.
2. The amount of pollution content added in air in a city due to x-diesel vehicles is given by
P(x) = 0.005x3 + 0.02x2 + 30x. Find the marginal increase in pollution content when 3 diesel
vehicles are added. [CBSE Delhi 2013]
Sol. We have to find [P'(x)]x = 3
Now, P(x) = 0.005x3 + 0.02x2 + 30x

P'(x) = 0.015x2 + 0.04x + 30
⇒ [P'(x)]x = 3 = 0.015 × 9 + 0.04 × 3 + 30

= 0.135 + 0.12 + 30 = 30.255
3. If x and y are the sides of two squares such that y = x – x2, then find the rate of change of the
area of second square with respect to the area of first square. [NCERT Exemplar]
Sol. Since, x and y are the sides of two squares such that y = x – x2.
∴ Area of the first square (A1) = x2

and area of the second square (A2) = y2 = (x – x2)2
dA2 d dx dx

= ( x – x 2 ) 2 = 2 ( x – x 2) d – 2x. n
dt dt dt dt
dx
= (1 – 2x) 2 (x – x 2)
dt
dA1 d 2 dx
=
dt = dx x 2x. dt
and

dx
dA2 dA2 /dt . (1 – 2x) (2x – 2x 2)

= = dt
dA1 dA1 /dt dx
2 x.
dt
(1 – 2x) 2x (1 – x)
= 2x
= (1 – 2x)(1 – x) = 1 – x – 2x + 2x2 = 2x2 – 3x + 1
4. Using differentials, find the approximate value of 49.5 . [CBSE Delhi 2012]
Sol. Let f ^xh = x, where x = 49 and dx = 0.5

f ^x + dxh = x + dx = 49.5
Now by definition, approximately we can write
f (x + dx) – f (x)
f l (x) = ...(i)
dx
Here f (x) = x = 49 = 7 and dx = 0.5

230 Xam idea Mathematics–XII

@Cbsebookshub - Join Us on Telegram


1 1 1

f l (x) = = =
2 x 2 49 14
Putting these values in (i), we get
1 49.5 – 7
=
14 0.5
0.5 0.5 + 98 98.5
49.5 = +7 = = = 7.036
14 14 14

5. Show that the function f given by f(x) = tan–1 (sin x + cos x) is decreasing for all �d d , n.
r r

4 2
[CBSE (F) 2017]
Sol. We have
f (x) = tan–1 (sin x + cos x)
1

f l (x) = × (cos x – sin x)
1 + (sin x + cos x) 2
cos x – sin x
f l (x) =
1 + (sin x + cos x) 2


 1 + (sin x + cos x )2 > 0 ∀x ∈ R

Also, 6x ! c , m sin x > cos x


r r
⇒ cos x – sin x < 0
4 2
–ve
∴ f l (x) = = – ve i.e., f ’(x) < 0
+ ve
f (x) is decreasing in c , m
r r

4 2
6. The volume of a cube is increasing at the rate of 9 cm3/s. How fast is its surface area increasing
when the length of an edge is 10 cm? [CBSE (AI) 2017]
Sol. Let V and S be the volume and surface area of a cube of side x cm respectively.
dV
Given = 9 cm3/sec
dt
dS
We require F
dt x = 10 cm
Now V = x3
dV dx dx
⇒ = 3x 2 . ⇒ 9 = 3x 2 .
dt dt dt
dx 9 3
⇒ = 2 = 2
dt 3x x
Again,  S = 6x2 [By formula for surface area of a cube]
dS dx
⇒ = 12.x.
dt dt
3 36
  = 12x. 2 = x
x
dS
F
36
⇒ = = 3.6 cm2/sec.
dt x = 10 cm 10
1
7.
Find the approximate change in the value of , when x changes from x = 2 to x = 2.002.
x2
 [CBSE Sample Paper 2018]

Application of Derivatives 231


@Cbsebookshub - Join Us on Telegram
1
Sol. Let y =. Let δx be small change in x and δy the corresponding change in y.
x2
Given δx = 2.002 – 2 = 0.002, where x = 2.
1 dy 2 dy 2 2
Now y = & dx
=– 3 & dx
=– 3 =–
8
x2 x 2

We know that, approximately,
dy 2
δy = dx .dx ∴ dy = – 8 × 0.002 = – 0.0005
r
8. Find whether the function f (x) = cos (2x + 4 ); is increasing or decreasing in the interval
3r 7r
d , n .  [CBSE 2019(65/5/3)]
8 8

Sol. We have f (x) = cos d 2x + n


r
4
We know that function f(x) is increasing in (a, b) if f '(x) > 0 ∀ x ∈ (a, b) & is decreasing if f '(x) < 0
∀ x ∈ (a, b).
Now, f '(x) = – 2 sin b 2x + 4 l
r

–2 sin b 2x + 4 l > 0
r
∴ f '(x) > 0 ⇒
sin b 2x + 4 l < 0
r

[a sin x < 0 6 x e ^r, 2r h]


r

r < 2x + < 2r
4
r r 3r 7r

< 2x < 2r –
r– ⇒ < 2x <
4 4 4 4
3r 7r

<x<
8 8
3r 7r
Thus f ' (x) > 0 6 x e d ,
n
8 8
3r 7r
⇒ f(x) is increasing function on d
, n.
8 8

Short Answer Questions-II [3 marks]


1. Prove that the curves xy = 4 and x2 + y2 = 8 touch each other. [NCERT Exemplar]
Sol. Given equation of curves are
xy = 4 ... (i)
and x 2 + y 2 = 8 ... (ii)
dy

x. +y = 0
dx
and dy
2x + 2y =0
dx
dy –y dy –2x

= and =
dx x dx 2y
dy –y

= = m1 (say)
dx x
dy –x
and = = m2 (say)
dx y
∴ Both the curves should have same slope.

232 Xam idea Mathematics–XII

@Cbsebookshub - Join Us on Telegram


–y –x


x
=
y
& –y 2 = –x 2 and x 2 = y 2 ... (iii)

Using the value of x2 in equation (ii), we get


y2 + y2 = 8 ⇒ y2 = 4 & y = ! 2
4 4
For = 2 and for y = –2, x =
y = 2, x = = –2
2 –2
Thus, the required points of intersection are (2, 2) and (–2, –2)
–y –2
For (2, 2), m1 = = = –1
x 2
–x –2
and m2 = = = –1
y 2

 m1 = m2
–y – (–2)
For (–2,
– 2), m1 = x = –2 = –1
–x – (–2)

 m2 = = = –1
y –2
For both the intersection points, we see that slope of both curve are same.
3 4
2. Find the intervals in which the function f ^ x h = x – 4x 3 – 45x 2 + 51 is
2
(a) strictly increasing (b) strictly decreasing. [CBSE (F) 2014]
3 4
Sol. Here, f ^ x h = x – 4x 3 – 45x 2 + 51
2
⇒ f '(x) = 6x3 – 12x2 – 90x ⇒ f '(x) = 6x(x2 – 2x – 15) = 6x(x + 3) (x – 5)
Now for critical point f (x) = 0
6x(x + 3)(x – 5) = 0 ⇒ x = 0, –3, 5
i.e., –3, 0, 5 are critical points which divides domain R of given function into four disjoint sub

intervals (– ∞, –3), (–3, 0), (0, 5), (5, ∞).
For (– ∞, –3) –ve +ve –ve +ve
–3 0 5
f (x) = +ve × (–ve) × (–ve) × (–ve) = –ve
i.e., f(x) is decreasing in (– ∞, –3).
For (– 3, 0)
f (x) = +ve × (–ve) × (+ve) × (–ve) = +ve
i.e., f(x) is increasing in (– 3, 0).
For (0, 5)
f'(x) = +ve × (+ve) × (+ve) × (–ve) = –ve
i.e., f(x) is decreasing in (0, 5).
For (5, ∞)
f'(x) = +ve × (+ve) × (+ve) × (+ve) = +ve
i.e., f(x) is increasing in (5, ∞).
Hence, f(x) is (a) strictly increasing in (–3, 0) ∪ (5, ∞)
(b) strictly decreasing in (– ∞, –3) ∪ (0, 5).

Application of Derivatives 233


@Cbsebookshub - Join Us on Telegram
3. Find the intervals in which f (x) = sin 3x – cos 3x, 0 < x < r , is strictly increasing or strictly
decreasing. [CBSE Delhi 2016]
Sol. Given function is
f (x) = sin 3x – cos 3x
f l (x) = 3 cos 3x + 3 sin 3x
For critical points of function f(x)
f l (x) = 0

3 cos 3x + 3 sin 3x = 0 ⇒ cos 3x + sin 3x = 0
sin 3x

sin 3x = – cos 3x ⇒ = –1
cos 3x

tan 3x = tan c r – m
r r

tan 3x = – tan ⇒
4 4
3r
⇒ tan 3x = tan
4
3r
⇒ 3x = nr + , where n = 0, ! 1, ! 2, ....
4
Putting n = 0, ! 1, ! 2, .... , we get
r 7r 11r
x= , , ! (0, r)
4 12 12
Hence, required possible intervals are c 0, m, c , mc , mc ,rm
r r 7r 7r 11r 11r
4 4 12 12 12 12

For c 0, m, f l (x) =+ ve
r
4

For c , m, f l (x) = –ve


r 7r
4 12

For c , m, f l (x) =+ ve
7r 11r
12 12

For c , r m, f l (x) = –ve


11r
12
Hence, given function f(x) is strictly increasing in c 0, m j c , m and strictly decreasing in
r 7r 11r
4 12 12
c , mjc , r m.
r 7r 11r
4 12 12
4. Find the equation of the normal at the point (am2, am3) for the curve ay2 = x3.
[CBSE (F) 2012; (South) 2016]
Sol. Given, curve ay2 = x3
On differentiating, we get
dy dy 3x 2
2ay
dx
= 3x 2 & =
dx 2ay
dy 3 # a 2 m 4 3m

= =
dx
at (am 2 , am 3) 2a # am 3 2
1 1 2
∴ Slope of normal = – =– =–
slope of tangent 3m 3m
2
2 3
Equation of normal at the point (am , am ) is given by
y – am 3 2
=– ⇒ 3my – 3am 4 = – 2x + 2am 2
x – am 2 3m

234 Xam idea Mathematics–XII

@Cbsebookshub - Join Us on Telegram


⇒ 2x + 3my – am2(2 + 3m2) = 0

Hence, equation of normal is 2x + 3my – am2(2 + 3m2) = 0
5. Show that y = log ^1 + x h – + , x > – 1 is an increasing function of x throughout its domain.
2x

2 x
[CBSE (F) 2012]
Sol. Here, f ^xh = log ^1 + xh – +
2 x
[where y = f(x)]
2 x
(2 + x) .1 – x
– 2> H
1

f l ( x) =
1+x (2 + x) 2
1 2 (2 + x – x) 1 4
= – = –
1+x (2 + x) 2 1 + x (2 + x) 2

4 + x 2 + 4x – 4 – 4x x2
= =
^x + 1h^x + 2h ^x + 1h^x + 2h
2 2

For f(x) being increasing function


f '(x) > 0
x2 1 x2
&
>0 & . >0
(x + 1)(x + 2) 2 x + 1 (x + 2) 2

x2
> > 0H
1
&

+
>0
x 1 (x + 2) 2
⇒ x + 1 > 0 or x>–1
2x
i.e., f ^ x h = y = log ^1 + x h –
is increasing function in its domain x > – 1 i.e., (–1, ∞).
2+x
6. Show that f(x) = 2x + cot–1x + log _ 1 + x 2 – x i is increasing in R. [NCERT Exemplar]
Sol. We have, f(x) = 2x + cot–1x + log _ 1 + x 2 – x i
–1 1
e . 2x – 1 o
1
f ' (x) = 2 + e 2 o+
1+ x _ 1 + x – x i 2 1 + x2
2

1 1 _ x – 1 + x2 i 1 1
= 2 – + . =2– –
1+ x 2
_ 1 + x – xi
2
+
1 x 2
1 + x2 1 + x2
2 + 2x 2 – 1 – 1 + x 2 1 + 2x 2 – 1 + x 2
= =
1 + x2 1 + x2
For increasing function, f ' (x) ≥ 0
1 + 2x 2 – 1 + x 2
$ 0 ⇒ 1 + 2x 2 $ 1 + x 2
1 + x2

(1 + 2x2)2 ≥ 1 + x2 ⇒ 1 + 4x4 + 4x2 ≥ 1 + x2
4 2
⇒ 4x + 3x ≥ 0 ⇒
x2 (4x2 + 3) ≥ 0
It is true for any real value of x.
Hence, f(x) is increasing in R.
7. Find the points on the curve y = x3 at which the slope of the tangent is equal to the y-coordinate
of the point. [CBSE Delhi 2010; (F) 2011]
Sol. Let P(x1, y1) be the required point on the curve
y = x3 …(i)
dy dy

= 3x 2 ⇒ ; E = 3x12
dx dx ^ x1, y1h

Application of Derivatives 235


@Cbsebookshub - Join Us on Telegram

Slope of tangent at (x1, y1) = 3 x12
According to the question,
3 x12 = y1 …(ii)
Also (x1, y1) lies on (i)

y1 = x13 …(iii)
From (ii) and (iii), we get
3
3 x12 = x1

x13 – 3 x12 = 0 ⇒ x12 (x1 – 3) = 0
⇒ x1 = 0 or x1 = 3

⇒ y1 = 0 or y1 = 27

Hence, required points are (0, 0) and (3, 27).
4
Find the intervals in which the function f(x) = –3 log (1 + x) + 4 log(2 + x) –
8. is strictly
2+x
increasing or strictly decreasing. [CBSE Sample Paper 2018]
Sol. Given f (x) = –3 log (1 + x ) + 4 log (2 + x) – 4
2+x
– 3 ^ 2 + x h + 4 ^1 + x h^ 2 + x h + 4 ^1 + x h
2
–3 4 4

f l^ x h = + + =
1 + x 2 + x ^2 + xh2 ^1 + x h^ 2 + x h
2

– 3 ^ 4 + 4x + x 2 h + 4 ^ 2 + x + 2x + x 2 h + 4 + 4x
=
^1 + x h^ 2 + x h
2

–12 – 12x – 3x 2 + 8 + 12x + 4x 2 + 4 + 4x


=
^1 + x h^ 2 + x h
2

x^x + 4h
f l^ x h =
^1 + x h^ 2 + x h
2

Now, f '(x) = 0
x^x + 4h

= 0 ⇒ x (x + 4 ) = 0
^1 + x h^ 2 + x h
2

⇒ x = 0 [  x ≠ –4 as f (x) is defined on (–1, ∞) ]


Hence, required intervals are (–1, 0) and (0, ∞).
For (–1, 0)
(–ve) # (+ ve)
f l (x) = = –ve ⇒ f (x) is strictly decreasing in (–1, 0)
(+ ve) # (+ ve)
For (0, ∞)
(+ ve) # (+ ve)
f l (x) = = + ve ⇒ f (x) is strictly increasing in (0, ∞)
(+ ve) # (+ ve)
i.e., f (x) is strictly decreasing on (–1, 0) and strictly increasing on (0, ∞).

9. Find the condition that curves 2x = y2 and 2xy = k intersect orthogonally. [NCERT Exemplar]
Sol. Given, equation of curves are 2 x = y2 ... (i)

and 2xy = k ... (ii)
k

y=
2x
2
k

From equation (i) 2x = d n
2x

236 Xam idea Mathematics–XII

@Cbsebookshub - Join Us on Telegram



8x 3 = k 2
1 2 1 2/3

x3 = k ⇒ x= k
8 2
k 1

y= = k ⇒ y = k(1/3)
2x 1
2. k 2/3
2
Thus, we get point of intersection of curves which is from equations (i) and (ii). dc m k(2/3), k(1/3) n
1

2
dy dy
2 = 2y and 2 <x. + y.1F = 0
dx dx
dy 1 dy –2y y
⇒ = and d n= =–
dx y dx 2x x
dy 1 2/3 1/3
d nd k , k n = 1/3
1

[say m1]
dx 2 k
dy 1 2/3 1/3 –k1/3

d
nd k , k n = = –2k –1/3 [say m2]
dx 2 1 2 /3
k
2
Since, the curves intersect orthogonally.
i.e., m1 .m2 = –1
1

. (–2k –1/3) = – 1 ⇒ –2k –2/3 = – 1
k1/3
2

= 1 ⇒ k 2/3 = 2
k 2/3

k 2 = 8
which is the required condition.

Long Answer Questions [5 marks]


1. Find the minimum value of (ax + by), where xy = c2. [CBSE Delhi 2015, 2020 (65/5/1)]
Sol. Let z = ax + by ...(i)
c2
Given xy = c2  ⇒  y = x
c2
Putting y = x in equation (i), we have
bc 2
z = ax + x
For z to be maximum or, minimum
dz bc 2 bc 2
= a – 2 = 0 ⇒ a=
dx x x2
bc 2 b

x 2 = a ⇒ x =! c a
d 2 z 2bc 2
Now, = 3
dx 2 x
b d2 z 2bc 2

at x = c a , dx 2
= 3
>0
bc l
b
a

Application of Derivatives 237


@Cbsebookshub - Join Us on Telegram
b

z will be minimum at x = c a
c2 c2 a

y= x = =c
b b
a c

Minimum value of z = ax + by
b a
= a#c +
a b#c b
= c ab + c ab
= 2c ab
2. Of all the closed right circular cylindrical cans of volume 128p cm3, find the dimensions of the
can which has minimum surface area. [CBSE Delhi 2014]
Sol. Let r, h be radius and height of closed right circular cylinder having volume 128p cm3.
If S be the surface area then
S = 2prh + 2pr2 ⇒ RS S = 2p(rh + r2)
V
SSa V = rr 2 hWWW
128
S = 2r d r. 2 + r 2 n SS& 128 r = rr 2 hWW
S W

r S
SS 128 WW
S ` h = 2 WW
128 dS 128
S = 2r c + r2 m = 2r d – 2 + 2r n
SS r WW

r
& dr T X
r
For extreme value of S
dS 128

dr
=0 & 2rd – 2 + 2r n = 0
r
128
&
– + 2r = 0
r2
128 128
&
2r = 2
& r3 =
2

r h
3
⇒ r = 64 ⇒ r=4
2
128 × 2 d2 S
G
d S
Again 2
= 2r d + 2n & =+ ve
dr r3 dr 2 r=4
r

Hence, for r = 4 cm, S (surface area) is minimum.


Therefore, dimensions for minimum surface area of cylindrical can are
128 128
radius r = 4 cm and h = 2 = = 8 cm .
r 16
3. Prove that the surface area of a solid cuboid, of square base and given volume, is minimum
when it is a cube. [CBSE (AI) 2017; (F) 2009; CBSE 2005]
Sol. Let x be the side of square base of cuboid and other side be y.
Then volume of cuboid with square base, V = x . x . y = x2y
As volume of cuboid is given so volume is taken constant throughout the question, therefore,
V
y = 2 ...(i)
x
In order to show that surface area is minimum when the given cuboid is cube, we have to show
S″ > 0 and x = y.
Let S be the surface area of cuboid, then
S = x2 + xy + xy + xy + xy + x2 = 2x2 + 4xy ...(ii)

238 Xam idea Mathematics–XII

@Cbsebookshub - Join Us on Telegram


V 4V
= 2x 2 + 4x. 2
& S = 2x 2 +
x
...(iii)
x
dS 4V
&

dx
= 4x – 2 ...(iv)
x
dS
For maximum/minimum value of S, we have dx = 0
4V
⇒ 4x – 2 = 0 & 4V = 4x 3 ⇒ V = x3 ...(v)
x
Putting V = x3 in (i), we have
x3
=x y=
x2
Here, y = x ⇒ cuboid is a cube.
Differentiating (iv) w.r.t x, we get
d2 S 8V
n>02
= d4 +
dx x3
Hence, surface area is minimum when given cuboid is a cube.
4. Show that the height of the cylinder of maximum volume that can be inscribed in a sphere of
2R
radius R is . Also find the maximum volume. [CBSE 2019 (65/2/1)]
3
Sol. Let x be radius and (y + R) be the height of cylinder given radius of sphere be R.
In ∆OAB, we have,
OB2 = OA2 + AB2

R2 = y2 + x2 ⇒ x2 + y2 = R2 ⇒ x2 = R2 – y2 ...(i)
Now, volume of cylinder = r x 2 × 2y
C

V = r (R 2 – y 2) × 2y y
For volume to be maximum or minimum O
y R
2r #_ R – y i × 1 + y × (–2y) -
dV 2 2
= 0 ⇒
dy A
B
2 2 2 2 2
⇒ R – y – 2y = 0 ⇒ R – 3y = 0 ⇒ R = 3y 2
2
x

R2 R

y2 = ⇒ y=
3 3
d2 V
= 2r ^ –6y h = –12ry =
–12Rr

<0
dy 2 e at y =
R
o
3
3

R
∴ Volume will be maximum when y =
3
2R

Height of cylinder = 2y =
3
and maximum volume = r _ R 2 – y 2 i × 2y
R2 2R 2R 2 2R 4rR 3
= r e R 2 – o× =r× × =
3 3 3 3 3 3
5. A tank with rectangular base and rectangular sides, open at the top is to be constructed so that
its depth is 2 m and volume is 8 m3. If building of tank costs ™70 per square metre for the base
and ™45 per square metre for the sides, what is the cost of least expensive tank?
[CBSE 2019 (65/1/1)]

Application of Derivatives 239


@Cbsebookshub - Join Us on Telegram
Sol. Let l, b and h metre be the length, breadth and height of the tank respectively.
Given h = 2m
and volume of tank = l × b × h
4

8 = l × b × 2 ⇒ lb = 4 ⇒ b =
l hm
Now, area of the base, lb = 4 m 2 bm
and, area of four walls, A = 2 (l + b) × h
4 4 lm
= 2 dl + n × 2 ⇒ A = 4 dl + n
l l
For minimum cost
dA 4
= 0 ⇒ 4 e1 – o=0
dl l2
4

= 1 ⇒ l2 = 4 ⇒ l = 2m
l2
4 4
∴ l = 2m ( length cannot be negative) and b = = = 2m
l 2
d2 A 32 32
Now, = 2 = = 8>0
dl 2 l 4
∴ Area will be minimum when l = 2m, b = 2m, h = 2m
∴ Cost of building of tank = 70 × (l × b) = 70 × 2 ×2 = ™ 280
and cost of building the walls = 45 × 2h (l + b) = 90 × 2 (2 + 2) = ™ 720
∴ Total cost for building the tank = 280 + 720 = ™ 1,000
6. If the sum of hypotenuse and a side of a right angled triangle is given, show that the area of
r
the triangle is maximum when the angle between them is .
3
[CBSE Delhi 2017; (AI) 2009, 2014; (Central) 2016]
Sol. Let h and x be the length of hypotenuse and one side of a right triangle and y is length of the third side.
If A be the area of triangle, then RSalso given V
1 1 SS WWW
A = xy = x h 2 – x 2 SSh + x = k (constant)WW
2 2 SS WW
1 1 S` h = k – x W
A = x (k – x) 2 – x 2 = x k 2 – 2kx + x 2 – x 2 T X
2 2
2
x 1
⇒ A 2 = (k 2 – 2kx) & A 2 = (k 2 x 2 – 2kx 3)
4 4
Differentiating with respect to x we get
d^ A2h 1
= ^ 2k 2 x – 6kx 2 h …(i)
dx 4
For maxima or minima of A2 RS VW
SSa V = lbh
d^ A2h
WW
1
=0 & ^ 2k 2 x – 6kx 2 h = 0 SS 8 = lb2 WW
dx 4 SS W
SS 8 4 WWW
⇒ 2k2x – 6kx2 = 0 ⇒ 2kx (k – 3x) = 0 SS` b = 2l = l WW

k – 3x = 0 ; 2kx ≠ 0 T X
k
⇒ x= 3
Differentiating (i) again with respect to x, we get
d 2 ( A 2) 1
= (2k 2 – 12kx)
dx 2 4

240 Xam idea Mathematics–XII

@Cbsebookshub - Join Us on Telegram


d 2 ( A 2) k2
H e 2k 2 – 12k. o = –
1 k
= <0
dx 2 4 3 2
x = k/3

2 k k 2k
Hence, A is maximum when x = and h = k – = .
3 3 3
k 2k
i.e., A is maximum when x = , h =

3 3
x k 3 1 1 r

cos i = = #
h 3 2k 2
= ⇒ cos i =
2
& i=
3
7. Show that the semi-vertical angle of the cone of the maximum volume and of given slant
1
height is cos -1 . [CBSE Delhi 2014; (North) 2016]
3
Sol. Let ABC be cone having slant height l and semi-vertical angle q. A

If V be the volume of cone then



1 r
V = .r # DC 2 # AD = # l 2 sin 2 i # l cos i
3 3 l
rl 3
&
V=
3
sin 2 i cos i

dV rl 3
&

di
=
3
[– sin 3 i + 2 sin i. cos 2 i] B
D
C

For maximum value of V.


dV = rl 3 6 @ =

di
0 & 3
+ 2
3 – sin i 2 sin i. cos i . 0

– sin3 q + 2sin q.cos2 q = 0 ⇒ – sin q (sin2 q – 2cos2 q) = 0
⇒ sin q = 0
or 1 – cos2 q – 2cos2 q = 0

q = 0 or 1 – 3cos2 q = 0
1

q = 0 or cos i =
3
d2 V rl 3
Now = {–3 sin 2 i. cos i – 4 sin 2 i. cos i + 2 cos 3 i}
di 2 3
d2 V rl 3 d2 V

= {–7 sin 2 i cos i + 2 cos 3 i} ⇒ G =+ ve
di 2 3 di 2 i=0

d2 V 1 2
G >Putting cos i = H
1 2
and 2
= –ve and sin i = 1–d n =
di cos i =
1 3 3 3
3
1 1
Hence, for cos i = or i = cos –1 d n, V is maximum.
3 3
8. Show that the height of a closed right circular cylinder of given surface and maximum volume,
is equal to the diameter of its base. [CBSE Delhi 2012]
Sol. Let r and h be radius and height of given cylinder of surface area S.
If V be the volume of cylinder then
V = pr2h
r r 2 . ( S – 2 r r 2) S – 2rr 2
V=
2rr
[a S = 2rr 2 + 2rrh & 2rr
= h]

Sr – 2rr 3 dV 1
V=
2
& dr
=
2
(S – 6rr 2)

Application of Derivatives 241


@Cbsebookshub - Join Us on Telegram
For maximum or minimum value of V
dV 1

dr
= 0    ⇒
2
(S – 6rr 2) = 0 & S – 6rr 2 = 0
S S

r2 =
6r
& r=
6r
d2 V d2 V d2 V
= G
1
Now =–
2
× 12rr    ⇒ = – 6rr & <0
dr 2 dr 2 dr 2 r=
S
6r
S
Hence, for r = 6r , volume V is maximum.
S
S – 2r. 3S – S 6r

h= 6 r & h = 3×2r ×
S S
2r
6r h
2S 6r S

h= . =2
6r S 6 r
:a r =
6r D
S r

h = 2r (diameter)

Therefore, for maximum volume, height of cylinder is equal to diameter of its base.
9. An open box with a square base is to be made out of a given quantity of cardboard of area c2

c3
square units. Show that the maximum volume of the box is cubic units.
6 3
[NCERT Exemplar; CBSE (AI) 2012]
Sol. Let the length, breadth and height of open box with square be x, x and h unit respectively.
If V be the volume of box then V = x.x. h    ⇒   V = x2h ....(i)
2 2
c –x
Also c 2 = x 2 + 4xh & h=
4x
Putting it in (i), we get
x 2 ^c 2 – x 2h c2 x x3 x h
V=
4x
& V=
4

4 x
Differentiating with respect to x, we get
dV c 2 3x 2
= –
dx 4 4
dV
Now for maxima or minima dx = 0

c 2 3x 2 3x 2 c 2
&

4

4
=0 & 4
=
4
c2 c
&
x2 =
3
& x=
3
d2 V d2 V
= G
6x 3x 3c
Now, =– =– ⇒ =– = –ve
dx 2 4 2dx x = c 2
3 2 3
c
Hence, for x = volume of box is maximum.
3
c2
c –x 2 2 2 c2 –

h= = 3 = 2c ×
3
=
c
4x c 3 4c 2 3
4
3

242 Xam idea Mathematics–XII

@Cbsebookshub - Join Us on Telegram


c2 c c3
Therefore maximum volume = x2 . h = . = cubic units
3 2 3 6 3
10. Find the shortest distance of the point (0, c) from the parabola y = x2, where 1 ≤ c ≤ 5. [CBSE 2013]
Sol. Let P (a, b) be required point on parabola y = x2 such that the distance of P to given point Q(0, c)
is shortest.
Let PQ = D y-axis
D = ^a – 0 h + ^b – c h D 2 = a 2 + ^b – c h
2 2 2
∴ & y = x2
(0, c) Q
2 2 2 2 2 2

D = a + (a – c) [(a, b) lie on y = x ⇒ b = a ] ...(i) P( , )

d ^D 2 h
= 2a + 2 ^a 2 – c h .2a = 2a ^1 + 2a 2 – 2c h = 2a + 4a 3 – 4ac
x-axis
Now, O
da
For extremum value of D or D2
d (D 2)

da
=0 & 2a (1 + 2a 2 – 2c) = 0
2c – 1
⇒ a = 0, or 1 + 2a 2 – 2c = 0 & a = 0 or a = !
2
d 2 ^D 2 h d 2 ^D 2 h
Again = 2 + 12a 2 – 4c ⇒ G = 2 – 4c = –ve [ 1 ≤ c ≤ 5]
da 2 da 2 a=0
d 2 ^D 2 h
= G
2c – 1
= 2 + 12 c m – 4c = 2 + 12 c – 6 – 4c = 8c – 4 > 0 [ 1 ≤ c ≤ 5]
da 2
a= ± 2c – 1 2
2

2c – 1
i.e., for a = ±
2
2 D i.e., D is minimum (shortest)
Now, the shortest distance D is
D = a 2 + (a 2 – c) 2 = a 4 + a 2 + c 2 – 2a 2 c [From (i)]
2c – 1 2 2c – 1 2c – 1 2c – 1
= d n + + c 2 –2c d n <a a 2 = F
2 2 2 2

]2c – 1g2 + 4c 2 + 4c – 2 – 4c (2c – 1)


=
4
1 1
4c 2 + 1 – 4c + 4c 2 + 4c – 2 – 8c 2 + 4c =
= 4c – 1
2 2
1
Hence, required shortest distance is 4c – 1 .
2
11. Show that the volume of the greatest cylinder that can be inscribed in a cone of height ‘h’ and
4
semi-vertical angle ‘a’ is r h 3 tan 2 a . [CBSE (AI) 2010, (East) 2016]
27
Sol. Let a cylinder of base radius r and height h1 is included in a cone of height h and semi-vertical
angle a.
Then AB = r, OA = (h – h1). In right angle triangle OAB,
AB r
= tan a ⇒ = tan a ⇒ r = (h – h1) tan a
OA h – h1

V = p [(h – h1) tan a]2 . h1 ( Volume of cylinder = pr2h)
= p tan2 a . h1 (h – h1)2 ...(i)
Differentiating with respect to h1, we get
= r tan 2 a 6h1 .2 ^ h – h1 h^ – 1 h + ^ h – h1 h ×1B
dV 2

dh1

Application of Derivatives 243


@Cbsebookshub - Join Us on Telegram
= p tan2 a (h – h1) [–2h1 + h – h1] O
2
= p tan a (h – h1) (h – 3h1)
dV =
For maximum volume V,
dh1 0
A r B
1 h

h – h1 = 0 or h – 3h1 = 0 & h = h1 or h1 = h
3 h1
1
⇒ h1 = h ( h = h1 is not possible)
3 r
Again differentiating with respect to h1, we get
d2 V
= r tan 2 a 6(h – h1 h^ –3 h + ^ h – 3h1 h^ –1 h]
dh12
1 d2 V 1
At h1 =
h, = r tan 2 a ;c h – h m^ –3 h + 0E = –2rh tan 2 a < 0
3 dh12 3
1
∴ Volume is maximum for h1 = h
3
1 1 2
Vmax = r tan 2 a. c h mc h – h m [Using (i)]
3 3
4
= rh 3 tan 2 a
27

12. The sum of the perimeter of a circle and a square is k, where k is some constant. Prove that the
sum of their areas is least when the side of the square is double the radius of the circle.
[CBSE (F) 2010, 2014]
Sol. Let side of square be a units and radius of circle be r units.
It is given that 4a + 2pr = k, where k is a constant
k – 4a
⇒ r=
2r
Sum of areas, A = a2 + pr2
k – 4a 2 1
= a2 + r; E = a2 + ^ k – 4a h
2

2r 4r
Differentiating with respect to a, we get
dA 1 2 ^ k – 4a h
= 2a + .2 ^ k – 4a h . ^ –4 h = 2a – …(i)
da 4r r
dA
For minimum area, =0
da
2 ^ k – 4a h 2 ^ k – 4a h

2a – = 0 & 2a =
r r
2 ^2rrh

2a = r [As k = 4a + 2pr given]

a = 2r
Now, again differentiating equation (i) with respect to a, we get
d2 A 2 8 d2 A 8
2
=2– (– 4) = 2 + at a = 2r, 2
= 2+ >0
da r r da r
∴ For ax = 2r, sum of areas is least.

Hence, sum of areas is least when side of the square is double the radius of the circle.

244 Xam idea Mathematics–XII

@Cbsebookshub - Join Us on Telegram


x2 y2
= 1 .
13. Find the area of the greatest rectangle that can be inscribed in an ellipse +
a2 b2
[CBSE (AI) 2013]
Sol. Let ABCD be rectangle having area A inscribed in an ellipse
x2 y2
2
+
= 1 ...(i)
a b2
Let the coordinate of A be (a, b).

Coordinate of B ≡ (a, – b), C ≡ (– a, – b), D ≡ (– a, b)
Now A = Length × Breadth = 2a × 2b = 4ab
RSa (a, b) lies on ellipse (i) VW
2 SS WW
b2 e1 – o
a SS 2 2 2 W

= 4a. SS` a + b = 1 i.e., b = b 2 e 1 – a oWWW
a2 SS a 2 b 2 a 2 WW
T X
a2 16b 2

A 2 = 16a 2 ) b 2 e 1 – o3 & A2 = ^ a2 a2 – a 4h Y
a2 a2
d ^ A 2 h 16b 2

= 2 ^ 2a 2 a – 4a 3 h (–, ) D A (, )
da a
For maximum or minimum value
d^ A2h
=0 O X
da
⇒ 2a2a – 4a3 = 0
⇒ 2a(a2 – 2a2) = 0
(–, –) C B (, –)
a

a = 0, a =
2
d2 ^ A2h 16b 2 d2 ^ A2h 16b 2 a2
Again = ^ 2a 2 – 12a 2 h ⇒ G = d 2a 2 – 12× n<0
da 2 a2 da 2 a=
a a2 2
2
a
⇒ For a =
, A 2 i.e., A is maximum.
2
a b
i.e., for greatest area A, a = and b = (Using (i))
2 2
a
∴ Greatest area = 4a . b = 4 × b = 2ab
2 2
14. Tangent to the circle x2 + y2 = 4 at any point on it in the first quadrant makes intercepts OA and
OB on x and y axes respectively, O being the centre of the circle. Find the minimum value of
(OA + OB). [CBSE Ajmer 2015]
Sol. Let AB be the tangent in the first quadrant to the circle Y
x2 + y2 = 4 which make intercepts OA and OB on x and
B (0, 2 cosec )
y axis respectively. Let S = OA + OB.
S = OA + OB … (i)
Let q be the angle made by OP with positive direction P (2 cos, 2 sin)

of x-axis. 2

∴ Coordinates of P = (2 cos q, 2 sin q) 


X
Coordinates of A = (2 sec q, 0) O A (2 sec, 0)
Coordinates of B = (0, 2 cosec q)
(i) ⇒ S = 2 sec q + 2 cosec q

2 "sec i tan i – cosec i cot i ,


dS =


di

Application of Derivatives 245


@Cbsebookshub - Join Us on Telegram
For extremum value of V

2 " sec i tan i – cosec i cot i , = 0


dS =


di
0 &
⇒ sec q tan q – cosec q cot q = 0

1 sin i 1 cos i sin i cos i

= & =
cos i cos i sin i sin i cos i sin 2 i
2

⇒ sin3 q = cos3 q   ⇒ sin q = cos q


i = 4 8a i lies in first quadrant & 0 # i # 4 B
r r

d2 S
Now, = 2 {^sec 3 i + tan 2 i sec i h + ^cosec 3 i + cosec i cot 2 i h}
di 2
d2 S

G =+ ve ⇒
r
S is minimum when i = 4
di 2 i=
r
4
r r
∴ Minimum value of S = OA + OB is 2 sec 4 + 2 cosec 4 = 2 2 + 2 2 = 4 2 units.

15. Find the absolute maximum and absolute minimum values of the function f given by
f(x) = sin 2 x – cos x, x ∈ [0, p]. [CBSE Panchkula 2015]
Sol. Here, f(x) = sin2 x – cos x
f '(x) = 2sin x.cos x + sin x ⇒ f '(x) = sin x(2cos x + 1)
For critical point: f '(x) = 0

1
&
sin x (2 cos x + 1) = 0 & sin x = 0 or cos x = –
2
2r 2r
&
x = 0 or cos x = cos
3
& x = 0 or x = 2nr !
3
, where n = 0, ±1, ±2 ...

⇒ 2r
x = 0 or x = 2 other values does not belong to [0, p].


For absolute maximum or minimum values:
f(0) = sin2 0 – cos 0 = 0 – 1 = – 1
2
=d n – c– m = + =
2r 2r 2r 3 1 3 1 5
fc m = sin 2 – cos
3 3 3 2 2 4 2 4
f(p) = sin2 p – cos p = 0 – (–1) = 1
5
Hence, absolute maximum value = 4 and absolute minimum value = –1 .

16. If the function f(x) = 2x3 – 9mx2 + 12m2x + 1, where m > 0 attains its maximum and minimum at
p and q respectively such that p2 = q, then find the value of m. [CBSE Patna 2015]
Sol. Given, f(x) = 2x3 – 9mx2 + 12m2x + 1
⇒ f '(x) = 6x2 – 18mx + 12m2
For extremum value of f(x), f '(x) = 0
⇒ 6x2 – 18mx + 12m2 = 0 ⇒ x2 – 3mx + 2m2 = 0
⇒ x2 – 2mx – mx + 2m2 = 0 ⇒ x(x – 2m) – m(x – 2m) = 0
⇒ (x – m)(x – 2m) = 0 ⇒ x = m or x = 2m
Now, f"(x) = 12x – 18m
⇒ f"(x) at [x = m] = f"(m) = 12m – 18m = – 6m < 0
And, f"(x) at [x = 2m] = f"(2m) = 24m – 18m = 6m > 0
Hence, f(x) attains maximum and minimum value at m and 2m respectively.

246 Xam idea Mathematics–XII

@Cbsebookshub - Join Us on Telegram


⇒ m = p and 2m = q
But, p2 = q [Given]
∴ m2 = 2m ⇒ m2 – 2m = 0
⇒ m(m – 2) ⇒ m = 0 or m = 2
⇒ m = 2 as m > 0
x
17. The sum of the surface areas of a cuboid with sides x, 2x and and a sphere is given to be
3
constant. Prove that the sum of their volumes is minimum, if x is equal to three times the
radius of sphere. Also find the minimum value of the sum of their volumes. [CBSE (F) 2016]
Sol. Let r be the radius of sphere and S, V be the sum of surface area and volume of cuboid and sphere.

Now V = d x . 2x . n + rr 3
x 4
3 3
RS
V=
2 3 4 3
x + rr ⇒ V=
2 3
(x + 2rr3 ) SSa S = 2 ; x.2x + x. x + x .2xE + 4rr 2
⇒ 3 3 3 SS 3 3
SS
SS& S = 18x + 4rr 2 = 6x 2 + 4rr 2
2
3

V = *e o + 2 rr 3 4
2 S – 4 rr 2 2 SS 3

SS 2 3/2
SS& x = – 4rr & x3 = d S – 4rr n
S
3 6 2
2
S 6 6
1
T
= * e o . . (– 8rr) + 6rr 2 4
dV 2 3 S – 4rr 2 2 1


dr 3 2 6 6
For maximum or minimum value
dV =
dr 0
Z] 1 _b 1
2 ]] S – 4rr 2 2 bb S – 4rr 2 2 6rr 2
& 3 ][] – 2rrf 6 p + 6rr 2 b`b = 0 & f
6
p =
2rr
] b
\ 1
a 1
S – 4rr 2 2 1 S – 4rr 2 2
&
f p = 3r & r = .f p
6 3 6
d2 V
Obviously, G 1
=+ ve
dr 2 1 S – 4rr 2 2
r= d n
3 6
1
1 S – 4r r 2 2
∴ V is minimum when r = d
n
3 6
1
S – 4rr 2 2 S – 4r r 2

3r = d n & 9r 2 = d n & 54r 2 = S – 4rr 2
6 6
⇒ 54r2 = 6x2 + 4pr2 – 4pr2
[ S = 6x2 + 4pr2]

x2 = 9r2 ⇒ x = 3r
i.e., x is equal to three times the radius of sphere.

' x + 2r a k 1
2 3 x 3
Now, minimum value of V (sum of volume) =
3 3
2 2r 3 2 3
= ' x3 + x 1= x ^ 27 + 2r h cubic unit.
3 27 81
18. Find the maximum and minimum values of f(x) = sec x + log cos2 x, 0 < x < 2p. [CBSE (South) 2016]
Sol. We have f(x) = sec x + log cos2 x
f ' ^xh = sec x. tan x + .2 cos x ^ – sin xh = sec x . tan x – 2 tan x = tan x (sec x – 2)
1

cos2 x

Application of Derivatives 247


@Cbsebookshub - Join Us on Telegram
For critical point
f'(x) = 0
⇒ tan x (sec x – 2) = 0
⇒ tan x = 0 or sec x – 2 = 0
1
& x = nr or sec x = 2 & x = nr or cos x =
2
r r
& x = nr or cos x = cos
3
& x = nr or x = 2nr ! , n = 0, ! 1, ! 2.......
3
r 5r
Thus possible value of x in interval 0 < x < 2p are x = , r,
3 3
2
f a k = sec + log cos 2 = 2 + log c m
r r r 1
Now,
3 3 3 2
= 2 + 2(log 1 – log 2) = 2 – 2 log 2 = 2(1 – log 2) [ log 1 = 0]
f(p) = sec p + log cos2 p = – 1 + log (–1)2 = – 1

= sec a 2r – k + 2 log cos a 2r – k


5r 5r 5r
fc m = sec
r r
+ 2 log cos
3 3 3 3 3
r r
= sec + 2 log cos = 2 + 2 log 1
3 3 2
= 2 + 2(log 1 – log 2) = 2 – 2 log 2 = 2(1 – log 2)
Hence, maximum value of f (x) = 2(1 – log 2)
minimum value of f (x) = – 1
19. Prove that the least perimeter of an isosceles triangle in which a circle of radius r can be
inscribed is 6 3r . [CBSE (Central) 2016]
Sol. Let ∆ABC be isosceles triangle having AB = AC in which a circle with centre O and radius r is
inscribed touching sides AB, BC and AC at E, D and F respectively.
Let AE = AF = x, BE = BD = y ar (DABC) = ar (DBOC) + ar (DAOC) + ar (DAOB)
Obviously, CF = CD = y 1 1 1 1
& 2 AD.BC = 2 .BC.OD + 2 . AC.OF + 2 .AB.OE
Let P be the perimeter of DABC.
& 2y. (r + r2 + x2 ) = 2y.r + (x + y) . r + (x + y) .r
∴ P = 2x + 4y & 2y. (r + r2 + x2 ) = 2yr + 2 (x + y) .r
4yr 2 & yr + y r2 + x2 = yr + xr + yr
⇒ P= + 4y (From (i))
y2 – r2 & y r2 + x2 = xr + yr
Differentiating w.r.t. y, we get & y2 (r2 + x2) = x2 r2 + y2 r2 + 2xyr2
&

dP (y – r ) .4r – 4yr (2y – 0)
=
2 2 2 2
+4
& y2 r2 + x2 y2 = x2 r2 + y2 r2 + 2xyr2
dy ( y 2 – r 2) 2 & x2 y2 = x2 r2 + 2xyr2
dP 4y r – 4r – 8y r
2 2 4 2 2 & xy2 = xr2 + 2yr2
&

dy
= +4
2yr 2
( y 2 – r 2) 2 &x= ... (i)
( y 2 – r 2)
dP – 4r (r + y )
2 2 2
&

dy
= +4
( y 2 – r 2) 2 A

For critical point dP = 0 x x


dy
– 4r 2 _ r 2 + y 2 i E F

+4 = 0 r r
_ y2 – r 2 i2
y
O
y
r

– 4r2(r2 + y2) + 4(y2 – r2)2 = 0
B y D y C

– r4 – r2y2 + y4 + r4 – 2y2r2 = 0

248 Xam idea Mathematics–XII

@Cbsebookshub - Join Us on Telegram



y4 – 3r2y2 = 0

y2[y2 – 3r2] = 0

y = 3 r 6a y ! 0@
d2 P
Also G =+ ve
dr 2 3r

⇒ when y = 3 r , the value of P is minimum.



4r 2 y 4r 2 . 3 r 4 3 r3
∴ Least perimeter = 4y +
= 4 3r + = 4 3r + = 6 3 r units
y2 – r2 3r 2 – r 2 2r 2
20. A window has the shape of a rectangle surmounted by an equilateral triangle. If the perimeter
of the window is 12 m, find the dimensions of the rectangle that will produce the largest area
of the window. [CBSE(AI) 2011] [HOTS]
Sol. Let x and y be the dimensions of rectangular part of window and x be side of equilateral part.
3 2
If A be the total area of window, then A = x.y + x ...(i)
4
Also, x + 2y + 2x = 12 ⇒ 3x + 2y = 12
12 – 3x x x
⇒ y= 2
^12 – 3x h
3 2

A = x. + x [From (i)]
2 4
y
3x 2 3 2 y

A = 6x – + x
2 4
3 x

A' = 6 – 3x + 2 x [Differentiating with respect to x]
Now, for maxima or minima
3 12
A' = 0 ⇒ 6 – 3x +
2
x=0 & x=
6– 3
3
Again A" = – 3 + 2 < 0 (for any value of x) ⇒ A"] 12 < 0 i.e., is maximum if
x=
6– 3

12 – 3 c m
12
12 6– 3
x = and y = 2 .
6– 3
i.e., for largest area of window, dimensions of rectangle are

12 18 – 6 3
x= and y =
6– 3 6– 3 .

PROFICIENCY EXERCISE
QQ Objective Type Questions: [1 mark each]
1.
Choose and write the correct option in each of the following questions.
(i) The points at which the tangents to the curve y = x3 – 12x + 18 are parallel to x-axis are
(a) (2, –2), (–2, –34) (b) (2, 34), (–2, 0) (c) (0, 34), (–2, 0) (d) (2, 2), (–2, 34)
(ii) The sides of an equilateral triangle are increasing at the rate of 2 cm/sec. The rate at which
the area increases, when side is 10 cm is
10
(a) 10 cm2/s (b) 3 cm2/s (c) 10 3 cm2/s (d) 3 cm2/s

Application of Derivatives 249


@Cbsebookshub - Join Us on Telegram
(iii) The maximum value of slope of the curve y = – x3 + 3x2 + 12x – 5 is [CBSE 2020 (65/3/1)]
(a) 15 (b) 12 (c) 9 (d) 0
2
(iv) If the function f(x) = 2x – kx + 5 is increasing on [1, 2], then k lies in the interval
(a) (– ∞, 4) (b) (4, ∞) (c) (– ∞, 8) (d) (8, ∞)
2 3
(v) If the curve ay + x = 7 and x = y, cut orthogonally at (1, 1), then the value of a is
(a) 1 (b) 0 (c) – 6 (d) 6
1
(vi) The approximate value of (33) 5 is
(a) 2.0125 (b) 2.1 (c) 2.01 (d) none of these
(vii) The equation of the normal to the curve y = x (2 – x) at the point (2, 0) is
(a) x – 2y = 2 (b) x – 2y + 2 = 0 (c) 2x + y = 4 (d) 2x + y – 4 = 0
2 2
(viii) The angle of intersection of the parabolas y = 4ax and x = 4ay at the origin, is
r r r r
(a) (b) (c) (d)
6 3 2 4
2.
Fill in the blanks.
(i) The slope of the tangent to the curve y = x3 – x at the point (2, 6) is _____________ .
 [CBSE 2020 (65/4/1)]
1
(ii) The maximum value of f (x) = x + x , x < 0 is _____________ .
(iii) The rate of change of the area of a circle with respect to its radius r, when r = 3 cm, is
_____________ .  [CBSE 2020 (65/4/1)]
1
(iv) If f (x) = 2
, then its maximum value is _____________ .
4x 2x + 1
+
QQ Very Short Answer Questions: [1 mark each]
3. If the rate of change of volume of a sphere is equal to the rate of change of its radius, find the
radius of the sphere.
4. Find the interval in which the function f given by f(x) = 7 – 4x – x2 is strictly increasing.
 [CBSE 2020 (65/3/1)]
5. At what points on the curve x2 + y2 – 2x – 4y + 1 = 0, the tangents are parallel to y-axis?
6. It is given that at x = 1 the function x4 – 62x2 + ax + 9 attains the maximum value on the interval
[0, 2]. Find the value of a.
7. Find the least value of l such that the function (x2 + lx + 1) is increasing on [1, 2].
QQ Short Answer Questions–I: [2 marks each]
8. The contentment obtained after eating x-units of a new dish at a trial function is given by the
function C(x) = x3 + 6x2 + 5x + 3. If the marginal contentment is defined as rate of change of C(x)
with respect to the number of units consumed at an instant, then find the marginal contentment
when three units of dish are consumed. [CBSE (F) 2013]

9. Prove that the function f(x) = tan x – 4x is strictly decreasing on c – , m .


r r

3 3
10. Find the value of a for which the function f(x) = sin x – ax + b increasing on R.
11. Show that the function f(x) = 4x3 – 18x2 + 27x –7 is always increasing on R. [CBSE Delhi 2017]
r
12. Prove that f(x) = sin x + 3 cos x has maximum value at x = .
6
x
13. Show that the function f defined by f(x) = (x – 1) e + 1 is an increasing function for all x > 0.
 [CBSE 2020 (65/4/1)]

250 Xam idea Mathematics–XII

@Cbsebookshub - Join Us on Telegram


QQ Short Answer Questions–II: [3 marks each]
x–7
14. Find the equations of the tangent and the normal to the curve y = at the point
(x – 2) (x – 3)
where it cuts the x-axis. [CBSE 2019 (65/3/1)]
15. A ladder 13 m long is leaning against a vertical wall. The bottom of the ladder is dragged
away from the wall along the ground at the rate of 2 cm/sec. How fast is the height on the wall
decreasing when the foot of the ladder is 5 m away from the wall? [CBSE 2019 (65/4/1)]
16. Find the intervals in which the function f(x) = 3x4 – 4x3 – 12x2 + 5 is
(a) strictly increasing (b) strictly decreasing. [CBSE Delhi 2014]
17. Find the point on the curve 9y 2 = x 3, where the normal to the curve makes equal intercepts on the
axes. [CBSE (F) 2015]
18. Find the equations of the normals to the curve y = x3 + 2x + 6 which are parallel to the line
x + 14y + 4 = 0. [CBSE Delhi 2010]
19. Prove that the semi-vertical angle of the right circular cone of given volume and least curved
surface area is cot –1 2 . [CBSE Delhi 2014]
20. Find all the points of local maxima and local minima of the function
3 45 2
f (x) = – x 4 – 8x 3 – x + 105
4 2
21. Using differentials, find the approximate value of 0.082 .

QQ Long Answer Questions: [5 marks each]


22. Find the dimensions of the rectangle of perimeter 36 cm which will sweep out a volume as large
as possible, when revolved about one of its side. Also, find the maximum volume.
 [CBSE 2020 (65/4/1)]
23. Show that a right circular cylinder of the given volume open at the top has minimum total surface
area, provided its height is equal to the radius of the base. [CBSE (F) 2014]
24. Show that the equation of normal at any point t on the curve x = 3 cot t – cos 3 t and y = 3 sin t – sin 3 t
is 4 (y cos 3 t – x sin 3 t) = 3 sin 4t. [CBSE Delhi 2016]
25. Find the angle of intersection of the curve y 2 = 4ax and x 2 = 4by. [CBSE (F) 2016]
26. The volume of a sphere is increasing at the rate of 3 cubic centimeter per second. Find the rate of
increase of its surface area, when the radius is 2 cm. [CBSE Delhi 2017]
27. Find the local maxima and local minima, of the function f(x) = sin x – cos x,0 <x<2π, Also find the
local maximum and local minimum values. [CBSE Delhi 2015]
28. Find the value of p for which the curves x2 = 9p(9–y) and x2 = p(y+1) cut each other at right angles.
[CBSE Allahabad 2015]
x
29. Find the point on the curve y = , where the tangent to the curve has the greatest slope.
 1 + x2 [CBSE Chennai 2015]
30. Find the absolute maximum and absolute minimum values of the function f given by
f(x) = cos2 x + sin x, x ∈ [0, p]. [CBSE Guwahati 2015]
31. Find the equation of tangents to the curve y = cos (x + y), – 2p ≤ x ≤ 2p, that are parallel to the line
x + 2y = 0. [CBSE (F) 2016]
32. Determine the intervals in which the function f(x) = x4 – 8x3 + 22x2 – 24x + 21 is strictly increasing
or strictly decreasing. [CBSE (South) 2016]
2
33. Find the equation of the normal at a point on the curve x = 4y which passes through the point
(1, 2). Also find the equation of the corresponding tangent. [CBSE Delhi 2013]

Application of Derivatives 251


@Cbsebookshub - Join Us on Telegram
34. A manufacturer can sell x items at a price of ™ c 5 – m each. The cost price of x items is
x

100
™ c + 500 m . Find the number of items he should sell to earn maximum profit. [CBSE (AI) 2009]
x
5
35. A wire of length 34 m is to be cut into two pieces. One of the pieces is to be made into a square and
the other into a rectangle whose length is twice its breadth. What should be the lengths of the two
pieces, so that the combined area of the square and the rectangle is minimum? [CBSE (F) 2017]
36. Show that the rectangle of maximum perimeter which can be inscribed in a circle of radius r is the
square of side r 2 . [CBSE Delhi 2011]
37. Show that the rectangle of maximum area that can be inscribed in a circle is a square.
 [CBSE Delhi 2008, 2011]
38. Show that the normal at any point q to the curve x = a cos i + ai sin i, y = a sin i – a i cos i is at a
constant distance from the origin. [CBSE(AI) 2011]

Answers
1. (i) (d) (ii) (c) (iii) (a) (iv) (a) (v) (d) (vi) (a)
(vii) (a) (viii) (c)
4
2. (i) 11 (ii) –2 (iii) 6p cm2/cm (iv)
3
1
3. units 4. (– ∞, –2) 5. (–1, 2) and (3, 2) 6. a = 120 7. l = – 2
2 r
8. 68 units 10. (– ∞, –1) 14. x – 20y – 7 = 0 and 20x + y – 140 = 0 respectively

17. c 4, m and c 4, m
5 8 –8
15. cm/sec 16. (a) (–1, 0) ∪ (2, ∞) (b) (– ∞, –1) ∪ (0, 2)
6 3 3
18. x + 14y – 254 = 0 and x + 14y + 86 = 0 20. Local maxima at 0, –5; and local minima at –3
216
21. 0.2867 22. Length = 12 cm, breadth = 6 cm and maximum volume = r cm3
25. 90° 26. 3 cm2/sec. 27. Local maximum value = 2 , local minimum value = – 2
28. p = 0, 4 29. (0, 0)
5 r 5r r
30. Absolute maximum value = at x = and , absolute minimum value = 1 at x = 0, and r
4 6 6 2
31. x + 2y = 0 32. (1, 2) ∪ (3, ∞); (– ∞, 1) ∪ (2, 3) 33. x + y – 3 = 0; x – y – 1 = 0 34. 240 items
35. 16 m, 18 m

SELF-ASSESSMENT TEST
Time allowed: 1 hour Max. marks: 30
1.
Choose and write the correct option in the following questions. (4 × 1 = 4)
4
(i) If y = x – 10 and x changes from 2 to 1.99, then what is the change in y?
(a) 0.32 (b) 0.032 (c) 5.68 (d) 5.698
3 2
(ii) The maximum slope of curve y = – x + 3x + 9x – 27 is
(a) 0 (b) 12 (c) 16 (d) 32
log x
(iii) The maximum value of x in [2, ∞) is
1
(a) 0 (b) 1 (c) e (d) e

252 Xam idea Mathematics–XII

@Cbsebookshub - Join Us on Telegram


(iv) The function f(x) = x3 – 27x + 5 is monotonically increasing when
(a) x < –3 (b) |x| > 3 (c) x ≤ –3 (d) |x| ≥ 3
2. Fill in the blanks. (2 × 1 = 2)
2
(i) The equation of normal to the curve 2y + x = 3 at point (1, 1) is _____________ .
(ii) The maximum value of sin x . cos x is _____________ .
QQ Solve the following questions. (2 × 1 = 2)
3. The maximum and minimum value of the function f(x) = |x + 2| – 1.
4. Show that f(x) = ex do not have maxima or minima.
QQ Solve the following questions. (4 × 2 = 8)
3
5. Show that the tangent to the curve y = 7x + 11 are parallel at the points x = 2 and x = –2.
6. Find two numbers whose sum is 24 and whose product is as large as possible.
7. Find the least value of l such that the function (x2 + lx + 1) is increasing on [1, 2].
8. Find the value of a for which the function f(x) = sin x – ax + b increasing on R.
QQ Solve the following questions. (3 × 3 = 9)
4 sin x – 2x – x cos x
9. Find the intervals in which the function f given by f (x) = is
2 + cos x
(i) increasing (ii) decreasing.
10. Find the equation of tangent to the curve y =
3x – 2 , which is parallel to the line 4x – 2y + 5 = 0.
11. The fuel cost for running a train is proportional to the square of the speed generated in km per
hour. If the fuel costs ` 48 per hour at speed 16 km per hour and the fixed charges amount to
` 1200 per hour then find the most economical speed of train, when total distance covered by train
is 5 km.
QQ Solve the following question. (1 × 5 = 5)
12. A square piece of tin of side 18 cm is to be made into a box without top by cutting a square from
each corner and folding up the flaps to form a box. Find the maximum volume of the box.

Answers
1. (i) (a) (ii) (b) (iii) (c) (iv) (d)
1
2. (i) x – y = 0 (ii)
2
3. Min value = – 1 & maximum value does not exist
6. Both numbers are same and is 12. 7. l = – 2 8. (– ∞, –1)
3r r 3r
9. (i) f(x) is increasing in the interval d 0, n and d , 2r n (ii) decreasing in the interval d , n
r
2 2 2 2
10. 48x – 24y – 23 = 0 11. v = 50 km/hour 12. 432 cm3
zzz

Application of Derivatives 253


@Cbsebookshub - Join Us on Telegram
8 Integrals

1. Antiderivative (or Primitive): A function f(x) is said to be antiderivative or primitive of a function


d
f(x) if f'(x) = f(x) i.e., {z (x)} = f (x) .
dx
2
For example, x is primitive or antiderivative of x because

2
d x2
d n = .2x = x
1

dx 2 2
d x2
d + 1 n = .2x + 0 = x
1
Similarly,
dx 2 2
h h h
2
d + C n = .2x + 0 = x
d x 1
Similarly,
dx 2 2
In this way, we see that a function has infinitely many antiderivatives or primitives.
i.e., if f(x) be an antiderivative of f(x), then f(x) + C is also antiderivative of f(x), where C is any
constant.
d
Because, {z (x) + C} = z' (x) + 0 = z' (x) = f (x)
dx
Indefinite Integrals: If f(x) is a function, then the family of all its antiderivatives is called Indefinite
Integral of f(x). It is represented by y f (x) dx (read as indefinite integral of f(x) with respect to x)

x3 x4
For example,
y x 2 dx = + C; y x 3 dx = +C
3 4

Why is it called Indefinite Integral?


It is called indefinite because it is not unique. Actually there exist infinitely many integrals of each function,
which can be obtained by choosing C arbitrarily from the set of real numbers.
2. Some Standard Integrals:
+
xn 1 dx
(i) y x n dx = + C (n ! –1) (ii) y
x
= log | x |+ C
n+1

(iii) y dx = x + C (iv) y cos x dx = sin x + C

(v) y sin x dx = – cos x + C (vi) y sec 2 x dx = tan x + C

254 Xam idea Mathematics–XII

@Cbsebookshub - Join Us on Telegram


(vii) y cosec 2 x dx = – cot x + C (viii) y sec x tan x dx = sec x + C

(ix) y cosec x cot x dx = – cosec x + C (x) y e x dx = e x + C

ax
(xi) y a x dx = +C
log a

= sin –1 ` j + C
1 dx x
(xii) (a) y dx = sin –1 x + C (b) y a
2 2 2
1– x a –x

dx = a tan –1 ` a j + C
1 1 1 x
(xiii) (a) y 2
dx = tan –1 x + C (b) y 2 2
+
1 x a x+
1 1
(xiv) y dx = sec –1 x + C (xv) y – dx = cosec –1 x + C
2 2
x x –1 x x –1

dx = a cot –1 ` a j + C (xvii) y dx = a sec –1 ` a j + C


1 1 x 1 1 x
(xvi) y – 2 2
a +x 2
x x –a 2

dx = a cosec –1 ` a j + C
1 1 x
(xviii) y –
2 2
x x –a
3. Methods of Integration: It is not possible to integrate each integral with the help of following
methods but a large number of various problems can be solved by these methods. So, we have the
following methods of integration:
(i) Integration by Substitution.
(ii) Integration by Parts.
(iii) Integration of Rational Algebraic Functions by Using Partial Fractions.
4. Integration by Substitution: The method of evaluating integrals of a function by suitable
substitution is called Integration by substitution.
We therefore give some of the fundamental integrals when x is replaced by ax + b.
+
(ax + b) n 1 1 1
(i) y (ax + b) n dx = + C, n ! –1 (ii) y dx = a log | (ax + b) |+ C
a (n + 1) ax + b
+
+b 1 + + 1 a bx c
(iii) y e ax dx = a e ax b + C (iv) y a bx c dx = . + C, a > 0 and a ! 1
b log a
1 1
(v) y sin (ax + b) dx = – a cos (ax + b) + C (vi) y cos (ax + b) dx = a sin (ax + b) + C

1 1
(vii) y sec 2 (ax + b) dx = a tan (ax + b) + C (viii) y cosec 2 (ax + b) dx = – a cot (ax + b) + C

1
(ix) y sec (ax + b) tan (ax + b) dx = a sec (ax + b) + C

1
(x) y cosec (ax + b) cot (ax + b) dx = – a cosec (ax + b) + C

1 1
(xi) y tan (ax + b) dx = – a log | cos (ax + b) |+ C (xii) y cot (ax + b) dx = a log | sin (ax + b) |+ C

5. More standard results:


y tan x dx = – log | cos x |+ C = log | sec x |+ C, provided x is not an odd multiple of r
2
y
cot x dx = log | sin x |+ C

Integrals 255
@Cbsebookshub - Join Us on Telegram
y sec x dx = log sec x + tan x + C = log tan c + m +C
r x

4 2

y cosec x dx = log | cosec x – cot x |+ C = log tan x + C


2

6. Integration by Parts: To integrate the product of two functions, we use integration by parts. The
method is as given below:
Let u and v be two functions of x then
y u.v dx = u y v dx – y ' du . y v dx 1 dx
dx

Note:
(i) To integrate the product of two functions we choose the 1st function according to word ILATE, where
I stands for inverse function, L stands for logarithmic function, A stands for the algebraic functions,
T stands for trigonometrical function and E stands for exponential function.
(ii) If the integrand has only one function then unity, i.e., 1 is taken to be the second function.

(iii) Integration by parts is not applicable to product of functions in all cases. For example, the method does
1
not work for y x . sin x dx . The reason is that there does not exist any function whose derivative is
1
x . sin x .
(iv) Observe that while finding the integral of the second function, we do not add a constant of integration on
both the sides.
7. Results of Some Special Integrals:
dx 1 x
(i) y 2 = tan –1 + C
+
a x 2 a a

dx 1 x–a dx 1 a+x
(ii) (a) y = log + + C ; (b) y = . log a – x + C
x – a 2 2a
2 x a 2
a –x 2 2a

1 x + x2 + a2
(iii) y dx = log + C or log | x + x 2 + a 2 |+ C
a
a2 + x2
1 x + x2 – a2
(iv) y dx = log + C or log | x + x 2 – a 2 |+C
x2 – a2 a
1 x x 2 a2 x
(v) (a) y dx = sin –1 a + C ; (b) y a 2 – x 2 dx = a – x 2 + sin –1 a + C
2
a –x 2 2 2
x a2
(vi) y x 2 – a 2 dx = x2 – a2 – log | x + x 2 – a 2 |+ C
2 2
x 2 a2
(vii) y a 2 + x 2 dx = a + x 2 + log | x + x 2 + a 2 |+ C
2 2

Theorem 1. The indefinite integral of an algebraic sum of two or more functions is equal to the algebraic sum

of their integrals,
i.e., y [f (x) + g (x)] dx = y f (x) dx + y g (x) dx

Theorem 2. A constant term may be taken outside from the integral sign i.e., if k is a constant then

y k f (x) dx = k y f (x) dx

256 Xam idea Mathematics–XII

@Cbsebookshub - Join Us on Telegram


Theorem 3. If the numerator in an integral is the exact derivative of denominator, then its integral is
logarithmic of denominator,
f l^ x h dx
i.e., y = log | f ^ x h |+ C
f^ xh
Theorem 4. To integrate a function whose numerator is unity and denominator is a homogeneous function
of 1st degree in cos x and sin x i.e., the integrals of these forms:
dx dx dx dx dx
y ,y ,y ,y ,y
a + b cos x a sin x + b a + b sin x a cos x + b sin x a sin x + b cos x
i.e., when integrand is a rational function of sin x and cos x.
To find these, we can use following substitution.
(i) By putting a = r cos a, b = r sin a respectively according to question
OR
a1 – tan 2 k
x x
2 tan
2 2
(ii) By putting sin x = x , cos x
=
a1 + tan 2 k
x
1 + tan 2
2 2
x
and putting tan = t and then simplify.
2
Theorem 5. To integrate a function whose numerator is 1 and denominator is a homogeneous function of the
second degree in cos x and sin x or both, i.e.,

y dx dx dx dx dx
,y ,y ,y ,y 2 2
a + b sin x a sin x + b a cos x + b a + b cos x a sin x + b 2 cos 2 x
2 2 2 2

To evaluate such type of integrals we proceed as follows:


(i) Divide the numerator and denominator by cos2 x and then
(ii) Putting tan x = z or cot x = z and then simplify.
Theorem 6. Integrals of the type y e f (x) .f l (x) dx, y f l (x) cos [f (x)] dx, y sin [f (x)] f l (x) dx,

y log [f (x)] f l (x) dx.

To evaluate these type of integrals, put f(x) = t so that f '(x) dx = dt and then integral converts to the standard

forms for which the integrals are known.
Note: If the integrand is a rational function of ex, then it always needs a replacement as the differentiation
and integration of ex is the same.
Thus, if on substituting denominator = t, the derivative of denominator is not present in the
problem, then we need to generate it by multiplying and dividing by a suitable term containing the
exponential function in numerator and denominator.

Integration by Partial Fractions


P ( x)
8. Rational function: Rational function is defined as the ratio of two polynomials in the form of
Q ( x)
where P(x) and Q(x) are polynomials in x. If the degree of P(x) is less than degree of Q(x) then it is
said to be Proper, otherwise it is called an Improper Rational Function.
P ( x)
Thus if is improper, then by long division method it can be reduced to proper function i.e.,
Q (x)
P (x) P1 (x) P1 (x)
= T (x)+ , where T(x) is a function of x and is a proper rational function. Such
Q(x) Q(x) Q(x)
fractions can be evaluated by breaking in factors given as follows:

Integrals 257
@Cbsebookshub - Join Us on Telegram
S. No. Form of the rational function Form of the partial fraction
px + q A B
1. , a!b +
(x – a)(x – b) (x – a) (x – b)
px + q A B
2. +
(x – a) 2 (x – a) (x – a) 2
px 2 + qx + r A B C
3. + +
(x – a)(x – b)(x – c) (x – a) (x – b) (x – c)
px 2 + qx + r A B C
4. + +
(x – a ) 2 (x – b ) (x – a) (x – a) 2 (x – b)
px 2 + qx + r A B C D
5. + + +
3
(x – a) (x – b) (x – a) (x – a) 2 (x – a) 3 (x – b)
px 2 + qx + r A Bx + C
6. + , where x 2 + bx + c cannot be factored further.
2
(x – a)(x + bx + c) (x – a) x 2 + bx + c
The constants A, B, C, etc. are obtained by equating coefficient of like terms from both sides or by
substituting any value for x on both sides.
dx
To find the integral of the form y 2 , we write
ax + bx + c
2
ax + bx + c = a <x 2 + a x + a F = a>d x + n + f a – 2 pH
2 b c b c b2

2a 4a
b c b2
Now putting x + = t so that dx = dt. Therefore, writing =
a 4a 2 k , and find the integral of

2a
1 y dt
reduced form .
| a | ^± t 2 ± k h
px + q
9. Integrals of the form y 2 dx
ax + bx + c
Step I. The numerator px + q is written in the form
d
px + q = A. (ax 2 + bx + c) + B
dx
⇒ px + q = A(2ax + b) + B
Step II. The value of A and B is obtained by equating the coefficients in the above equation.
Step III. (px + q) is replaced by A(2ax + b) + B and we write the given integral as
^ px + q h A^ 2ax + b h + B
y dx = y dx
2
ax + bx + c ax 2 + bx + c
px + q
10. Integrals of the form y dx .
ax 2 + bx + c
Step I. The numerator px + q is written in the form
d
px + q = A (ax 2 + bx + c) + B
dx
⇒ px + q = A(2ax + b) + B
Step II. The values of A and B are obtained by equating the coeffcients in the above equation.

Step III. (px + q) is replaced by A (2ax + b) + B in given integration as
^ px + q h A {^ 2ax + b h + B ,
y dx = y dx and then solved.
ax 2 + bx + c ax 2 + bx + c

258 Xam idea Mathematics–XII

@Cbsebookshub - Join Us on Telegram


p (x)
11. Integration of the form y dx , where p(x) and q(x) are polynomials such that
q (x)
degree of p(x) ≥ degree q(x).
Step I. p(x) is divided by q(x) and it is written as

p^ xh R^ xh
   = Q^ xh + , where Q(x) is quotient polynomial and R(x) is remainder polynomial.
q^ xh q^ xh
p^ xh R^xh p^ xh R^ xh

Step II. is replaced by e Q ^ x h + o as y dx = y e Q ^ x h) + o dx and then solved.
q^ xh q^xh q^ xh q^ xh

12. Integral of the form y sin m x. cos n x dx


(i) If the exponent of sin x is an odd positive integer, then put cos x = t.
(ii) If the exponent of cos x is an odd integer, then put sin x = t.
13. y e x (f (x) + f l (x)) dx = f (x) .e x + C

Definite Integrals
1. Definition: If F(x) is the integral of f(x) over the interval [a, b], i.e., y f (x) dx = F (x) then the definite
b
integral of f(x) over the interval [a, b] is denoted by y f (x) is defined as
a
b
y f (x) dx = F (b) – F (a)
a

where 'a' is called the lower limit and 'b' is called the upper limit of integration and the interval [a, b] is called

the interval of integration.
2. Integration as a Limit of Sum: If a function f(x) is continuous in an interval [a, b] then it is integrable on
that interval.
b
Therefore, we have y f (x) dx = lim Sn .
a n"3
b
Or, y f (x) dx = lim h [f (a) + f (a + h) + f (a + 2h) + ... + f (a + n – 1h)]
a n"3

∴ lim Sn = lim h [f (a) + f (a + h) + f (a + 2h) + ... + f (a + n – 1h)]


n"3 n"3

Since when n → ∞, i.e., number of intervals is very large, then the width of the interval is very small
which implies that h → 0, so that nh = b – a is a constant.
3. Some Useful Results: The following results will be useful in evaluating the definite integrals as the
limit of sum.
n(n – 1)
(i) / (n – 1) = 1 + 2 + 3 + ... + (n – 1) =
2
n (n – 1)(2n – 1)
(ii) / (n – 1) 2 = 1 2 + 2 2 + 3 2 + ... + (n – 1) 2 =
6
2
n(n – 1)
(iii) / (n – 1) 3 = 1 3 + 2 3 + 3 3 + ... + (n – 1) 3 = f p
2

rn – 1 1 – rn
(iv) a + ar + ar 2 + ... + ar n –1 = a c m^if r > 1h or a c m^if r < 1h
r–1 1–r

Integrals 259
@Cbsebookshub - Join Us on Telegram
(n – 1) h
sin * a + 4 sin
nh
2 2
(v) sin a + sin (a + h)+ sin (a + 2h) + ... + sin {a + (n – 1) h} =
h
sin
2
(n – 1) h
cos * a + 4 sin
nh
2 2
(vi) cos a + cos(a + h)+ cos(a + 2h) + ... + cos{a + (n – 1) h} =
h
sin
2
4. Fundamental Properties of Definite Integrals: There are certain properties of definite integrals
which can be used while solving the definite integral.
b b
(i) y f ^xh dx = y f ^ z h dz (Change of variable)
a a

b a
(ii) y f ^ x h dx = – y f ^ x h dx (Inter change the limits)
a b

b c b
(iii) y f ^ x h dx = y f ^ x h dx + y f ^ x h dx, where a < c < b (Change the limits)
a a c

a a b b
(iv) (a) y f ^ x h dx = y f ^ a – x h dx (b) y f ^ x h dx = y f ^ a + b – x h dx
0 0 a a

2a a a
(v) y f ^ x h dx = y f ^ x h dx + y f ^ 2a – x h dx , then following cases will occur:
0 0 0

2a a 2a
(a) y f (x) dx = 2 y f (x) dx, if f (2a – x) = f (x) (b) y f (x) dx = 0, if f (2a – x) = – f (x)
0 0 0

na a
(vi) y f ^xh dx = n y f ^xh dx, if f ^xh = f ^a + xh
0 0

a a
(vii) (a) y f ^ x h dx = 2 y f ^ x h dx , if f is an even function, i.e., f(– x) = f(x)
–a 0

a
(b) y f (x) dx = 0 , if f is an odd function, i.e., f (–x) = – f(x)
–a

Selected NCERT Questions


dx
1. Find y .
x– x
dx dx
Sol. I = y =y
x– x x ( x –1)
1
Let x – 1 = t ⇒ dx = dt
2 x
dx

= 2dt
x
dt
∴ I = 2y = 2 log t + C = 2 log x –1 + C
t

260 Xam idea Mathematics–XII

@Cbsebookshub - Join Us on Telegram


e 2x – e –2x
2. Find : y dx [CBSE (F) 2011]
e 2x + e –2x
dt
Sol. Put (e2x + e –2x) = t ⇒ (2e2x – 2e– 2x) dx = dt ⇒ (e2x – e–2x) dx =
2
e 2x – e –2x 1 1 1 1
∴ y
dx = y dt = log | t |+ C = log | e 2x + e –2x |+ C
2x
e +e –2x 2 t 2 2

tan x
3. Find : y dx
sin x cos x
tan x tan x sec 2 x
Sol. y dx = y dx = y dx
sin x cos x sin x tan x
. cos 2 x
cos x
Put tan x = t ⇒ sec2 x dx = dt.
sec 2 x 1 t1/2

y dx = y dt = y t –1/2 dt = + C = 2 t + C = 2 tan x + C
tan x t 1/2
(x + 1) (x + log x) 2
4. Find : y x dx

(x + 1) (x + log x) 2 x+1
dx = y d n (x + log x) 2 dx = y d1 + n (x + log x) 2 dx
1
Sol. y
x x x
1
Put (x + log x) = t
& d1 + n dx = dt
x
1 t3
y d1 + n (x + log x) 2 dx = y t 2 dt = + C = (x + log x) 3 + C
1

x 3 3
x3 sin (tan –1 x 4)
5. Find : y dx
1 + x8
x 3 sin (tan –1 x 4) 4x 3 x3 dt
Sol. y 8
dx , put tan –1 4
( x ) = t & 8
dx = dt & 8
dx =
4
1+x 1+x 1+x
x 3 sin (tan –1 x 4) 1 1 –1

y dx = y sin t dt = (– cos t) + C = cos (tan –1 x 4) + C
1 x+ 8 4 4 4
cos 2x – cos 2a
6. Find : y cos x – cos a dx
(CBSE (AI) 2013)

cos 2x – cos 2a (2 cos 2 x –1) – (2 cos 2 a–1) cos 2 x – cos 2 a


Sol. Let I=y dx = y dx = 2 y dx
cos x – cos a cos x – cos a cos x – cos a
(cos x – cos a) (cos x + cos a)
= 2y dx = 2 y (cos x + cos a) dx
(cos x – cos a)

= 2 y cos x dx + 2 cos a y 1.dx = 2 sin x + 2x cos a + C


1
7. Find : y dx
2
x + 2x + 2
1 1 1
Sol. y 2
dx = y 2
dx = y dx
x 2x + 2
+ (x 2x + 1) + 1
+ (x 1) 2 + 1
+
Putting x + 1 = t ⇒ dx = dt
1 1
` y dx = y dt
2
+
(x 1) 1 + ( t ) + (1 ) 2
2

Integrals 261
@Cbsebookshub - Join Us on Telegram
= log | t + t 2 + 1 |+ C = log | (x + 1) + (x 2 + 2x + 1) + 1 |+ C

= log | (x + 1) + x 2 + 2x + 2 |+ C

1
8. Find : y dx
9x 2 + 6x + 5
1 1 1
Sol. y 2 dx = y dx
9x + 6x + 5 9 6 5
x2 + x +
9 9
1 1 1 1 1
= y
dx = y dx, putting x + = t ⇒ dx = dt
9 1 2
1 2 9 1 2
2 2 3
x2 + x + + d n –d n
2 5
dx + n + d n
3 9 3 3 3 3
RS V
S 3 d x + 1 n WWW 3x + 1
dt = . tan d n + C = tan SS 3 WW + C = tan –1 d n+ C
1 1 1 1 –1 t 1 –1 S 1
= y

9 2 2 9 2 2/3 6 SS WW 6 2
t +d n
2
3 T
2
X
3
5x + 3
9. Evaluate: y 2
dx [CBSE Delhi 2011; (AI) 2010]
x + 4x + 10

d 2
Sol. We can express the Nr as 5x + 3 = A (x + 4x + 10) + B
dx
⇒ 5x + 3 = A (2x + 4) + B ⇒ 5x + 3 = 2Ax + (4A + B)

Equating the coefficients, we get
2A = 5 and 4A + B = 3
5 5
A= & 4 # + B = 3 & B = 3 – 10 = – 7
2 2
5
∴ 5x + 3 = (2x + 4) + ( – 7)
2
5
(2x + 4) – 7 (2x + 4)
2 5 dx
∴ I=y dx = y dx – 7 y
2
x + 4x + 10 2 2
x + 4x + 10 2
x + 4x + 10
5
I = I1 – 7I2 …(i)
2
2x + 4 dx
where I1 = y dx and I2 = y
2 2
+
x 4x 10 + x 4x + 10
+
(2x + 4)
Now, I1 = y dx
x 2 + 4x + 10
Let x2 + 4x + 10 = t ⇒ (2x + 4)dx = dt
+
dt t –1/2 1

I1 = y = y t –1/2 dt = + C1 = 2 t + C1
t 1
– + 1
2
I1 = 2 x 2 + 4x + 10 + C1
dx dx
Again, I2 = y =y
2 2
x + 2.x.2 + 2 – 4 + 10 ( x + 2) 2 + ( 6 ) 2

= log| (x + 2) + x 2 + 4x + 10 |+ C2

Putting the value of I1 and I2 in (i), we get

262 Xam idea Mathematics–XII

@Cbsebookshub - Join Us on Telegram


5 5
I= # 2 x 2 + 4x + 10 – 7 log | (x + 2) + x 2 + 4x + 10 |+ c C1 – 7C2 m
2 2
= 5 x 2 + 4x + 10 – 7 log | (x + 2) + x 2 + 4x + 10 |+ C
5
where C = d C1 –7C2 n
2
1 – x2
10. Find : y dx
x (1 – 2x)
RS 1 VW
1 – x2 SS – x + 1 WW
Sol. y dx = y SS 1 + 2 WW dx = 1 y dx – 1 y x – 2 dx = x – 1 I ...(i)
x ( 1 – 2x) SS 2 W 2 2 x (1 – 2x) 2 2 1
x (1 – 2x) W
T X
x–2
Now, I1 = y dx
x (1 – 2x)
x–2

 is a proper rational function
x (1 – 2x)
x–2 A B
∴ = + ...(ii)
x (1 – 2x) x 1 – 2x

⇒ x – 2 = A (1 – 2x) + Bx ⇒ x – 2 = (–2A + B)x + A



A = – 2 and – 2A + B =1 ⇒ B = 1 + 2A = 1 + 2 (–2) = –3
Putting values of A and B in (ii), we have
x–2 –2 3
= –
x (1 – 2x) x 1 – 2x

dx = y ; x – E dx
x–2 –2 3
∴ y

x (1 – 2x) 1 – 2x
1 1 log | 1 – 2x |
= – 2 y dx – 3 y dx = –2 log | x | – 3 + C1
x 1 – 2x –2
3
= –2 log | x |+ log | 1– 2x |+ C1
2
Putting the value I1 in (i), we have
1 –x 2
dx = – ;–2 log| x |+ log | 1 – 2x |+ C1E
x 1 3
y
x (1 – 2x) 2 2 2
x 3 C
= + log | x | – log | 1 – 2x | – 1
2 4 2
x 3 C
= + log x – log 1 – 2x + C , where C = – 1
2 4 2
x3 + x + 1
11. Find : y dx
x2 – 1
x3 + x + 1 2x + 1 2x + 1 x2
= G
2x 1
Sol. y dx = y x + 2 dx = y xdx + y dx = +y 2 dx + y 2 dx
2
x –1 x –1 2
x –1 2 x –1 x –1
Putting x2 – 1 = t ⇒ 2x dx = dt in second integral, we get
x2 y 1 1 x2 1 x–1
= + dt + y 2 dx = + log | t |+ log + + C
2 t x – (1) 2 2 2 (1) x 1
x2 1 x – 1
= + log | x 2 – 1 |+ log +C
2 2 x+1

Integrals 263
@Cbsebookshub - Join Us on Telegram
x cos – 1 x
12. Evaluate: y dx [CBSE (F) 2014]
1 – x2
x cos –1 x
Sol. Let I=y dx
1 – x2
1 1
Put cos –1 x = z & – dx = dz & dx = – dz
2
1– x 1 – x2
I = – y cos z.z dz = – z. sin z + y sin zdz + C = – z sin z – cos z + C

I = – cos –1 x. 1 – x 2 – x + C [a x = cos z & sin z = 1 – x 2 ]

I = – 1 – x 2 cos –1 x – x + C
1 + sin x
13. Find : y e x d n dx
1 + cos x

KJK 1 + 2 sin x cos x ONO


1 + sin x K 2 OO
I= ye d n dx = y e KK
x x 2
Sol. Let OO dx
1 + cos x KK 2 x OO
K 2 cos
L 2 P
JK x x NO
KK 2 sin cos OO
e c sec + tan m dx
1 2 2O =y x 1 2 x x
= y e x KK + OO dx
KK 2 x 2 x O 2 2 2
K 2 cos 2 2 cos
2 OP
L
x 1 x

= y e x tan dx + y e x sec 2 dx
2 2 2
I II

= c tan m e – y c sec 2 m e x dx + y e x sec 2 dx ⇒


x x 1 x 1 x x
I = e x tan +C
2 2 2 2 2 2
4
14. Evaluate: y1 (x 2 – x) dx as limit of sums. [CBSE Delhi 2010; (F) 2011]
4
Sol. y1 (x 2 – x) dx

We have to solve it by using limit of sums.


b–a 4–1
Here, a = 1, b = 4, h = n
=
n i.e., nh = 3
4
Limit of sum for y1 (x 2 – x) dx is

= lim h [f (1) + f (1 + h) + f (1 + 2h) +.... + f {1 + (n – 1) h}]


h"0

Now, f (1) = 1 – 1 = 0
f (1 + h) = (1 + h)2 – (1 + h) = h2 + h
f (1 + 2h) = (1 + 2h)2 – (1 + 2h) = 4h2 + 2h
……………………………………………
……………………………………………
……………………………………………
f [1 + (n – 1)h] = {1 + (n –1)h}2 – {1 + (n – 1)h} = (n – 1)2 h2 + (n – 1) h
2

y1 (x 2 – x)dx = lim h [0 + h 2 + h + 4h 2 + 2h +.... (n – 1) 2 h 2 + (n – 1) h]
h"0

= lim h [h 2 {1 + 4 + .. + (n – 1) 2} + h {1 + 2 + ... (n – 1)}]


h"0

264 Xam idea Mathematics–XII

@Cbsebookshub - Join Us on Telegram


(n) (n – 1) (2n – 1) n (n – 1)
= lim h ;h 2 . +h E
h"0 6 2
n (n – 1) (2n – 1) n (n – 1)
[a 1 + 4 + ... + (n – 1) 2 = and 1 + 2 + ... + (n – 1) = E
6 2
nh (nh – h) (2nh – h) nh (nh – h)
= lim ; + E
h"0 6 2

(3 – h) (3) (6 – h) (3 – h) (3) 3#3#6 3#3 9 27


= lim ; + E=c m+c m= 9+ =
h"0 6 2 6 2 2 2

5
15. Evaluate: y–5| x + 2 | dx [CBSE (F) 2010]

Sol. Here, function is |x + 2| which is defined as


|x + 2| = (x + 2), if x $ – 2
– (x + 2), if x < – 2 {
So, we have
b c b
>a H

5 –2 5
y–5| x + 2 | dx = y–5 – (x + 2) dx + y–2 (x + 2) dx y f (x) dx = ya f (x) dx + yc f (x) dx
a<c<b
–2 5
x2 x2
= <– – 2xF + < + 2xF
2 –5 2 –2

(–2) 2 (–5) 2 (5) 2 (–2) 2


=– – 2 (–2) + + 2 # (–5) + + 2 # (5) – – 2 # (–2)
2 2 2 2
25 25
= –2 + 4 + – 10 + + 10 – 2 + 4 = 29
2 2
r /4
16. Evaluate: y0 log(1 + tanx) dx [CBSE (AI) 2011]

r/ 4
Sol. Let I = y0 log (1 + tan x) dx …(i)

log :1 + tan a
– x kD dx (By using property y0 f (x) dx = y0 f (a – x) dx)
r/ 4 r a a

I = y0
4
RS r V
SS tan – tan x WWW
4
= y0 log SSS1 + WW dx
r/ 4
r W
SS 1 tan .tan x WW
+
T 4 X
1 + tan x + 1 – tan x
= 0 log ;1 + E dx = 0 log ; E dx
r/ 4 1 – tan x r/ 4
y y
1 + tan x 1 + tan x
r/ 4 2 r/ 4
I = y0 log dx = y0 [log 2 – log (1 + tan x)] dx …(ii)
1 + tan x

Adding (i) and (ii), we get


r/ 4 r/ 4
2I = y0 log 2 dx = log 2 y0 dx = log 2 [x] r0 /4
r r
2I = log 2 ⇒ I= log 2
4 8
3/2
17. Evaluate : y–1 | x sin rx | dx [CBSE (F) 2010; Central 2016]
3/2
Sol. y–1 | x sin rx | dx

As we know

Integrals 265
@Cbsebookshub - Join Us on Telegram
sin q = 0 ⇒ q = np, n ∈ Z
∴ sin px = 0
x = 0, 1, 2, ...
For – 1 < x < 0, x < 0, sin px < 0 ⇒ x sin px > 0
For 0<x<1
x > 0, sin px > 0   ⇒ x sin px > 0
3
For 1 1 x 1 , x 2 0, sin rx 1 0   ⇒ x sin r x 1 0
2
3/2 1 3/2

y–1 | x sin rx | dx = y–1 x sin rx dx + y1 (– x sin rx) dx

(– cos rx) 1
= ;x. E – y –1 1.
– cos rx – cos rx 3/2 3/2 – cos rx
dx – :x. D + y1 1.
1
dx
r –1 r r 1 r
1 3/2
x 1 x 1
= <– cos rx + 2 sin rxF – <– cos rx + 2 sin rxF
r r –1 r r 1

=; + 0 + – 0E – <0 – 2 – F = < + + 2 + F
1 1 1 1 1 1 1 1

r r r r r r r r
1 3 1 + 3r
= 2+ =
r r r2
18. Find y e.x (cos x – sin x) cosec 2 x dx .[CBSE 2019(65/5/1)]
Sol. Let I = y e x (cos x – sin x) cosec 2 x dx = y e x (cot x . cosec x – cosec x) dx

= y e x cosec x cot x dx – y e x cosec x dx


II I

= y e cosec x. cot x – cosec x e + C + y – cosec x cot x e x dx


x x

[Using integration by parts for 2nd integral]


= y e x cosec x. cot x dx – e x cosec x + C – y e x cosec x. cot x dx
= – excosec x + C.
r/3 sin x + cos x
19. Evaluate: yr/6 dx [CBSE Delhi 2010]
sin 2x
Sol. We have,
r/3 sin x + cos x r/3 sin x + cos x
I = yr/6 dx   ⇒ I = yr/6 dx
sin 2x 1 – (cos x – sin x) 2
Let t = (cos x – sin x) ⇒ dt = – (sin x + cos x) dx
r r r 3 –1
The limits are, when x = ⇒ t = cos – sin =
6 6 6 2
r r r 1– 3
and x = ⇒ t = cos – sin =
3 3 3 2
(1– 3 )/2 1
∴ I = – y( 3 –1)/2 dt
1 – t2
1– 3
= – <sin –1 F = <– sin – 1 F
2 1– 3 3 –1 1– 3 3 –1
  = – [sin –1 t] 3 –1 – sin –1 + sin – 1
2 2 2 2
2

3 –1

I = 2 sin –1 [ sin–1 (–x) = – sin–1 x]
2

266 Xam idea Mathematics–XII

@Cbsebookshub - Join Us on Telegram


r/4 sin x + cos x
20. Evaluate: y0 dx [CBSE (F) 2011, 2013, 2014]
9 + 16 sin 2x
sin x + cos x
r/ 4
Sol. Let I = y0 dx
9 + 16 sin 2x
Here, we express denominator in terms of sin x – cos x which is integral of the numerator.
We have, (sin x – cos x)2 = sin2 x + cos2 x – 2sin x cos x = 1 – sin 2x

sin 2x = 1 – (sin x – cos x)2

r/ 4 sin x + cos x

I = y0 dx
9 16 {1 – (sin x – cos x) 2}
+

r/ 4 sin x + cos x

I = y0 dx
25 – 16(sin x – cos x) 2
Let sin x – cos x = t ⇒ (cos x + sin x)dx = dt
r r r
The limits are, when x = 0 & t = sin 0 – cos 0 = –1 and x = & t = sin – cos = 0
4 4 4
0 dt
∴ I = y–1
25 – 16t 2
RS 5 VW0
SS + t WW
1 0 dt 1 0 dt 1 1 S WW
⇒ I= y–1 = y–1 & I= . SSlog 4 WW
16 25 16 5 2 16 5 S 5
– t2 c m – t2 2 c m SS – t WW
16 4 4 4
T X–1
<log 1 – log d nF
1 1/4 1 1 1
⇒ I= & I= ;0 – log c mE = log 9
40 9/4 40 9 40

r x tan x
21. Evaluate: y0 dx [CBSE Delhi 2008, 2010, (AI) 2008, 2017, (F) 2010, 2013, 2014]
sec x + tan x
r x tan x
Sol. Let I = y0 dx …(i)
sec x + tan x
( r – x)tan( r – x)
7a y0 f (x) dx = y0 f (a – x) dxA
a a
= y0r dx
sec( r – x) + tan( r – x)
(r – x) tan x
= y0r dx …(ii)
sec x + tan x

By adding equations (i) and (ii), we get
tan x r
2I = r y0 dx
sec x + tan x
Multiplying and dividing by (sec x – tan x), we get
r tan x (sec x – tan x) r
2I = r y0 dx = r y0 (sec x tan x – tan 2 x) dx
sec 2 x – tan 2 x
= r y0r sec x tan x dx – r y0r sec 2 x dx + y0r dx

= r [sec x] r0 – r [tan x] r0 + r [x] r0 = r (–1 – 1) – 0 + r (r – 0) = r (r – 2)

r
⇒ 2I = p(p – 2) ⇒ I= ( r – 2)
2

Integrals 267
@Cbsebookshub - Join Us on Telegram
Multiple Choice Questions [1 mark]
Choose and write the correct option in the following questions.
3
1. y x 2 e x dx is equal to [CBSE 2020 (65/5/1)]
1 3 1 4 1 3 1 x2
(a) e x + C (b) e x + C (c) e x + C (d) e +C
3 3 2 2
x
e (1 + x)
2. y dx is equal to [CBSE 2020 (65/3/1)]
cos 2 (xe x)

(a) tan (xe x) + C (b) cot (xe x) + C (c) cot (e x) + C (d) tan [e x (1 + x)] + C
3. y e x (cos x – sin x) dx is equal to
(a) e x cos x + C (b) e x sin x + C (c) – e x cos x + C (d) – e x sin x + C
dx
4. y is equal to [NCERT Exemplar]
sin 2 x cos 2 x
(a) tan x + cot x + C (b) (tan x + cot x) 2 + C (c) tan x – cot x + C (d) (tan x – cot x) 2 + C
3e x – 5e –x
5. If y dx = ax + b log 4e x + 5e –x + C then
4e x + 5e –x
–1 7 1 7 –1 –7 1 –7
(a) a = , b = (b) a = , b = (c) a = ,b= (d) a = , b =
8 8 8 8 8 8 8 8
cos 2x – cos 2i
6. y dx is equal to [NCERT Exemplar]
cos x – cos i

(a) 2 (sin x + x cos i) + C (b) 2 (sin x – x cos i) + C


(c) 2 (sin x + 2x cos i) + C (d) 2 (sin x – 2x cos i) + C
dx
7. y is equal to [NCERT Exemplar]
sin (x – a) sin (x – b)
sin (x – b) sin (x – a)
(a) sin (b – a) log + C (b) cosec (b – a) log +C
sin (x – a) sin (x – b)

sin (x – b) sin (x – a)
(c) cosec (b – a) log + C (d) sin (b – a) log +C
sin (x – a) sin (x – b)

8. y tan –1 x dx is equal to [NCERT Exemplar]


(a) (x + 1) tan –1 x – x + C (b) x tan –1 x – x + C
(c) x – x tan –1 x + C (d) x – (x + 1) tan –1 x + C

1–x 2
9. y e x d n dx is equal to [NCERT Exemplar]
1 + x2
ex – ex ex – ex
(a) + C (b) + C (c) + C (d) +C
1 + x2 1 + x2 ( 1 + x 2) 2 ( 1 + x 2) 2
sin6 x
10. y dx is equal to
cos8 x
tan6 x tan7 x tan7 x
(a) + C (b) (c) + C (d) none of these
5 5 7

268 Xam idea Mathematics–XII

@Cbsebookshub - Join Us on Telegram


x3
11. If y dx = a (1 + x 2) 3/2 + b 1 + x 2 + C, then [NCERT Exemplar]
1 + x2
1 1 –1 1
(a) a = , b = 1 (b) a = – , b = 1 (c) a = , b = –1 (d) a = , b = –1
3 3 3 3
x9
12. The integral y dx is equal to [NCERT Exemplar]
(4x 2 + 1) 6
1 1 –5 1 1 –5
(a) e 4 + o + C (b) e 4 + o +C
5x x2 5 x2
–5
1 1
]5g–5 + C (d)
1
(c) e 2 + 4o + C
10x 10 x
r /8
13. y tan 2 (2x) dx is equal to [CBSE 2020, (65/4/1)]
0
4–r 4+r 4–r 4–r
(a) (b) (c) (d)
8 8 4 2
b+c
14. ya + c f (x) dx is equal to
b b b b–c
(a) ya f (x – c) dx (b) ya f (x + c) dx (c) ya f (x) dx (d) ya – c f (x) dx

15. If f and g are continuous functions in [0, 1] satisfying f (x) = f(a – x) and g(x) + g(a – x) = a, then
a
y0 f (x) .g (x) dx is equal to [NCERT Exemplar]
a a a a a
(a) (b) y0 f (x) dx (c) y0 f (x) dx (d) a y0 f (x) dx
2 2

1 x3 + x + 1
16. y–1 dx is equal to [NCERT Exemplar]
x2 + 2 x + 1
1
(a) log 2 (b) 2 log 2 (c) log 2 (d) 4 log 2
2
et 1 1 et
17. y0dt = a, then y0 dt is equal to
1+t (1 + t ) 2
e e e e
(a) a – 1 + (b) a + 1 – (c) a – 1 – (d) a + 1 +
2 2 2 2
2
18. y x cos r x dx is equal to [NCERT Exemplar]
–2
8 4 2 1
(a) r (b) r (c) r (d) r
r
2 tan x
19. The integral value of y 0 dx, m > 0 , is
1 m 2 tan 2 x
+
log m m2 – m
(a) (b) log d n (c) log 3 m (d) 0
(m 2 – 1) 2

2
20. y–1| x | dx is equal to
3 5
(a) 1 (b) (c) 2 (d)
2 2

Integrals 269
@Cbsebookshub - Join Us on Telegram
Answers
1. (a) 2. (a) 3. (a) 4. (c) 5. (a) 6. (c)
7. (c) 8. (a) 9. (a) 10. (c) 11. (b) 12. (d)
13. (a) 14. (b) 15. (b) 16. (b) 17. (b) 18. (a)
19. (a) 20. (d)

Solutions of Selected Multiple Choice Questions


3
1. We have, I = y x 2 e x dx
dt
x3 = t & 3x2 dx = dt & x2 dx = 3
dt 1 t 1

I = y et = y e dt = et + C
3 3 3
1 x3
= e +C
3
e x (1 + x)
2. Let I = y
cos 2 (xe x)
Let xe x = t & (xe x + e x) dx = dt
& (x + 1) e x dx = dt
dt
` I=y
cos 2 t

I = y sec 2 tdt = tan t + C = tan (xe x) + C

dx (sin 2 x + cos 2 x) dx
4. I = y = y
sin 2 x cos 2 x sin 2 x cos 2 x
= y sec 2 xdx + y cosec 2 xdx = tan x – cot x + C

dx 1 sin (b – a)
7. I = y = y dx
sin (x – a) sin (x – b) sin (b – a) sin (x – a) sin (x – b)
1 sin (x – a – x + b)
= y dx
sin (b – a) sin (x – a) sin (x – b)

1 sin "(x – a) – (x – b) ,
= y dx
sin (b – a) sin (x – a) sin (x – b)
1 sin (x – a) cos (x – b) – cos (x – a) sin (x – b))
= y dx
sin (b – a) sin (x – a) sin (x – b)
1 y [cot (x – b) – cot (x – a)] dx
  =
sin (b – a)
1
  = [log| sin (x – b)| – log| sin (x – a) |] + C
sin (b – a)
sin (x – b)
  = cosec (b – a) log +C
sin (x – a)
r/8 r/ 8
13. y tan 2 (2x) dx = y _ sec 2 (2x) –1 i dx
0 0

270 Xam idea Mathematics–XII

@Cbsebookshub - Join Us on Telegram


r/8
tan (2x)
<
= –xF
2 0
r
tan
= 4 – r –0
2 8
1 r 4–r
= – =
2 8 8
a a a
15. I = y f (x) .g (x) dx = y f (a – x) g (a – x) dx = y f (x) (a – g (x)) dx
0 0 0
a a a
a a
= a y f (x) dx – y f (x) .g (x) dx ⇒ I = a y f (x) dx – I ⇒ I =
y f (x) dx
0 0 0 20
3
x + x +1
1
16. I = y 2 dx
–1 x + 2 x + 1

x3 1 1 x +1 1 x +1
= y + y dx = 0 + 2 y dx
0 ^ x + 1h
2 2 2
–1 x + 2 x + 1 –1 x + 2 x + 1

[odd function + even function]


1 +1 1
x 1 1
=2y dx = 2 y dx = 2 log x + 1 0 = 2 log 2.
0 (x + 1)
2
0 x + 1
2 2
18. Since I = y x cos rx dx = 2 y x cos rx dx
–2 0
]Z] 1 3 b_b
]] 2 2 2 bb 8
= 2 [] y x cos rx dx + y x cos rx dx + y x cos rx dx `b = r
]]] 0 1 3 bb
b
2 2
\ a
Fill in the Blanks [1 mark]
r/ 2
sin n x dx
1. y = _____________ .
0 sin n x + cos n x

2. y0r/2 cos x esinx dx = _____________ . [NCERT Exemplar]

x
y etan x d1 + n dx = _____________ .
–1
3.
1 + x2
4. A primitive of |x|, when x < 0 is _____________ .
r
5. The value of y sin 3 x cos 2 x dx = _____________ .
–r

Answers
r –1 1
1. 2. e – 1 3. xetan x
+ C 4. – x 2 + C 5. 0
4 2

Solutions of Selected Fill in the Blanks


2. Let I, = y0r/2 cos x esinx dx
Let sin x = t ⇒ cos x dx = dt

When x = 0 ⇒ t=0
r
x= ⇒ t=1
2

Integrals 271
@Cbsebookshub - Join Us on Telegram

I= y01 e t dt = 6e t@10 = e1 –e0 = e –1
x
y etan x d1 + n dx
–1
3. Let I =
1 + x2
1
Let tan –1 x = t ⇒ dx = dt
1 + x2
1 + x2 + x
I = y e tan x e o dx
–1

1 + x2
= y et (1 + tan 2 t + tan t) dx

= y et ^sec 2 t + tan t h dx = y et (tan t + sec 2 t) dt


= et tan t + C
–1
x
= xe tan +C
r
5. Let I = y sin 3 x cos 2 x dx
–r

Let f (x) = sin 3 x. cos 2 x ⇒ f (–x) = – sin 3 x cos 2 x


⇒ f (–x) = –f (x)
∴ f (x) is an odd function
a
` e y f (x) dx = 0, when f (x) is an odd function o
r

I = y sin 3 x cos 2 x dx = 0
–r –a

Very Short Answer Questions [1 mark]


dx
1. Evaluate: y  [CBSE 2020 (65/3/1)]
9 4x 2
+
dx 1 dx 1 dx
Sol. y = y = y
9 + 4x 2 4 9 4 3 2
4
+ x2 d n + x2
2

× tan –1 d n+ C
1y dx 1 2 x
= =
4 2 4 3 3/2
x2 + c m
3
2

tan –1 c m + C
1 2x
=
6 3
+1
2x –5 x–1
2. Find: y dx  [CBSE 2020 (65/4/1)]
10 x
+1 +
2x – 5 x–1 2x 1 5 x–1
Sol. y dx = y dx – y dx
10 x ( 5 × 2) x ( 5 × 2) x
2 x ×2 5x 1
= y x x dx – y x x × dx
5 ×2 5 ×2 5
1 –2×5 –x 1 –2 –x
= 2 y 5 dx – y 2 –x dx =
– x
– × +C
5 log 5 5 log 2
–2 –x 1 2 –x
= 5 + × +C
log 5 5 log 2
1 1 2 1
= . – . +C
5 log 2 2 x log 5 5 x

272 Xam idea Mathematics–XII

@Cbsebookshub - Join Us on Telegram


( x 2 + 2)
3. y dx [NCERT Exemplar]
x+1
x2 + 2
Sol. Let I=y dx
x+1
3
= y dx – 1 + n dx
x+1
1
= y (x – 1) dx + 3 y dx
x+1
x2
= – x + 3 log x + 1 + C
2
4. Evaluate: y sec 2 (7 – x)dx [CBSE Delhi 2009; (AI) 2010]
tan (7 –x)
Sol. y sec 2 (7 – x)dx = + C = – tan (7 – x) + C
–1
sec 2 x
5. Evaluate: y dx [CBSE (AI) 2009]
x
1 dx
Sol. Let x=z ⇒ dx = dz ⇒ = 2dz
2 x x
sec 2 x

y dx = 2 y sec 2 z dz = 2 tan z + C = 2 tan x + C
x
dx
6 Evaluate: y [CBSE (F) 2009]
x + x log x
dx
Sol. Let I = y
x (1 + log x)
1
Put 1 + log x = z, ⇒ x dx = dz
dz
∴ I=y = log z + C = log | 1 + log x |+ C
z
a
dx r
7 If y = , then find the value of a. [CBSE 2020 (65/4/1)]
0 1 + 4x
2 8
a
dx r
Sol. We have, y =
0 1 + 4x
2 8
1 ya dx r

=
40 1 2 8
+x
4
1 ya dx r

=
40 2 8
(x) 2 + c m
1
2
× 2 6tan –1 2x@0 =
1 a r


4 8
–1 –1 r
⇒ tan 2a – tan 0 =
4
r r
⇒ tan –1 2a = & 2a = tan = 1
4 4
1

a=
2

Integrals 273
@Cbsebookshub - Join Us on Telegram
4
8 Find the value of y1 | x – 5 | dx [CBSE 2020 (65/5/1)]
4 4
Sol. We have, y x – 5 dx = y – (x – 5) dx
1 1
4
(4) 2 (1) 2
–< – 5xF = – =
2
+ 5 × 1G
x
= – 5×4 –
2 1 2 2

–; – 20 – + 5E = – ; – 15E
16 1 15
=
2 2 2
15
=
2
1
9. If y0 (3x 2 + 2x + k)dx = 0, then find the value of k. [CBSE Delhi 2009]
1
3x 3 2x 2
< + kxF = 0
1
Sol. Given, y0 (3x 2 + 2x + k) dx = 0 ⇒ +
3 2 0

6x 3 + x 2 + kx@0 = 0 ⇒
1

(1 + 1 + k) – (0) = 0 ⇒ k = –2

Short Answer Questions-I [2 marks]


tan 3 x
1. Find y dx [CBSE 2020 (65/2/1)]

cos 3 x
Sol. We have,
sin 3 x
tan 3 x cos 3 x
I=y
3
dx = y dx
cos x cos 3 x
sin 3 x
⇒ I= y dx
cos6 x
Let cos x = t ⇒ – sin x dx = dt
⇒ sin x dx = – dt
sin 2 x. sin x dx (1 – cos 2 x) . sin x

I= y = y dx
cos6 x cos6 x
1 – t2 t2
– ye o y 1 y
=
dt = – dt + dt
t6 t6 t6
t –5 t –3
=
– y t –6 dt + y t –4 dt = + +C
5 –3
1 1 1 1
=
× – × +C
5 t5 3 t3
1 1 1 1
= 5
– 3
+C= – +C
5 (cos x) 3 (cos x) 5 cos x 3 cos 3 x
5

1 + sin 2x
2. Find y e x dx . [CBSE Sample Paper 2018]
1 + cos 2x
1 + sin 2x sin 2 x + cos 2 x + 2 sin x. cos x
Sol. I = y e x dx = y e x dx
1 + cos 2x 1 + cos 2x
(sin x + cos x) 2 sin x + cos x 1 sin x cos x
= y ex dx = y e x dx = y e x d + n dx
1 + cos 2x 2
2 cos x 2 cos x cos 2 x
2

274 Xam idea Mathematics–XII

@Cbsebookshub - Join Us on Telegram


1 y x
= e (sec x + sec x . tan x) dx
2
1
= e x . sec x + C     [a y e x (f (x)+ f l(x))dx = e x f (x)+C ]
2
x–1
3. Find y dx.  [CBSE 2019 (65/5/1)]
(x – 2) (x – 3)
x–1 A B
Sol. a = +
(x – 2) (x – 3) x – 2 x – 3
x–1 x–1
where A = &B=
x–3 x=2 x–2 x=3

1

A= = –1 & B = 2
–1
x–1 –1 2
⇒ = +
(x – 2) (x – 3) (x – 2) (x – 3)
x–1 dx dx
⇒ y dx = – y + 2y = – log (x – 2 ) + 2 log (x – 3) + C
(x – 2) (x –3) x–2 x–3
x–1 2
(x – 3) 2
y = +
dx – log (x – 2) log (x – 3) + C = log +C
(x – 2) (x –3) ( x – 2)
0
1 + tan x
4. Find y dx . [CBSE 2019 (65/5/1)]
–r /4 1 – tan x

dx = y tan c + x m dx
0 0
1 + tan x r
Sol. y
–r / 4 1 – tan x –r / 4
4

= log sec c + xm = log sec c m – log sec c – m


r 0 r r r
4 –r/4 4 4 4
= log ^ 2 h – log (sec (0)) = log ^ 2 h – log 1
1
= log 2 = log 2
2
dx
5. Find y .[CBSE 2019 (65/4/1)]
5 – 4x – 2x 2
dx dx
Sol. y =y
2
5 – 4x –2x 7 – 2 – 4x –2x 2
dx 1 y dx
=y
=
7 – 2 (1 + 2x + x ) 2 2 7
– (x + 1) 2
2
JK x + 1 NO
K O
sin –1 c (x + 1) m + C
1 y dx 1 1 2
=
= sin KK 7 OOO + C =
–1 K
2 2 2 K 2 O 2 7
c m – (x + 1) 2
7
L P
2
1 tan – 1 x
6. Evaluate y0 dx . [CBSE Delhi 2011]
1 + x2
1
Sol. Let t = tan –1 x & dt = dx
1 x2
+
r
Also when, x = 0, t = 0 and when x = 1, t =
4

Integrals 275
@Cbsebookshub - Join Us on Telegram
r
1 tan –1 x 4

y0 dx = y0 tdt
2
1+x
r/ 4
t2 1 r2 r2
=< F = < – 0F =
2 0 2 16 32

1 dx
7. Evaluate: y0 . [CBSE (F) 2009]
2x + 3

1 dx 1
Sol. Let I = y0 = y0 (2x + 3) –1/2 dx
2x + 3
RS –1/2 + 1 V
WW1 RSS (2x + 3) 1/2 VWW1
SS (2x + 3) WW = SS WW = 51/2 –31/2 = 5 – 3
=S
SS c – 2 + 1 m # 2 WW SS 2 # 2 WWW
SS 1 WW SS 1 W
T X0 T X0
Short Answer Questions-II [3 marks]
2x
1. Evaluate: y dx [CBSE Delhi 2011]
(x 1)(x 2 + 3)
+ 2

Sol. Let x2 = z ⇒ 2x dx = dz
2x dz
∴ y dx = y
(x 2 + 1)(x 2 + 3) (z + 1)(z + 3)

Using partial fraction.


1 A B
Let = + …(i)
(z + 1)(z + 3) z + 1 z + 3
1 A(z + 3)+ B(z + 1)
=
+
(z 1)(z 3)+ (z + 1)(z + 3)

1 = A(z + 3) + B(z + 1) ⇒ 1 = (A + B)z + (3A + B)
Equating the coefficient of z and constant, we get
A + B = 0 …(ii)
and 3A + B = 1 …(iii)
Subtracting (ii) from (iii), we get
1 1
2A = 1 & A = 2  and B = – 2
Putting the values of A and B in (i), we get
1 1 1
  = –
(z + 1) (z + 3) 2 (z + 1) 2 (z + 3)
2x dx 1 1 1 dz 1 dz

y = yd – n dz = y – y
2 2
(x + 1) (x + 3) 2 (z + 1) 2 (z + 3) 2 z+1 2 z+3
1 1 1 1
= log| z + 1 |– log | z + 3 |+ C = log | x 2 + 1 | – log | x 2 + 3 |+ C
2 2 2 2
1 x2 + 1 Note: log m + log n = log m . n
=
2
log 2 + C = G
x +3 and log m – log n = log m / n
x2 + 1
= log +C
x2 + 3

276 Xam idea Mathematics–XII

@Cbsebookshub - Join Us on Telegram


x2
2. Evaluate: y dx [NCERT Exemplar]
1 – x4 2 2
1 x 1 x
2 + – +
x 2 2 2 2
Sol. Let I=y dx = y dx [ a2 – b2 = (a + b) (a – b)]
1 – x4 (1 – x 2) (1 + x 2)
1 1 1
2
(1 + x 2) – (1 – x 2)
2 2
( 1 + x 2)
1y ]1 – xg2
=y dx = y dx – dx
(1 – x 2) (1 + x 2) ]1 – xg2 (1 + x 2) 2 ]1 – xg2 ]1 + xg2

1y 1 1 1 1 1 1+x 1
dx – y dx = . log + C1 – tan –1 x + C2
2 1– x 2 2 1+x 2 2 2 1 – x 2
1 +
1 x 1
= log – tan –1 x + C [ C = C1 + C2]
4 1– x 2
3. Evaluate: y sin x sin 2x sin 3x dx [CBSE (F) 2010, Delhi 2012, 2019 (65/5/3)]
Sol. Let I = y sin x sin 2x sin 3x dx
1y 1
= 2 sin x.sin 2x.sin 3xdx = y sin x.(2 sin 2x.sin 3x)dx
2 2
1y
= sin x.(cos x – cos 5x)dx [ 2sin A sin B = cos (A – B) – cos (A + B)]
2

2 sin x.cos 5xdx 6a 2 cos A sin B = sin (A + B) – sin (A–B)@


1 y 1 y
= 2 sin x.cos xdx –
2#2 2#2
1 1
= y sin 2xdx – y (sin 6x – sin 4x) dx
4 4
cos 2x cos 6x cos 4x
=– + – +C
8 24 16
sin6 x + cos6 x
4. Evaluate: y dx [CBSE Delhi 2014]
sin 2 x. cos 2 x
sin6 x + cos6 x (sin 2 x) 3 + (cos 2 x) 3
Sol. Let I=y dx  ⇒ I = y dx
sin 2 x. cos 2 x sin 2 x.cos 2 x
(sin 2 x + cos 2 x)(sin 4 x – sin 2 x. cos 2 x + cos 4 x)
⇒ I= y dx
sin 2 x. cos 2 x
sin 4 x – sin 2 x. cos 2 x + cos 4 x

I= y dx = y tan 2 xdx – y dx + y cot 2 xdx
sin 2 x. cos 2 x

I= y (sec 2 x – 1) dx – x + y (cosec 2 x – 1) dx

I= y sec 2 xdx + y cosec 2 xdx – x – x – x + C = tan x – cot x – 3x + C

sin(x – a)
5. Evaluate: y dx [CBSE Delhi 2013; (F) 2015]
sin(x + a)
sin (x – a)
Sol. Let I = y dx
sin (x + a)
Let x + a = t ⇒ x=t–a ⇒ dx = dt
sin (t – 2a) sin t. cos 2a – cos t. sin 2a

I=y dt = y dt
sin t sin t
= cos 2a y dt – y sin 2a. cot t dt = cos 2a.t – sin 2a. log | sin t |+ C
= cos 2a . (x + a) – sin 2a . log |sin (x + a)| + C
= x cos 2a + a cos 2a – (sin 2a) log |sin (x + a)|+ C

Integrals 277
@Cbsebookshub - Join Us on Telegram
ex
6. Evaluate: y dx [CBSE Delhi 2009, 2019 (65/5/1)]
5 – 4e x – e 2x
ex
Sol. Let I=y dx
5 – 4e x – e 2x
Put ex = t ⇒ ex dx = dt, we get
dt dt dt

I=y 2
=y 2
=y
5 – 4t – t – (t + 4t – 5) – (t + 2.t.2 + 2 2 – 9)
2

ex + 2
+ C = sin – 1 d n+ C
dt t+2
=y = sin – 1
3 2 – (t + 2) 2 3 3

sin 4x – 4
7. Evaluate: y e x c m dx [CBSE Delhi 2010]
1 – cos 4x
sin 4x – 4
Sol. Let I = y ex c m dx
1 – cos 4x
2 sin 2x.cos 2x –4
= y ex d n dx [ sin 2x = 2 sin x . cos x and cos 2x = 1 – 2 sin2 x]
2 sin 2 2x
= y e x (cot 2x – 2 cosec 2 2x) dx
Let f(x) = cot 2x ∴ f l (x) = – 2cosec2 2x

I = y e x (f (x) + f l (x)) dx
⇒ I = ex . f(x) + C = ex . cot 2x + C
[a y e x (f (x) + f l (x)) dx = e x f (x) + C]

x+2
8. Evaluate: y 2
dx  [CBSE (AI) 2014]
x + 5x + 6
x+2
Sol. Let I=y 2
dx
x + 5x + 6
Now, we can express as
d
x + 2 = A (x 2 + 5x + 6) + B
dx
⇒ x + 2 = A(2x + 5) + B ⇒ x + 2 = 2Ax + (5A + B)
Equating coefficients both sides, we get
1 5 1
2A = 1, 5A + B = 2 ⇒ A= ,B = 2 – = –
2 2 2
1 1

x+2 =
(2x + 5) –
2 2
1 1
(2x + 5) – 2x + 5 dx
Hence, I = y 2 2 2 dx = 1 y 1
dx – y
x + 5x + 6 2 2
x + 5x + 6 2 2
x + 5x + 6
1 1
I=
. I – I … (i)
2 1 2 2
2x + 5 dx
where, I1 = y 2
dx, I2 = y 2
x + 5x + 6 x + 5x + 6
2x + 5
Now, I1 = y dx
x 2 + 5x + 6
Let x2 + 5x + 6 = z ⇒ (2x + 5)dx = dz

278 Xam idea Mathematics–XII

@Cbsebookshub - Join Us on Telegram


1
+
dz –1 z– 2 1
∴ I1 = y
= y z 2 dz = + C1 = 2 z + C1 = 2 x 2 + 5x + 6 + C1
z –1
+1
2
dx dx
Again I2 = y 2
=y
x + 5x + 6 5 5 2 25
x2 + 2 # x # + c m – +6
2 2 4
dx dx
=y 2
=y
5 1 5 2 1 2
cx + m – cx + m – c m
2 4 2 2
5
= log | c x + m + x 2 + 5x + 6 |+ C2
2
Putting the value of I1 and I2 in (i), we get
1 1 5
{2 x 2 + 5x + 6 + C1} – ' log|c x + m + x 2 + 5x + 6 |+ C2 1
I=
2 2 2
1 5 1 1
= x 2 + 5x + 6 – log | c x + m + x 2 + 5x + 6 |+ C1 – C2
2 2 2 2
1 5 1 1
= x + 5x + 6 – log | c x + m + x + 5x + 6 |+ C
2 2
[Here, C = C1 – C2]
2 2 2 2
(x 2 – 3x)
9. Evaluate: y dx [CBSE (F) 2010]
(x – 1)(x – 2)
(x 2 – 3x) (x 2 – 3x)
Sol. Let I=y dx = y 2 dx
(x – 1)(x – 2) x – 3x + 2
x 2 – 3x + 2 – 2 2dx
=y 2
dx = y dx – y 2
x – 3x + 2 x – 3x + 2
dx dx
= x – 2y = x – 2y
3 9 9 3 2 1 2
x 2 – 2. x. + – + 2 cx – m – c m
2 4 4 2 2
3 1
x– – dx 1 x–a
<a y 2 F
= x – 2 log 2 2 +C = log
3 1 x –a 2 2 a x+a
x– +
2 2
x–2
= x – 2 log +C
x–1
x
10. Find: y sin –1 dx [NCERT Exemplar]
a+x
x
Sol. Let I = y sin –1 dx
a+x
Put x = a tan2θ ⇒ dx = 2a tanθ sec2 θ d θ
a tan 2 i
I = y sin –1 e o (2a tan i. sec 2 i) di = 2a y sin –1 d
tan i
∴ n tan i. sec 2 idi
2
a + a tan i sec i
= 2a y sin –1 (sin i) tan i. sec 2 idi = 2a y i. tan i sec 2 idi
I II

= 2a <i. y tan i. sec idi – y d i. tan i. sec 2 i di n diF


d y 2
di

= 2a <i. tan diF


2
i tan 2 i
–y
2 2

Integrals 279
@Cbsebookshub - Join Us on Telegram
= ai tan 2 i – a y (sec 2 i– 1) di = ai. tan 2 i – a tan i + ai + C

= a : x tan –1 x x
+ –1
D C
x +
a a – a tan a

dx
11. Find : y  [CBSE Delhi 2012]
sin x + sin 2x
1
Sol. Here, I = y dx
sin x + sin 2x
1 1
⇒ I=y dx ⇒ I=y dx
sin x + 2 sin x cos x sin x (1 + 2 cos x)

sin x sin x
⇒ I=y 2
dx ⇒ I=y 2
dx
sin x (1 + 2 cos x) (1– cos x) (1 + 2 cos x)
Let cos x = z ⇒ - sin x dx = dz
–dz dz

I=y ⇒ I = –y
(1–z 2) (1 + 2z) (1 + z) (1–z) (1 + 2z)

Here, integrand is proper rational function. Therefore, by the form of partial fraction, we can write
1 A B C ...(i)
= + +
(1 + z) (1–z) (1 + 2z) 1 + z 1–z 1 + 2z
1 A (1 – z) (1 + 2z) + B (1 + z) (1 + 2z) + C (1 + z) (1–z)

=
+ +
(1 z) (1–z) (1 2z) (1 + z) (1 – z) (1 + 2z)
⇒ 1 = A(1 – z)(1 + 2z) + B(1 + z)(1 + 2z) + C (1 + z)(1 – z) ....(ii)
Putting the value of z = –1 in (ii), we get

1= –2A + 0 + 0 ⇒ A = –1/2
Again, putting the value of z = 1 in (ii), we get
1

1= 0 + B.2.(1 + 2) + 0 ⇒ 1 = 6B ⇒ B=
1 6
Similarly, putting the value of z = – in (ii), we get
2

1 = 0 + 0 + C c mc m ⇒
1 3 3 4
⇒ 1 = C ⇒ C=
2 2 4 3
Putting the value of A, B, C in (i), we get
1 –1 1 4
= + +
(1 + z) (1 – z) (1 – 2z) 2 (1 + z) 6 (1 – z) 3 (1 + 2z)

F dz = y = F dz
1 1 4 1 1 4

I = – y =–
+
+ +
+
– –
2 (1 z) 6 (1 – z) 3 (1 2z) 2 (1 z) 6 (1 – z) 3 (1 + 2z)
+
1 1 4
⇒ I = log | 1 + z |+ log | 1– z | – log | 1 + 2z |+ C
2 6 3×2
Putting the value of z, we get
1 1 2
⇒ I = log | 1 + cos x |+ log | 1– cos x | – log | 1 + 2 cos x |+ C
2 6 3
x2
12. Find: y dx [NCERT Exemplar]
x 4 – x 2 – 12
x2 x2
Sol. Let I=y 4 dx = y dx
x – x 2 – 12 x 4 – 4x 2 + 3x 2 – 12
x 2 dx
= y 2 2
x (x – 4) + 3 (x 2 – 4)

280 Xam idea Mathematics–XII

@Cbsebookshub - Join Us on Telegram


x2
= y dx
(x – 4) (x 2 + 3) 2

t A B

= + [let x2 = t] ⇒ t = A (t + 3) + B (t – 4)
( t – 4) ( t + 3) t – 4 t + 3
On comparing the coefficient of t on both sides, we get

A + B = 1

3A – 4B = 0 ⇒ 3(1 – B) – 4B = 0
3

3 – 3B – 4B = 0 ⇒ 7B = 3 ⇒ B=
7
3 3 3 4
If B =
, then A + = 1 ⇒ A = 1– =
7 7 7 7
x2 4 3
Now, 2 2
= 2
+ 2
( x – 4 ) ( x 3 ) 7 ( x – 4 ) 7 ( x + 3)
+
4y 1 3 1
∴ = dx + y 2 dx
7 x 2 – (2) 2 7 x + ( 3) 2
4 1 x–2 3 1 x
= . log + + . tan –1 +C
7 2.2 x 2 7 3 3
1 x–2 3 x
= log + + tan –1 +C
7 x 2 7 3
x2
13. Evaluate : y dx [CBSE Delhi 2013; (F) 2015]
(x + 4)(x 2 + 9)
2

x2
Sol. Let I=y dx
(x 2 + 4)(x 2 + 9)
Put x2 = t, we get
x2 t
∴ =
(x + 4)(x + 9) (t 4)(t + 9)
2 2 +
t A B A(t + 9) + B(t + 4)
Now, = + =
(t + 4)(t + 9) t + 4 t + 9 (t + 4)(t + 9)

⇒ t = (A + B)t + (9A + 4B)



Equating the coefficients, we get
A + B = 1, 9A + 4B = 0
Solving above two equations, we get
4 9
A = – ,B =
5 5
x2 4 9

2 2
=– 2
+ 2
(x + 4)(x + 9) 5 (x + 4 ) 5 ( x + 9)
x 2 dx 4 dx 9 dx 4 1 x 9 1 x

2
y
2
=– y 2 + y 2 = – # tan – 1 + # tan –1 + C
(x + 4) (x + 9) 5 x +2 2 5 x +3 2 5 2 2 5 3 3
2 x 3 x
= – tan –1 + tan –1 + C
5 2 5 3

Integrals 281
@Cbsebookshub - Join Us on Telegram
(3 sin i – 2)cos i
14. Find: y di [CBSE Delhi 2016]
5 – cos 2 i – 4 sin i
Sol. We have
(3 sin i – 2)cosi
I=y di
5 – cos 2 i – 4 sin i
Let sin q = z ⇒ cos q dq = dz
(3z – 2)dz

I= y
5 – (1 – z 2) – 4z
(3z – 2)dz (3z – 2) 3z – 2 3z dz
=y 2
=y 2
dz = y 2
dz = y 2
dz – 2 y
5 – 1 + z – 4z 4 – 4z + z (z – 2) ( z – 2) (z – 2) 2
Let z – 2 = t ⇒ dz = dt
+
3 (t + 2) dt dt t.dt dt dt dt dt t –2 1
=y – 2y = 3y + 6y – 2y = 3y + 4 y 2 = 3 log | t |+4 +C
t2 t2 t2 t2 t2 t t –2 + 1
1
= 3 log | t | – 4 . +C
t
Putting value of t in terms of z then z in terms of q, we get
4
= 3 log| sin i – 2 | – +C
sin i – 2
x
15. Find: y dx [CBSE Delhi 2016]
a3 – x3
x x1/2 dx x1/2 dx
Sol. We have I=y dx = y =y
a3 – x3 a3 – x3 (a 3/2) 2 – (x 3/2) 2
3 1/2 2
Let x 3/2 = t & 2
x dx = dt & x1/2 dx =
3
dt
2 dt
I= y [a x 3 2
= t & x 3 = t 2]
3 (a ) – t 2
3/2 2

x 3/ 2
sin –1 d 3/2 n + C = sin –1 e 3/2 o + C
2 t 2
=
3 a 3 a
2 x3
= sin –1 +C
3 a3
(2x – 5) e 2x
16. Find: y dx [CBSE (North) 2016]
(2x – 3) 3
Sol. We have,
(2x – 5) e 2x (2x – 3) – 2
y 3
dx = y e 2x = G dx
(2x – 3) (2x – 3) 3
1 2 1 2
= y e 3 .e 2x –3 = 2
– 3G
dx = e 3 y e 2x –3 = 2
– G dx
(2x – 3) (2x – 3) (2x – 3) (2x – 3) 3
dt
Let 2x – 3 = t ⇒ 2dx = dt & dx =
2
e3 t 1 2 e3 t 1

I= y e < 2 – 3 F dt ⇒ I= e . 2 +C
2 t t 2 t
Putting t = 2x – 3

282 Xam idea Mathematics–XII

@Cbsebookshub - Join Us on Telegram


e 3 2x –3 1 e 2x
I= e +C ⇒ I= +C
2 (2x – 3) 2 2 (2x – 3) 2

17. Find : y (2x + 5) 10 – 4x – 3x 2 dx [CBSE (F) 2016]

Sol. Let, I = y (2x + 5) 10 – 4x – 3x 2 dx


d
Let (2x + 5) = A (10 – 4x – 3x 2) + B
dx
⇒ 2x + 5 = A(– 4 – 6x) + B
⇒ 2x + 5 = – 4A – 6Ax + B
Equating, we get
– 4A + B = 5 ...(i) and – 6A = 2 ...(ii)
1
(ii) & A = –
3
4 4 11
Now, from (i) + B = 5 ⇒ B = 5 – =
3 3 3
1 11
∴ 2x + 5 = – (– 4 – 6x) +
3 3
1 11
Now, I = y ' – (– 4 – 6x) + 1 10 – 4x – 3x 2 dx
3 3
1 11
= – y (– 4 – 6x) 10 – 4x – 3x 2 dx + y 10 – 4x – 3x 2 dx
3 3
1 11
I = – I1 + I ...(iii)
3 3 2
where I1 = y (– 4 – 6x) 10 – 4x – 3x 2 dx and I2 = y (10 – 4x – 3x 2) dx
Now, I1 = y (– 4 – 6x) 10 – 4x – 3x 2 dx
Let 10 – 4x – 3x2 = z   ⇒ (– 4 – 6x) dx = dz
1
+1
z2

I1 = y z dz = + C1
1
+1
2
2
(10 – 4x – 3x 2) 3/2 + C1 ...(iv)
=
3
4 10
Again I2 = y 10 – 4x – 3x 2 dx = y – 3 c x 2 + x – m dx
3 3
2 2 34
– (c x + 2 dx
2 4 4 10
= 3y – ' x 2 + 2x. + – – 1 dx = 3 y m –
3 9 9 3 3 9
2
2 2 2 2
d
n – c x + m dx
34 34
= 3y – c x + m dx = 3 y
9 3 3 3
JK 2N
3 2 3 34 KK x + OOO
= c x + m 10 – 4x – 3x 2 + # sin –1 KK 3 O + C ...(v)
2 3 2 9 KK 34 OOO 2
K O
L 3 P
Putting the value of I1 and I2 in (iii), we get
2 11 2 187 3 3 2
I=
– (10 – 4x – 3x 2) 3/2 + c x + m 10 – 4x – 3x 2 + sin –1 d (x + ) n + C
9 2 3 3 27 34 3

Integrals 283
@Cbsebookshub - Join Us on Telegram
1
18. Evaluate: y0 x log (1 + 2x) dx [NCERT Exemplar]
I
1 II
Sol. Let I = y0 x log (1 + 2x) dx
1
x2 x2
= <log (1 + 2x) F – y
x2 1 1 1
.2. dx = [x 2 log (1 + 2x)] 10 – y dx
2 0 1 + 2x 2 2 +
0 1 2x
RS J x NO VWW
1 SS 1 KK O W 1 1 1 1 1 x
= [1 log 3 – 0] – SS y0 KK x 2 OO dxW = log 3 – y0 x dx + y0 dx
2 SS KK 2 – 1 + 2x OO WWW 2 2 2 1 + 2x
T L P X
1
1 +
(2x 1 – 1)
= log 3 – < F + y0 dx = log 3 – ; – 0E + y0 dx – y0
1 1 x2 1 1 2 1 1 1 1 1 1 1 1
dx
2 2 2 0 2 (2x + 1) 2 2 2 4 4 1 + 2x
1 1 1 1 1 1 1 1
= log 3 – + [x] 10 – [log (1 + 2x ) ] 10 = log 3 – + – [log 3 – log 1]
2 4 4 8 2 4 4 8
1 1 3
= log 3 – log 3 = log 3
2 8 8
r
4x sin x
19. Evaluate: y dx [CBSE (AI) 2014]
0 1 + cos 2 x
r 4x sin x
Sol. Let I= y dx …(i)
0 1 + cos 2 x
r 4 (r – x) . sin (r – x)
= y dx
0 1 + cos 2 (r – x)
r 4 (r – x) . sin x
I= y dx …(ii)
0 1 + cos 2 x
Adding (i) and (ii), we get
4 (x + r – x) sin x
r r
r sin x
2I = y dx ⇒ 2I = 4 y dx
0 1 + cos 2 x 0 1 + cos 2 x
r sin x
I = 2r y 2
dx
0 1 + cos x
Let cos x = z ⇒ – sin x dx = dz ⇒ sin x dx = – dz
The limits are, x = 0 ⇒ z=1
x = p ⇒ z = – 1
–1 – dz

I = 2r y = 2r [tan –1 z] 1–1
1 1 + z2
= 2r [tan –1 1 – tan –1 (–1)] = 2r : + D = 2r #
r r r

4 4 2
⇒ I = p2.

r
20. Evaluate: y (cos ax – sin bx) 2 dx [CBSE Delhi 2015]
–r
r
Sol. Here, I = y–r (cos ax– sin bx) 2 dx
r

I = y–r (cos 2 ax + sin 2 bx – 2 cos ax sin bx) dx
r r r

I = y–r cos 2 ax dx + y–r sin 2 bx dx – y–r 2 cos ax sin bx dx
r r
⇒ I = 2 y0 cos 2 ax dx + 2 y0 sin 2 bx dx – 0 [First two integrands are even function while third
is odd function.]

284 Xam idea Mathematics–XII

@Cbsebookshub - Join Us on Telegram


r r r r

I = y0 2 cos 2 ax dx + y0 2 sin 2 bx dx ⇒ I = y0 (1 + cos 2ax) dx + y0 (1 – cos 2bx) dx
r r r r r r r

I = y0 dx + y0 cos 2ax dx + y0 dx – y0 cos 2bx dx ⇒ I = 2 y0 dx + y0 cos 2 ax dx – y0 cos 2 bx dx
sin 2 ax r sin 2 bx r

I = 2 [x] r0 + ; E –< F
2a 0 2b 0

sin 2 ar sin 2br


⇒ I = 2r + –
2a 2b
r
x sin x
21. Evaluate: y 2
dx [CBSE Delhi 2017; (AI) 2012, CBSE 2020 (65/4/1)]
0 1 + cos x
r x sin x r (r – x) sin (r – x) dx
Sol. Let I = y0 dx I = y0
1 + cos x 2
1 + cos 2 (r – x)
(r – x) sin x dx sin x dx
= y 0r 2
= r y 0r –I
1 + cos x 1 + cos 2 x
sin x dx r r r sin x dx
or 2I = r y 0 or I = y0
1 + cos 2 x 2 1 + cos 2 x
Put cos x = t so that – sin x dx = dt.
The limits are, when x = 0, t = 1 and x = p, t = – 1, we get

8a ya f (x) dx = – y–a f (x) dx and y0 f (x) dx = 2 y0 f (x) dxB


–r – 1 dt 1 dt –a a 2a a
I= y = r y0
2 1 1 + t2 1 + t2
r2
= r [tan –1 t] 10 = r [tan –1 1 – tan –1 0] = r : – 0D =
r

4 4
r /4
dx
22. Find: y 3
[CBSE (Allahabad) 2015]
0 cos x 2 sin 2x
r/ 4 r/ 4
dx dx
Sol. Let I = y 3
= y
3
0 cos x 2 sin 2 x 0 cos x 2 . 2 sin x. cos x
1 r/ 4 dx 1 r/ 4 dx
= y0 = y0
2 sin x 2 4
cos x tan x
cos 3 x cos x . cos 2 x

1 y r/4 sec 4 x dx 1 y r/4 sec 2 x. sec 2 x dx


= = 0
2 0 tan x 2 tan x
r
Let tan x = t ⇒ sec2 x dx = dt,
x=0 ⇒ t = 0 and x = ⇒ t=1
4
2 1 1
1 1 (1 + t ) dt –1/2 + 1
1 t 3/ 2 + 1
= y0 (t –1/2 + t 3/2) dt = = t G + = G
1 1 1

I = y0
2 t 2 2 –1/2 + 1 0 2 3/2 + 1 0


= 1 × 2 6 t @1 + 1 × 2 6t 5/2@1 = 1 + 1 = 6
2 1 0 2 5 0 5 5
r/2
23. Evaluate: cos x [CBSE (F) 2015]
y x dx
–r/2 1 + e
r/2 cos x
Sol. Let I = y–r/2 dx In 1st Integrand
1 + ex
0 cos x r/2 cos x
Let x = –t
= y–r/2 y
x dx + 0 dx dx = –dt
1+e 1 + ex
0 cos t r/2 cos x x = –r/2 & t = r/2
= yr/2 (–dt) + y0 dx
1+e –t 1 + ex x=0 &t=0

Integrals 285
@Cbsebookshub - Join Us on Telegram
t
cos t
r/ 2 r/2 cos x r/2 e . cos t r/2 cos x
= y0 dt + y0 dx = y dt + y0 dx
1 1 + ex 0
1 + e t 1 + ex
1+ t
e
x
r/2 e . cos x r/2 cos x b b
= y0 dx + y0 dx [By property ya f (x) dx = ya f (t) dt]
1 + ex 1 + ex
x
r/2 (e + 1) . cos x r/ 2 r
= y0 x dx = y0 cos x dx = [sin x] 02 = sin π/2 – sin 0
1+e
= 1.

dx
r/3
24. Evaluate: yr/6 [CBSE (AI) 2011]
1 + tanx
r/ 3 dx
Sol. Let I = yr/6
1 + tan x
dx b b
= yrr//63 [By using property ya f (x) dx = ya f (a + b – x) dx] ...(i)
1 + tan a + – x k
r r
6 3
r/ 3 dx r/ 3 dx
= yr/6 = yr/6
1 + tan a – x k
r 1
1+
2 tanx
r/ 3 tan x
= yr/6 dx ...(ii)
1 + tan x
Adding (i) and (ii), we get
(1 + tan x ) r/ 3
2I = yr/6
dx
(1 + tan x )
r/ 3 r r r
  = yr/6 dx = [x] rr//36 = – =
3 6 6
r r
2I = or I=
6 12
3
25. Evaluate: y [| x – 1 |+| x – 2 |+| x – 3 |]dx [CBSE Delhi 2013]
1
3 3 3 3
Sol. Let I = y [| x – 1 |+| x – 2 |+| x – 3 |]dx = y | x – 1 | dx + y | x – 2 | dx + y | x – 3 | dx
1 1 1 1
3 2 3 3
= y | x – 1 | dx + y | x – 2 | dx + y | x – 2 | dx + y | x – 3 | dx
1 1 2 1
[By property of definite integral]
3 2 3 3
= y (x – 1) dx + y – (x – 2) dx + y (x – 2) dx + y – (x – 3) dx
1 1 2 1
]Z] x – 1 $ 0, if 1 # x # 3 b_b
]] bb
]] x – 2 # 0, if 1 # x # 2 bb
[] `
]] x – 2 $ 0, if 2 # x # 3 bbb
]] b
] x– 3 # 0, if 1 # x # 3 bb
3 2 3 3 \ a
=< F –< F +< F –< F
(x – 1) 2 ( x – 2) 2 (x – 2) 2 ( x – 3) 2
2 1 2 1 2 2 2 1

= a – 0 k – a0 – k + a – 0 k – a – 0 – k = 2 + + + 2 = 5
4 1 1 4 1 1
2 2 2 2 2 2

286 Xam idea Mathematics–XII

@Cbsebookshub - Join Us on Telegram


r x tan x
26. Evaluate: y0 dx [CBSE Delhi 2008, 2014; Chennai 2015]
sec x .cosec x
sin x
x tan x x.
Sol. Let I= y0r dx = y0
r cos x dx
sec x.cosec x 1 1
.
cos x sin x
a a
I = y0 x sin 2 xdx = y0 (r – x) sin 2 (r – x) dx
r r
[a y0 f (x) dx = y0 f (a – x) dx]
r r r r
I = y0 r sin 2 xdx – y0 x sin 2 x dx & 2I = y0 2 sin 2 x dx
2
r sin 2x r
= y0 (1 – cos 2x) dx = y0 dx – y0 cos 2xdx = [x] r0 – ; E
r r r r r r r

2 2 2 2 2 2 0
r r
& 2I = (r – 0) – (sin 2r – sin 0)
2 4
2
r r2
& 2I =
2
–0 & I=
4
dx
27. Evaluate y .[CBSE 2019(65/4/2)]
sin (x – a) cos (x – b)
dx 1 cos (a – b) dx
Sol. Let   I = y = y
sin (x – a) cos (x – b) cos (a – b) sin (x – a) cos (x – b)

1 cos (a – x + x – b) 1 cos ((x – b) – (x – a))


= y dx = y dx
cos (a – b) sin (x – a) cos (x – b) cos (a – b) sin (x – a) cos (x – b)
1 cos (x – b) cos (x – a) + sin (x – b) sin (x – a)
= y dx
cos (a – b) sin (x – a) cos (x – b)
cos (x – a) sin (x – b) cos (x – a) sin (x – b)
=
1 y) + 3 dx =
1
>y dx + y dxH
cos (a – b) sin (x – a) cos (x – b) cos (a – b) sin (x – a) cos (x – b)

7log| sin (x – a) |– log| cos (x – b) |A + C


1
=
cos (a – b)
1 sin (x – a)
= log +C
cos (a – b) cos (x – b)
1
28. Evaluate: y0 cot –1 (1 – x + x 2)dx [CBSE Delhi 2008; (AI) 2008; (South) 2016]
1
Sol. Let I = y cot –1 (1 – x + x 2 ) dx
0
1 1
;a cot –1 x = tan –1 E
1
= y0 tan –1 dx
1 – x + x2 x
1 x + ( 1 – x)
= y0 tan –1 dx
1 – x ( 1 – x)
x+y
1
  = y0 {tan –1 x + tan –1 (1 – x)} dx <a tan –1 (x + y) = tan –1 F
1 – xy
1 1

= y0 tan –1 x dx + y0 tan –1 (1 – x) dx
= y0 tan –1 xdx + y0 tan –1 (1 – (1 – x)) dx 7a y0 f (x) dx = y0 f (a – x) dxA
1 1 a a

1 1 1 1
= y0 tan –1 xdx + y0 tan –1 xdx = 2 y0 tan –1 xdx = 2 y0 1.tan –1 x dx
1 1 2x dx
= 2 ([tan –1 x.x] 10 – y0
. x.dx 2 = 2 – y0
r 1 r
= – [log | 1 + x 2 |] 10
1+x 2 4 1+x 2 2
r r
  = – [log 2 – log 1] = – log 2
2 2

Integrals 287
@Cbsebookshub - Join Us on Telegram
1
29. Evaluate: y0 x 2 (1 – x) n dx [CBSE (F) 2010, 2019 (65/4/3)]
1
Sol. Let I = y0 x 2 (1 – x) n dx
7a y0 f (x) dx = y0 f (a – x) dxA
1 a a
⇒ I = y0 (1 – x) 2 [1 – (1 – x)] n dx
1 1
= y0 (1 – 2x + x 2) x n dx = y0 (x n – 2x n + 1 + x n + 2) dx
+ + + 1
xn 1 xn 2 xn 3
== G =
1 2 1
– 2. + – +
n+1 n+2 n+3 0 n+1 n+2 n+3

(n + 2)(n + 3) – 2(n + 1)(n + 3) + (n + 1)(n + 2)


=
(n + 1)(n + 2)(n + 3)
n + 5n + 6 – 2n 2 – 8n – 6 + n 2 + 3n + 2
2
2
= =
(n + 1)(n + 2)(n + 3) (n + 1)(n + 2)(n + 3)
3
30. Evaluate: y1 (2x 2 + 5x)dx as a limit of a sum. [CBSE Delhi 2012]
Sol. Let f(x) = 2x2 + 5x
b–a 3–1 2
Here a = 1, b = 3 ∴ h= = = ⇒ nh = 2
n n n
Also, n → ∞ ⇔ h → 0.

a
b
ya f (x) dx = lim h [f (a) + f (a + h) + ... + f {a + (n – 1) h}]
h"0
3
` y1 (2x 2 + 5x) dx = lim h [f (1) + f (1 + h) + ... + f {1 + (n – 1) h}]
h"0

= lim h [{2 # 1 2 + 5 # 1} + {2 (1 + h) 2 + 5 (1 + h)} + ... + {2 (1 + (n – 1) h) 2 + 5 ((1 + (n – 1) h}]


h"0
= lim h [(2 + 5) + {2 + 4h + 2h 2 + 5 + 5h} + ... + {2 + 4 (n – 1) h + 2 (n – 1) 2 h 2 + 5 + 5 (n – 1) h}]
h"0

= lim h [7 + {7 + 9h + 2h 2} + ... + {7 + 9 (n – 1) h + 2 (n – 1) 2 h 2}]


h"0
= lim h [7n + 9h {1 + 2 + ... + (n – 1)} + 2h 2 {1 2 + 2 2 + ... + (n – 1) 2}]
h"0

(n – 1) .n (n – 1) .n(2n – 1)
= lim ;7nh + 9h 2 + 2h 3 E
h"0 2 6
1 1 1
9 (nh) 2 . c1 – m 2 (nh) 3 .c1 – m .c 2 – m
>
= lim 7 (nh) + n + n n H
h"0 2 6
1 1 1
36 c 1 – m 16 c 1 – m .c 2 – m
= lim >14 + n + n n H
[ nh = 2]
n"3 2 6
1 8 1 1
= lim ;14 + 18 c1 – m + c1 – m .c 2 – mE
n"3 n 3 n n
8 16 96 + 16 112
= 14 + 18 + ×1×2 = 32 + = =
3 3 3 3
2
31. Evaluate : y | x 3 – x | dx [CBSE Delhi 2016; (AI)2012], [CBSE 2019 (65/5/3)]
–1
Sol. If x3 – x = 0

x(x2 – 1) = 0 ⇒ x = 0 or x2 = 1

x = 0 or x = ±1 ⇒ x = 0, –1, 1

288 Xam idea Mathematics–XII

@Cbsebookshub - Join Us on Telegram


Hence [–1, 2] is divided into three sub intervals [–1, 0], [0, 1] and [1, 2] such that
x3 – x ≥ 0 on [–1, 0]
x3 – x ≤ 0 on [0, 1]
3
and x – x ≥ 0 on [1, 2]
2 0 1 2
Now, y | x 3 – x | dx = y | x 3 – x | dx + y | x 3 – x | dx + y | x 3 – x | dx
–1 –1 0 1
0 1 2
x4 x2 x4 x2 x4 x2
   = y (x 3 – x) dx + y – (x 3 – x) dx + y (x 3 – x) dx = < – F – < – F +< – F
0 1 2

–1 0 1 4 2 –1 4 2 0 4 2 1
1 1 1 1 1 1
   = ' 0 – c – m1 – 'c – m – 0 1 + '(4 – 2) – c – m1
4 2 4 2 4 2
1 1 1 1 1 1 3 3 11
   = – + – + +2– + = – +2 =
4 2 4 2 4 2 2 4 4

32. Evaluate y e 2x . sina + x k dx . [CBSE Delhi 2016]


r r

0 4

Sol. We have I = y e 2x . sin a + x k dx


r r
0 4
Integrating by part, we get
r
e 2x e 2x
I = <sin a + x k. F – y0r cos a + x k .
r r
dx
4 2 0 4 2

.e – sin E – y0 e 2x . cos a + x k dx
1 5r 2 r 1 r
= ;sin
r r
2 4 4 2 4
r
o – =( cos a + x k . dxG
e 2r e 2x e 2x
e– 2 + y0r sin a + x k .
1 1 1 r r
= –
2 2 2 2 4 2 0 4 2
e 2r + 1 1 5r e 2r 1
– <cos – cos F – y e 2x .sin a + x k dx
r 1 r
=– .
2 2 2 4 2 2 4 4 4
e 2r + 1 1 2r 5r 1 1
I=– – .e . cos + – I
2 2 4 4 4 2 4
5I e 2r + 1 e 2r 1 e 2r + 1 e 2r + 1 e 2r + 1 e 2r + 1
=– + + =– + = ( – 2 + 1) = –
4 2 2 4 2 4 2 2 2 4 2 4 2 4 2
e 2r + 1
I=–
5 2
2 x2
33. Evaluate : y dx [CBSE (North) 2016]
–2 1 + 5x
2 x2
Sol. Let I= y dx ...(i)
–2 1 + 5x
2 (2 + (–2) – x) 2 b
= y
(2 + (–2) –x)
dx = y f (x) dx = y f (a + b – x) dxG
–2 1+5 a
2 2 2 2
(–x) x
= y dx = y dx
–2 1 + 5 –x –2 1
1+
5x

Integrals 289
@Cbsebookshub - Join Us on Telegram
5x x2 2
I= y x
dx …(ii)
–2 1 + 5
Adding (i) and (ii), we get
2
( 1 + 5 x) x 2 x3
dx = y x 2 dx = < F
2 2
2I = y
–2 1+5 x
–2 3 –2
1 16 8

2I = [8 – (– 8)] ⇒ I= =
3 3#2 3
1
34. Find : y [log (log x) + ] dx [CBSE Bhubaneswar 2015, (South) 2016]
(log x) 2
1
Sol. Let I = y [log (log x) + ]dx
(log x) 2
Let log x = t ⇒ x = et ⇒ dx = et dt
1
∴ I = y ( log t +
2 et dt
t2
1 1 1 1 1 1
   = y ( log t + – + 2 2 et dt = y c log t + m et + d – + 2 n et dt
t t t t t t
1
   = et . log t – .et + C [a y (f (x) + f l (x)) e x dx = f (x) e x + C]
t
1
= e log x log (log x) – e log x + C [Put t = log x]
log x
x
= x. log (log x) – +C
log x
5x x
35. Find: y 55 . 55 .5 x dx [HOTS]

5x x
Sol. Let I = y 5 5 . 5 5 .5 x dx
dt
Putting 5 x = t & 5 x . log 5 dx = dt or 5 x . dx =
(log 5)
5x x t dt 1 t
Therefore, I = y 5 5 . 5 5 . 5 x . dx = y 5 5 . 5t . = y 5 5 . 5t . dt
(log 5) (log 5)
du
Again, putting 5t = u, 5t dt =
(log 5)
1 du 1 5u
Therefore, I= y 5u . = y 5u du = +C
(log 5) (log 5) (log 5) 2
(log 5) 2 . (log 5)
t 5x
5u 55 55
= + C = + C = +C
(log 5) 3 (log 5) 3 (log 5) 3

Long Answer Questions [5 marks]


4
1. Evaluate the following integral as the limit of sums: y1 (x 2 –x) dx [CBSE 2020 65/5/1]
4
Sol. Let I = y1 (x 2 – x) dx


f (x) = x 2 – x, a = 1, b = 4
b–a 4–1 3

 h= n
=
n
=
n & nh = 3

290 Xam idea Mathematics–XII

@Cbsebookshub - Join Us on Telegram


As n " 3, h " 0
We know that
lim
h 7 f (a) + f (a + h) + ... + f (a + n–1h)A
b
(ya f x) dx =
h"0
b n–1

ya f (x) dx = nlim
" 3h
/ f (a + rh) ...(i)
r=0


f (a + rh) = (a + rh) 2 – (a + rh)


f (1 + rh) = (1 + rh) 2 – (1 + rh) = r 2 h 2 + rh
Using (i), we have
4 n–1
y1 (x 2 – x) dx = nlim
" 3h
/ (r 2 h 2 + rh)
r

h (h / r + h / r2
n–1 n–1
2 2

I= lim
h"0 r=0 r=0

n (n–1) (2n–1) n (n–1)


= lim h ( h 2 × + h× 2
h"0 6 2
n 2 h 2 b 1– n l
1 1 1
> n h (1– n ) (2– n ) + H
3 3
= lim
h"0
6 2
3 2
(3) (1–0) (2–0) (3) (1–0)
= +
6 2
9 27
= 9+ =
2 2
2. Evaluate: y ( cot x + tan x )dx [CBSE (AI) 2014; Patna 2015]

Sol. Let I = y ( cot x + tan x ) dx


(cos x + sin x)
I = ye o dx = y
cos x sin x
+ dx
sin x cos x sin x. cos x
Let (sin x – cos x) = t ⇒ (cos x + sin x) dx = dt
Also (sin x – cos x)2 = t2 ⇒ sin2 x + cos2 x – 2 sin x . cos x = t2
1 – t2

1 – 2 sin x . cos x = t2 ⇒ sin x.cos x =
2
dt dt
Therefore, I=y 2
= 2y
1–t 1 – t2
2
= 2 sin –1 t + C = 2 sin –1 (sin x – cos x) + C

1
3. Evaluate: y dx [CBSE (AI) 2014]
sin x + sin x cos 2 x + cos 4 x
4 2

1
Sol. Let I=y dx
sin 4 x + sin 2 x.cos 2 x + cos 4 x
Dividing Nr and Dr by cos4 x, we get
sec 4 x
I=y dx
tan x + tan 2 x + 1
4

Put z = tan x ⇒ dz = sec2 x dx

Integrals 291
@Cbsebookshub - Join Us on Telegram
1 1 1
z 2 d1 +
n d1 + 2 n d1 + 2 n
(1 + z 2) dz z2 z z

I=y 4 =y dz = y dz = y dz
z + z2 + 1 1 2
1 2
z (z + 2 + 12
1
cz – m + 3 c z – m + ( 3) 2
2 2
z z z
1 1
Again, let z – =t & d1 + n dz = dt
z z2 1
tan 1 f z p+ C
1 dt t 1 z– 1
⇒ I = y 2 = d tan –1 n+ C =
-
;a z – = tE
t + ( 3) 2 3 3 3 3 z

z2 – 1 tan 2 x – 1
tan –1 e o+ C = tan –1 e o+ C
1 1
=
3 3z 3 3 tan x

r /2 sin 2 x
4. Evaluate: y0 dx [CBSE Panchkula 2015; (South) 2016]
sin x + cos x
r/ 2 sin 2 x
Sol. Let I = y0 dx ..(i)
sin x + cos x
sin 2 a – x k
r
a a
2
<a y f (x) dx = y f (a – x) dxF
r/ 2
I = y0 dx
sin a – x k + cos a – x k
r r 0 0
2 2
r/ 2
cos 2 x
I= y dx ...(ii)
0 cos x + sin x

Adding (i) and (ii), we get

r/ 2
sin 2 x + cos 2 x r/ 2
dx
2I = y dx = y
0 +
sin x cos x 0 sin x + cos x
r/ 2
dx 1 r/ 2 dx
= y = y
1 1
2d sin x n
0 20 r r
cos x + cos x.cos + sin x.sin
2 2 4 4
1 r/ 2 1 r/ 2
y sec a x – k dx [ cos (A – B) = cos A cos B + sin A sin B]
dx r
= y =
cosa x – k
20 r 20 4
4

:log &sec a x – k + tan a x – k0D


1 r r r/ 2
= [a y sec xdx = log (sec x + tan x)]
2 4 4 0

:log asec + tan k – log &sec a – k + tan a – k0D


1 r r r r
=
2 4 4 4 4

:log ( 2 + 1) – log asec – tan kD


1 r r
=
2 4 4
2+1
log ) 3
1 1
= [log ( 2 + 1) – log ( 2 – 1)] =
2 2 2 –1
( 2 + 1) 2
log ( 2=
1 1 2
= log ( 2 + 1) 2 = log ( 2 + 1)
2 2–1 2 2
1
Hence, I = log ( 2 + 1)
2

292 Xam idea Mathematics–XII

@Cbsebookshub - Join Us on Telegram


r /2 x sin x cos x
5. Evaluate: y dx [CBSE Delhi 2011, 2014; Sample Paper 2017]
0 sin 4 x + cos 4 x
r/ 2
x sin x cos x
Sol. Let I= y dx
0 sin 4 x + cos 4 x

a
– x k . sin a – x k . cos a – x k
r r r
r/ 2
2 2 2 By Property

I= y dx > a H
sin a – x k + cos a – x k
0 4 r 4 r a
y0 f (x) dx = y0 f (a – x) dx
2 2

r/ 2 a – x k cos x. sin x
r
:a sin a – x k = cos x and cos a – x k = sin xD
2 r r

I= y dx
0
4
cos x + sin x 4 2 2

r r/2 cos x. sin x r/ 2


x sin x. cos x

I= y dx – y dx
2 0 sin 4 x + cos 4 x 0 sin 4 x + cos 4 x
r r/2 cos x.sin x r r/2 sin x.cos x

I= y dx – I ⇒ 2 I = y dx
2 0 sin 4 x + cos 4 x 2 0 sin 4 x + cos 4 x
sin x . cos x
r/ 2
dx
r y cos 4 x
= [Dividing numerator and denominator by cos4 x]
20 tan 4 x + 1
r r/2 2 tan x. sec 2 x
= y dx
2 # 2 0 1 + (tan 2 x) 2
Let tan2 x = z; 2tan x . sec2 x dx = dz
r
The limits are, when x = 0, z = 0; x = ,z = 3
2
r 3 dz r r

2I = y = [tan –1 z] 3 = (tan –1 3 – tan –1 0)
40 1+z 2 4 0 4
r2
a – 0k
r r

2I =
4 2
& I=
16
r /2
6. Evaluate: y 2 sin x cos x tan –1 (sin x) dx [CBSE Delhi 2011]
0
r/ 2
Sol. Let I = 2 y0 sin x.cos x.tan –1 (sin x)dx
Put sin x = z ⇒ cos x dx = dz
r r
The limits are, when x = 0, z = sin 0 = 0; x = , z = sin = 1
2 2
1
z2 z2
I = 2 y0 z tan –1 (z) dz = 2 <tan –1 z. F – 2 y0
1 1 1

. dz
2 0 1 + z2 2
r 1 2 1 z2 2
1 1+z –1 1 dz
= 2 ; . – 0E – y0
r r 1
dz = – y dz = – y0 dz + y0
4 2 2 1+z 2 4 0
1+z 2 4 1 + z2
= – [z] 10 + [tan –1 z] 10 = – 1 + : – 0D = – 1
r r r r
4 4 4 2
r x
7. Evaluate: y0 2 dx [CBSE (AI) 2009; (F) 2014]
a cos 2 x + b 2 sin 2 x
r x
Sol. Let I = y 2 2 2 2
dx …(i)
0 a cos x + b sin x

Integrals 293
@Cbsebookshub - Join Us on Telegram
r r–x a a
I= y dx [a y f (x) dx = y f (a – x) dx]
0 a cos (r – x) + b 2 sin 2 (r – x)
2 2
0 0
r r–x
I= y dx …(ii)
0 a cos x + b 2 sin 2 x
2 2

Adding (i) and (ii), we get


r r r r dx
2I = y 2 2 2 2
dx & I= y
2 0 a 2 cos 2 x + b 2 sin 2 x
0 a cos x + b sin x

r r sec 2 x
I= y 2 dx [Divide numerator and denominator by cos2 x]
2 0 a + b 2 tan 2 x
r
2 sec 2 x 2a a
=ry dx [a y f (x) dx = 2 y f (x) dx]
0 a + b 2 tan 2 x
2
0 0
2
Put b tan x = t ⇒ b sec x dx = dt
r
The limits are, when x = 0, t = 0 and x = ,t = 3
2
r 3 dt r 1 t 3
I= y 2 2 = . tan –1 E
b 0 a +t b a a 0
r r r r2
I=
ab
(tan –1 3 – tan –1 0) = .
ab 2
& I=
2ab

( a + x)
8. Solve: y dx [NCERT Exemplar]
(a – x)
a+x
Sol. Let I=y dx
a–x
Put x = a cos 2θ

dx = – a. sin2θ. 2. dθ
a + a cos 2i x x 1 x
∴ I = –2 y . a sin 2idi <a cos 2i = & 2i = cos –1 & i= cos –1 F
a – a cos 2i a a 2 a

1 + cos 2i 2 cos 2 i
= – 2a y sin 2idi = –2a y sin 2idi
1 – cos 2i 2 sin 2 i
cos i
= – 2a y cot i. sin 2idi = –2a y 2 sin i. cos idi = – 4a y cos 2 idi = –2a y (1 + cos 2i) di
sin i
x2
= – 2a ;i + E + C = – 2a > cos –1 a +
x 1
H+ C
sin 2i 1
1–
2 2 2 a2
x2
= – a >cos –1 b a l + H+C
x
1–
a2
3/2
9. Evaluate the following: y0 | x cos r x | dx [CBSE (F) 2010; Patna 2015; (Central) 2016]
3/2
Sol. y0 | x cos r x | dx
r
As we know, cos x = 0 ⇒ x = (2n – 1) , n ! Z
2
1 3
∴ cos px = 0
⇒ x= ,
2 2
1
For 0 1 x 1 , x 2 0 then cos px > 0 ⇒ x cos px > 0
2

294 Xam idea Mathematics–XII

@Cbsebookshub - Join Us on Telegram


1 3
For 1x 1 , x 2 0 then cos px < 0 ⇒ x cos px < 0
2 2
3/2 1/2 3/2

y0 | x cos rx | dx = y0 x cos r xdx + y1/2 (– x cos rx) dx …(i)

sin rx 1/2 1/2 sin rx x sin rx 3/2


= ;x E – y0 1. dx – ; E + y1/2
3/2 sin rx
dx
r 0 r r 1/ 2 r
1/2 3/2
x 1 x 1
= < sin rx + 2 cos rxF – < sin rx – 2 cos rxF
r r 0 r r 1/ 2
1 1 3 1 5 1
=d +0– 2n–c – – m= –
2r r 2r 2r 2r r 2
r x
10. Evaluate: y dx [CBSE (F) 2016]
0 1 + sin a sinx
r x r r–x
Sol. Let I= y dx = y dx
0 +
1 sin a sin x +
0 1 sin a.sin (r – x)
r r r x
= y dx – y dx
0 1 + sin a sin x 0 1 + sin a.sin x
r dx
I=ry –I
0 1 + sin a.sin x
r dx r dx

2I = r y =ry
0 1 + sin a.sin x 0 x
2 tan
1 + sin a. 2
x
1 + tan 2
2

a1 + tan 2 k
x x
r r sec 2
=ry 2 dx = r y 2 dx
x x x x
0
1 + tan 2 + 2 sin a.tan 0
tan 2
+ 2 sin a.tan + 1
2 2 2 2
x x
Let tan
2
=t & sec 2 dx = 2dt; x = 0
2
& t = 0 and x = r & t=3

3 dt

2I = 2r y 2
0 t + 2 sin at + 1
3 dt
I=ry
0 t 2 sin at sin 2 a – sin 2 a + 1
+ 2
+
3 dt 3 dt
=ry 2 2
= r y 2 2
0 (t + sin a) + (1 – sin a) 0 (t + sin a) + cos a
x 3
+ >tan –1 tan + sin a
H
3
;tan –1 E =
r t sin a r 2
=
cos a cos a 0 cos a cos a 0

: – tan –1 (tan a)D = a – ak


r r r r
=
cos a 2 cosa 2
r (r – 2a)
=
2 cos a

Integrals 295
@Cbsebookshub - Join Us on Telegram
1– x
11. Find: y dx  (HOTS)
1+ x
1– x
Sol. Let I = y dx
1+ x
Putting x = cos i, i.e., x = cos 2 i & i = cos –1 x and dx = – 2 cos i sinidi, we get
1 – cos i
I=y (– 2 sin icosi) di
1 + cos i
i i
2 sin 2 sin
= – 2y 2 (sin i cos i) di = – 2 y 2 c 2 sin i cos i cos i m di
i i 2 2
2 cos 2 cos
2 2
i
= – 2 y 2 sin 2 cos idi = – 2 y (1 – cos i)cosidi
2

= –2 y (1 – cos i) cos i . di = – 2 y (cos i – cos 2 i) . di
= –2 y cos i + y 2 cos 2 i. di = – 2 sin i + y (1 + cos 2i) .di
sin 2i
= – 2 sin i + y 1.di + y cos 2i. di = – 2 sin i + i + +C
2
2 1 – cos 2 i. cos i
= –2 1 – cos 2 i + i + + C = – 2 1 – x + cos –1 x + x 1– x + C
2
x2
12. Find: y dx [HOTS]
(x sin x + cos x) 2

x 2 dx x cos x x
Sol. Let I = y 2
dx = y .
2 cos x
dx
(x sin x + cos x) (x sin x + cos x)
x x cos x
Integrating by parts, taking as the first function and as the second function,
cosx (x sin x + cos x) 2
we get

.dx – y < a ky c
x x cos x d x x cos x
I= y m .dxF dx
cos x (x sin x + cos x) 2 dx cos x x sin x + cos x
x cos xdx
Now, let us first evaluate y
(x sin x + cos x) 2
Putting (x sin x + cos x) = t, then (sin x + x cos x – sin x)dx = dt i.e., x cos x dx = dt, we get
x cos x dt 1 1
y dx = y 2 = – = –
(x sin x + cos x) 2
t t (x sin x + cos x)
x –1 cos x + x sin x –1
Hence, I = . –y # .dx
cos x (x sin x + cos x) cos x2 (x sin x + cos x)
x –1 –x
= # + y sec 2 x. dx = + tan x + C
cos x (x sin x + cos x) cos x (x sin x + cos x)
–x + x sin 2 x + sin x cos x – x (1 – s in 2 x) + sinxcosx
= +C= +C
cos x (x sin x + cos x) cos x (x sin x + cos x)
cos x (sin x – x cos x)
= +C
cos x (x sin x + cos x)
x 2 dx (sin x – x cos x)
y = +C
(x sin x + cos x) 2 (x sin x + cos x)

296 Xam idea Mathematics–XII

@Cbsebookshub - Join Us on Telegram


PROFICIENCY EXERCISE
QQ Objective Type Questions: [1 mark each]
1. Choose and write the correct option in each of the following questions.
x9
(i) y 2
dx is equal to [NCERT Exemplar]
(4x + 1) 6
–5 –5 –5

]1 + 4g– 5 + C (d)
1 4+ 1 1 4+ 1 1 1 +4
d n d n d n +C
1
(a) + C (b) + C (c)
5x x2 5 x2 10x 10 x 2
x
(ii) The integral of y dx is equal to
x+1

(a) 2 < – + x – log ( x + 1) F + C (b)


x x x x x x
+ – x + log ( x + 1) + C
3 2 3 2
(c) x – log ( x + 1) + C (d) None of these

(iii) y e x [f (x) + f l (x)] dx = e x sin x + C then f(x) is equal to


(a) sin x (b) – sin x (c) cos x – sin x (d) sin x + cos x
x
a
(iv) y dx is
x
x x x 2a x
(a) a log a + C (b) 2a log e a + C (c) 2a log10 a + C (d) +C
log e a
3 3 cos (log x)
(v) y1 x dx is equal to
r
(a) sin (log 3) (b) cos (log 3) (c) 1 (d)
4
–1
1 tan x
(vi) y0 dx is equal to
1 + x2
r2 r2
(a) 1 (b) (c) (d) none of these
4 32
2. Fill in the blanks. [1 mark each]
sin x
(i) y dx = _____________ .  [CBSE 2020 (65/4/1)]
3 + 4 cos 2 x
2dx
(ii) y = _____________ .
1 – 4x 2
a
(iii) y f (x) dx = 0 if f (x) is an _____________ function.
–a [CBSE 2020 (65/4/1)]
6
(iv) y 2 [x] dx =_____________ , where [x] is the greatest integer function.
3

QQ Very Short Answer Questions: [1 mark each]


2
3x
3. If y (e ax + bx) dx = 4e 4x + , find the values of a and b. [CBSE (AI) 2008]
2
a
4. If y 3x 2 dx = 8, write the value of 'a'. [CBSE (F) 2012, 2014]
0

1
5. If y (3x 2 + 2x + k) dx = 0, find the value of k. [CBSE Delhi 2009]
0

Integrals 297
@Cbsebookshub - Join Us on Telegram
x
6. If f (x) = y t sin t dt, then write the value of f ' (x). [CBSE (AI) 2014]
0
1
Write the antiderivative of e 3 x +
7. o . [CBSE Delhi 2014]
x
8. If y (ax + b) 2 dx = f (x) + C, find f (x) . [CBSE (F) 2010]
1
1
9. Evaluate: y dx [CBSE (F) 2009]
0 2x + 3
dx
10. Evaluate: y [CBSE (F) 2014]
sin 2 x cos 2 x
r/4
11. Evaluate: y tan x dx [CBSE (F) 2014]
0

12. Evaluate: y cos –1 (sin x) dx [CBSE Delhi 2014]


2
e
dx
13. Evaluate : y [CBSE (AI) 2014]
e x log x
dx
14. Evaluate: y  [CBSE (AI) 2011]
1 – x2
dx
15. Write the value of y 2
. [CBSE Delhi 2011]
x + 16
QQ Short Answer Questions–I: [2 marks each]
x x
16. Given y e (tan x + 1) sec x dx = e f (x) + C [CBSE (AI) 2012]

Write f(x) satisfying the above.
17. Evaluate: y (1 – x) x dx [CBSE Delhi 2012]
x–1
18. If y e o e x dx = f (x) e x + C, find the value of f(x). [CBSE (F) 2012]
x2
r x sin x
19. Evaluate : y0 dx  [CBSE Delhi 2008]
1 + cos 2 x
^ tan x + cot x dx h = 2 r. 
r/ 2
20. Show that y0 [CBSE (AI) 2008]
a 1 r
21. If y dx = , find the value of a.  [CBSE (AI) 2014]
0 4+x 2 8
r/ 4
sin x + cos x
22. Evaluate : y e o dx  [CBSE (Ajmer) 2014]
0 3 + sin 2x
x dx
23. Find : y  [CBSE Bhubneshwar 2015]
1 + x tan x
r/ 2
5 sin x + 3 cos x
24. Evaluate : y e o dx  [CBSE Bhubneshwar 2015]
0 sin x + cos x
(x + 3) e x
25. Evaluate : y dx  [CBSE Punchkula 2015]
(x + 5) 3
r/2
2 sin x
26. Evaluate : y dx  [CBSE Patna 2015]
0 2 sin x + 2 cos x
r/2

27. Evaluate : y e x (sin x – cos x) dx  [CBSE Delhi 2014]


0

298 Xam idea Mathematics–XII

@Cbsebookshub - Join Us on Telegram


cos x
28. Evaluate: y dx  [CBSE (AI) 2009]
x
r/ 2
29. Write the value of the following integral y –r/2 sin 5 x dx . [CBSE (AI) 2010]
x sin –1 x
30. Evaluate: y dx  [CBSE (AI) 2012, (F) 2016]
1 – x2
QQ Short Answer Questions–II: [3 marks each]

31. Evaluate: y x sin –1 x dx [CBSE (AI) 2009; Chennai 2015]


dx
32. Evaluate: y  [CBSE (AI) 2013]
x ( x 5 + 3)
33. Evaluate: y x 2 . cos –1 x dx [CBSE (F) 2009]
dx
34. Evaluate: y [CBSE (AI) 2013]
x^x3 + 8h
3x + 5
35. Evaluate: y 2
dx  [CBSE (F) 2011]
x – 8x + 7
1 – x2
36. Evaluate: y dx  [CBSE Delhi 2010, (F) 2013]
x (1 – 2x)
( x + 2)
37. Evaluate: y dx  [CBSE (AI) 2010]
(x – 2)(x – 3)
r x
38. Evaluate: y dx [CBSE Delhi 2010]
0 1 + sinx
4
39. Evaluate: y (| x |+| x – 2 |+| x – 4 |) dx  [CBSE Delhi 2013]
0
sin z
40. Evaluate the following indefinite integral : y 2
dz [CBSE Sample Paper 2016]
2
sin z + 2 cos z + 3
x
41. Find : y dx [CBSE (Central) 2016]
x4 + x2 – 2

42. Find: y ^3x + 1h 4 – 3x – 2x 2 dx  [CBSE (Central) 2016]


r x sin x
43. Evaluate : y dx  [CBSE (East) 2016]
0 1 + 3 cos 2 x
x2 + x + 1
44. Find : y dx  [CBSE (North) 2016]
^ x 2 + 1 h^ x + 2 h

45. Find : y ^ x + 3 h) 3 – 4x – x 2 dx  [CBSE (North) 2016]


^ x + 1 h^ x + 4 h
2 2
46. Find : y dx  [CBSE (F) 2016]
^ x 2 + 3 h^ x 2 – 5 h
47. Evaluate the following definite integral :  [CBSE Sample Paper 2016]
r 2x ^1 + sin x h
y dx
–r 1 + cos 2 x
r/ 2
1 – sin 2x
48. Evaluate : y e 2x d n dx  [CBSE Guwahati 2015]
–r/2 1 – cos 2x
r/ 2
49. Evaluate : y0 log sin x dx  [CBSE (AI) 2008]
2
2x + 3
50. Evaluate : y 2
dx  [CBSE (F) 2013]
x + 5x + 6

Integrals 299
@Cbsebookshub - Join Us on Telegram
51. Evaluate : y e 2x . sin (3x + 1) dx  [CBSE (F) 2015]
1 – cos x
52. Evaluate : y dx  [CBSE Chennai 2015]
cos x (1 + cos x)
sin 2x
53. Find: y dx  [CBSE 2019 (65/3/1)]
(sin x + 1) ^sin 2 x + 3h
2

QQ Long Answer Questions: [5 marks each]


r /3 dx
54. Evaluate: y [CBSE Delhi 2014]
r /6 1 + cotx
3
55. Evaluate y ^ e 2 – 3x + x 2 + 1 h dx as a limit of a sum. [CBSE Delhi 2015]
1
sin x – x cos x
56. Evaluate: y dx [CBSE Ajmer 2015]
x ^ x + sin x h
r/4
dx
57. Find: y 3
[CBSE Allahabad 2015]
0 cos x 2 sin 2x
r/2
cos 2 x dx
58. Evaluate : y  [CBSE Ajmer 2015]
0 1 + 3 sin 2 x

Answers
1. (i) (d) (ii) (a) (iii) (a) (iv) (d) (v) (a) (vi) (c)
2 cos x
tan –1 e o + C (ii) sin–1 (2x) (iii) odd
1
2. (i) – (iv) 24
2 3 3
3. a can't be determined, b = 3 4. a = 2 5. k = – 2 6. x sin x
]ax + bg3
7. 2x 3/2 + 2 x + C 8. 9. 5– 3 10. tan x – 1/ tan x + C
3a
1 rx x 2 1 x
11. log 2 12. – + C 13. log 2 14. sin–1 x + C 15. tan –1 + C
2 2 2 4 4
2 3/2 2 5/2 1 r2
16. f (x) = sec x 17. x – x + C 18. f ( x ) = 19. 21. a = 2
3 5 x 2 2
1 1
22. log 3 23. log |cos x + x sin x | + C 24. 2π 25. e x . +C
4 (x + 5) 2
r
26. 27. 1 28. 2 sin x + C 29. 0 30. x – 1 – x 2 sin –1 x + C
4
x2 1 x 1 x5
31. sin –1 x – sin –1 x + 1 – x2 + C 32. log 5 +C
2 4 4 15 x +3
x3 1 1 1 x3
33. cos 1 x  1  x 2  (1  x 2 )3/2  C 34. log 3 +C
3 3 9 24 x +8
1 3
35. 3 x 2  8 x  7  17 log|( x  4)  x 2  8 x  7 | C 36. x  log| x|  log| 2 x  1| C
2 4
9  5
37. x 2  5 x  6  log  x    x 2  5 x  6  C
2  2
cos z – 1
38. r 39. 20 40. – sin –1 f p+ �
5
2  x  1 x 1
41. tan 1    C
3  2  6 log x  1

300 Xam idea Mathematics–XII

@Cbsebookshub - Join Us on Telegram


1 5  3 3 205 1  4  3 
42. ( 4  3 x  2 x 2 )3/2  2
 x  4  2  2 x  x  64 2 sin  41  x  4    C
2 4 2  

3 2 3 1 1
43.
 44. log| x  2|  log| x 2  1|  tan 1 x  C
9 5 5 5
1 x2 7 x2
45.  (3  4 x  x 2 )3/2  3  4 x  x 2  sin 1  C
3 2 2  7 

46. 2 (10  4 x  3 x 2 )3/2  11  x  2  10  4 x  3 x 2  17 sin 1  3  x  2    C


    
9 2 3 3 9  34  3 
e r/2 r
47. p2 48. 49. – log 2 50. 2x + 11 log |x + 2| – 21 log |x + 3|+ C
2 2
2e 2x . sin (3x + 1) 3e 2x . cos (3x + 1) x
51. – +C 52. log sec x + tan x – 2 tan +C
13 13 2

log ^sin 2 x + 1h – log ^sin 2 x + 3h + C


1 1 r
53. 54.
2 2 12
b
32  (e 1  e 7 )
55.
3
Hint: Apply  f (x) dx  lim
h 0
h{ f ( a  h)  f ( a  2 h)  ...  f ( a  nh)}
a
3 3 3
Write (e 2  3 x  x 2  1)dx  e 2  3 x dx  ( x 2  1) dx
  
1 1 3 1
3 3 x e 2 e 3 (1  e 6 ) 32
 and  ( x 2  1)dx 
2
Get e e dx 
1 3 1
3
56. log|x|– log|x + sin x|+ C
sin x  x cos x ( x  sin x )  x (1  cos x)

Hint: Write  x ( x  sin x)
dx  
x ( x  sin x )
dx

6 dx 1 /4 dx 1 /2 dx

/ 4
57. ; Hint :     4 Then put tan x = t
5 0 3
cos x 2 sin 2 x 2 0 sin x 2 0 cos x tan x
cos 3 x .cos 2 x
cos x
r
58.
6

SELF-ASSESSMENT TEST
Time allowed: 1 hour Max. marks: 30
1.
Choose and write the correct option in the following questions. (4 × 1 = 4)
0
dx
(i) The value of 2
y dx is
–1 x + 2x + 2
r –r r
(a) 0 (b) (c) (d)
4 4 2
dx
(ii) The value of y is
x x4 – 1
1
(a) sec –1 (x2) + C (b) sec –1 (x2) + C (c) sec –1 x + C (d) tan –1 (x2) + C
2

Integrals 301
@Cbsebookshub - Join Us on Telegram
y
dt d2 y
(iii) If x = y and = ay, then a is equal to
0 1 + 9t 2 dx2
(a) 3 (b) 6 (c) 9 (d) 1

(iv) If y x6 sin (5x7) dx = k cos (5x7), x ! 0 then k is equal to


5
1 1
(a) 7 (b) – 7 (c) (d) –
7 7
2. Fill in the blanks. (2 × 1 = 2)
^log x h
2
(i) y dx = _____________ .
x
4
(ii) y x – 2 dx = _____________ .
0

QQ Solve the following questions. (2 × 1 = 2)


3. Find the antiderivative of y sin 2xdx.
2
1
4. Find: y e x – o dx
x
QQ Solve the following questions. (4 × 2 = 8)
x
(x – 3) e
5. Find: y dx
( x – 1) 3
1 1
6. Evaluate: y d n e 2x dx
2

1 x 2x 2
^ tan x + cot x dx h = 2 r.
r/ 2
7. Show that y0
a 1 r
8. If y dx = , find the value of a.
0 +
4 x 2 8

QQ Solve the following questions. (3 × 3 = 9)


r
x sin x
9. Evaluate: I = y 2
dx
0 1 + 3 cos x
1
10. Find: y dx
x ( x 4 – 1)
r/ 2
11. Evaluate: I = y (2 log sin x – log sin 2x) dx
0

QQ Solve the following question. (1 × 5 = 5)


r/2
12. Evaluate: y log (sin x) dx
0

Answers
^log x h + C (ii) 4
1 3
1. (i) (b) (ii) (a) (iii) (c) (iv) (d) 2. (i)
3
1 x2 ex e 2 (e 2 – 2)
3. – cos .2x 4. – 2x + log x + C 5. +C 6.
2 2 (x – 1) 2 4
3 2 x4 – 1
log c m 12. – log 2
1 r 1 r
8. a = 2 9. r 10. log +C 11.
9 4 x4 2 2 2
zzz

302 Xam idea Mathematics–XII

@Cbsebookshub - Join Us on Telegram


Application 9
of Integrals

Y
1. Area of Bounded Regions: Let f (x) be a continuous function defined
on [a, b], then the area bounded by the curve y = f (x), the x-axis and
the straight line x = a and x = b is given by

x=b
x=a
b b
y f (x) dx = y ydx
a a
X′ X
O
Theorem 1. If the curve y = f (x) lies below x-axis, then the area
Y′
bounded by y = f (x), y = 0, x = a and x = b will be negative Y
and in this situation we take the modulus of the area i.e., O (a,0) (b,0)
X′ X
the area is represented by

x=a

x=b
b
y f (x)dx
a
y = f(
x)
Theorem 2. If the curve is given in the form x = f(y), then the area of
Y′
curve x = f(x) bounded between x = 0, y = c and y = d is
Y
given by y=d
(0, d)
d (y)
y f (y)dy
f
x=
c

(0, c) y=c
Theorem 3. The area bounded between the curves y = f(x) and

y = g(x) is given by X′ X
O

y 6 f (x) – g (x)@dx
b Y′

a
Y
 where x = a, x = b are the abscissae of the point of y = f (x)
intersection of two curves y = f(x) and y = g(x) and
f(x) > g(x) ∀ x ∈ (a, b)
Note: If y changes sign in [a, b], obtain the area of each part
x=b

y = g (x)
x=a

separately and then add after taking their modulus.


X′ X
O (a,0) (b,0)
Y′

Application of Integrals 303


@Cbsebookshub - Join Us on Telegram
Selected NCERT Questions
1. Using integration, find the area lying above x-axis and included between the circle x2 + y2 = 8x
and inside the parabola y2 = 4x. [CBSE 2019 (65/1/1), CBSE 2020 (65/2/1)]
Sol. Given equation of curves:
x2 + y2 = 8x  ⇒  x2 – 8x + y2 = 0  ⇒ (x – 4)2 + y2 = (4)2 ...(i)
It is circle with centre (4, 0) and radius 4.
and given parabola y2 = 4x ...(ii)
On plotting these two curves, we have the required region (shaded) OAB.
Points of intersection of the curves (i) and (ii) are Y
O(0, 0) and A(4, 4) in the first quadrant
A (4, 4) y2 = 4x
4 8
∴ Area of shaded region y yp dx + y yc dx

x2 + y2 = 8x
0 4
4 8
X′ X
∴ A = y
4x dx + y (4) 2 – (x – 4) 2 dx O (4, 0) B (8, 0)
0 4
4 8
= 2 y x dx + y (4) 2 – (x – 4) 2 dx
0 4
8
8 B
2 3 4 + < ( x – 4)
sin –1 c mF
16 x–4
= 2 # 8x – x 2 + Y′
3 x2 0 2 2 4 4

# 8 + ; # 0 + 8 sin –1 (1) – 0 – 0E
4 4
=
3 2
32 + 32 + r
= 8 sin –1 (1) = 8#
3 3 2
∴ Required area = c 4r m sq. units.
32 +

3
2. Using integration, find the area of the greatest rectangle that can be inscribed in an ellipse.
[CBSE 2019 (65/4/1)]
Sol. Let ABCD is the required rectangle whose area to be Y
found.
b
Let OB makes angle θ with positive direction of x-axis.
A Q B (a cos θ, b sin θ)
Then co-ordinate of the point B is (a cos θ, b sin θ).
∴  Area of rectangle OPBQ = ab sin θ cos θ
X′
θ
X
–a O P a
So, ar ( ABCD) = 4 × ar ( OPBQ).
r r
2 D C
4ab y2
= 4 y ab sin i cos i di = 2 sin i cos i di –b
0 2 0
r r Y′
2
= 2ab y sin 2i di = 2ab ;– E
cos 2i 2
0
2 0

= –ab 5cos 2i?0 = –ab 5cos r – cos 0? = – ab (–1 –1) = 2ab. sq. units.
2

3. Find the area of the region enclosed between the two circles x2 + y2 = 1 and (x – 1)2 + y2 = 1.
[CBSE 2013; (AI) 2008, CBSE 2019 (65/3/1)]
OR
Find the area of the region enclosed between the two circles: x2 + y2 = 4 and (x – 2)2 + y2 = 4.
[CBSE Delhi 2008, 2013]

304 Xam idea Mathematics–XII

@Cbsebookshub - Join Us on Telegram


Sol. The equations of the given curves are Y
2 2
x + y = 1 …(i)

2 , 3
2
A 1
2 2
and (x – 1) + (y – 0) = 1 …(ii)
Obviously, curve (i) is a circle having centre at (0, 0) and radius
1 while curve (ii) are circle having centre at (1, 0) and radius 1. X′ X
O E B (1, 0)
Shaded region is required region which is symmetrical about
x-axis.
D
Now for intersection points (i) and (ii) are solved as
(x – 1)2 + 1 – x2 = 1 Y′
1 3
2
x – 2x + 1 – x = 0 2
& x=
2
` y=
2
So, coordinates of A are c ,
1 3m
.
2 2
Hence, area of required region = 2 × area of OABO = 2 [area OAEO + area EABE]
= 2 8 y0 1 – (x – 1) 2 dx + y1/2 1 – x 2 dxB
1/2 1

1/2
x – 1 1/ 2 + 1 x 1
⇒ A = 2 <: . (x – 1) 1 – (x – 1) 2D + : sin –1 a kD : x 1 – x 2 + sin –1 ` jD F
1 1 1

2 0 2 1 0 2 2 1 1/2

⇒ A = =) – + sin –1 d – 1 n – sin –1 (– 1) 3 + )sin –1 (1) – – sin –1 d n3G


3 3 1

4 2 4 2

– =e o sq units.
3 r r r 3 r 2r 3
⇒ A = –
– + + – –
4 6 2 2 4 6 3 2
OR
8r
Solution is similar as above. Ans.
– 2 3 sq. units
3
4. Find the area of the region
{(x, y) : 0 ≤ y ≤ x2 + 1, 0 ≤ y ≤ x + 1, 0 ≤ x ≤ 2}

Sol. Let us first sketch the region whose area is to be found out. This region is the intersection of the
following regions.
A1 = { (x, y) : 0 ≤ y ≤ x2 + 1)}, A2 = { (x, y) : 0 ≤ y ≤ x + 1},
and A3 = { (x, y) : 0 ≤ x ≤ 2}
The points of intersection of y = x2 + 1 and y = x + 1 are points
P (0, 1) and Q (1, 2). From the fig., the required region is the
x=1

x=2

shaded region OPQRSTO whose area


= area of the region OTQPO + area of the region TSRQT
1 2
= y0 (x 2 + 1) dx + y1 (x + 1) dx
1 2
x3 x2
+ x oH + >e + x oH = =d + 1 n – 0G + =(2 + 2) – d + 1 nF =
1 1
= >e
23
sq. units
3 0 2 1
3 2 6
x2 y2 x y
5. Find the area of smaller region bounded by the ellipse + = 1 and the line + = 1.
a 2
b 2 a b
2
y x2
Sol. The given curve+ = 1 represents an ellipse with centre at origin, major axis of length 2a,
a2 b2
minor axis of length 2b and vertices (± a, 0) and (0, ± b).
x y
The line of equation is + = 1.
a b

Application of Integrals 305


@Cbsebookshub - Join Us on Telegram
Point of intersection of ellipse and line are A(a, 0) and B(0, b). Y

Required Area = Area of OACBO – Area of OADBO


x2 y2 B (0, b)

= (Area under the curve 2
+ 2
= 1 , x-axis, x = 0 and x = a) C
a b
x y D
A (a, 0)
– (area under the line + = 1, x-axis, x = 0 and x = a) X′ X
a b (–a, 0) O
a b b a
= y a 2 – x 2 dx – y (a – x) dx
0 a a0 (0, –b)
2 a 2 a
b x 2 b
= = a – x 2 + sin –1 G – =ax – G
a x x

a 2 2 a 0 a 2 0 Y′
2 2
sin –1 1G – 60 + 0@3 – *e a 2 – – 0 o3
b b
= *=0 +
a a

a 2 a 2
ab ab ab r
=
r– = d – 1 n sq. units.
4 2 2 2
6. Using the method of integration find the area bounded by the curve x + y = 1 .

Sol. The given curve can be redefined in the form of 4 straight


lines as
_
x + y = 1 bbb
b
–x + y = 1 bb
`b
–x – y = 1 bb
bb
x – y = 1b
a
Required area = 4 × area OABO
= 4 (area between line AB, x-axis, x = 0 and x =1)
1
1
x2 12
= 4 y (1 – x) dx = 4 =x –
G = 4 >e 1 – o – 0H = 2 sq. units.
0 2 0 2

7. Find the area bounded by curves {(x, y) : y ≥ x2, and y = |x|}.


OR
Find the area of the region {(x, y) : x ≤ y ≤ |x|}. [CBSE (AI) 2013]
Sol. The required area is bounded between two curves y = x2 and y = |x|. Both of these curves are
symmetric about y-axis and shaded region in the figure shows the region whose area is required.
Therefore, the required area
A = 2 × area of the region R1

Now, to find the point of intersection of the curves y =|x| and y = x2, we solve them simultaneously.
Clearly, the region R1 is in the first quadrant, where x > 0 Y
y=x


|x|= x ⇒ y = x ...(i)
x
y=

y=
2

2
and y = x ...(ii) B(–1,1)
–x

A(1,1)
Solving these two equations, we get, x = x2 R1
⇒ Either x = 0 or x = 1

X′ X
O
The limits are, when x = 0, y = 0 and when x = 1, y = 1
So, the points of intersection of the curves are O (0, 0) and A(1, 1,).
Now, required area = 2 area of the region R1 Y′

306 Xam idea Mathematics–XII

@Cbsebookshub - Join Us on Telegram


1
= 2 y0 [(y of the line y = x) – (y of the parabola y = x 2)] dx
1
x2 x3
= 2 y0 (x – x 2) dx = 2 > – H
1
2 3 0

1 1 1
= 2 = – G = sq unit.
2 3 3
8. Find the area of the region {(x, y) : y2 ≤ 4x, 4x2 + 4y2 ≤ 9}.
[CBSE Compartment 2017, CBSE 2019 (65/4/1)]
Sol. Required area is bounded by the circle 4x2 + 4y2 = 9 and the parabola y2 = 4x. It is interior to both
these curves.
Given circle is 4x2 + 4y2 = 9 …(i)
2
and the given parabola is y = 4x …(ii)
2 2 2
Now, y = 4x and 4x + 4y = 9 meet at x of
4x2 + 16x = 9, i. e., 4x2 + 16x – 9 = 0
– 16 ! 256 + 144 –16 ! 20 1 9
x = = = ,– .
8 8 2 2
But distance can’t be negative, therefore, the two curves meet
1
at x = .
2
Required area = 2 [area (OACO) + area (ABCA)]
1/2 3/2
9 – 4x 2
Required area = 2 > y 4x dx + y dxH
0 1/2 4
1/2
x 3/2 3/2
3 2
= 2 # 2 = G +2 y d n – x 2 dx
3/2 0 1/2 2
3/2
8 1 3/2 9
>d n – 0H + 2 = G
x 9 x
= – x 2 + 4 sin –1
3 2 2 4 2 3/2 1/2

8 1 9 9 1 9
= . + 2 <0 + sin –1 (1) – – – sin –1 (1/3)F
3 2 2 8 4 4 8

2 2 9 r 2 9 1 9r 2 9 1
= + . – – sin –1 d n = + – sin –1 d n sq. units.
3 4 2 2 4 3 8 6 4 3

Multiple Choice Questions [1 mark]


Choose and write the correct option in the following questions.

1. The area enclosed by the circle x2 + y2 = 2 is equal to [NCERT Exemplar]

(a) 4π sq units (b) 2 2 r sq units (c) 4π2 sq units (d) 2π sq units

x2 y2
2. The area enclosed by the ellipse + = 1 is equal to [NCERT Exemplar]
a2 b2
(a) π2 ab sq units (b) π ab sq units (c) πa2 b sq units (d) π ab2 sq units

Application of Integrals 307


@Cbsebookshub - Join Us on Telegram
r
3. The area of the region bounded by the y-axis, y = cos x and y = sin x, 0 # x # is
2
[NCERT Exemplar]
(a) 2 sq units (b) ( 2 + 1) sq units (c) ( 2 – 1) sq unit (d) (2 2 – 1) sq units

4. The area of the region bounded by the curve x2 = 4y and the straight line x = 4y – 2 is
[NCERT Exemplar]
3 5 7 9
(a) sq unit (b) sq unit (c) sq unit (d) sq units
8 8 8 8
5. The area of the region bounded by the curve y = 16 – x 2 and x-axis is [NCERT Exemplar]
(a) 8 p sq units (b) 20π sq units (c) 16π sq units (d) 256π sq units

6. Area of the region in the first quadrant enclosed by the x-axis, the line y = x and the circle
x2 + y2 = 32 is [NCERT Exemplar]
(a) 16π sq units (b) 4π sq units (c) 32π sq units (d) 24 sq units

7. Area of the region bounded by the curve y = cos x between x = 0 and x = π is [NCERT Exemplar]
(a) 2 sq units (b) 4 sq units (c) 3 sq units (d) 1 sq unit

8. The area of the region bounded by the curve x = 2y + 3 and the lines y = 1 and y = –1 is
[NCERT Exemplar]
3
(a) 4 sq units (b) sq units (c) 6 sq units (d) 8 sq units
2
9. The area of the region bounded by the curve y = x2 and the line y = 16 is
37 256 64 128
(a) sq units (b) sq units (c) sq units (d) sq units
3 3 3 3
10. The area of the region bounded by the curve y2 = 9x, y = 3x is
1
(a) 1 sq unit (b) sq unit (c) 4 sq units (d) 14 sq units
2
11. The area of the curve y = sin x between 0 and π is
(a) 2 sq units (b) 4 sq units (c) 12 sq units (d) 14 sq units
2 3
12. The area of the region bounded by the curve ay = x , the y-axis and the lines y = a and

y = 2a is
3
(a) 3 sq units (b) a 2 2 # 2 2/3 –1 sq units
5
3
(c) a 2 3/2 –1 sq units (d) 1 sq unit
5
13. The area enclosed by the curve x = 3 cos t, y = 2 sin t is
(a) 4π sq units (b) 6π sq units (c) 14π sq units (d) 7π sq units
14. The area of the region bounded by the curves x = at2 and y = 2at between the ordinate
corresponding to t = 1 and t = 2 is
56 2 40 2
(a) a sq units (b) a sq units (c) 5π sq units (d) None of these
3 3
a
15. The area of a minor segment of the circle x2 + y2 = a2 cut off by the line x = is
2
a2 a2
(a) (4r – 3 3 ) sq units (b) (4r – 3) sq units
12 4
a2
(c) (3r – 4) sq units (d) None of these
12

308 Xam idea Mathematics–XII

@Cbsebookshub - Join Us on Telegram


16. The area of the region bounded by the curve y = x3 and y = x + 6 and x = 0 is
(a) 7 sq units (b) 6 sq units (c) 10 sq units (d) 14 sq units
17. The area under the curve y = 2 x included between the lines x = 0 and x = 1 is
4
(a) 4 sq units (b) 3 sq units (c) sq units (d) None of these
3
18. The area under the curve y = a 2 – x 2 included between the lines x = 0 and x = a is
ra 2 a2
(a) sq units (b) sq units (c) πa2 sq units (d) 4π sq units
4 4
19. The area of the region bounded by the triangle whose vertices are (–1, 1), (0, 5) and (3, 2) is
15
(a) sq units (b) 15 sq units (c) 4 sq units (d) 10 sq units
2
20. The area of the region bounded by the line y – 1 = x, the x-axis and the ordinates x = – 2 and
x = 3 is
4 7 17 16
(a) sq units (b) sq units (c) sq units (d) sq units
3 2 2 3

Answers
1. (d) 2. (b) 3. (c) 4. (d) 5. (a) 6. (b)
7. (a) 8. (c) 9. (b) 10. (b) 11. (a) 12. (b)
13. (b) 14. (a) 15. (a) 16. (c) 17. (c) 18. (a)
19. (a) 20. (c)

Solutions of Selected Multiple Choice Questions


2
1. Area = 4 y 2 – x 2 dx
0
2
= 4 d 2 2 – x + sin n = 2r sq units.
x 2 –1 x
2 0
r
3. We have, Y-axis i.e., x = 0, y = cos x and y = sin x, where 0 # x # .
2
r/ 4
∴ Required area = y0
(cos x – sin x) dx Y

5sin x?0 + 5cos x?0


r/4 r/4 y = sin x
= 1

= c sin – sin 0 m + c cos – cos 0 m


r r X′ X
O π/4 π/2
4 4
1 1
=e – 0o + e – 1o =
1 + 1
–1 y = cos x
2 2 2 2
Y′
2 =
= –1 + ( 2 – 1) sq units.
2
6. We have, area enclosed by x-axis i.e., y = 0, y = x and the circle x2 + y2 = 32 in first quadrant.
Since, x2 + (x)2 = 32 [ y = x]
2

2x = 32 ⇒ x=±4

Application of Integrals 309


@Cbsebookshub - Join Us on Telegram
So, the intersection point of circle x2 + y2 = 32 and line y = x are (4, 4) or (– 4, 4).
Y
and x2 + y2 = (4 2 ) 2
y=x

x2 + (0)2 = 32 [ y = 0]
(4, 4)

x=± 4 2 (0, 0)
X′ X
So, the circle intersects the x-axis at ^! 4 2 , 0 h .
O (4, 0)
= 32
2

4 4 2 2 +y
(–4, –4) x
Area of shaded region = y xdx + y (4 2 ) 2 – x 2 dx
0 4
Y′
4 4 2
x2 (4 2 ) 2
= + x (4 2 ) 2 – x 2 + sin –1
x
2 0 2 2 4 2 4

16 + > 4 2 (4 2 ) 4
H
4
= $ 0 + 16 sin –1 – (4 2 ) 2 – 16 – 16 sin –1
2 2 (4 2 ) 2 4 2

= 8 + ;16 $ – 2 $ 16 – 16 $ E = 8 + 58r – 8 – 4r? = 4r sq units.


r r
2 4
7. Required area enclosed by the curve y = cos x, x = 0 and x = π is Y
r/ 2 r
A = y0
cos x dx + yr/2 cos xdx

= ;sin – sin 0E + sin – sin r


r r O π/2 π
X
2 2

= 1 + 1 = 2 sq units.
y = cos x
Y
1
8. Required area, A = y–1 (2y + 3) dy
y=1
2 1
= = + 3yG
2y
2 –1 O
X
= 8y 2 + 3yB–1
1

= [1 + 3 – 1 + 3] = 6 sq units. 3
y=–1
+
2y
=
x

Fill in the Blanks [1 mark]


1. The area of the region bounded by the curve x = y2, y-axis and the line y = 3 and y = 4 is ___________.
2. The area bounded by the curve y = sin x, x-axis and the ordinates x = 0 and x = p is ___________.
3. The area of the region bounded by the curve y = x – x2 between x = 0 and x = 1 is ___________.
x2 y2
4. The area of the region bounded by the ellipse + = 1 is ___________.
25 16

Answers
37 1
1. sq. units 2. 2 sq. units 3. sq. unit 4. 20p sq. units
3 6

310 Xam idea Mathematics–XII

@Cbsebookshub - Join Us on Telegram


Solutions of Selected Fill in the Blanks
Y
1. Required area
y=4
4
A = y x dy y=3
3
X
O
4 3 4 3 3
= y y 2 dy = = G =
y (4) – (3)
3 3 3 3
64 – 27 = 37
= sq units.
3 3
Area = y y dx = y sin x dx = 5– cos x?0
r r
r
2.
0 0

  = – 5cos r – cos 0? = – 5–1 –1?


  = 2 sq. units.

Very Short Answer Questions [1 mark]


1. Sketch the region {(x, 0) : y = 4 – x 2 } and X-axis. Find the area of the region using integration.
[NCERT Exemplar]
2
Sol. Given region is {(x, 0) : y = 4 – x } and X-axis.

We have,    y = 4 – x2 & y2 = 4 – x2 & x2 + y2 = 4


2 2 Y
∴  Area of shaded region, A = y–2 4 – x 2 dx = y–2 2 2 – x 2 dx

2
22
=< $ sin –1 F
x 2 x
2 – x2 +
2 2 2 –2 X
(–2, 0) (2, 0)
2 + r+2
= $0 2$ $ 0 – 2 sin –1 (–1)
2 2 2
r r
= 2$ +2$ = 2π sq units.
2 2
2. Determine the area under the curve y = a 2 – x 2 included between the lines x = 0 and x = a.
[NCERT Exemplar]

Sol. Given equation of the curve is y= a2 – x2 . Y


y2 = a 2 – x 2 & y 2 + x 2 = a 2
a X
∴  Required area of shaded region, A = y0
a 2 – x 2 dx –a O a

a
a2
== a – x 2 + sin –1 G
x 2 x

2 2 a 0
x=0 x=a
a2 a2
= <0 + sin –1 (–1) – 0 – sin –1 0F
2 2
a2 r r a2
= $ = sq units.
2 2 4

Application of Integrals 311


@Cbsebookshub - Join Us on Telegram
3. Find the area bounded by the curve y = sin x between x = 0 and x = 2π. [NCERT Exemplar]
2r r 2r
Sol. Required area = y0 sin x dx = y0 sin x dx + yr sin x dx Y
y = sin x
= – 5cos x?0 + 5– cos x?r2r
r

= – 5cos r – cos 0? + – 5cos 2r – cos r? O π



X

= – [ – 1 – 1 ] + |– (1 + 1)|
= 2 + 2 = 4 sq units.

Long Answer Questions [5 marks]


1. Find the area of the following region using integration {(x, y) : y ≤ |x| + 2, y ≥ x 2 }
[CBSE 2020 (65/3/1)]
Sol. We have given regions
{(x, y) : y ≤ |x| + 2, y ≥ x2}
Now, we have, y ≤ |x| + 2 ...(i)
2
y ≥ x ...(ii)
and
On plotting inequalities (i) and (ii), we get the required region Y
(shaded) OABC.
y= 2
Now, points of intersection of the curve (i) and (ii), are (–2, 4)
C(–2,4) –x x+ A(2,4)
+2 y=
and (2, 4). B (0,2)
2
Here, region is symmetric about y-axis
∴ Area of required region OABCO = 2 × area of region OABO

X' X
2 O
∴ Area
= 2 y #(x + 2) – x - dx2
0 Y'
2
x2 x3 (2) 2 (2) 3
= 2 < + 2x – F = 2 < +2#2– – 0F
2 3 0 2 3

= 2 ;2 + 4 – E = 2 c 6 – m =
8 8 20
sq. units.
3 3 3
2. Using the method of integration, find the area of the region bounded by the lines
3x – 2y + 1 = 0, 2x + 3y – 21 = 0 and x – 5y + 9 = 0. [CBSE Delhi 2012, 2019 (65/4/1)]
Sol. Given lines are
3x – 2y + 1 = 0 ...(i)
2x + 3y – 21 = 0 ...(ii)
x – 5y + 9 = 0 ...(iii)
For intersection of (i) and (ii)

Applying (i) × 3 + (ii) × 2, we get
9x – 6y + 3 + 4x + 6y – 42 = 0

13x – 39 = 0 ⇒ x=3
Putting it in (i), we get
9 – 2y + 1 = 0

2y = 10 ⇒ y=5

312 Xam idea Mathematics–XII

@Cbsebookshub - Join Us on Telegram


Intersection point of (i) and (ii) is (3, 5) Y

For intersection of (ii) and (iii) 8

0
1=
Applying (ii) – (iii) × 2, we get 2x

+
7
+3

2y
2x + 3y – 21 – 2x + 10y – 18 = 0 y–


21

3x
6 =0
⇒ 13y – 39 = 0
⇒ y=3
Putting y = 3 in (ii), we get 5 (3, 5)

2x + 9 – 21 = 0
4
⇒ 2x – 12 = 0
⇒ x=6
(6, 3)
Intersection point of (ii) and (iii) is (6, 3) 3

+9= 0
For intersection of (i) and (iii)
2
x – 5y
(1, 2)
Applying (i) – (iii) × 3, we get
1
3x – 2y + 1 – 3x + 15y – 27 = 0
⇒ 13y – 26 = 0
⇒ y=2 X' X
–1 0 1 2 3 4 5 6 7 8
Putting y = 2 in (i), we get
Y'
3x – 4 + 1 = 0 ⇒ x=1
Intersection point of (i) and (iii) is (1, 2).
With the help of point of intersection we draw the graph of lines (i), (ii) and (iii)
Shaded region is required region.
3
3x + 1 + y6 –2x + 21 6
x+9
∴ Area of required region = y
dx dx – y dx
1 2 3 3 1 5
3 y3 1 3 2 6 6
1 6 9 6
= x dx + y dx – y x dx + 7 y dx – y x dx – y dx
21 21 33 3 51 51
3 6 6
3 ; x2 E + 1 2 x2 1 x2
[x] 13 – ; E + 7 [x] 63 – ; E – [x] 16
9
=
2 2 1 2 3 2 3 5 2 1 5
3 1 2 1 9
= (9 – 1) + (3 – 1) – (36 – 9) + 7 (6 – 3) – (36 – 1) – (6 – 1)
4 2 6 10 5
7 7 20 – 7 = 13
= 6 + 1 – 9 + 21 – – 9 = 10 – = sq units.
2 2 2 2
3. Find the area of the region {(x, y) : x2 + y2 ≤ 4, x + y ≥ 2}. [CBSE (AI) 2012]
Sol. Let R = {(x, y) : x2 + y2 ≤ 4, x + y ≥ 2}
⇒ R = {(x, y) : x2 + y2 ≤ 4} ∩ {(x, y) : x + y ≥ 2}

i.e., R = R1 ∩ R2, where R1 = {(x, y) : x2 + y2 ≤ 4} and
Y
R2 = {(x, y) : x + y ≥ 2}
For region R1 2 x 2+
2 2
Obviously x + y = 4 is a circle having centre at (0,0) and
y
2
=4

1
radius 2.
Since (0,0) satisfy x2 + y2 ≤ 4. Therefore region R1 is the X′ X
–2 –1 O 1 2
x+

region lying interior of circle x2 + y2 = 4 –1


y=

For region R2
2

x 0 2
y 2 0
Y′

Application of Integrals 313


@Cbsebookshub - Join Us on Telegram
x + y = 2 is a straight line passing through (0, 2) and (2, 0). Since (0, 0) does not satisfy x + y ≥ 2

therefore R2 is that region which does not contain origin (0, 0) i.e., above the line x + y = 2.
Hence, shaded region is required area.
Now, area of required region
2 2
= y 4 – x 2 dx – y (2 – x) dx
0 0
2
x 2 x2
= : x 4 – x 2 + 4 sin –1 ` jD – 2 [x] 02 + ; E = [2 sin –1 1 – 0] – 2 [2 – 0] + : – 0D
1 1 4
2 2 2 0 2 0 2
r
= 2 # – 4 + 2 = (r – 2) sq units .
2
4. Find the area of the region included between the parabolas y2 = 4ax and x2 = 4ay, where a > 0.
[CBSE (AI) 2009; (F) 2013]
Sol. Given parabolas are y2 = 4ax ...(i), x2 = 4ay ...(ii)
Obviously, curve (i) is right handed parabola having vertex at (0, 0), while curve (ii) is upward
parabola having vertex at (0, 0).
Shaded region is required region. Y
For coordinate of intersection point A, (i) and (ii) are solved as
2
x2
d n = 4ax ⇒ x4 = 64a3x
4a

x2 =
D
⇒ x(x3 – 64a3) = 0 ⇒ x = 0 or x3 – 64 a3 = 0 C
A(4a, 4a)

4ay
⇒ x = 4a and y = 4a

D
X’ X
Hence, coordinate of A ≡ (4a, 4a). O B(4a, 0)

Therefore, area of required region


y2=
= area of OCABO – area of ODABO 4ax
4a

A=< F
2 4 a 3/ 2 x 3
4a 4a x Y’
= y0 4ax dx – y0
4a
dx & 3
x –
12a 0
4 a (4a) 3 32a 2 16a 2 16a 2

A= (4a) 3/2 – = – = sq units.
3 12a 3 3 3
5. Find the area of the region in the first quadrant enclosed by the x-axis, the line y = x and the
circle x2 + y2 = 32. [CBSE Delhi 2014, CBSE 2020 (65/4/1)]
Sol. The given equations are
Y
y = x ...(i)

)
,4

x2 + y2 = 32
x

and ...(ii)
(4
y=
B

Solving (i) and (ii), we find that the line and the circle meet at
0)
B(4, 4) in the first quadrant. Draw perpendicular BM to the x-axis. 2,
4
A(
X′ X
Therefore, the required area = area of the region OBMO + area of O M
the region BMAB.
Now, the area of the region OBMO
4 4 1
= y0 y dx = y0 x dx = [x 2] 04 = 8 sq. units ...(iii) Y′
2
Again, the area of the region BMAB
4 2
32 – x 2 dx = = x 32 – x 2 + # 32 # sin –1 G
4 2 4 2 1 1 x
= y4 y dx = y4
2 2 4 2 4

314 Xam idea Mathematics–XII

@Cbsebookshub - Join Us on Telegram


= d 4 2 # 0 + # 32 # sin –1 1 n – e
1 1 4 1 1 o
32 – 16 + # 32 # sin –1
2 2 2 2 2
= 8p – (8 + 4p) = 4p – 8. ...(iv)
Adding (iii) and (iv), we get the required area = 4p sq units.
6. Find the area of that part of the circle x2 + y2 = 16, which is exterior to the parabola y2 = 6x.
[CBSE (AI) 2008]
Sol. Given curves are Y
x
x2 + y2 = 16 …(i) =6
16
E y2
y2 = 6x …(ii) = B (2, 2√3)

y2
x2+
Obviously curve (i) is a circle having centre at (0, 0) and
radius 4 unit. While curve (ii) is right handed parabola X′ A(4, 0)
X
having vertex at (0, 0) and axis along +ve direction of (–4, 0) D O F

x-axis.
Required part is the shaded region.
C
Now, for intersection point of curve (i) & (ii)
x2 + 6x = 16 ⇒ x2 + 6x – 16 = 0 Y′

⇒ x2 + 8x – 2x – 16 = 0
⇒ x(x + 8) – 2 (x + 8) = 0
⇒ (x + 8) (x – 2) = 0

⇒ x = – 8 or 2
[ x = – 8 is not possible as y2 is +ve]
∴ x = 2 and y = ! 12 = ! 2 3

Hence, coordinate of B is (2, 2 3 ) .
 Shaded region i.e., required part is symmetrical about x-axis.

∴ Area of required part = 2 [area OBEDO]



= 2 [area DEBFD – area OBFO]

= 2 : y–4 16 – x 2 dx – y0 6x dxD
2 2

2
= 2 = sin – 1 G – 2 6 . [x 3/2] 02
x 16 x 2
16 – x 2 +
2 2 4 –4 3

= =d 2 # 12 + 16 sin –1 n – (– 4) # 0 + 16 sin –1 (– 1)G –


1 4 6
[2 2 – 0]
2 3

= d 8 nr + d 4 – n 3
8+ 16
3 3

= d 3 n sq units.
32 4
r–
3 3
7. Find the area of the region included between the parabola 4y = 3x2 and the line 3x – 2y + 12 = 0.
[CBSE (AI) 2009, (F) 2013]
3x 2
Sol. Given equation of parabola 4y = 3x 2 & y=
4
… (i)
3x + 12 =
and the line 3x – 2y + 12 = 0 ⇒ y … (ii)
2

Application of Integrals 315


@Cbsebookshub - Join Us on Telegram
The line intersect the parabola at (–2, 3) and (4, 12).
=0
12
Hence, the required area will be the shaded region. Y y+
–2
3x

3x 2 = 4y
4 4
3x + 12 3x 2
Required area = y dx – y dx 12 A(4,12)
–2 2 –2 4
10
4
x3
= = x 2 + 6x – G
3 8
4 4 –2 6
4
= (12 + 24 – 16) – (3 – 12 + 2) (–2,3)B
2
= 20 + 7 = 27 square units.
X' X
–4 –2 O 2 4

Y'

8. Using integration, find the area of the following region:


{(x, y) : | x – 1 | # y # 5 – x 2 } [CBSE Sample Paper 2017; Delhi 2010]
Y
Sol. We have provided {(x, y) : | x – 1 | # y # 5 – x 2 }

y=
3
Equation of curve is y = 5 – x 2 or y 2 + x 2 = 5 ,


x+
5

1
which is a circle with centre at (0, 0) and radius .

1
2

x–
2
2

x
5–

y=
Equation of line is y = |x – 1|

y=
1
Consider, y = x – 1 and y = 5 – x2
X‘ X
Eliminating y, we get x – 1 = 5 – x 2 –3 –2 P(–1,0) O 1 Q(2,0) 3

⇒ x2 + 1 – 2x = 5 – x2 ⇒ 2x2 – 2x – 4 = 0

Y‘
⇒ x2 – x – 2 = 0
⇒ (x – 2) (x – 1) = 0
⇒ x = 2, – 1

The required area is
2 1 2
= y–1 5 – x 2 dx – y-1 ( – x + 1) dx – y1 (x – 1) dx
2 1 2
x2 + x2
= = G –=– xG – = – xG
x 5 x
5 – x 2 + sin –1
2 2 5 –1 2 -1 2 1

1 o d –1 + + 1 + n d
= e 1 + sin –1 1 – sin –1 e – 1 – 2 – 2 – + 1n
5 2 o+ 5 1
– 1
2 5 2 5 2 2 2

2 – 2 – = sin – 1 = G–
5 e –1 2 + 1 o+ 1 5 2 1 + 1 4 1
= sin sin –1 1– 1–
2 5 5 2 2 5 5 5 5 2

sin d n –
5 = –1 4 + 1 G 1 = 5 1
= sin –1 (1) –
2 5 5 2 2 2

= d – n sq units.
5r 1
4 2
9. Find the area of the circle 4x2 + 4y2 = 9, which is interior to the parabola x2 = 4y.
[CBSE (AI) 2010]
Sol. Equation of circle and parabola is
9
4x 2 + 4y 2 = 9 & x2 + y2 =
4
…(i)
x2
x 2 = 4y & y=
4
…(ii)

316 Xam idea Mathematics–XII

@Cbsebookshub - Join Us on Telegram


Y

4y
3
Obviously given circle have centre at (0, 0) and radius , while

x 2=
2 A
– 2, 1 1
given parabola is upward parabola having vertex at (0, 0) and 2 B 2, 2

axis along +ve direction of y-axis. Shaded region is required


D
region which is symmetrical along y-axis. X X
E(– 2,0) O C( 2,0)
By putting value of equation (ii) in equation (i), we get
2
x2
4x 2 + 4 d n =9
4
Y
⇒ x4 + 16x2 – 36 = 0 ⇒ (x2 + 18) (x2 – 2) = 0

x2 + 18 = 0, x2 – 2 = 0 ⇒ x 2 = –18, x = ! 2

x = ! 2 (a x 2 = –18 is not possible i.e., intersecting points are (– 2 , 1/2), ( 2 , 1/2))
∴ Required area = 2 [area of OABCO – area of ODBCO]

x2
= 2 = y0 dxG
2 9 2
– x 2 dx – y0
4 4
2 2
= 2 *< 4
3

x 9 9
– x 2 + sin –1
x
F – =x G
2 4 8 3/2 0 12 0

= 2= G=e o sq units.
2 9 2 2 2 2 9 2 2
+ sin –1 – + sin –1
4 8 3 6 6 4 3
10. Using integration, find the area of the triangle ABC, co-ordinates of whose vertices are A(4, 1),
B(6, 6) and C(8, 4). [CBSE (AI) 2010, 2017]
Sol. Given triangle ABC, coordinates of whose vertices are Y
A(4, 1), B(6, 6) and C(8, 4).
7–
Equation of AB is given by B (6,6)
6–
6–1 5 5–
y – 6 = (x – 6) or y = x – 9
6–4 2 4– C (8,4)

Equation of BC is given by 3–
2–
4–6
y – 4 = (x – 8) or y = – x + 12 1– A (4,1)
8–6 D E F
X′ X






Equation of AC is given by 0 1 2 3 4 5 6 7 8

4–1 3 Y′
y – 4 = (x – 8) or y = x – 2
8–4 4
∴ Area of DABC = area of trap. DABE + area of trap. EBCF – area of trap. DACF

= y4 d x – 9 n dx + y6 (– x + 12) dx – y4 d x – 2 n dx
6 5 8 8 3

2 4
6 8 8
5 x2 x2 3 x2
= = G – 9 [x] 64 – = G + 12 [x] 86 – = G + 2 [x] 84

2 2 4 2 6 4 2 4
5 1 3

= (36 – 16) – 9 (6 – 4) – (64 – 36) + 12 (8 – 6) – (64 – 16) + 8
4 2 8
5 28 + 3

= # 20 – 18 – 24 – # 48 + 8
4 2 8
= 25 – 18 – 14 + 24 – 18 + 8 = 7 sq units.

Application of Integrals 317


@Cbsebookshub - Join Us on Telegram
11. Using integration find the area of the triangular region whose sides have equations y = 2x + 1,
y = 3x + 1 and x = 4. [CBSE Delhi 2008; 2011]

Sol. The given lines are

y = 2x + 1 …(i)
Y
y = 3x + 1 …(ii)
14 –
x = 4 …(iii) 13 – B(4,13)

1
x+
For intersection point of (i) and (iii) 12 –

2
11 –

y=
+1
y = 2 × 4 + 1 = 9 10 –

3x
9– C(4,9)

y=
Coordinates of intersecting point of (i) and (iii) is (4, 9) 8–
7–
For intersection point of (ii) and (iii) 6–

x=4
5 –
y = 3 × 4 + 1 = 13 4–
i.e., coordinates of intersection point of (ii) and (iii) is (4, 13)
3–
2–
For intersection point of (i) and (ii) 1 – A (0,1)
D(4,0)
X′ | | | | | |
X
2x + 1 = 3x + 1 ⇒ x = 0 O 1 2 3 4 5 6


y=1 Y′

i.e., coordinates of intersection point of (i) and (ii) is (0, 1).


Shaded region is required triangular region.
∴ Required area = area of trapezium OABD – area of trapezium OACD

2 4 2 4
= y0 (3x + 1) dx – y0 (2x + 1) dx = =3 + xG – = xG
4 x 4 2x +

2 0
2 0

= [(24 + 4) – 0] – [(16 + 4) – 0] = 28 – 20 = 8 sq units.


12. Find the area of the region bounded by the parabola y2 = 2x and the line x – y = 4. [CBSE (F) 2013]
Sol. Given curves are y2 = 2x ....(i) Y
and x – y = 4 ....(ii)
Obviously, curve (i) is right handed parabola having vertex 4
P(8,4)
2x
at (0, 0) and axis along +ve direction of x-axis while curve y2 = 4
y=
(ii) is a straight line. x–
For intersection point of curve (i) and (ii),we get X‘ X
O
(x – 4)2 = 2x –2 Q(2,–2)
⇒ x2 – 8x + 16 = 2x
⇒ x2 – 10x + 16 = 0

x2 – 8x – 2x + 16 = 0 ⇒ x(x – 8) –2(x – 8) = 0
⇒ (x – 8) (x – 2) = 0 ⇒ x = 2, 8
Y‘
⇒ y = –2, 4

Intersection points are (2, –2), (8, 4)
Therefore, required area = area of shaded region
4 4
4 y24 ( y + 4) 2 1 y
3
= y (y + 4) dy – y dy = > H – > H
–2 –2 2 2 –2
2 3 –2
1 1 72
= . [64 – 4] – [64 + 8] = 30 – = 18 sq units.
2 6 6

318 Xam idea Mathematics–XII

@Cbsebookshub - Join Us on Telegram


13. Using integration find the area of the region {(x, y) : x2 + y2 ≤ 2ax, y2 ≥ ax, x, y ≥ 0}.
[CBSE Delhi 2016]
Sol. Given region R is R = {(x, y) : x2 + y2 ≤ 2ax, y2 ≥ ax, x, y ≥ 0}.
⇒ R = R1 ∩ R2 ∩ R3, where R1 = {(x, y) : x2 + y2 ≤ 2ax},

a a
R2 = {(x, y) : y2 ≥ ax} and R3 = {(x, y) : x ≥ 0, y ≥ 0} A y2 = ax
Obviously, x2 + y2 = 2ax ⇒ (x – a)2 + (y – 0)2 = a2 is a
circle having centre at (a, 0) and radius r = a.
Therefore the region R1 ≡ {(x, y) : x2 + y2 ≤ 2ax} is the
region inside the circle with centre (a, 0) and radius a. O C
Also y2 = ax is right handed parabola with vertex at a
origin.
x2 y2 2ax
So, region R2 ≡ {(x, y) : y2 ≥ ax} is the region out side
parabola.
Also, R3 ≡ {(x, y) : x ≥ 0, y ≥ 0} is region in first quadrant.
Hence, R = R1 ∩ R2 ∩ R3 is the shaded region shown above in figure.
Now for co-ordinate of A, we solve y2 = ax and x2 + y2 = 2ax as follows
x2 + ax = 2ax [Putting y2 = ax]
x2 – ax = 0 ⇒ x(x – a) = 0 ⇒ x = 0, a
For x = a, y = a
Hence, co-ordinate of A is (a, a)
a a a a
∴ Required area = y0
2ax – x 2 dx – y0 ax dx = y0 – (x 2 – 2ax + a 2 – a 2) dx – a y0 x1/2 dx
a
x 3/2 2 a 3/2
– (x – a) + a dx – a > H = y0 a 2 – (x – a) 2 dx –
a 2 2 a
= y0 [a – 0]
3/2 0 3
a
1 1 x–a 2a 2
= > (x – a) a 2 – (x – a) 2 + a 2 sin –1 e oH –
2 2 a 0
3

2a 2 2a 2
= =0 – )0 + a e – o3G – = e – o a 2 sq units.
1 2 r r r 2
= a2 –
2 2 3 4 3 4 3

14. Find the area of the region {(x, y): x2 + y2 ≤ 1 ≤ x + y}. [CBSE (F) 2017] [HOTS]
Sol. The required area is the area between the circle Y

x2 + y2 = 1 …(i) B(0,1)
and line x + y = 1 …(ii) y = 1–x2
Circle (i) has centre (0, 0) and radius 1. Line (ii) meets x-axis at A(1,0)
X′ X
A(1, 0) and y-axis at B(0, 1). The circle (i) also passes through A O
and B. Hence, points of intersection of (i) and (ii) are A(1, 0) and
x+
y=

B(0, 1).
1

1 1
Required area = y0 1 – x 2 dx – y0 (1 – x) dx Y′

1 1
x. 1 – x 2 1 x2
=< + sin –1 xF – =x – G = sin –1 (1) –
1 1

2 2 0 2 0
2 2

e o – = e – o sq unit.
1 r 1 r 1
=
2 2 2 4 2

Application of Integrals 319


@Cbsebookshub - Join Us on Telegram
15. Using integration find the area of the triangle formed by positive x-axis and tangent and
normal to the circle x 2 + y 2 = 4 at (1, 3 ) . [CBSE Delhi 2015]
Sol. Given circle is x 2 + y 2 = 4
dy

2x + 2y = 0 [By differentiating]
dx
dy x

=–
dx y
dy
G
1
Now, slope of tangent at (1, 3 ) = =– .
dx (1, 3 ) 3
∴ Slope of normal at (1, 3 ) = 3

O
Therefore, equation of tangent is

y– 3 1
=–
x–1 3

x + 3 y = 4 … (i)
Again, equation of normal is
y– 3
= 3 ⇒ y – 3x = 0 …(ii)
x–1
To draw the graph of the triangle formed by the lines x-axis, (i) and (ii), we find the intersecting
points of these three lines which give vertices of required triangle. Let O, A, B be the intersecting
points of these lines.
Obviously, the coordinate of O, A and B are (0, 0), (1, 3 ) and (4, 0) respectively.
Required area = area of triangle OAB = area of region OAC + area of region CAB
1 4 4–x
= y0 y dx + y1 y dx [Here in 1st integrand y = 3 x and in 2nd y = ]
3
1 2 4
dx = 3 < F – = G
y0
1
y1
4 4–x x2 1 (4 – x)
= 3 xdx +
3 2 0 3 2 1

;0 – E =
3 1 9 3 9 12
= – + = = 2 3 sq units.
2 3 2 2 2 3 2 3

16. Using integration, find the area bounded by the tangent to the curve 4y = x2 at the point (2, 1)
and the lines whose equations are x = 2y and x = 3y – 3. [CBSE Sample Paper 2016]
Sol. Obviously 4y = x2 is upward parabola having vertex at origin.
Now 4y = x2
dy dy 1 dy
⇒ 4
= 2x & = x & G = 1 #2 = 1
dx dx 2 dx (2, 1) 2

⇒ Slope of tangent at (2, 1) to given curve 4y = x2 is 1.



y–1
Equation of tangent = =1
x–2
⇒ y – 1 = x – 2
⇒ y=x–1

320 Xam idea Mathematics–XII

@Cbsebookshub - Join Us on Telegram


Y
1
5
x2 = 4y x–
y=
nt
4
nge
Ta 2y
x=
3

)
B (6, 3)

,2
(3
C
2

1 A (2, 1)
–3
x = 3y
X' O 1 2 3 4 5 6 X
L M N

Y'

Now, for graph of x = 2y Also for graph of x = 3y – 3


x 0 2 x 0 3
y 0 1 y 1 2

After plotting the graph, we get shaded region ABC as required region, area of which is to be
calculated.
After solving the respective equation, we get
Coordinate of A ≡ (2, 1); B ≡ (6, 3); C ≡ (3, 2)
Now, the required area = area of shaded region ABC
= ar(region ALMC) + ar(region CMNB) – ar(region ALNB)
3 6 6
x+3 x
= y (x – 1) dx + y dx – y dx
2 3 3 2 2
3 6 6
x2 1 x2 1 x2
= > – xH + > + 3xH – > H
2 2
3 2 3
2 2 2

9 4 1 36 9 1
= >e – 3 o – e – 2 oH + >e + 18 o – e + 9 oH – (36 – 4)
2 2 3 2 2 4
3 1 27 3 45
= + e 36 – o–8= + – 8 = 1 square unit.
2 3 2 2 6
17. Prove that the curves y2 = 4x and x2 = 4y divide the area of the square bounded by x = 0, x = 4,
y = 4 and y = 0 into three equal parts. [CBSE Delhi 2009; (Central) 2016, NCERT Exemplar]
OR
Find the area of the region bounded by the curve y2 = 4x and x2 = 4y. [NCERT Exemplar]
Sol. We plot the curves y2 = 4x and x2 = 4y and also the various areas of the square.
To show that area of region I = II = III
4 4
Area of region I = y 4dx – y 2 x dx
0 0

Application of Integrals 321


@Cbsebookshub - Join Us on Telegram
4 Y
x 3/2
= =4x – 2 G x2=4 y

3/2 0
4 16

= 16 – #8 = square units y=4 y2 =4 x
3 3 4
4 4 2
x
Area of region II = 2 y x dx – y dx I
0 0 4 x= 4
II
3 4
H
2 x III
= 2. x 3/2 –
3 12 0 X′ X
O 4
4 64
= #8– –0
3 12
Y′
128 – 64 64 16
= = = square units
12 12 3
4 2 4
x3
dx = ; E =
x 64 = 16
Area of region III = y square units.
0 4 12 0 12 3
Thus, the curves y2 = 4x and x2 = 4y divide the area of given square into three equal parts.

18. Find the area of the region {(x, y) : y2 ≤ 6ax and x2 + y2 ≤ 16a2}, using method of integration.
[CBSE (AI) 2013; (South) 2016, NCERT Exemplar]
Sol. Corresponding curves of given region Y

{(x, y): y2 ≤ 6ax and x2 + y2 ≤ 16a2} are a2


16 B (2 a, 2 3 a)
x2 + y2 = 16a2 ...(i) y2
=
y2 = 6ax
x 2+

...(ii)

Obviously, curve (i) is a circle having centre (0, 0) and X
O A(4 a, 0)
X
radius 4a. While curve (ii) is right handed parabola
having vertex at (0, 0) and axis along +ve direction of
x-axis. y2 = 6ax
C
Obviously, shaded region OCAB is area represented by
y2 ≤ 6ax and x2 + y2 ≤ 16a2 Y

Now, we find intersection point of curve (i) and (ii).


x2 + 6ax = 16a2 [Putting the value of y2 in (i)]

⇒ x2 + 6ax – 16a2 = 0
⇒ x2 + 8ax – 2ax – 16a2 = 0

⇒ x(x + 8a) – 2a(x + 8a) = 0 ⇒ (x + 8a) (x – 2a) = 0


⇒ x = 2a, – 8a
⇒ x = 2a [∴ x = – 8a is not possible as y2 is +ve]


y = 2 3a
Since, shaded region is symmetrical about x-axis

Required area = 2 [area of OABO]

= 2 > y 16a 2 – x 2 dxH = 2 > 6a y (4a) 2 – x 2 dxH


2a 4a 2a 4a
6ax dx + y x dx + y
0 2a 0 2a
4a
= 2 6a # [x 2/3] 02a + 2 = 16a 2 – x 2 + 16a 2 sin –1 G
2 x 1 x
3 2 2 4a 2a

322 Xam idea Mathematics–XII

@Cbsebookshub - Join Us on Telegram


4a 2 r o
[(2a) 2/3 – 0] + 2 =(0 + 4a 2 r) – e 2 3 a 2 + G
4 6a
=
3 3

8a 2 12 8a 2 r
= + 8a 2 r – 4 3 a 2 –
3 3
16 16a 2 r 4 3 2 16a 2 r
= 3 a2 + – 4 3 a2 = a +
3 3 3 3
1
19. Sketch the graph y =|x + 1|. Evaluate y–3 x + 1 dx . What does this value represent on the
graph? [HOTS]

Sol. We have, y = | x + 1 | = ) x + 1, if x + 1 $ 0 i.e., x $ –1


Y
– (x + 1), if x + 1 < 0 i.e., x < –1

y=

1
x+
So, we have y = x + 1 for x ≥ – 1 and y = – x – 1 for x < – 1. Clearly, B

–(

y=
y = x + 1 is a straight line cutting x and y-axes at (–1, 0) and (0, 1) D

x+
x = –3

1)
respectively. So, y = x + 1, x ≥ –1 represents that portion of the

A(–1, 0)
line which lies on the right side of x = –1. Similarly, y = – x – 1,
x < – 1 represents that part of the line y = – x – 1 which is on the
X X
left side of x = –1. A rough sketch of y = |x + 1| is shown in fig. C(–3, 0) O E(1, 0)
1 –1 1
Now, y–3 x + 1 dx = y–3 – (x + 1) dx + y–1 (x + 1) dx Y
–1 1
(x + 1) 2 ( x + 1) 2
= – > H +> H = – =0 – G + = – 0G = 4 sq units
4 4
2 2 -
2 2
–3 1
This value represents the area of the shaded portion shown in figure.

PROFICIENCY EXERCISE
QQ Objective Type Questions: [1 mark each]
1.
Choose and write the correct option in each of the following questions.
(i) The area bounded by the curve y = x |x|, x-axis and the ordinates x = – 1 and x = 1 is given
by
1 2 4
(a) 0 sq. units (b) sq. unit (c) sq. unit (d) sq. units
3 3 3
(ii) The area bounded by the curve y = |sin x|, x-axis and ordinates x = p and x = 10p is equal to
(a) 8 sq. units (b) 10 sq. units (c) 18 sq. units (d) 20 sq. units
(iii) The area of the region bounded by the parabola y2 = x and the straight line 2y = x is
4 2 1
(a) sq. units (b) 1 sq. unit (c) sq. unit (d) sq. unit
3 3 3
(iv) The area of the circle x2 + y2 = 16 exterior to the parabola y2 = 6x is
(a) ^4r – 3 h sq. units (b) ^4r + 3 h sq. units
4 4
3 3
(c) ^8r – 3 h sq. units (d) ^8r + 3 h sq. units
4 4
3 3
(v) Area lying in the first quadrant and bounded by the circle x2 + y2 = 4 and the line x = 0 and
x = 2 is
r r r
(a) r sq. units (b) sq. units (c) sq. units (d) sq. units
2 3 4

Application of Integrals 323


@Cbsebookshub - Join Us on Telegram
2.
Fill in the blanks.
(i) The area of the region bounded by the curve y = x2 + x, x-axis and the line x = 2 and x = 5 is
equal to _____________ .
(ii) The area bounded by the curve y = ex, x-axis and ordinates x = 0 and x = 2 is _____________ .
(iii) The area bounded by the curves y = |x|, and x = – 1 and x = 1 is _____________ .

QQ Very Short Answer Questions: [1 mark each]


2
Find the area bounded by the curve y = x , x = 2, x = 3 and x-axis.
3.
Calculate the area under the curve y = 2 x included between the lines x = 0 and x = 1.
4.
Find the area under the curve y = x – 1 between the lines x = 1 and x = 5.
5.

QQ Long Answer Questions: [5 marks each]


Find the area bounded by the lines y = 4x + 5, y = 5 – x and 4y = x + 5.
6. [NCERT Exemplar]
2
Find the area bounded by the curve x = 4y and the straight line x = 4y – 2.
7.
 [CBSE Delhi 2010, 2013]
Using integration, find the area of the region {(x, y)} : 9x2 + y2 ≤ 36 and 3x + y ≥ 6. [CBSE (F) 2009]
8.
9. Find the area of the region {(x, y) : x2 ≤ y ≤ x}.
3 2
10. Find the area of the region bounded by the curve y = x and the line 3x – 2y + 12 = 0.
4
11. Using integration, find the area of the triangle ABC, where A is (2, 3), B is (4, 7) and C is (6, 2).
12. Make a rough sketch of the region given below and find its area, using integration:
{(x, y) : 0 ≤ y ≤ x2 + 3; 0 ≤ y ≤ 2x + 3, 0 < x ≤ 3}
13. Using integration, find the area of the triangle ABC, whose vertices have coordinates
A (2, 0), B (4, 5) and C (6, 3). [CBSE (F) 2012]
2 2
x y x y
14. F ind the area of the smaller region bounded by the ellipse + = 1 and the line + = 1 .
9 4 3 2
 [CBSE Delhi 2010; (F) 2014]
15. Using integration, find the area of the triangle formed by negative x-axis and tangent and normal
to the circle x 2 + y 2 = 9 at (–1, 2 2 ) . [CBSE East 2016]
a2
16. If the area bounded by the parabola y2 = 16ax and the line y = 4mx is sq units, then using
12
integration, find the value of m. [CBSE Ajmer 2015]
17. Using integration, find the area bounded by the curves y =|x – 1| and y = 3 –|x|.
 [CBSE Chennai 2015]
18. Using the method of integration, find the area of the triangular region whose vertices are (2, –2),
(4, 3) and (1, 2). [CBSE (North) 2016]

19. Using integration, find the area of the region bounded by the curves y = 4 – x 2 , x 2 + y 2 – 4x = 0
and the x-axis. [CBSE (F) 2016]
20. Find the area of the triangle whose vertices are (–1, 1), (0, 5) and (3, 2), using integration.
 [CBSE 2019 (65/3/1)]
21. Using integration, find the area of the triangle whose vertices are (2, 3), (3, 5) and (4, 4).
 [CBSE 2019 (65/1/3)]

324 Xam idea Mathematics–XII

@Cbsebookshub - Join Us on Telegram


22. Using integration, find the area of the following region: {(x, y) : x2 + y2 ≤ 16a2 and y2 ≤ 6ax}
 [CBSE 2019 (65/5/1)]
23. Using integration find the area of the region bounded between the two circles x2 + y2 = 9 and
(x – 3)2 + y2 = 9. [CBSE 2020 (65/5/1)]

Answers
1. (i) (c) (ii) (c) (iii) (a) (iv) (c) (v) (a)
297
2. (i) sq. units (ii) (e2 – 1) sq units (iii) 1 sq. unit
6
19 4 16 15 9
3. sq. units 4. sq. units 5. sq. units 6. sq. units 7. sq. units
3 3 3 2 8
1
8. 3 (p – 2) sq. units 9. sq. unit 10. 27 sq. units 11. 9 sq. units
6

sq. units 13. 7 sq. units 14. c – 3 m sq. units


50 3r
12. 15. 9 2 16. m = 2 2
3 2

sq. units 19. c – 3 m sq. units


13 4r 15 3
17. 4 sq. units 18. 20. sq. units 21. sq. units
2 3 2 2
4a �
^4r + 3 h 23. 2 c 3r – m sq. units
9 3
22.
3 4

SELF-ASSESSMENT TEST
Time allowed: 1 hour Max. marks: 30
1. Choose and write the correct option in the following questions. (3 × 1 = 3)

(i) The area of the region bounded by the curve y = 2x – x2 and the line y = x is
1 1 1 1
(a) sq. unit (b) sq. unit (c) sq. unit (d) sq. unit
6 4 3 2
(ii) Using integration, the area of the region bounded by the line 2y = 5x + 7, x-axis and the lines
x = 2 and x = 8 is
(a) 90 sq units (b) 96 sq units (c) 40 sq units (d) 10 sq units
(iii) The area of the parabola y2 = 4ax bounded by its latus rectum is
4a 2 8a 2 9a 2 8a 2
(a) sq units (b) sq units (c) sq units (d) sq units
3 3 4 5
2.
Fill in the blanks. (2 × 1 = 2)
2
(i) The area bounded by x = 4 – y and y-axis is _____________ .
(ii) The area between x-axis and the curve y = cos x when 0 ≤ x ≤ 2p, is _____________ .
QQ Solve the following questions. (2 × 1 = 2)
kx 3
3. If the area above x-axis, bounded by curves y = 2 , x = 0 and x = 2 is , then find the value
log e 2
of k.
4. Find the area common to parabola y = 2x2 and y = x2 + 4.

Application of Integrals 325


@Cbsebookshub - Join Us on Telegram
QQ Solve the following questions. (2 × 2 = 4)
5. Find the area of the region {(x, y) : x + y 2 # 1 # x + y} .
2

6. Find the area bounded by parabola y2 = x and straight line 2y = x.


QQ Solve the following questions. (3 × 3 = 9)
2 2 2 a
7. Find the area of the smaller part of the circle x + y = a cut off by the line x = .
2
8. Find the area lying above x-axis and included between the circle x2 + y2 = 8x and inside of the
parabola y2 = 4x. [CBSE Delhi 2008]
9. Find the area enclosed by the parabola y2 = x and line y + x = 2. [CBSE (AI) 2009]
QQ Solve the following question. (2 × 5 = 10)
10. Using integration, find the area of the triangle ABC with vertices as A(–1, 0), B(1, 3) and C(3, 2).
 [CBSE (F) 2009]
11. Using integration, find the area of the triangle ABC, where A is (2, 3), B is (4, 7) and C is (6, 2).

Answers
1. (i) (a) (ii) (b) (iii) (b)
32
2. (i) sq. units (ii) 4 sq. units
3
1 4
3. k = 1 4. 4 sq. units 5. (r – 1) sq. units 6. sq. units
2 3
a2 4 + 9
7. (r – 2) sq. units 8. (8 3r) sq. units 9. sq. units
4 3 2

10. 4 sq. units 11. 9 sq. units


zzz

326 Xam idea Mathematics–XII

@Cbsebookshub - Join Us on Telegram


Differential 10
Equations

1. Definition: An equation involving the independent variable x (say), dependent variable y (say)
and the differential coefficients of dependent variable with respect to independent variable i.e.,
dy d 2 y
, ,.... , etc. is called a differential equation.
dx dx 2
dy d 2 y dy
e.g.,
+ 4y = x, 2 –3 + 5y = x 2 are differential equations.
dx dx dx
2. Order and Degree of a Differential Equation: The order of a differential equation is the order of
the highest derivative occurring in the differential equation.
The degree of a differential equation is the degree of the highest order derivative occurring in the equation,
when the differential coefficients are made free from radicals, fractions and it is written as a polynomial
in differential co-efficient.
Example: Consider three differential equations:
3
d3 y d2 y d2 y d2 y
+ 2e o– (iii) e 2 o + sin c m = 0
dy dy dy
(i) + y = 0 (ii) 2 = 1 +
dx 3 dx 2 dx dx dx dx dx
Solution:
(i) In this equation, the highest order derivative is 3 and its power is 1. Therefore, its order is 3 and
degree 1.
(ii) In this equation, the differential co-efficient is not free from radical. Therefore, it is made free
from radical as
2
d2 y d2 y d 2 y dy
⇒ e 2 o +1– 2 2 –
dy
2 – 1 = = 0 [Squaring both sides]
dx dx dx dx dx

Hence, order is 2 and degree is 2.


(iii) In this equation order of highest order derivative is 2 therefore, its order is 2, but this differential
equation cannot be written in the form of polynomial in differential co-efficient.
Hence, its degree is not defined.
[Note : The order and degree of differential equations are always positive integers.]
3. Classification of Differential Equations:
(A) Differential equations are classified according to their order:
(i) First order differential equations: First order differential equations are those in which
only the first order derivative of the dependent variable occurs.

Differential Equations 327


@Cbsebookshub - Join Us on Telegram
(ii) Higher order differential equations: Differential equations of order two or more are
referred as higher order differential equations.
(B) Another classification of differential equations refers to its linearity means linear and non linear
differential equations:
Linear and non-linear differential equations: A differential equation, in which the dependent variable

and its derivatives occur only in the 1st degree and are not multiplied together, is called a linear differential
equation otherwise it is non linear.
Note: Every linear differential equation is always of the 1st degree but every differential equation of the 1st

degree need not be the linear differential equation.
4. Solution of a differential equation: The solution of a differential equation is a relation between
dependent and the independent variables which satisfies the given differential equation i.e., when
this relation is substituted in given differential equation, makes left hand and right hand sides
identically equal.
Note: If any relation contains n arbitrary constants, then the differential equation of nth order will be obtained

after eliminating all the arbitrary constants.
5. General and particular solutions of differential equations: The general solution of a differential
equation of nth order is a relation between dependent and independent variables having n arbitrary
constants.
The solution obtained from the general solution by giving the particular values to these arbitrary constants is

called the particular solution.
6. Forms of the solution of differential equations: The general solution may have more than one
forms but the arbitrary constants must be same in the number.
7. Formation of differential equations: By forming a differential equation from a given equation
representing family of curves, means finding a differential equation whose solution is the given
equation. If an equation, representing a family of curves contains n arbitrary constants, then
we differentiate the given equation n times to obtain n equations. Using all these equations, we
eliminate the arbitrary constants. The equation so obtained is the differential equation of the nth
order for the family of the given curves.
(i) If the given equation contains only one arbitrary constant then differentiate only one time and
eliminate the constant then differential equation of the first order is obtained.
(ii) If the differential equation contains two arbitrary constant then differentiate only two times and
eliminate the constants, then the differential equation of the second order is obtained.
8. Solution of differential equations: In this chapter, we shall only find the solutions of differential
equations viz. differential equations with variables separable form, homogeneous and linear
differential equations.
9. Type 1:
(A) Variables separable form: If in the given equation, it is possible to get all the terms containing
x and dx to one side and all the terms containing y and dy to the other, the variables are said to
be separable.
Procedure to solve the differential equations with variables separable form:
dy
Consider the equation = X.Y where X is a function of x only and Y is a function of y only.
dx
1
(i) Put the equation in the form . dy = X. dx
Y
(ii) Integrating both the sides, we get
dy
y = y X dx + C , where C is an arbitrary constant.
Y
Thus, the required solution is obtained.

328 Xam idea Mathematics–XII

@Cbsebookshub - Join Us on Telegram


dy
(B) Equations Reducible to Variables Separable Form: Equations of the form
= f ^ax + by + ch
dx
can be reduced to form in which the variables are separable form.
Procedure to solve an equation reducible to variables separable form:
dy
(i) Write the given equation in form = f ^ax + by + ch .
dx
dy 1 dz
(ii) Put ax + by + c = z, so that = c – am .
dx b dx
dy 1 dz
(iii) Putting this in the given equation, we get c – a m = f (z) . This equation is reduced
dx b dx
dz
in the form : = dx. After integrating, we get the required result.
a + b f^ z h

Type 2 : Homogeneous Function and Homogeneous Differential Equation


Homogeneous function : A function F(x, y) is called homogeneous function of degree n if
F(lx, ly) = ln F(x, y), where l is non-zero real number.
dy
A differential equation of the form = F ^x, yh is called homogeneous differential equation, if
dx
F(x, y) is a homogeneous function of degree zero, i.e., F(lx, ly) =l0 F(x, y).
Example : (x2 + xy)dy = (x2 + y2)dx

dy x 2 + y 2

= is homogeneous differential equation because
dx x 2 + xy
x2 + y2
Here, F ^x, yh =
x 2 + xy
m2 x2 + m2 y2 m2 _ x2 + y2 i

F ^ m x, my h = = = m 0 F ^ x, y h
m 2 x 2 + mx. my m 2 _ x 2 + xy i
Hence, F(x, y) is homogeneous function of degree zero.
dy x 2 + y 2
Therefore, = is a homogeneous differential equation.
dx x 2 + xy
To solve this type of equation we proceed as follows:
dy dv
(i) Suppose y = vx and so = v+x .
dx dx
dy dv
(ii) The value y = vx and = v+x is substituted in given equation. The equation reduces to
dx dx
variable separable form, which can be solved by integrating both sides.
y
(iii) Finally, v is replaced by to get the required solution.
x
dx
[Note : If the homogeneous differential equation is in the form = F ^x, yh then we substitute x = vy and
dy
dx dv
so = v+y and proceed as above.]
dy dy
Type 3: Linear Differential Equations Form: A linear differential equation is that in which the
dependent variable and its differential co-efficient occur in the first degree and not multiplied
together.
Thus, the standard form of a linear differential equation of the first order is
dy
+ Py = Q , where P and Q are functions of x or constants.
dx

Differential Equations 329


@Cbsebookshub - Join Us on Telegram
Now, we find a function F of x, by which we can multiply both sides of the given equation so that
the LHS becomes a complete differential. Such a function F is called the integrating factor (IF)

In this case IF = e
y Pdx
and solution is given by y e
y Pdx
= y _Q e y P dx i dx + C

10. Sometimes the Equation can be Made Linear Differential as Follows:


dx
+ P x = Q in which x is treated as dependent variable while y is treated as independent variable
dy
and P, Q are function of y or constant.
y P dy
In this case IF = e and solution is given by,
xe
y P dy
= y Q ^e y P dy h dy + C

Selected NCERT Questions


Verify that the given function is a solution of the corresponding differential equation (Q. 1 & 2):
y2
1. xy = log y + C : yl = (xy ! 1) .
1 – xy
Sol. The given function is xy = log y + C. Now,
dy 1 dy dy dy y2
e – xo
1
x.
dx
+y =
y dx
& y dx
=y & =
dx 1 – xy
which is given differential equation.
Thus, xy = log y + C is a solution of the given differential equation.
y – cos y = x : (y sin y + cos y + x) y′ = y.
2.
Sol. The given function is y – cos y = x.
dy dy dy 1
Now,
dx
+ sin y
dx
=1 = &
dx 1 + sin y
Putting values of y′ and x in given differential equation, we have
1 1
LHS = (y sin y + cos y + y – cos y) . = y (1 + sin y) . = y = RHS
(1 + sin y) (1 + sin y)
Thus, y – cos y = x is a solution of the given differential equation.
3. Form the differential equation of the family of circles touching the x-axis at origin. [HOTS]
Sol. Let C denote the family of circles touching x-axis at origin. Let Y
(0, a) be the coordinates of the centre of any member of the
family (shown in fig.). Therefore, equation of family C is
x2 + (y – a)2 = a2 or x2 + y2 = 2ay ...(i)
where, a is an arbitrary constant.
Differentiating both sides of equation (i) with respect to x, we get
X' X
dy dy dy dy O
2x + 2y = 2a or x+y =a
dx dx dx dx
Y'
dy
x+y
or a = dx ...(ii)
dy
dx

330 Xam idea Mathematics–XII

@Cbsebookshub - Join Us on Telegram


Substituting the value of a from equation (ii) in equation (i), we get

=x + y G
dy
dx dy 2 dy dy 2xy
x 2 + y 2 = 2y or (x + y 2) = 2xy + 2y 2 or =
dy dx dx dx x 2 – y 2
dx
This is the required differential equation of the given family of circles.
4. Form the differential equation of the family of hyperbolas having foci on x-axis and centre at
origin.
Sol. We know that the differential equation of the family of hyperbolas having foci on x-axis and
centre at origin is
x2 y2 2x 2y dy y dy b 2
2 – 2 = 1 & 2 – 2 =0 & =
a b a b dx x dx a 2
Again differentiating both sides w.r.t. x, we have
J dy N
y d 2 y dy KKK x – y OOO y d 2 y 1 dy 2 y dy
. 2 + KK dx OO = 0 & + d n– 2 =0
x dx dx K 2 O x dx 2 x dx x dx
L x P
d2 y dy 2 dy
⇒ xy
+xd n – yd n=0
dx 2 dx dx
which is the required differential equation.
5. Write the solution of the differential equation
(e x + e –x h dy = (e x – e –x) dx
e x – e –x
Sol. We have, dy = x –x .dx
e +e
Integrating both sides, we get
e x – e –x
y = y x –x dx = log | e x + e –x |+ C
e +e
6. Find the general solution of the following differential equation:
ex tan y dx + (1 – ex) sec2 y dy = 0 [CBSE Delhi 2011]
Sol. Given differential equation,
ex tan y dx + (1 – ex) sec2 y dy = 0 ⇒ ex tan y dx = – (1 – ex) sec2 y dy
ex tan y sec 2 y dy ex
∴ dy =
. dx & tan y
= x dx
e x – 1 sec 2 y e –1
Integrating both sides, we get
sec 2 y dy ex
y =y x dx & log | tan y | = log | e x – 1 |+ log C
tan y e –1
⇒ log |tan y|= log|(ex – 1) C|


tan y = (ex – 1) C
dy
7. For the differential equation xy = (x + 2) (y + 2) , find the solution curve passing through the
dx
point (1, –1).
dy y x+2
Sol. The given equation is xy
dx
= ( x + 2) ( y + 2) & dy =
x
dx
y+2

y e1 – o dy = y d 1 + n dx
y x+2 2
dy = y d n dx
2
⇒ y

+ x
& y+2 x
y 2

Differential Equations 331


@Cbsebookshub - Join Us on Telegram
⇒ y – 2 log | y + 2| = x + 2 log | x | + C
⇒ y = x + 2 log | x | + 2 log | y + 2 | + C

⇒ y = x + 2 log | x (y + 2) | + C

Since the line passes through the point (1, –1). So, putting x =1, y = –1.
We have, –1 = 1 + 2 log |1(–1 + 2) | + C ⇒ C = – 2
∴ y = x + 2 log | x (y + 2) | – 2, which is the required equation of the curve.

8. Show that the differential equation 2yex/y dx + (y – 2xex/y) dy = 0 is homogeneous and find its
particular solution, given that x = 0 when y = 1. [CBSE (North) 2016; (South) 2016; (F) 2017]
Sol. Given: 2y. ex/y dx + (y – 2xex/y) dy = 0
dx y – 2x e x/y dx 2x e x/y – y
&

dy
=– & dy
=
2y . e x/y 2y . e x/y
2x . e x/y – y
Let F ^x, yh) =
2y . e x/y
2mx.e mx/my – my 2x.e x/y – y
∴ F (mx, my) =
= m0 = m 0 . F (x, y)
2my .e mx/my 2y .e x/y
Hence, given differential equation is homogeneous.
dx 2xe x/y – y
Now, = …(i)
dy 2y . e x/y
dx dv
Let x = vy ⇒ = v+y
dy dy
vy
dv 2vy. e y – y
∴ (i) ⇒
v + y. = vy
dy
2y. e y

dv y ^2v e v – 1h dv 2v . e v – 1
&
y.
dy
= –v & y.
dy
= –v
2y . e v 2e v
dv 1
&
y.
dy
=– v & 2ye v dv = –dy
2e
x
dy
&
2 y e v dv = – y
y
& 2e v = – log y + C & 2e y + log y = C
When x = 0, y = 1
∴ 2e0 + log 1 = C or C = 2

Hence, the required solution is
2ex/y + log y = 2
9. Show that the given differential equation is homogeneous and solve it.
y y y y
) x cos d n + y sin d n3 ydx = ) y sin d n – x cos d n3 x dy.
x x x x

Sol. The given differential equation can be expressed as


y y y y y y
) x cos d
n + y sin d n3 y ) cos d n + sin d n3
dy x x x x x x
= = = f (x, y) (say)
dx y y y y y
) y sin d n – x cos d n3 x ) sin d n – cos d n3
x x x x x

332 Xam idea Mathematics–XII

@Cbsebookshub - Join Us on Telegram


my my my my
) cos d n+d n sin d n3d n
mx mx mx mx
Now f (λx, λy) = = m 0 f (x, y)
my my my
)d n sin d n – cos d n3
mx mx mx
Therefore, f (x, y) is a homogeneous function of degree zero. So, the given differential equation
is a homogeneous differential equation.
dy dv
Put y = vx so that =v+x
dx dx
dy
Putting values of and y in the given equation, we have
dx
dv " x cos v + vx sin v , vx dv (cos v + v sin v) v
v + x
dx
=
!vx sin v – x cos v + x
& x
dx
=
v sin v – cos v
–v

dv v cos v + v 2 sin v – v 2 sin v + v cos v dv 2v cos v


⇒ x

dx
=
v sin v – cos v
& x =
dx v sin v – cos v
v sin v – cos v 2 1 2
⇒ dv = dx & d tan v – n dv = dx
v cos v x v x
1 1
⇒ y d tan v –
n dv = 2 y dx & log ; sec v ; – log v = 2 log x + log C
v x
sec v
∴ log | sec v | – log | v | – log x2 = log C ⇒ log = log C
vx 2
y
sec d n y
sec v
sec d n = Cxy
x
⇒ =C & y 2
=C & x
vx 2
.x
x
which is the required solution.

10. Form the differential equation representing the family of ellipses foci on x-axis and centre at
the origin. [CBSE (F) 2010]
x2 y2
Sol. The family of ellipses having foci on x-axis and centre at the origin, is given by + =1
a2 b2
Differentiating with respect to x, we get
y
2y dy
2x 2y dy 2x
+ 2c m=0 & 2 dx = – 2
a 2
b dx b a
dy dy
y y c m –b 2
& dx = – x & dx = O
x
b2 a2 x a2
Again by differentiating with respect to x, we get
d2 y dy 2
x =y m G – c y. m
dy
+c
dx 2 dx dx
=0
x2
d2 y dy 2 dy
∴ The required equation is xy
+ xc m–y = 0.
dx 2 dx dx
11. Form the differential equation of the family of parabolas having vertex at the origin and axis
along positive y-axis. [CBSE Delhi 2011]
Sol. The equation of parabola having vertex at origin and axis along +ve y-axis is
x2 = 4ay …(i) , where a is parameter.

Differential Equations 333


@Cbsebookshub - Join Us on Telegram
Differentiating w.r.t. x we get, y
dy x dy
2x = 4a . i.e., x = 2ay' ⇒ a= where yl =
dx 2yl dx
x
Putting a = in (i), we get
2yl
x 2y x
x 2 = 4 . . y & yl = & xyl = 2y & xyl –2y = 0 O
2yl x
It is required differential equation.
dy
12. Solve: x + y – x + xy cot x = 0 (x ! 0) [CBSE (East) 2016]
dx
dy
Sol. The given differential equation x + y – x + xy cot x = 0 ^x ! 0h
dx
dy 1
& + c cot x + m y = 1 (Dividing both sides by x)  ...(i)
dx x
dy 1
This is a linear differential equation of the form + Py = Q, where P = cot x + and Q = 1 .
dx x
1
y c cot x + m dx log| sin x|+ log| x |
So, IF = e x =e

= e log x sin x = x sin x = x sin x (x sin x is always +ve for any x)


Multiplying both sides by IF in equation (i), we get
dy
+ x sin x e cot x + o y = x sin x
1
x sin x
dx x
dy d
⇒ x sin x

dx
+ (x cos x + sin x) y = x sin x & dx
(yx sin x) = x sin x  [By product rule]

On integrating both sides, we get


y ^x sin xh = y x sin x dx + C  ...(ii)

Let I = y x sin x dx = x× ^ – cos xh – y 1.^ – cos xh dx (Using by parts)


I = – x cos x + sin x
Putting the value of I in (ii), we get
y (x sin x) = – x cos x + sin x + C ⇒ y (x sin x) = sin x – x cos x + C
1 C
Hence, y = – cot x + is the required solution.
x x sin x
13. Find the particular solution of the differential equation
(1 + e2x)dy + (1 + y2) ex dx = 0 given that y = 1 when x = 0. [CBSE (F) 2011]
2x 2 x
Sol. We have, (1 + e ) dy + (1 + y ) e dx = 0 and given that y = 1, when x = 0
dy – _1 + y 2 i e x dy e x dx
= & =
– _1 + y 2 i 1 + e 2x
`
dx 1 + e 2x
Integrating both sides, we get
dy e x dx e x dx
– y = y & – tan –1 y = y
1 + y2 1 + e 2x 1 + (e x) 2
dt
⇒ – tan –1 y = y
[Putting e x = t & e x dx = dt]
1 + t2
⇒ – tan y = tan–1 (t) + C
–1
⇒ – tan–1 y = tan–1 (ex) + C ...(i)
Put x = 0, y = 1 in (i), we get

334 Xam idea Mathematics–XII

@Cbsebookshub - Join Us on Telegram


r r r
– tan–1 1 = tan–1 (e0) + C ⇒ – = +C
4 4
& C=–
2
Putting the value of C in (i), we get
– tan –1 y = tan –1 ^ e x h –
r r
2
& 2 = tan –1 (e x) + tan –1 y
r
Hence, tan–1(ex) + tan–1 y = is the required solution.
2
14. Show that the differential equation is homogeneous and solve it.

`1 + e y j dx + e y c 1– m dy = 0
x x x
y
x x
Sol. We have, ^1 + e x/yh dx + e x/y c1 – m dy = 0 ⇒ ^1 + e x/yh dx = – e x/y c1– m dy
y y
x
– e x/y c1 – m
dx y x
∴ = = g c m …(i)
dy ^1 + e h
x / y y
x
Here, RHS of differential equation is of the form g c m , so it is a homogeneous function of degree
y
zero.
dx dv
Now, we put x = vy and = v+y
dy dy
dv –e v ^1 – vh
From (i), we get v + y =
dy 1 + ev
dv –e v (1 – v) – (v + e v ) 1 + ev dy
⇒ y

dy
= – v = & dv =
y
1 + ev 1 + ev +
– (v e )v

On integrating both sides, we get


– log |v + ev|+ log C = log|y| ⇒ log C = log|y|+ log|v + ev|
x
⇒ log C = log | y ^v + e vh |= log |y c
+ e x/y m|
y
x
⇒ C = y c
+ e x/y m or C = x + ye x/y
y
Hence, x + yex/y = C is the required solution.
15. Solve the following differential equation:
dy r
+ 2y tan x = sin x, given that y = 0, when x = [CBSE (F) 2011, 2014]
dx 3
dy
Sol. Given differential equation is + 2 tan x . y = sin x .
dx
dy
Comparing it with + Py = Q, we get P = 2 tan x, Q = sin x
dx
y 2 tan xdx 2
`
IF = e = e 2 logsec x = e logsec x = sec 2 x [a e log z = z]

Hence, general solution is y. sec 2 x = y sin x. sec 2 x dx + C .


y. sec 2 x = y sec x. tan x dx + C & y. sec 2 x = sec x + C ⇒ y = cos x + C cos2 x
r r r
Putting y = 0 and x = , we get 0 = cos + C. cos 2
3 3 3
1 C
⇒ 0 =
+
2 4
& C = –2

∴ Required solution is y = cos x – 2 cos2 x.


Differential Equations 335


@Cbsebookshub - Join Us on Telegram
dy y 2 + y + 1
16. Show that the general solution of the differential equation + = 0 is given by
dx x 2 + x + 1
(x + y + 1) = A(1 – x – y – 2xy), where A is a parameter.
dy y 2 + y + 1
Sol. The given equation is + = 0.
dx x 2 + x + 1
dy – dx dy dx
∴ 2
= & y =– y
y + y + 1 x2 + x + 1 y2 + y + 1 x2 + x + 1
dy dx dy dx
⇒ y
=– y ⇒ y =–y
1 1 1 1 2 2
y 2 + y + + d 1– n x 2 + x + + d1– n
2
1 2
d n d n
1
d n d n
3 3
4 4 4 4 y + + x + +
2 2 2 2
JK 1 NO JK 1 NO
1 K y + OO 1 K x + OO

tan –1 KKK 2 OO = – tan –1 KKK 2 O+ C
3 /2 K 3 /2 O 3 / 2 K 3 /2 OO
L P L P
+ + 2y + 1 2x + 1
tan – 1 e o+ tan – 1 e o=C ⇒ >tan – 1 e o + tan –1 e oH = C
2 2 y 1 2 2 x 1 2


3 3 3 3 3 3 3
RS V
SS 2y + 1 + 2x + 1 WWW
SS WW 3 (2y + 1 + 2x + 1
tan –1 > H=C
2 3 3 2

tan –1 SS WW = C ⇒
2y + 1 2x + 1 WW
e o
SS
e o
3 3 3 – (2y + 1) (2x + 1)
S 1– W
S 3 3 W
T X
2 3 ( x + y + 1) 2 3 (x + y + 1 )
H = tan d Cn
3
⇒ tan –1 > H= ⇒ >
3
C
2 (1 – x – y – 2xy) 2 2 (1 – x – y – 2xy) 2
x+y+1
tan d C n = A
1 3

= ⇒ (x + y + 1) = A (1 – x – y – 2xy) .
1 – x – y – 2xy 3 2
17. Solve the differential equation ye x/y dx = (xe x/y + y 2) dy (y ! 0) .
Sol. The given equation is ye x/y dx = (xe x/y + y 2) dy.
dx xe x/y + y 2

= , is a homogeneous differential equation. ...(i)
dy ye x/y
dx dv

Put x = vy so that =v+y
dy dy
dv vye v + y 2 dv y (ve v + y) dv ve v + y
∴ v + y

dy
= & v+y
dy
= ⇒ y
dy
= –v
ye v ye v ev
dv ve + y – ve v
v
dv y
⇒ y

dy
= ⇒ y =
dy e v
& e v dv = dy
ev
⇒ y e v dv = y dy
& ev = y + C & e x/y = y + C
18. Find the particular solution of the differential equation:
dy r
+ y cot x = 4x cosec x (x ! 0), given that y = 0 when x = [CBSE (AI) 2012; (F) 2009, 2011]
dx 2
dy
Sol. Given differential equation is + y cot x = 4x cosec x .
dx
dy
It is of the type + Py = Q, where P = cot x, Q = 4x cosec x .
dx
y Pdx y cot xdx

` IF = e = e = e log|sin x| = sin x

336 Xam idea Mathematics–XII

@Cbsebookshub - Join Us on Telegram


Its solution is given by
sin x. y = y 4x cosec x.sin xdx
4x 2
⇒ y sin x = y 4xdx = +C ⇒ y sin x = 2x 2 + C … (i)
2
r
Now, put y = 0 when x = in (i), we get
2
r2 r2
∴ 0 = 2× + C ⇒ C=–
4 2
r2
Hence, the particular solution of given differential equation is y sin x = 2x 2 –
2

Multiple Choice Questions [1 mark]


Choose and write the correct option in the following questions.
3
d2 y dy
1. The degree of the differential equation x 2 = dx – y n is [CBSE 2020 (65/3/1)]
dx 2 dx
(a) 1 (b) 2 (c) 3 (d) 6
2 2 2
d y dy d y
2. The degree of the differential equation + 3d n = x 2 log f 2 p is  [NCERT Exemplar]
dx 2 dx dx
(a) 1 (b) 2 (c) 3 (d) Not defined
2 2
d2 y
3. The order and degree of differential equation >1 + d nH =
dy
respectively, are
dx dx 2
(a) 1, 2 (b) 2, 2 (c) 2, 1 (d) 4, 2
4. The order of the differential equation of all circles of given radius a is
(a) 1 (b) 2 (c) 3 (d) 4
dy
5. The solution of the differential equation 2x. – y = 3 represents a family of
dx
(a) straight lines (b) circles (c) parabolas (d) ellipses
dy
6. The integrating factor of the differential equation (x log x) + y = 2 log x is
 dx [NCERT Exemplar]
(a) e x (b) log x (c) log(log x) (d) x
dy 2 dy
7. A solution of the differential equation d n–x + y = 0 is [NCERT Exemplar]
dx dx
(a) y = 2 (b) y = 2x (c) y = 2x – 4 (d) y = 2x2 – 4
8. Which of the following is not a homogeneous function of x and y?
y y
(a) x 2 + 2xy (b) 2x – y (c) cos 2 d n + (d) sin x – cos y
x x
dx dy
9. Solution of the differential equation + = 0 is
x y
1 1
(a) + = c (b) log x. log y = c (c) xy = c (d) x + y = c
x y
dy
10. The solution of the differential equation x + 2y = x 2 is
dx
x2 + C x2 x4 + C x4 + C
(a) y = (b) y = + C (c) y = (d) y =
4x 2 4 x2 4x 2

Differential Equations 337


@Cbsebookshub - Join Us on Telegram
2
d2 y dy 2 dy
11. The degree of the differential equation f p +d n = x sin d n is
dx 2 dx dx
(a) 1 (b) 2 (c) 3 (d) Not defined
3/2
dy 2 d2 y
12. The degree of the differential equation >1 + d nH = is
dx dx 2
3
(a) 4 (b) (c) Not defined (d) 2
2
1
1
dy 4 d2 y
13. The order and degree of a differential equation +d n + x 5 = 0, respectively, are
dx 2 dx
(a) 2 and not defined (b) 2 and 2 (c) 2 and 3 (d) 3 and 3
14. If y = e –x (A cos x + B sin x) , then it is a solution of
d2 y dy d2 y d2 y
(a) +2 = 0 (b) –2 + 2y = 0
dx 2 dx dx 2 dx 2
d2 y dy d2 y
(c) +2 + 2y = 0 (d) + 2y = 0
dx 2 dx dx 2
15. Differential equation which has solution of the form y = A cos ax + B sin ax , where A and B
are arbitrary constants is
d2 y d2 y d2 y d2 y
(a) – a 2 y = 0 (b) + a 2 y = 0 (c) + ay = 0 (d) – ay = 0
dx 2 dx 2 dx 2 dx 2
dy
16. Integrating factor of x – y = x 4 – 3x is
dx
1
(a) x (b) log x (c) (d) –x
x
dy
– y = 1, y (0) = 1 is given by
17. Solution of
dx
(a) xy = –e x (b) xy = –e –x (c) xy = –1 (d) y = 2ex – 1
dy y + 1
18. The number of solution of = when y(1) = 2 is
dx x – 1
(a) none (b) one (c) two (d) infinite

19. Which of the following is a second order differential equation?
(a) (y') 2 + x = y 2 (b) y'' + y = sin x (c) y''' + (y'') 2 + y = 0 (d) y' = y 2
dy
20. Integrating factor of the differential equation (1 – x 2) – xy = 1 is
dx
x 1
(a) –x (b) (c) 1 – x 2 (d) log (1 – x 2)
1+x 2 2

Answers
1. (a) 2. (d) 3. (c) 4. (c) 5. (c) 6. (b)
7. (c) 8. (d) 9. (c) 10. (d) 11. (d) 12. (d)
13. (a) 14. (c) 15. (b) 16. (c) 17. (d) 18. (b)
19. (b) 20. (c)

338 Xam idea Mathematics–XII

@Cbsebookshub - Join Us on Telegram


Solutions of Selected Multiple Choice Questions
2. The given differential equation is not a polynomial equation in terms of its derivatives, so its
degree is not defined.
4. Let the equation of given family be (x – h)2 + (y – k)2 = a2. It has three arbitrary constants h, k and a.
Therefore, the order of the given differential equation will be 3.
y
2 2 x4 x4 + C
10. IF = e x dx = e 2 log x = e log x = x 2 . Therefore, the solution is y . x 2 = y x 2 . xdx = + k, i.e., y = ,
4 4x 2
where C = 4k.
15. Given, y = A cos ax + B sin ax
dy

= –aA sin ax + aB cos ax
dx
Again, differentiating both sides w.r.t. x, we get
d2 y d2 y
2
= –Aa 2 cos ax – a 2 B sin ax ⇒ 2
= –a 2 (A cos ax + B sin ax)
dx dx
d2 y d2 y
⇒ = –a 2 y ⇒ + a2 y = 0
dx 2 dx 2
dy
17. Given that, –y=1
dx
dy dy
= 1 + y ⇒
⇒ = dx
dx 1+y
On integrating both sides, we get log (1 + y) = x + C
When x = 0 and y = 1, then
log 2 = 0 + C ⇒ C = log 2
The required solution is log (1 + y) = x + log 2
1+y 1+y
⇒ log d n = x ⇒ = ex
2 2
⇒ 1 + y = 2ex ⇒ y = 2ex – 1
dy
20. Given that, (1 – x 2) – xy = 1
dx
dy x 1
⇒ – y= , which is a linear differential equation.
dx 1 – x 2
1 – x2
–x dt

` IF = e y 2
dx , Let 1 – x 2 = t & –2xdx = dt & – xdx = 2
1– x
1 dt 1 1
y log t log (1 – x 2)
e 2 t = e2 = e2 = 1 – x2

Fill in the Blanks [1 mark]


dy
1. The integrating factor of the differential equation x + 2y = x 2 is _____________ .
dx
 [CBSE 2020 (65/5/1)]
2. The number of arbitrary constants in the general solution of a differential equation of order
three is _____________ .
3. The solution of differential equation cot y dx = x dy is _____________ . [NCERT Exemplar]

Differential Equations 339


@Cbsebookshub - Join Us on Telegram
dy 2
4. The degree of the differential equation 1 + d n = x is _____________ . [CBSE 2020 (65/5/1)]
dx
d2 y dy 3
5. The order of the differential equation 3 – 5d n + 2y = 0 is _____________ .
dx 2 dx

Answers
1. x2 2. 3 3. x = C sec y 4. 2 5. 2

Solutions of Selected Fill in the Blanks


1. Given differential equation is
dy
x + 2y = x 2
dx
dy 2
⇒ + .y=x
dx x
2
y Pdx
` Integrating factor, IF = e
= e y x dx
2
  = e 2 log x = e log x = x 2
3. We have, cot y dx = x dy
dx = dy =

tan y dy
x cot y
dx = y
⇒ y
tan y dy   ⇒ log|x| = log|sec y| + log|C|
x
⇒ log|x|= log|C sec y|

⇒ x = C sec y

Very Short Answer Questions [1 mark]


dy
1. Find the general solution of the differential equation e y – x = 1.
dx
dy e y dy =
Sol. e y – x = 1   ⇒  1
dx e x dx
⇒ ey dy = ex dx

On integrating we have
y e y dx = y e x dx ⇒  ey = ex + C
⇒  y = log (ex + C)
2. Find the order and degree of differential equation:
d4 y d3 y
+ sin e o = 0 [NCERT Exemplar]
dx 4 dx 3
Sol. Order is 4 but degree is not defined because given differential equation cannot be written in the
form of polynomial in differential co-efficient.
3. Find the differential equation representing the curve y = cx + c 2 . [CBSE Patna 2015]
Sol. Given y = cx + c 2 ...(i)
dy dy

= c + 0 ⇒ = c [Differentiating with respect to x]
dx dx

340 Xam idea Mathematics–XII

@Cbsebookshub - Join Us on Telegram


Putting the value of c in eq n (i) , we get
2
dy dy 2 dy dy
y = x +d n ⇒ e o +x –y=0
dx dx dx dx
4. Find the differential equation representing the curve y = e –x + ax + b, where a and b are
arbitrary constants. [CBSE Guwahati 2015]
–x
Sol. Given curve is y = e + ax + b .
dy
⇒ = –e –x + a
[Differentiating with respect to x]
dx
d2 y

= e –x [Differentiating again with respect to x]
dx 2
A
5. Find the differential equation representing the family of curves v = r + B, where A and B are
arbitrary constants. [CBSE Delhi 2015]
A
Sol. Given family of curve is v = r + B .
dv –A
= [Differentiating with respect to r]
dr r2
d 2 v 2A d2 v 2 A
= 3 & = . 2
r r [Again differentiating with respect to r]
dr 2 r dr 2
d2 v 2 dv d2 v dv d2 v dv

= .d – n & r = –2   ⇒ r +2 =0
dr 2 r dr dr 2 dr dr 2 dr

6. Write the sum of the order and degree of the following differential equation:
dy 3
*e o 4 = 0
d
[CBSE Allahabad 2015]
dx dx
Sol. Given differential equation is
3
dy 2
d2 y
>e o H = 0
d dy
& 3e o. =0
dx dx dx dx 2
i.e., order = 2, degree = 1
  ∴ Required sum = 2 + 1 = 3.

Short Answer Questions-I [2 marks]


1. For a differential equation representing the faimily of curves y = A sin x, by eliminating the
arbitrary constant. [CBSE 2019 (65/4/3)]
Sol. We have, y = A sin x
y
⇒ =A
sin x
Differentiating with respect to x, we get
dy
sin x – y cos x
⇒ dx =0
sin 2 x
dy
sin x – y cos x = 0
dx
dy

sin x = y cos x
dx
dy

= y cot x
dx

Differential Equations 341


@Cbsebookshub - Join Us on Telegram
2. Find the differential equation of the family of curves represented by y2 = a(b2 – x2).
[CBSE 2019 (65/5/3)]
Sol. We have, y 2 = a (b 2 – x 2) = ab 2 – ax 2
Differentiating with respect to x, we get
dy
2y = –2ax
dx
dy y dy
⇒ y = –ax ... (i) ⇒ = –a ... (ii)
dx x dx
Again differentiating (i) with respect to x, we get
d2 y dy 2
y +d n = –a
dx 2 dx
Using (ii), we get
d2 y dy 2 y dy d2 y dy 2 dy
y +d n = ⇒ xy + xd n–y =0
dx 2 dx x dx dx 2 dx dx

3. Find the general solution of y 2 dx + (x 2 – xy + y 2) dy = 0. [NCERT Exemplar]


Sol. Given, differential equation is y 2 dx + (x 2 – xy + y 2) dy = 0 .
dx

y 2 dx = – (x 2 – xy + y 2) dy ⇒ y2 = – (x 2 – xy + y 2)
dy
x2 x
= –f 2 – + 1 p
dx

...(i)
dy y y
Which is a homogeneous differential equation.
x dx dv
Put = v or x = vy ⇒ = v+y
y dy dy
On substituting these values in equation (i), we get
dv dv
v + y = – [v 2 – v + 1] ⇒ y = –v 2 + v – 1 – v
dy dy
dv dv dy

y = –v 2 –1 ( 2 =–
dy v 1+ y
On integrating both sides, we get
tan –1 (v) = – log y + C

tan –1 e o + log y = C =a v = G
x x


y y
dx
4. Solve the differential equation (y + 3x 2) = x . [CBSE 2019 (65/5/2)]
dy
Sol. (y + 3x2)dx = xdy ⇒ ydx + 3x2dx = xdy
⇒ 3x2dx = xdy – ydx

= dc m
xdy – ydx y

3dx =2
x x
Integrating, we get
y
⇒ 3x = + C ⇒ 3x2 = y + Cx ⇒ y – 3x2 + Cx = 0.
x

342 Xam idea Mathematics–XII

@Cbsebookshub - Join Us on Telegram


5. Write the integrating factor of the following differential equation:
dy
(1 + y 2) + (2xy – cot y) = 0 [CBSE Allahabad 2015]
dx
dy
Sol. (1 + y 2) + (2xy – cot y) =0
dx
dy dy 1 + y2
⇒ (2xy – cot y)
dx
2
= – (1 + y ) & dx
=–
2xy – cot y
dx (2xy – cot y) dx 2y cot y


dy
=– & +
dy 1 + y 2
.x =
1 + y2 1 + y2
dx
It is in the form + Px = Q, where P and Q are function of y.
dy
2y 2
y p dy
⇒ IF = e
= e y 1 + y2 dy = e log | 1 + y |
= 1 + y2

Short Answer Questions-II [3 marks]


1. Solve the following differential equation: ^1 + e y/x h dy + e y/x d 1 –
y
n dx = 0, (x ! 0) .
x
[CBSE 2020 (65/2/1)]
Sol. Given differential equation

^1 + e y/xh dy + e y/x d1 –
y
n dx = 0
x

⇒ ^1 + e y/xh dy = d
y
n e y/x dx
x –1
y
d n e y/ x
dy x –1

=
dx ^1 + e y/xh
It is a homogeneous differential equation
dy dv
Put y = vx and = v+x
dx dx
We have,
dv (v – 1) v ve v – e v
v + x = e =
dx 1 + ev 1 + ev
v v
dv ve – e ve v – e v – v – ve v
⇒ x = v –v
=
dx 1+e 1 + ev
dv (v + e v)
⇒ x =–
dx 1 + ev
v
1+e dx
⇒ dv = – x
v + ev
On integrating both sides, we have
1 + ev dx
y dv = – y x
v + ev
⇒ log|v + ev| = – log|x| + log|C|
⇒ log|v + ev| + log|x| = log|C|

⇒ log|x (v + ev)| = log|C|

Differential Equations 343


@Cbsebookshub - Join Us on Telegram
y
⇒ x (v + e v) = C   ⇒  x d
+ y/x n = C
x e
⇒ y + x ey/x = C

dy
2. Find the particular solution of the differential equation log c m = 3x + 4y , given that y = 0
dx
when x = 0. [CBSE AI 2014]
dy
Sol. Given differential equation is log c m = 3x + 4y
dx
dy dy


dx
= e 3x + 4y & dx
= e 3x . e 4y

dy
⇒ 4y = e 3x .dx & e –4y dy = e 3x dx
e
Integrating both sides, we get
y e –4y dy = y e 3x dx
e –4y e 3x

= + C1 ⇒ –3e–4y = 4e3x + 12C1
–4 3
⇒ 4e3x + 3e–4y = – 12C1
⇒ 4e3x + 3e–4y = C … (i)
It is general solution.
Now for particular solution we put x = 0 and y = 0 in (i), we get
4 + 3 = C ⇒ C = 7
Putting C = 7 in (i), we get
4e3x + 3e–4y = 7
It is required particular solution.
3. Solve the following differential equation:
dy
2x 2 – 2xy + y 2 = 0 [CBSE Delhi 2012]
dx
dy
Sol. Given 2x 2 – 2xy + y 2 = 0
dx
dy dy 2xy – y 2
⇒ 2x 2

dx
= 2xy – y 2
dx
= & ...(i)
2x 2
It is homogeneous differential equation.
dy dv
Let y = vx & dx
= v+x
dx

Equation (i) becomes


dv 2x.vx – v 2 x 2
v + x =
dx 2x 2
v2
dv 2x 2 d v – n
dv v2
2
⇒ v + x

dx
= & x
dx
=v–
2
– v
2x 2
dv v2 dx 2dv
⇒ x

dx
=–
2
& x
= – 2
v
Integrating both sides, we get
dx dv
⇒ y = – 2y 2
x v

344 Xam idea Mathematics–XII

@Cbsebookshub - Join Us on Telegram


+
v –2 1 1
⇒ log | x |+ C = –2
& log | x |+ C = 2 .
v

–2 + 1
y 2x
Putting v = , we get log | x |+ C =
x y
4. Find the particular solution of the differential equation:
dy
x (x 2 – 1) = 1; y = 0 ; when x = 2 [CBSE (AI) 2012]
dx
Sol. Given differential equation is,
dy dx dx
x ^x 2 – 1h = 1 & dy = & dy = ^ h^ + h
dx ^ 2
x x –1 h x x – 1 x 1
Integrating both sides, we get,
dx dx
y dy = y
^ h
& y=y ...(i)
x ^x – 1h x 1
+ x ^ x – 1 h^ x + 1 h
1 A B C
Let = + +
x ^x – 1h^x + 1h x x –1 x+1

1 A ^x – 1h^x + 1h + Bx ^x + 1h + Cx ^x – 1h

=
x ^x – 1h^x + 1h x ^x – 1h^x + 1h
⇒ 1 = A(x – 1)(x + 1) + Bx(x + 1) + Cx(x – 1)

1
Putting x = 1, we get 1 = 0 + B. 1. 2 + 0 ⇒ B=
2
1
Putting x = –1, we get 1 = 0 + 0 + C .(–1).(–2) ⇒ C=
2
Putting x = 0, we get 1 = A (–1).1 ⇒ A = –1
1 –1 1 1
Hence, = + +
x ^x – 1h^x + 1h x 2 ^x – 1h 2 ^x + 1h
1 1 1
From (i) y = y d –
+ + n dx
x 2 ^x – 1h 2 ^x + 1h
dx 1 dx 1 dx 1 1
⇒ y = – y

x
+ y + y
2 x –1 2 x+1
& y = – log x + 2 log | x – 1 |+ 2 log | x + 1 |+ log C1
1 x2 – 1
⇒ 2y = 2 log
+ log | x 2 - 1 |+ 2 log C1 & 2y = log | |+ log C12 ...(ii)
x x2
When x = 2, y = 0
4–1 3
⇒ 0 = log|
4
|+ log C12 & log C12 = – log 4
3
Putting log C12 = – log in (ii), we get
4
x2 – 1 3 1 x2 – 1 1 3
2y = log| 2
| – log
4
& y =
2
log | 2
| – log
2 4
x x
dy
5. Solve the differential equation ^1 + x 2 h
–1
+ y = e tan x . [CBSE (AI) 2014]
dx
Sol. Given differential equation is
–1
dy dy 1 e tan x
^1 + x 2h
–1
+ y = e tan x ⇒ + y = …(i)
dx dx 1 + x 2 1 + x2
Equation (i) is of the form
–1
dy 1 e tan x
+ Py = Q, where P = , Q =
dx 1 + x2 1 + x2

Differential Equations 345


@Cbsebookshub - Join Us on Telegram
1 –1
y Pdx
∴ IF = e
= e y 1 + x2 dx = e tan x

Therefore, general solution of required differential equation is


–1 –1
tan –1 x –1
x e tan x tan –1 x e 2 tan x
y.e = y e tan . dx + C ⇒ y.e =y dx + C …(ii)
1 + x2 1 + x2
1
Let tan–1 x = z ⇒ dx = dz
1 + x2
(ii) becomes
–1 –1 e 2z
y.e tan x
= y e 2z dz + C & y.e tan x
=
2
+C
–1
x
–1
x e 2 tan
⇒ y.e tan
= + C [Putting z = tan–1 x]
2
–1
e tan x –1 –1
⇒ y = + C.e – tan x [Dividing both sides by e tan x
]
2
It is the required solution.
y
6. Find the particular solution of the differential equation e x 1 – y 2 dx + dy = 0 given that
x
y = 1 when x = 0. [CBSE Delhi 2014]
y
Sol. We have, e x 1 – y 2 dx + dy = 0
x
y y
⇒ e x 1 – y 2 dx = –

x
dy & xe x dx = – dy
1 – y2
y
⇒ y x e x dx = – y
dy
I II 1 – y2
1 dt
⇒ xe x – y e x dx =
y , where t = 1 – y2 (Using ILATE on LHS)
2 t
1 t1/2
⇒ xe x – e x =
e o+ C & xe x – e x = t + C
2 1/2
⇒ xe x – e x = 1 – y 2 + C , is the general solution.

y = 1 and x = 0, we get
Putting

0e 0 – e 0 = 1 – 1 2 + C & C = –1

Therefore, required particular solution is xe x – e x = 1 – y 2 – 1.


7. Solve the differential equation: [CBSE (AI) 2013; Delhi 2015]
–1 2
(tan y – x) dy = (1 + y ) dx
Sol. The given differential equation can be written as
dx x tan –1 y
+ = ...(i)
dy 1 + y 2 1 + y2
dx 1 tan –1 y
Now, (i) is of the form + Px = Q, where P = and Q =
dy 1 + y2 1 + y2
1 –1
y dy y
Therefore, IF = e 1 + y2 = e tan
Thus, the solution of the given differential equation is
tan –1 y tan –1 y
= yf pe
–1
y
xe tan dy + C ...(ii)
1 + y2

346 Xam idea Mathematics–XII

@Cbsebookshub - Join Us on Telegram


tan –1 y tan –1 y
Let I = y f pe dy
1 + y2
1
Substituting tan–1 y = t so that e o dy = dt , we get
1 + y2
I = y t et dt = t et – y 1. et dt = t et – et / et ^t – 1h

_tan –1 y – 1 i
–1
y
or I = e tan
Substituting the value of I in equation (ii), we get
_tan –1 y – 1 i + C
–1 –1
y y
x. e tan = e tan
–1
y
or x = (tan –1 y – 1) + C e – tan is the required solution.

8. Solve the following differential equation:


e –2 x
> H
y dx
– = 1, x ! 0 [CBSE (F) 2012]
x x dy
e –2 x
Sol. Given f p
y dx
– = 1, x ! 0
x x dy

dy e –2 x y dy 1 e –2 x


dx
= – & dx
+ .y=
x x x x

dy 1 e –2 x
It is in the form + Py = Q, where P = ,Q =
dx x x
–1
+1
x 2
1 1
1 –
– +1
∴ IF = e y P dx = e y = ey x
dx 2 dx x
x =e 2 = e2
Therefore general solution is
e –2 x 2 x
y .e 2 x
= y Q # IF dx + C & y .e 2 x
=y .e dx + C
x
1
– +1
dx x 2
⇒ y e 2
x
=y +C & y . e2 x
=
1
+C
x
– +1
2
x
⇒ y . e 2
= 2 x+C
dy 2
9. Solve the differential equation (x 2 – 1) + 2xy = 2 , where x ! (– 3, – 1) ' (1, 3) .
dx x –1
[CBSE Delhi 2014; (AI) 2010; (F) 2009, 2011]
dy 2
Sol. The given differential equation is ^x 2 – 1h + 2xy = 2
dx x –1
dy 2x 2
⇒ + y= 2 ...(i)
dx x 2 – 1 (x – 1) 2
dy 2x 2
This is a linear differential equation of the form + Py = Q, where P = 2 and Q =
dx ^x – 1h
2
x –1 2

= e y 2 x/ ^ x –1hdx
y P dx 2 2
∴ IF = e
= e log x –1 = x2 – 1
dy 2
Multiplying both sides of (i) by IF = x 2 – 1, we get ^x 2 – 1h + 2xy = 2
dx x –1

Differential Equations 347


@Cbsebookshub - Join Us on Telegram
Integrating both sides, we get
2
y ^x 2 – 1h = y 2 dx + C 6Using: y ^IFh = y Q .^IFh dx + C]
x –1
2 x–1 x–1
⇒ y ^x 2 – 1h = log +C & y ^x 2 – 1h = log +C
2 x+1 x+1
This is the required solution.
dy
10. Find the particular solution of the differential equation = 1 + x + y + xy given that y = 0
dx
when x = 1. [CBSE (AI) 2014]
dy
Sol. Given differential equation is = 1 + x + y + xy
dx
dy dy dy
⇒ = ^1 + xh + y ^1 + xh & = ^1 + xh^1 + yh & = ^1 + xh dx
dx dx 1+y
Integrating both sides, we get log | 1 + y |= y ^1 + xh dx
x2
⇒ log | 1 + y |= x +
+ C is the general solution.
2
Putting x = 1, y = 0, we get
1 3 –3
log 1 = 1 + + C & 0 = + C & C =
2 2 2
x2 3
Hence, particular solution is log | 1 + y | = x + – .
2 2
dy 2
11. Solve the differential equation x log x + y = log x . [CBSE (F) 2014]
dx x
dy 2
Sol. Given differential equation is x log x + y = log x
dx x
dy
+d n .y = 2
1 2

(Divide each term by x log x)
dx x . log x x
dy 1 2
It is in the form + Py = Q , where P = and Q = 2 .
dx x. log x x
dx
y Pdx
∴ IF = e
= e y x log x

dx 1
Put log x = z
x
& = dz = e y z dz = e log z = z = log x
∴ General solution is
2 log x
y. log x = y log x. 2 dx + C & y log x = 2 y 2 dx + C
x x
1
Let log x = z & dx = dz also log x = z ⇒ x = ez
x
z
` y log x = 2 y z dz + C & y log x = 2 y z.e – z dz + C
e
e– z e– z
& y log x = 2>z. –1 – y –1 dzH + C & y log x = 2 [–ze – z + y e – z dz] + C
⇒ y log x = –2ze– z – 2e– z + C
⇒ y log x = –2log x e– log x – 2e–log x + C
1 2 1
;a e – log x = e log x = E
1
⇒ y log x = – 2 log x.
– +C
x x x
2
⇒ y log x = –
^1 + logxh + C
x

348 Xam idea Mathematics–XII

@Cbsebookshub - Join Us on Telegram


sin a k + x – y sin a k = 0 is homogeneous. Find the
dy y y
12. Show that the differential equation x
dx x x
r
particular solution of this differential equation, given that x = 1 when y = . [CBSE Delhi 2013]
2
dy y y
Sol. Given differential equation is x sin + x – y sin = 0
dx x x
y
Dividing both sides by x sin , we get
x
dy y y dy y y
+ cosec – = 0 & = – cosec …(i)
dx x x dx x x
y y
Let F ^x, yh = – cosec
x x
my my y y
∴ F ^mx, myh = – cosec = m 0 : – cosec D = m 0 F ^x, yh
mx mx x x
Hence, differential equation (i) is homogeneous.
dy dv
Let y = vx & dx = v + x . dx
Now, equation (i) becomes
dv vx vx
v + x. = – cosec
dx x x
dv dv
v + x.
dx
= v – cosec v & x.
dx
= – cosec v

dx dx
⇒ – sin vdv =
x
& – y sin vdv = y
x
y
⇒ cos v = log | x |+ C & cos x = log | x |+ C ...(ii)
r
Putting y = , x = 1 in (ii), we get
2
r
` cos = log 1 + C
2
& 0 = 0+C & C = 0
Hence, particular solution is
y y
cos = log | x |+ 0 i.e., cos = log | x |
x x
13. Solve the differential equation:
dy
1 + x 2 + y 2 + x 2 y 2 + xy = 0 [CBSE (AI) 2010; (F) 2015]
dx
dy
Sol. Given 1 + x 2 + y 2 + x 2 y 2 + xy =0
dx
By simplifying the equation, we get
dy
xy = – 1 + x 2 + y 2 + x 2 y 2 = – 1 + x 2 + y 2 ^1 + x 2h
dx
= – ^1 + x 2h _1 + y 2 i = – ^1 + x 2h _1 + y 2 i
dy
&
xy
dx
y ^1 + x 2h
&
dy = – dx
_1 + y 2 i x
Integrating both sides, we get
y ^1 + x 2h
y dy = – y dx ...(i)
_1 + y 2 i x

Differential Equations 349


@Cbsebookshub - Join Us on Telegram
Let 1 + y2 = t ⇒ 2y dy = dt and 1 + x2 = m2 ⇒ 2x dx = 2m dm ⇒ x dx = m dm
1 1 m
∴ (i)
& 2
y dt = – y 2 . mdm
t m –1
1 t1/2 m2 m2 + 1 – 1
&

2 1/2
+y 2 dm = 0 & t +y dm = 0
m –1 m2 – 1
1 1 m–1
& t + y d1 + 2 n dm = 0 & t + m + log | |= 0
m –1 2 m+1
Now, substituting these value of t and m, we get
1 1 + x 2 –1
1 + y2 + 1 + x2 + log | |+ C = 0
2 1 + x2 + 1
14. (x 2 + y 2) dy = xy dx. If y (1) = 1 and y (x0) = e, then find the value of x0.
[CBSE Bhubneshwar 2015]
Sol. Given differential equation is (x 2 + y 2) dy = xy dx
It is also written as
dy xy
= …(i)
dx x 2 + y 2
Now, to solve let y = vx. [ (i) is a homogeneous equation]
Differentiating y = vx with respect to x, we get
dy xdy
= v+
dx dx
dy dy
Putting y = vx and = v+x in (i), we get
dx dx
dv x.vx
v + x =
dx x 2 + (vx) 2

dv vx 2 dv vx 2
v + x = 2
dx x + v 2 x 2
& v+x = 2
dx x (1 + v 2)

dv v dv v – v – v 3
& x =
dx (1 + v 2)
–v & x
dx
=
( 1 + v 2)
dv – v3 (1 + v 2) dv dx
& x =
dx (1 + v 2)
& 3
=–
x
v
Integrating both sides, we get
(1 + v 2) dv dx
y 3
= –y
x
v
dv dv 1
&
y 3
+y
v
= – log | x |+ C & – + log | v | = – log | x |+ C
v 2v 2
x2 y x2
&
– + log |
x
| = – log | x |+ C & – + log | y | – log | x | = – log | x |+ C
2y 2 2y 2
x2
&
+ log | y | = C
– … (ii)
2y 2
Given, x = 1, y = 1
1 1
& –
2×1
+ log | 1 | = C & –
2
=C [a log 1 = 0]

350 Xam idea Mathematics–XII

@Cbsebookshub - Join Us on Telegram


Now (ii) becomes
x2 1 x2 1 x2 – y2
– + log | y | = –
2
& log | y | = – & log | y | = ...(iii)
2y 2 2y 2 2 2y 2
Putting x = x0 and y = e in (iii), we get
x 02 – e 2 x 02 – e 2
log | e | = & 1= & x 02 – e 2 = 2e 2
2e 2 2e 2
& x 02 = 3e 2 & x0 = 3 e
15. Find the particular solution of the differential equation. [CBSE Bhubneshwar 2015]
dy r r
+ y tan x = 3x 2 + x 3 tan x, x ! , given that y = 0 when x = .
dx 2 3
dy
Sol. Given,   + y tan x = 3x 2 + x 3 tan x
dx
dy
&

dx
+ tan x.y = 3x 2 + x 3 tan x

dy

This is of the form + Py = Q, where P = tan x, Q = 3x 2 + x 3 tan x.
dx
y tan x dx

` IF = e = e log sec x = sec x
Therefore, general solution is given by
y. sec x = y (3x 2 + x 3 tan x) . sec x dx + C
&
y. sec x = y 3x 2 sec x dx + y x 3 tan x. sec x dx + C
&
y sec x = y 3x 2 sec x dx + x 3 . sec x – y 3x 2 . sec x dx + C
&
y sec x = x 3 sec x + C & y = x 3 + C cos x
r
Now x = , y = 0
3
r 3 r3 C 2r 3
0=c m + C. cos c m
r

`
3 3
& 0= +
27 2
& C=–
27
2r 3
Hence, required particular solution is y = x 3 – cos x.
27
dy
16. Show that the differential equation (x – y) = x + 2y is homogeneous and solve it.
dx
[CBSE (AI) 2010, 2017; (F) 2013; Ajmer 2015]
dy
Sol. Given, (x – y) = x + 2y
dx
By simplifying the above equation, we get
dy x + 2y
= …(i)
dx x–y
x + 2y
Let F ^x, yh =
x–y
mx + 2my m ^x + 2yh
then F ^mx, myh = = = m° F ^ x , y h
mx – my m ^x – yh
F(x, y) is homogeneous function and hence given differential equation is homogeneous.
dy dv
Now, let y = vx ⇒ = v+x
dx dx

Differential Equations 351


@Cbsebookshub - Join Us on Telegram
Substituting these values in equation (i), we get
dv x + 2vx
v + x =
dx x – vx
dv 1 + 2v 1 + 2v – v + v 2 1 + v + v 2
& x

dx
=
1– v
–v=
1– v
=
1– v
1– v dx
& dv = x
1 + v + v2
By integrating both sides, we get
1– v dx
y dv = y …(ii)
1+v+v 2 x
1– v
LHS = y 2
dv
v +v+1
Let 1 – v = A(2v +1) + B = 2Av + (A + B)
Comparing coefficients of both sides, we get
1 3
2A = –1, A + B = 1 or A=– , B=
2 2
1 3
1– v – ^2v + 1h +
∴ y 2 dv = y 2 2 dv
v +v+1 v2 + v + 1
1 2v + 1 3 dv
= – y 2 dv + y 2
2 v +v+1 2 v +v+1
1 2v + 1 3 dv
= – y 2 dv + y
2 v +v+1 2 1 2 3
cv + m +
2 4
JK 1 NO
1 3 2 KK v + OO
= – log | v 2 + v + 1 |+ × tan –1 KK 2 OO
2 2 3 KK 3 OO
K O
L 2 P
Now, substituting it in equation (ii), we get
2v + 1
log | v 2 + v + 1 |+ 3 tan –1 d n = log x + C
1

2 3

KJK 2y + 1 OON
y2 y
& – 2 log 2 + x + 1 + 3 tan –1 KKKK x OOOO = log x + C

1
x K 3 O
L P
2y + x
& – 12 log | x2 + xy + y2 |+ 12 log x2 + 3 tan –1 f
p = log x + C
3x
2y + x
log | x 2 + xy + y 2 |+ 3 tan –1 f p=C
1
& –

2 3x
dy
17. Solve = cos (x + y) + sin (x + y) . [NCERT Exemplar]
dx
dy
Sol. Given, = cos (x + y) + sin (x + y)
dx
dy dz
Put x + y = z ⇒ 1+ =
dx dx

352 Xam idea Mathematics–XII

@Cbsebookshub - Join Us on Telegram


On substituting these values in equation (i), we get
dz dz dz
d – 1 n = cos z + sin z ⇒ = (cos z + sin z + 1) ⇒ = dx
dx dx cos z + sin z + 1
On integrating both sides, we get
dz
y = y 1d x
cos z + sin z + 1
dz

y 2
= y dx
1– tan z/2 2 tan z/2
+ +1
1 + tan 2 z/2 1 + tan 2 z/2
dz

y = y dx
1– tan 2 z/2 + 2 tan z/2 + 1 + tan 2 z/2
(1 + tan 2 z/2)
(1 + tan 2 z/2) dz sec 2 z/2 dz

y = y dx ⇒ y = y dx
2 + 2 tan z/2 2 (1 + tan z/2)
1
Put 1 + tan z/2 = t ⇒ d sec 2 z/2 n dz = dt
2
dt

y = y dx ⇒ log | t | = x + C
t
(x + y)

log | 1 + tan z/2 | = x + C ⇒ log 1 + tan = x+C
2
18. Find the equation of the curve through the point (1, 0), if the slope of the tangent to the curve
y–1
at any point (x, y) is 2 . [NCERT Exemplar]
x +x
y–1
Sol. It is given that, slope of tangent to the curve at any point (x, y) is 2 .
x +x
dy y–1
` d n = 2
dx (x, y) x + x
dy y–1 dy dx

= 2
⇒ = 2
dx x + x y –1 x +x
dy dx
On integrating both sides, we get y =y 2
y–1 x +x
1 1
= yd – n dx
dy dx dy
⇒ y =y ⇒ y
y–1 x (x + 1) y–1 x x+1
⇒ log (y – 1) = log x – log (x + 1) + log C
xC

log (y – 1) = log d
n
x+1
Since, the given curve passes through point (1, 0).
1. C

` 0–1= ( C = –2
1+1
–2x
The particular solution is y–1= ⇒ (y – 1) (x + 1) = –2x ⇒ (y – 1) (x + 1) + 2x = 0 .
x+1
19. Find the particular solution of the differential equation:
(1 – y2)(1 + log x) dx + 2xy dy = 0 given that y = 0 when x = 1 [CBSE Delhi 2016]
2
Sol. We have (1 – y )(1 + log x) dx + 2xy dy = 0
2y dy ^1 + log xh dx
⇒ 2xy dy = – (1 – y2)(1 + log x) dx
& 2
=–
x
1– y

Differential Equations 353


@Cbsebookshub - Join Us on Telegram
Integrating both sides, we get
2y ^1 + log xh ^1 + log xh
&
y 2
dy = – y
x
dx & – log | 1 – y 2 | = – y
x
dx
1– y

& – log | 1 – y2 | = – y zdz =Let 1 + log x = z dx = dzG


1
& x
z2 (1 + log x) 2
&
log | 1 – y 2 | =
2
+C & log | 1 – y 2 | =
2
+C

Putting x = 1 and y = 0, we get


(1 + log 1) 2 1 1
& log 1 =

2
+C & 0=
2
+C & C=–
2
^1 + log xh
2
1 2
Hence, particular solution is log | 1 – y |= – .
2 2
20. Find the general solution of the following differential equation:
–1 dy
(1 + y 2) + (x – e tan y) = 0 [CBSE Delhi 2016]
dx

Sol. We have _1 + y 2 i + _ x – e tan y i dy


–1
=0
dx

_ x – e tan y i dy
= – _1 + y 2 i
–1
&

dx
1 + y2
–1

=–f p
y
=–f –1 p
dy dx x – e tan
&

dx
& dy
x – e tan y 1 + y2
–1 –1
dx x e tan y dx 1 e tan y
&

dy
=– + & +
dy 1 + y 2
x =
1 + y2 1 + y2 1 + y2
–1
dx 1 e tan y
It is in the form + Px = Q , where P = and Q = .
dy 1+y 2
1 + y2

y P. dy 1 –1
` IF = e
= e y 1 + y2 dy = e tan y

–1
tan –1 y e tan y tan –1 y
Therefore, general solution is x . e =y .e dy + C .
1 + y2
1
>Let tan y = z & dy = dzH
–1
&
x . e tan y
= y e z . e z dz + C –1
1 + y2
–1
&
x . e tan y
= y e 2z dz + C
–1
tan –1 y e 2z tan –1 y e 2 tan y
&
x.e =
2
+C & x.e =
2
+C

1 tan –1 y –1
&
x=
2
e + C . e – tan y

dy x + y cos x
21. Find the particular solution of differential equation: =– given that y = 1 when
dx 1 + sin x
x = 0. [CBSE (North) 2016]
Sol. We have
dy x + y cos x
=–
dx 1 + sin x

354 Xam idea Mathematics–XII

@Cbsebookshub - Join Us on Telegram


dy x y cos x dy cos x x
&

dx
=– – & +
dx 1 + sin x
y=–
1 sin x 1 + sin x
+ +
1 sin x
dy cos x x
It is in the form + Py = Q , where P = ,Q = – .
dx 1 + sin x 1 + sin x
cos x
y
Now IF = e 1 + sin x
dx
= e log| 1 + sin x| = 1 + sin x
Therefore, general solution is
x
y (1 + sin x) = y – ^1 + sin xh dx + C = – y x dx + C
1 + sin x
x2
⇒ y (1 + sin x) = –
+C
2
1(1 + sin 0) = 0 + C ⇒ C = 1 [Given y = 1 and x = 0]
Hence, particular solution is
x2 2 – x2
y (1 + sin x) = – + 1 ⇒ y=
2 2 ^1 + sin xh
22. Solve the following differential equation :
(cot–1 y + x)dy = (1 + y2)dx [CBSE (F) 2016]
Sol. We have (cot–1 y + x)dy = (1 + y2)dx
This can be written as
dx cot –1 y + x cot –1 y x dx 1 cot –1 y

= = + ⇒ – . x =
dy 1+y 2
1+y 2
1 + y2 dy 1 + y 2 1 + y2
dx –1 cot –1 y
It is of the form + Px = Q, where P = and Q = .
dy 1+y 2
1 + y2
1 –1
y– dy y

` IF = e 1 + y2 = e cot
Therefore, required solution of differential equation is
–1 cot –1 y cot –1 y –1
x.e cot y
=y .e dy + C & x.e cot y
= I + C … (i)
1 + y2
cot –1 y cot –1 y
Here, I = y .e dy
1 + y2
1 1
cot
Let –1
y=t ⇒ – dy = dt & dy = – dt
1 + y2 1 + y2
–1
& I = – y t . et dt = –[t.et – y et dt] = – t . et + et
= et (1 – t) = e cot y
(1 – cot –1 y)
–1 –1
y y
Hence, required solution is x.e cot = e cot (1– cot –1 y) + C. [From equation (i)]
–1
& x = (1 – cot –1 y) + Ce – cot
y

23. Form the differential equation of the family of circles in the second quadrant and touching the
coordinate axes. [CBSE (AI) 2012; (Central) 2016]
Sol. Let C denotes the family of circles in the second quadrant and touching the coordinate axes. Let
(–a, a) be the coordinate of the centre of any member of this family (see figure).
Equation representing the family C is
(x + a)2 + (y – a)2 = a2 ...(i)
or x2 + y2 + 2ax – 2ay + a2 = 0 ...(ii)

Differential Equations 355


@Cbsebookshub - Join Us on Telegram
Differentiating equation (ii) with respect to x, we get Y
dy dy dy dy
2x + 2y + 2a – 2a = 0 & x+y = ac – 1m
dx dx dx dx (–a, a)
x + yyl dy
or a= c a yl = m
yl – 1 dx X'
O
X

Substituting the value of a in equation (i), we get


2 2 2
x + yyl x + yyl x + yyl
=x + G + =y – G == G
yl – 1 yl – 1 yl – 1
Y'
⇒ [xy' – x + x +yy']2 + [yy' – y – x – yy']2 = [x + yy']2

or (x + y)2 y'2 + (x + y)2 = (x + yy')2 ⇒ (x + y)2 [(y')2 + 1] = [x + yy']2, is the required differential
equation representing the given family of circles.
24. Find the general solution of the following differential equation: [CBSE Delhi 2017 (C)]

x cosa x k dx = y cosa x k+ x
y dy y

Sol. Given differential equation is x cos a k = y cos a k + x


y dy y
x dx x
dy y cos (y/x) + x
= … (i)
dx x cos (y/x)
It is homogeneous differential equation.
dy dv
Let y = vx ⇒ = v+x
dx dx
dv vx cos v x +
(i)   ⇒ v+x =
dx x .cos v
dv v cos v + 1 dv v cos v + 1 – v cos v
  ⇒ x = – v ⇒ x =
dx cos v dx cos v
dv 1 dx
  ⇒ x dx = cos v ⇒ cosvdv =
x
Integrating both sides
y
  ⇒ sin v = log|x| + C ⇒ sin = log |x| + C is the required solution.
x

Long Answer Questions [5 marks]


1. Solve the following differential equation:
r
3ex tan y dx + (2 – ex) sec2 y dy = 0, given that when x = 0, y =
[CBSE (F) 2012]
4
Sol. Given, 3ex tan y dx + (2 – ex) sec2 y dy = 0
⇒ (2 – ex) sec2 y dy = – 3ex tan y dx

sec 2 y –3e x sec 2 y dy – e x dx
&

tan y
dy = dx & y
tan y
= 3y
2 – ex 2 – ex
⇒ log |tan y|= 3 log |2 – ex| + log C

⇒ log |tan y| = log |C. (2 – ex)3| ⇒ tan y = C (2 – ex)3
r
Putting x = 0, y = , we get
4
tan = C ^2 – e 0h & 1 = C ^2 – 1 h 3 & 1 = C
r 3
& 4
Therefore, particular solution is tan y = (2 – ex)3.

356 Xam idea Mathematics–XII

@Cbsebookshub - Join Us on Telegram


2. Solve: x dy – y dx = x 2 + y 2 dx [CBSE (AI) 2011]

Sol. The given differential equation can be written as


dy x2 + y2 + y
= ,x!0
dx x
Clearly, it is a homogeneous differential equation.
dy dv
Putting y = vx and = v+x in it, we get
dx dx
dv x 2 + v 2 x 2 + vx dv
v+x
dx
=
x
& v+x
dx
= 1 + v2 + v

dv dv dx
&
x
dx
= 1 + v2 & 2
=
x
1+v
Integrating both sides, we get
1 1
y
2
dv = y dx
x
& log | v + 1 + v 2 |= log | x |+ log C
1+v
y y2
&
| v + 1 + v 2 |=| Cx | & x
+ 1+ =| Cx | [a v = y/x]
x2
# y + x2 + y2 - = C2 x2 
2
&
[Squaring both sides]

Hence, # y + x 2 + y 2 - = C 2 x 2 gives the required solution.


2

3. Show that the differential equation (xey/x + y) dx = x dy is homogeneous. Find the particular
solution of this differential equation, given that x = 1 when y = 1. [CBSE Delhi 2013]
y

` x.e x + y j dx = xdy
y
dy x. e x + y
Sol. Given differential equation is & dx
=
x
…(i)

y my y
x. e x + y mx. e mx + my x. e x + y
Let F ^x, yh = & F ^mx, myh = = m0 = m 0 F ^x, yh
x mx x
Hence, given differential equation (i) is homogeneous.
dy dv
Let y = vx ⇒ = v + x.
dx dx
Now, given differential equation (i) would become
vx
dv x.e x + vx dv dv
v + x
dx
=
x
& v + x.
dx
= ev + v & x.
dx
= ev

dv dx dv e –v
⇒ v =
x
& y e –v dv = y
v
& –1
= log x + C
e
y y y
– 1
⇒ –e x = log x + C & – y = log x + C & e x . log x + Ce x + 1 = 0
ex
Putting x = 1, y = 1, we get
1
∴ e log 1 + Ce + 1 = 0
⇒ C=–
e
∴ The required particular solution is

y y y
1 xy –1
e x . log x – e +1 = 0 or e x log x – e x +1 = 0
e

Differential Equations 357


@Cbsebookshub - Join Us on Telegram
4. Show that the differential equation :x sin 2 a k – yD dx + x dy = 0 is homogeneous. Find the
y

x
r
particular solution of this differential equation, given that y = when x = 1. [CBSE (AI) 2013]
4

dy y – x sin a x k
2 y

Sol. Given differential equation is :x sin a k – yD dx + x dy = 0


2 y
& dx = ...(i)
x x
y – x sin 2 a k
y
Let F ^x, yh = x
x
my y
my – mx sin 2 y – x sin 2
Then F ^mx, myh = m x =m 0 x = m 0 F ^ x, y h
mx x
Hence, differential equation (i) is homogeneous.
dy dv
Now, let y = vx ⇒ = v+x
dx dx
Putting these value in (i), we get
2 vx
dv vx – x sin x dv x " v – sin 2 v ,
v + x
dx
=
x
& v+x
dx
=
x
dv dv dv dx
& v + x dx = v – sin2 v & x dx = – sin2 v & =–
x
sin 2 v
Integrating both sides, we get

& – cot v = – log x + C & log x – cot a x k = C …(ii)


1 y
& y cosec 2 vdv = – y dx
x
r
Putting y = and x = 1 in (ii), we get
4
r
log 1 – cot = C
4
& 0–1=C & C=–1
Hence, particular solution is
log x – cot a k = – 1 & log x – cot a x k + 1 = 0
y y
x
5. Find the differential equation of the family of curves (x – h) 2 + (y – k) 2 = r 2, where h and k are
arbitrary constants. [CBSE Ajmer 2015]
2 2 2
Sol. Given family of curve is (x – h) + (y – k) = r . …(i)
Differentiating with respect to x, we get
dy dy x–h
& 2 (x – h) + 2 (y – k) . dx = 0 & dx
=–
y–k
…(ii)

Differentiating again with respect to x, we get


Z] dy _bb Z] _
]] ]] (y – k) + (x – h) . x – h bbb
d2 y ] (y – k) – (x – h) . b
b ]
= – [] dx `b = – [] y – k b` [From (ii)]
2 ] b ] bb
dx ] 2 b ] 2 b
] (y – k) b ] (y – k ) b
2 \ 2 2
a \ a
= –* 4=–
d y ( y – k ) + ( x – h) r 2
& 2
[From (i)] …(iii)
dx (y – k ) 3 (y – k ) 3
2 2
dy x–h
From (ii) e o =e o
dx y–k
2
dy (x – h) 2
&
e o =
dx ( y – k) 2

358 Xam idea Mathematics–XII

@Cbsebookshub - Join Us on Telegram


Adding 1 both the sides, we get
dy 2
(x – h) 2 (x – h) 2 + (y – k) 2
&
e o +1 = +1 =
dx ( y – k) 2 ( y – k) 2
3
Putting exponent (power) both sides, we get
2
3 3
2
r2
>e o + 1H = > H
dy 2 2
r3
& =
dx (y – k ) 2 (y – k ) 3
3
2 d2 y
>e o + 1H = r.
dy 2
r2
&
= –r [Using (iii)]
dx (y – k ) 3 dx 2
3/2
d2 y 2
+ >e o + 1H
dy
& r =0
dx 2 dx
dy x (2 log x + 1)
6. Find the particular solution of the differential equation = given that
dx sin y + y cos y
r
y= when x = 1. [CBSE Delhi 2014] [HOTS]
2
dy x `2 log x + 1 j
Sol. Given differential equation is =
dx sin y + y cos y
⇒ (sin y + y cos y) dy = x (2 log x + 1)dx

&
y sin y dy + y y cos y dy = 2 y x log x dx + y x dx
x2 1 x2
&
y sin y dy + 7y sin y – y sin y dyA = 2 <log x – y . dxF + y x dx
2 x 2
&
y sin y dy + y sin y – y sin y dy = x 2 log x – y x dx + y x dx + C
⇒ y sin y = x2 log x + C, is general solution. … (i)
r
For particular solution, we put y = when x = 1
2
6a log 1 = 0@
r r r
(i) becomes sin = 1. log 1 + C ⇒ =C
2 2 2
Putting the value of C in (i), we get the required particular solution
r
y sin y = x 2 log x +
2
7. Show that the family of curves for which the slope of the tangent at any point (x, y) on it is
x2 + y2
, is given by x2 – y2 = Cx. [CBSE Delhi 2017] [HOTS]
2xy
dy
Sol. We know that the slope of the tangent at any point on a curve is .
dx
y2
2 2 1+
dy x + y x 2 ...(i) dy
Therefore,
dx
=
2xy 2y
& dx
=

x
Clearly, equation (i) is a homogeneous differential equation. To solve it we make substitution.
dy dv
y = vx & dx
= v+x
dx

Differential Equations 359


@Cbsebookshub - Join Us on Telegram
dy
Putting the value of y and in equation (i), we get
dx
dv 1 + v 2 dv 1 – v 2
v + x
dx
=
2v
& x
dx
=
2v
2v dx 2v dx

2
dv =
x
& 2
dv = –
x
1– v v –1
Integrating both sides, we get
2v 1
y dv = – y x dx & log | v 2 – 1 |= – log | x |+ log | C1 |
v2 – 1
⇒ log |(v2 – 1) (x)| = log |C1| ⇒ (v2 – 1) x = ± C1
y
Replacing v by , we get
x
y2
e – 1 o x = ± C1 & _ y 2 – x 2 i = ± C1 x & x 2 – y 2 = Cx ^where ± C1 = Ch
x2

PROFICIENCY EXERCISE
QQ Objective Type Questions: [1 mark each]
1. Choose and write the correct option in each of the following questions.
2
dy 3 d2 y
(i) The degree of the differential equation d 1 + n =e o is [NCERT Exemplar]
dx dx
(a) 1 (b) 2 (c) 3 (d) 4
4 4
d y dy
(ii) The order and degree of the differential equation = y +d n are respectively
dx 4 dx
(a) 4, 1 (b) 4, 2 (c) 2, 2 (d) 2, 4
dy
(iii) The integrating factor of the differential equation x – y = 2x 2 is
dx
1
(a) e–x (b) e–y (c) (d) x
x
(iv) The differential equation of the family of lines passing through the origin is
dy dy dy dy
(a) = x (b) = y (c) x – y = 0 (d) x + =0
dx dx dx dx
dy
(v) Solution of the differential equation x + y = x e x is
dx
(a) xy = ex (1 – x) + C (b) xy = ex (x + 1) + C
(c) xy = ey (y – 1) + C (d) xy = ex (x – 1) + C
(vi) The general solution of the differential equation ex dy + (y ex + 2x) dx = 0 is
(a) x ey + x2 = C (b) x ey + y2 = C (c) y ex + x2 = C (d) y ey + x2 = C
2. Fill in the blanks.
(i) The number of arbitrary constant (s) in a particular solution of the differential equation
tan x dx + tan y dy = 0 is _____________ .
dy
(ii) The solution of the differential equation x + 2y = x 2 is _____________ .
dx
dy x 2
(iii) The general solution of the differential equation = is _____________ .
dx y 2

360 Xam idea Mathematics–XII

@Cbsebookshub - Join Us on Telegram


d2 y
(iv) The degree of the differential equation + e dy/dx = 0 is _____________ .
dx 2
QQ Very Short Answer Questions: [1 mark each]
3. What is the degree of the following differential equation:
dy 2 d 2 y
5x c m– – 6y = log x ? [CBSE Delhi 2010]
dx dx 2
4. Write the degree of the following differential equation:
2
d2 y dy 4
x e 3
o + xc m = 0 [CBSE Delhi 2013]
dx 2 dx
5. Write the sum of the order and degree of the following differential equation:
2 3
d2 y dy
f p +e o + x4 = 0  [CBSE (F) 2015]
dx 2 dx
2 2
d2 y dy
6. Find the product of the order and degree x f p +e o + y 2 = 0 .
[CBSE Chennai 2015]
dx2 dx
7. Write the differential equation formed from the equation y = mx + c, where m and c are arbitrary
constants. [CBSE (F) 2013]
dy
8. Write the integrating factor of ^x log xh + y = 2 log x . [CBSE Punchkula 2015]
dx
9. Solve : edy/dx = x2
10. State whether y = e–x(x + a) is the solution of differential equation:
dy
+ y = e –x
dx
dy y (x + 1)
11. Solve : – =0
dx x
QQ Short Answer Questions–I: [2 marks each]
dy y
12. Write the general solution of the differential equation = .
dx x
dy 1+y
13. Write the integrating factor of +y = .
dx x
dy
14. Given that = e –2y and y = 0 when x = 5. Find the value of x when y = 3.
dx
dy
15. Find the general solution of the differential equation = 2y – x .
dx
16. Find the differential equation of the family of curves y = A e2x + B e–2x.

QQ Short Answer Questions–II: [3 marks each]


17. Solve the following differential equation:
dy
cos 2 x + y = tan x [CBSE Delhi 2008, 2011; (AI) 2009]
dx
18. Solve the differential equation:
dy
(x 2 + 1) + 2xy = x 2 + 4 [CBSE (AI) 2010]
dx
dy
19. Solve : (1 + x 2) + y = tan –1 x [CBSE (AI) 2014]
dx

Differential Equations 361


@Cbsebookshub - Join Us on Telegram
dy 2
20. Solve : ^x 2 – 1h + 2xy = 2 [CBSE (AI) 2010]
dx x –1
dy dy
21. Solve the differential equation: y + x =x–y [CBSE (Central) 2016]
dx dx
22. Solve the differential equation:
(x2 + 3xy + y2)dx – x2dy = 0 given that y = 0, when x = 1 [CBSE (East) 2016]
dy
23. Solve the differential equation : ^ x + 1 h) – y = e 3x ^ x + 1 h
3
[CBSE (South) 2016]
dx
24. Solve the following differential equation :
y2dx + (x2 – xy + y2)dy = 0 [CBSE (F) 2016]
25. Find the particular solution of the differential equation: [CBSE (F) 2011]
2x 2 x
(1 + e ) dy + (1 + y ) e dx = 0, given that y = 1, when x = 0
dy
– xy = 1 + cos c m, x ! 0. Find the
y
26. Find the particular solution of this differential equation x 2
dx x
r
particular solution of this differential equation, given that when x = 1, y = . [CBSE (F) 2013]
2
dy xy
27. Find the particular solution of the differential equation = given that y = 1, when x = 0.
dx x + y2
2

 [CBSE Delhi 2015]


28. Solve the following differential equation:
dy 2
(x 2 – 1) + 2xy = 2 [CBSE (AI) 2009]
dx x –1
dy r
29. Solve the differential equation + y cot x = 2 cos x, given that y = 0, when x = .
dx 2
 [CBSE (F) 2014]
30. Solve the following differential equation:
dy
(1 + x 2) + y = tan –1 x [CBSE Delhi 2009, (AI) 2008, 2009]
dx
31. Solve the differential equation (x2 – yx2) dy + (y2 + x2y2) dx = 0, given that y = 1, when x = 1.
 [CBSE (F) 2014]
32. Solve the differential equation:
dy x + y
= x – y  [CBSE 2019 (65/3/1)]
dx
33. Solve the differential equation:
(1 + x 2) dy + 2xy dx = cot xdx  [CBSE 2019 (65/3/1)]
2 3 3
34. Find the general solution of the differential equation x y dx – (x + y ) dy = 0. [CBSE 2020 (65/3/1)]

QQ Long Answer Questions: [5 marks each]


r
35. Solve the following differential equation, given that y = 0, when x = :
4
dy
sin 2x – y = tan x [CBSE Chennai 2015]
dx
36. Find the differential equation for all the straight lines, which are at a unit distance from the
origin. [CBSE Patna 2015]

362 Xam idea Mathematics–XII

@Cbsebookshub - Join Us on Telegram


37. Solve the following differential equation:

=y – x cos e oG dy + =y cos e o – 2x sin e oG dx = 0


y y y
[CBSE (F) 2015]
x x x

= ` e m tan x – y j given that y = 1,


dy
38. Find the particular solution of the differential equation ^1 + x 2 h
–1

dx
when x = 0. [CBSE Panchkula 2015]
39. Find the particular solution of the following differential equation:
dy
xy = (x + 2) (y + 2); y = –1 when x = 1  [CBSE Delhi 2012]
dx
40. Find the particular solution of the differential equation
dx r
+ x cot y = 2y + y 2 cot y, (y ! 0) given that x = 0 when y = . [CBSE (AI) 2013]
dy 2
41. Find the particular solution of the differential equation x(1 + y2)dx – y(1 + x2)dy = 0 given that
y = 1 when x = 0. [CBSE (AI) 2014]
42. Find the particular solution of the differential equation satisfying the given conditions
x2 dy + (xy + y2) dx = 0; y = 1 when x = 1. [CBSE Delhi 2010]
43. (x2 + y2) dy = xy dx. If y(1) = 1 and y(x0) = e, then find the value of x0. [CBSE Bhubaneswar 2015]
44. Find the particular solution of the differential equation (y – sin x) dx + (tan x) dy = 0 satisfying the
condition that y = 0 when x = 0. [CBSE Guwahati 2015]

Answers
1. (i) (b) (ii) (a) (iii) (c) (iv) (c) (v) (d) (vi) (c)
x2 +
2. (i) zero (ii) y = C x –2 (iii) x 3 – y 3 = C (iv) not defined
4
d2 y
3. 1 4. 2 5. 4 6. 4 7. = 0 8. log x
dx 2
ex
9. y = 2 (x log x – x) + C 10. Yes 11. y = x ex + C 12. y = C x 13. x

e6 + 9 d2 y
14. –x –y
15. 2 – 2 = C 16. – 4y = 0 17. y = tan x – 1 + C e– tan x
2 dx 2
x –1 –1
18. (x 2 + 1) y = x 2 + 4 + 2 log x + x 2 + 4 19. y e tan x = (tan –1 x – 1) e tan x + C
2
x–1 x log x
20. y (x 2 – 1) = log + + C 21. y2 + 2xy – x2 = C2 22. y =
x 1 1 – log x
y e 3x
e 3x y r
23. = (x + 1) – + C 24. y = C e tan –1 25. tan –1 y + tan –1 e x =
x+1 3 9 x 2
y 2
x–1
26. tan e o = – 2 +
1 3 x 1 C
27. – 2 + log y = 0 28. y = 2 log + + 2
2x 2x 2 2y x –1 x 1 x +1
–1 1 1
29. 2y sin x = – (1 + cos 2x) 30. y = (tan –1 x – 1) + C e tan x 31. log y + = – + x + 1
y x
y x3
32. tan –1 d n = log (x 2 + y 2) + C 33. y =
1 1 C
log sin x + 34. log y = +C
x 2 1 + x2 1 + x2 3y 3
y
35. y = tan x – tan x 36. y = x yl ! (yl ) 2 + 1 37. y 2 – 2x 2 cos d n = C
x

Differential Equations 363


@Cbsebookshub - Join Us on Telegram
–1
(m + 1) tan x
r2
38. y e tan –1 x
= e + m
39. x + 2 log|x| – 2 40. x sin y = y 2 sin y –
m+1 m+1 4
1
41. y2 = 2x2 + 1 42. 3x2y = y + 2x 43. x0 = 3 e 44. y = sin x
2

SELF-ASSESSMENT TEST
Time allowed: 1 hour Max. marks: 30

1.
Choose and write the correct option in the following questions. (4 × 1 = 4)

d2 y 2 3/2
= )1 + d n3
dy
(i) The order and degree of the differential equation , respectively are
dx 2 dx
3
(a) , 2 (b) 2, 2 (c) 2, 3 (d) 3, 4
2
(ii) The general solution of ex cos y dx – ex sin y dy = 0 is

(a) ex cos y = k (b) ex sin y = k (c) ex = k cos y (d) ex = k sin y
(iii) The differential equation of family of curves y2 = 4a (x+a) is

dy dy dy
(a) y2 = 4 e x + o (b) 2y = 4a
dx dx dx
d2 y dy 2
dy dy 2
(c) y +e o = 0 (d) 2x +ye o – y=0
dx2 dx dx dx
dy
(iv) The integrating factor of
+ y sec x = tan x is
dx
(a) sec x + tan x (b) log|sec x + tan x| (c) esec x (d) sec x
2.
Fill in the blanks. (2 × 1 = 2)

(i) The solution of the differential equation y dx + (x + xy) dy = 0 is _____________ .


dy y
(ii) The general solution of the differential equation + = 1 is _____________ .
dx x
QQ Solve the following questions. (3 × 1 = 3)
3. Solve : edy/dx = x2
dy y ^x + 1h
4. Solve : – =0
dx x
dy
5. Write the integrating factor of ^x log xh + y = 2 log x .
dx
QQ Solve the following questions. (2 × 2 = 4)
2
6. Write the order of the differential equation representing the family of parabola y = 4ax.
dy
7. Given that = e –2y and y = 0 when x = 5. Find the value of x when y = 3.
dx
QQ Solve the following questions. (4 × 3 = 12)
dy
8. Solve : ^1 + x 2h + y = tan –1 x
dx

364 Xam idea Mathematics–XII

@Cbsebookshub - Join Us on Telegram


9. Find the equation of a curve passing through a1, k , if the slope of the tangent to the curve at any
r

4
y y
point P(x, y) is – cos 2 .
x x
d
10. Solve: y + ^xyh = x ^sin x + log xh
dx
dy dy
11. Solve the differential equation: y + x =x–y
dx dx
QQ Solve the following question. (1 × 5 = 5)

sin a k + x – y sin = 0, y ^1 h =
dy y y r
12. Solve : x
dx x x 2

Answers
1. (i) (b) (ii) (a) (iii) (d) (iv) (a)
x2
2. (i) xy = C e– y (ii) xy = +C
2
e6 + 11
3. y = 2(x log x – x) + C 4. y = xex + C 5. log x 6. 1 7.
2
tan d y n – 1
x
–1 –1
x x
8. ye tan = (tan –1 x – 1) e tan + C 9. xe =1
2 sin x 2 cos x x log x x
10. y = – cos x + + + – + Cx –2 11. y2 + 2xy – x2 = C2
x x2 x 9
y
12. log | x | = cos d n, x ! 0
x

zzz

Differential Equations 365


@Cbsebookshub - Join Us on Telegram
11 Vector
Algebra

1. Vector : Those physical quantities, which are defined by z axis

both magnitude and direction are called vector e.g., velocity,


P(x, , )
acceleration, force, etc.
2. Scalar : Those physical quantities which have only magnitude
are called scalar, e.g., area, volume, mass, etc.
x axis
3. Position vector : Let (x, y, z) be a point in space with respect to O (0, 0, 0)

the origin O (0, 0, 0). The vector OP having O as initial and P


as terminal point is called position vector of P.
y axis
Here, position vector of P = OP = xit + yjt + zkt
z axis
4. Direction cosines : If r = ait + bjt + ckt makes angle a, b, g with
+ve direction of x-axis, y-axis and z-axis respectively, then P(a, , )

cos a, cos b and cos g are the direction cosines of r and are
denoted by l, m and n where
a b
l = cos a = , m = cos b =
a2 + b2 + c2 a2 + b2 + c2 O x axis
c
n = cos c =
a + b2 + c2
2

y axis
5. Direction ratios : If numbers a, b, c are proportional to direction
cosine l, m and n respectively of r , then a, b, c are called direction ratios of r .
z axis
6. Vector joining two points: If A (x1, y1, z1) and B (x2, y2, z2) are
B(x2, y2, z2)
two points, then the vector joining A and B is the vector AB
given by
AB = (x2 – x1) it + (y2 – y1) jt + (z2 – z1) kt . A(x1, y1, z1)

x axis

Proof : AB = AO + OB [By addition of vectors] O

AB = OB – OA = (x2 it + y2 jt + z2 kt) – (x1 it + y1 jt + z1 kt)


= (x2 – x1) it + (y2 – y1) jt + (z2 – z1) kt y axis

366 Xam idea Mathematics–XII

@Cbsebookshub - Join Us on Telegram


7. Components of a vector: If r = xit + yjt + zkt then x, y, z are called scalar components of r and
xit, yjt, zkt are called vector components of r .

8. If a and b are the position vectors of two points A and B, then AB = b – a .

9. If a = a1 it + b1 jt + c1 kt then | a | = a12 + b12 + c12 .

a
10. If a is any given vector then unit vector in direction a , i.e., a = .
a
11. (i) Collinearity of three points: Three points with position vectors a , b , c are collinear, iff there
exist scalars x, y, z not all zero such that x a + y b + z c = 0 , where x + y + z = 0.

(ii) Coplanarity of three vectors: Let a and b be two given non-zero non-collinear, vectors. Then
any vector r , coplanar with a and b can be uniquely expressed as r = x a + y b for some
scalars x and y.

12. Section formula:


(i) For internal division: The position vector of a point C, which divides A C B
m n
internally the line-segment joining two points A and B with position a
vectors a and b in the ratio m : n (as figure given alongside) is given by

mb+na
position vector of C = .
m+n
(ii) For external division: The position vector of a point C, which divides m
A B n C
externally the line-segment joining two points A and B with position a

vectors a and b in the ratio m : n (as figure given alongside) is given by


mb –na
position vector of C = .
m–n
13. Two vectors are said to be orthogonal if they are perpendicular to each other.

14. The dot product (scalar product) of two vectors a and b is given by a . b = a b cos i , where q is
the angle between a and b .

15. Properties of dot product of vectors:

(i) a . b = b . a (commutativity) (ii) a . (m b ) = (m a ) . b = m (a . b ), m is a scalar

(iii) a . (b + c ) = a . b + a . c (Distributive property) (iv) a . b = 0 + a = 0, b = 0 or a = b


(v) If a = a1 it + a2 jt + a3 kt and b = b1 it + b2 jt + b3 kt , then a . b = a1 b1 + a2 b2 + a3 b3

and projection vector of a on b = f p.b


a .b a .b
(vi) Projection of a on b = 2
|b | b

and projection vector of b on a = f p. a


a .b a .b
(vii) Projection of b on a =
2
a a

Vector Algebra 367


@Cbsebookshub - Join Us on Telegram
16. it . it = jt . jt = kt . kt = 1 and it . jt = jt . kt = kt . it = 0 , where it, jt and kt are unit vectors along x-axis, y-axis
and z-axis respectively.
17. If q is the angle between two vectors a = a1 it + a2 jt + a3 kt and b = b1 it + b2 jt + b3 kt , then

a .b a b +a b +a b
1 1 2 2 3 3
cos i = =
a b a1 a 2 a 3 b1 b 22 + b 32
2
+ 2
+ 2 2
+

18. If a = b , then a . b = 0 & a1 a2 + b1 b2 + c1 c2 = 0 , where a = a1 it + b1 jt + c1 kt and b = a2 it + b2 jt + c2 kt

19. The cross product or vector product of two vectors a and b is given by a × b = a b sin int , where
q is the angle between a and b and nt is a unit vector perpendicular to the plane of a and b and
+ ve for a right handed rotation from a to b .

a ×b
20. a × b = a b sin i and sin i = , where q is the angle between a and b .
a b
21. Properties of cross product of vectors:

(i) a × b = – b × a (ii) a × a = b × b = c × c = it × it = jt × jt = kt × kt = 0

(iii) it × jt = kt, jt × kt = it and kt × it = jt (iv) it × jt = –jt × it, jt × kt = – kt × jt and kt × it = – it × kt

(v) If a × b = 0 + a = 0, b = 0 or a || b
it jt kt
(vi) If a = a1 it + a2 jt + a3 kt and b = b1 it + b2 jt + b3 kt then a × b = a1 a2 a3
b1 b2 b3

22. Area of a parallelogram, whose sides are represented by a , b is a × b .

a #b
23. Unit vector perpendicular to a and b is .
a #b
1
24. Area of parallelogram, whose diagonals are represented by a and b is .
2 a ×b
a ×b
25. Area of triangle, whose two sides are represented by a and b is given by .
2
26. If F is a force applied at a point A, then moment of force about the point P is given by |AP × F |.
27. Cosine formulae : If a, b, c are lengths of the opposite sides respectively to the angles A, B and C of
a triangle ABC, then
b2 + c2 – a2 c2 + a2 – b2 a2 + b2 – c2
(i) cos A = (ii) cos B = (iii) cos C =
2bc 2ac 2ab
28. Projection formulae : If a, b, c are lengths of the sides opposite respectively to the angles A, B, C of
a triangle ABC, then
(i) a = b cos C + c cos B (ii) b = c cos A + a cos C (iii) c = a cos B + b cos A

2 a. a a. b
29. Lagrange's identity : a × b =
a. b b. b
2 2
or (a . b ) 2 + (a × b ) 2 = a b

368 Xam idea Mathematics–XII

@Cbsebookshub - Join Us on Telegram


30. For any two vectors a and b , we have
2 2 2 2 2 2
(i) a + b = a + b +2 a b (ii) a –b = a + b –2 a b

= 29 a + b C
2 2 2 2 2 2
(iii) a + b + a –b (iv) (a + b ) . (a – b ) = a – b .
31. Scalar triple product of vectors:
The scalar triple product of three vectors a , b and c denoted by [a b c ] is equal to the dot product
of the first vector by the cross product of remaining two in order.
i.e.,
[ a b c ] = a . ( b × c ) = b . ( c ×a ) = c . ( a × b )
The scalar triple product is a pseudoscalar (i.e., it reverses sign under inversion).
Since, the cross product of two vectors is calculated by using a determinant as

it jt kt
b × c = b1 b2 b3
c1 c2 c3

it jt kt
a . (b × c ) = (a1 it + a2 jt + a3 kt) . b1 b2 b3
c1 c2 c3

a1 a2 a3
This gives [a b c ] = a . (b × c ) = 1 b2 b3
b
c1 c2 c3

a = a1 it + a2 jt + a3 kt; b = b1 it + b2 jt + b3 kt and c = c1 it + c2 jt + c3 kt
where
Properties of scalar triple product:
(i) The scalar triple product of vectors does not change if the order of its factors are circularly
rotated, but it changes its sign if they are transposed.

i.e.,
[a b c ] = [ b c a ] = [ c a b ]
(ii) If any vector out of three is equal to any other vector with multiplication of a scalar quantity
then the value of scalar triple product is zero.
e.g., let a = a it + a jt + a kt ; b = ma it + ma jt + ma kt = m a and c = c it + c jt + c kt
1 2 3 1 2 3 1 2 3

a1 a2 a3 a1 a2 a3

[a b c ] = ma1 ma2 ma3 = m a1 a2 a3 = 0
c1 c2 c3 c1 c2 c3

[Note: The value of determinant is zero if any two rows/columns are same.]
Geometrical interpretation of scalar triple product:
Let a , b and c be non-zero, non parallel vectors. A G F
Normal of base

parallelopiped is constructed in which three adjacent sides


A
OA, OB and OC are represented by a , b and c respectively
in magnitude and direction. a
B
Let normal of base OBDC of parallelopiped makes angle θ D
with a . O M C

Vector Algebra 369


@Cbsebookshub - Join Us on Telegram
` +OAM = i , where AM = height of parallelopiped.
Volume of parallelopiped = area of base OBDC × height

= | b × c | . AM
AM
= | b × c | . OA cos i [ In DOAM, cos i =
OA
& AM = OA cos i ]
= | b × c | . | a | cos i = | a | . | b × c | cos i

= a . (b × c ) = [a b c ] = Scalar triple product of a , b , c

[Note: Base OBDC is parallelogram and thus area is b # c . Also normal vector of base OBDC is
b # c ].

Coplanarity: Three vectors a , b , c are coplanar, if the scalar triple product of these three vector is
zero, i.e., the volume of parallelopiped so formed is zero and thus it would be flat.
i.e.,
[a b c ] = a . ( b × c ) = 0 + a , b , c are coplanar.

Selected NCERT Questions


Find the scalar components of the vector AB with initial point A(2, 1) and terminal point B(–5, 7).
1.
 [CBSE (AI) 2012]
Sol. Let AB = (–5 –2) it + (7–1) jt = –7it + 6jt
Hence, scalar components are –7, 6.
[Note: If r = xit + yjt + zkt then x, y, z are called scalar components and xit, yjt, zkt are called vector

components.]
2. Find a vector in the direction of vector 5it –jt + 2kt which has a magnitude of 8 units.
2 2 2
Sol. a = (5) + (–1) + (2) = 25 + 1 + 4 = 30

The unit vector in the direction of vector a is

a 1
= (5it – jt + 2kt)
a 30

Now, vector in the direction of a having magnitude 8 units is


8 40 t 8 t 16 t
8at = (5it – jt + 2kt) = i– j+ k
30 30 30 30
3. Find the position vector of a point R which divides the line joining two points P and Q whose
position vectors are it + 2jt – kt and –it + jt + kt respectively, in the ratio 2 : 1
(i) internally (ii) externally
Sol. (i) Let R be the point which divides the line joining the point P and Q internally in the ratio 2 : 1.

2 (– it + jt + kt) + 1 (it + 2jt – kt) 1 4 1


The position vector of point R = = – it + jt + kt.
2+1 3 3 3
(ii) Let R be the point which divides the line joining the points P and Q externally in the ratio 2 : 1

2 (– it + jt + kt) – 1 (it + 2jt – kt) –2it + 2jt + 2kt –it –2jt + kt


= = = –3it + 3kt
2–1 1

370 Xam idea Mathematics–XII

@Cbsebookshub - Join Us on Telegram


4. Find x , if for a unit vector a , (x – a ) . ^ x + a h = 12 .

Sol. Here a = 1 and ^x – a h . ^x + a h = 12

Now, ^x – a h . ^x + a h = 12 &x .x+x .a – a . x – a . a = 12

– ]1 g = 12
2 2 2 2 2 2

x +x .a –x .a – a = 12 ⇒ x – a = 12 ⇒ x
2

x = 12 + 1 ⇒ x = 13
5. Using vectors, prove that the points (2, –1, 3), (3, –5, 1) and (–1, 11, 9) are collinear.
[CBSE 2019 (65/5/3)]
Sol. Let A (2, – 1, 3), B (3, –5, 1) and C (–1, 11, 9) are three points.
To show that A, B, C are collinear.


AB = (3 – 2) it + (–5 + 1) jt + (1 – 3) kt = it – 4jt – 2kt = 1 2 + (–4) 2 + (–2) 2 = 21

BC = (– 1 – 3) it + (11 + 5) jt + (9 – 1) kt = – 4it + 16jt + 8kt = (–4) 2 + (16) 2 + (8) 2 = 4 21

AC = (–1 – 2) it + (11 + 1) jt + (9 – 3) kt = –3it + 12jt + 6kt = (–3) 2 + (12) 2 + (6) 2 = 3 21


 AC + AB = BC

A, B, C are collinear.

6. Find λ and µ if (2it + 6jt + 27kt) × (it + mjt + nkt) = 0 .

Sol. Here (2it + 6jt + 27kt) × (it + mjt + nkt) = 0


it jt kt

2 6 27 = 0
1 m n

(6n – 27m) it – (2n –27) jt + (2m – 6) kt = 0


6n – 27m = 0, 2n – 27 = 0 and 2m – 6 = 0
27

n= and λ = 3
2
7. Find a unit vector perpendicular to each of the vectors a + b and a – b , where a = 3it + 2jt + 2kt
and b = it + 2jt – 2kt . [CBSE Delhi 2011]

Sol. Given, a = 3it + 2jt + 2kt and b = it + 2jt – 2kt


a + b = 4it + 4jt and a – b = 2it + 4kt

Now, vector perpendicular to (a + b ) and (a – b ) is

it jt kt
(a + b ) × (a – b ) = 4 4 0 = (16 – 0) it– (16 – 0) jt + (0 – 8) kt = 16it – 16jt – 8kt
2 0 4

∴ Unit vector perpendicular to (a + b ) and (a – b ) is given by


Vector Algebra 371


@Cbsebookshub - Join Us on Telegram
(a + b ) × (a – b ) 16it – 16jt – 8kt 8 (2it–2jt – kt)
! =! = !
|(a + b ) × (a – b ) | 16 2 + (–16) 2 + (–8) 2 8 22 + 22 + 12

2it – 2jt – kt t
= ! c 2 it – 2 jt – k m = ! it " jt " kt
2 2 1
= !
9 3 3 3 3 3 3

The two adjacent sides of a parallelogram are 2it – 4jt + 5kt and it– 2jt – 3kt . Find the unit vector
8.
parallel to one of its diagonals. Also, find its area. [CBSE (F) 2012]

Sol. Let two adjacent sides AB and AC of a parallelogram ABDC be represented by 2it – 4jt + 5kt and
it – 2jt – 3kt in magnitude and direction respectively.
C D

i.e., AB = 2it – 4jt + 5kt and AC = it – 2jt – 3kt



By parallelogram law of addition
AD = AB + BD ⇒ AD = AB + AC [a BD = AC ]
A B
AD = (2it – 4jt + 5kt) + (it – 2jt – 3kt) = 3it – 6jt + 2kt

AD = 3it – 6jt + 2kt = 3 + (– 6) + 2 = 9 + 36 + 4 = 49 = 7


2 2 2

1
Therefore, unit vector parallel to diagonal AD = AD
AD
1 t 3 6 2
= (3i – 6jt + 2kt) = it – jt + kt
7 7 7 7
it jt kt
Again, =
AB × AC 2 –4 5
1 –2 –3

AB × AC = (12 + 10) it – (–6 –5) jt + (–4 + 4) kt = 22it + 11jt

AB × AC = | 22it + 11jt | = 22 + 11 = 484 + 121 = 605 = 11 5


2 2

Now the area of parallelogram ABDC whose adjacent sides are AB and AC
= AB ×AC = 11 5 sq units.

Let a = it + 4jt + 2kt, b = 3it – 2jt + 7kt and c = 2it – jt + 4kt . Find a vector d which is perpendicular
9.
to both a and b and c . d = 15. [CBSE Ajmer 2015]

Sol. The vector d is perpendicular to both a and b , so we have d = m (a × b ) .


it jt kt
Now a × b = 1 4 2 = 32it – jt – 14kt
3 –2 7


So, d = 32mit – mjt – 14mkt

We have c . d = 15 ⇒ (2it – jt + 4kt) . (32mit – mit – 14mkt) = 15


15 5

64m + m – 56m = 15 ⇒ 9m = 15 & m=
9
=
3
5
∴ Required vector d =
(32it – jt – 14kt)
3

372 Xam idea Mathematics–XII

@Cbsebookshub - Join Us on Telegram


5 5 5 160 t 5 t 70 t
or d = 32 × it – jt – 14 × kt = i– j– k.
3 3 3 3 3 3
10. The scalar product of the vector it + jt + kt with the unit vector along the sum of vectors
2it + 4jt – 5kt and mit + 2jt + 3kt is equal to one. Find the value of λ. [CBSE (AI) 2009, 2014]

Sol. Let sum of vectors 2it + 4jt – 5kt and mit + 2jt + 3kt equal to a then a = (2 + m) it + 6jt – 2kt

a (2 + m) it + 6jt – 2kt
The unit vector in the direction of a = at = =
a (2 + m) 2 + 36 + 4
(2 + m) i + 6j – 2kt
t t
Here, (it + jt + kt) . at = (it + jt + kt) . =1
(2 + m) 2 + 40

(2 + m) + 6 – 2 = (2 + m) 2 + 40 & (m + 6) 2 = (2 + m) 2 + 40

m 2 + 36 + 12m = 4 + m 2 + 4m + 40 & 8m = 8 & m = 1.
11. If with reference to the right handed system of mutually perpendicular unit vectors
it, jt and kt, a = 3it – jt, b = 2it + jt – 3kt, then express b in the form b = b 1 + b 2 , where b 1 is
parallel to a and b2 is perpendicular to a .

Sol. Let b1 = ma , m is a scalar, i.e., b1 = 3mit – mjt


and b 2 = b – b1 = (2 – 3m) it + (1 + m) jt – 3kt
Now, since b2 is to be perpendicular to a , we should have a . b 2 = 0 i.e.,
1
3(2 – 3λ) – (1 + λ) = 0 ⇒ m =
2
3 1 1 3
Therefore, b1 = it – jt and b 2 = it + jt – 3kt
2 2 2 2

Multiple Choice Questions [1 mark]


Choose and write the correct option in the following questions.
1. The vectors 3it – jt + 2kt, 2it + jt + 3kt and it + mjt – kt are coplanar if [CBSE 2020 (65/2/1)]
(a) –2 (b) 0 (c) 2 (d) Any real number
2. The area of a triangle formed by vertices O, A, B where OA = it + 2jt + 3kt and OB = –3it – 2jt + kt
is [CBSE 2020 (65/2/1)]
(a) 3 5 sq. units (b) 5 5 sq. units (c) 6 5 sq. units (d) 4 sq. units
3. The value of it. (jt×kt) + jt. (it×kt) + kt. (it×jt) is
(a) 0 (b) – 1 (c) 1 (d) 3
4. If θ is the angle between any two vectors a and b then a – b = a + b , where θ is equal to
r r
(a) 0 (b) (c) (d) r
4 2
5. The vector of the direction of the vector it – 2jt + 2kt that has magnitude 9 is [NCERT Exemplar]
it – 2jt + 2kt
(a) it – 2jt + 2kt (b) (c) 3 (it – 2jt + 2kt) (d) 9 (it – 2jt + 2kt)
3

Vector Algebra 373


@Cbsebookshub - Join Us on Telegram
6. The position vector of the point which divides the join of point 2a – 3b and a + b in the ratio
3 : 1 is [NCERT Exemplar]
3a –2b 7a –8b 3a 5a
(a) (b) (c) (d)
2 4 4 4
7. The vector having initial and terminal points as (2, 5, 0) and (–3, 7, 4) respectively is
(a) –it + 12jt + 4kt (b) 5it + 2jt – 4kt (c) –5it + 2jt + 4kt (d) it + jt + kt

8. The angle between two vectors a and b with magnitudes 3 and 4 respectively and
a . b = 2 3 is [NCERT Exemplar]
r r r 5r
(a) (b) (c) (d)
6 3 2 2
t t t t t t
9. Find the value of λ such that the vectors a = 2i + mj + k and b = i + 2j + 3k are orthogonal
3 5
(a) 0 (b) 1 (c)
(d) –
2 2
10. The value of λ for which the vectors 3it – 6jt + kt and 2it – 4jt + mkt are parallel is
2 3 5 2
(a) (b) (c) (d)
3 2 2 5
11. The vector from origin to the points A and B are a = 2it – 3jt + 2kt and b = 2it + 3jt + kt, respectively
then the area of triangle OAB is [NCERT Exemplar]
1
(a) 340 (b) 25 (c) 229 (d) 229
2
12. For any vector a, the value of ^a × it h + `a × jt j + ^a × kt h is equal to
2 2 2
[NCERT Exemplar]
2 2 2 2
(a) a (b) 3 a (c) 4 a (d) 2 a
13. If a = 10, b = 2 and a . b = 12, then value of a × b is
(a) 5 (b) 10 (c) 14 (d) 16
14. The vector mit + jt + 2kt, it + mjt – kt and 2it – jt + mkt are coplanar if
(a) m = –2 (b) m = 0 (c) λ = 1 (d) λ = – 1

15. If a , b , c are unit vectors such that a + b + c = 0 , then the value of a . b + b . c + c . a is


3
(a) 1 (b) 3 (c) – (d) None of these
2
16. Projection vector of a on b is [NCERT Exemplar]

(a) f p . b (b) f p bt
a .b a .b a .b a .b
2 (c) (d) 2
b b a b

17. If a , b and c are three vectors such that a + b + c = 0 and a = 2, b = 3, c = 5, then value
of a . b + b . c + c . a is
(a) 0 (b) 1 (c) – 19 (d) 38
18. If a = 4 and –3 # m $ 2, then the range of m a is [NCERT Exemplar]
(a) [0, 8] (b) [– 12, 8] (c) [0, 12] (d) [8, 12]
19. The number of vectors of unit length perpendicular to the vectors a = 2it + jt + 2kt and b = jt + kt is
(a) one (b) two (c) three (d) infinite

374 Xam idea Mathematics–XII

@Cbsebookshub - Join Us on Telegram


20. The position vector of the point which divides the join of points with position vectors a + b
and 2a – b in the ratio 1 : 2 is [NCERT Exemplar]
3a + 2 b 5a – b 4a + b
(a) (b) a (c) (d)
3 3 3

Answers
1. (a) 2. (a) 3. (b) 4. (c) 5. (c) 6. (d)
7. (c) 8. (b) 9. (d) 10. (a) 11. (d) 12. (d)
13. (d) 14. (a) 15. (c) 16. (a) 17. (c) 18. (c)
19. (b) 20. (d)

Solutions of Selected Multiple Choice Questions


1. Given vectors are coplanar if
3 –1 2
2 1 3 =0
1 m –1

3 (–1 –3m) + 1 (–2 –3) + 2 (2m – 1) = 0

–5m – 10 = 0   ⇒  5m = – 10   ⇒  m = – 2
2. We have,
it jt kt
OA # OB = 1 2 3 = 8it – 10jt + 4kt
–3 –2 1
2 2 2

OA # OB = (8) + (–10) + (4) = 64 + 100 + 16 = 180 = 6 5
1 1

Area of D OAB = = # 6 5 = 3 5 sq. units.
2 OA # OB 2
8. Here, a = 3 , b = 4 and a . b = 2 3
We know that, a . b = a b cos i ⇒ 2 3 = 3 . 4 . cos i
2 3 1 r

cos i = = ⇒ i=
4 3 2 3
2
12. Let a = xit + yjt + zkt ⇒ a = x2 + y2 + z2
it jt kt
a × it = x y z = it (0) – jt (–z) + kt (–y) = zjt – ykt
1 0 0


(a × it) 2 = (zjt – ykt) (zjt – ykt) = y2 + z2

Similarly, ` a × jt j = x 2 + z 2 and ^ a × kt h = x 2 + y 2
2 2

2

(a × it) 2 + (a × jt) 2 + (a × kt) 2 = y 2 + z 2 + x 2 + z 2 + x 2 + y 2 = 2 (x 2 + y 2 + z 2) = 2 a
2 2 2
15. We have, a + b + c = 0 and a = 1, b = 1, c =1

 ^ a + b + c h^ a + b + c h = 0
2 2 2

a + a .b + a . c + b . a + b + b . c + c .a + c . b + c =0

Vector Algebra 375


@Cbsebookshub - Join Us on Telegram
2 2 2

a + b + c
+ 2 (a .b + b . c + c . a ) = 0 [a a . b = b . a , b . c = c . b and c . a = a . c ]
3

1 + 1 + 1 + 2 ( a . b + b . c + c .a ) = 0 ⇒ a . b + b . c + c . a = –
2
2 2 2
17. Here, a + b + c = 0 and a = 4, b = 9, c = 25


^a + b + c h.^a + b + c h = 0
2 2 2

a + a .b + a . c + b . a + b + b . c + c .a + c . b + c =0
2 2 2

a + b + c + 2 (a . b + b . c + c . a ) = 0
[a a . b = b . a ]


4 + 9 + 25 + 2 ( a .b + b . c + c .a ) = 0
38
⇒ a . b + b . c + c . a = –
= –19
2

Fill in the Blanks [1 mark]


1. The projection of the vector it – jt on the vector it + jt is _____________ . [CBSE 2020 (65/4/1)]
2 2
2. If a # b + a .b = 144 and a = 4 , then b is equal to _____________ . [NCERT Exemplar]
3. If a is a non-zero vector, then _ a . iti it + ` a . jtj jt + _ a . kti kt equals _____________ .
 [CBSE 2020 (65/4/1]
4. If a = 1 and a # it = jt , then angle between a and it is _____________ .

5. The area of the triangle whose adjacent sides are a = it + 4jt – kt and b = it + jt + 2kt is _____________
sq. units.

Answers
r 3
1. 0 2. 3 3. a 4. 5. 11 Sq. units.
2 2
Solutions of Selected Fill in the Blanks
1. Let a = it – jt and b = it + jt

b (it + jt)
∴  Projection of a on b = a .
= (it – jt) .
b (1) 2 + (1) 2
1–1 0
= = =0
2 2
2. We have,
2 2
a#b + a .b = 144

_ a b sin i i + _ a b cos i i = 144


2 2

_ 4 b sin i i + _ 4 b cos i i = 144


2 2


2

16 b (sin 2 i + cos 2 i) = 144
2 2

16 b = 144 ⇒  b =9

⇒  b =3

376 Xam idea Mathematics–XII

@Cbsebookshub - Join Us on Telegram


5. We have,
it jt kt
a ×b = 1 4 –1 = 9it – 3jt – 3kt
1 1 2
2 2 2

a # b = (9) + (–3) + (–3) = 81 + 9 + 9 = 99
   = 3 11
1 3
∴ Area of triangle = a ×b = 11 Sq. units.
2 2

Very Short Answer Questions [1 mark]


1. Find a vector in the direction of vector a = it – 2jt that has magnitude 7 units. [CBSE (AI) 2008]
Sol. The unit vector in the direction of the given vector a is
1 1 t 1 t 2 t
at = a= (i – 2jt) = i– j
|a | 5 5 5
Therefore, the vector having magnitude equal to 7 and in the direction of a is

7at = 7 d jn =
1 t 2 t 7 t 14 t
i– i– j
5 5 5 5
2. Write the number of vectors of unit length perpendicular to both the vectors a = 2it + jt + 2kt
and b = jt + kt.  [CBSE Central 2016]
Sol. Number of vectors of unit length perpendicular to both vectors = 2

3. Write the value of p for which a = 3it + 2jt + 9kt and b = it + pjt + 3kt are parallel vector.
[CBSE Delhi 2009]
Sol. Since a || b , therefore a = m b ⇒ 3it + 2jt + 9kt = m (it + pjt + 3kt)
2

l = 3, 2 = lp, 9 = 3l or l = 3, p = [By comparing the coefficients]
3
4. Write a vector of magnitude 15 units in the direction of vector it – 2jt + 2kt .[CBSE Delhi 2010]

Sol. Let a = it – 2jt + 2kt


it – 2jt + 2kt 1
Unit vector in the direction of a is at = = (it – 2jt + 2kt)
(1) 2 + (–2) 2 + (2) 2 3
(it – 2jt + 2kt)
Vector of magnitude 15 units in the direction of a = 15 at = 15
3
15 t 30 t 30 t = t
= i – j+ k 5i – 10jt + 10kt
3 3 3

5. What is the cosine of the angle, which the vector 2 it + jt + kt makes with y-axis?
[CBSE Delhi 2010]
Sol. We will consider a = 2 it + jt + kt
2 it + jt + kt 2 it + jt + kt 2 it + jt + kt
Unit vector in the direction of a is at = = =
( 2 ) 2 + (1) 2 + (1) 2 4 2
2 t 1t 1 t 1 t 1t 1 t
= i+ j+ k= i+ j+ k
2 2 2 2 2 2

2 it + jt + kt makes with y-axis is c 2 m .


1
The cosine of the angle which the vector

Vector Algebra 377


@Cbsebookshub - Join Us on Telegram
6. If a = 4, b = 3 and a . b = 6 3, then the value of a × b . [CBSE East 2016]
Sol. We have,
a . b = 6 3 ⇒ a . b cos i = 6 3
6 3 3 r

4 × 3 cos θ = 6 3 ⇒ = cos i =
⇒ i=
4×3 2 6
r 1
Now, a × b = a . b sin i = 4 × 3 sin = 4 × 3 × = 6
6 2
7. Write the value of the area of the parallelogram determined by the vectors 2it and 3jt .
[CBSE (F) 2012]
Sol. Required area of parallelogram = | 2it × 3jt | = 6 | it ×jt | = 6 | kt | = 6 sq units.
[Note: Area of parallelogram whose sides are represented by a and b is| a ×b | ]

8. Write the value of (it × jt) . kt + (jt × kt) .it [CBSE (F) 2012]
Sol. (it ×jt) . kt + (jt×kt) . it = kt. kt + it .it = 1 + 1 = 2

[Note: a .b = | a | . | b |cos i. Also| it | = | jt | = | kt | = 1 ]


9. For what value of ‘a’ the vectors 2it – 3jt + 4kt and ait + 6jt – 8kt are collinear? [CBSE Delhi 2011]

Sol.  2it – 3jt + 4kt and ait + 6jt – 8kt are collinear
2 –3 4 2×6 2× (– 8)

a
=
6
=
–8
& a=
–3
or a =
4
⇒ a=–4

[Note: If a and b are collinear vectors then the respective components of a and b are proportional.]

10. Write the direction cosines of the vector – 2it + jt – 5kt .[CBSE Delhi 2011]

Sol. Direction cosines of vector – 2it + jt – 5kt are


–2 1 –5 –2 1 –5
2 2 2
, 2 2 2
, 2 2 2
= , ,
(–2) + 1 + (–5) (–2) + 1 + (–5) (–2) + 1 + (–5) 30 30 30
Note: If l, m, n are direction cosine of ait + bjt + ckt then
a b c
l= , m= , n=
a2 + b2 + c2 a2 + b2 + c2 a2 + b2 + c2
1 4 1
11. If vectors a and b are such that a = 2 , b = and a × b = , then find a . b .
3 3
[CBSE South 2016]
1 1
Sol. We have, a × b = ⇒ a b sin i nt =
3 3
1

a b sin i = [a i is angle between a and b ]
3
1 4 1 1 3

× . sin i = ⇒ sin i = ×
2 3 3 3 2
1
⇒ sin i = ⇒ θ = 30°
2
Now, a . b = a b cos i
3
= 1 × 4 . cos 30° = 2 . =1
2 3 3 2

378 Xam idea Mathematics–XII

@Cbsebookshub - Join Us on Telegram


2
12. Let a and b be two vectors such that a = 3 and b = and a × b is a unit vector. What is
3
the angle between a and b ?
2
Sol. We have, | a | = 3,| b | =
3
Now, a × b = | a || b |sin i
a ×b 1 1

sin i = = = & i = 45°
|a || b | 2 2

3

13. Give an example of vectors a and b such that | a | = b but a ! b . [CBSE Sample Paper 2018]
Sol. Let a = xit + yjt; b = yit + xjt
| a | = x2 + y2, | b | =
y 2 + x 2 Hence, a ! b but a = b

14. Find the unit vector in the direction of sum of vectors a = 2it – jt + kt and b = 2jt + kt .
[NCERT Exemplar]
Sol. Let c denote the sum of a and b
We have, c = a + b = 2it – jt + kt + 2jt + kt = 2it + jt + 2kt
t t t t t t t t t
c = 2i + j + 2k = 2i + j + 2k = 2i + j + 2k
∴ Unit vector in the direction of c =
3
c 22 + 12 + 22 9

Short Answer Questions-I [2 marks]


1. Show that for any two non-zero vectors a and b , a + b = a – b iff a and b are perpendicular
vectors.[CBSE 2020 (65/3/1)]
Sol. We have,
a+b = a –b
2 2

a+b = a –b (Squaring both sides)
2 2 2 2

a + b +2 a b cos i = a + b –2 a b cos i

(Where q is the angle between vectors a and b )



4 a b cos i = 0
r

cos i = 0 = cos
2
r

i=
2

a and b are perpendicular vectors.
2. The x-coordinate of a point on the line joining the point P(2, 2, 1) and Q(5, 1, –2) is 4. Find its
z-coordinate.[CBSE (AI) 2017]
Sol. Let required point be R(4, y1, z1) which divides PQ in ratio k : 1.
By section formula
5k + 2
4= ⇒ 4k + 4 = 5k + 2
k+1
⇒ k = 2
2 × (–2) + 1×1 –4 + 1 –3
∴ z1 = = = = –1
2+1 3 3

Vector Algebra 379


@Cbsebookshub - Join Us on Telegram
3. Find ‘l’ when the projection of a = mit + jt + 4kt on b = 2it + 6jt + 3kt is 4 units. [CBSE Delhi 2012]

a. b a. b
Sol. We know that projection of a on b = ⇒ 4= ...(i)
|b | |b |

Now,
a . b = 2m + 6 + 12 = 2m + 18 also | b | = 2 2 + 6 2 + 3 2 = 4 + 36 + 9 = 7
Putting in (i), we get

2m + 18

7
4= & 2m = 28 – 18 & m = 10 2
=5

4. What are the direction cosines of a line, which makes equal angles with the co-ordinate axes?
 [CBSE (F) 2011]
Sol. Let a be the angle made by line with coordinate axes.
⇒ Direction cosines of line are cos a, cos a, cos a.
& cos 2 a + cos 2 a + cos 2 a = 1
1 1
& 3 cos2 a = 1
& cos 2 a = & cos a = !
3 3
Hence, the direction cosines of the line equally inclined to the coordinate axes are
1 1 1
! ,! ,!
3 3 3
[Note: If l, m, n are direction cosines of line then l2+m2+n2=1]
t p a unit vector?
5. For what value of p, is (it + jt + k) [CBSE (F) 2010]

Sol. Let, a = p (it + jt + kt)


Magnitude of a =| a | = (p) 2 + (p) 2 + (p) 2 = ! 3 p
As a is a unit vector, | a | = 1
1
& ! 3 p = 1 & p =! .
3
6. X and Y are two points with position vectors 3a + b and a – 3b respectively. Write the position
vector of a point Z which divides the line segment XY in the ratio 2 : 1 externally.
[CBSE 2019 (65/4/1)]
Sol. We have OX = 3a + b , OY = a – 3b , OZ = ?
2 ( a – 3 b ) – 1 (3 a + b )
OZ = 2–1 = – a – 7b = – a – 7b
1

OZ = – a – 7b
7. Write the position vector of the mid-point of the vector joining the points P(2, 3, 4) and
Q(4, 1, –2). [CBSE (F) 2011]
Sol. Let a , b be position vector of points P(2, 3, 4) and Q(4, 1, –2) respectively.

a = 2it + 3jt + 4kt and b = 4it + jt – 2kt
a+b 6it + 4jt + 2kt

Position vector of mid point of P and Q == = 3it + 2jt + kt
2 2
If | a | = a, then find the value of the following :
8. [CBSE Bhubneshwar 2015]
2 2 2
a × it + a × jt + a × kt
Sol. Let a makes angle α, β, γ with x, y and z axis.
∴ a × it = a .1. sin a = a sin a similarly | a × jt | = a sin b and | a × kt | = a sin c

380 Xam idea Mathematics–XII

@Cbsebookshub - Join Us on Telegram


∴ | a × it |2 +| a × jt |2 +| a × kt |2 = a2 sin2 α+ a2 sin β + a2 sin2 γ = a2[sin2 α + sin2 β + sin2 γ]

= a2 [1 – cos2 α + 1 – cos2 β + 1 – cos2 γ]
= a2 [3 – (cos2 α + cos2 β + cos2 g)]
a l2 + m2 + n2 = 1
= a 2 (3 – 1) > H
& cos 2 a + cos 2 b + cos 2 c = 1
= 2a2
9. The vectors a = 3it + xjt and b = 2it + jt + ykt are mutually perpendicular. If | a | = | b |, then find
the value of y.[CBSE Bhubneshwar 2015]
Sol.  a and b are mutually perpendicular.

a . b = 0 ⇒ (3it + xjt) . (2it + jt + ykt) = 0 ⇒ 6 + x + 0. y = 0

6 + x = 0 ⇒ x=–6
Again, | a| = | b|

3 2 + x 2 = 2 2 + 1 + y 2 ⇒ 9 + 36 = 5 + y 2 [x = – 6]

45 = 5 + y 2 ⇒ y2 = 45 – 5

y = ! 40 = ! 2 10

10. Find the value of a . b if| a | = 10,| b | = 2 and| a × b | = 16. [CBSE Guwahati 2015]
Sol.  | a × b |= 16 & | a| | b |sin i = 16
16 4
⇒ 10 × 2 sin θ = 16 ⇒ sin i = =
20 5
16 3

⇒ cos i = 1 – sin 2 i = 1– =!
25 5
3
∴ a . b = | a|| b| cos i = ! 10 × 2 × = ! 12
5
11. Find the vector of magnitude 6, which is perpendicular to both the vectors 2it – jt + 2kt and
4it – jt + 3kt . [NCERT Exemplar]
Sol. Let a = 2it – jt + 2kt and b = 4it – jt + 3kt
So, any vector perpendicular to both the vectors a and b is given by
it jt kt
a × b = 2 –1 2 = it (– 3 + 2) – jt (6 – 8) + kt (–2 + 4) = –it + 2jt + 2kt = r [say]
4 –1 3
A vector of magnitude 6 in the direction of r
r –it + 2jt + 2kt –6 t 12 t 12 t
= .6= .6 = i+ j+ k = –2it + 4jt + 4kt
r 12 + 22 + 22 3 3 3

12. Let a = it + 2jt – 3kt and b = 3it – jt + 2kt be two vectors. Show that the vectors (a + b ) and (a – b )
are perpendicular to each other. [CBSE 2019 (65/4/1)]
Sol. We have a = it + 2jt – 3kt and b = 3it – jt + 2kt.
Then, a + b = 4it + tj – kt and a – b = –2it + 3tj –5kt

 (a + b ) . (a – b ) = –8 + 3 + 5 = 0

^a + b h = ^a – b h

Vector Algebra 381


@Cbsebookshub - Join Us on Telegram
13. For any two vectors a and b , prove that (a × b ) 2 = a b – ^ a . b h [CBSE 2019 (65/5/3)]
2 2 2

Sol.  ^ a × b h = a b sin i nt and a .b = a b cos i

RHS = a b – _ a . b i = a
2 2 2 2 2 2 2
b – a b cos 2 i
2 2 2 2
2 2
= a b (1 – cos i) = a b sin i
= ^ a × b h = LHS
2
Hence proved.

Short Answer Questions-II [3 marks]


1. The scalar product of the vector a = it + jt + kt with a unit vector along the sum of the vectors
b = 2it + 4jt – 5kt and c = mit + 2jt + 3kt is equal to 1. Find the value of λ and hence find the unit
vector along b + c .  [CBSE 2019 (65/2/1)]
Sol. We have, b + c = (2it + 4jt – 5kt) + (mit + 2jt + 3kt)

b + c = (2 + m) it + 6jt – 2kt

b +c (2 + m) it + 6jt – 2kt
∴ Unit vector of b + c = =
b +c (2 + m) 2 + 36 + 4
b +c _^2 + mh it + 6jt – 2kti
Now, a . = 1 ⇒ _it + jt + kti . =1
^2 + mh + 40
2
b +c
⇒ 2 + m + 6 – 2 = (2 + m) 2 + 40 ⇒ m + 6 = (2 + m) 2 + 40
Squaring both sides, we have
m 2 + 12m + 36 = (2 + m) 2 + 40 = 4 + m 2 + 4m + 40
8

8m = 44 – 36 = 8 ⇒ m= = 1 ⇒ λ=1
8
3 t 6t 2 t
By putting the value of λ = 1, unit vector of b + c = i+ j– k
7 7 7
2. Let a , b and c be three vectors such that a = 1, b = 2 and c = 3. If the projection of b
along a is equal to the projection of c along a ; and b , c are perpendicular to each other,
then find 3a – 2b + 2c . [CBSE 2019 (65/3/1)]
Sol. Given projection of b along a is equal to the projection of c along a .

b. a c. a

=
a a


b . a = c . a ...(i)
Also given b = c & b . c = 0 ...(ii)
2 2 2 2
Now, 3a – 2b + 2 c =9 a +4 b +4 c – 12 a . b – 8 b . c + 12 a . c

= 9 × (1) 2 + 4 × (2) 2 + 4 × (3) 2 – 12 a . b – 8 × 0 + 12 a . b


= 9 + 4× 4 + 4 × 9 = 9 + 16 + 36 = 61

3a – 2b + 2 c = 61

382 Xam idea Mathematics–XII

@Cbsebookshub - Join Us on Telegram


3. Prove that, for any three vectors a , b , c

[a + b b + c c + a ] = 2 [a b c ] [CBSE Delhi 2014]


Sol. LHS = [a + b b + c c + a ] = (a + b ) . {(b + c ) × ( c + a )}

= (a + b ) . { b × c + b × a + c × c + c × a } = (a + b ) . { b × c + b × a + c × a } [ a c ×c = 0 ]
= a . (b × c ) + a . (b × a ) + a . ( c × a ) + b . (b × c ) + b . (b × a ) + b . ( c × a )

= [a b c ] + [a b a ] + [a c a ] + [ b b c ] + [ b b a ] + [ b c a ]

= [a b c ] + 0 + 0 + 0 + 0 + [b c a ] [By property of scalar triple product]


= [a b c ] + [ b c a ] = [a b c ] + [a b c ] [By property of circularly rotation]
= 2 [a b c ] = RHS
4. Find the value of x such that the points A(3, 2, 1), B(4, x, 5), C(4, 2, –2) and D(6, 5, –1) are coplanar.
[CBSE (AI) 2017]
Sol. We have A(3, 2, 1), B(4, x, 5), C(4, 2, –2) and D(6, 5, –1) points.
AB = it + (x – 2) jt + 4kt; AC = it + 0jt – 3kt; AD = 3it + 3jt – 2kt

 Points A, B, C and D are coplanar ⇒ AB , AC , AD are coplanar ⇒ [ AB AC AD ] = 0

1 x–2 4

1 0 –3 = 0
3 3 –2

1(0 + 9) – (x – 2)(–2 + 9) + 4(3 – 0) = 0 ⇒ 9 – 7x + 14 + 12 = 0

7x = 35 ⇒ x=5

5. Show that the vectors a , b , c are coplanar, iff a + b , b + c and c + a are coplanar.
[CBSE (F) 2014, Delhi 2016]
Sol. If part: Let a , b , c are coplanar

Scalar triple product of a , b and c is zero.


[a b c ] = 0 & a . ( b × c ) = b . ( c ×a ) = c . (a × b ) = 0

Now, [a + b b + c c + a ] = (a + b ) . {(b + c ) × ( c + a )}

= (a + b ) . {b × c + b ×a + c × c + c ×a } = (a + b ) . {b × c + b ×a + c ×a } [a c × c = 0]

= a . (b × c ) + a . (b ×a ) + a . ( c ×a ) + b . (b × c ) + b . (b ×a ) + b . ( c ×a )

= [a b c ] + 0 + 0 + 0 + 0 + [b c a ] [By property of scalar triple product]

= [a b c ] + [a b c ] = 2 [a b c ] = 2 × 0 = 0 [ a [a b c ] = 0]

Hence, a + b , b + c and c + a are coplanar.

Only if part: Let a + b , b + c , c + a are coplanar.


⇒ [a + b b + c c + a ] = 0
⇒ (a + b ) . {(b + c ) × ( c + a )} = 0

⇒ (a + b ) . {b × c + b ×a + c × c + c ×a } = 0
⇒ (a + b ) . {b × c + b ×a + c ×a } = 0 [a c × c = 0]

⇒ a . (b × c ) + a . (b ×a ) + a . ( c ×a ) + b . (b × c ) + b . (b ×a ) + b . ( c ×a ) = 0

Vector Algebra 383


@Cbsebookshub - Join Us on Telegram
⇒ [a b c ] + 0 + 0 + 0 + 0 + [b c a ] = 0
⇒ 2 [a b c ] = 0 [a [a b c ] = [b c a ]]

⇒ [a b c ] = 0

Hence, a , b , c are coplanar.

Let a = it + jt + kt, b = it and c = c1 it + c2 jt + c3 kt then


6.
(a) Let c1 = 1 and c2 = 2, find c3 which makes a , b and c coplanar.
(b) If c2 = –1 and c3 = 1, show that no value of c1 can make a , b and c coplanar.
[CBSE Delhi 2017]

Sol. Given a = it + jt + kt; b = it and c = c1 it + c2 jt + c3 kt

(a) Since a b and c vectors are coplanar


⇒ [a b c ] = 0
1 1 1 1 1 1

1 0 0 = 0 ⇒ 1 0 0 = 0 [Given that c1 = 1 and c2 = 2]
c1 c2 c3 1 2 c3


1(0 – 0) –1(c3 – 0) + 1(2 – 0) = 0

– c3 + 2 = 0 ⇒ c3 = 2

(b) To make a b and c coplanar.


1 1 1 1 1 1

1 0 0 = 0 ⇒ 1 0 0 = 0 [Given that c2 = –1 and c3 = 1]
c1 c2 c3 c1 –1 1


1(0 – 0) – 1(1 – 0) + 1(–1 – 0) = 0 ⇒ –1 – 1 = 0

–2 = 0, which is never possible.
Hence, if c2 = –1 and c3 = 1, there is no value of c1 which can make a , b and c coplanar.
7. If a , b , c are mutually perpendicular vectors of equal magnitudes, show that the vector
a + b + c is equally inclined to a , b and c . Also, find the angle which a + b + c makes
with a or b or c .[CBSE Delhi 2017]
Sol. Let | a| = | b | = | c |= x (say)

Since a , b , c are mutually perpendicular vectors.


Therefore, a . b = b . c = c . a = 0 = b . a = c . b = a . c
Now, | a + b + c |2 = (a + b + c ) . (a + b + c )

= a .a +a .b +a .c +b .a +b .b +b .c +c .a +c .b +c .c
= x2 + 0 + 0 + 0 + x2 + 0 + 0 + 0 + x2 = 3x2

| a + b + c| = 3 x
Let θ1, θ2 and θ3 be the angles made by (a + b + c ) with a , b and c respectively.

a. (a + b + c ) a .a +a .b +a.c x2 + 0 + 0 1

cos i1 = = = 2
=
| a| . | a + b + c | x. 3 x 3x 3

384 Xam idea Mathematics–XII

@Cbsebookshub - Join Us on Telegram


1 1 1

i1 = cos –1 e o similarly i 2 = cos –1 e o and i 3 = cos –1 e o
3 3 3

i.e., (a + b + c ) is equally inclined with a , b and c .


8. If a = it + jt + kt and b = jt – kt , then find a vector c such that a ×c = b and a . c = 3 .


[CBSE Delhi 2008, 2013]
Sol. Let c = c1 it + c2 jt + c3 kt . Then,

it jt kt
(a × c ) = 1 1 1 = (c3 – c2) it + (c1 – c3) jt + (c2 – c1) kt

c1 c2 c3

 (a × c ) = b

(c3 – c2) it + (c1 – c3) jt + (c2 – c1) kt = jt – kt

c3 – c2 = 0, c1 – c3 = 1 and c2 – c1 = – 1 ...(i)

Also, a . c = (it + jt + kt) . (c1 it + c2 jt + c3 kt)

⇒ a . c = c1 + c2 + c3

c1 + c2 + c3 = 3 [ a a . c = 3 ] ...(ii)

c1 + c2 + c1 – 1 = 3 [ a c1 – c3 = 1 ] ... (iii)

2c1 + c2 = 4
On solving c1 – c2 = 1 and 2c1 + c2 = 4 , we get
5
3c1 = 5c1 =
3
&
c2 = (c1 – 1) = c 3 – 1 m = and
5 2 2
∴ c3 = c2 =
3 3
Hence, c = c i + j + k m .
5t 2t 2 t
3 3 3

9. If a + b + c = 0 and a = 3, b = 5 and c = 7 then show that the angle between a and b is 60°.
[CBSE Delhi 2008, 2014]
Sol. a + b + c = 0 ⇒ (a + b ) 2 = (– c ) 2

(a + b ) . (a + b ) = c . c
2 2 2

a + b + 2a . b = c ⇒ 9 + 25 + 2a . b = 49


2a . b = 49 – 25 – 9

2 a b cos i = 15 ⇒ 30 cos i = 15
1

cos i = = cos 60º ⇒ q = 60°
2
10. If a , b , c are three vectors such that a + b + c = 0 , then prove that a × b = b × c = c × a , and
hence show that 8a b c B = 0.  [CBSE Sample Paper 2018]

Sol. Given a + b + c = 0

a # (a + b + c ) = a # 0

a #b + a #c =0 ⇒ a # b = c # a …(i)

Vector Algebra 385


@Cbsebookshub - Join Us on Telegram
Again, b # (a + b + c ) = b # 0

b #a +b #c =0 ⇒ b # c = a # b …(ii)
From (i) and (ii), we get
a #b = b #c = c #a
Now, [a b c ] = a . (b # c )
= a . (a × b ) = [a a b ] = 0 [ Scalar triple product of three vectors
is zero if any two of them are equal.]

11. Let a = it + 4jt + 2kt, b = 3it – 2jt + 7kt and c = 2it – jt + 4kt .

Find a vector p which is perpendicular to both a and b and p . c = 18.[CBSE (AI) 2012]
Sol. Given, a = it + 4jt + 2kt, b = 3it – 2jt + 7kt, c = 2it – jt + 4kt
Vector p is perpendicular to both a and b i.e., p is parallel to vector a × b .

it jt kt
4 2 t 1 2 t 1 4

a ×b = 1 4 2 = it –j +k = 32it – jt – 14kt
–2 7 3 7 3 –2
3 –2 7

Since p is parallel to a # b ⇒ p = n (32it – jt – 14kt)


Also, p . c = 18 ⇒ n (32it – jt – 14kt) . (2it – jt + 4kt) = 18

n (64 + 1 – 56) = 18 & 9n = 18 or n = 2


p = 2 (32it – jt – 14kt) = 64it – 2jt – 28kt

12. The magnitude of the vector product of the vector it + jt + kt with a unit vector along the sum of

vectors 2it + 4jt – 5kt and mit + 2jt + 3kt is equal to 2 . Find the value of l. [CBSE (F) 2013]

Sol. Let a = it + jt + kt; b = 2it + 4jt – 5kt; c = mit + 2jt + 3kt


From question
b+c a × (b + c )
a× = 2 & = 2 ...(i)
b+c b+c

b + c = (2 + m) it + 6jt – 2kt

∴ b + c = (2 + m) 2 + 6 2 + (–2) 2 =
4 + m 2 + 4m + 36 + 4 = m 2 + 4m + 44
it jt kt
⇒ a × (b + c ) =
1 1 1 = (– 2 – 6) it – (– 2 – 2 – m) jt + (6 – 2 – m) kt = – 8it + (4 + m) jt + (4 – m) kt
2 + m 6 –2
Putting it in (i), we get
– 8it + (4 + m) jt + (4 – m) kt (– 8) 2 + (4 + m) 2 + (4 – m) 2
= 2 & = 2
m 2 + 4m + 44 m 2 + 4m + 44
Squaring both sides, we get
64 + 16 + m 2 + 8m + 16 + m 2 – 8m 96 + 2m 2
=2 & =2
m 2 + 4m + 44 2
m + 4m + 44
⇒ 8λ = 8
⇒ λ=1

386 Xam idea Mathematics–XII

@Cbsebookshub - Join Us on Telegram


13. Show that the points A, B, C with position vectors 2it – jt + kt, it – 3jt– 5kt and 3it – 4jt – 4kt
respectively, are the vertices of a right-angled triangle. Hence find the area of the triangle.
[CBSE (AI) 2017]
Sol. Given, position vector of A = 2it – jt + kt
position vector of B = it – 3jt – 5kt
position vector of C = 3it – 4jt – 4kt

AB = –it – 2jt – 6kt ; AC = it – 3jt – 5kt and BC = 2it – jt + kt
2 2 2
Now, AB = AB . AB = 1 + 4 + 36 = 41 ; AC = 1 + 9 + 25 = 35 ; BC = 4+1+1 = 6
2 2 2

 AB = AC
+ BC ⇒ A, B, C are the vertices of right triangle.
it jt kt
Now, AB × AC = –1 –2 –6 = it (10 – 18) – jt (5 + 6) + kt (3 + 2) = –8it –11jt + 5kt
1 –3 –5
2 2 2

AB × AC = (–8) + (–11) + 5 = 64 + 121 + 25 = 210
1 210
∴ Area (∆ABC) =
AB × AC = sq. units
2 2
Alternate method to find area:
1 1 210
Area of ∆ABC = × BC × AC = × 35 × 6 = sq. units
2 2 2
14. Find a unit vector perpendicular to the plane of triangle ABC, where the coordinates of its
vertices are A(3, –1, 2), B(1, –1, –3) and C(4, –3, 1). [CBSE Bhubaneshwar 2015]
Sol. Here, AB = (1 – 3) it + (– 1 + 1) jt + (– 3 – 2) kt B (1, –1, –3)

= – 2it + 0. jt – 5kt

And AC = (4 – 3) it + (– 3 + 1) jt + (1 – 2) kt
AB × AC
= it – 2jt – kt
it jt kt
AB ×AC = –2 0 –5
A (3, –1, 2) C (4, –3, 1)
1 –2 –1

= (0 –10) it – (2 + 5) jt + (4 – 0) kt = –10it –7jt + 4kt

Since, AB # AC is perpendicular to both AB and AC .



AB # AC is perpendicular to the plane of triangle ABC.
AB # AC –10it – 7jt + 4kt 1

Required vector = = 2 2 2
= (– 10it – 7jt + 4kt)
AB # AC (– 10) + (– 7) + 4 165
– 10 t 7 t 4 t
= i– j+ k
165 165 165
15. Find the area of a parallelogram ABCD whose side AB and the diagonal AC are given by the
vectors 3it + jt + 4kt and 4it + 5kt respectively. [CBSE (F) 2017]
D C
Sol. Here, BC = BA + AC = – AB + AC
= – 3it – jt – 4kt + 4it + 5kt = it – jt + kt


` AD = BC = it – jt + kt
A B

Vector Algebra 387


@Cbsebookshub - Join Us on Telegram
∴ Area of parallelogram = AB # AD
it jt kt
= | 3 1 4 | = (1 + 4) it – (3 – 4) jt + (–3 – 1) kt = 5it + jt – 4kt
1 –1 1
= 5 2 + 1 2 + (–4) 2 = 25 + 1 + 16 = 42 sq. units.

16. If a = 2it – jt – 2kt and b = 7it + 2jt – 3kt then express b in the from of b = b 1 + b 2, where b 1 is
parallel to a and b 2 is perpendicular to a .[CBSE (AI) 2017]
Sol. Since b 1 || a
&
b 1 = ma = m (2it – jt – 2kt) = 2mit – mjt – 2mkt

a b2 = b – b1 b1 + b2 = b &
= (7it + 2jt – 3kt) – (2mit –mjt – 2mkt) = 7it + 2jt – 3kt – 2mit + mjt + 2mkt
= (7 – 2m) it + (2 + m) jt – (3 – 2m) kt
It is given that b 2 is perpendicular to a .

b 2 . a = 0 ⇒ (7 – 2λ).2 – (2 + λ).1 + (3 – 2λ).2 = 0
18

14 – 4λ – 2 – λ + 6 – 4λ = 0 ⇒ –9λ + 18 = 0 ⇒ m= =2
9
Hence, b 1 = 4it – 2jt – 4kt; b 2 = 3it + 4jt + kt
Now, 7it + 2jt – 3kt = (4it – 2jt – 4kt) + (3it + 4jt + kt), i.e., b = b 1 + b 2

17. Given that vectors a , b , c form a triangle such that a = b + c .Find p, q, r, s such that area of
triangle is 5 6 where a = pit + qjt + rkt, b = sit + 3jt + 4kt and c = 3it + jt – 2kt .[CBSE (South) 2016]
Sol. Given, a = b + c
C
⇒ pit + qjt + rkt = (sit + 3jt + 4kt) + (3it + jt – 2kt)
⇒ pit + qjt + rkt = (s + 3) it + 4jt + 2kt
Equating the co-efficient of it, jt, kt from both sides, we get a


s + 3 = p; q = 4 and r = 2 ...(i)
1
Now, area of triangle =
2 b ×c A B
it jt kt
1 1
⇒ 5 6 = | s 3 4 | = (– 6 – 4) it – (– 2s – 12) jt + (s – 9) kt
2 2
3 1 –2
1 1

5 6= 10 2 + (2s + 12) 2 + (s – 9) 2 = 100 + 4s 2 + 144 + 48s + s 2 + 81 – 18s
2 2
1

325 + 5s 2 + 30s
5 6=
2
Squaring both sides
1

150 = (325 + 5s 2 + 30s)
4

600 – 325 = 5s2 + 30s ⇒ 5s2 + 30s – 275=0
–30 ! 900 + 4×5×275 –30 ! 6400 –30 ! 80

s = = =
10 10 10

388 Xam idea Mathematics–XII

@Cbsebookshub - Join Us on Telegram



s = –11, 5 ...(ii)
From (i) and (ii)
s = –11, 5; p = – 8, 8; q = 4 and r = 2

18. If a and b are unit vectors, then what is the angle between a and b for a – 2 b to be a
unit vector?  [CBSE South 2016]

Sol. Given, a – 2 b is an unit vector


2

a – 2 b = 1 ⇒ a – 2 b =1


( a – 2 b).( a – 2 b) = 1 ⇒ a . a – 2 a . b – 2 b . a +2 b . b =1
2 2
⇒ a –2 2 a . b + 2 b = 1 [a a . b = b . a ]

1 – 2 2 a . b + 2 = 1 [a a = b = 1]
–2

–2 2 a . b = –2 ⇒ a. b =
–2 2
1 1

a .b = ⇒ a . b cos i = [a a . b = a . b cos i]
2 2
1 r r
⇒ 1 . 1 . cos i = ⇒ cos i = cos ⇒ i=
2 4 4
19. The two vectors jt + kt and 3it –jt + 4kt represent the two side vectors AB and AC respectively
of triangle ABC. Find the length of the median through A.[CBSE (F) 2015]

Sol. Here, AB = jt + kt and AC = 3it – jt + 4kt


A

BC = BA + AC
= –AB + AC = – jt – kt + 3it – jt + 4kt = 3it – 2jt + 3kt
1 j+k 3i – j + 4k

 BD = BC
2
1 3 3
= (3it –2jt + 3kt) = it –jt + kt
2 2 2
Now, AD = AB + BD B D C

= (jt + kt) + c it – jt + kt m = it + kt
3 3 3 5

2 2 2 2
3 2 5 2
c m +c m =
34
Length of AD = AD = units .
2 2 2
1
20. If a , b and c determine the vertices of a triangle, show that 2 [b × c + c × a + a × b ] gives the
vector area of the triangle. Hence, deduce the condition that the three point a , b and c are
collinear. Also, find the unit vector normal to the plane of the triangle. [NCERT Exemplar]

Sol. Since, a b and c are the vertices of a DABC as shown. C


1
∴ Area of DABC = 2 AB × AC
a

c–
c–

Now, AB = b – a and AC = c – a
b

1

Area of DABC = 2 [(b – a ) × ( c – a )] A B
b –a

Vector Algebra 389


@Cbsebookshub - Join Us on Telegram
= 1 ( b × c ) – ( b × a ) – (a × c ) + (a × a )
2

= 1 ( b × c ) + (a × b ) + ( c × a ) + 0
2

= 1 (b × c ) + (a × b ) + ( c × a ) ...(i)
2
For three points to be collinear, area of the DABC should be equal to zero.
1 + + =

2 [b × c c × a a × b ] 0

b × c + c × a + a × b = 0 ...(ii)
This is the required condition for collinearity of three points a , b and c .
Let nt be the unit vector normal to the plane of the DABC.

AB × AC + +

nt = = a ×b b ×c c ×a
AB × AC a ×b b ×c + c ×a
+
a ×b
21. Show that area of the parallelogram whose diagonals are given by a and b is 2 . Also, find
the area of the parallelogram, whose diagonals are 2it – jt + kt and it + 3jt – kt. [NCERT Exemplar]

Sol. Let ABCD be a parallelogram such that


AB = p , AD = q & BC = q
q
By triangle law of addition, we get a b
AC = p + q = a [say] ...(i)
Similarly, BD = – p + q = b [say] ...(ii)
On adding equation (i) and (ii), we get
a + b = 2 q & q = 2 ^a + b h
1

On subtracting equation (ii) from equation (i), we get
a – b = 2p & p = 2 ^ a + b h
1

p × q = 4 ^a – b h × ^a + b h = 4 ^a × a + a × b – b × a – b × b h
1 1
Now,

   = 4 [a × b + a × b ] = 2 ^a × b h
1 1

1
So, area of a parallelogram ABCD = p × q = 2 a × b

Now, area of a parallelogram, whose diagonals are 2it – jt + kt and it + 3jt – kt .

= 2 _ 2it – jt + kti × _it + 3jt – kti


1

it jt kt
1
= 2 | 2 –1 1 |
1 3 –1
1
= 2 [it (1 – 3) – jt (–2 – 1) + kt (6 + 1)]
1
= 2 –2it + 3jt + 7kt
1
= 2 4 + 9 + 49

1
= 2 62 sq. units

390 Xam idea Mathematics–XII

@Cbsebookshub - Join Us on Telegram


22. If a ×b = c ×d and a × c = b × d , then show that (a – d ) is parallel to (b – c ) , it is being given
that a ! d and b ! c .[CBSE (F) 2016] [HOTS]

Sol. Given, a ×b = c ×d and a × c = b ×d


a ×b – a × c = c ×d – b ×d

⇒ a × b – a × c + b ×d – c ×d = 0


a × (b – c ) + (b – c ) ×d = 0 [By left and right distributive law]


a × (b – c ) – d × (b – c ) = 0 [ a a × b = – b × a ]


(a – d ) × (b – c ) = 0 [By right distributive law]


(a – d ) || (b – c )

= a . a a . b [HOTS]
2
23. Prove that : a ×b
a. b b . b

Sol. Let q be the angle between a and b . Then,


2
LHS = a ×b = (a × b ) . (a × b )
= (ab sin i) nt . (ab sin i) nt = (a 2 b 2 sin 2 i) (nt .nt ) = a 2 b 2 sin 2 i
= a 2 b 2 (1– cos 2 i) = a 2 b 2 – (ab cos i) 2

= (a . a ) (b . b ) – _ a . b i ...(i)
2

a. a a. b =
Also, RHS = (a . a ) . ( b . b ) – ( a . b ) . ( a . b )
a. b b. b

= (a . a ) . (b . b ) – (a . b ) 2 ...(ii)
From (i) and (ii) RHS = LHS Hence proved.

24. If a , b are unit vectors such that the vector a + 3 b is perpendicular to 7 a – 5 b and a – 4 b
is perpendicular to 7 a – 2 b , then find the angle between a and b . [HOTS]

Sol. Let angle between a and b be q

Given, (a + 3 b ) = (7 a – 5 b ) ⇒ ( a + 3 b ) . (7 a – 5 b ) = 0
2 2

7 a + 16 (a .b ) – 15 b =0
2 2

7+16 cos q – 15= 0 [a a = b = 1]
8 1 r

cos i = = ⇒ i=
16 2 3
Also, given that (a – 4 b ) = (7 a – 2 b )
2 2

(a – 4 b ) . (7 a – 2 b ) = 0 ⇒ 7 a +8 b – 30 (a .b ) = 0
1 r

15 – 30 cos q = 0 ⇒ cos i = ⇒ i=
2 3

Vector Algebra 391


@Cbsebookshub - Join Us on Telegram
25. Find the value of x such that the four points with position vectors,
A (3it + 2jt + kt), B (4it + xjt + 5kt), C (4it + 2jt – 2kt) and D (6it + 5jt – kt) are coplanar .
 [CBSE 2019 (65/4/3)]

Sol. Given, AB = it + (x – 2) jt + 4kt, AC = it + 0jt – 3kt, AD = 3it + 3jt – 2kt


As A, B, C, D are coplanar so AB . (AC × AD ) = 0
1 x–2 4

1 0 –3 = 0 ⇒ 1(9) – (x – 2) 7 + 4(3) = 0 ⇒ 9 – 7x + 14 + 12 = 0
3 3 –2

35 = 7x ⇒x= 5

PROFICIENCY EXERCISE
QQ Objective Type Questions: [1 mark each]
1. Choose and write the correct option in each of the following questions.
1
(i) If a . b = b , then the angle between a and b is [CBSE 2020 (65/4/1)]
2 a
(a) 0° (b) 30° (c) 60° (d) 90°

(ii) Let a and b be two unit vectors and θ is the angle between them. Then a + b is unit vector if q is
r r r 2r
(a) (b) (c) (d)
4 3 2 3
(iii) The magnitude of the vector 6it + 2jt + 3kt is [NCERT Exemplar]
(a) 5 (b) 7 (c) 12 (d) 1
2
(iv) Let a = it – 2jt + 3kt . If b is a vector such that a . b = b and a – b = 7 then b equals
 [CBSE 2020 (65/4/2)]
(a) 7 (b) 14 (c) 7 (d) 21
2 2
(v) If a # b = 4 and a . b = 2 then a b is equal to
(a) 2 (b) 6 (c) 8 (d) 20
(vi) The value of p for which p (it + jt + kt) is a unit vector is [CBSE 2020 (65/3/1)]
1
(a) 0 (b) (c) 1 (d) 3
3
2.
Fill in the blanks. [1 mark each]
t t
(i) The area of the parallelogram whose diagonals are 2i and –3k is ___________ square units.
 [CBSE 2020 (65/3/1)]

(ii) The sine of the angle between vectors a = 2it – 6jt – 3kt and b = 4it + 3jt – kt is equal to _________.

(iii) The value of l for which the vectors 2it – mjt + kt and it + 2jt – kt are orthogonal is ____________.
 [CBSE 2020 (65/3/1)]

(iv) If a = 3it – 2jt + 2kt, b = 6it + 4jt – 2kt and c = –3it – 2jt + 4kt. Then a . _ b # c i is equal to _________.

(v) The vectors a = 3it – 2jt + 2kt and b = –it –2kt are the adjacent sides of a parallelogram. The acute
angle between its diagonals is _____________ .

392 Xam idea Mathematics–XII

@Cbsebookshub - Join Us on Telegram


QQ Very Short Answer Questions: [1 mark each]
3. If a = 3 , b = 2 and angle between a and b is 60°, then find a . b .  [CBSE (AI) 2008]

4. Find the sum of the vectors a = it – 2jt + kt, b = – 2it + 4jt + 5kt and c = it – 6jt – 7kt . [CBSE Delhi 2012]

5. Find the angle between two vectors a and b with magnitudes 1 and 2 respectively and when
a × b = 3 . [CBSE Delhi 2009]
Find a vector of magnitude
6. 171 , which is perpendicular to both of the vectors a = it + 2jt – 3kt
and b = 3it – jt + 2kt.  [CBSE Ajmer 2015]
Write the distance of the point (3, –5, 12) form X-axis.
7. [CBSE (F) 2017]
If a . a = 0 and a . b = 0, then what can be concluded about the vector b .
8. [CBSE (F) 2011]
If a = 4it – jt + kt and b = 2it – 2jt + kt, then find a unit vector parallel to the vector a + b .
9.
 [CBSE (North) 2016]
10. If a = 2it + jt + 3kt and b = 3it + 5jt – 2kt, then find| a × b | .  [CBSE Panchkula 2015]
r r
11. If a unit vector a makes angles with i , with j and an acute angle θ with kt , then find the
t t
3 4
value of θ. [CBSE Delhi 2013]
12. In a triangle OAC, if B is the mid-point of side AC and OA = a . OB = b , then what is OC .
 [CBSE Ajmer 2015]
2
13. If| a × b| +| a . b| = 400 and| a| = 5, then write the value of| b | .  [CBSE (F) 2016]
14. If at, bt and ct are mutually perpendicular unit vectors, then find the value of 2at + bt + ct .
[CBSE Allahabad 2015]
t t t
15. Find a unit vector in the direction of a = 3i – 2j + 6k . [CBSE Delhi 2008]
16. Write a vector of magnitude 9 units in the direction of vector –2it + jt + 2kt .[CBSE (AI) 2010]

QQ Short Answer Questions–I: [2 marks each]

17. Find a vector of magnitude 5 units and parallel to resultant of the vectors a = 2it + 3it – kt and
b = it – 2jt + kt . [CBSE Allahabad 2015]

18. For any three vectors a , b and c , find the value of a × (b + c ) + b × ( c + a ) + c × (a + b ) .


 [CBSE (F) 2013]

19. Find | x |, if for a unit vector a

(x – a ) . (x + a ) = 15 . [CBSE (F) 2010]

20. If a and b are two unit vectors such that a + b is also a unit vector, then find the angle between
a and b . [CBSE (AI) 2014]
21. Find the value of a + b, if the points (2, a, 3)(3, –5, b) and (–1, 11, 9) are collinear. [CBSE Guwahati 2015]

22. Find a vector r equally inclined to the three axes and whose magnitude is 3 3 units.
 [CBSE 2020 (65/2/1)]

23. Find the angle between unit vectors a and b so that 3 a – b is also a unit vector.
 [CBSE 2020 (65/2/1)]

Vector Algebra 393


@Cbsebookshub - Join Us on Telegram
24. Find a and b , if a = 2 b and _ a + b i . _ a – b i = 12 [CBSE 2020 (65/4/1)]

25. Find the unit vector perpendicular to each of the vectors a = 4it + 3jt + kt and b = 2it – jt + 2kt .
 [CBSE 2020 (65/4/1)]
QQ Short Answer Questions–II: [3 marks each]
26. Find a unit vector perpendicular to both of the vectors a + b and a – b where a = it + jt + kt ,
b = it + 2jt + 3kt.  [CBSE (F) 2014]

27. If p = 5it + mjt – 3kt and q = it + 3jt – 5kt then find the value of m , so that p + q and p – q are
perpendicular vectors. [CBSE (AI) 2013]

28. Let a = it + 4jt + 2kt, b = 3it – 2jt + 7kt and c = 2it – jt + 4kt . Find a vector d which is perpendicular to
both a and b and c . d = 27. [CBSE Ajmer 2015]
29. Show that the four points with position vectors 4it + 8jt + 12kt, 2it + 4jt + 6kt, 3it + 5jt + 4kt and 5it + 8jt + 5kt
are coplanar. [CBSE Guwahati 2015]
30. Find x such that four points A(4, 1, 2), B(5, x, 6), C(5, 1, –1) and D(7, 4, 0) are coplanar.
 [CBSE Panchkula 2015]

31. For three vectors a , b and c if a × b = c and a × c = b , then prove that a , b and c are
mutually perpendicular vectors, b = c and a = 1 . [CBSE Sample Paper 2015]
r
32. If a , b , c are unit vectors such that a . b = a . c = 0 and the angle between b and c is , then
6
prove that (i) a = ! 2 (b # c ) (ii) 7a + b b + c c + a A = ! 1 . [CBSE Sample Paper 2016]
33. The two adjacent sides of a parallelogram are 2it – 4jt – 5kt and 2it + 2jt + 3kt . Find the two unit
vectors parallel to its diagonals. Using the diagonal vectors, find the area of the parallelogram.
 [CBSE (Central) 2016]

34. Find the angle between the vectors a + b and a – b if a = 2it – jt + 3kt and b = 3it + jt – 2kt , and hence
find a vector perpendicular to both a + b and a – b . [CBSE (East) 2016]
35. If a = 2it + jt – kt, b = 4it – 7jt + kt, find a vector c such that a × c = b and a . c = 6 . [CBSE (F) 2017]
36. Using vectors find the area of triangle ABC with vertices A(1, 2, 3), B(2, –1, 4) and C(4, 5, –1).
[CBSE Delhi 2017]
37. Find the value of x such that the four points with position vectors, A _ 3it + 2jt + kt i, B _ 4it + xjt + 5kt i,
C _ 4it + 2jt – 2kt i and D _6it + 5jt – kti are coplanar. [CBSE 2019 (65/4/3)]

38. If a = it + 2jt + 3kt and b = 2it + 4jt – 5kt represent two adjacent sides of a parallelogram, find unit
vectors parallel to the diagonals of the parallelogram. [CBSE 2020 (65/5/1)]
39. Using vectors, find the area of the triangle ABC with vertices A(1, 2, 3), B(2, –1, 4) and C(4, 5, –1).
 [CBSE 2020 (65/5/1)]

Answers
1. (i) (c) (ii) (d) (iii) (b) (iv) (c) (v) (d) (vi) (b)
5 1 r
2. (i) 3 sq. units (ii) (iii) m = (iv) 72 (v)
26 2 4

394 Xam idea Mathematics–XII

@Cbsebookshub - Join Us on Telegram


r
3. 3 4. –4jt – kt 5. 6. it – 11jt – 7kt 7. 13 units
3
9. _6it – 3jt + 2kti
1 r
8. b may be any vector 10. 507 11.
7 3
15. _ 3it – 2jt + 6kti
1
12. 2b – a 13. b = 4 14. 6 16. –6it + 3jt + 6kt
7
_ 3it + jti
5 2r
17. ! 18. 0 19. 4 20. 21. a + b = 0
10 3

22. r = ! 3 _it + jt + kti


r
23. 24. a = 4, b = 2
6
_7it – 6jt – 10kti
1 1 t 2 t 1 t
25. 26. – i+ j– k 27. m = ! 1
185 6 6 6
28. d = 96it – 3jt – 42kt 30. x = 4

_ 4it – 2jt – 2kti, ^6i + 8kth and area of ;; gm = 2 101 sq. units 34. i = ; 2it – 26jt – 10kt
1 1 t r
33.
2 6 10 2
1
35. c = 3it + 2jt + 2kt 36. 274 sq. units 37. x = 5
2
38. ! _ 3it + 6jt – 2kti, ! _i + 2j – 8kti
1 1 t t 1
39. 274 sq. units
7 69 2

SELF-ASSESSMENT TEST
Time allowed: 1 hour Max. marks: 30

1.
Choose and write the correct option in the following questions. (4 × 1 = 4)
(i) The area of the parallelogram having diagonals d1 = 3it + jt – 2kt and d2 = it – 3jt + 4kt is equal to
(a) 8 (b) 9 (c) 10 3 (d) 5 3
(ii) If a = 3 and –1 # k # 2, then k a lies in the interval
(a) [0, 6] (b) [–3, 6] (c) [3, 6] (d) [1, 2]

(iii) If a = 10, b = 2 and a . b = 12, then value of a # b is


(a) 5 (b) 10 (c) 14 (d) 16
(iv) If O and O′ are circumcentre and orthocentre of ∆ABC, then OA + OB + OC equals
(a) 2 Ol O (b) Ol O (c) OOl (d) 2 OOl
2.
Fill in the blanks. (2 × 1 = 2)
(i) If a # b = 1 and a . b = 3 , then acute angle between a and b is _____________ .

(ii) The projection of a = it – 2jt + kt on b = 4it – 4jt + 7kt is _____________ .

QQ Solve the following questions. (2 × 1 = 2)


Find the area of a parallelogram whose adjacent sides are given by the vectors a = 3it + jt + 4kt and
3.
b = it – jt + kt .
4. Find a unit vector in the direction of a = 3it – 2jt + 6kt .

QQ Solve the following questions. (4 × 2 = 8)

5. If a and b are perpendicular vectors, a + b =13 and a = 5 find the value of b .

Vector Algebra 395


@Cbsebookshub - Join Us on Telegram
6. If a is a unit vector and (2 x – 3 a ) . (2 x + 3a ) = 91 then find x .

7. Find | x |, if for a unit vector a

(x – a ) . (x + a ) = 15 .
8. Find the value of a + b, if the points (2, a, 3)(3, –5, b) and (–1, 11, 9) are collinear.

QQ Solve the following questions. (3 × 3 = 9)

9. If a and b are two vectors, then prove that a – b # a + b .

10. Given that a = it + 2jt + kt, b = 3it + 2jt – 7kt and c = 5it + 6jt – 5kt , verify that

a × ( b × c ) = (a . c ) b – ( a . b ) c .
11. Find a vector whose magnitude is 3 units and which is perpendicular to the following two vectors:
a = 3it + jt – 4kt; b = 6it + 5jt – 2kt.

QQ Solve the following question. (1 × 5 = 5)

12. Let u , v and w be vectors such that u + v + w = 0 . If u = 3, v = 4 and w = 5 , find


u. v + v. w + w. u .

Answers
1. (i) (d) (ii) (a) (iii) (d) (iv) (c)
r 19
2. (i) (ii)
6 9
1 t t
3. 42 sq. units 4. (3i –2j + 6kt) 5. | b |= 12 6. | x |= 5
7
7. 4 8. a + b = 0 11. 2it –2jt + kt 12. –25
zzz

396 Xam idea Mathematics–XII

@Cbsebookshub - Join Us on Telegram


Three Dimensional 12
Geometry

1. Distance between two given points P(x1, y1, z1) and Q(x2, y2, z2) is
PQ = (x2 – x1) 2 + (y2 – y1) 2 + (z2 – z1) 2 .

2. Direction ratios of a line joining the points (x1, y1, z1) and (x2, y2, z2) are x2 – x1, y2 – y1, z2 – z1.
3. Angle between two lines, whose direction ratios are a1, b1, c1 and a2, b2, c2 is given by
a1 a2 + b1 b2 + c1 c2
cos i = .
a12 + b12 + c12 . a 22 + b 22 + c 22
(i) If lines are perpendicular, then a1a2 + b1b2 + c1c2 = 0.
a1 b1 c1
(ii) If lines are parallel, then = = .
a2 b2 c2
4. Vector equation of a straight line passing through a fixed point with the position vector a and
parallel to a given vector b is r = a + m b , where l is a parameter and r = xit + yjt + zkt.

5. Cartesian equation (symmetrical form) of the straight line passing through a fixed point (x1, y1, z1)
x – x1 y – y1 z – z1
having the direction ratios a, b, c is given by = = .
a b c

x – x1 y – y1 z – z1
6. The parametric equations of the line = = are
a b c
x = x1 + al, y = y1 + bl, z = z1 + cl, where l is a parameter.
x – x1 y – y1 z – z1
7. The coordinates of any point on the line = = are
a b c
(x1 + al, y1 + bl, z1 + cl), where l ∈ R.
8. Equation of straight line passing through the point (x1, y1, z1) having direction cosines l, m, n is
x – x1 y – y1 z – z1
= = .
l m n
9. Vector equation of two straight lines passing through two given points with position vector a and
b is r = a + m (b – a ) , where l is a parameter.

10. Cartesian equation of a straight line passing through two given points A (x1, y1, z1) and B (x2, y2, z2)
x – x1 y – y1 z – z1
is given by = = .
x2 – x1 y2 – y1 z2 – z1

Three Dimensional Geometry 397


@Cbsebookshub - Join Us on Telegram
11. Angle between two straight lines: Angle between two straight lines whose vector equations are
r = a1 + mb1 and r = a2 + mb2 , is equal to the angle between b1 and b2 , because b1 and b2 are
parallel vector to the lines r = a1 + mb1 and r = a2 + mb2 respectively.
If q is angle between both lines, then
b1 . b2
cos i =

+ 

b2
b1 . b2 


b2

a2
r=

(i) If b1 . b2 = 0 , then cos q = 0°  

b1


q = 90° ⇒ b1 = b2 
  
r = a1 + b1

Both lines are perpendicular to each other.
(ii) If b1 = mb2 , then
2
mb2 . b2 m b2

cos i = = 2
m b2 . b2 m b2

⇒ cos q = 1
⇒ q = 0° ⇒ b1 || b2

Both lines are parallel to each other.
12. Shortest distance between two lines: Let l1 and l2 be two skew lines given by r = a1 + mb1 and
r = a2 + mb2 respectively, where a1 and a2 are position vectors of points on l1 and l2 then shortest
distance between two given points is given by

(a2 – a1) . (b1 ×b2)



b1 × b2
Note: If two lines are intersecting, then shortest distance between them is zero, i.e.,

(a2 – a1) . (b1 ×b2) = 0


13. Shortest distance between two parallel lines: Let l1 and l2 be two parallel lines given by r = a1 + mb
b . (a2 – a1)
and r = a2 + mb respectively. Then shortest distance between them is .
b
x – x1 y – y1 z – z1
14. Shortest distance between two skew lines in cartesian form: Let = = and
a1 b1 c1
x – x2 y – y2 z – z2
= = are two skew lines, then shortest distance between them is given by
a2 b2 c2
x2 – x1 y2 – y1 z2 – z1
a1 b1 c1 .
a2 b2 c2
x2 – x1 y2 – y1 z2 – z1

Note: If a1 b1 c1 = 0 . Then lines are intersecting.
a2 b2 c2

15. Equation of a plane passing through given point, whose position vector is a and perpendicular to
n
a given vector n , is r . n = a . n or ( r – a ) . n = 0 or ( r – a ) . nt = 0 when nt = .
n

398 Xam idea Mathematics–XII

@Cbsebookshub - Join Us on Telegram


16. Cartesian equation of plane passing through a given point (x1, y1, z1) and perpendicular to the
normal, whose direction ratios are a, b, c respectively is given by
a (x – x1)+b(y – y1)+c (z –z1) =0
17. Equation of a plane in normal form:
(i) When a unit vector (nt ) perpendicular (normal) to the plane is given and its perpendicular
distance d from the origin is also given, then the equation of plane is r . nt = d.
(ii) If unit vector nt = lit + mjt + nkt where l, m, n are direction cosines and p, the perpendicular
distance from origin to normal are given, then equation of the plane is lx +my +nz = p.
18. Angle between line and plane: Angle between line r = a + mb and plane r . n = d is complementary
to the angle between line and the normal to the plane.
Let q be the angle between line and plane and φ be the angle between line and normal of plane.
b. n

Now, cos z =
b . n
   

 q = 90 – f or f = 90 – q 
r b r = a + b

b. n  90–  = 
∴ cos (90 – i) =  
.n=d
b . n
b. n
⇒ sin i =
b . n

m n. n
(i) If b = mn , then sin i = = 1 ⇒ q = 90º
m n. n

Line is perpendicular to the plane.
(ii) If b . n = 0 ⇒ sin q = 0 ⇒    q = 0º

Line is parallel to the plane.
19. Angle between two planes: The angle between two planes is defined as the angle between their
normals.

Case I. If q be the angle between normals n1 and n2 of the planes r . n1 = d1 and r . n2 = d2



respectively, then
n1 . n2
cos i =
n1 . n2 
n1
Note: (i) If n1 . n2 = 0 then given planes are perpendicular.

(ii) If n1 = mn2 , then both planes are parallel.


(iii) The angle between two planes is always taken as acute

angle.  n2

Case II. If a1 x + b1 y + c1z + d1 = 0 and a2x + b2y + c2z + d2 = 0 be two


planes and q is the angle between them, where a1, b1, c1 and a2, b2, c2
are direction ratios of normals to the planes, then
a1 a2 + b1 b2 + c1 c2
cos q =
a1 + b12 + c12 . a 22 + b 22 + c 22
2

Three Dimensional Geometry 399


@Cbsebookshub - Join Us on Telegram
(i) If a1 a2 + b1b2 + c1c2 = 0, then planes are perpendicular to each other.
a1 b1 c1
(ii) If = = then given planes are parallel to each other.
a2 b2 c2
Determination of plane under given conditions:
20. An equation of first degree in x, y, z of the form ax + by + cz + d = 0 where at least one of a, b, c is
non-zero real number, i.e., a2 + b2 + c2 ≠ 0 represents a plane.
x y z
21. Equation of plane in intercept form: + + = 1 where a, b, c are intercepts made by the plane
a b c
on the x-axis, y-axis and z-axis respectively.
22. Equation of plane passing through three given points:

Case I. Vector equation of the plane passing through three given points having position vector
a , b and c is given by

[( r – a ) (b – a ) ( c – a )] = 0

or ( r – a ) . (b – a ) × ( c – a ) = 0
Case II. Cartesian equation of the plane passing through points A(x1, y1, z1), B(x2, y2, z2) and

C(x3, y3, z3) is given by

x – x1 y – y1 z – z1
x2 – x1 y2 – y1 z2 – z1 = 0
x3 – x1 y3 – y1 z3 – z1

23. Condition for coplanarity of two lines:


Case I. When lines are in vector form:
(i) Let r = a1 + m b1 and r = a2 + m b2 be two lines then these lines are coplanar if

[a2 – a1 b1 b2] = 0 i.e a a2 –a1 k . a b1 × b2 k = 0


(ii) Equation of plane containing these two lines is

( r – a1) . (b1 ×b2) = 0 or ( r – a2) . (b1 ×b2) = 0 .

Case II. When lines are in cartesian form:


x – x1 y – y1 z – z1 x – x2 y – y2 z – z2
(i) Let = = and = = be the two lines
a1 b1 c1 a2 b2 c2
then these lines are coplanar iff

x2 – x1 y2 – y1 z2 – z1
a1 b1 c1 = 0
a2 b2 c2
(ii) Equation of plane containing these lines is
x – x1 y – y1 z – z1 x – x2 y – y2 z – z2
= = or = =
b1 c2 – b2 c1 a2 c1 – a1 c2 a1 b2 – a2 b1 b1 c2 – b2 c1 a2 c1 – a1 c2 a1 b2 – a2 b1

(iii) The length of perpendicular from a point having position vector a to the plane

r . nt = d is given by d – a . nt .

400 Xam idea Mathematics–XII

@Cbsebookshub - Join Us on Telegram


(iv) The length of perpendicular from a point P (x1, y1, z1)to the plane ax + by + cz + d = 0 is given by

ax1 + by1 + cz1 + d



a2 + b2 + c2

Selected NCERT Questions


1. If a line makes angles 90°, 135°, 45° with the x, y and z axis respectively, find its direction
cosines.
Sol. Since the line makes angle 90°, 135°, 45° with the x, y and z axis respectively
then α = 90°, b =135° and g = 45°
1 1
l = cos 90° = 0, m = cos 135° = cos (180 – 45)° = – cos 45° = – and n = cos 45° =
2 2
–1 1

Thus, direction cosines of the line are 0, and .
2 2
2. Show that the line through the points (1, –1, 2), (3, 4, –2) is perpendicular to the line through
the points (0, 3, 2) and (3, 5, 6).
Sol. Let A (1, –1, 2) and B (3, 4, – 2) be given points.
Direction ratios of AB are
(3 – 1), {(4 – (–1)}, (–2 –2) i.e., 2, 5, – 4.
Let C (0, 3, 2) and D (3, 5, 6) be given points.
Direction ratios of CD are
(3 – 0), (5 – 3), (6 – 2) i.e., 3, 2, 4.
We know that two lines with direction ratios a1, b1, c1 and a2 , b2 , c2 are perpendicular if
a1a2 + b1b2 + c1c2 = 0.

∴ 2 × 3 + 5 × 2 + (– 4) × 4 = 6 +10 –16 = 0, which is true.

It will shows that lines AB and CD are perpendicular.
3. Find the Cartesian equation of the line which passes through the point (–2, 4, –5) and parallel
x+3 y – 4 z+8
to the line given by = = .
3 5 6
Sol. The equation of given line is
x+3 y – 4 z+8
= = .
3 5 6
The direction ratios of the given line are 3, 5, 6. Since the required line is parallel to given line, so,
the direction ratios of required line are proportional i.e., 3, 5, 6.
Now the equation of the line passing through point (– 2, 4, –5) and having direction ratios 3, 5, 6 is
x+2 y – 4 z+5
= =
3 5 6
which is equation of required line.
4. Find the angle between the following pair of lines:
x y z x–5 y–2 z–3
= = and = = .
2 2 1 4 1 8
Sol. Here the equation of given lines are
x y z x–5 y–2 z–3
= = and = =
2 2 1 4 1 8

Three Dimensional Geometry 401


@Cbsebookshub - Join Us on Telegram
∴ Direction ratios of two lines are 2, 2, 1 and 4, 1, 8.
Let θ be the angle between two given lines then
a1 a2 + b1 b2 + c1 c2
cos i =
a1 + b12 + c12 a 22 + b 22 + c 22
2

2#4+2#1+1#8 8+2+8

= =
(2) + (2) 2 + (1) 2 . (4) 2 + (1) 2 + (8) 2 .
2 4 + 4 + 1 16 + 1 + 64

2 2
∴ cos i =

3
& i = cos –1 .
3
Find the value of p so that the lines
5.
1 – x 7y –14 5z – 10 7 – 7x y – 5 6 – z
= = and = = [CBSE Delhi 2009]
3 2p 11 3p 1 5
are perpendicular to each other.
Sol. The given lines
1 – x 7y – 14 5z – 10 7 – 7x y – 5 6 – z
= = and = = are rearranged to get
3 2p 11 3p 1 5
x–1 y–2 z–2
= = ...(i)
–3 2p/7 11/5
x–1 y–5 z–6
= = ...(ii)
–3p/7 1 –5
Direction ratios of lines are
2p 11 –3p
–3, , and , 1, –5
7 5 7
As the lines are perpendicular, we get

–3 c m+
–3p 2p 11
∴ ×1 + (–5) = 0
7 7 5
9p 2p 11


7
+
7
–11 = 0 & 7
p = 11 & p=7

6. Find the shortest distance between the lines :

r = (it + 2jt + kt) + m (it – jt + kt) and r = (2it – jt – kt) + n (2it + jt + 2kt) [CBSE (F) 2011]
Sol. Given lines are
r = (it + 2jt + kt) + m (it – jt + kt) … (i)

r = (2it – jt – kt) + n (2it + jt + 2kt) … (ii)

Comparing the equation (i) and (ii) with r = a1 + m b1 and r = a2 + m b2 , we get

a1 = it + 2jt + kt a2 = 2it – jt – kt

b1 = it – jt + kt b2 = 2it + jt + 2kt

Now, a2 – a1 = (2i – jt –kt) – (it + 2jt + kt) = it – 3jt – 2kt


it jt kt
b ×b = 1 –1 1 = (–2–1) it – (2 – 2) jt + (1 + 2) kt = –3it + 3kt
1 2
2 1 2


b1 ×b2 = (–3) 2 + (3) 2 = 3 2

402 Xam idea Mathematics–XII

@Cbsebookshub - Join Us on Telegram


(a2 – a1) . (b1 ×b2) (it – 3jt – 2kt) . (– 3it + 0jt + 3kt)
∴ Shortest distance =
=
b1 ×b2 b1 ×b2

–3 – 0 – 6 9 2 9 2 3 2
= = × = =
3 2 3 2 2 3×2 2
7. Find the vector equation of a plane which is at a distance of 7 units from the origin and normal
to the plane is 3it + 5jt – 6kt.
Sol. Normal vector of the plane is
n = 3it + 5jt – 6kt
2 2 2

n = (3) + (5) + (– 6) = 9 + 25 + 36 = 70

n 1 3 t 5 t 6 t
nt = = (3it + 5jt – 6kt) = i+ j– k.
n 70 70 70 70

The required equation of plane is r .nt = 7.


3 t 5 t 6 t
∴ r . e i+ j– ko = 7
70 70 70
8.
Find the vector equation of the plane passing through the intersection of the planes
r . (2it + 2jt – 3kt = 7), r . (2it + 5jt + 3kt) = 9 and through the point (2, 1, 3).
Sol. Let the equation of plane passing through the intersection of two planes be.
r . 7(2it + 2jt – 3kt) + m (2it + 5jt + 3kt)A = 7 + 9 m
r . 7(2 + 2m) it + (2 + 5m) jt + (– 3 + 3m) ktA = 7 + 9 m ...(i)
_ xit + yjt + zkti . 7^2 + 2mh it + ^2 + 5mh jt + ^ – 3 + 3mh ktA = 7 + 9m
^2 + 2mh x + (2 + 5m) y + (– 3 + 3m) z = 7 + 9m
a It contains point (2, 1, 3).
` ^2 + 2mh # 2 + ^2 + 5mh # 1 + ^ – 3 + 3mh # 3 = 7 + 9m
4 + 4m + 2 + 5m – 9 + 9m = 7 + 9m & 18m – 3 = 7 + 9m & 18m – 9m = 7 + 3
10
9m = 10 & m =
9
Put in equation (i), we get
38 68 t 3 t
r . d it + j + k n = 17
9 9 9
r . _ 38i + 68j + 3k i = 153 is the required vector equation in plane.
t t t

9. Find the equation of the plane through the line of intersection of the planes
x + y + z = 1 and 2x + 3y + 4z = 5 which is perpendicular to the plane x – y + z = 0.

Sol. Let the equation of the plane passing through the intersection of two planes be
(equation of (i) plane) + λ (equation of (ii) plane) = d1+λ d2
(x + y + z) + λ (2x + 3y + 4z) = 1 + 5λ ... (i)
x(1 + 2λ) + y (1 + 3λ) + z (1 + 4λ) = 1 + 5λ
a1= (1 + 2λ), b1 = (1 + 3λ), c1= (1 + 4λ)
This plane is perpendicular to the plane x – y + z = 0.
a2 = 1, b2 = – 1, c2 = 1
As plane is perpendicular to another plane then, a1a2 + b1b2 + c1c2 = 0
(1 + 2λ) × 1 + (1 + 3λ) × ( – 1) + (1 + 4 λ) × 1 = 0

Three Dimensional Geometry 403


@Cbsebookshub - Join Us on Telegram
1 + 2λ – 1 – 3λ + 1 + 4λ = 0 ⇒ – λ + 4 λ = – 1 ⇒ 3λ = – 1
1
λ = –
3
Put value of λ in equation (i), we get
–1
(2 x + 3y + 4z) = 1+5 d n
1
(x + y + z) –
3 3
3x + 3y + 3z – (2x + 3y + 4z) 3 – 5
=
3 3
x – z = – 2 ⇒ x – z + 2 = 0 is the required equation of the plane.
10. Find the coordinates of the point where the line through (3, – 4, – 5) and (2, – 3, 1) crosses the
plane 2x + y + z = 7.
Sol. The equation of given plane is 2x + y + z = 7 ...(i)
Equation of the line passing through points (3, – 4, – 5) and (2, –3, 1) is
x–3 y+4 z+5 x– 3 y+4 z+5
= = ⇒ = = = m (say)
2– 3 – 3+4 1+5 –1 1 6
x–3

–1
=m & x – 3=– m & x = 3– m
y+4
=m & y+4 = m & y=– 4+m
1
z+5
6
=m & z + 5 = 6m & z = – 5 + 6m
Putting value of x, y and z in (i), we have
2(3 –λ) + (– 4 + λ) + (– 5 + 6λ) = 7

6–2λ–4+λ–5+6λ=7

5 λ = 7 + 3 ⇒ λ=2
∴ x = 3 – 2 = 1, y = – 4 + 2 = – 2 and z = – 5 + 6 × 2 = – 5 + 12 = 7

Thus, coordinates of required point are (1, – 2, 7).
11. Find the equation of the plane passing through the line of intersection of the planes
r . (it + jt + kt) = 1 and r . (2it + 3jt – kt) + 4 = 0 and parallel to x-axis.
Sol. Here the equations of given planes are
r . (it + jt + kt) = 1 and r . (2it + 3jt – kt) + 4 = 0
The equation of a plane passing through the intersection of the given planes is
8r . (it + jt + kt) – 1B + m 8r . (2it + 3jt – kt) + 4B = 0

r . 7(2m + 1) it + (3m + 1) jt + (1 – m) ktA + 4m – 1 = 0 ...(i)
Since the above plane is parallel to x-axis i.e., it + 0jt + 0kt.
∴ 7^2m + 1h it + (3m + 1) jt + (1 – m) ktA . (it + 0jt + 0kt) = 0
1
⇒  2λ + 1 = 0  ⇒  λ = –
2
Putting value of λ in (i), we have

r . =d 2 # d – n + 1 n it + d 3 # d – n + 1 n jt + d1 + n ktG + 4 # d – n –1 = 0
1 1 1 1

2 2 2 2
1 3
⇒ r . d – jt + kt n –3 = 0 & r . (–jt + 3kt) – 6 = 0
2 2
Putting r = xi + yjt + zkt, we have
t

(xit + yjt + zkt) (–jt + 3kt) – 6 = 0 & – y + 3z – 6 = 0 & y – 3z + 6 = 0


which is required equation of the plane.

404 Xam idea Mathematics–XII

@Cbsebookshub - Join Us on Telegram


12. Find the equation of the plane which contains the line of intersection of the planes
r . (it + 2jt + 3kt) – 4 = 0 and r . (2it + jt – kt) + 5 = 0 and which is perpendicular to the plane
r . (5it + 3jt – 6kt) + 8 = 0 .[CBSE Delhi 2011, 2013]

Sol. The given planes are

r . (it + 2jt + 3kt) – 4 = 0 ...(i)


and r . (2it + jt – kt) + 5 = 0 ...(ii)


Therefore, a plane which contains the line of intersection of the planes (i) and (ii) is


r . (it + 2jt + 3kt) – 4 + m { r . (2it + jt – kt) + 5} = 0


r . [(1 + 2m) it + (2 + m) jt + (3 – m) kt] – 4 + 5m = 0 ...(iii)
Now, the plane (iii) is perpendicular to the plane

r . (5it + 3jt – 6kt) + 8 = 0 ...(iv)


Therefore from (iii) and (iv), we get
(1 + 2l). 5 + (2 + l). 3 + (3 – l). (– 6) = 0 [ a n1 . n2 = 0 ]


5 + 10l + 6 + 3l – 18 + 6l = 0
7
⇒ 19l – 7 = 0 ⇒ λ=
19
Now, putting the value of λ in (iii), we get

r . <c1 + m + c2 + m + c3 – m F
14 t 7 t 7 t 7
+ =
19 i 19 j 19 k – 4 5× 19 0

r . ; it + kE +
33 45 t 50 t 35 – 76
J+ =0
19 19 19 19

r . (33it + 45jt + 50kt) – 41 = 0 , which is the required equation.

13. Find the distance of the point (–1, –5, –10), from the point of intersection of the line
r = (2it – jt + 2kt) + m (3it + 4jt + 2kt) and the plane r . (it – jt + kt) = 5 .
[CBSE Delhi 2014; (AI) 2011; (F) 2014]
Sol. Given line and plane are
r = (2it – jt + 2kt) + m (3it + 4jt + 2kt) …(i)

r . (it – jt + kt) = 5 …(ii)

For intersection point, we solve equations (i) and (ii) by putting the value of r from (i) in (ii).
[(2it – jt + 2kt) + m (3it + 4jt + 2kt)] . (it – jt + kt) = 5
P(–1,–5,–10)

(2 + 1 + 2)+l (3 – 4 + 2) = 5 ⇒ 5 + l = 5 ⇒ l = 0
Hence, position vector of intersecting point is 2it – jt + 2kt
d

i.e., coordinates of intersection of line and plane is (2, –1, 2).


(2,–1,2)

Hence, required distance


= (2 + 1) 2 + (– 1 + 5) 2 + (2 + 10) 2 = 9 + 16 + 144
= 169 = 13 units

Three Dimensional Geometry 405


@Cbsebookshub - Join Us on Telegram
14. Find the vector equation of the line passing through (1, 2, – 4) and perpendicular to the two lines:
x – 8 y + 19 z – 10 x – 15 y – 29 z – 5
= = and = =
3 – 16 7 3 8 –5
Sol. Equation of any line through the point (1, 2, – 4) is
x–1 y –2 z+4
= = ...(i)
a b c
where a, b and c are direction ratios of line (i).
Now the line (i) is perpendicular to the lines
x – 8 y + 19 z – 10 x – 15 y – 29 z – 5
= = and = =
3 –16 7 3 8 –5
having direction ratios 3, – 16, 7 and 3, 8, – 5 respectively.
∴ 3a – 16b + 7c = 0
…(ii)
3a + 8b – 5c = 0 …(iii)
Solving (ii) and (iii) by cross-multiplication method, we have
a b c a b c
= =
80 – 56 21 + 15 24 + 48
& 24 = 36 = 72
a b c

= =
2 3 6
a b c
Let = = = m ⇒ a = 2λ, b = 3 λ and c = 6λ
2 3 6
The equation of required line which passes through point (1, 2, –4) and parallel to vector
2it + 3jt + 6kt is r = (it + 2jt – 4kt) + m (2it + 3jt + 6kt) .
15. Prove that if a plane has the intercepts a, b, c and is at a distance of p units from the origin then
1 1 1 1
2
+ 2+ 2 = 2.
a b c p
x y z
Sol. Let the equation of plane be + + = 1 …(i)
a b c
∴ Length of perpendicular from origin to plane (i) is
0 0 0
+ + –1
a b c –1 1
= =
1 2
1 2
1 2 1 1 1 1 1 1
d n +d n +d n 2
+ 2+ 2 2
+ 2+ 2
a b c a b c a b c
It is given that
1 1 1 1 1 1 1 1 1
p=
1 1 1
& 2
+ 2
+ 2
=
p
& 2
+ 2
+ 2
= . (on squaring both sides)
+ 2+ 2 a b c a b c p2
2
a b c

Multiple Choice Questions [1 mark]


Choose and write the correct option in the following questions.
1. The co-ordinates of the foot of the perpendicular drawn from the point (2, –3, 4) on the y-axis
is [CBSE 2020, (65/2/1)]
(a) (2, 3, 4) (b) (– 2, – 3, – 4) (c) (0, –3, 0) (d) (2, 0, 4)
2. The two planes x – 2y + 4z = 10 and 18x + 17y + kz = 50 are perpendicular, if k is equal to
[CBSE 2020, (65/4/1)]
(a) –4 (b) 4 (c) 2 (d) – 2

406 Xam idea Mathematics–XII

@Cbsebookshub - Join Us on Telegram


3. Distance of the point (a, b, c) from y-axis is
(a) b (b) b (c) b + c (d) a2 + c2
4. If the direction cosines of a line are k, k, k then [NCERT Exemplar]
1 1
(a) k > 0 (b) 0 < k < 1 (c) k = 1 (d) k = or –
3 3

5. The distance of the plane r . c it + jt – kt m = 1 from the origin is


2 3 6

7 7 7
1
(a) 1 (b) 7 (c) (d) none of these
7
x–2 = y–3 = z–4
6. The sine of the angle between the straight line and the plane
3 4 5
2x – 2y + z = 5 is [NCERT Exemplar]
10 4 2 3 2
(a) (b) (c) (d)
6 5 5 2 5 10
7. The reflection of the point (a, b, c) in the xy-plane is [NCERT Exemplar]
(a) (a, b, 0) (b) (0, 0, c) (c) (–a, –b, c) (d) (a, b, –c)
8. P is a point on the line segment joining the points (3, 2, –1) and (6, 2, –2). If x co-ordinate of
P is 5, then its y co-ordinate is [NCERT Exemplar]
(a) 2 (b) 1 (c) –1 (d) –2
9. If a, b, c are the angles that a line makes with the positive direction of x, y, z axis, respectively,
then the direction cosines of the line are [NCERT Exemplar]
(a) sin a, sin b, sin g (b) cos a, cos b, cos g (c) tan a, tan b, tan g (d) cos2 a, cos2 b, cos2 g
10. The distance of a point P (a, b, c) from x-axis is
(a) a 2 + c 2 (b) a 2 + b 2 (c) b 2 + c 2 (d) b 2 + c 2
11. The equations of x-axis in space are [NCERT Exemplar]
(a) x = 0, y = 0 (b) x = 0, z = 0 (c) x = 0 (d) y = 0, z = 0
12. A line makes equal angles with co-ordinate axis. Direction cosines of this line are
[NCERT Exemplar]

(b) ! d n (c) ! c , , m (d) ! d n


1 1 1 1 1 1 1 –1 –1
(a) ± (1, 1, 1) , , , ,
3 3 3 3 3 3 3 3 3
13. P is the point on the line segment joining the points (3, 2, –1) and (6, 2, –2). If x co-ordinate of
P is 5, then its y co-ordinate is
(a) 2 (b) 1 (c) –1 (d) –2
x–2 = y–3 = z–4

14. The sine of the angle between the straight line and the plane
3 4 5
2x – 2y + z = 5 is
10 4 2 3 2
(a) (b) (c) (d)
6 5 5 2 5 10
15. The area of the quadrilateral ABCD where A (0, 4, 1), B (2, 3, –1), C (4, 5, 0) and D (2, 6, 2) is equal
to
(a) 9 sq units (b) 18 sq units (c) 27 sq units (d) 81 sq units

16. The intercepts made by the plane 2x –3y + 5z + 4 = 0 on the coordinate axes are
4 4 4 4 4 4 7 4 4
(a) – 2, and – (b) – 2, – and (c) , – and (d) – 2, – and –
3 5 3 5 3 3 3 3 5

Three Dimensional Geometry 407


@Cbsebookshub - Join Us on Telegram
17. The shortest distance between the lines given by
( r = 8 + 3m) it – (9 + 16m) jt + (10 + 7m) kt and r = 15it + 29jt + 5kt + n (3it + 8jt – 5kt) is
(a) 7 units (b) 2 units (c) 14 units (d) 3 units
x = y–1 = z–2
18. The image of the point (1, 6, 3) in the line is
1 2 2
(a) (2, 0, 5) (b) (1, 3, 4) (c) (1, 0, 7) (d) (– 3, – 2, 0 )
19. The coordinates of the point where the line through (3, – 4, – 5) and (2, –3, 1) crosses the plane
passing through three points (2, 2, 1), (3, 0, 1) and (4, – 1, 0) are
(a) (0, – 2, 7) (b) (3, – 2, 5) (c) (1, – 2, – 7) (d) (1, – 2, 7)
20. The co-ordinates of the foot of perpendicular drawn from point A(1, 8, 4) to the line joining the
points B(0, –1, 3) and C(2, –3, –1) are

(a) c m (b) c m (c) c , , m (d) None of these


– 7 2 11 – 5 2 19 4 2 11
, , , ,
3 3 3 3 3 3 3 3 3
Answers
1. (c) 2. (b) 3. (d) 4. (d) 5. (a) 6. (d)
7. (d) 8. (a) 9. (b) 10. (c) 11. (d) 12. (b)
13. (a) 14. (d) 15. (a) 16. (a) 17. (c) 18. (c)
19. (d) 20. (b)

Solutions of Selected Multiple Choice Questions


1. The x and z co-ordinates on y-axis are 0.
∴ Required point is (0, –3, 0) on y-axis.

2. We have angle between two given plants is given by
r 1 × 18 + (–2) × 17 + 4 × k
cos =
2 (1) + (–2) 2 + (4) 2 . (18) 2 + (17) 2 (k) 2
2

⇒ 0 = 18 – 34 + 4k ⇒
4k = 16
⇒ k=4
4. Since, direction cosines of a line are k, k and k.
∴ l = k, m = k and n = k

1
We know that, l2 + m2 + n2 = 1  ⇒  k2 + k2 + k2 = 1  ⇒  k2 =
3
1

k = !
3
6. We have, the equation of line as
x–2 = y–3 = z–4

3 4 5
Now, the line passes through point (2, 3, 4) and having direction ratios (3, 4, 5).
Since, the line passes through point (2, 3, 4) and parallel to the vector (3it + 4jt + 5kt) .

b = 3it + 4jt + 5kt
Also, the cartesian form of the given plane is 2x – 2y + z = 5.
⇒ (xit + yjt + zkt) (2it – 2jt + kt) = 5

n = (2it – 2jt + kt)

408 Xam idea Mathematics–XII

@Cbsebookshub - Join Us on Telegram


b .n (3it + 4jt + 5kt) . (2it – 2jt + kt)
=
We know that, sin θ =
b .n 32 + 42 + 52 . 4 + 4 + 1
6–8+5
= = 3 = 1
50 . 3 15 2 5 2
2
sin θ =
10
7. In xy-plane, the reflection of the point (a, b, c) is (a, b, –c) .
8. Let P divides the line segment in the ratio of l : 1, x-coordinate of the point P may be expressed
6m + 3 6m + 3 = 2m + 2 =
as x = giving 5 so that l = 2. Thus y-coordinate of P is 2.
+
m 1 +
m 1 m+1
10. The required distance is the distance of P (a, b, c) from Q (a, 0, 0), which is b2 + c2 .
11. On x-axis the y-co-ordinate and z-co-ordinate are zero.
12. Let the line makes angle a with each of the axis.
Then, its direction cosines are cos a, cos a, cos a.
1
Since cos2 a + cos2 a + cos2 a = 1. Therefore, cos a = !
3

Fill in the Blanks [1 mark]


1. The distance between parallel planes 2x + y – 2z – 6 = 0 and 4x + 2y – 4z = 0 is _____________
units. [CBSE 2020 (65/2/1)]
2. If p(1, 0, –3) is the foot of the perpendicular from the origin to the plane, then the Cartesian
equation of the plane is _____________ . [CBSE 2020 (65/2/1)]
x–5 y+4 z–6
3. Vector equation of the line = = is ___________ .
3 7 2
r
4. If a line makes an angle of with each of y and z–axis, then the angle which it makes with
4
x – axis is _____________ . [CBSE 2020 (65/3/1)]

5. The cartesian equation of the plane r . _ it + jt – kt i = 2 is _____________ .

Answers
1. 2 2. x – 3z – 10 = 0 3. r = 5it – 4jt + 6kt + m (3it + 7jt + 2kt)
r
4. 5. x + y – z = 2
2
Solutions of Selected Fill in the Blanks
1. Given parallel planes are
2x + y – 2z – 6 = 0 ...(i)
and 4x + 2y – 4z = 0
⇒ 2x +y – 2z = 0 ...(ii)

Required distance between planes (i) and (ii) is given by
–6–0 6
D= = = 2 units
(2) 2 + (1) 2 + (–2) 2 3
2. Direction ratios of the normal to the plane are given by 1– 0, 0 – 0, –3 – 0

⇒ 1, 0, –3

∴ Equation of the plane be

Three Dimensional Geometry 409


@Cbsebookshub - Join Us on Telegram
a (x – 1) + b (y – 0) + c ^ z – (–3) h = 0
⇒ 1. (x – 1) + 0. (y – 0) + (–3) (z + 3) = 0
⇒ x –1 + 0 – 3z – 9 = 0
⇒ x –3z – 10 = 0

4. Let the line makes angle \ with x–axis.


r r
∴ cos 2 \ + cos 2 + cos 2 = 1 ( a l 2 + m 2 + n 2 = 1)
4 4
2 2
1 1
cos a + e o +e o = 1
2 1 1
⇒ ⇒ cos 2 a + + =1
2 2 2 2
⇒ cos 2 a = 0 ⇒ cos a = 0
r
⇒ a =
2
5. We have, r . (it + jt – kt) = 2
⇒ (xit + yjt + zkt) . (it + jt – kt) = 2
⇒ x + y – z = 2

Very Short Answer Questions [1 mark]


1. If a line has direction ratios 2, –1, –2, then what are its direction cosines? [CBSE Delhi 2012]
Sol. Here direction ratios of line are 2, –1, –2

2 –1 –2
∴ Direction cosines of line are
, ,
2 2 + (– 1) 2 + (– 2) 2 2 2 + (– 1) 2 + (– 2) 2 2 2 + (– 1) 2 + (– 2) 2

2 –1 –2
i.e.,
, ,
3 3 3

Note: If a, b, c are the direction ratios of a line, the direction cosines are
a b c
, ,
a2 + b2 + c2 a2 + b2 + c2 a2 + b2 + c2
x+2 y –5 z+1
2. Find the co-ordinate of the point, where the line = = cuts the yz-plane.
1 3 5
[CBSE 2019 (65/5/3)]
Sol. Let the required point be (a, b, c) where given line cuts yz-plane.
a+2 = b– 5 = c+1
∴ = k(say)
1 3 5
a+2 =
If k & a = – 2 + k, b = 5 + 3k, c = – 1 + 5k
1
Since this point lies in yz-plane.
∴ a = 0
& –2 + k = 0 & k=2
So, a = 0, b = 11, c = 9
∴ Required point is (0, 11, 9) where given line cuts yz-plane.

3. Write the direction cosine of a line equally inclined to the three coordinate axes.
[CBSE (AI) 2009, (F) 2011]
Sol. Any line equally inclined to coordinate axes will have direction cosines l, l, l

l2 + l2 + l2 = 1

410 Xam idea Mathematics–XII

@Cbsebookshub - Join Us on Telegram


1
3l 2 = 1 & l =!
3
1 1 1 1 1 1
∴ Direction cosines are +
,+ ,+ or – ,– ,–
3 3 3 3 3 3
4. Write the distance of the following plane from the origin. [CBSE (AI) 2010]
2x – y + 2z + 1 = 0
Sol. We have given plane
2x – y + 2z + 1 = 0
(2×0) – (1×0) + (2×0) + 1 1 1
Distance from origin = = =
(2) 2 + (– 1) 2 + (2) 2 4+1+4 3

5. Find the acute angle between the planes [CBSE 2019 (65/4/1)]
r . (it – 2jt – 2kt) = 1 and r . (3it – 6jt + 2kt) = 0.
Sol. We have, n1 = it – 2jt – 2kt and n2 = 3it – 6jt + 2kt
Let θ be the angle between the normals to the planes drawns from some common point.

n1 . n2 3 + 12 – 4 11
We have, cos i = = = = 11
n1 n2 9 49 3 #7 21

∴    i = cos –1 c m .
11

21
6. Write the direction cosines of a line parallel to z-axis. [CBSE (F) 2012]
Sol. The angle made by a line parallel to z-axis with x, y and z-axis are 90°, 90° and 0° respectively.
∴ The direction cosines of the line are cos 90º, cos 90º, cos 0º i.e., 0, 0, 1.

7. Write the cartesian equation of a plane, bisecting the line segment joining the points A(2, 3, 5)
and B(4, 5, 7) at right angles. [CBSE (F) 2013]
Sol. One point of required plane = mid point of given line segment.
2+4 3+5 5+7
= d , , n = (3, 4, 6)
2 2 2
Also dr's of normal to the plane = (4 – 2), (5 – 3), (7 – 5) = (2, 2, 2)
Therefore, required equation of plane is
2(x – 3) + 2(y – 4) + 2(z – 6) = 0
2x + 2y + 2z = 26 or x + y + z = 13

8. Write the vector equation of the plane, passing through the point (a, b, c) and parallel to the
plane r . (it + jt + kt) = 2. [CBSE Delhi 2014]
Sol. Since, the required plane is parallel to plane r . (it + jt + kt) = 2.

Normal of required plane is normal of given plane.

Normal of required plane = it + jt + kt

Required vector equation of plane

{ r – (ait + bjt + ckt)} . (it + jt + kt) = 0

Three Dimensional Geometry 411


@Cbsebookshub - Join Us on Telegram
9. Find the sum of the intercepts cut off by the plane
2x + y – z = 5, on the coordinate axes. [CBSE (F) 2015]
Sol. Let a, b, c be the intercepts cut off by the plane
2x + y – z = 5 … (i) on x, y and z-axis respectively.
⇒ A(a, 0, 0), B(0, b, 0) and C(0, 0, c) satisfy the
equation (i) A (a, 0, 0)
5
Hence, 2a + 0 – 0 = 5 ⇒ a=
2
and 2 × 0 + b – 0 = 5 ⇒ b=5

and 2 × 0 + 0 – c = 5 ⇒ c=–5
5 5

` a+b+c= +5– 5 =
2 2
10. Write the coordinates of the point which is the reflection of the point (a , b , c ) in the XZ-
plane. [CBSE East 2016]

Sol. The reflection of the point (a, b, c) in the XZ plane is (a, – b, c) .

(α, β, γ)

(α, 0, γ)

(α, –β, γ)

11. Find the distance between the planes r . (2it – 3jt + 6kt) – 4 = 0 and r . (6it – 9jt + 18kt ) + 30 = 0
[CBSE South 2016]
Sol. Given two planes are
r . (2it – 3jt + 6kt) – 4 = 0 and
r . (6it – 9jt + 18kt) + 30 = 0
Given planes may be written in cartesian form as
2x – 3y + 6z – 4 = 0 … (i)
6x – 9y + 18z + 30 = 0 …(ii)
Let P (x1, y1, z1) be a point on plane (i)

2x1 – 3y1 + 6z1 – 4 = 0

2x1 – 3y1 + 6z1 = 4 …(iii)
The length of the perpendicular from P (x1, y1, z1) to plane (ii)

412 Xam idea Mathematics–XII

@Cbsebookshub - Join Us on Telegram


6x1 – 9y1 + 18z1 + 30 3 (2x1 – 3y1 + 6z1) + 30
= =
2 2 2
6 + (– 9) + 18 36 + 81 + 324
3 × 4 + 30 42
= = = 2 [Using (iii)]
441 21

12. Write the equation of a plane which is at a distance of 5 3 units from origin and the normal
to which is equally inclined to coordinate axes. [CBSE (F) 2016]
Sol. Obviously, a vector equally inclined to co-ordinate axes is given by it + jt + kt
it + jt + kt 1 t t t
∴ Unit vector equally inclined to co-ordinate axes =
= (i + j + k )
2 2 2 3
1 +1 +1
Therefore, required equation of plane is
1 t t t
r . ) (i + j + k ) 3 = 5 3 ⇒ r . (it + jt + kt) = 15 or x + y + z = 15
3
13. If a line makes angles 90° and 60° respectively with the positive directions of x and y axes, find
the angle which it makes with the positive direction of z-axis. [CBSE Delhi 2017]
Sol. Let the angle made by line with positive direction of z-axis be i then,
We know that
cos 2 90° + cos 2 60° + cos 2 i = 1
1 2
0 + c m + cos 2 i = 1
1

⇒ + cos 2 i = 1
2 4
1 3

cos 2 i = 1 – ⇒ cos 2 i =
4 4
3

cos i = !
2
r 3 5r 3

i = 60° or if cos i = and i = 150° or if cos i = –
3 2 6 2
14. Find the distance between the planes 2x – y + 2z = 5 and 5x – 2.5y + 5z = 20. [CBSE (AI) 2017]
Sol. Let P (x1, y1, z1) be any point on plane 2x – y + 2z = 5.

2x1 – y1 + 2z1 = 5
Now distance of point P (x1, y1, z1) from plane 5x – 2.5y + 5z = 20 is given by
5x1 – 2.5y1 + 5z1 – 20 2.5 (2x1 –y1 + 2z1 –8) 2.5 (5 – 8)
d = = =
5 2 + (2.5) 2 + (5) 2 25 + 6.25 + 25 56.25
7.5
= = 1 unit
7.5
15. Find the equation of a plane that cuts the coordinates axes at (a, 0, 0), (0, b, 0) and (0, 0, c).
[NCERT Exemplar]
x y z
Sol. The equation of such plane is + + = 1
a b c
16. Find the angle between the line r = (2it – jt + 3kt) + m (3it – jt + 2kt) and the plane r . (it + jt + kt) = 3.
[CBSE 2019 (65/5/3)]
Sol. We have equation of line
r = (2it – jt + 3kt) + m (3it – jt + 2kt)
∴ b = 3it – jt + 2kt

Three Dimensional Geometry 413


@Cbsebookshub - Join Us on Telegram
Equation of plane r . (it + jt + kt) = 3
n = it + jt + kt
Let θ be the required angle
b .n 3–1+2 4 4
∴ cos i =
= = =
b n 9+1+4 1+1+1 14 3 42

⇒ i = cos –1 d n.
4

42

Short Answer Questions-I [2 marks]


1. Find the points of intersection of the line rt = 2it – jt + 2kt + m (3it + 4jt + 2kt) and the plane
rt. (it – jt + kt ) = 5 [CBSE 2020, (65/2/1)]
Sol. Given line be
rt = 2it – jt + 2kt + m (3it + 4jt + 2kt)
Its Cartesian form is
x –2 y+1 z– 2
= = = m (let)
3 4 2
∴ points on this line be (3m + 2, 4m – 1, 2m + 2)
i.e., x = 3m + 2, y = 4m – 1, z = 2m + 2
...(i)

and plane be r . (it –jt + kt) = 5


⇒ x – y + z = 5
⇒ (3m + 2) – (4m – 1) + 2m + 2 = 5 (from (i)
⇒ m + 5 = 5 ⇒ m = 0
Putting m = 0 in (i), we get the co-ordinates of the point, x = 2, y = – 1, z = 2
∴ Point of intersection be (2, –1, 2).

2. If the x-coordinate of a point P on the join of Q(2, 2, 1) and R(5, 1, –2) is 4, then find its
z-coordinate. [NCERT Exemplar]
Sol. Let P divides QR in the ratio l: 1
5m + 2 m + 2 –2m + 1
Then coordinates of P are d , , n
m+1 m+1 m+1
It is given that x-coordinate of P is 4.
5m + 2
∴ = 4 & 5m + 2 = 4m + 4 & m = 2
m+1
–2m + 1 – 4 + 1
So,z -coordinate of P = = = – 1.
m+1 2+1
3. Show that the points (it – jt + 3kt) and 3 (it + jt + kt) are equidistant from the plane
r . (5it + 2jt – 7kt) + 9 = 0 and lies on opposite side of it. [NCERT Exemplar]
t t t t t t
Sol. To show that these given points (i – j + 3k) and 3 (i + j + k) are equidistant from the plane
r . (5it + 2jt – 7kt) + 9 = 0, we first find out the mid-point of the points which is 2it + jt + 3kt.
On substituting r by the mid-point in plane, we get
LHS = (2it + jt + 3kt) $ (5it + 2jt – 7kt) + 9 = 10 + 2 – 21 + 9 = 0 = RHS
Hence, the two points lie on opposite sides of the plane are equidistant from the plane.

414 Xam idea Mathematics–XII

@Cbsebookshub - Join Us on Telegram


4. If the plane ax + by = 0 is rotated about its line of intersection with the plane z = 0 through an
angle a, then prove that the equation of the plane in its new position is ax + by ! _ a �+ b �tan ai
z = 0. [NCERT Exemplar]
Sol. Given, planes are ax + by
= 0 ...(i)
and z
= 0 ...(ii)
Therefore, the equation of any plane passing through the line of intersection of planes
(i) and (ii) may be taken as ax + by + k = 0. ...(iii)
Then, direction cosines of a normal to the plane (iii) are
a b c
2 2 2
, , and direction cosines of the normal to the plane (i) are
+
a b k + a b2 + k2
2
+ a b2 + k2
+ 2

a b
2 2
, , 0.
a +b a + b2
2

Since, the angle between the planes (i) and (ii) is α,


a$a+b$b+k$0 a2 + b2
∴ cos a = =
a + b2 + k2 a2 + b2
2
a2 + b2 + k2


k2 cos2 a = a2 (1 – cos2 a) + b2 (1 – cos2 a)

(a 2 + b 2) sin 2 a

k2 =
cos 2 a

k = ! a 2 + b 2 tan a

On putting this value in plane (iii), we get the equation of the plane as
ax + by + z a 2 + b 2 tan a = 0
5. Find the co-ordinates of the point where the line through ^ –1, 1, – 8 h and ^5, –2, 10 h crosses the
ZX - plane. [CBSE 2020, (65/3/1)]
Sol. We have,
Equation of the line passing through (– 1, 1, – 8) and (5, – 2, 10) be
x – (– 1) y–1 z – ( – 8)
= =
5+1 –2 – 1 10 + 8
x+1 y –1 z+8

= = . ..(i)
6 –3 18
Now, for the co-ordinates of the point where the line (i) crosses the ZX–plane, put y = 0 in (i),
we get
x+1 0 –1 z+8
= =
6 –3 18
x+1 1 1 z+8

= and =
6 3 3 18


x = 1 and z = – 2
∴ Co-ordinates of the required point be (1, 0, – 2)

Three Dimensional Geometry 415


@Cbsebookshub - Join Us on Telegram
Short Answer Questions-II [3 marks]
1. Find the shortest distance between the lines whose vector equations are:
r = (it + jt) + m (2 it –jt + kt) and r = (2 it + jt –kt) + n (3 it –5 jt + 2 kt) . . [CBSE (F) 2014]

Sol. Comparing the given equations with equations r = a1 + m b1 and r = a2 + n b2 .

We get a1 = it + jt, b1 = 2it – jt + kt and a2 = 2it + jt – kt, b2 = 3it – 5jt + 2kt

Therefore, a2 – a1 = (it – kt) and


it jt kt
b1 ×b2 = (2 it – jt + kt) × (3it – 5jt + 2kt) = 2 – 1 1 = 3it – jt – 7kt
3 –5 2

b1 ×b2 = 9 + 1 + 49 = 59
Hence, the shortest distance between the given lines is given by

(b1 ×b2) . (a2 – a1) 3–0+7 10


d = = = units.
b1 ×b2 59 59
2. Find the distance between the lines l1 and l2 given by
t
l1: r = it + 2jt – 4kt + m (2it + 3jt + 6kt ); l2: r = 3it + 3jt – 5kt + n (4it + 6jt + 12kt ) [CBSE (F) 2014]

Sol. Given lines are

l1: r = it + 2jt – 4kt + m (2it + 3jt + 6kt)

l2: r = 3it + 3jt – 5kt + n (4it + 6jt + 12kt)


After observation, we get l1 ||l2
Therefore, it is sufficient to find the perpendicular distance of a point of line l1 to line l2.
The coordinate of a point of l1 is P(1, 2, –4)
P(1,2,-4)
Also the cartesian form of line l2 is l1
x – 3 y – 3 z+5
= = ... (i)
4 6 12
Let Q(α, β, γ) be foot of perpendicular drawn from P to line l2

 Q(a, b, g) lie on line l2
a– 3 b– 3 c+5

= = = m (say)
4 6 12
⇒ a = 4l + 3, b = 6l + 3, g = 12l – 5
l2
Q(,,)
Again, a PQ is perpendicular to line l2.


PQ . b = 0, where b is parallel vector of l2

⇒ (a –1).4 + (b –2).6 + (g + 4). 12 = 0 [ a PQ = (a – 1) it + (b – 2) jt + (c + 4) kt ]


⇒ 4a – 4 + 6b – 12 + 12g + 48 = 0
⇒ 4a + 6b + 12g + 32 = 0

4 (4l + 3) + 6 (6l + 3) + 12(12l – 5) + 32 = 0  ⇒  16l + 12 + 36l + 18 + 144l – 60 + 32 = 0
–2 –1
⇒ 196l + 2 = 0
⇒ m= =
196 98

416 Xam idea Mathematics–XII

@Cbsebookshub - Join Us on Telegram


Coordinate of Q / c 4× c – m + 3, 6× c – m + 3, 12× c – m m
1 1 1
98 98 98 – 5

/ c – 49 + 3, – 49 + 3, – 49 – 5 m / c 49 , 49 , – 49 m
2 3 6 145 144 251

Therefore required perpendicular distance is


2 2 2
96 2 46 2 – 55 2
c – 1m + c – 2m + c + 4m = c m +c m +c m
145 144 – 251

49 49 49 49 49 49

96 2 + 46 2 + 55 2 9216 + 2116 + 3025


= =
49 2 49 2
14357 7 293 293
= = = units
49 49 7
3. Find the coordinates of the point where the line through the points (3, –4, –5) and (2, –3, 1)
crosses the plane 2x + y + z = 7. [CBSE (AI) 2012]
Sol. The equation of line passing through the points (3, – 4, – 5) and (2, – 3, 1) is
x–3 y+4 z+5 y+4 +
=
2– 3 – 3+4 1+5
= & x––13 = 1 = z 6 5 ...(i)
A (3, –4, –5)
Let the line (i) crosses at point P (a, b, g) to the plane 2x + y + z = 7 ...(ii)
 P lies on line (i), therefore (a, b, g) satisfy equation (i)
a– 3 b+4 c+5
∴ = = = m (say) P (,,)
–1 1 6
a = –l + 3; b = l – 4 and g = 6l –5
Also P (a, b, g) lie on plane (ii) 2x + y + z = 7

∴ 2a + b + g = 7  ⇒  2 (–l + 3)+ (l – 4)+(6l – 5) = 7


B (2, –3,1)
⇒ –2 l + 6 + l – 4 + 6l – 5 = 7  ⇒  5l = 10
⇒ l=2
Hence, the coordinate of required point P is (–2 + 3, 2 – 4, 6 × 2 – 5) i.e., (1, –2, 7)

4. A line passes through (2, –1, 3) and is perpendicular to the lines r = it + jt – kt + m (2it – 2jt + kt) and
r = (2it – jt – 3kt) + n (it + 2jt + 2kt) . Obtain its equation in vector and cartesian form.
[CBSE (AI) 2014]
Sol. Let b be parallel vector of required line.


b is perpendicular to both given line.

b = (2it – 2jt + kt) × (it + 2jt + 2kt)




it jt kt
= 2 –2 1 = (– 4 – 2) it – (4 – 1) jt + (4 + 2) kt = – 6it – 3jt + 6kt .
1 2 2

Hence, the equation of line in vector form is

r = (2it – jt + 3kt) + m (– 6it – 3jt + 6kt) r = (2it – jt + 3kt) – 3m (2it + jt – 2kt)

r = (2it – jt + 3kt) + n (2it + jt – 2kt) [ n = –3m ]


Equation in cartesian form is
x–2 y+1 z– 3
= =
2 1 –2

Three Dimensional Geometry 417


@Cbsebookshub - Join Us on Telegram
5. Find the shortest distance between the following lines :
x–3 y–5 z–7 x+1 y+1 z+1
= = and = = [CBSE Delhi 2008, (F) 2013, 2014]
1 –2 1 7 –6 1
x–3 y–5 z–7 x+1 y+1 z+1
Sol. Let = = = m and = = =k
1 –2 1 7 –6 1
A line 1
Now, let’s take a point on first line as
A (l + 3, – 2l + 5, l + 7) and let
B (7k – 1, – 6k – 1, k – 1) be point on the second line
The direction ratio of the line AB
B line 2
7k –l –4, – 6k + 2l – 6, k – l – 8
Now, as AB is the shortest distance between line 1 and line 2 so,
(7k –l – 4) × 1 + ( –6k + 2l –6) × (–2) + (k –l – 8) × 1 = 0 ...(i)
and (7k – l – 4) × 7 + (–6k + 2l – 6)× (–6) + (k – l – 8 ) × 1 = 0 ...(ii)
Solving equation (i) and (ii), we get
l = 0 and k = 0

A ≡ (3, 5, 7) and B ≡ (–1, –1, –1)

Hence, AB = (3 + 1) 2 + (5 + 1) 2 + (7 + 1) 2 = 16 + 36 + 64 = 116 units = 2 29 units

6. Find the equation of planes passing through the intersection of the planes r . (2it + 6jt) + 12 = 0
and r . (3it – jt + 4kt) = 0 and are at a unit distance from the origin. [CBSE 2019 (65/5/3)]
Sol. We are given planes:

r . (2it + 6jt) + 12 = 0 ...(i)

r . (3it – jt + 4kt) = 0 ...(ii)


So equation of the required plane can be written as:
( r . (2it + 6jt) + 12) + m ( r . (3it – jt + 4kt)) = 0

⇒ r . {(2 + 3m) it + (6 – m) jt + 4mkt} + 12 = 0 ...(iii)



In cartesian form
(2 + 3m) x + (6 – m) y + 4mz + 12 = 0 ...(iv)
Since direction ratios of the normal to the plane are (2 + 3m), (6 – m), 4m; the direction cosines of it
are:
2 + 3m 6–m 4m
, ,
2 2 2 2 2 2
(2 + 3m) + (6 – m) + (4m) ( 2 + 3 m) + ( 6 – m) + ( 4 m ) (2 + 3m) + (6 – m) 2 + (4m) 2
2

12
So the distance of the plane from the origin is
(2 + 3m) + (6 – m) 2 + (4m) 2
2

We are given that distance from origin is unity


12
∴ =1
(2 + 3m) 2 + (6 – m) 2 + (4m) 2
144
⇒ = 1   ⇒  144 = 26m 2 + 40 (Squaring both sides)
4 + 9m + 12m + 36 – 12m + m 2 + 16m 2
2

418 Xam idea Mathematics–XII

@Cbsebookshub - Join Us on Telegram


104
⇒ 26m 2 = 104 ⇒
m2 = =4
26
⇒ m = ! 2

∴ Required equation of the plane is 8x + 4y + 8z + 12 = 0.

In vector form
r . (8it + 4jt + 8kt) + 12 = 0
7. Find the vector equation of the plane determined by the points A(3, –1, 2), B(5, 2, 4) and
C(–1, –1, 6). Hence, find the distance of the plane, thus obtained from the origin.
[CBSE 2019 (65/4/2)]
Sol. Required equation of plane is given by:
x – 3 y+1 z– 2
2 3 2 0 ⇒ (x – 3) 12 – (y + 1) 16 + (z – 2) 12 = 0
–4 0 4
⇒ 12x – 16y + 12z – 36 – 16 – 24 = 0  ⇒  12x – 16y + 12z – 76 = 0  ⇒  3x – 4y + 3z – 19 = 0

Vector form:
r . (3it – 4jt + 3kt) = 19. ...(i)
Distance of plane (i) from origin
3i – 4j + 3k 19
r. = [a r .nt = d]
9 + 16 + 9 34
19
Therefore distance of plane from origin is units.
34
8. Find the vector and cartesian equations of the line passing through the point (2, 1, 3) and
x–1 y–2 z–3 x y z
perpendicular to the lines = = and = = . [CBSE (AI) 2014]
1 2 3 –3 2 5
Sol. Let the cartesian equation of the line passing through (2, 1, 3) be
x–2 y–1 z–3
= = …(i)
a b c
Since, line (i) is perpendicular to given line
x–1 y–2 z–3
= = …(ii)
1 2 3
x y z
and = = …(iii)
–3 2 5

a + 2b + 3c = 0 …(iv)
–3a + 2b + 5c = 0 …(v)
From equation (iv) and (v), we get
a b c a b c
= =
10 – 6 – 9 – 5 2 + 6
& = = = m
4 – 14 8
(say)


a = 4l, b = –14l, c = 8l
Putting the value of a, b and c in (i), we get
x–2 y–1 z–3 y–1

4m
=
– 14m
=
8m
& x –4 2 = –14 = z –8 3
x–2 y–1 z–3
⇒ = = , which is the cartesian form.
2 –7 4
The vector form is r = (2it + jt + 3kt) + m (2it – 7jt + 4kt) .

Three Dimensional Geometry 419


@Cbsebookshub - Join Us on Telegram
9. Find the distance of the point P (6, 5, 9) from the plane determined by the points A (3, –1, 2),
B (5, 2, 4) and C (–1, –1, 6). [CBSE (AI) 2010; (F) 2012; Delhi 2013; Ajmer 2015]
Sol. Plane determined by the points A (3, –1, 2), B (5, 2, 4) and C (–1, –1, 6) is
x – 3 y+1 z– 2 x – 3 y+1 x – 2
5– 3 2+1 4– 2 = 0 & 2 3 2 =0
–1– 3 –1+1 6– 2 –4 0 4
3 2 2 2 2 3
(x – 3) – (y + 1) + ( z – 2) =0
0 4 –4 4 –4 0
⇒ 12x – 36 – 16y – 16 + 12z – 24 = 0  ⇒  3x – 4y + 3z – 19 = 0

Distance of this plane from point P ( 6, 5, 9) is
(3×6) – (4×5) + (3×9) – 19 18 – 20 + 27 – 19 6
= = units.
(3) 2 + (4) 2 + (3) 2 9 + 16 + 9 34
+
10. Show that the lines x + 1 = y 3 = z + 5 and x – 2 = y – 4 = z – 6 intersect. Also find their
3 5 7 1 3 5
point of intersection. [CBSE Delhi 2014]
Sol. Given lines are
x+1 y+3 z+5
= = …(i)
3 5 7
x–2 y–4 z–6
= = …(ii)
1 3 5
Let two lines (i) and (ii) intersect at a point P(a, b, γ).
⇒ (a, b, g) satisfy line (i)

a+1 b+3 c+5

= = = m (say)
3 5 7
⇒ a = 3l – 1, b = 5l – 3,
g = 7l – 5 , …(iii)
Again (a, b, g) also lie on (ii), we get
a–2 b–4 c–6 3m – 1 – 2 5 m – 3 – 4 7 m – 5 – 6

1
=
3
=
5
& 1
=
3
=
5
3m – 3 5m – 7 7m – 11
⇒ = =
1 3 5
I II III
From I and II From II and III
3m – 3 5m – 7 5m – 7 7m – 11
= =
1 3 3 5
⇒ 9l – 9 = 5l – 7 ⇒ 25l – 35 = 21l – 33
⇒ 4l = 2
⇒ 4l = 2
1 1
⇒ m =
⇒ m=
2 2
Since, the value of l in both the cases is same

Both lines intersect each other at a point.

∴ Intersecting point = (a, b, g) = c – 1, – 3, – 5 m


3 5 7
[From (iii)]
2 2 2

   = c , – , m
1 1 –3
2 2 2

420 Xam idea Mathematics–XII

@Cbsebookshub - Join Us on Telegram


11. Find the vector and cartesian equations of the line passing through the point (1, 2, –4) and
y + 19 z – 10 y – 29 z – 5
perpendicular to the two lines x – 8 = = and x – 15 = = .
3 – 16 7 3 8 –5
[CBSE Delhi 2012, 2017]
OR
Find the equation of a line passing through the point (1, 2, –4) and perpendicular to two lines
r = (8it – 19jt + 10kt) + m (3it – 16jt + 7kt) and r = (15it + 29jt + 5kt) + n (3it + 8jt – 5kt) .
[CBSE Allahabad 2015; Delhi 2016]
Sol. Let the cartesian equation of line passing through (1, 2, – 4) be
y – 2 z+4
x – 1 = = ...(i)
a b c
Given lines are
x – 8 y + 19 z – 10
= = ...(ii)
3 – 16 7
x – 15 y – 29 z – 5
= = ...(iii)
3 8 –5

Obviously parallel vectors b1 , b2 and b3 of (i), (ii) and (iii) respectively are given as

b1 = ait + bjt + ckt; b2 = 3it – 16jt + 7kt; b3 = 3it + 8jt – 5kt


According to question
(i) ⊥ (ii) ⇒ b1 = b2 ⇒ b1 .b2 = 0

(i) ⊥ (iii) ⇒ b1 = b3 ⇒ b1 . b3 = 0
Hence, 3a – 16b + 7c = 0 ...(iv)
and 3a + 8b – 5c = 0 ...(v)
From equation (iv) and (v), we get
a b c
= =
80 – 56 21 + 15 24 + 48
a b c a b c

= =
24 36 72
& = = = m
2 3 6
(say)

⇒ a= 2l, b = 3l, c = 6l

Putting the value of a, b, c in (i), we get the required cartesian equation of line as
x –1 y – 2 z+4 x –1 y – 2 z+4

2m
=
3m
=
6m
& 2
=
3
=
6
Hence, vector equation is
r = (it + 2jt – 4kt) + m (2it + 3jt + 6kt)
12. A line passing through the point A with position vector a = 4it + 2jt + 2kt is parallel to the vector
b = 2it + 3jt + 6kt . Find the length of the perpendicular drawn on this line from a point P with
position vector r1 = it + 2jt + 3kt . [CBSE Panchkula 2015]

Sol. The equation of line passing through the point A and parallel to b is given in cartesian form as
x–4 y–2 z–2
= = …(i)
2 3 6
Let Q (a, b, c) be foot of perpendicular drawn from point P to the line (i).
Co-ordinate or point P / (1, 2, 3) [a P.V. of P is it + 2jt + 3kt]

Three Dimensional Geometry 421


@Cbsebookshub - Join Us on Telegram
Since, Q lie on line (i)
a–4 b–2 c–2
= = = m   ⇒   a = 2m + 4, b = 3m + 2, c = 6m + 2
2 3 6
Now, PQ = (a – 1) it + (b – 2) jt + (c – 3) kt P (1, 2, 3)

Obviously, PQ = b ` PQ . b = 0

2 (a – 1) + 3 (b – 2) + 6 (c – 3) = 0

2a – 2 + 3b – 6 + 6c – 18 = 0   ⇒  2a + 3b + 6c – 26 = 0
b = 2i+3j+6k
Putting the value of a, b, c; we get
2 (2m + 4) + 3 (3m + 2) + 6 (6m + 2) – 26 = 0 Q (α, β, γ)
x–4 y–2 z–2

4m + 8 + 9m + 6 + 36m + 12 – 26 = 0 = =
2 3 6

49m = 0 & m=0
Hence, the co-ordinate of Q / (4, 2, 2)
∴ Length of perpendicular PQ = (4 – 1) 2 + (2 – 2) 2 + (2 – 3) 2

= 9 + 0 + 1 = 10 units.
y+1 z–2
13. Find the coordinates of the point, where the line x – 2 = = intersects the plane
3 4 2
x – y + z – 5 = 0. Also find the angle between the line and the plane. [CBSE Delhi 2013]
Sol. Let the given line
x –2 y+1 z– 2
= = …(i)
3 4 2
intersect at point P (a, b, g) to the plane x – y + z – 5 = 0 ...(ii)
 P (a, b, g) lie on line (i)

a– 2 b+1 c– 2

= = = m (say)
3 4 2
a = 3l + 2; b = 4l – 1; g = 2l + 2
Also, P (a, b, g) lies on plane (ii)
P (,,)
∴ (3l + 2) – (4l – 1)+ (2l + 2) – 5 = 0

⇒ 3l + 2 – 4l + 1 + 2l + 2 – 5 = 0
⇒ l=0
∴ a = 2, b = – 1, g = 2

Hence, co-ordinate of required point = (2, – 1, 2)
Now, find angle between line (i) and plane (ii)
If q be the required angle, then RS VW
SSa b = 3it + 4jt + 2kt WW
b .n 1 1 SS WW
sin i = = = t t
SS n = i – j + k t WW
b .n 9 + 16 + 4 . 1 2 + (–1) 2 + 1 2 29 . 3 SS W
` b . n = 3 – 4 + 2 = 1W
i = sin –1 d n
1 1 T X
sin i = & 87
87
14. Find the coordinates of the point where the line through the points A(3, 4, 1) and B(5, 1, 6)
crosses the XZ plane. Also find the angle which this line makes with the XZ plane.
[CBSE (Central) 2016]
Sol. Let P (a, b, g) be the point at which the given line crosses the XZ plane.
Now the equation of given line AB is

422 Xam idea Mathematics–XII

@Cbsebookshub - Join Us on Telegram


x–3 y–4 z–1
= = ...(i)
2 –3 5 A (3, 4, 1)

5k
j+
Since P (a, b, g) lies on line (i)

–3
2i
a–3 b–4 c–1
∴ = = = m (say)

b=
2 –3 5 n=j

⇒ a = 2l + 3; b = –3l + 4 and g = 5l + 1
P( , , )
y=0
Also P (a, b, g) lie on XZ plane, i.e., y = 0 (0x + 1y + 0z = 0)
0a + 1. b + 0. g = 0 XZ plane
4
⇒ b = 0 ⇒ –3l + 4 = 0
⇒ m=
3
Hence, the co-ordinates of required point P is B (5, 1, 6)
4 8 17 4 4 23
a = 2× + 3 = + 3 = ; b = –3× + 4 = 0 ; c = 5× + 1 =
3 3 3 3 3 3

∴ Co-ordinate of required point is c , 0, m .


17 23

3 3
Let q be the angle made by line AB with XZ plane.
n .b
∴ sin q =

b n
Here n = j and b = 2it – 3jt + 5kt
t

n = 1 and b = 4 + 9 + 25 = 38
jt. (2it – 3jt + 5kt) –3
⇒ sin q = =
1. 38 38

i = sin –1 d n
3 3

sin i = & 38
38
15. Find the vector equation of the line passing through the point A(1, 2, –1) and parallel to the line
5x – 25 = 14 – 7y = 35z. [CBSE Delhi 2017]
Sol. Given line is
5x – 25 = 14 – 7y = 35z
x–5 2–y z–0 x–5 y–2 z–0
⇒ = =   ⇒  = =
1 1 1 1 1 1

5 7 35 5 7 35
x–5 y–2 z–0
= = = … (i)
7 –5 1
Hence, parallel vector of given line i.e., b = 7it – 5jt + kt
Since required line is parallel to given line (i)
⇒ b = 7it – 5jt + kt will also be parallel vector of required line which passes through A(1, 2, –1).
Therefore, required vector equation of line is
r = (it + 2jt – kt) + m (7it – 5jt + kt)

16. Find the co-ordinates of the point where the line r = (– it – 2jt – 3kt) + m (3it + 4jt + 3kt) meets the
4
plane which is perpendicular to the vector n = it + jt + 3kt and at a distance of from origin.
11
[CBSE (South) 2016]
Sol. We know that the equation of plane is

Three Dimensional Geometry 423


@Cbsebookshub - Join Us on Telegram
r . nt = d; where nt is normal unit vector and d is perpendicular distance from origin.
it + jt + 3kt 1 t t 4
Here, nt = = (i + j + 3kt) and d =
12 + 12 + 32 11 11
∴ Equation of plane
1 t t 4
r . (i + j + 3kt) =   ⇒  r . (it + jt + 3kt) = 4
11 11
⇒ x + y + 3z = 4 ...(i)
Equation of given line
r = (–it – 2jt – 3kt) + m (3it + 4jt + 3kt) Q (, , )
Its cartesian form is
x+1 y+2 z+3
= = ...(ii)
3 4 3 x  y  3z  
Let Q (a, b, g) be the point of intersection of (i) & (ii)
 Q lies on (ii)
x 1 y  2 z  3
 
a+1 b+2 c+3 3 4 3

= = = m
3 4 3
⇒ a = 3l – 1, b = 4l – 2, g = 3l – 3

Also, Q lies on (i)
∴ a + b + 3g = 4  ⇒  3l – 1 + 4l – 2 + 9l – 9 = 4  ⇒  16l = 16 ⇒ λ = 1
∴ a = 2, b = 2, g = 0

∴ Required point of intersection ≡ (2, 2, 0)

17. A variable plane which remains at a constant distance 3p from the origin cuts the coordinate
1 1 1 1
axes at A, B, C. Show that the locus of the centroid of triangle ABC is 2 + 2 + 2 = 2 .
x y z p
[CBSE (AI) 2017]
Sol. Let the given variable plane meets X, Y and Z axes at A(a, 0, 0), B(0, b, 0), C(0, 0, c).
Therefore the equation of given plane is given by
x y z
a + + c = 1 … (i)
b
Let (a, b, c) be the coordinates of the centroid of triangle ABC. Then
a+0+0 a + +
a =
3
=
3
& a = 3a; b = 0 3b 0 = 3b & b = 3b
0+0+c c
c =
3
=
3
& c = 3c
 3p is the distance from origin to the plane (i)
1 1 1
0. a + 0. + 0. c –1
1 2
b l +c m +b l = –
b 1 2 1 2 1
⇒ 3p = ⇒ a c
2 b 3p
b l +c m +b l
2 2
1 1 1
a b c
Squaring both sides, we have
1 1 1 1 1 1 1 1
2 + 2 + 2 = 2   ⇒ 2 + 2 + 2 = 2 [Putting value of a = 3a, b = 3b, c = 3c ]
a b c 9p 9a 9b 9c 9p
1 1 1 1

2
+ 2+ 2 = 2
a b c p
1 1 1 1
Therefore, locus of (a, b, c) is 2
+ 2 + 2 = 2 Hence proved.
x y z p

424 Xam idea Mathematics–XII

@Cbsebookshub - Join Us on Telegram


18. Find the image P′ of the point P having position vector it + 3jt + 4kt in the plane
r . (2it – jt + kt ) + 3 = 0 . Hence find the length of PP′. [CBSE (F) 2013, 2017]
Sol. Let given point be P (1, 3, 4) and the equation of given plane in P (i + 3 j + 4 k)
cartesian form be
2x – y + z + 3 = 0 ...(i) n = (2 i – j + k )

Let R (x1, y1, z1) be the foot of perpendicular and P′ (a, b, g) be the

image of P R (x1, y1, z1)
Since, R (x1, y1, z1) lie on plane (i)
2x1 – y1 + z1 + 3 = 0 ...(ii)
r (2i – j + k ) + 3 = 0
Also, normal vector n of plane (i) is n = 2it – jt + kt

2x – y + z + 3 = 0
and PR = (x1 – 1) it + (y1 – 3) jt + (z1 – 4) kt P′ ( , , )

∴ PR || n
Since R is the mid point of PP′
x1 – 1 y1 – 3 z1 – 4 a+1

2
=
–1
=
1
= m ∴ –1=
2
& a=–3
b+3
⇒ x1 = 2l +1, y1 = –l + 3, z1 = l + 4 4=
2
& b=5
c+4
Putting x1, y1, z1 in (ii), we get 3=
2
& c=2
⇒ 2 (2l + 1) – (–l + 3) + (l + 4) + 3 = 0 Hence, image P′ = (–3, 5, 2)
⇒ 4l + 2 + l – 3 + l + 4 + 3 = 0

⇒ 6l + 6 = 0
⇒ λ=–1
∴ R ≡ (x1, y1, z1) ≡ (–1, 4, 3)

PPl = (– 3 – 1) 2 + (5 – 3) 2 + (2 – 4) 2 = 16 + 4 + 4 = 24 = 2 6 units
x+2 y–3 z+1
19. Find the equation of the line which intersects the lines = = and
1 2 4
x–1 y–2 z–3
= = and passes through the point (1, 1, 1). [CBSE Sample Paper 2018]
2 3 4
Sol. Let l1, l2 be given lines as
x+2 y – 3 z+1 x–1 y–2 z–3
l1 : = = ; l2 : = =
1 2 4 2 3 4
Let l be the required line, which passes through P (1, 1, 1) and intersect l1 and l2 at
Q (a1, b1, g1) and R (a2, b2, g2) respectively.
Now, Q (a1, b1, g1) lie on line l1
a1 + 2 b1 – 3 c1 + 1
\
= = = m (say)
1 2 4
  a1 = λ – 2, b1 = 2λ + 3, g1 = 4λ – 1
Similarly, R (a2 ,b2 , g2) lie on line l2
a2 – 1 b2 – 2 c2 – 3
= = = n (say)
2 3 4

a2 = 2m+1, b2 = 3m + 2, g2 = 4m + 3
\
PQ = (a1 – 1) it + (b1 – 1) jt + (c1 – 1) kt

Three Dimensional Geometry 425


@Cbsebookshub - Join Us on Telegram
= (m – 3) it + (2m + 2) jt + (4m – 2) kt

Similarly, PR = 2nit + (3n + 1) jt + (4n + 2) kt


m – 3 = 2m + 2 = 4m – 2 =
 PQ || PR ⇒ M (say)
2n 3n + 1 4n + 2
m–3 = m–3
Now, M & m – 3 = 2 Mn & Mn =
2n 2
2m + 2 =
Also, M & 2m + 2 = 3Mn + M
3n + 1
3m – 9 + 3m – 9 =
& 2m + 2 = M & 2m + 2 – M
2 2
4m + 4 – 3m + 9 = m + 13 =
& M & M
2 2
4m – 2 =
Also, M & 4m – 2 = 4Mn + 2M
4n + 2
8m – 4 – 4m + 12 =
4m – 12 +
& 4m – 2 = 2M 2M &
2
2
+
&
4m + 8 =
2
2M & 4 (m2 2) = 2M
⇒ λ + 2 = M & m + 2 = m +2 13
3
⇒ 2λ + 4 = λ + 13 ⇒ λ = 9 ⇒ M = 11 & n =
11
∴ PQ = 6it + 20jt + 34kt
x–1 = y–1 = z–1 x–1 = y–1 = z–1
Hence, equation of required line is
6 20 34
& 3 10 17

Long Answer Questions [5 marks]


1. Find the vector and cartesian equations of the line which is perpendicular to the lines with
x+2 z–3 z+1 x–1 y–2 z–3
equations = = and = = and passes through the point (1, 1, 1).
1 2 4 2 3 4
Also find the angle between the given lines . [CBSE 2020 (65/5/1)]
Sol. Let the cartesian equation of the required line be
x–1 y–1 z–1
= = ...(i)
a b c
where, a, b, c are direction ratios and given lines are
x+2 y – 3 z+1
= = ...(ii)
1 2 4
x–1 y–2 z–3
and = = ...(iii)
2 3 4
Since the line (i) is perpendicular to both the lines (ii) and (iii)

∴ a × 1 + b × 2 + c × 4 = 0
⇒ a + 2b + 4c = 0

Also, a × 2 + b × 3 + c × 4 = 0 ⇒ 2a + 3b + 4c = 0

On solving these two equations, we get


a –b c a b c
= = ⇒ = =
8 – 12 4 – 8 3 – 4 –4 4 –1

426 Xam idea Mathematics–XII

@Cbsebookshub - Join Us on Telegram


∴ Direction ratios of the required line be – 4, 4, –1
∴ Vector and cartesian equation of required line be
r = it + jt + kt + m (– 4it + 4jt –kt)
x–1 y–1 z–1
and, = = respectively.
–4 4 –1
Let θ be the angle between given lines
1× 2 + 2 × 3 + 4 × 4 24
∴ cos i = =
1 + 4 + 16 × 4 + 9 + 16 609
24
⇒ i = cos –1 e
o
609

2. Find the coordinates of the foot of perpendicular and the length of the perpendicular drawn
from the point P (5, 4, 2) to the line r = –it + 3jt + kt + m (2it + 3jt –kt) . Also find the image of P in
this line. [CBSE (AI) 2012]
Sol. Given line is

r = –it + 3jt + kt + m (2it + 3jt –kt)


It can be written in cartesian form as
x + 1 y –3 z –1
= = ...(i)
2 3 –1
Let Q (a, b, g) be the foot of perpendicular drawn from P(5, 4, 2) to the line (i) and P’ (x1, y1, z1) be
the image of P on the line (i)
 Q (a, b, g) lie on line (i)

a + 1 b –3 c –1

= = = m (say)
2 3 –1
⇒ a = 2l –1; b = 3l + 3 and g = –l + 1 ...(ii)
P (5, 4, 2)

Now, PQ = (a –5) it + (b – 4) jt + (c – 2) kt

Parallel vector of line (i) b = 2it + 3jt –kt . b

Obviously PQ = b & PQ . b = 0 Q (, , )


      
2 (a – 5) + 3 (b – 4) + (–1) (g – 2) = 0 r = –i + 3j + k +  (2i + 3j – k)

⇒ 2a – 10 + 3b – 12 – g + 2 = 0
⇒ 2a + 3b – g – 20 = 0
⇒ 2 (2l – 1) + 3 (3l + 3) – (–l + 1) – 20 = 0
[Putting value of a, b, g from (ii)] P' (x1, y1, z1)
⇒ 4l – 2 + 9l + 9 + l – 1 – 20 = 0
⇒ 14l – 14 = 0 ⇒ l = 1
Hence the coordinates of foot of perpendicular Q are (2 × 1 – 1, 3 × 1 + 3, – 1 + 1), i.e., (1, 6, 0)

Length of perpendicular = (5 –1) 2 + (4 – 6) 2 + (2 – 0) 2
= 16 + 4 + 4 = 24 = 2 6 units.
Also, since Q is mid-point of PP′
x +5
∴ 1 = 1
2
& x1 = –3
y +4 z +2
   6 = 1 ⇒ y1 = 8   0 = 1 ⇒ z1 = –2
2 2
Therefore required image is (–3, 8, –2).

Three Dimensional Geometry 427


@Cbsebookshub - Join Us on Telegram
3. Find the equation of the plane that contains the point (1, – 1, 2) and is perpendicular to both
the planes 2x + 3y – 2z = 5 and x + 2y – 3z = 8. Hence find the distance of point P(–2, 5, 5) from
the plane obtained above. [CBSE (F) 2014]
Sol. Equation of plane containing the point (1, –1, 2) is given by
a(x – 1) + b (y + 1) + c (z – 2) = 0
...(i)

 (i) is perpendicular to plane 2x + 3y – 2z = 5
∴ 2a + 3b – 2c = 0 ...(ii)
Also, (i) is perpendicular to plane x + 2y – 3z = 8
a + 2b – 3c = 0 ...(iii)
From (ii) and (iii), we get
a b c
= =
–9 + 4 –2 + 6 4 –3
a b c
⇒ = = = m (say)   ⇒ a = –5l, b = 4l, c = l
–5 4 1
Putting these values in (i), we get

–5l (x – 1) + 4l(y + 1) +l (z – 2) = 0

–5 (x – 1) + 4(y + 1) + (z – 2) = 0
⇒ –5x + 5 + 4y + 4 + z – 2 = 0
⇒ –5x + 4y + z + 7 = 0
⇒ 5x – 4y – z – 7 = 0
... (iv) is the required equation of plane.
Again, if d be the distance of point P (–2, 5, 5) to plane (iv), then
5× ]–2g + (–4) ×5 + (–1) ×5 –7 –10 –20 – 5 – 7 42
d = = = = 42 units
5 2 + (–4) 2 + (–1) 2 25 + 16 + 1 42
4. Find the vector equation of the plane passing through the points (2, 1, –1) and (–1, 3, 4) and
perpendicular to the plane x – 2y + 4z = 10. Also show that the plane thus obtained contains the
line r = – it + 3jt + 4kt + m (3it – 2jt – 5kt) . [CBSE (AI) 2013, (F) 2012, 2013]
Sol. Let the equation of plane through (2, 1, –1) be
(i)
a (x – 2) + b ( y – 1)+ c (z + 1) = 0 ...(i)
   

 (i) passes through (–1, 3, 4) n1 (ai + bj + ck )

∴ a (–1 –2) + b (3 – 1) + c (4 + 1) = 0
   
n2 (i – 2j + 4k )
⇒ –3a + 2b + 5c = 0 ...(ii)
Also plane (i) is perpendicular to plane x – 2y + 4z = 10

⇒ n1 = n2 & n1 . n2 = 0 x – 2y + 4z = 10

∴ 1 a – 2b + 4c = 0 ...(iii)

From (ii) and (iii), we get
a b c a b c
= =
8 + 10 5 + 12 6 – 2
& =
18 17 4
= = m (say)

⇒ a = 18l, b = 17 l , c = 4l

Putting the value of a, b, c in (i), we get


18 l (x – 2) + 17l (y – 1) + 4l (z + 1) = 0

⇒ 18x – 36 + 17y – 17 + 4z + 4 = 0  ⇒  18x + 17y + 4z = 49


428 Xam idea Mathematics–XII

@Cbsebookshub - Join Us on Telegram


∴ Required vector equation of plane is

r . (18it + 17jt + 4kt) = 49 ...(iv)


Obviously plane (iv) contains the line

r = (–it + 3jt + 4kt) + m (3it – 2jt –5kt) ...(v)


Since, point (–it + 3jt + 4kt) satisfy equation (iv) and vector (18it + 17jt + 4kt) is perpendicular to,
(3it – 2jt – 5kt), as (–it + 3jt + 4kt) . (18it + 17jt + 4kt) = –18 + 51 + 16 = 49
and (18it + 17jt + 4kt) . (3it – 2jt –5kt) = 54 – 34 – 20 = 0
Therefore, (iv) contains line (v).
5. Find the vector and Cartesian equations of a plane containing the two lines.
r = 2it + jt – 3kt) + m (it + 2jt + 5kt)
( and r = (3it + 3jt + 2kt) + n (3it – 2jt + 5kt) [CBSE Delhi 2013]
Sol. Given lines are
r = (2it + jt – 3kt) + m (it + 2jt + 5kt) ...(i)

r = (3it + 3jt + 2kt) + n (3it – 2jt + 5kt) ...(ii)

Here a1 = 2it + jt + 3kt; a2 = 3it + 3jt + 2kt

b1 = it + 2jt + 5kt; b2 = 3it – 2jt + 5kt


it jt kt
Now, b ×b = 1 2 5
1 2
3 –2 5
= (10 + 10) it – (5 – 15) jt + (– 2 – 6) kt = 20it + 10jt – 8kt
Hence, vector equation of required plane is

( r – a1) . (b1 ×b2) = 0   ⇒  r . (b1 ×b2) = a1 . (b1 ×b2)



r . (20it + 10jt – 8kt) = (2it + jt – 3kt) . (20it + 10jt – 8kt)


r . (20it + 10jt – 8kt) = 40 + 10 + 24


r . (20it + 10jt – 8kt) = 74 & r . (20it + 10jt – 8kt) = 74 & r . (10it + 5jt – 4kt) = 37
Therefore, Cartesian equation is 10x + 5y – 4z = 37
6. Find the distance of the point (2, 12, 5) from the point of intersection of the line
r = 2it – 4jt + 2kt + m (3it + 4jt + 2kt) and the plane r . (it – 2jt + kt) = 0 . [CBSE (AI) 2014]

Sol. Given line and plane are


P(2,12,5)
r = (2it – 4jt + 2kt) + m (3it + 4jt + 2kt) …(i)

and r . (it – 2jt + kt) = 0 … (ii)


Q
For intersection point Q, we solve equations (i) and (ii) by putting the
value of r from (i) in (ii)
[(2it – 4jt + 2kt) + m (3it + 4jt + 2kt)] . (it – 2jt + kt) = 0

[(2 + 3m) it – (4 – 4m) jt + (2 + 2m) kt] . (it – 2jt + kt) = 0   ⇒  (2+3l) + 2 (4 – 4l) + (2 + 2l) = 0

2 + 3l + 8 – 8l + 2 + 2l = 0

Three Dimensional Geometry 429


@Cbsebookshub - Join Us on Telegram
⇒ 12 – 3l = 0

⇒ l = 4

Hence, position vector of intersecting point is 14it + 12it + 10kt .
Co-ordinate of intersecting point, Q ≡ (14, 12, 10)

Required distance = (14 – 2) 2 + (12 – 12) 2 + (10 – 5) 2 = 144 + 25 = 169 units


= 13 units.
7. Find the coordinate of the point P where the line through A(3, – 4, – 5) and B (2, – 3, 1) crosses
the plane passing through three points L(2, 2, 1), M(3, 0, 1) and N(4, –1, 0). Also, find the ratio
in which P divides the line segment AB. [CBSE Delhi 2016]
A (3, –4, –5)
Sol. Let the coordinate of P be (a, b, g).
Equation of plane passing through L(2, 2, 1), M(3, 0, 1) and K

N(4, –1, 0) is given by


L
x–2 y –2 z –1 x –2 y –2 z –1 (2, 2, 1)
P (, , )
3 – 2 0 – 2 1 – 1 = 0⇒ 1 –2 0 =0
4 – 2 –1 – 2 0 – 1 2 –3 –1 N
(4, –1, 0)
M (3, 0, 1)
⇒ (x – 2) (2 – 0) – (y – 2) (–1 – 0) + (z – 1) (–3 + 4) = 0

L

2 (x – 2) + ( y – 2) + z – 1 = 0
⇒ 2x – 4 + y – 2 + z – 1 = 0
B (2, –3, 1)
⇒ 2x + y + z – 7 = 0
...(i)
Now, the equation of line passing through A(3, –4, –5) and B(2, –3, 1) is given by
x–3 y+4 z+5 x – 3 y+4 z+5
= = ⇒ = = ...(ii)
2 – 3 –3 + 4 1 + 5 –1 1 6
 P (a, b, g) lie on line AB
a– 3 b+4 c+5
⇒ = = = m (say)  ⇒ a = – l + 3, b = l – 4, g = 6l – 5
–1 1 6
Also P (a, b, g) lie on plane (i)
⇒ 2a + b + g – 7 = 0
⇒ 2 (–l + 3) + (l – 4) + (6l – 5) – 7 = 0
⇒ –2l + 6 + l – 4 + 6l – 5 – 7 = 0
⇒ 5l – 10 = 0  ⇒  l = 2
∴ a = 1, b = –2, g = 7


Co-ordinate of P ≡ (1, –2, 7)
Let P divides AB in the ratio K : 1.
K × 2 + 1× 3
∴ 1 =   ⇒  K + 1 = 2K + 3  ⇒  K = –2
K+1
⇒ P divides AB externally in the ratio 2 : 1.

8. From the point P(a, b, c), perpendiculars PL and PM are


drawn to YZ and ZX planes respectively. Find the equation
of the plane OLM. [CBSE Bhubaneshwar 2015]
Sol. Obviously, the coordinates of O, L and M are (0, 0, 0), (0, b, c)
and (a, 0, c).
Therefore, the equation of required plane is given by
x–0 y–0 z–0 x y z
0 – 0 b – 0 c – 0 = 0   ⇒  0 b c =0
a–0 0–0 c–0 a 0 c

430 Xam idea Mathematics–XII

@Cbsebookshub - Join Us on Telegram


⇒ x(bc – 0) – y(0– ac) + z(0 – ab) = 0

⇒ bcx + acy – abz = 0

9. Find the equation of the plane through the line of intersection of the planes x + y + z = 1 and
2x + 3y + 4z = 5 which is perpendicular to the plane x – y + z = 0. Also find the distance of the
plane obtained above, from the origin. [CBSE (AI) 2014; (F) 2017]
Sol. The equation of a plane passing through the intersection of the given planes is
(x + y + z –1) + l (2x + 3y + 4z – 5) = 0

(1 + 2l ) x + ( 1 + 3l) y + (1 + 4l ) z – (1+5l) = 0 … (i)
Since, (i) is perpendicular to x – y + z = 0

(1 + 2l) 1 + (1 + 3l) (–1) + (1 + 4l) 1 = 0
1

1 + 2l – 1 – 3l + 1 + 4l = 0  ⇒  3l + 1 = 0  ⇒  l = –
3
Putting the value of l in (i), we get

c1– m x + (1 –1) y + c1– m z – c1– m = 0


2 4 5 x z 2
3 3 3
& – + =0
3 3 3
⇒ x – z + 2 = 0, it is required plane.

Let d be the distance of this plane from origin.
0. x + 0. y + 0. (–z) + 2 2

d= = = 2 units.
1 2 + 0 2 + (–1) 2 2
aa + bb + cc + d
[Note: The distance of the point ( a, b, g) to the plane ax + by + cz + d = 0 is given by .
a2 + b2 + c2
10. Find the vector equation of the plane passing through three points with position vectors
it + jt – 2kt , 2it – jt + kt and it + 2jt + kt . Also find the coordinates of the point of intersection of
this plane and the line r = 3it – jt – kt + m (2it – 2jt + kt) . [CBSE Delhi 2013]

Sol. The equation of plane passing through three points it + jt – 2kt, 2it – jt + kt and it + 2jt + kt
i.e., (1, 1, –2), (2, – 1, 1) and (1, 2, 1) is

x –1 y –1 z+2
2– 1 – 1 – 1 1 + 2 = 0
1–1 2–1 1+2

x –1 y –1 z+2  ^ ^ ^ ^ ^
r = 3i – j – k + (2i – 2j + k)
^


1 –2 3 =0
0 1 3
⇒ (x –1)(–6 – 3) – (y –1) (3 – 0) + (z + 2) (1 + 0) =0

(1,1,–2)
⇒ –9x + 9 – 3y + 3 + z + 2 = 0 ()
⇒ 9x + 3y – z = 14
…(i) (2,–1,1) (1,2,1)

Its vector form is r . (9it + 3jt – kt) = 14

The given line is r = (3it – jt –kt) + m (2it – 2jt + kt)


Its cartesian form is
x – 3 y+1 z+1
= = …(ii)
2 –2 1
Let the line (ii) intersect plane (i) at (a, b, g)

Three Dimensional Geometry 431


@Cbsebookshub - Join Us on Telegram
 (a, b, g) lie on (ii)
a– 3 b+1 c+1
⇒ = = = m (say)   ⇒ a = 2l + 3; b = –2l – 1; g = l – 1
2 –2 1
Also, point (a, b, g) lie on plane (i)
⇒ 9 a + 3b – g = 14   ⇒
9 (2l + 3) + 3 (–2l – 1) – (l – 1) = 14
⇒ 18l + 27 – 6l – 3 – l + 1 = 14   ⇒ 11l + 25 = 14

⇒ 11l = 14 – 25   ⇒ 11l = – 11

⇒ l = – 1

Therefore, point of intersection ≡ (1, 1, – 2).

11. Find the direction ratios of the normal to the plane, which passes through the points (1, 0, 0)
r
and (0, 1, 0) and makes angle with the plane x + y = 3. Also find the equation of the plane.
4
[CBSE Patna 2015]
Sol. Let the equation of plane passing through the point (1, 0, 0) be
a (x – 1) + b ( y – 0)+ c (z – 0) = 0
⇒ ax – a + by + cz = 0
⇒ ax + by + cz = a ...(i)
Since, (i) also passes through (0, 1, 0)
⇒ 0 + b + 0 = a

b = a ...(ii)
r
Given, the angle between plane (i) and plane x + y = 3 is .
4
r a.1 + b.1 + c.0 1 a+b
∴ cos = 2   ⇒  =
4 2 2 2
a +b +c 1 +1 2
2 a + b + c2 1 + 1
2 2

1 a+b a+b

= & 1=
2 a b2 + c2 2
2
+ a + b2 + c2
2


a 2 + b 2 + c 2 = ! (a + b) & a 2 + b 2 + c 2 = (a + b) 2
⇒ a2 + b2 + c2 = a2 + b2 + 2ab

⇒ c2 = 2ab
⇒ c2 = 2a2 [From (ii)]

c =! 2 a
Now, equation (i) becomes ax + ay ! 2 az = a .

x + y ! 2 z = 1, is the required equation of plane.
Therefore, required direction ratios are 1, 1, ! 2 .
12. Find the equation of the plane which contains the line of intersection of the planes
r . (it – 2jt + 3kt) – 4 = 0 and r . (–2it + jt + kt) + 5 = 0
and whose intercept on x-axis is equal to that of on y-axis. [CBSE (North) 2016]

Sol. Given planes are r . (it – 2jt + 3kt) – 4 = 0 and r . (–2it + jt + kt) + 5 = 0

These can be written in cartesian form as
x – 2y + 3z – 4 = 0 ...(i)
and –2x + y + z + 5 = 0 . ..(ii)
Now the equation of plane containing the line of intersection of the planes (i) and (ii) is given by
(x – 2y + 3z – 4) + l(–2x + y + z + 5) = 0 ...(iii)

(1 – 2l ) x – (2 – l) y + (3 + l) z – 4 + 5l = 0 ⇒ (1 – 2l ) x – (2 – l ) y + (3 + l) z = 4 – 5l

432 Xam idea Mathematics–XII

@Cbsebookshub - Join Us on Telegram


x y z

+ + =1
4 – 5m 4 – 5m 4 – 5m
1 – 2m –2 + m 3+m
4 – 5m 4 – 5m
According to question =
1– 2m –2 + m
⇒ 1 – 2l = – 2 + l ⇒ 3l = 3 ⇒ l=1
Putting the value of l = 1 in (iii), we get
(x – 2y + 3z – 4) + 1 (–2x + y + z + 5) = 0
–x – y + 4z + 1 = 0 ⇒ x + y – 4z – 1 = 0
Its vector form is r . (it + jt – 4kt) – 1 = 0

13. If l1, m1, n1, l2, m2, n2 and l3, m3, n3 are the direction cosines of three mutually perpendicular lines,
then prove that the line whose direction cosines are proportional to l1 + l2 + l3, m1 + m2 + m3 and
n1 + n2 + n3 makes equal angles with them. [NCERT Exemplar]
Sol. Let a = l1 it + m1 jt + n1 kt;   b = l2 it + m2 jt + n2 kt;   c = l3 it + m3 jt + n3 kt
d = (l1 + l2 + l3) it + (m1 + m2 + m3) jt + (n1 + n2 + n3) kt
Also, let α, β and γ are the angles between a and d , b and d , c and d .
∴ cos α = l1 (l1 + l2 + l3) + m1 (m1 + m2 + m3) + n1 (n1 + n2 + n3)

= l12 + l1 l2 + l1 l3 + m12 + m1 m2 + m1 m3 + n12 + n1 n2 + n1 n3
= (l12 + m12 + n12) + (l1 l2 + l1 l3 + m1 m2 + m1 m3 + n1 n2 + n1 n3)
= 1 + 0 = 1
[a l12 + m12 + n12 = 1 and l1 = l2, l1 = l3, m1 = m2, m1 = m3, n1 = n2, n1 = n3]
Similarly, cos b = l2 (l1 + l2 + l3) + m2 (m1 + m2 + m3) + n2 (n1 + n2 + n3)
= 1 + 0 and cos g = 1 + 0

cos a = cos b = cos g

a = b = g
So, the line whose direction cosines are proportional to l1 + l2 + l3, m1 + m2 + m3, n1 + n2 + n3 makes
equal angles with the three mutually perpendicular lines whose direction cosines are l1, m1, n1, l2,
m2, n2 and l3, m3, n3 respectively.
14. Find the distance of the point (1, –2, 3) from the plane x – y + z = 5 measured parallel to the line
whose direction cosines are proportional to 2, 3, –6. [CBSE (F) 2015] [HOTS]
Sol. Let Q (a, b, g)be the point on the given plane
x – y + z = 5 ...(i)
Since PQ is parallel to given line
x–1 y+2 z–3
= = ...(ii) where P (1, –2, 3) is the given point.
2 3 –6
 PQ is parallel to given line (ii).
P(1, –2, 3) x–1 y–3 z+2
∴ PQ b (parallel vector of line).
2
=
3
=
–6
a–1 b+2 c– 3

= = =m
2 3 –6
⇒ a = 2l + 1, b = 3l – 2, g = – 6l + 3
Q( , , )
Now,  Q (a, b, g) lie on plane (i)

Three Dimensional Geometry 433


@Cbsebookshub - Join Us on Telegram
a – b + g = 5 ⇒  2λ + 1 – 3λ + 2 – 6λ + 3 = 5
1
–7λ + 6 = 5 ⇒ –7λ = –1 ⇒  m =
7
1 9 1 11 1 15
a = 2× + 1 = ; b = 3× – 2 = – and c = – 6× + 3 =
7 7 7 7 7 7
Therefore required distance
2 2 2
c – 1m + c– + 2m + c – 3m =
9 11 15 4 9 36
PQ = + + = 1 = 1 unit.
7 7 7 49 49 49
15. A plane meets the coordinate axes in A, B, C, such that the centroid of the triangle ABC is the
x y z
point (a, b, g). Show that the equation of the plane is + + = 3.  [HOTS]
a b c
Sol. Let the equation of required plane be
x y z
+ + = 1 ...(i)
a b c
Then the coordinates of A, B, C are (a, 0, 0), (0, b, 0) and (0, 0, c) respectively. So, the centroid of
triangle ABC is c , , m . But the coordinates of the centroid are (a, b, g) as given in problem.
a b c
3 3 3
a b c
a = , b = and c = & a = 3a, b = 3b, c = 3c
3 3 3
Substituting the values of a, b and c in equation (i), we get the required equation of the plane as follows
x y z x y z
+
3a 3b 3c
+ =1 & + + = 3.
a b c

PROFICIENCY EXERCISE
QQ Objective Type Questions: [1 mark each]
1. Choose and write the correct option in each of the following questions.
(i) The coordinates of the foot of the perpendicular drawn from the point (2, 5, 7) on the x-axis
are given by [NCERT Exemplar]
(a) (2, 0, 0) (b) (0, 5, 0) (c) (0, 0, 7) (d) (0, 5, 7)
(ii) The co-ordinates of the foot of the perpendicular drawn from the point (–2, 8, 7) on the
XZ-plane is
(a) (–2, –8, 7) (b) (2, 8, –7) (c) (–2, 0, 7) (d) (0, 8, 0)
(iii) The locus represented by xy + yz = 0 is
(a) a pair of perpendicular lines (b) a pair of parallel lines
(c) a pair of perpendicular planes (d) a pair of parallel planes
(iv) The vector equation of XY-plane is [CBSE 2020 (65/3/1)]
(a) r . kt = 0 (b) r . jt = 0 (c) r . it = 0 (d) r . n = 1
x–2 y–3 4–z x–1 y–4 z–5
(v) The lines= = and = = are mutually perpendicular if the
1 1 k k 2 –2
value of k is [CBSE 2020 (65/5/1)]
2 2
(a) – (b) (c) – 2 (d) 2
3 3
(vi) The angle between the planes 2x – y + z = 6 and x + y + 2z = 7 is
r r r r
(a) (b) (c) (d)
4 6 3 2

434 Xam idea Mathematics–XII

@Cbsebookshub - Join Us on Telegram


2. Fill in the blanks.
r 3r r
(i) If a line makes angles , and with X, Y, Z axes respectively, then its direction cosines
2 4 4
are _____________ .
x–1 y+1 z– 2 x+1 y –1 z– 3
(ii) The value of a for which the lines = = and = = are
1 a 3 2a 1 1
perpendicular to each other, is _____________ .
(iii) The vector equation of the line through the points (3, 4, –7) and (1, –1, 6) is _____________ .
(iv) The equation of the plane 2x + 5y – 3z = 4 in the vector form is _____________ .
(v) A plane passes through the points (2, 0, 0), (0, 3, 0) and (0, 0, 4). The equation of the plane is
_____________ .

QQ Very Short Answer Questions: [1 mark each]


2x – 1 4 – y z + 1
3. Cartesian equation of a line AB is = = .
2 7 2
Write the direction ratios of a line parallel to AB. [CBSE Sample Paper]
x–1 y–2 z–3 x–1 y–1 z–6
4. If the lines = = and = = are perpendicular, find the value of k.
–3 2k 2 3k 1 –5
5. If a line makes angles a, b, g with the direction of the axes, then write the value of
sin2 a + sin2 b + sin2 g .
6. If a line makes angles 90°, 135°, 45° with the X, Y and Z axes respectively, find its direction
cosines. [CBSE 2019 (65/1/1)]
7. Find the vector equation of the line which passes through the point (3, 4, 5) and is parallel to the
vector 2it + 2jt – 3kt . [CBSE 2019 (65/1/1)]
8. A line passes through the point with position vector 2it – jt + 4kt and is in the direction of the
vector it + jt – 2kt . Find the equation of the line in cartesian form. [CBSE 2019 (65/2/1)]
9. If a line has the direction ratios – 18, 12, – 4, then what are its direction cosines?
 [CBSE 2019 (65/3/1)]
x –1 y+4 z+4
10. Find the co-ordinates of the point where the line = = cuts the XY-plane.
3 7 2
 [CBSE 2020 (65/2/1)]
11. Find the distance of the plane 3x – 4y + 12z = 3 from the origin. [CBSE (AI) 2012]
12. Find the Cartesian equation of the line which passes through the point (–2, 4, – 5) and is parallel
x+3 4– y z+8
to the line = = . [CBSE Delhi 2013]
3 5 6
13. Find the length of the perpendicular drawn from the origin to the plane 2x – 3y + 6z + 21 = 0.
 [CBSE (AI) 2013]
3–x y + 4 2z – 6
14. If the cartesian equations of a line are = = , write the vector equation for the
5 7 4
line. [CBSE (AI) 2014]
15. Find the angle between the lines r = 2it – 5jt + kt + m (3it + 2jt + 6kt) and r = 7it – 6kt + n (it + 2jt + 2kt) .
 [CBSE (F) 2014]
t t t
16. Write the sum of intercepts cut off by the plane r . (2i + j – k) – 5 = 0 on the three axes.
 [CBSE North 2016]
17. Find the vector equation of the plane with intercepts 3, – 4 and 2 on x, y and z axes.
 [CBSE Central 2016]

Three Dimensional Geometry 435


@Cbsebookshub - Join Us on Telegram
QQ Short Answer Questions–I: [2 marks each]
18. Find the coordinates of the point where the line
x+1 y+2 z+3
= = meets the plane x + y + 4z = 6.
2 3 4
19. If the line drawn from the point (–2, –1, –3) meets a plane at right angle at the point (1, –3, 3), find
the equation of the plane.
20. Find the distance of the point whose position vector is (2it + jt – kt) from the plane r . (it – 2jt + 4kt) = 9
21. Write the unit vector normal to the plane
x + 2y + 3z – 6 = 0.
22. Find the vector equation of a plane which is at a distance of 5 units from the origin and its normal
vector is 2it – 3jt + 6kt . [CBSE Delhi 2016]
23. Find the equation of the plane passing through the line of intersection of the planes
2x + 2y – 3z = 7 and 2x + 5y + 3z = 9 the point (2, 1, 3).
24. Find the angle between the planes, whose vector equations are r . (2it + 2jt – 3kt) = 5 and
r . (3it – 3jt + 5kt) = 3 .

QQ Short Answer Questions–II: [3 marks each]


25. Find the shortest distance between the following pair of skew lines:
x –1 2 –y z + 1 x + 2 y – 3 z
= = , = = [CBSE Sample Paper 2016]
2 3 4 1 2 3
x –1 y –1 z + 1 x –4 y z + 1
26. Show that the lines = = and = = intersect. Find their point of
3 –1 0 2 0 3
intersection. [CBSE (East) 2016]
27. Find the coordinates of the foot of perpendicular drawn from the point A(–1, 8, 4) to the line
joining the points B(0, –1, 3) and C(2, –3, –1) Hence find the image of the point A in the line BC.
[CBSE (North) 2016]
28. Find the equation of plane passing through the points A(3, 2, 1), B(4, 2, –2) and C(6, 5, –1) and
hence find the value of λ for which A(3, 2, 1), B(4, 2, –2), C(6, 5, –1) and D(l, 5, 5) are coplanar.
 [CBSE (South) 2016]
29. Prove that the line through A(0, –1, –1) and B(4, 5, 1) intersects the line through C(3, 9, 4) and
D(–4, 4, 4). [CBSE (F) 2016]
30. Show that the following two lines are coplanar:
x – a+d y – a z– a– d x –b+c y –b z–b–c
= = and = = [CBSE Patna 2015]
a–d a +
a d b–c b b+c
31. Find the acute angle between the plane 5x – 4y + 7z – 13 = 0 and the y-axis. [CBSE Patna 2015]
32. Find the equation of a plane which is at a distance of 3 3 units from origin and the normal to
which is equally inclined to the coordinate axes. [CBSE (F) 2013]
33. Let P(3, 2, 6) be a point in the space and Q be a point on the line r = (it – jt + 2kt) + n (– 3it + jt + 5kt),
then find the value of n for which the vector PQ is parallel to the plane x – 4y + 3z = 1.
 [CBSE Chennai 2015]
34. Find the vector and cartesian equations of the plane which bisects the line joining the points
(3, –2, 1) and (1, 4, –3) at right angles. [CBSE Chennai 2015]
35. Find the distance of the point P(3, 4, 4) from the point, where the line joining the points
A(3, –4, –5) and B(2, –3, 1) intersect the plane 2x + y + z = 7.
[CBSE Allahabad 2015]

436 Xam idea Mathematics–XII

@Cbsebookshub - Join Us on Telegram


36. Find the distance of the point (1, –2, 3) from the plane x – y + z = 5 measured parallel to the line
whose direction cosines are proportional to 2, 3, –6. [CBSE (F) 2015]
x –1 y+1 z
37. Find the equation of the plane containing two parallel lines = = and
2 –1 3
x y – 2 z+1 x–2 y–1 z–2
= = . Also, find if the plane thus obtained contains the line = =
4 –2 6 3 1 5
or not.  [CBSE (South) 2016]
38. Find the vector and cartesian equations of a line through the point (1, –1, 1) and perpendicular to
the lines joining the points (4, 3, 2), (1, –1, 0) and (1, 2, –1), (2, 1, 1). [CBSE Guwahati 2015]
39. Find the shortest distance between the lines whose vector equations are
r = (1 – t) it + (t – 2) jt + (3 – 2t) kt and r = (s + 1) it + (2s – 1) jt + (2s + 1) kt . [CBSE (AI) 2011]
40. Find the vector and cartesian equations of the line passing through the point P (1, 2, 3) and
parallel to the planes r . (it –jt + 2kt) = 5 and r . (3it + jt + kt) = 6 . [CBSE (F) 2012]

QQ Long Answer Questions: [5 marks each]


41. Find the equation of the plane passing through the point P(1, 1, 1) and containing the line
r = (–3it + jt + 5kt) + m (3it – jt – 5kt) . Also, show that the plane contains the line
r = (–it + 2jt + 5kt) + n (it – 2jt – 5kt) . [CBSE (AI) 2010]
42. Find the vector and cartesian equations of the plane passing through the intersection of the planes
r . (it + jt + kt) = 6 and r . (2it + 3jt + 4kt) = – 5 and the point (1, 1, 1). [CBSE Chennai 2015]
43. Find the value of k for which the following lines are perpendicular to each other:
x+3 y –1 5– z x+2 2– y z
= = ; = =
k–5 1 – 2k – 1 – 1 –k 5
Hence, find the equation of the plane containing the above lines. [CBSE Guwahati 2015]
44. Show that the lines:
r = it + jt + kt + m (it – jt + kt) and r = 4jt + 2kt + n (2it – jt + 3kt) are coplanar.
Also, find the equation of the plane containing these lines. [CBSE Panchkula 2015]
45. Show that the lines r = (– 3it + jt + 5kt) + m (– 3it + jt + 5kt) and r = (– it + 2jt + 5kt) + n (– it + 2jt + 5kt) are
co-planar. Also, find the equation of the plane containing these lines. [CBSE Sample Paper 2015]
46. Find the position vector of the foot of perpendicular and the perpendicular distance from the
point P with position vector 2it + 3jt + 4kt to the plane r (2it + jt + 3kt) – 26 = 0. Also, find image of P
in the plane. [CBSE (Central) 2016]
47. Find the coordinates of the foot of perpendicular and perpendicular distance from the point
P(4, 3, 2) to the plane x + 2y + 3z = 2 Also find the image of P in the plane. [CBSE (East) 2016]
48. Find the equation of the plane which contains the line of intersection of the planes x + 2y + 3z – 4 = 0
and 2x + y – z + 5 = 0 and whose x-intercept is twice its z-intercept.
Hence write the vector equation of a plane passing through the point (2, 3, –1) and parallel to the
plane obtained above. [CBSE (F) 2016]
49. Find the coordinate of the point where the line through (3, –4, –5) and (2, –3, 1) crosses the plane,
passing through the points (2, 2, 1), (3, 0, 1) and (4, –1, 0). [CBSE Delhi 2013]
50. Show that lines r = (it + jt – kt) + m (3it – jt) and r = (4it – kt) + n (2it + 3kt) intersect. Also find their
point of intersection. [CBSE Delhi 2014]

Three Dimensional Geometry 437


@Cbsebookshub - Join Us on Telegram
51. Find the distance between the point (7, 2, 4) and the plane determine by the points A(2, 5, –3),
B(–2, –3, 5) and C(5, 3, –3). [CBSE Delhi 2014]
x–2 y–2 z–3 x–2 y–3 z–4
52. Show that the lines = = and = = intersect. Also, find the
1 3 1 1 4 2
co-ordinate of the point of intersection and equation of the plane containing the two lines.
 [CBSE 2020 (65/3/1)]

Answers
1. (i) (a) (ii) (c) (iii) (c) (iv) (a) (v) (a) (vi) (c)

2. (i) ! d 0, – n (ii) –1 (iii) r = 3it + 4jt – 7kt + m _ –2it – 5jt + 13kti


1 1
,
2 2
x y z
(iv) r . _ 2it + 5jt – 3kti = 4 (v) + + = 1
2 3 4
–10 1 1
3. 1, –7, 2 4. k = 5. 2 6. 0, – ,
7 2 2
x –2 y+1 z– 4
7. r = (3it + 4jt + 5kt) + m (2it + 2jt – 3kt) 8. = =
1 1 –2
–9 6 –2 3 x+2 y – 4 z+5
9. l = ,m= ,n= 10. (7, 10, 0) 11. 12. = =
11 11 11 13 3 –5 6
13. 3 units 14. r = (3it – 4jt + 3kt) + m (–5it + 7jt + 2kt) 15. i = cos c m
–1 19
21
jt kt
17. r . d – + n = 1
5 it
16. 18. (1, 1, 1) 19. 3x – 2y + 6z – 27 = 0
2 3 4 2
2it 3jt 6kt n =
22. r . d
13 1 t 2 t 3 t
20. 21. i+ j+ k – + 5
21 14 14 14 7 7 7

24. cos –1 d n
15 42
23. 38x + 68y + 3z = 153 units 26. (4, 0, –1)
25.
731 390

28. 9x – 7y + 3z – 16 = 0 ; m = 4 31. i = sin –1 d n


4
27. (–2, 1, 7); (–3, –6, 10)
3 10
1
32. x + y + z = 9 33. n = 34. r . (it – 3jt + 2kt) + 3 = 0; x – 3y + 2z + 3 = 0 35. 7 units
4
36. 1 unit 37. 8x + y – 5z = 7; yes the plane contains the given line
x–1 y+1 z–1 8
38. = = ; r = (it – jt + kt) + m (10it – 4jt – 7kt) 39. units
10 –4 –7 29
x–1 y–2 z–3
40. r = (it + 2jt + 3kt) + m (–3it + 5jt + 4kt), = = 41. r . (it – 2jt + kt) = 0
–3 5 4

42. 20x + 23y + 26z – 69 = 0; r . (20it + 23jt + 26kt) = 69 43. k = – 1, 4x + 31y + 7z = 54

44. r . (–2it – jt + kt) + 2 = 0 45. x – 2y + z = 0


7 11 t 7
46. 3it + jt + k; units, (4, 4, 7) 47. (3, 1, –1); 14 sq. units; (2, –1, –4)
2 2 2
48. 13x + 14y + 11z = 0 or 7x + 11y + 14z – 15 = 0, { r – (2it + 3jt – kt)} . (7it + 11jt + 14kt) = 0

49. (1, –2, 7) 50. (4, 0, –1) 51. 29 units 52. (1, –1, 2); 2x – y + z – 5 = 0

438 Xam idea Mathematics–XII

@Cbsebookshub - Join Us on Telegram


SELF-ASSESSMENT TEST
Time allowed: 1 hour Max. marks: 30
1. Choose and write the correct option in the following questions. (4 × 1 = 4)
(i) The foot of perpendicular from (a, b, g) on y-axis is
(a) (a, 0, 0) (b) (0, b, 0) (c) (0, 0, g) (d) (0, b, g)

(ii) Find the equation of the plane through the points (2, 1, –1), (–1, 3, 4) and perpendicular to the
plane x – 2y + 4z = 10
(a) 18x + 17y + 4z = 49 (b) 20x – 12y + 3z = 11
(c) 3x – 2y – 4z = 17 (d) 7x – 2y – 3z = 0

(iii) The coordinates of the point where the line through (3, – 4, – 5) and (2, –3, 1) crosses the plane
passing through three points (2, 2, 1), (3, 0, 1) and (4, – 1, 0) are
(a) (0, – 2, 7) (b) (3, – 2, 5) (c) (1, – 2, – 7) (d) (1, – 2, 7)

(iv) The distance between the parallel planes x + 2y – 3z = 2 and 2x + 4y – 6z + 7 = 0 is


2 11 7
(a) unit (b) unit (c) unit (d) none of these
14 56 56

2. Fill in the blanks. (2 × 1 = 2)


(i) The distance of the plane 2x – 3y + 6z + 14 = 0 from origin is _____________ units.

(ii) The Cartesian equation of the line joining the points (–2, 1, 3) and (3, 1, –2) is _____________ .

QQ Solve the following questions. (3 × 1 = 3)


x – 2 y+1 z– 3
3. Find the angle between the line : = = and the plane 3x + 4y + z + 5 = 0.
3 –1 2

4. Find the angle between the line: r = (5it – jt – 4kt) + m (2it – jt + 3kt) and the plane r . (3it + 4jt + kt) + 5 = 0.
x –1 y+4 z+4
5. Find the co-ordinates of the point where the line = = cuts the XY-plane.
3 7 2

QQ Solve the following questions. (2 × 2 = 4)


6. If the line drawn from the point (–2, –1, –3) meets a plane at right angle at the point (1, –3, 3), find
the equation of the plane.

7. Find the distance of the point whose position vector is (2it + jt – kt) from the plane r . (it – 2jt + 4kt) = 9.

QQ Solve the following questions. (4 × 3 = 12)

8. If a plane meets the coordinate axes in A, B, C such that the centroid of the ∆ABC is the point
(a, b, g), then find the equation of the plane.

9. Find the distance of the point (2, 3, 4) from the plane 3x + 2y + 2z + 5 =0 measured parallel to the
x + 3 y –2 z
line = = .
3 6 2

Three Dimensional Geometry 439


@Cbsebookshub - Join Us on Telegram
10. Find the distance of the point, whose position vector is (2it + jt –kt) from the plane r . (it –2jt + 4kt) = 9 .
11. Let P(3, 2, 6) be a point in the space and Q be a point on the line r = (it – jt + 2kt) + n (– 3it + jt + 5kt),
then find the value of n for which the vector PQ is parallel to the plane x – 4y + 3z = 1.

QQ Solve the following questions. (1 × 5 = 5)


x y –2 z –3
12. Find the equation of perpendicular from the point (3, –1, 11) to the line = = . Also
2 3 4
find the foot of the perpendicular and length of the perpendicular.

Answers
1. (i) (b) (ii) (a) (iii) (d) (iv) (b)

x+2 y –1 z– 3
2. (i) 2 (ii) = =
5 0 –5

3. sin –1 d n 4. sin –1 d n
7 5
5. (7, 10, 0)
2 91 2 91

13 x y z
6. 3x – 2y + 6z – 27 = 0 7.
8. + + =3
21 a b c

13 1 x – 3 = y + 1 = z – 11
9. 7 units 10. 11. n = 12. , (2, 5, 7), 53 units
21 4 1 –6 4
zzz

440 Xam idea Mathematics–XII

@Cbsebookshub - Join Us on Telegram


Linear 13
Programming

1. Definition: Linear programming (LP) is an optimisation technique in which a linear function is


optimised (i.e., minimised or maximised) subject to certain constraints which are in the form of
linear inequalities or/and equations. The function to be optimised is called objective function.
2. Applications of Linear Programming: Linear programming is used in determining optimum
combination of several variables subject to certain constraints or restrictions.
3. Formation of Linear Programming Problem (LPP): The basic problem in the formulation of a
linear programming problem is to set-up some mathematical model. This can be done by asking
the following questions:
(a) What are the unknowns (variables)?
(b) What is the objective?
(c) What are the restrictions?
For this, let x1, x2, x3, ..........xn be the variables. Let the objective function to be optimized (i.e.,
minimised or maximised) be given by Z.
(i) Z= c1 x1 + c2 x2 + ....... + cn xn, where ci xi (i = 1, 2, ...........n) are constraints.
(ii) Let there be mn constants and let a be a set of constants such that
a11 x1 + a12 x2 + ...... + a1n xn (≤, = or ≥) b1
a21 x1 + a22 x2 + ...... + a2n xn (≤, = or ≥) b2
…… …… ……
…… …… ……
aml x1 + am2 x2 + ....... + amn xn (≤, = or ≥) bm

(iii) Finally, let x1 ≥ 0, x2 ≥ 0, ...........xn ≥ 0, called non-negative constraints.
The problem of determining the values of x1, x2, ............., xn which makes Z, a minimum or
maximum and which satisfies (ii) and (iii) is called the general linear programming problem.
4. General LPP:
(a) Decision variables: The variables x1, x2, x3, ........, xn whose values are to be decided, are called
decision variables.

Linear Programming 441


@Cbsebookshub - Join Us on Telegram
(b) Objective function: The linear function Z = c1 x1 + c2 x2 + ........ + cn xn which is to be optimized
(maximised or minimised) is called the objective function or preference function of the general
linear programming problem.
(c) Structural constraints: The inequalities given in (ii), are called the structural constraints of the
general linear programming problem. The structural constraints are generally in the form of
inequalities of ≥ type or ≤ type, but occasionally, a structural constraint may be in the form of
an equation.
(d) Non-negative constraints: The set of inequalities (iii) is usually known as the set of non-negative
constraints of the general LPP. These constraints imply that the variables x1, x2, ...., xn cannot take
negative values.
(e) Feasible solution: Any solution of a general LPP which satisfies all the constraints, structural
and non-negative, of the problem, is called a feasible solution to the general LPP.
(f) Optimum solution: Any feasible solution which optimizes ( i.e., minimises or maximises) the
objective function of the LPP is called optimum solution.
5. Requirements for Mathematical Formulation of LPP: Before getting the mathematical form of a
linear programming problem, it is important to recognize the problem which can be handled by
linear programming problem. For the formulation of a linear programming problem, the problem
must satisfy the following requirements:
(i) There must be an objective to minimise or maximise something. The objective must be capable
of being clearly defined mathematically as a linear function.
(ii) There must be alternative sources of action so that the problem of selecting the best course of
actions may arise.
(iii) The resources must be in economically quantifiable limited supply. This gives the constraints to
LPP.
(iv) The constraints (restrictions) must be capable of being expressed in the form of linear equations
or inequalities.
6. Solving Linear Programming Problem: To solve linear programming problems, Corner Point
Method is adopted. Under this method following steps are performed:
Step I. At first, feasible region is obtained by plotting the graph of given linear constraints and
its corner points are obtained by solving the two equations of the lines intersecting at that
point.
Step II. The value of objective function Z = ax + by is obtained for each corner point by putting its
x and y-coordinate in place of x and y in Z = ax + by . Let M and m be largest and smallest
value of Z respectively.
Case I: If the feasible region is bounded, then M and m are the maximum and minimum
values of Z.
Case II: If the feasible region is unbounded, then we proceed as follows:
Step III. The open half plane determined by ax + by > M and ax + by < m are obtained.

Case I: If there is no common point in the half plane determined by ax + by > M and
feasible region, then M is maximum value of Z, otherwise Z has no maximum
value.
Case II: If there is no common point in the half plane determined by ax + by < m and
feasible region, then m is minimum value of Z, otherwise Z has no minimum
value.

442 Xam idea Mathematics–XII

@Cbsebookshub - Join Us on Telegram


Above facts can be represented by arrow diagram as

Selected NCERT Questions


1. Maximize Z = – x + 2y
Subject to x ≥ 3, x + y ≥ 5, x + 2y ≥ 6, y ≥ 0.
Sol. Let us graph the given inequalities

The feasible region is shown shaded which is unbounded.


The coordinates of the corner points of the feasible region are A(6, 0), B(4, 1) and C(3, 2).
Let us evaluate the objective function.

Linear Programming 443


@Cbsebookshub - Join Us on Telegram
Corner Points Objective Function
Z = –x + 2y
A (6, 0) Z=–6+2×0=–6+0=–6
B (4, 1) Z = – 4 + 2 × 1 = – 4 + 2 = –2
C (3, 2) Z=–3+2×2=–3+4=1 Maximum


We see that maximum value of Z at (3, 2) is 1.
Since the region is unbounded, so 1 may or may not be the maximum value of Z.
First draw the graph of the inequality –x + 2y > 1.
–x + 2y > 1 is away from origin.
Since the open half plane of –x + 2y > 1 has points in common with the feasible region.
Thus, Z has no maximum value.
A factory manufactures two types of screws, A and B. Each type of screw requires the use of
2.
two machines, an automatic and a hand operated. It takes 4 minutes on the automatic and 6
minutes on hand operated machines to manufacture a package of screws A, while it takes 6
minutes on automatic and 3 minutes on the hand operated machines to manufacture a package
of screws B. Each machine is available for at the most 4 hours on any day. The manufacturer
can sell a package of screws A at a profit of ™7 and screws B at a profit of ™10. Assuming that
he can sell all the screws he manufactures, how many packages of each type should the factory
owner produce in a day in order to maximise his profit? Determine the maximum profit.
Sol. Let x be the number of packages of screws A, y be the number of packages of screws B produced
in a day and Z be the total profit of the manufacture in a day.
Automatic Machine Hand Operated Machine Profit

Package of Screws A 4 minutes 6 minutes ™7


Package of Screws B 6 minutes 3 minutes ™ 10

Time Available 4 hours 4 hours

Thus, the mathematical formulation of the given LPP is


Maximize Z = 7x + 10y
Subject to 4x + 6y ≤ 240, 6x + 3y ≤ 240, x, y ≥ 0.

Let us draw the graph for system of inequalities representing constraints.

444 Xam idea Mathematics–XII

@Cbsebookshub - Join Us on Telegram


The feasible region is shown (shaded) in figure, which is bounded.
The coordinates of the corner points of the feasible region are O(0, 0), A(40, 0), B(30, 20) and C(0, 40).
Let us evaluate the objective function.
Corner Points Objective Function
Z = 7x + 10y
O (0, 0) Z = 7 × 0 + 10 × 0 = 0
A (40, 0) Z = 7 × 40 + 10 × 0 = 280 + 0 = 280
B (30, 20) Z = 7 × 30 + 10 × 20 = 210 + 200 = 410 Maximum
C (0, 40) Z = 7 × 0 + 10 × 40 = 0 + 400 = 400

Thus, Z is maximum at (30, 20) and maximum value = 410


∴ Maximum profit = ™410 when 30 packages of screws A and 20 packages of screws B are

produced in a day.
3. A factory makes two types of items A and B, made of plywood. One piece of item A requires 5
minutes for cutting and 10 minutes for assembling. One piece of item B requires 8 minutes for
cutting and 8 minutes for assembling. There are 3 hours and 20 minutes available for cutting
and 4 hours for assembling. The profit on one piece of item A is `5 and that on item B is `6.
How many pieces of each type should the factory make so as to maximise profit? Make it as an
LPP and solve it graphically. [CBSE (F) 2010]
Sol. Let the factory makes x pieces of item A and y pieces of item B.
Time required by item A (one piece)
cutting = 5 minutes, assembling = 10 minutes
Time required by item B (one piece) Y

cutting = 8 minutes, assembling = 8 minutes 30


Total time for A(0,25)

cutting = 3 hours and 20 minutes and 20 B (8, 20)

assembling = 4 hours
5x
10

+8
x+

Profit on one piece 10 y=


8y

20
0
=

For item A = `5, item B = `6


24

C (24, 0)
0

Thus, our problem is maximized as X′ X


O 10 20 30 40
Z = 5x + 6y ...(i) Y′
Subject to constraints:
x ≥ 0, y ≥ 0 ...(ii)
5x + 8y ≤ 200 ...(iii)
10x + 8y ≤ 240 ...(iv)
From figure, possible points for maximum value of Z are at O(0, 0) A(0, 25), B(8, 20), C(24, 0).
Corner Points Z = 5x + 6y
O (0, 0) 0

A (0, 25) 150

B (8, 20) 40 + 120 = 160 Maximum

C (24, 0) 120
Hence, 8 pieces of item A and 20 pieces of item B produce maximum profit of `160.

Linear Programming 445


@Cbsebookshub - Join Us on Telegram
4. A merchant plans to sell two types of personal computers— a desktop model and a portable
model that will cost `25000 and `40000 respectively. He estimates that the total monthly
demand of computers will not exceed 250 units. Determine the number of units of each type
of computers which the merchant should stock to get maximum profit if he does not want to
invest more than `70 lakhs and if his profit on the desktop model is `4500 and on portable
model is `5000. [CBSE (AI) 2011]
Sol. Let the number of desktop models be x and the number of portable models be y.
Since, the total monthly demand of computers does not exceed 250. Then we have
x + y ≤ 250
Also, cost of one desktop computer is ` 25000 and one portable computer is ` 40000.
Therefore, the cost of x desktop and y portable computers = ` (25000x + 40000y).
We have maximum investment = ` 7000000
⇒ 25000x + 40000y ≤ 7000000
⇒ 5x + 8y ≤ 1400
Now, profit on x desktop and y portable computers is given by
Z = 4500x + 5000y

250

200
C(0,175)
5x
x+

150 +8
y=

y=
25

14
00
0

100

50 B (200, 50)

A (250, 0)
X′ X
O 50 100 150 200 250 300
Y′

Hence our LPP is


Maximise, Z = 4500x + 5000y …(i)
Subject to the constraints:
x + y ≤ 250 …(ii)
5x + 8y ≤ 1400 …(iii)
x, y ≥ 0 …(iv)
Now, the shaded region OABC is the feasible region which is bounded.
The coordinates of corner points are O (0, 0), A (250, 0), B (200, 50), C (0, 175) .

446 Xam idea Mathematics–XII

@Cbsebookshub - Join Us on Telegram


Now, we evaluate Z at each corner point

Corner Points Z = 4500x + 5000y

O (0, 0) 0

A (250, 0) 1125000

B (200, 50) 1150000 Maximum

C (0, 175) 875000

Hence, the maximum profit of `1150000 is obtained, when he stocks 200 desktop and 50 portable
computers.

5. A diet is to contain at least 80 units of Vitamin A and 100 units of minerals. Two foods, F1 and
F2 are available costing `5 per unit and `6 per unit respectively. One unit of food F1 contains
4 units of vitamin A and 3 units of minerals whereas one unit of food F2 contains 3 units of
vitamin A and 6 units of minerals. Formulate this as a linear programming problem. Find the
minimum cost of diet that consists of mixture of these two foods and also meets minimum
nutritional requirement. [CBSE (South) 2016]

Sol. Let x units of food F1 and y units of food F2 are required to be mixed.
Y

45

40

35

30
A(0, 80 )
25 3

3x 20
+6
y=
10
0
15

10 B(12, 32 )
3
5
C ( 100 , 0 )
3
X'
O 5 10 15 20 25 30 35 40 45 X

5x + 6y = 124
Y'
4x + 3y = 80

Cost Z = 5x + 6y ...(i) is to be minimised

Linear Programming 447


@Cbsebookshub - Join Us on Telegram
Subject to following constraints.
4x + 3y ≥ 80 ...(ii)
3x + 6y ≥ 100 ...(iii)
x ≥ 0, y ≥ 0 ...(iv)
To solve the LPP graphically, the graph of inequations (ii), (iii) and (iv) is plotted as shown:
The shaded region in the graph is the feasible region of the problem. The corner points are
80 32 100
A c 0, m, B c 12, m, C c , 0m
3 3 3
The value of Z at corner point is given as

Corner Points Z = 5x + 6y

80
A d 0, n
80
5×0+6× = 160
3 3
32
B d 12, n
32
12 × 5 + 6 × = 124 Minimum
3 3
100
Cd , 0n
100
5× + 6 × 0 = 166.6
3 3

Since, feasible region is unbounded therefore, a graph of 5x + 6y = 124 is drawn which is shown
in figure by dotted line.
Also, since there is no point common in feasible region and region 5x + 6y < 124.
32
Hence, the minimum cost is `124 and 12 units of F1 and units of F2 are required.
3
6. Two godowns A and B have grain capacity of 100 quintals and 50 quintals respectively. They supply
to 3 ration shops, D, E and F whose requirements are 60, 50 and 40 quintals respectively. The cost of
transportation per quintal from the godowns to the shops are given in the following table :

Transportation cost per quintal (in ™)


From/To A B
D 6 4
E 3 2
F 2.50 3

How should the supplier be transported in order that the transportation cost is minimum?
What is the minimum cost? [HOTS]
Sol. Let the godown A transport x quintals of grain to ration shop D, y quintals of grain to ration shop E.
Since the total capacity of godown A is 100 quintals, so the remaining (100 – x – y) quintals of
grain can be transported to ration shop F.
Now the requirement of ration shop D is 60 quintals, out of which x quintals are transported from
godown A. The remaining (60 – x) quintals will be transported from godown B.
Also the requirement of ration shop E is 50 quintals, out of which y quintals are transported from
godown A. The remaining (50 – y) quintals will be transported from godown B.

448 Xam idea Mathematics–XII

@Cbsebookshub - Join Us on Telegram


x x

y y

y
x x
y

Since the total capacity of godown B is 50 quintals, so the remaining


50 – (60 – x + 50 – y) = (x + y – 60) quintals can be transported to ration shop F.
Let Z be the total transportation cost then
Z = 6x + 3y + 2.50(100 – x – y) + 4(60 – x) + 2(50 – y) + 3(x + y – 60)
= 6x + 3y + 250 – 2.50x – 2.50y + 240 – 4x + 100 – 2y + 3x + 3y – 180
= 2.50x + 1.50y + 410
Thus, the mathematical formulation of the given LPP is
Minimize Z = 2.50x + 1.50y + 410
Subject to x + y ≤ 100, x + y ≥ 60, x ≤ 60, y ≤ 50, x, y ≥ 0
Let us graph the system of inequalities representing the constraints.

Linear Programming 449


@Cbsebookshub - Join Us on Telegram
The feasible region ABCD is shown (shaded) in figure which is bounded.
The coordinates of the corner points of the feasible region are A (60, 0), B (60, 40), C (50, 50) and
D (10, 50). Let us evaluate the objective function.
Corner Points Objective Function
Z = 2.50x + 1.50y + 410

A (60, 0) Z = 2.50 × 60 + 1.50 × 0 + 410 = 150 + 410 = 560


B (60, 40) Z = 2.50 × 60 + 1.50 × 40 + 410 = 150 + 60 + 410 = 620 Maximum
C (50, 50) Z = 2.50 × 50 + 1.50 × 50 + 410 =125 + 75 + 410 = 610
D (10, 50) Z = 2.50 × 10 + 1.50 × 50 + 410 = 25 + 75 + 410 = 510 Minimum

Thus, Z is minimum at (10, 50) and minimum value = 510.


So, the minimum transportation cost is ™510 when 10 quintals, 50 quintals and 40 quintals are
transported from godown A and 50 quintals 0 quintal and 0 quintal are transported from godown
B to ration shops D, E and F respectively.

Multiple Choice Questions [1 mark]


Choose and write the correct option in the following questions.
1. The feasible region for an LPP is shown below: [NCERT Exemplar, CBSE 2020 (65/3/1)]
Let Z = 3x – 4y be the objective function. Minimum of Z occurs at

Y
(4, 10)
10
9

(0, 8) 8 (6, 8)

5 (6, 5)

4
3

2
1

X
(0, 0) 1 2 3 4 5 6
(5, 0)

(a) (0, 0) (b) (0, 8) (c) (5, 0) (d) (4, 10)


2. In an LPP, if the objective function Z = ax + by has the same maximum value on two corner
points of the feasible region, then the number of points of which Zmax occurs is
[CBSE 2020 (65/4/1)]
(a) 0 (b) 2 (c) finite (d) infinite

450 Xam idea Mathematics–XII

@Cbsebookshub - Join Us on Telegram


3. Corner points of the feasible region determined by the system of linear constraints are (0, 3),
(1, 1) and (3, 0). Let Z = px + qy, where p, q > 0. Condition on p and q so that the minimum of Z
occurs at (3, 0) and (1, 1) is
q
(a) p = 2q (b) p = (c) p = 3q (d) p = q
2
4. The optimal value of the objective function is attained at the points
(a) given by intersection of inequation with y-axis only.
(b) given by intersection of inequation with x-axis only.
(c) given by corner points of the feasible region.
(d) none of these

Answers
1. (b) 2. (d) 3. (b) 4. (c)

Solutions of Selected Multiple Choice Questions


1. Given objective function Z = 3x – 4y
on putting the corner points, we get
Zmin = – 32 at (0, 8)
3. At (3, 0), Zmin = 3p + q × 0 = 3p
and, at (1, 1), Zmin = p × 1 + q + 1 = p + q
` 3p = p + q
q
⇒ 2p = q ⇒ p =
2

Fill in the Blanks [1 mark]


1. The corner points of the feasible region of an LPP are (0, 0), (0, 8), (2, 7), (5, 4) and (6, 0). The
maximum profit P = 3x + 2y occurs at the point ____________. [CBSE 2020, (65/2/1)]
2. The common region determined by all the linear constraints of a LPP is called the ____________
region.
3. If the feasible region for a LPP is ____________, then the optimal value of the objective function
Z = ax + by may or may not exist.
[NCERT Exemplar]

4. The feasible region for an LPP is always a ____________ polygon.

Answers
1. (5, 4) 2. feasible 3. unbounded 4. convex

Solutions of Selected Fill in the Blanks


1. We have,
P = 3x + 2y
` At point (5, 4)
P = 3 × 5 + 2×4 = 15 + 8 = 23 will be maximum profit.

Linear Programming 451


@Cbsebookshub - Join Us on Telegram
Very Short Answer Questions [1 mark]
1. In figure, which half plane (A) or (B) is the solution of x + y > 1? Justify your answer.
Y

1 B

X' X
O x+y = 1
1

Y'

Sol. Half plane B because (0, 0) does not satisfy x + y > 1.


2. What is the maximum value of objective function Z = 3x + y under given feasible region?

1 ( 23 , 23 (
2
X' X
O 1
x+2y = 2
Y'
Sol. 3, 2x+y = 2

 Z = 3x + y attains maximum value at (1, 0).


∴ Z = 3 × 1 + 0 = 3
3. Is feasible region represented by x + y ≥ 1, x ≥ 0, y ≥ 0 bounded? Justify your answer.
Sol. No, feasible region obtained is unbounded as shown in figure.

X' X
O

Y' x+y = 1

452 Xam idea Mathematics–XII

@Cbsebookshub - Join Us on Telegram


Short Answer Questions-I [2 marks]
1. A small firm manufactures necklaces and bracelets. The total number of necklaces and bracelets
that it can handle per day is at most 24. It takes one hour to make a bracelet and half an hour
to make a necklace. The maximum number of hours available per day is 16. If the profit on a
necklace is `100 and that on a bracelet is `300. Formulate an LPP for finding how many of each
should be produced daily to maximise the profit. It is being given that at least one of each must
be produced. [CBSE Delhi 2017]
Sol. Let x and y be the number of necklaces and bracelets manufactured by small firm per day. If P be
the profit, then objective function is given by
P = 100x + 300y which is to be maximised under the constraints
x + y ≤ 24
1
x + y # 16
2
x ≥ 1, y ≥ 1
2. Two tailors, A and B, earn `300 and `400 per day respectively. A can stitch 6 shirts and 4 pairs
of trousers while B can stitch 10 shirts and 4 pairs of trousers per day. To find how many
days should each of them work and if it is desired to produce at least 60 shirts and 32 pairs of
trousers at a minimum labour cost, formulate this as an LPP. [CBSE (AI) 2017]
Sol. Let A and B work for x and y days respectively.
Let Z be the labour cost.
Z = 300x + 400y
Subject to constraints
6x + 10y ≥ 60
4x + 4y ≥ 32
x, y ≥ 0
3. A company produces two types of goods A and B, that require gold and silver. Each unit of
type A requires 3 g of silver and 1 g of gold while that of type B requires 1 g of silver and 2 g
of gold. The company can produce a maximum of 9 g of silver and 8 g of gold. If each unit of
type A brings a profit of ™40 and that of type B ™50, formulate LPP to maximize profit.
 [CBSE (F) 2017]
Sol. Let x and y be the number of goods A and goods B respectively. If P be the profit then
P = 40x + 50y which is to be maximised under constraints
3x + y ≤ 9
x + 2y ≤ 8
x ≥ 0, y ≥ 0
4. A firm has to transport atleast 1200 packages daily using large vans which carry 200 packages
each and small vans which can take 80 packages each. The cost for engaging each large van is
™400 and each small van is ™200. Not more than ™3,000 is to be spent daily on the job and the
number of large vans cannot exceed the number of small vans. Formulate this problem as a
LPP given that the objective is to minimize cost. [CBSE Delhi (C) 2017]
Sol. Let the number of large vans and small vans be x and y respectively.
Here transportation cost Z be objective function, then

Z = 400x + 200y, which is to be minimized under constraints

200 x + 80y ≥ 1200 ⇒ 5x + 2y ≥ 30
400 x + 200y ≤ 3000 ⇒ 2x + y ≤ 15
x ≤ y, x ≥ 0, y ≥ 0

Linear Programming 453


@Cbsebookshub - Join Us on Telegram
5. Minimise Z = 13x – 15y subject to the
constraints x + y ≤ 7, 2x – 3y + 6 ≥ 0, x ≥ 0 Y
and y ≥ 0.     [NCERT Exemplar]
7 A (0, 7)
Sol. Minimise Z = 13x – 15y ...(i)
6
Subject to the constraints
5
x + y ≤ 7 ...(ii)
4 B (3, 4)
2x – 3y + 6≥ 0 ...(iii)
3
x ≥ 0, y ≥ 0 ...(iv) 2 C (0, 2)
Shaded region shown as OABC is 1
(-3, 0) A (7, 0)
bounded and coordinates of its corner X′ X
O
points are (0, 0), (7, 0), (3, 4) and (0, 2) -3 -2 -1 1 2 3 4 5 6 7 x+y = 7
2x–3y+6 = 0 Y′
respectively.
Corner Points Z = 13x – 15y
O (0, 0) 0
A (7, 0) 91
B (3, 4) –21
C (0, 2) –30 Minimum
Hence, the minimum value of Z is –30 at (0, 2).
6. The feasible region for a LPP is shown in the following figure. Evaluate Z = 4x + y at each of
the corner points of this region. Find the minimum value of Z, if it exists. [NCERT Exemplar]
Y

3 C (0, 3)

2 (0, 2)
x+2
y=
4
B (2, 1)
1

A (4, 0)
X′ x+ X
O 1 2 3 y= 4
3
Y′

Sol. From the fig, it is clear that feasible region is unbounded with the corner points A (4, 0), B (2, 1) and
C (0, 3). [ x + 2y = 4 and x + y = 3 ⇒ y = 1 and x = 2]
Also, we have Z = 4x + y
Corner Points Z = 4x + y
A (4, 0) 16
B (2, 1) 9
C (0, 3) 3 Minimum

454 Xam idea Mathematics–XII

@Cbsebookshub - Join Us on Telegram


Y

C (0, 3)

B (2, 1)

X' X
O (3/4, 0) (4, 0) A
x+2y=4

x+
y=
Y' 4x+y=3

3
Now, we see that 3 is the smallest value of Z at the corner point (0, 3). Note that here we see that,
the region is unbounded, therefore 3 may or may not be the minimum value of Z.
To decide this issue, we graph the inequality 4x + y < 3 and check wether the resulting open half
plane has no point in common with feasible region otherwise, Z has no minimum value.
From the shown graph, it is clear that there is no point common with feasible region and hence,
Z has minimum value 3 at (0, 3).

Short Answer Questions-II [3 marks]


1. Solve the following LPP graphically:
Minimise Z = 5x + 7y
Subject to the constraints
2x + y $ 8
x + 2y $ 10
x, y $ 0 [CBSE (F) 2020, (65/3/1]
Sol. Given constraints are
2x + y $ 8
x + 2y $ 10
and x, y $ 0
For the graph of 2x + y $ 8, we draw the graph of 2x + y = 8
x 0 4
y 8 0

Now, checking for (0, 0) we have 2×0 + 0 $ 8 ⇒ 0$8


` Origin (0, 0) does not satisfy 2x + y $ 8


` Region lies away from origin.
For the graph of x + 2y $ 10, we draw the graph of x + 2y = 10

x 0 10
y 5 0

Linear Programming 455


@Cbsebookshub - Join Us on Telegram
Now, checking for origin (0, 0), we have

0 + 2×0 $ 10 ⇒ 0 $ 10
` Origin (0, 0) does not satisfy x + 2y $ 10


` Region lies away from origin.
Now x, y $ 0, it means region will lie in first quadrant.
On plotting graph of given inequalities (or constraints)
We get the region (shaded) with corner points
A (10, 0), B(2, 4) and C(0, 8).

10

8 C(0, 8)

6
(0, 38)
7
(0, 5)
4 B (2, 4)

5x
x+2
y=
2x

2 +7 10
y=
+y

38
=8

A (10, 0)
X′ X
O (38, 0)
2 4 6 5 8 10
Y′

Now, the value of Z is evaluated at corner points in the following table.


Corner Points Z = 5x + 7y
A (10, 0) 50

B (2, 4) 38 Minimum

C (0, 8) 56

Since, feasible region is ubbounded. Therefore, we have to draw the graph of the inequality.
5x + 7y < 38
Since, the graph of this inequality does not have any point common.
So, the minimum value of Z is 38 at (2, 4).
Hence, Zmin = 38 at (2, 4).
2. Maximise Z = 8x + 9y subject to the constraints given below :
2x + 3y ≤ 6; 3x – 2y ≤ 6; y ≤ 1; x, y ≥ 0 [CBSE (F) 2015]
Sol. Given constraints are
2x + 3y ≤ 6
3x – 2y ≤ 6

456 Xam idea Mathematics–XII

@Cbsebookshub - Join Us on Telegram


y ≤ 1
x, y ≥ 0

For graph of 2x + 3y ≤ 6
We draw the graph of 2x + 3y = 6
x 0 3
y 2 0
2 × 0 + 3 × 0 ≤ 6 ⇒ (0,0) satisfy the constraints.
Hence, feasible region lie towards origin side
of line.

For graph of 3x – 2y ≤ 6
We draw the graph of line 3x – 2y = 6.
x 0 2
y –3 0
3×0–2×0≤6
⇒ Origin (0, 0) satisfy 3x – 2y = 6.

Hence, feasible region lie towards origin side of line.
For graph of y ≤ 1
We draw the graph of line y = 1, which is parallel to x-axis and meet y-axis at 1.
0 ≤ 1 ⇒ feasible region lie towards origin side of y = 1.
Also, x ≥ 0, y ≥ 0 says feasible region is in Ist quadrant.

Therefore, OABCDO is the required feasible region, having corner point O(0, 0), A(0, 1)
3 30 6
B c , 1 m, C c , m , D(2, 0).
2 13 13
Here, feasible region is bounded. Now the value of objective function Z = 8x + 9y is obtained as.
Corner Points Z = 8x + 9y
O (0, 0) 0
A (0, 1) 9

Bd , 1 n
3
21
2

Cd , n
30 6 22.6 Maximum
13 13
D(2, 0) 16

30 6
Z is maximum when x =
and y = .
13 13
3. Minimize and maximize Z = 5x + 2y subject to the following constraints:
x – 2y ≤ 2, 3x + 2y ≤ 12, –3x + 2y ≤ 3, x ≥ 0, y ≥ 0 [CBSE Panchkula 2015]
Sol. Here, objective function is
Z = 5x + 2y ...(i)
Subject to the constraints :
x – 2y ≤ 2 ...(ii)

Linear Programming 457


@Cbsebookshub - Join Us on Telegram
3x + 2y ≤ 12 ...(iii)
–3x + 2y ≤ 3 ...(iv)
x ≥ 0, y ≥ 0 ...(v)
Graph for x – 2y ≤ 2

We draw graph of x – 2y = 2 as
x 0 2
y –1 0
0 – 2 × 0 ≤ 2
[By putting x = y = 0 in the equation]
i.e., (0, 0) satisfy (ii) ⇒ feasible

region lie origin side of line x – 2y = 2.

Graph for 3x + 2y ≤ 12
We draw the graph of 3x + 2y =12.
x 0 4
y 6 0
3 × 0 + 2 × 0 ≤ 12 [By putting x = y
= 0 in the given equation]
i.e., (0, 0) satisfy (iii) ⇒ feasible region lie origin side of line 3x + 2y = 12.


Graph for –3x + 2y ≤ 3
We draw the graph of –3x + 2y ≤ 3
x –1 0
y 0 1.5
–3 × 0 + 2 × 0 ≤ 3 [By putting x = y = 0]
i.e., (0, 0) satisfy (iv) ⇒ feasible region lie origin side of line –3x + 2y = 3.

x ≥ 0, y ≥ 0 ⇒ feasible region is in Ist quadrant.

Now, we get shaded region having corner points O, A, B, C and D as feasible region.
7 3 3 15 3
The co-ordinates of O, A, B, C and D are O(0, 0), A(2, 0), B c , m, C c , m and D c 0, m
2 4 2 4 2
respectively. Now, we evaluate Z at the corner points.
Corner Points Z = 5x + 2y
O (0, 0) 0 Minimum

A (2, 0) 10

Bd , n
7 3
19 Maximum
2 4

Cd , n
3 15
15
2 4

D d 0, n
3
3
2
Hence, Z is minimum at x = 0, y = 0 and minimum value = 0
7 3
also Z is maximum at x = ,y= and maximum value = 19.
2 4

458 Xam idea Mathematics–XII

@Cbsebookshub - Join Us on Telegram


4. A company manufactures two types of novelty souvenirs made of plywood. Souvenirs of type
A require 5 minutes each for cutting and 10 minutes each for assembling. Souvenirs of type
B require 8 minutes each for cutting and 8 minutes each for assembling. There are 3 hours
20 minutes available for cutting and 4 hours for assembling. The profit for type A souvenirs
is `100 each and for type B souvenirs profit is `120 each. How many souvenirs of each type
should the company manufacture in order to maximize the profit? Formulate the problem as
LPP and then solve it graphically. [CBSE 2020 (65/5/1), 2019 (65/5/3)]
Sol. Let x be the number of souvenirs of type A and y be the number of souvenirs of type B.

Souvenirs A(x) Souvenirs B(y) Time


Cutting Machine (min) 5 8 3 × 60 + 20 = 200
Assembling (min) 10 8 4 × 60 = 240
Profits 100 120

Maximize Z = 100x + 120y ...(i)


Subject to 5x + 8y ≤ 200 ...(ii)
10x + 8y ≤ 240 ...(iii)
i.e., 5x + 4y ≤ 120
...(iii)
x, y ≥ 0 ...(iv)
Plotting the constraints

40

30 (0, 30)

(0, 25)
20 (8, 20)

5x
+8
5x

10 y=
+4

20
0
y=
12

(24, 0) (40, 0)
0

X′ X
O 10 20 30 40
Y′

Feasible region is shaded region.


Value of Z = 100 x + 120 y

At (0, 0), Z = 0
At (0, 25) Z = 3000
At (24, 0) Z = 2400
At (8, 20) Z = 3200 Maximum

∴ Maximum profit is `3200 at point (8, 20).


So, 8 types A and 20 types B souvenirs should be made to maximise profit.

Linear Programming 459


@Cbsebookshub - Join Us on Telegram
5. A small firm manufacturers gold rings and chains. The total number of rings and chains
manufactured per day is atmost 24. It takes 1 hour to make a ring and 30 minutes to make a
chain. The maximum number of hours available per day is 16. If the profit on a ring is `300 and
that on a chain is `190, find the number of rings and chains that should be manufactured per
day, so as to earn the maximum profit. Make it as an LPP and solve it graphically.
Sol. Total number of rings and chains manufactured per day = 24
Time taken in manufacturing ring = 1 hour
Time taken in manufacturing chain = 30 minutes
Time available per day = 16 hours
Maximum profit on ring = `300
Maximum profit on chain = `190
Let number of gold rings manufactured per day = x and chains manufactured per day = y
LPP is
Maximize Z = 300x + 190y ...(i)
Subject to constraints x ≥ 0, y ≥ 0 ...(ii)
x + y ≤ 24 ...(iii)
1
x + y # 16 ...(iv)
2
Possible points for maximum Z are A(0, 24), B(8, 16) and C(16, 0).

28

24 A (0, 24)

20

16 B (8, 16)

12
x+ 2
1 =1

x+
y

y=

8
24
6

C(16,0)
X′ X
O 4 8 12 16 20
Y′

Corner Points Z = 300x + 190y


A(0, 24) 4560
B(8, 16) 5440 Maximum
C(16, 0) 4800

460 Xam idea Mathematics–XII

@Cbsebookshub - Join Us on Telegram


Z is maximum at (8, 16).

Hence, 8 gold rings and 16 chains must be manufactured per day.
6. A manufacturing company makes two types of teaching aids A and B of mathematics for
class XII. Each type of A requires 9 labour hours for fabricating and 1 labour hour for finishing.
Each type of B requires 12 labour hours for fabricating and 3 labour hours for finishing. For
fabricating and finishing, the maximum labour hours available are 180 and 30, respectively.
The company makes a profit of `80 on each piece of type A and `120 on each piece of type B.
How many pieces of type A and B should be manufactured per week to get a maximum profit?
What is the maximum profit per week?

Sol. Let x and y be the number of pieces of type A and B manufactured per week respectively. If Z be
the profit then, Y

Objective function, Z = 80x + 120y ...(i)

3x
We have to maximize Z, subject to the

+4
25

y=
constraints

60
9x + 12y ≤ 180 20
x+
⇒ 3x + 4y ≤ 60
...(ii) 3y
= 315
0
x + 3y ≤ 30 ...(iii)
10 C(0,10)
x ≥ 0, y ≥ 0 ...(iv) B(12,6)
5
The graph of constraints are drawn and
A(20,0)
feasible region OABC is obtained, which X′ X
O 10 20 30 40 50 60
is bounded having corner points O (0, 0),
A (20, 0), B (12, 6) and C (0, 10) Y′

Now the value of objective function is obtained at corner points as


Corner points Z = 80x + 120y
O (0, 0) 0
A (20, 0) 1600
B (12, 6) 1680 Maximum
C (0, 10) 1200

Hence, the company will get the maximum profit of `1680 by making 12 pieces of type A and 6
pieces of type B of teaching aid.
7. The standard weight of a special purpose brick is 5 kg and it must contain two basic ingredients
B1 and B2. B1 costs ` 5 per kg and B2 costs ` 8 per kg. Strength considerations dictate that the
brick should contain not more than 4 kg of B1 and minimum 2 kg of B2. Since the demand
for the product is likely to be related to the price of the brick, find the minimum cost of brick
satisfying the above conditions. Formulate this situation as an LPP and solve it graphically.
Sol. Let x kg of B1 and y kg of B2 are taken for making brick.
Here, Z = 5x + 8y is the cost which is objective function and is to be maximised subjected to
following constraints.

Linear Programming 461


@Cbsebookshub - Join Us on Telegram
x + y = 5 … (i)
x ≤ 4 … (ii)
y ≥ 2 … (iii)
x ≥ 0, y ≥ 0 … (iv)
In this case, constraint (i) is a line passing through the feasible region determined by constraints
(ii), (iii) and (iv).

Therefore, maximum or minimum value of objective function ‘Z’ exist on end points of line
(constraint) (i) in feasible region i.e., at A or B.
At A (3, 2) Z = 5 × 3 + 8 × 2 = 15 + 16 = 31
At B (0, 5) Z = 5 × 0 + 8 × 5 = 0 + 40 = 40
Hence, cost of brick is minimum when 3 kg of B1 and 2 kg of B2 are taken.

Long Answer Questions [5 marks]


1. A factory makes tennis rackets and cricket bats. A tennis racket takes 1.5 hours of machine
time and 3 hours of craftman’s time in its making while a cricket bat takes 3 hours of machine
time and 1 hour of craftman’s time. In a day, the factory has the availability of not more than
42 hours of machine time and 24 hours of craftsman’s time. If the profit on a racket and on a bat
is `20 and `10 respectively, find the number of tennis rackets and cricket bats that the factory
must manufacture to earn the maximum profit. Make it as an LPP and solve graphically.
[CBSE Delhi 2011]
Sol. Let the number of tennis rackets and cricket bats manufactured by factory be x and y respectively.
Here, profit on x rackets and y bats is the objective function Z.
Z = 20x + 10y …(i)

462 Xam idea Mathematics–XII

@Cbsebookshub - Join Us on Telegram


We have to maximise Z subject to the constraints:
1.5x + 3y ≤ 42 …(ii) [Constraint for machine hour]
3x + y ≤ 24 …(iii) [Constraint for craft man’s hour]
x, y ≥ 0 …(iv) [Non-negative constraints]
Graph of x = 0 and y = 0 is the y-axis and x-axis respectively.
∴ Graph of x ≥, y ≥ 0 is the Ist quadrant.

Y
Graph of 1.5x + 3y = 42
24

x 0 28

3 x+
20

y= 2
y 14 0

4
16
∴ Graph for 1.5x + 3y ≤ 42 is the part of Ist
B (0, 14)

C(4,12)
quadrant which contains the origin. 12

Graph of 3x + y = 24 8
1.5
x+3
y=
x 0 8 4 42

y 24 0 X′
A (8, 0)
X
O 4 8 12 16 20 24 28
Y′
∴ Graph of 3x + y ≤ 24 is the part of Ist quadrant

in which origin lie.
Hence, shaded area OACB is the feasible region.
For coordinate of C, equation 1.5x + 3y = 42 and 3x + y = 24 are solved as
1.5x + 3y = 42 …(v)
3x + y = 24 …(vi)
2 × (v) – (vi) ⇒ 3x + 6y = 84
– 3x ± y = – 24
5y = 60
⇒ y = 12 ⇒ x=4 (Substituting y = 12 in (vi))
Now, value of objective function Z at each corner of feasible region is
Corner Points Z = 20x + 10y
O (0, 0) 0
A (8, 0) 20 × 8 + 10 × 0 = 160

B (0, 14) 20 × 0 + 10 × 14 = 140

C (4, 12) 20 × 4 + 10 × 12 = 200 Maximum

Therefore, maximum profit is `200, when factory make 4 tennis rackets and 12 cricket bats.

2. A dealer wishes to purchase a number of fans and sewing machines. He has only `5,760 to
invest and has space for at the most 20 items. A fan costs him `360 and a sewing machine `240.
He expects to sell a fan at a profit of `22 and a sewing machine for a profit of `18. Assuming
that he can sell all the items that he buys, how should he invest his money to maximise his
profit ? Solve it graphically. [CBSE (AI) 2007, 2009, Delhi 2014]

Linear Programming 463


@Cbsebookshub - Join Us on Telegram
OR
A dealer in a rural area wishes to purchase some sewing machines. He has only `57,600
to invest and has space for at most 20 items. An electronic machine costs him `3,600 and a
manually operated machine costs `2,400. He can sell an electronic machine at a profit of `220
and a manually operated machine at a profit of `180. Assuming that he can sell all the machines
that he buys, how should he invest his money in order to maximise his profit? Make it as an
LPP and solve it graphically. [CBSE Bhubaneshwar 2015]

Sol. Let the dealer purchases x fans and y sewing machines,then cost of x fans and y sewing machines
is given by 360x + 240y
∴ 360x + 240y ≤ 5, 760 ⇒ 3x + 2y ≤ 48
As, he has space for at most 20 items,
∴ x + y ≤ 20

Now, profit earned by the dealer on selling x fans and y sewing machines = 22x + 18y
Hence, our LPP is to
Maximise Z = 22x + 18y …(i)
Subject to the constraints:
3x + 2y ≤ 48 …(ii)
x + y ≤ 20 …(iii)
x, y ≥ 0 …(iv)

24

20 C(0,20)
x+

3x
y=

16
+2
20

y=
48

12 B (8, 12)

A (16, 0)
X′ X
O 4 8 12 16 20
Y′

464 Xam idea Mathematics–XII

@Cbsebookshub - Join Us on Telegram


Let us evaluate, Z = 22x + 18y at each corner point.
The region satisfying inequalities (ii) to (iv) is shown (shaded) in the figure.
Corner Points Z = 22x + 18y
O (0, 0) 0
A (16, 0) 352
B (8, 12) 392 Maximum
C (0, 20) 360

Thus, maximum value of Z is 392 at B (8, 12).


Hence, the profit is maximum i.e., ` 392 when he buys 8 fans and 12 sewing machines.
OR
Solve yourself as above solution. Here Z = 220x + 180y is objective function.
3. A company produces soft drinks that has a contract which requires that a minimum of 80 units
of the chemical A and 60 units of the chemical B go into each bottle of the drink. The chemicals
are available in prepared mix packets from two different suppliers. Supplier S had a packet of
mix of 4 units of A and 2 units of B that costs `10. The supplier T has a packet of mix of 1 unit
of A and 1 unit of B that costs `4. How many mix packets mix from S and T should the company
purchase to honour the contract requirement and yet minimize cost? Make a LPP and solve
graphically. [CBSE (F) 2012] [HOTS]
Sol. Let x and y units of packet of mixes be Y

purchased from S and T respectively. If Z


is total cost then 80 B(0, 80)

Z = 10x + 4y ...(i)
70
is objective function, which we have to
minimize. 60

Here, constraints are:


50
4x + y ≥ 80 ...(ii)
2x + y ≥ 60 ...(iii) 40 P(10, 40)

Also, x, y ≥ 0 ...(iv)
30
On plotting graph of above constraints
or inequalities (ii), (iii) and (iv), we get 20

shaded region having corner point A, P,


B as feasible region. 10

For coordinate of P. X′
A (30, 0)
X
–1 O 10 20 30 40 50 60 70 80
Point of intersection of
4x+y
10x
2x+ =260

–1
2x + y = 60 ...(v)
+4y
y=
=80

60

and 4x + y = 80 ...(vi) Y′

(v) – (vi)
⇒ 2x + y – 4x – y = 60 – 80

⇒ –2x = –20

⇒ x = 10

⇒ y = 40

coordinate of P ≡ (10, 40)

Linear Programming 465


@Cbsebookshub - Join Us on Telegram
Now the value of Z is evaluated at corner point in the following table

Corner Points Z = 10x + 4y


A (30, 0) 300
P (10, 40) 260 Minimum
B (0, 80) 320

Since, feasible region is unbounded. Therefore we have to draw the graph of the inequality.
10x + 4y < 260 ...(vii)
Since, the graph of inequality (vii) does not have any point common.
So, the minimum value of Z is 260 at (10, 40).
i.e., minimum cost of each bottle is `260 if the company purchases 10 packets of mixes from S and

40 packets of mixes from supplier T.
4. A cooperative society of farmers has 50 hectares of land to grow two crops A and B. The profits
from crops A and B per hectare are estimated as ` 10,500 and ` 9,000 respectively. To control
weeds, a liquid herbicide has to be used for crops A and B at the rate of 20 litres and 10 litres
per hectare, respectively. Further not more than 800 litres of herbicide should be used in order
to protect fish and wildlife using a pond which collects drainage from this land. Keeping in
mind that the protection of fish and other wildlife is more important than earning profit, how
much land should be allocated to each crop so as to maximize the total profit? Form an LPP
from the above and solve it graphically. [CBSE Delhi 2013]
Sol. Let x and y hectares of land be allocated to crop A and B respectively. If Z is the profit then
Z = 10500x + 9000y …(i)
Y

C(0, 50)
50
2x

40
+y
=8

30
0

20 B(30, 20)
x+
y=

10
50

A(40, 0)
X′ X
O 10 20 30 40 50

Y′

We have to maximize Z subject to the constraints:


x + y ≤ 50 …(ii)
20x + 10y ≤ 800 ⇒ 2x + y ≤ 80 …(iii)
x ≥ 0, y ≥ 0 …(iv)

466 Xam idea Mathematics–XII

@Cbsebookshub - Join Us on Telegram


Table for x + y = 50
x 0 50
y 50 0

Table for 2x + y = 80
x 0 40
y 80 0

The graph of system of inequalities (ii) to (iv) are drawn, which gives feasible region OABC with
corner points O (0, 0), A (40, 0), B (30, 20) and C (0, 50).
Feasible region is bounded.
Now,
Corner points Z = 10500x + 9000y
O (0, 0) 0
A (40, 0) 420000
B (30, 20) 495000 Maximum
C (0, 50) 450000

Hence, the co-operative society of farmers will get the maximum profit of ` 495000 by allocating
30 hectares for crop A and 20 hectares for crop B.
5. A manufacturer considers that men and women workers are equally efficient and so he
pays them at the same rate. He has 30 and 17 units of workers (male and female) and capital
respectively, which he uses to produce two types of goods A and B. To produce one unit of A,
2 workers and 3 units of capital are required while 3 workers and 1 unit of capital is required
to produce one unit of B. If A and B are priced at `100 and `120 per unit respectively, how
should he use his resources to maximise the total revenue? Form the above as an LPP and solve
graphically. [CBSE (AI) 2013]
Y
Sol. Let x and y units of goods A and B are
produced respectively. 14

Let Z be total revenue 13


3x +

12
Here, Z = 100x + 120y ....(i)
11
y=1

A(
Subjects to constraints: 10 0,1
0)
7

Also 2x + 3y ≤ 30 ....(ii) 9
C(3, 8)
3x + y ≤ 17 ....(iii) 8

x, y ≥ 0 ....(iv) 7
2x
6 +3
On plotting graph of above inequalities (ii), y=
5 30
(iii) and (iv). We get shaded region as feasible 4
region having corner points A, O, B and C. 3

For coordinate of 'C' 2


1 17 , 0)
Two equations (ii) and (iii) are solved and we B( 3
X′ X
get coordinate of C = (3, 8) O 1 2 3 4 5 6 7 8 9 10 11

–1

Y′

Linear Programming 467


@Cbsebookshub - Join Us on Telegram
Now, the value of Z is evaluated at corner points as:
Corner points Z = 100x + 120y
O (0, 0) 0
A (0, 10) 1200

B c 3 , 0m
17 1700
3
C (3, 8) 1260 Maximum

Therefore, maximum revenue is `1, 260 when 3 workers and 8 units capital are used for production.
6. An aeroplane can carry a maximum of 200 passengers. A profit of `500 is made on each
executive class ticket out of which 20% will go to the welfare fund of the employees. Similarly
a profit of `400 is made on each economy ticket out of which 25% will go for the improvement
of facilities provided to economy class passengers. In both cases, the remaining profit goes to
the airline’s fund. The airline reserves at least 20 seats for executive class. However, at least
four times as many passengers prefer to travel by economy class than by the executive class.
Determine, how many tickets of each type must be sold in order to maximise the net profit of
the airline. Make the above as an LPP and solve graphically. [CBSE (F) 2013]
Sol. Let there be x tickets of executive class and y tickets of economy class be sold. Let Z be net profit
of the airline.
Here, we have to maximise Z
80 75
Now, Z = 500x × + 400y ×
100 100
Z = 400x + 300y ...(i)
Subject to constraints:
x ≥ 20 ...(ii)
Also x + y ≤ 200 ...(iii)
x + 4x ≤ 200 [ y = 4x]
⇒ 5x ≤ 200

⇒ x ≤ 40 ...(iv)

x = 40
x = 20

E(0, 200)
D(20, 180)

x+
y=
20
0

A(20, 0)
X′

Y′

468 Xam idea Mathematics–XII

@Cbsebookshub - Join Us on Telegram


Shaded region is feasible region having corner points A(20, 0), B(40, 0), C(40, 160), D(20, 180).
Now, value of Z is calculated at corner point as
Corner points Z = 400x + 300y
A (20, 0) 8000
B (40, 0) 16000
C (40, 160) 64000 Maximum
E (0, 200) 60000

Hence, 40 tickets of executive class and 160 tickets of economy class should be sold to maximise
the net profit of the airlines.
7. A factory owner purchases two types of machines, A and B for his factory. The requirements
and the limitations for the machines are as follows :
Machine Area occupied Labour force Daily output (in units)
2
A 1000 m 12 men 60
2
B 1200 m 8 men 40
He has maximum area of 9000 m2 available, and 72 skilled labourers who can operate both the
machines. How many machines of each type should he buy to maximise daily output?
Sol. Let the owner buys x machines of type A and y machine of type B.
Then 1000x + 1200y ≤ 9000 ...(i)
12x + 8y ≤ 72 ...(ii)
Objective function is to maximize Z = 60x + 40y
From (i)
10x + 12y ≤ 90
or 5x + 6y ≤ 45 ...(iii)
3x + 2y ≤ 18 ...(iv) [from (ii)]
We plot the graph of inequations shaded region in the feasible solution (iii) and (iv).
The shaded region in the figure represents the feasible region which is bounded. Let us now
evaluate Z at each corner point.

10
(0, 9)

8
3x
+2
y=
18

5x
+
2 6y
=
45

X′ X
O 2 4 6 8 10
Y′

Linear Programming 469


@Cbsebookshub - Join Us on Telegram
Z at (0, 0) is 60 × 0 + 40 × 0 = 0
15
Z at c 0, m is 60 × 0 + 40 ×
15
= 300
2 2
9 45 9 45
Z at d , n is 60 × + 40 × =135 + 225 = 360 Maximum
4 8 4 8
Z at (6, 0) is 60 × 6 + 0 = 360 Maximum
9 45
Therefore, either x = 6, y = 0 or x = , y = but second case is not possible as x and y are whole
4 8
numbers, because number of machines cannot be fraction.
Hence, there must be 6 machines of type A and no machine of type B is required for maximum
daily output.
8. A manufacturer produces two models of bikes-model X and model Y. Model X takes a 6
man-hours to make per unit, while model Y takes 10 man hours per unit. There is a total of
450 man-hour available per week. Handling and marketing costs are `2000 and `1000 per unit
for models X and Y, respectively. The total funds available for these purposes are `80000 per
week. Profits per unit for models X and Y are `1000 and `500, respectively. How many bikes
of each model should the manufacturer produce, so as to yield a maximum profit? Find the
maximum profit. [NCERT Exemplar]
Sol. Let the manufacturer produces x number of model X and y number of model Y bikes.
Model X takes a 6 man-hours to make per unit and model Y takes a 10 man-hours to make per unit.
There is total of 450 man-hour available per week.
∴ 6x + 10y ≤ 450


3x + 5y ≤ 225 ...(i)
For models X and Y, handling and marketing costs `2000 and `1000, respectively, total funds
available for these purposes are `80000 per week.
∴ 2000x + 1000y ≤ 80000

⇒ 2x + y ≤ 80
...(ii)
Also, x ≥ 0, y ≥ 0
Here, the profits per unit for models X and Y are `1000 and `500, respectively.
∴ Required LPP is

Maximise Z = 1000x + 500y
Subject to, 3x + 5y ≤ 225
2x + y ≤ 80
x ≥ 0, y ≥ 0
From the shaded feasible region, it is clear that coordinates of corner points are (0, 0), (40, 0),
(25, 30) and (0, 45).
On Solving 3x + 5y = 225 and 2x + y = 80, we get x = 25, y = 30

Corner Points Z = 1000x + 500y


(0, 0) 0
(40, 0) 40000 Maximum
(25, 30) 25000 +15000 = 40000 Maximum
(0, 45) 22500

470 Xam idea Mathematics–XII

@Cbsebookshub - Join Us on Telegram


Y

80 (0, 80)

70

60

50
(0, 45)

40

30 (25, 30)

20 3x
+5
y=
2x

22
+y

5
=8

10
0

(40, 0) (75, 0)
X′ X
O 10 20 30 40 50 60 70 80
Y′

So, the manufacturer should produce 25 bikes of model X and 30 bikes of model Y to get a
maximum profit of `40000.
Since, in question it is asked that each model bikes should be produced, so the value (40, 0) is
ignored.
9. A manufacturer has employed 5 skilled men and 10 semi-skilled men and makes two models
A and B of an article. The making of one item of model A requires 2 hours work by a skilled
man and 2 hours work by a semi-skilled man. One item of model B requires 1 hour by a skilled
man and 3 hours by a semi-skilled man. No man is expected to work more than 8 hours per
day. The manufacturer’s profit on an item of model A is `15 and on an item of model B is ` 10.
How many of items of each model should be made per day in order to maximize daily profit?
Formulate the above LPP and solve it graphically and find the maximum profit.
[CBSE Delhi, 2019]
Sol. Let x items of model A and y items of model B be made.

x, y  0 (number of items can not be negative)
According to question, we have
The making of model A requires 2 hours work by a skilled man and the model B requires 1 hour
by a skilled man.

2x + y  40
and, the making of model A requires 2 hours work by a semi skilled man and model B requires
3 hours work by a semi-skilled man.
∴ 2x + 3y  80

Linear Programming 471


@Cbsebookshub - Join Us on Telegram
and, the profit, Z = 15x + 10y, which is to be maximised.
Thus, we have the mathematical formulation of the given linear programming problem as
ZMax. = 15x + 10y
Subject to constraints
2x + y # 40 ... (i)
2x + 3y # 80 ...(ii)
x, y $ 0 ...(iii)
The feasible region determined by the system of constraints is OABC.

2x
+3
2x

y=
+y

80
=4
0

80
The corner points are A d 0, n, B ^10, 20 h, C (20, 0) .
3
Corner points Z = 15x + 10y

80 800
A d 0, n
3 3
B (10, 20) 350 Maximum

C (20, 0) 300

The maximum value of Z = 350 which is attained at B (10, 20).


Hence, the maximum profit is ™350 when 10 units of model A and 20 units of model B are
produced.

472 Xam idea Mathematics–XII

@Cbsebookshub - Join Us on Telegram


PROFICIENCY EXERCISE
QQ Objective Type Questions: [1 mark each]
1. Choose and write the correct option in the following questions.
(i) Feasible region (shaded) for a LPP is shown in the given figure. Minimum of z = 4x + 3y
occurs at the point.

10

8 D (0, 8)

6
C (2, 5)

4
B (4, 3)

A (9, 0)
X′ X
O 2 4 6 8 10
Y′

(a) (0, 8) (b) (2, 5) (c) (4, 3) (d) (9, 0)


(ii) The solution set of the inequation 3x + 2y > 3 is
(a) half plane not containing the origin (b) half plane containing the origin
(c) the point being on the line 3x + 2y = 3 (d) None of these
(iii) If the constraints in a linear programming problem are changed
(a) solution is not defined (b) the objective function has to be modified
(c) the problems is to be re-evaluated (d) none of these
(iv) Which of the following statement is correct?
(a) Every LPP admits an optimal solution.
(b) Every LPP admits unique optimal solution.
(c) If a LPP gives two optimal solutions it has infinite number of solutions.
(d) None of these
(v) The maximum value of p = x + 3y such that 2x + y # 20 , x + 2y # 20, x $ 0, y $ 0 is
80
(a) 10 (b) 30 (c) 60 (d)
3
2. Fill in the blanks.
(i) In a LPP, the linear function which has to be maximised or minimised is called a linear
____________ function.
(ii) The maximum value of Z = 6x + 16y satisfying the conditions x + y $ 2 , x $ 0, y $ 0 is
____________.
(iii) In a LPP, the inequalities or restrictions on the variables are called ____________.

Linear Programming 473


@Cbsebookshub - Join Us on Telegram
QQ Very Short Answer Questions: [1 mark each]
3. Determine the maximum value of Z = 3x + 4y,
Subject to the constraints: x + y # 1, x $ 0, y $ 0.
4. If a linear programming problem is Zmax = 3x + 2y,
Subject to the constraints: x + y # 2, find Zmax
5. Maximise the function Z = 11x + 7y, subject to the constraints: x # 3, y # 2, x $ 0, y $ 0.

QQ Short Answer Questions–I: [2 marks each]


6. A manufacturing company makes two models A and B of a product. Each piece of model A
requires 9 labour hours for fabricating and 1 labour hour for finishing. Each piece of model B
requires 12 labour hours for fabricating and 3 labour hours for finishing. For fabricating and
finishing the maximum labour hours available are 180 and 30 respectively. The company makes
a profit of `8,000 on each piece of model A and `12,000 on each piece of model B. To realise a
maximum profit formulate above problem in LPP.
7. One kind of cake requires 200 g of flour and 25 g of fat, and another kind of cake requires 100 g
of flour and 50 g of fat. To get the maximum number of cakes can be made from 5 kg of flour and
1 kg of fat, formulate the problem in LPP.
8. A manufacturer produces nuts and bolts. It takes 1 hour of work on machine A and 3 hours on
machine B to produce a package of nuts. It takes 3 hours on machine A and 1 hour on machine B to
produce a package of bolts. He earns a profit of `17.50 per package on nuts and `7.00 per package
on bolts. If he operates his machines for at most 12 hours a day, the formulate the problems in
LPP to maximise his profit.
9. A merchant plans to sell two types of personal computers a desktop model and a portable model
that will cost `25,000 and `40,000 respectively. He estimates that the total monthly demand of
computers will not exceed 250 units. If he does not want to invest more than `70 lakhs and if his
profit on the desktop model is `4500 and on portable mode is `5000, then formulate the problems
as LPP to get maximum profit.
10. A manufacturing company makes two toys A and B. Each piece of toy A requires 8 labour hours for
fabricating and 2 labour hours for finishing. Each piece of toy B requires 16 labour hours for fabricating
and 4 labour hours for finishing. For fabricating and finishing, the maximum labour hours available
are 200 and 50 respectively. The company makes a profit of `10,000 on each piece of toy A and `14,000
on each piece of toy B. Formulate the problem as LPP to realise a maximum profit.

QQ Short Answer Questions–II: [3 marks each]


11. A cottage manufactures pedestal lamps and wooden shades. Both the products require machine
time as well as craftsman time in the making. The number of hour(s) required for producing 1
unit of each and the corresponding profit is given in the following table:
Item Machine Time Craftsman time Profit (in `)
Pedestal lamp 1.5 3 30
Wooden shades 3 hours 1 hours 20
In day, the factory has availability of not more than 42 hours of machine time and 24 hours of
craftsman time.
Assuming that all items manufactured are sold, how should the manufacturer schedule his daily
production in order to maximise the profit? Formulate it as an LPP and solve it graphically.
[CBSE 2020, (65/2/1)]

474 Xam idea Mathematics–XII

@Cbsebookshub - Join Us on Telegram


12. A manufacturer has three machines I, II and III installed in his factory. Machine I and II are
capable of being operated for atmost 12 hours whereas machine III must be operated for atleast
5 hours a day. He produces only two items M and N each requiring the use of all the three
machines.
The number of hours required for producing 1 unit of M and N on three machines are given in
the following table:
Item Number of hours required on machines
I II III
­
M 1 2 1
N 2 1 1.25
He makes a profit of `600 and `400 on one unit of items M and N respectively. How many units
of each item should he produce so as to maximise his profit assuming that he can sell all the items
that he produced. What will be the maximum profit? [CBSE 2020, (65/4/1)]
13. A man rides his motorcycle at the speed of 50 km/hour. He has to spend `2 per km on petrol. If
he rides it at a faster speed of 80 km/hour, the petrol cost increases to `3 per km. He has at the
most `120 to spend on petrol and one hour time. He wishes to find the maximum distance that he
can travel. Express this problem as a linear programming problem.
14. Determine the maximum value of Z = 3x + 4y of the feasible region (shaded) for a LPP is shown
in figure.
Y

X x X
O A 2x + + 2y
y =7
Y = 6
10
4

15. In figure, the feasible region (shaded) for a LPP is shown. Determine the maximum and minimum
value of Z = x + 2y.
Y

3 15
Q 2, 4
3 , 24
13 13 P

7 3
R 2,4
18 , 2
7 7 S
X X
O

Linear Programming 475


@Cbsebookshub - Join Us on Telegram
16. Solve the following linear programming problem graphically:
Minimise Z = x – 5y + 20
Subject to constraints: x – y $ 0, – x + 2y $ 2;
x $ 3, y # 4, x, y $ 0
17. Solve the following LPP:
Maximise Z = 5x1 + 7x2,
Subject to constraints: x1 + x2 # 4,
3x1 + 8x2 # 24,
10x1 + 7x2 # 35,
x1, x2 $ 0 .
18. Maximise Z = x + y subject to x + 4y # 8, 2x + 3y # 12, 3x + y # 9, x $ 0, y $ 0.
19. Solve the following LPP graphically:
Maximise Z = 1000x + 600y
Subject to the constraints
x + y # 200
x $ 20
y – 4x $ 0
x, y $ 0 [CBSE (F) 2017]
20. Solve the following LPP graphically:
Maximise Z = 4x + y
Subject to following constraints x + y # 50 ,
3x + y # 90 ,
x $ 10
x, y $ 0 [CBSE Delhi 2017]
21. Solve the following linear programming problem graphically:
Maximise Z = 7x + 10y
Subject to constraints
4x + 6y # 240
6x + 3y # 240
x $ 10
x $ 0, y $ 0 [CBSE (AI) 2017]
22. Fund graphically, the maximum value of Z = 2x + 5y, subject to constraints given below:
2x + 4y # 8,
3x + y # 6,
x + y # 4,
x $ 0, y $ 0 [CBSE Delhi 2015]
23. Solve the following linear programming problem graphically.
Minimise Z = 3x + 5y
Subject to the constraints:
x + 2y $ 10; x + y $ 6; 3x + y $ 8; x, y $ 0 [CBSE Ajmer 2015]

476 Xam idea Mathematics–XII

@Cbsebookshub - Join Us on Telegram


QQ Long Answer Questions: [5 marks each]
24. (Diet problem) A dietician has to develop a special diet using two foods P and Q. Each packet
(containing 30 g) of food P contains 12 units of calcium, 4 units of iron, 6 units of cholesterol and
6 units of vitamin A. Each packet of the same quantity of food Q contain 3 units of calcium, 20
units of iron, 4 units of cholesterol and 3 units of vitamin A. The diet requires at least 240 units
of calcium, at least 460 units of iron and at most 300 units of cholesterol. How many packets of
each food should be used to minimise the amount of vitamin A in the diet? What is the minimum
amount of vitamin A? [CBSE Chennai 2015]
25. One kind of cake requires 300 g of flour and 15 g of fat, another kind of cake requires 150 g of flour
and 30 g of fat. Find the maximum number of cakes which can be made from 7.5 kg of flour and
600 g of fat, assuming that there is no shortage of the other ingredients used in making the cakes.
Make it as an LPP and solve it graphically? [CBSE (AI) 2010]
26. In order to supplement daily diet, a person wishes to take some X and some Y tablets. The contents
of iron, calcium and vitamins in X and Y (in miligrams per tablet) are given below:
Tablets Iron Calcium Vitamins
X 6 3 2
Y 2 3 4
The person needs at least 18 milligrams of iron, 21 milligrams of calcium and 16 milligrams of
vitamins. The price of each tablet of X and Y is `2 and `1 respectively. How many tablets of each
should the person take in order to satisfy the above requirement at the minimum cost?
[CBSE (F) 2016]
27. A company manufactures three kinds of calculators: A, B and C in its two factories I and II. The
company has got an order for manufacturing at least 6400 calculators of kind A, 4000 of kind B
and 4,800 of kind C. The daily output of factory I is of 50 calculators of kind A, 50 calculators of
kind B, and 30 calculators of kind C. The daily output of factory II is of 40 calculators of Kind A,
20 of kind B and 40 of kind C. The cost per day to run factory I is `12,000 and factory II is `15,000.
How many days do the two factories have to be in operation to produce the order with the
minimum cost? Formulate this problem as an LPP and solve it graphically.
[CBSE Allahabad 2015]
28. A dealer deals in two items only – item A and item B. He has `50,000 to invest and a space to store
at most 60 items. An item A costs `2,500 and an item B costs `500. A net profit to him on item A is
`500 and on item B `150. If he can sell all the items that he purchases, how should he invest his
amount to have maximum profit? Formulate an LPP and solve it graphically.
[CBSE Chennai 2015]
29. The postmaster of a local post office wishes to hire extra helpers during the Deepawali season,
because of a large increase in the volume of mail handling and delivery. Because of the limited
office space and the budgetary conditions, the number of temporary helpers must not exceed 10.
According to past experience, a man can handle 300 letters and 80 packages per day, on the average,
and a woman can handle 400 letters and 50 packets per day. The postmaster believes that the daily
volume of extra mail and packages will be no less than 3400 and 680 respectively. A man receives
`225 a day and a woman receives `200 a day. How many men and women helpers should be hired
to keep the pay-roll at a minimum? Formulate an LPP and solve it graphically. [CBSE Patna 2015]
30. A retired person wants to invest an amount of `50,000. His broker recommends investing in two
type of bonds ‘A’ and ‘B’ yielding 10% and 9% return respectively on the invested amount. He
decides to invest at least 20,000 in bond ‘A’ and at least 10,000 in bond ‘B’. He also wants to invest
at least as much in bond ‘A’ as in bond ‘B’. Solve this linear programming problem graphically to
maximise his returns. [CBSE (North) 2016]

Linear Programming 477


@Cbsebookshub - Join Us on Telegram
31. A company manufactures two types of cardigans: type A and type B. It costs `360 to make a type
A cardigan and `120 to make a type B cardigan. The company can make at most 300 cardigans
and spend at most `72000 a day. The number of cardigans of type B cannot exceed the number of
cardigans of type A by more than 200. The company makes a profit of `100 for each cardigan of
type A and `50 for every cardigan of type B.
Formulate this problem as a linear programming problem to maximise the profit of the company.
Solve it graphically and find maximum profit. [CBSE (East) 2016]
32. There are two types of fertilisers ‘A’ and ‘B’. ‘A’ consists of 12% nitrogen and 5% phosphoric acid
whereas ‘B’ consists of 4% nitrogen and 5% phosphoric acid. After testing the soil conditions, farmer
finds that he needs at least 12 kg of nitrogen and 12 kg of phosphoric acid for his crops. If ‘A’ costs
`10 per kg and ‘B’ costs `8 per kg, then graphically determine how much of each type of fertiliser
should be used so that nutrient requirements are met at a minimum cost. [CBSE (Central) 2016]
33. A manufacturer produces two products A and B. Both the products are processed on two
different machines. The available capacity of first machine is 12 hours and that of second machine
is 9 hours per day. Each unit of product A requires 3 hours on both machines and each unit of
product B requires 2 hours on first machine and 1 hour on second machine. Each unit of product
A is sold at `7 profit and that of B at a profit of `4. Find the production level per day for maximum
profit graphically. [CBSE Delhi 2016]
34. A manufacturer produces nuts and bolts. It takes 1 hour of work on machine A and 3 hours on
machine B to produce a package of nuts. It takes 3 hours on machine A and 1 hour on machine B
to produce a package of bolts. He earns a profit of `17.50 per package on nuts and `7 per package
on bolts. How many packages of each should be produced each day so as to maximize his profits
if he operates his machines for at the most 12 hours a day? Form the linear programming problem
and solve it graphically. [CBSE Delhi 2012]
35. A dietician wishes to mix two types of foods in such a way that the vitamin contents of the
mixture contains at least 8 units of vitamin A and 10 units of vitamin C. Food I contains
2 units/kg of vitamin A and 1 unit/kg of vitamin C while Food II contains 1 units/kg of
vitamin A and 2 units/kg of vitamin C. It costs `5 per kg to purchase Food I and `7 per kg to
purchase Food II. Determine the minimum cost of such a mixture. Formulate the above as a
LPP and solve it graphically. [CBSE (AI) 2012]

Answers
1. (i) (b) (ii) (a) (iii) (c) (iv) (c) (v) (b)
2. (i) objective (ii) 12 (iii) linear constraints
3. 4 4. 6 5. 47
6. Z = 8000x + 12000y is to be maximised under constraints
9x + 12y # 180; x + 3y # 30; x $ 0, y $ 0
7. Z = x + y is to be maximised under the constraints
2x + y # 50
x + 2y # 40

x, y $ 0

where x and y are number of first and second kind of cake and Z the total number of cakes.
8. Z = 17.5x + 7y w hich is to be maximised under constraints
x + 3y # 12; 3x + y # 12; x, y $ 0
where x nuts and y bolts are produced and Z is the profit.

478 Xam idea Mathematics–XII

@Cbsebookshub - Join Us on Telegram


9. Z = 4500 x + 5000y which is to be maximised under constraints
x + y # 250; 5x + 8y # 1400; x, y $ 0
10. Z = 10,000x + 14,000y which is to be maximised under constraints
8x + 16y # 200; 2x + 4y # 50; x $ 0, y $ 0
where Z is total profit and x, y are the number of toy A and toy B.
11. The manufacturer should produce 4 pedestal lamps and 12 wooden shades to get maximum profit
of 360.
12. Maximum profit is `4000 whan a manufacturer produced 4 units of items M and 4 units of item of N.
13. Maximise Z = x + y. Subject to constraints: 2x + 3y # 120, 8x + 5y # 400, x $ 0, y $ 0 .
1
14. 196 15. Maximum = 9, minimum = 3 16. At (4, 4), Zmin = 4
7
at c , m
8 12 124 43 28 15
17. x1 = , x2 = ; Zmax = 18. 19. Zmax = 136000 at x = 40 and y = 160
5 5 5 11 11 11
20. Zmax = 120 when x = 30, y = 0 21. Zmax = 410 for x = 30, y = 20

22. Maximum value of Z is 10 at x = 0, y = 2 23. Minimum value of Z is 26 at (2, 4)


24. 15 packets of food P and 20 packets of food Q, minimum amount of vitamin A is 150 units.
25. 20 cakes of type 1 and 10 cakes of type II to get maximum number of cakes
26. x = 1, y = 6 minimum value B
27. Factory I run for 80 days and
Factory II run for 60 days to get minimum cost 2184000
Hint: Objective function Z = 12000x + 15000y
Subject to constraints:
5x + 4y ≥ 640 5x + 2y ≥ 400 3x + 4y ≥ 480 x, y ≥ 0
28. Dealer deals in 10 items of A and 50 items of B to get maximum profit `12500
Hint: Objective function: Z = 500x + 150y
Subject to constraints:
x + y ≤ 60 5x + y ≤ 100 x, y ≥ 0
29. Minimum payroll 6 men and 4 women must be hired minimum cost `2150
Hint: Objective function, Z = 225x + 200y

Subject to constraints: x + y ≤ 10 3x + 4y ≥ 34 8x + 5y ≥ 68 x, y ≥ 0
30. `40000 in bond A and `10,000 in bond B for a maximum return of `4900.
31. 150 cardigans of type A and 150 of type B for a maximum profit of 22,500.
32. Minimum cost `1980, 30 kg of fertiliser A and 210 kg of fertiliser B should be used.
33. Manufacturer will get maximum profit of `26 by producing 2 units of A and 3 units of B.
34. Maximum profit is `73.5, when 3 package of nuts and 3 package of bolts are produced.
35. Minimum cost of food mixture is `38, when 2kg of Food I and 4 kg of food II are mixed.

SELF-ASSESSMENT TEST
Time allowed: 1 hour Max. marks: 30
1. Choose and write the correct option in the following questions. (2 × 1 = 2)
(i) The maximum value of Z = 4x + 3y subjected to the constraints 2x + 3y ≤ 18, x + y ≥ 10;
x, y ≥ 0 is
(a) 36 (b) 40 (c) 20 (d) none of these

Linear Programming 479


@Cbsebookshub - Join Us on Telegram
(ii) The objective function Z = 4x + 3y can be maximised subjected to the constraints 3x + 4y ≤ 24,
8x + 6y ≤ 48, x ≤ 5, y ≤ 6; x, y ≥ 0
(a) at only one point (b) at two points only
(c) at an infinite number of points (d) none of these
2. Fill in the blanks. (2 × 1 = 2)
(i) A corner point of a feasible region is a point in the region which is the ____________ of two
boundary lines.
(ii) In a LPP, the objective function is always ____________.
QQ Solve the following questions. (1 × 1 = 1)
3. Solve the linear inequation –3x +2y ≥ 6 graphically.
QQ Solve the following questions. (5 × 5 = 25)
4. A diet is to contain at least 80 units of Vitamin A and 100 units of minerals. Two foods F1 and
F2 are available. Food F1 costs `4 per unit and F2 costs `6 per unit. One unit of food F1 contains
3 units of Vitamin A and 4 units of minerals. One unit of food F2 contains 6 units of Vitamin A
and 3 units of minerals. Formulate this as a linear programming problem and find graphically
the minimum cost for diet that consists of mixture of these two foods and also meets the minimal
nutritional requirements. [CBSE Delhi 2009]
5. A retired person has `70,000 to invest and two types of bonds are available in the market for
investment. First type of bonds yields an annual income of 8% on the amount invested and
the second type of bonds yields 10% per annum. As per norms, he has to invest a minimum of
`10,000 in the first type and not more than `30,000 in the second type. How should he plan his
investment, so as to get maximum return, after one year of investment?
6. To maintain his health, a person must fulfil certain minimum daily requirements of several kinds
of nutrients. Assuming that there are only three kinds of nutrients – calcium, protein and calories
and the person’s diet consists of only two food items, I and II, whose price and nutrient contents
are shown below:
Food I (per lb) Food II (per lb) Minimum daily
requirement of the nutrient
Calcium 10 4 20
Protein 5 5 20
Calories 2 6 13
Price (`) 60 100

What combination of two food items will satisfy the daily requirement and entail the least cost ?
Formulate this as an LPP.
7. One kind of cake required 200 g of flour and 25 g of fat, and another kind of cake requires
100 g of flour and 50 g of fat. Find the maximum number of cakes which can be made from 5 kg of
flour and 1 kg of fat, assuming that there is no shortage of other ingradients used in making the
cakes. Formulate the above as a linear programming problem and solve graphically.
8. A company sells two different products A and B. The two products are produced in a common
production process which has a total capacity of 500 man hours. It takes 5 hours to produce a unit of
A and 3 hours to produce a unit of B. The demand in the market shows that the maximum number
of units of A that can be sold is 70 and that for B is 125. Profit on each unit of A is `20 and that on B
is `15. How many units of A and B should be produced to maximize the profit? Solve it graphically.

480 Xam idea Mathematics–XII

@Cbsebookshub - Join Us on Telegram


Answers
1. (i) (d) (ii) (c)
2. (i) intersection (ii) linear
3. Y

(0, 3)

X' X
(–2, 0) O

Y'

4
4. For minimum cost `104, 24 units of food F1 and units of food F2 is required.
3
5. `40,000 in I Type and `30,000 in II Type maximum return of `6200.
6. Minimise Z = 0.60x + 0.100y
Subject to constraints.
5x + 2y ≥ 10 2x + 6y ≥ 13
x + y ≥ 4 x, y ≥0
11 5
ZMin = ` 290 at c , m

4 4
7. Maximise
Z=x+y
Subject to constraints
2x + y ≤ 50
x + 2y ≤ 40
x, y ≥ 0
Maximum number of cakes = 30

20 cakes of I kind and 10 cakes of II kind should be made to get maximum number of cakes.
8. 25 of type A and 125 of type B to get maximum profit of `2375
zzz

Linear Programming 481


@Cbsebookshub - Join Us on Telegram
14 Probability

1. Probability: Probability is a branch of mathematics in which the chance of an event happening is


assigned a numerical value that predicts how likely that event is to occur.
Random Experiment: The experiment, in which the outcomes may not be same even if the
experiment is performed in identical condition, is called random experiment. e.g., Tossing a coin is
a random experiment because if we toss a coin in identical condition, outcomes may be head or tail.
2. Outcome: An outcome is a result of some activity or experiment.
3. Sample Space: A sample space is a set of all possible outcomes for a random experiment.
4. Event: An event is a subset of the sample space.
5. Theoretical Probability: The theoretical probability of an event is the number of ways that the
event can occur, divided by the total number of possibilities in the sample space.
n (E)
Symbolically, we write P (E) = , where P(E) represents the probability of the event.
n ( S)
6. In general, for any sample space S containing k possible outcomes, we say n(S) = k. When the event
E is certain, every possible outcome for the sample space is also an outcome for event E or n(E) = k.
Thus, the probability of a certain or sure event is given as
n (E) k
P (E) = = =1
n ( S) k

Note: (i) The probability of an event that is certain to occur is 1.
 he probability of any event E must be equal to or greater than 0; and less than or equal
(ii) T
to 1, i.e., 0 ≤ P(E) ≤ 1.
7. Another way of expressing probability is in term of axioms, laid by Russian mathematician A. N.
Kolmogorov.
If S is a sample space, then probability P is a real valued function defined on S and take values
[0 , 1], satisfying following axiom
(i) Probability of any event ≥ 0.
(ii) Sum of probabilities assigned to all members of S is 1.
(iii) For any two mutually exclusive events E and F, P(E ∪ F) = P(E) + P(F).

482 Xam idea Mathematics–XII

@Cbsebookshub - Join Us on Telegram


8. Theorems of Probability:
(i) Addition theorem:
(a) When the events are not mutually exclusive: The probability that at least one of the two
events A and B which are not mutually exclusive will occur is given
Symbolically, P(A ∪ B) = P(A) + P(B) – P(A ∩ B)
In the case of three events:
P(A ∪ B ∪ C) = P(A) + P(B) + P(C) – P(A ∩ B) – P(B ∩ C) – P(C ∩ A) + P(A ∩ B ∩ C)
(b) When A and B are mutually exclusive: The addition theorem states that if two events
A and B are mutually exclusive, the probability of the occurrence of either A or B is the sum
of the individual probability of A and B
Symbolically, P(A ∪ B) = P(A) + P(B)
The theorem can be extended to three or more mutually exclusive events
thus, P(A ∪ B ∪ C) = P(A) + P(B) + P(C)
(ii) Multiplication theorem: This theorem states that if two events A and B are independent, the
probability that they both will occur is equal to the product of their individual probabilities.
Symbolically, P(A ∩ B) = P(A) . P(B)
The theorem can be extended to three or more independent events
thus, P(A ∩ B ∩ C) = P(A) . P(B) . P(C)
Note: If A and B are mutually exclusive and exhaustive, then
P(A ∪ B) = P(A) + P(B) = 1
9. A rule for the probability of the event not A: If P(A) is the probability that some given event will
occur, and P (not A) is the probability that the given event will not occur, then
Symbolically: P(A) + P(not A) = 1 or P(A) = 1 – P(not A) or P(not A) = 1 – P(A)

We write P (not A) as P(A).
10. Problems, related to withdrawal of balls, cards, letters, etc. with replacement and without
replacement:
In such type of problems, the sample space will not change when the articles (balls, cards, letters, etc.) are

replaced after each withdrawal. While in case when the article is not replaced (without replacement), the
sample space will change after each withdrawal.

Note:
(i) If the problem does not specifically mention “with replacement” or “without replacement”, ask
yourself: ‘’Is this problem with or without replacement?’’
(ii) For many compound events, the probability can be determined most easily by using the
counting principle i.e., permutations and combinations.
(iii) Every probability problem can always be solved by
OO Counting the number of elements in the sample space n(S);

OO Counting the number of outcomes in the events, n(E);

OO And substituting these numbers in the probability formula.

n (E)
P (E) =
n ( S)
(iv) Taking out 2 or more objects (e.g. balls) randomly from a bag one by one without replacement
is same as taking out 2 or more objects simultaneously.
n!
The number of ways in which r objects can be taken out of n objects is nCr or C (n, r) = .
(n – r ) ! . r !

Probability 483
@Cbsebookshub - Join Us on Telegram
11. Conditional Probability: If A and B are two events associated with the same random experiment,
then the probability of occurrence of event A, when the event B has already occurred is called
conditional probability of A when B is given. It is represented by P (A/B) and is given by
A
P d n = Probability of event A when B has already occurred
B
= Probability of event ‘A ∩ B’ when B behaves like sample space
n (A + B) S
= B A
n (B)
n (A + B)
n ( S)
= [Dividing Nr and Dr by n(S)]
n (B)
n ( S)
P (A + B)
=
P (B)

B P (A + B)
Similarly, P d
n=
A P (A)
Theorem of Total Probability : Let E1, E2, ..., En be the events of a sample space ‘S’ such that they
are pair wise disjoint, exhaustive and have non-zero probabilities. If A is any event associated with
S, then
A A A
P (A) = P (E1) .P e E o + P (E2) .P e E o + .... + P (En) .P e E o
1 2 n

12. Bayes’ Theorem: If B1, B2, ... Bn are mutually exclusive and exhaustive events and A is any event
that occurs with B1 or B2 or Bn then
P ^Bi h .P ^ A/Bi h
P ^Bi /A h = n
, i = 1, 2, ..., n. .
/ P ^Bi h .P ^ A/Bi h
i=1

Note: The probabilities P(Bi) i = 1, 2, ..., n which were already known before performing an

experiment are known as prior probabilities and conditional probabilities P(Bi/A), i = 1, 2, 3, ..., n
which are calculated after the experiment is performed are known as posterior probabilities. The
events B1, B2, ... Bn are usually called causes for event A to occur.
13. Random Variable: Random variable is simply a variable whose values are determined by the
outcomes of a random experiment; generally it is denoted by capital letters such as X, Y, Z, etc. and
their values are denoted by the corresponding small letters x, y, z, etc.
14. Probability Distribution: The system consisting of a random variable X along with P(X) is called
the probability distribution of X.
15. Mean and Variance of a Random Variable: Let a random variable X assume values x1, x2,...xn with
n
probabilities p1, p2, ... pn respectively, such that pi $ 0, R pi = 1. Then, the mean of X, denoted by m,
i=1
[or expected value of X denoted by E(X) is defined as
n
m = E(X) = R xi pi and
i=1
2
Variance denoted by s is defined as

v 2 = / ^xi – nh pi = / xi2 pi – n 2
n 2 n

i=1 i=1

16. Standard Deviation, s= variance

484 Xam idea Mathematics–XII

@Cbsebookshub - Join Us on Telegram


17. Bernoullian Trials: A sequence of independent trials which can result in one of the two mutually
exclusive possibilities success or failure such that the probability of success or failure in each trial is
constant, then such repeated independent trials are called Bernoullian trials.
Suppose we perform a series on n Bernoullian trails for each trial, p is the probability of success and
q is the probability of failure, then p + q = 1.
18. Binomial Distribution: A random variable X taking values 0, 1, 2, ... n is said to have a binomial
distribution with parameters n and p, if its probability distribution is given by
P (X = r) = n Cr p r q n–r …(i)
Where, p represents probability of success while q represents probability of non-success, or failure
and n is the number of trials. A binomial distribution with n-Bernoulli’s trials and probability of
success in each trial as p, is denoted by B(n, p).

Note: While using above probability density functions of the binomial distribution in solving any
problem we should, first of all examine whether all the conditions, given below are satisfied:
(i) There should be a finite number of trials.
(ii) The trials are independent.
(iii) Each trial has exactly two outcomes: success or failure.
(iv) The probability of an outcome remains the same in each trial.
19. Recurrence or recursion formula for the binomial distribution:
n–r p
P ( r + 1) = . P (r)
r+1 q
20. Mean, Variance and Standard Deviation:
(i) Mean = np (iii) Standard Deviation = npq
(ii) Variance = npq

LIST OF IMPORTANT FORMULAE


(i) P(A ∩ B) = P(A) × P(B/A), where A and B are any two events.

(ii) P(A ∩ B) = P(B) × P(A/B), where A and B are any two events.

(iii) Two events A and B are independent, if and only if P(A ∩ B) = P(A) × P(B).

(iv) If A, B, C are three independent events, then P(A ∩ B ∩ C) = P(A) × P(B) × P(C).

(v) P(A ∩ B) = P(B) – P(A ∩ B), where A and B are independent events.

(vi) P(A ∩ B) = P(A) – P(A ∩ B), where A and B are independent events.

(vii) P(A ∩ B) = P(A ∪ B) = 1 – P(A ∪ B) = P(A) × P(B), where A and B are mutually exclusive events.

P ^ A + Bh P^Bh – P^ A + Bh
(viii) P ^ A /B h = = , where A and B are independent events and P(B) ≠ 0.
P^Bh P^Bh

P^ A + B h 1 – P^ A , Bh
(ix) P ^ B / A h = = , where A and B are independent events and P(A) ≠ 1.
P^ A h 1 – P^ Ah

Probability 485
@Cbsebookshub - Join Us on Telegram
Selected NCERT Questions
1. A black and red die are rolled.
(a) Find the conditional probability of obtaining a sum greater than 9 given that the black die
resulted in a 5.
(b) Find the conditional probability of obtaining the sum 8 given that the red die resulted in
a number less than 4.
Sol. When a black and a red die are rolled then n(S) = 36.
(a) Let A be the event getting sum greater than 9 and B be the event getting a 5 on the black die.

A = {(4, 6), (5, 5), (5, 6), (6, 4), (6, 5), (6, 6)}
B = {(5, 1), (5, 2), (5, 3), (5, 4), (5, 5), (5, 6)}
A ∩ B = {(5, 5), (5, 6)}
6 1 6 1 2 1
∴ P (A) = = , P (B) = = and P (A k B) = =
36 6 36 6 36 18
1
P (A k B) 1 6 1
∴ P (A/B) = = 18 = # =
P (B) 1 18 1 3
6
(b) Let A be the event getting the sum 8 and B be the event getting a number less than 4 on red
die.
A = {(2, 6), (3, 5), (4, 4), (5, 3), (6, 2)}
B = {(1, 1), (1, 2), (1, 3), (1, 4), (1, 5), (1, 6), (2, 1), (2, 2), (2, 3), (2, 4), (2, 5), (2, 6),
  (3, 1), (3, 2), (3, 3), (3, 4), (3, 5), (3, 6)}
A ∩ B = {(2, 6), (3, 5)}
5 18 1 2 1

P (A) = , P (B) = = , P (A k B) = =
36 36 2 36 18
1
P (A k B) 1 2 1
∴ P (A/B) = = 18 = # =
P (B) 1 18 1 9
2
2. An instructor has a question bank consisting of 300 easy true/false questions, 200 difficult,
500 easy multiple choice questions and 400 difficult multiple choice questions. If a question
is selected at random from the question bank, what is the probability that it will be an easy
question given that it is a multiple choice question?
Sol. Here, total questions = 300 + 200 + 500 + 400 = 1400
Let A be the event that selected question is an easy question.
300 + 500 800 4
∴ P (A) = = =
1400 1400 7
Let B be the event that selected question is a multiple choice question.
500 + 400 900 9
P (B) = = =
1400 1400 14
Now A ∩ B is the event so that the selected question is a easy multiple choice question.
500 5
∴ P (A k B) = =
1400 14
5
P (A k B) 14 5
∴ P (A/B) = = =
P (B) 9 9
14
486 Xam idea Mathematics–XII

@Cbsebookshub - Join Us on Telegram


3. Consider the experiment of throwing a die. If a multiple of 3 comes up, a die is again thrown
and if any other number comes, a coin is tossed. Find the conditional probability of the event,
‘the coin shows a tail’ given that ‘at least one die shows a 3’.
Sol. Here, S = {(3, 1), (3, 2), (3, 3), (3, 4), (3, 5), (3, 6), (6, 1), (6, 2), (6, 3), (6, 4), (6, 5), (6, 6),
(1, H), (1, T), (2, H), (2, T), (4, H), (4, T), (5, H), (5, T)}
Let A be the event of getting a tail on coin.
A = {(1, T), (2, T), (4, T), (5, T)}
Let B be the event of getting 3 on at least one die.
B = {(3, 1), (3, 2), (3, 3), (3, 4), (3, 5), (3, 6), (6, 3)}
∴ A ∩ B = φ
4 1 7 0
∴ P (A) = = , P (B) = and P (A k B) = =0
20 5 20 20
0
P (A k B) 20
∴ P (A/B) = = =0
P (B) 7
20
1 7 1
4. Events A and B are such that P(A) = , P(B) = and P (not A or not B) = . State whether A
2 12 4
and B are independent.
1 7 1
Sol. Here P (A) = , P (B) = and P ( A j B ) =
2 12 4
= =
Now P ( A j B ) P ( A k B ) 1 – P (A k B)
1 1 3
= 1 – P (A k B) & P (A k B) = 1 – =
4 4 4
1 7 7
Now P(A) × P(B) = # =
2 12 24

P(A ∩ B) ≠ P(A) × P(B)
Thus, A and B are not independent.
1 1
5. Probabilities of solving specific problem independently by A and B are and respectively.
2 3
If both try to solve the problem independently. Find the probability that (i) the problem is
solved (ii) exactly one of them solves the problem.
1 1
Sol. Here, P(A) = and P(B) =
2 3
1 1 1 2
Now P ( A ) = 1 – P (A) = 1 – = ; P ( B ) = 1–P (B) = 1– =
2 2 3 3
(i) P (the problem is solved) = 1 – P ( A + B )
= 1 – P ( A ) P ( B )
1 2
= 1– d
# n= 1– =
1 2

2 3 3 3
(ii) P (exactly one of them solves) = +
P (A) P ( B ) P ( A ) P (B)
1 2 1 1 1 1 2+1 3 1
= # + # = + = = = .
2 3 2 3 3 6 6 6 2

Probability 487
@Cbsebookshub - Join Us on Telegram
6. In a hostel, 60% of the students read Hindi newspaper, 40% read English newspaper and 20%
read both Hindi and English newspaper. A student is selected at random.
(a) Find the probability that the student reads neither Hindi nor English newspaper.
(b) If she reads Hindi newspaper, find the probability that she reads English newspaper.
(c) If she reads English newspaper, find the probability that she reads Hindi newspaper.
Sol. Let A be the event that a student reads Hindi newspaper and B be the event that a student reads
English newspaper.
60 40 20
P (A) = = 0.6, P (B) = = 0.4 and P (A k B) = = 0.2
100 100 100
(a) Now P(A∪ B) = P(A)+P(B) – P(A ∩ B)

= 0.6 + 0.4 – 0.2 = 0.8

Probability that she reads neither Hindi nor English newspaper


1
= 1 – P(A ∪ B) = 1– 0.8 = 0.2 =
5
P (A k B) 0.2 1
(b) P (B/A) = = =
P (A) 0.6 3
P (A k B) 0.2 1
(c) P (A/B) = = =
P (B) 0.4 2
7. An urn contains 5 red and 5 black balls. A ball is drawn at random, its colour is noted and is
returned to the urn. Moreover, 2 additional balls of the colour drawn are put in the urn and
then a ball is drawn at random. What is the probability that the second ball is red?
Sol. Let E1 and E2 be the events that red ball is drawn in first draw and black ball is drawn in first draw
respectively. Let A be the event that ball drawn in second draw is red. There are 5 red and 5 black
balls in the urn.
5 1 5 1
∴ P (E1) = = and P (E2) = =
10 2 10 2
When 2 additional balls of red colour are put in the urn there are 7 red and 5 black balls in the urn.
A
Pe E o =
7

1 12
When 2 additional balls of black colour are put in the urn there are 5 red and 7 black balls in the
urn.
A
Pe E o =
5

2 12
By theorem of total probability
A A
P (A) = P (E1) P e E o + P (E2) P e E o
1 2

1 7 1 5 7 5 12 1

P (A) = # + # = + = =
2 12 2 12 24 24 24 2
8. Of the students in a college, it is known that 60% reside in hostel and 40% are day scholars (not
residing in hostel). Previous year results report that 30% of all students who reside in hostel
attain A grade and 20% of day scholars attain A grade in their annual examination. At the end
of the year, one student is chosen at random from the college and he has an A grade. What is
the probability that the student is a hostelier?
Sol. Let the event be defined as
E1 = selection of hostelier

488 Xam idea Mathematics–XII

@Cbsebookshub - Join Us on Telegram


E2 = selection of day scholar

A = selection of student getting A grade
60 3 40 2
P (E1) = = P (E2) = =
100 5 100 5

Pd E n = Pd E n =
A 30 3 A 20 1
= =
1 100 10 2 100 5
E1
P d n = required
A

P (E1) .P d E n
A 3 3 9
E .
Pd 1 n =
1 5 10 50 9
= = =
P (E1) .P d E n + P (E2) .P d E n
A A A 3 3 2 1 9 2 13
. + . +
1 2
5 10 5 5 50 25

9. A Laboratory blood test is 99% effective in detecting a certain disease when it is in fact, present.
However, the test also yields a false positive result for 0.5% of the healthy person tested (i.e., if
a healthy person is tested then with probability 0.005, the test will imply he has the disease). If
0.1 % of the population actually has the disease then what is the probability that a person has
the disease given that his test result is positive.
Sol. Let E1 and E2 denote the events that a person has disease and does not have disease respectively.
Let A be the event that the test result is positive.
Now, the probability that a person has the disease is
0.1
P (E1) = 0.1% =
= 0.001
100
Probability that a person does not have the disease

P (E2) = 1 – 0.001 = 0.999
Probability that a person has disease and test result is positive.
99
∴ P (A/E1) = 99% = = 0.99
100
Probability that a person does not have disease and test result is positive.
0.5

P (A/E2) = 0.5% = = 0.005
100
By Bayes’ theorem,
P (E1) .P (A/E1)
P (E1 /A) =
P (E1) .P (A/E1) + P (E2) .P (A/E2)
0.001 # 0.99 0.00099
= =
0.001 # 0.99 + 0.999 # 0.005 0.00099 + 0.004995
0.00099 990 22
= = =
0.005985 5985 133
10. A card from a pack of 52 cards is lost. From the remaining cards of the pack, two cards are drawn
and are found to be both diamonds. Find the probability of the lost card being a diamond.
Sol. Let E1, E2 , E3 and E4 be the events that the missing card is a heart, spade, club and diamond
respectively. Let A be the event of drawing two diamond cards from 51 cards.
There are four events and each event is equally likely to be selected.
13 1
∴ P (E1) = P (E2) = P (E3) = P (E4) = =
52 4

Probability 489
@Cbsebookshub - Join Us on Telegram
It is given that
13 C 13 C 13 C 12 C
2 2 2 2
P (A/E1) = 51 C
, P (A/E2) = 51 C
, P (A/E3) = 51 C
and P (A/E4) = 51 C
2 2 2 2

By Bayes’ theorem,
P (E4) P (A/E4)
P (E4 /A) =

P (E1) P (A/E1) + P (E2) P (A/E2) + P (E3) P (A/E3) + P (E4) P (A/E4)

1 12 C2
#
4 51 C2 12 C
2
12 C
2
= = =
1 13 C2 1 13 C2 1 13 C
2 1 12 C
2
13 C + 13
C2 + 13 C2 + 12 C2 3. 13
C2 + 12 C2
# + × + # + × 2
4 51 C2 4 51 C2 4 51 C
2 4 51 C
2
12!
2 !10! 66 66 11
= = = =
13! 12! 3 # 78 + 66 300 50
3# +
2!11! 2!10!
11. A coin is biased so that the head is 3 times as likely to occur as tail. If the coin is tossed twice,
find the probability distribution of number of tails.
Sol. Let X denote the random variable which denotes the number of tails when a biased coin is tossed
twice.
So, X may have values 0, 1 or 2.
Since the coin is biased in which head is 3 times as likely to occur as a tail.
3 1
∴ P (H) = and P (T) =
4 4
Now, P(X = 0) ⇒ Probability of getting two heads
3 3 9
P(X = 0) = # =
4 4 16
P(X =1) ⇒ Probability of getting one tail and one head
3 1 1 3 3 3 6 3
P(X =1) = # + # = + = =
4 4 4 4 16 16 16 8
P(X = 2) ⇒ Probability of getting two tails
1 1 1
P(X = 2) = # =
4 4 16

Thus, required probability distribution is
X 0 1 2
9 3 1
P(X)
16 8 16
12. The random variable X has a probability distribution P(X) of the following form, where k is
some number:
Z]
]] k if x = 0
]]
] 2k if x = 1
P (X) = []
]] 3k if x = 2
]]
] 0 if otherwise
\
(a) Determine the value of k. [CBSE 2019 (65/1/2)]
(b) Find P(X < 2), P(X ≤ 2), P(X ≥ 2).

490 Xam idea Mathematics–XII

@Cbsebookshub - Join Us on Telegram


Sol. (a) k + 2k + 3k =1 [... p1 + p2 + p3 + ... + pn =1]
1
⇒ 6k = 1 ⇒k=
6
1 1
(b) P (X < 2) = k + 2k = 3k = 3 # =
6 2
1
P (X ≤ 2) = 3k + 2k + k = 6k = 6 # =1
6
1 1
P (X ≥ 2) = 3k = 3 # =
6 2
13. Two numbers are selected at random (without replacement) from the first six positive integers.
Let X denote the larger of the two numbers obtained. Find E(X) or the mean of the distribution.
[CBSE (AI) 2014]
Sol. First six positive integers are 1, 2, 3, 4, 5, 6.
If two numbers are selected at random from above six numbers then sample space S is given by
S = {(1, 2), (1, 3), (1, 4), (1, 5), (1, 6), (2,1), (2, 3), (2, 4), (2, 5), (2, 6), (3, 1), (3, 2), (3, 4), (3, 5), (3, 6),
(4, 1), (4, 2), (4, 3), (4, 5), (4, 6), (5, 1), 5, 2), (5, 3), (5, 4), (5, 6), (6, 1), (6, 2), (6, 3), (6, 4), (6, 5)}
Here, n (S) = 30 and X is random variable, which may have values 2, 3, 4, 5 or 6.
Therefore, required probability distribution is given as
2
P (X = 2) = probability of event getting (1, 2), (2, 1) =
30
4
P (X = 3) = probability of event getting (1, 3), (2, 3), (3, 1), (3, 2) =
30
6
P (X = 4) = probability of event getting (1, 4), (2, 4), (3, 4), (4, 1), (4, 2), (4, 3) =

30
8
P (X = 5) = probability of event getting (1, 5), (2, 5), (3, 5), (4, 5), (5, 1), (5, 2), (5, 3), (5, 4) =

30
10
P(X = 6) = probability of event getting (1, 6), (2, 6), (3, 6), (4, 6), (5, 6), (6, 1), (6, 2), (6, 3), (6, 4), (6, 5) =

30
It is represented in tabular form as
X 2 3 4 5 6

P(X) 2 4 6 8 10
30 30 30 30 30
∴ Required mean = E (X) = / pi xi
2 4 6 8 10 4 + 12 + 24 + 40 + 60 140 14 2
= 2× + 3× + 4× + 5× + 6× = = = =4
30 30 30 30 30 30 30 3 3
14. A class has 15 students whose ages are 14, 17, 15, 14, 21, 17, 19, 20, 16, 18, 20, 17, 16, 19 and
20 years. One student is selected in such a manner that each has the same chance of being
chosen and the age X of the selected student is recorded. What is the probability distribution
of the random variable X? Find mean, variance and SD of X.
Sol. Here X = 14, 15, 16, 17, 18, 19, 20, 21 and
2 1 2 3
P (X = 14) = , P (X = 15) = , P (X = 16) = , P (X = 17) = ,
15 15 15 15
1 2 3 1
P (X = 18) =
, P (X = 19) = , P (X = 20) = , P (X = 21) = ,
15 15 15 15

Probability 491
@Cbsebookshub - Join Us on Telegram
Probability distribution

X 14 15 16 17 18 19 20 21
2 1 2 3 1 2 3 1
P(X)
15 15 15 15 15 15 15 15

28 + 15 + 32 + 51 + 18 + 38 + 60 + 21
Now,
∑X P(X) =
15
263

E(X) =
15
392 + 225 + 512 + 867 + 324 + 722 + 1200 + 441

∑X2 P(X) =
15
2 4683
⇒ E(X ) =
15
263
Mean (µ) = E(X) = ∑X P(X) = = 17.53
15
Var (X) = E(X2) – [E(X)]2
263 2 4683 # 15 – 263 # 263 1076
–d n =
4683
= = = 4.78
15 15 225 225

SD (X) = var (X) = 4.78 = 2.19
3
15. The probability of a shooter hitting a target is . How many minimum number of times must
4
he/she fire so that the probability of hitting the target at least once is more than 0.99?
Sol. Let the shooter fire n times.
3
We have, p = probability of hitting the target =
4
1

` q = probability of not hitting the target =
4
Now, using Binomial Probability Distribution, we have
P (X = r) = n Cr p r .q n – r
3 r 1 n – r n 3r
= n Cr d n . d n = Cr n
4 4 4
Given that
P(hitting the target at least once) > 0.99
i.e., P (r $ 1) > 0.99

30 1

` 1 – P (r = 0) > 0.99 ⇒ 1 – n C0 n
> 0.99 ⇒ 1– n
2 0.99 (a n C0 = 1)
4 4
1 1 1
⇒ < 0.01 ⇒ <
4n 4 n 100
⇒ 4 n > 100 …(i)

` The minimum value of n satisfying the inequality (i) is 4 i.e., n = 4
Thus, the shooter must fire 4 times.
16. Assume that the chances of a patient having a heart attack is 40%. Assuming that a meditation
and yoga course reduces the risk of heart attack by 30% and prescription of certain drug
reduces its chance by 25%. At a time a patient can choose any one of the two options with equal
probabilities. It is given that after going through one of the two options, the patient selected
at random suffers a heart attack. Find the probability that the patient followed a course of
meditation and yoga. [CBSE Delhi 2013; Chennai 2015]

492 Xam idea Mathematics–XII

@Cbsebookshub - Join Us on Telegram


Sol. Let E1, E2 , A be events defined as
E1 = treatment of heart attack with Yoga and meditation
E2 = treatment of heart attack with certain drugs
A = person getting heart attack
1 1
P (E1) = , P (E2) =
2 2
Now P d E n = 40% – c 40×
A
m =
30 28
= =
1 100 % 40% – 12% 28% 100

P d E n = 40% – c 40× 100 m % = 40% – 10% = 30% =


A 25 30

2 100
E1
We have to find P d n
A
P (E1) .P d E n
A 1 28
E ×
∴ P d 1 n =
1 2 100 28 100 14
= = × =
P (E1) .P d E n + P (E2) .P d E n
A A A 1 28 1 30 100 58 29
× + ×
1 2
2 100 2 100
17. Bag I contains 3 red and 4 black balls and bag II contains 4 red and 5 black balls. One ball is
transferred from bag I to bag II and then a ball is drawn from bag II. The ball so drawn is found
to be red in colour. Find the probability that the transferred ball is black. [CBSE (F) 2011]
Sol. Let E1 = Event that a red ball is drawn from bag I
E2 = Event that a black ball is drawn from bag I
3 4
Therefore, P (E1) = , P (E2) =
7 7
After transferring a red ball from bag I to bag II, the bag II will have 5 red and 5 black balls.
Let A be the event of drawing red ball
5 1
∴ P (A / E1) = =
10 2
Further, when a black ball is transferred from bag I to bag II, it will contain 4 red and 6 black balls.
4 2
P (A / E2) = =
10 5
∴ By Baye's theorem, we have
P (E2) .P (A / E2)
P (E2 / A) =
P (E1) .P (A / E1) + P (E2) .P (A/E2)
4 2 8
× 8 70 16
= 7 5 = 35 = × =
3 1 4 2 3 8 35 31 31
× + × +
7 2 7 5 14 35

Multiple Choice Questions [1 mark]


Choose and write the correct option in the following questions.
1. From the set {1, 2, 3, 4, 5}, two numbers a and b ( a ! b ) are chosen at random. The probability
a
that is an integer is [CBSE 2020 (65/4/1)]
b
1 1 1 3
(a) (b) (c) (d)
3 4 2 5

Probability 493
@Cbsebookshub - Join Us on Telegram
2. A bag contains 3 white, 4 black and 2 red balls. If 2 balls are drawn at random (without
replacement), then the probability that both the balls are white is [CBSE 2020 (65/4/1)]
1 1 1 1
(a) (b) (c) (d)
18 36 12 24
3. A and B are events such that P(A) = 0.4, P(B) = 0.3 and P (A , B) = 0 . 5. Then P (B' + A) equals
[NCERT Exemplar]
2 1 3 1
(a) (b) (c) (d)
3 2 10 5
3 1 4
4. You are given that A and B are two events such that P (B) = , P (A | B) = and P (A , B) = ,
5 2 5
then P(A) equals
3 1 1 3
(a) (b) (c) (d)
10 5 2 5
5. Three persons, A, B and C, fire at a target in turn, starting with A. Their probability of hitting
the target are 0.4, 0.3 and 0.2 respectively. The probability of two hits is
(a) 0.024 (b) 0.188 (c) 0.336 (d) 0.452
6. Assume that in a family, each child is equally likely to be a boy or a girl. A family with three
children is chosen at random. The probability that the eldest child is a girl given that the
family has at least one girl is [NCERT Exemplar]
1 1 2 4
(a) (b) (c) (d)
2 3 3 7
7. A die is thrown and a card is selected at random from a deck of 52 playing cards. The probability
of getting an even number on the die and a spade card is
1 1 1 3
(a) (b) (c) (d)
2 4 8 4
8. A box contains 3 orange balls, 3 green balls and 2 blue balls. Three balls are drawn at random
from the box without replacement. The probability of drawing 2 green balls and one blue ball
is
3 2 1 167
(a) (b) (c) (d)
28 21 28 168
9. A flashlight has 8 batteries out of which 3 are dead. If two batteries are selected without
replacement and tested, the probability that both are dead is
33 9 1 3
(a) (b) (c) (d)
56 64 14 28
10. Two dice are thrown. If it is known that the sum of number on the dice was less than 6, the
probability of getting a sum 3, is
1 5 1 2
(a) (b) (c) (d)
18 18 5 5
11. Two cards are drawn from a well shuffled deck of 52 playing cards with replacement. The
probability, that both cards are queens, is
1 1 1 1 1 1 1 4
(a) × (b) × (c) × (d) ×
13 13 13 12 13 17 13 51
12. A bag contains 5 red and 3 blue balls. If 3 balls are drawn at random without replacement, then
the probability of getting exactly one red ball is
15 131 15 15
(a) (b) (c) (d)
196 392 56 29
13. Three persons A, B and C, fire at a target in turn, standing with A. Their probability of hitting
the target are 0.4, 0.3 and 0.2 respectively. The probability of two hits is
(a) 0.025 (b) 0.188 (c) 0.339 (d) 0.475

494 Xam idea Mathematics–XII

@Cbsebookshub - Join Us on Telegram


14. The probability distribution of a discrete random variable X is given below:

X 2 3 4 5

P(X) 5 7 9 11
k k k k
The value of k is

(a) 8 (b) 16 (c) 32 (d) 48
15. Two dice are thrown together. Let A be the event ‘getting 6 on the first die’ and B be the event
‘getting 2 on the second die’, then P(A∩B) is
1 7 9
(a) (b) (c) (d) None of these
36 4 20
16. In a college, 30% students fail in Physics, 25% fail in Mathematics and 10% fail in both. One
student is chosen at random. The probability that she fails in Physics if she has failed in
Mathematics is
1 2 9 1
(a) (b) (c) (d)
10 5 20 3
1 1
17. A and B are two students. Their chances of solving a problem correctly are and ,
3 4
1
respectively. If the probability of their making a common error is, and they obtain the
20
same answer, then the probability of their answer to be correct is [NCERT Exemplar]
1 1 13 10
(a) (b) (c) (d)
12 40 120 13
18. Let X be a discrete random variable assuming values x1, x2, ..., xn with probabilities p1, p2, ..., pn,
respectively. Then variance of X is given by [NCERT Exemplar]
(a) E(X2) (b) E(X2) + E(X) (c) E(X2) – [E(X)]2 (d) E (X 2) – [E (X)] 2
1
19. The probability that a student is not a swimmer is . Then the probability that out of five
5
students, four are swimmers is
4 41 4 4 1 1 4 4
(a) 5 C4 d n (b) c m . (c) 5 C1 d n (d) None of these
5 5 5 5 5 5
20. Which one of the following is not a requirement of a binomial distribution?
(a) There are two outcomes for each trial
(b) There is a fixed number of trials
(c) The outcomes must be dependent on each other.
(d) The probability of success must be the same for all trials.

Answers
1. (b) 2. (c) 3. (d) 4. (c) 5. (b) 6. (d)
7. (c) 8. (a) 9. (d) 10. (c) 11. (a) 12. (c)
13. (b) 14. (c) 15. (a) 16. (b) 17. (d) 18. (c)
19. (a) 20. (c)

Probability 495
@Cbsebookshub - Join Us on Telegram
Solutions of Selected Multiple Choice Questions
a
1. In �b can be any of the five values and correspondingly a can assume four values.
b
Thus, the number of fractions possible is 5 × 4 = 20
2 3 4 5 4
Now, out of these, only five i.e., , , , and
1 1 1 1 2
will be integers

n (E) = 5
n (E) 5 1
` Requried probability =
= =
n (S) 20 4
3C
P (Both balls are white) = 9 2
2.
C2
3!
1
  = ! 1! =
2
9! 12
2! 7!
3. Here, P (A) = 0.4, P (B) = 0.3 and P (A , B) = 0.5
 P (A , B) = P (A) + P (B) – P (A + B)
&
P (A + B) = 0.4 + 0.3 – 0.5 = 0.2
` P (B' + A) = P (A) – P (A + B)
1
= 0.4 – 0.2 = 0.2 =
5
6. Here, S = {(B, B, B), (G, G, G), (B, G, G), (G, B, G), (G, G, B), (G, B, B), (B, G, B), (B, B, G)}
E1 = Event that a family has atleast one girl, then

E1 = {(G, B, B), (B, G, B), (B, B, G), (G, G, B), (B, G, G), (G, B, G), (G, G, G)}
E2 = Event that the eldest child is a girl, then

E2 = {(G, B, B), (G, G, B), (G, B, G), (G, G, G)}

` E1 + E2 = {(G, B, B), (G, G, B), (G, B, G), (G, G, G)}

P (E1 + E2) 4/8 4


` P (E2 /E1) =
= =
P (E1) 7/8 7

17. Let E1 = Event that both A and B solve the problem


1 1 1
` P (E1) =
× =
3 4 12
Let E2 = Event that both A and B got incorrect solution of the problem
2 3 1
` P (E2) =
× =
3 4 2
Let E = Event that they got same answer
1
Here, P (E/E1) = 1, P (E/E2) =
20
P (E1 + E) P (E1) .P (E/E1)

` P (E1 /E) = =
P (E) P (E1) .P (E/E1) + P (E2) P (E/E2)

496 Xam idea Mathematics–XII

@Cbsebookshub - Join Us on Telegram


1
×1 1/12 120 10
= 12 = = =
1 1 1 10 + 3 12×13 13
×1 + ×
12 2 20 120
1
19. We have, p = probability that a student is swimmer = 1 –
5
4
& p=
5
1
` q = probability that a student is not a swimmer =
5
1
& q=
5
n = 5 and r = 4
4 4 1 5–4 5 4 4 1

` P (r) = n Cr .p r .q n – r = 5 C4 d n . d n = C4 d n .
5 5 5 5

Fill in the Blanks [1 mark]


5
1. If E and F are independent events such that P (E) = p, P (F) = 2p and P (Exactly one of E, F) = ,
9
then p = ____________.
1 1
2. If P (A) = , P (A + B) = then the value of P(B/A) = ____________.
2 3
3. The mean and variance of a binomial variate X are 2 and 1 respectively, then value of p =
____________.

4. A random variable X has the following probability distribution:

X 0 1 2 3 4 5 6 7 8
P(X = x) a 3a 5a 7a 9a 11a 13a 15a 17a

then the value of a = ____________.

Answers
1 5 2 1 1
1. , 2. 3. 4.
3 12 3 2 81
Solutions of Selected Fill in the Blanks
1. P(Exactly one of E, F) = P (E) . P ( F ) + P ( E ) .P (F)
= p (1 – 2p) + (1 – p) 2p
5
= p – 2p + 2p – 2p 2 = 3p – 4p 2
2
9
& 5 = 27p – 36p2

& 36p2 – 27p + 5 = 0


27 ! (27) 2 – 4 × 36 × 5 27 ! 3
& p =

2 × 36
=
72
30 24 5 1
& p = 72 , 72 = 12 , 3

Probability 497
@Cbsebookshub - Join Us on Telegram
1 5
` p =
,
3 12
1
P (A + B)
2. P (B/A) = = 3
P (A) 1
2
2
& P (B/A) = 3
3. We have, mean = np = 2 & np = 2 …(i)

and variance = npq = 1 & npq = 1 …(ii)

Dividing (ii) by (i) we have,

npq 1 1

np
=
2
&q= 2
1 1
∴ p = 1 – q = 1 –
=
2 2
1
4. We have, RP (X) = 1 & 81 a = 1 & a=
81

Very Short Answer Questions [1 mark]


1. An unbiased coin is tossed 4 times. Find the probability of getting atleast one head.
[CBSE 2020 (65/3/1)]
Sol. When an unbiased coin is tossed once, then
1
P (H) = P (T) = , where H and T denote head and tail respectively.
2
∴ Probability of getting atleast one head

= 1 – P (No head)
1 1 1 1
= 1 – P (all tails) = 1 – × × ×
2 2 2 2
1 15
= 1 – =
16 16
2. A die, whose faces are marked 1, 2, 3 in red and 4, 5, 6 in green, is tossed. Let A be the event
"number obtained is even" and B be the event "number obtained is red". Find if A and B are
independent events. [CBSE (AI) 2017]
Sol. Here,
A = Event that "number obtained is even".

B = Event that "number obtained is red".

3 1 3 1 1 1 1 1
P (A) = = ; P (B) = = ; P (A + B) = ; P (A) × P (B) = × =
6 2 6 2 6 2 2 4
i.e., P (A + B) ! P (A) .P (B)

Hence, A and B are not independent events.
3. Write the probability of an even prime number on each die, when a pair of dice is rolled.
1 1 1
Sol. The probability of getting even number on each die = # =
6 6 36
(As there is only one even prime number on each die i.e., 2).

498 Xam idea Mathematics–XII

@Cbsebookshub - Join Us on Telegram


4. Two Independent events A and B are given such that P(A) = 0.3 and P(B) = 0.6, find P(A and not B).
Sol. We have,
P(A and not B) = P(A ∩ B') = P(A) – P(A ∩ B)
= 0.3 – 0.18 [ P(A ∩ B) = P(A) × P(B)]
= 0.12
5. The probability distribution of X is:
X 0 1 2 3
P(X) 0.2 k k 2k
Write the value of k.
Sol. We have,
SP(X) = 1 ⇒ 0.2 + 4k = 1 ⇒ 4k = 0.8 ⇒ k = 0.2
6. The probability that atleast one of the two events A and B occurs is 0.6. If A and B occur
simultaneously with probability 0.3, evaluate P ( A ) + P ( B ) . [NCERT Exemplar]
Sol. We know that, A , B denotes the occurrence of atleast one of A and B and A + B denotes the
occurrence of both A and B, simultaneously.
Thus, P (A , B) = 0.6 and P (A + B) = 0.3
Also, P (A , B) = P (A) + P (B) – P (A + B) ⇒ 0.6 = P (A) + P (B) – 0.3

P (A) + P (B) = 0.9

[1 – P ( A )] + [1 – P ( B )] = 0.9 [a P (A) = 1 – P ( A ) and P (B) = 1 – P ( B )]
⇒ P ( A ) + P ( B ) = 2 – 0.9 = 1.1

A
7. Let A and B be two events. If P(A) = 0.2, P(B) = 0.4, P(A ∪ B) = 0.6 then find P c m .
B
[NCERT Exemplar]
Sol. Since, P(A ∪ B) = P(A) + P(B) – P(A ∩ B)
0.6 = 0.2 + 0.4 – P(A ∩ B) ⇒ P(A ∩ B) = 0.6 – 0.6 = 0
A P ^ A + Bh 0
\
Pc m= = =0
B P ^B h 0.4

Short Answer Questions-I [2 marks]


1. Prove that if E and F are independent events, then the events E and F′ are also independent.
[CBSE Delhi 2017]
Sol. Since, E and F are independent events.

P(E ∩ F) = P(E). P(F)
Now, P(E ∩ F′) = P(E) – P(E ∩ F)

= P(E) – P(E). P(F) = P(E)(1 – P(F))

P(E ∩ F′) = P(E). P(F′)

Hence, E and F′ are independent events.
2. From a set of 100 cards numbered 1 to 100, one card is drawn at random. Find the probability
that the number on the card is divisible by 6 or 8, but not by 24. [CBSE Chennai 2015]
Sol. Numbers divisible by 6 from 1 to 100 = 6, 12, 18, 24, 30, 36, 42, 48, 54, 60, 66, 72, 78, 84, 90, 96
Numbers divisible by 8 from 1 to 100 = 8, 16, 24, 32, 40, 48, 56, 64, 72, 80, 88, 96

Probability 499
@Cbsebookshub - Join Us on Telegram
∴ Numbers divisible by 6 or 8 but not by 24 from 1 to 100 = 6, 8, 12, 16, 18, 30, 32, 36, 40, 42, 54,
56, 60, 64, 66, 78, 80, 84, 88, 90.
20 1
∴ Required probability = = .
100 5
3. If P (A) = 0.6, P (B) = 0.5 and P (B/A) = 0.4, find P (A , B) and P (A/B) . [CBSE 2019 (65/5/1)]
Sol. We have P (A) = 0.6, P (B) = 0.5 and P (B/A) = 0.4
P (A + B)
a P (B/A) = & P (A + B) = P (A) P (B/A)
P (A)
& P (A + B) = 0.6 × 0.4 = 0.24

Now P (A , B) = P (A) + P (B) – P (A + B)
= 0.6 + 0.5 – 0.24 = 1.1 – 0.24 = 0.86
P (A + B) 0.24
P (A/B) = = = 0.48
P (B) 0.5
2 4
4. The probability of two students A and B coming to school in time are and , respectively.
7 7
Assuming that the events ‘A coming on time’ and ‘B coming on time’ are independent, find the
probability of only one of them coming to school on time. [CBSE 2019 (65/5/3)]
2 4
Sol. P (A) = i.e.; A is coming on time, P (B) = i.e.; B is coming on time,
7 7
2 5 4 3
P (A') = 1 – = , P (B') = 1 – =
7 7 7 7
` Probability of only one of them coming to school on time = P (A) P (B') + P (A') P (B)
2 3 5 4 6 20 26
= × + × = + =
7 7 7 7 49 49 49
5. Four cards are drawn one by one with replacement from a well-shuffled deck of playing cards.
Find the probability that at least three cards are of diamonds. [CBSE 2019 (65/5/3)]
Sol. X be the random variable of drawing at least three cards are of diamonds.
P(At least three cards are of diamonds)
= P (X = 3) + P (X = 4)
13 C × 39 C 13 C
8 13C3 × 39 + C4B
3 1 4 1 13
= 52 C
+ 52 C
= 52 C
1 4 4 4
13! 13!
= 152 < × 39 + F
4! 48! 3! 10! 4! 9!
1
= 49 × 50 × 51 × 52 < 11 × 12 × 13 × 39 + 10 × 11 × 12 × 13 F
4× 3×2×1 3× 2×1 4× 3×2×1
11 × 12 × 13 × 39 × 4 + 10 × 11 × 12 × 13
< F
4× 3×2×1
=
49 × 50 × 51 × 52 4× 3×2×1
1 11 × 12 × 13 × 166
= × 11 × 12 × 13 {.156 + 10} = = 0.04
49 × 50 × 51 × 52 49 × 50 × 51 × 52

6. The mean and variance of a binomial distribution are 12 and 3 respectively. Find the binomial
distribution.
Sol. Mean = np = 12 & np = 12
and, variance = npq = 3 & npq = 3

500 Xam idea Mathematics–XII

@Cbsebookshub - Join Us on Telegram


npq 3 1 1 1
` np = =
12 4
& q=
4
& 
p = 1–q = 1–
4

& p = 34

3

Now, np = 12 & n×
4
= 12 & n = 16

` Binomial distribution is

B c16, m
1

4

Short Answer Questions-II [3 marks]


1. Assume that each born child is equally likely to be a boy or a girl. If a family has two children,
what is the conditional probability that both are girls given that [CBSE Delhi 2014]
(i) the youngest is a girl? (ii) atleast one is a girl?
Sol. A family has 2 children,
then sample space = S = {BB, BG, GB, GG}, where B stands for boy and G for girl.
(i) Let A and B be two event such that
A = Both are girls = {GG} and
B
= The youngest is a girl = {BG, GG}
1
A P(A + B) 1
Pc m = = 4 = [ A ∩ B = {GG}]
B P(B) 2 2
4
(ii) Let C be event such that
C = at least one is a girl = {BG, GB, GG}
1
P(A + C) 1
Now P(A/C) = = 4 = [ A ∩ C = {GG}]
P(C) 3 3
4
2. A committee of 4 students is selected at random from a group consisting of 7 boys and 4 girls.
Find the probability that there are exactly 2 boys in the committee, given that at least one girl
must be there in the committee. [CBSE (East) 2016]
Sol. Let A and B be two events such that
A = selection of committee having exactly 2 boys.
B = selection of committee having at least one girl.
The required probability is P c m
A
B

Now, P c m=
A P (A + B)
B P (B)
4! 7! 4! 7! 4! 7! 4!
4 C ×7 C + 4 C ×7 C + 4 C ×7 C + 4 C × + × + × +
1 3 2 2 3 1 4
= 1 !3 ! 3 !× 4 ! 2 ! 2 ! 2 ! 5 ! 3 ! 1 ! 1 ! 6! 4 ! 0!
P (B) = 11 C
4 11 !
4!7!
7×6×5 4×3 7×6
4× + × + 4×7 + 1 140 + 126 + 28 + 1 295 59
= 3×2 2×1 2×1 = = =
11×10×9×8 330 330 66
4×3×2×1

Probability 501
@Cbsebookshub - Join Us on Telegram
4! 7! 4×3 7×6
4 C ×7 C × × 126 21
P(A ∩ B) = 2 2
= 2 ! 2 ! 2 ! 5 ! = ×1 2×1 =
2 =
11 C 11! 330 330 55
4
4!7!
21
A 21 66 126
∴ P d n = 55 = × =
B 59 55 59 295
66
3. In a hockey match, both teams A and B scored same number of goals up to the end of the game,
so to decide the winner, the referee asked both the captains to throw a die alternately and
decided that the team, whose captain gets a six first, will be declared the winner. If the captain
of team A was asked to start, find their respective probabilities of winning the match and state
whether the decision of the referee was fair or not. [CBSE (AI) 2013]
Sol. Let E1, E2 be two events such that
E1 = The captain of team ‘A’ gets a six.
E2 = The captain of team ‘B’ gets a six.
1 1 1 5 1 5
Here, P (E1) = , P (E2) = P (El1) = 1 – = , P (El2) = 1 – =
6 6 6 6 6 6
1 5 5 1 5 5 5 5 1
Now, P (winning the match by team A) = + × × + × × × × + ...
6 6 6 6 6 6 6 6 6
1
5 2 1 5 4 1
= + c m . + c m . + ..... =
1 6 1 36 6
= × =
6 6 6 6 6 5 2 6 11 11
1 –c 6 m
6 5
P (winning the match by team B) = 1 – =
11 11
a
[Note: If a be the first term and r the common ratio then sum of infinite terms S∞ = ]
1–r
Decision: Referee’s decision was fair became team spirit enhances co-operation and unity.
4. A bag A contains 4 black and 6 red balls and bag B contains 7 black and 3 red balls. A die is
thrown. If 1 or 2 appears on it, then bag A is chosen, otherwise bag B. If two balls are drawn at
random (without replacement) from the selected bag, find the probability of one of them being
red and another black. [CBSE Delhi 2015]
Sol. Let E, F and A be three events such that
E = selection of bag A and F = selection of bag B
A = getting one red and one black ball out of two
2 1
Here, P(E) = P(getting 1 or 2 in a throw of die) = =
6 3
1 2
∴ P(F) = 1 – =
3 3
6 C ×4 C
1 1 24
Also, P(A/E) = P (getting one red and one black if bag A is selected) = 10 C =
2 45
3 C ×7 C
1 1 21
and P(A/F) = P (getting one red and one black if bag B is selected) = 10 C =
2 45
Now, by theorem of total probability,
P(A) = P (E). P (A/E) + P (F). P (A/F)
1 24 2 21 8 + 14 22
= × + × = =
3 45 3 45 45 45
5. Bag I contains 3 red and 4 black balls while another bag II contains 5 red and 6 black balls. One
ball is drawn at random from one of the bags and it is found to be red. Find the probability that
it was drawn from bag II. [CBSE Delhi 2011]

502 Xam idea Mathematics–XII

@Cbsebookshub - Join Us on Telegram


Sol. Let E1 be the event of choosing the bag I, E2 the event of choosing the bag II and A be the event of
drawing a red ball.
1
Then P(E1) = P(E2) =
2
3
Also P(A/E1) = P (drawing a red ball from bag I) =
7
5
and P(A/E2) = P (drawing a red ball from bag II) =
11
Now, the probability of drawing a ball from bag II, being given that it is red, is P(E2/A). By using
Bayes’ theorem, we have
1 5
P(E2) P(A/E2) # 35
P(E2 /A) = = 2 11 =
P ( E ) P ( A /E ) + P ( E ) P ( A /E ) 1 3 1 5 68
1 1 2 2 # + #
2 7 2 11

6.
Suppose 5% of men and 0.25% of women have grey hair. A grey haired person is selected
at random. What is the probability of this person being male? Assume that there are equal
number of males and females. [CBSE Delhi 2011]
Sol. Let E1 , E2 and A be event such that
E1 = Selecting male person

E2 = Selecting women (female person)

A = Selecting grey haired person.

1 1
Then P (E1) = , P (E2) =
2 2

A A
Pe E o = Pe E o =
5 0.25
,
1 100 2 100

E1
Here, required probability is P d n.
A
A
P (E1) . P e E o
E
Pd 1 n =
1


A A A
P (E1) . P e E o + P (E2) . P e E o
1 2

1 5
E1 #
Pd n = 2 100 5 500 20

= = =
A 1 5 1 0.25 5 + 0.25 525 21
# + #
2 100 2 100
7. Three persons A, B and C apply for a job of manager in a private company. Chances of their
selection (A, B and C) are in the ratio 1 : 2 : 4. The probabilities that A, B and C can introduce
changes to improve profits of the company are 0.8, 0.5 and 0.3 respectively. If the change does
not take place, find the probability that it is due to the appointment of C. [CBSE Delhi 2016]
Sol. Let E1, E2, E3 and A be events such that
E1 = person selected is A; E2 = person selected is B; E3 = person selected is C
A = changes to improve profit does not take place.
1 2 4
Now P (E1) = , P (E2) = , P (E3) =
7 7 7

Probability 503
@Cbsebookshub - Join Us on Telegram
P d E n = 1– ; P d E n = 1– Pd E n = 1 –
A 8 2 A 5 5 A 3 7
= = ; =
1 10 10 2 10 10 3 10 10
E3
We require P d n
A

P (E3) .P d E n
A 4 7
E ×
P d 3 n = 7 10
3
=
P (E1) .P d E n + P (E2) .P d E n + P (E3) .P d E n
A A A A 1 2 2 5 4 7
× + × + ×
1 2 3
7 10 7 10 7 10
28 70 28 7
= × = =
70 2 + 10 + 28 40 10
8. A bag contains 4 balls. Two balls are drawn at random (without replacement) and are found to
be white. What is the probability that all balls in the bag are white? [CBSE (Central) 2016]
Sol. There may be three situations as events.
E1 = bag contains 2 white balls, E2 = bag contains 3 white balls,
E3 = bag contains all 4 white balls, A = getting two white balls.
E3
We have required P d n=?
A
1 1 1
Now, P (E1) = ; P (E2) = ; P (E3) =
3 3 3
2C 3C 4C
Pd E n = 4 2 = ; Pd E n = 4 2 = = ; Pd E n = 4 2 = 1
A 1 A 3 1 A


1 C2 6 2 C2 6 2 3 C2

P (E3) .P d E n
A
E3
Now, P d n=
3

P (E1) .P d E n + P (E2) .P d E n + P (E3) .P d E n


A A A A
1 2 3

1 1 1
×1 1 18 3
= 3 = 3 = 3 = × =
1 1 1 1 1 1 1 1 10 3 10 5
× + × + ×1 + +
3 6 3 2 3 18 6 3 18
9. A card from a pack of 52 cards is lost. From the remaining cards of the pack, two cards are
drawn at random and are found to be both clubs. Find the probability of the lost card being of
clubs.[CBSE Delhi 2010]
Sol. Let A, E1 and E2 be the events defined as follows:
A : cards drawn are both clubs
E1 : lost card is club; E2 : lost card is not a club
13 1 39 3
Then, P (E1) =
= , P (E2) = =
52 4 52 4
12 11
P(A/E1) = Probability of drawing both club cards when lost card is club = #
51 50
13 12
P(A/E2) = Probability of drawing both club cards when lost card is not a club = #
51 50
To find : P(E1/A)
By Baye’s Theorem,
P (E1) P (A/E1)
P (E1 /A) =
P (E1) P (A/E1) + P (E2) P (A/E2)

504 Xam idea Mathematics–XII

@Cbsebookshub - Join Us on Telegram


1 12 11
# # 12 # 11 11 11
= 4 51 50 = = =
1 12 11 3 13 12 12 # 11 + 3 # 13 # 12 11 + 39 50
# # + # #
4 51 50 4 51 50
10. Often it is taken that a truthful person commands, more respect in the society. A man is known
to speak the truth 4 out of 5 times. He throws a die and reports that it is actually a six. Find the
probability that it is actually a six.  [CBSE Delhi 2017; (F) 2013]
Sol. Let E1, E2 and E be three events such that
E1 = six occurs; E2 = six does not occur and
E = man reports that six occurs in the throwing of the dice.

, P (E2) = ; P d E n = , P d E n = 1– =
1 5 E 4 E 4 1
Now, P (E1) =
6 6 1 5 2 5 5
E1
We have to find P d n
E

P (E1) .P d E n
E 1 4
E ×
Pd 1 n =
1 6 5 4 30 4
= = × =
P (E1) .P d E n + P (E2) .P d E n
E E E 1 4 5 1 30 4 + 5 9
× + ×
1 2
6 5 6 5
11. In shop A, 30 tin pure ghee and 40 tin adulterated ghee are kept for sale while in shop B, 50
tin pure ghee and 60 tin adulterated ghee are there. One tin of ghee is purchased from one of
the shops randomly and it is found to be adulterated. Find the probability that it is purchased
from shop B.[CBSE (F) 2017]
OR
In a shop X, 30 tins of ghee of type A and 40 tins of ghee of type B which look alike, are kept for
sale. While in shop Y, similar 50 tins of ghee of type A and 60 tins of ghee of type B are there.
One tin of ghee is purchased from one of the randomly selected shop and is found to be of
type B. Find the probability that it is purchased from shop Y. [CBSE 2020 (65/5/1)]
Sol. Let the event be defined as
E1 = selection of shop A; E2 = selection of shop B and
A = purchasing of a tin having adulterated ghee

P (E1) = , P (E2) = , P d E n = = , Pd E n =
1 1 A 40 4 A 60 6
=
2 2 1 70 7 2 110 11
∴ Required probability is given by

P (E2) .P d E n
A 1 6 3
E2 .
P d n =
2 2 11 11 21
= = =
P (E1) .P d E n + P (E2) .P d E n
A A A 1 4 1 6 2 3 43
. + . +
1 2
2 7 2 11 7 11

12. A random variable X has the following probability distribution:


X 0 1 2 3 4 5 6 7
2 2 2
P(X) 0 k 2k 2k 3k k 2k 7k + k

Determine:
(i) k (ii) P (X < 3) (iii) P (X > 6) (iv) P (0 < X < 3) [CBSE (AI) 2011]
n
Sol.  / pi = 1
i =1

Probability 505
@Cbsebookshub - Join Us on Telegram
∴ 0 + k + 2k + 2k +3k + k2 + 2k2 + 7k2 + k = 1

10k2 + 9k – 1 = 0
2
⇒ 10k + 10k – k – 1 = 0
⇒ 10k (k + 1) – 1(k + 1) = 0
1

(k + 1) (10k – 1) = 0 ⇒ k = – 1 and k =
10
1
(i)  k can never be negative as probability is never negative then k = 10
(ii) P(X < 3) = P(X = 0) + P(X = 1) + P(X = 2)
3
= 0 + k + 2k = 3k =
10
1 1 17
(iii) P(X 2 6) = P (X = 7) = 7k 2 + k = 7 # + =
100 10 100
3
(iv) P(0 1 X 1 3) = P (X = 1) + P(X = 2) = k + 2k = 3k =
10
13. Three numbers are selected at random (without replacement) from first six positive integers. If
X denotes the smallest of the three numbers obtained, find the probability distribution of X.
Also, find the mean and variance of the distribution. [CBSE (South) 2016]
Sol. First six positive integers are 1, 2, 3, 4, 5 and 6.
If three numbers are selected at random from above six numbers then the number of elements in
sample space S is given by
6! 6#5#4
i.e., n (s) = 6 C3 = = = 20
3!3! 3#2
Here X, smallest of the three numbers obtained, is random variable X may have value 1, 2, 3
and 4. Therefore, required probability distribution is given as
P(X = 1) = Probability of event getting 1 as smallest number
5C
2 5! 5#4 10 1 5
= = = = = [ C2 / selection of two numbers out of 2, 3, 4, 5, 6]
20 2 ! 3 ! # 20 2 # 20 20 2
P(X = 2) = Probability of events getting 2 as smallest number.

4C
2 4! 6 3
= = = = [4C2 ≡ selection of two numbers out of 3, 4, 5, 6]
20 2 ! 2 ! # 20 20 10
P(X = 3) = Probability of events getting 3 as smallest number

3C
2 3! 3
= = = [3C2 ≡ selection of two numbers out 4, 5, 6]
20 2!1!×20 20
P(X = 4) = Probability of events getting 4 as smallest number.

2
C2 1
= = [2C2 ≡ selection of two numbers out of 5, 6]
20 20
Required probability distribution table is
X or xi 1 2 3 4

P(X) or pi 1 3 3 1
2 10 20 20
Mean = E(X) = Spixi
1 3 3 1
= 1# +2# +3# +4#
2 10 20 20
1 6 9 4 10 + 12 + 9 + 4 35 7
= + + + = = =
2 10 20 20 20 20 4

506 Xam idea Mathematics–XII

@Cbsebookshub - Join Us on Telegram


Variance = / xi2 pi – [E (X)] 2
1 3 3 1 7 2
= '1 2 # + 22 # + 32 # + 42 # 1–c m
2 10 20 20 4
1 12 27 16 49 10 + 24 + 27 + 16 49
= + + + – = –
2 10 20 20 16 20 16
77 49 308 – 245 63
=– = =
20 16 80 80
14. In a game, a man wins `5 for getting a number greater than 4 and loses `1 otherwise, when a fair
die is thrown. The man decided to throw a die thrice but to quit as and when he gets a number
greater than 4. Find the expected value of the amount he wins/loses. [CBSE (Central) 2016]
Sol. Let X be random variable, which is possible value of winning or loosing of rupee occur with
probability of getting a number greater than 4 in 1st, 2nd, 3rd or in any throw respectively.
Obviously X may have value `5, `4, `3 and – `3 respectively.
Now, P(X = 5) = P (getting number greater than 4 in first throw)
2 1
= =
6 3
P(X = 4) = P (getting number greater than 4 in 2nd throw)
4 2 2 1 2
× = × = =
6 6 3 3 9
P(X = 3) = P (getting number greater than 4 in 3rd throw)
4 4 2 2 2 1 4
= × × = × × =
6 6 6 3 3 3 27
P(X = –3) = P (getting number greater than 4 in no throw)
4 4 4 2 2 2 8
=
× × = × × =
6 6 6 3 3 3 27
Therefore, probability distribution is as
X or xi 5 4 3 –3

P(X) or pi 1 2 4 8
3 9 27 27
∴ Expected value of the amount he wins/loses = E (X)
1 2 4 8
E (X) = / xi pi = 5× + 4× + 3× + (– 3) ×
3 9 27 27
5 8 12 24 45 + 24 + 12 – 24 57 19 1
= + + – = = =` =`2
3 9 27 27 27 27 9 9

15. There are 4 cards numbered 1 to 4, one number on one card. Two cards are drawn at random
without replacement. Let X denote the sum of the numbers on the two drawn cards. Find the
mean and variance of X. [CBSE (F) 2017]
Sol. If two cards, from four cards having numbers 1, 2, 3, 4 each are drawn at random then sample
space S is given by
S = {(1, 2), (1, 3), (1, 4), (2, 1), (2, 3), (2, 4), (4, 1), (4, 2), (4, 3), (3, 1), (3, 2), (3, 4)}
Let X, sum of the numbers, be random variable. X may have values 3, 4, 5, 6, 7.
2 1
Now P(X = 3) = Probability of event getting (1, 2), (2, 1) = =
12 6

Probability 507
@Cbsebookshub - Join Us on Telegram
2 1
P(X = 4) = Probability of event getting (1, 3), (3, 1) = =
12 6
4 1
P(X = 5) = Probability of event getting (1, 4), (4, 1), (2, 3), (3, 2) = =
12 3
2 1
P(X = 6) = Probability of event getting (4, 2), (2, 4) = =
12 6
2 1
P(X = 7) = Probability of event getting (4, 3), (3, 4) = =
12 6
Thus, probability distribution is represented in tabular form as
X 3 4 5 6 7
P(X) 1 1 1 1 1
6 6 3 6 6
X.P(X) 3 4 5 6 7
6 6 3 6 6
X2P(X) 9 16 25 36 49
6 6 3 6 6

3 4 5 6 7 3 + 4 + 10 + 6 + 7 30
∴ Mean = ∑X.P(X) =
+ + + + = = =5
6 6 3 6 6 6 6
Variance = ∑X2 P(X) – (∑X. P(X))2
9 16 25 36 49 9 + 16 + 50 + 36 + 49

=d + + + + n – (5) 2 = – 25
6 6 3 6 6 6
160 160 – 150 10 5
= – 25 = = = .
6 6 6 3

16. The random variable X can take only the values 0, 1, 2, 3. Given that P(X = 0) = P(X = 1) = p and
P(X = 2) = P(X = 3) = a such that / pi x i2 = 2∑pixi , find the value of p. [CBSE Delhi 2017]
Sol. Given X is a random variable with values 0, 1, 2, 3. Given probability distributions are as
X(xi) 0 1 2 3
P(x)(pi) p p a a
x ip i 0 p 2a 3a

x i2 pi 0 p 4a 9a

∴ ∑xipi = 0 + p + 2a + 3a = p + 5a

∑ xi2 pi = 0 + p + 4a + 9a = p + 13a

According to question
/ pi xi2 = 2 / pi xi
p + 13a = 2p + 10a ⇒ p = 3a

Also p + p +a +a = 1 ⇒ 2 p + 2a = 1
1 – 2p
2a = 1 – 2p ⇒ a=
2
^1–2p h

p = 3# ⇒ 2 p = 3 – 6p
2
3

8p = 3 ⇒ p=
8

508 Xam idea Mathematics–XII

@Cbsebookshub - Join Us on Telegram


17. Three machines E1, E2, E3 in a certain factory produce 50%, 25% and 25% respectively, of
the total daily output of electric tubes. It is known that 4% of the tube produced on each of
machines E1 and E2 are defective and that 5% of those produced on E3, are defective. If one tube
is picked up at random from a day’s production, calculate the probability that it is defective.
[NCERT Exemplar; CBSE (F) 2015]
Sol. Let A be the event that the picked up tube is defective.
Let A1, A2, A3 be events such that
A1 = event of producing tube by machine E1
A2 = event of producing tube by machine E2
A3 = event of producing tube by machine E3

50 1 25 1 25 1
P (A1) = = , P (A2) = = , P (A3) = =
100 2 100 4 100 4

Also, P d A n = ; Pd A n = Pd A n =
A 4 1 A 4 1 A 5 1
= = and =
1 100 25 2 100 25 3 100 20
Now, P (A) is required.
From concept of total probability,

P (A) = P (A1) .P d A n + P (A2) .P d A n + P (A3) .P d A n


A A A
1 2 3

1 1 1 1 1 1 1 1 1
= × + × + × = + +
2 25 4 25 4 20 50 100 80
8+4+5 17
= = = 0.0425
400 400

18. Four bad oranges are accidentally mixed with 16 good ones. Find the probability distribution
of the number of bad oranges when two oranges are drawn at random from this lot. Find the
mean and variance of the distribution. [CBSE Sample Paper 2018]

Sol. Let X be the number of bad oranges in two draws of orange from the lot. Here, X is random
variable and may have value 0, 1, 2.
16
C2 16! 2! ×18! 18×17×16×15 60
Now, P^X = 0h = = × = =
20
C2 2! ×14! 20! 20×19×18×17 95

4
C1 # 16C1 4 × 16 × 2 32 4
C2 4×3 3
P (X = 1) = 20 C
= = ; P (X = 2) = = =
2 20 × 19 95 20
C2 20 × 19 95

Now, required probability distribution is


X 0 1 2
60 32 3
P (X)
95 95 95

60 32 3 32 6 38 2
Now, Mean = R Xi P ^ Xi h = 0 × + 1× + 2× = + = =
95 95 95 95 95 95 5
32 3 2 2 44 4 144
Variance = RXi2 P ^ Xi h – R (Xi P ^ Xi h) = 1× –c m =
2
+ 4× – =
95 95 5 95 25 475

Probability 509
@Cbsebookshub - Join Us on Telegram
19. A coin is biased so that the head is three times as likely to occur as tail. If the coin is tossed
twice, find the probability distribution of number of tails. Hence find the mean of the number
of tails. [CBSE 2020 (65/4/1]

Sol. A coin is biased such that


3 1
P(H) = = and P(T) =
4 4
Let X = No. of tails when coin is tossed twice.


When X = 0 (i.e. No Tail)
3 3 9

P (X = 0) = P (H) .P (H) = × =
4 4 16
when X = 1 (i.e., only one tail)


P (X = 1) = P (T) .P (H) + P (H) .P (T)
1 3 3 1 6

P (X = 1) = × + × =
4 4 4 4 16
when X = 2 (i.e., both tails).
1 1 1
P(X = 2) = P(T).P(T) = × =
4 4 16
Probability distribution is given by
X 0 1 2

P (X) 9 6 1
16 16 16

We have, mean = Rpi (X) . Xi


9 6 1

Mean = 0 × + 1× +2×
16 16 16
6 2 8 1
= + = =
16 16 16 2
20. Five cards are drawn successively with replacement from a well-shuffled deck of 52 cards.
What is the probability that
(i) all the five cards are spades?
(ii) only three cards are spades?
Sol. Let X denote the number of spade card
p = probability that drawn card is spade
and,  
13 1
 = =
52 4
1
⇒   p =
4
1 3

q = probability that drawn card is not spade = 1 – =
4 4
3
⇒  
q=
4
and n = 5.
1 5 3 5–5 1 5
(i) P (X = 5) = 5 C5 d n × d n = 1× d n ×1
4 4 4

510 Xam idea Mathematics–XII

@Cbsebookshub - Join Us on Telegram


1 5
     = c m =
1
4 1024
1 3 3 5–3 1 3 3 2
(ii) P (X = 3) = 5 C3 c m × c m ×c m ×c m
5!
=
4 4 3! 2! 4 4
9 90 45
     = 10× = =
(4) 5 1024 512

Long Answer Questions [5 marks]


1. There are two boxes I and II. Box I contains 3 red and 6 black balls. Box II contains 5 red and ‘n’
black balls. One of the two boxes, box I and box II is selected at random and a ball is drawn at
random. The ball drawn is found to be red. If the probability that this red ball comes out from
3
box II is , find the value of ‘n’. [CBSE 2019 (65/3/1)]
5
Sol. We have events
E1: bag I is selected; E2: bag II is selected; A : getting a red ball
1 1 3 1 5
So, P (E1) =, P (E2) = , P (A/E1) = = and P (A/E1) =
2 2 9 3 5+n
P (E2) .P (A/E2)
Now, P (E2 /A) =
P (E1) .P (A/E1) + P (E2) .P (A/E2)
1 5 5
×
3 2 5+n 5+n
⇒  
= =
5 1 1 1 5 1 5
× + × +
2 3 2 5+n 3 5+n
3 5 3 (5 + n) 15
⇒  
= × =
5 5 + n (5 + n) + 15 20 + n
⇒  
60 + 3n = 75 ⇒ 3n = 15   ⇒    n = 5
A card from a pack of 52 playing cards is lost. From the remaining cards of the pack three
2.
cards are drawn at random (without replacement) and are found to be all spades. Find the
probability of the lost card being a spade. [CBSE Delhi 2014]
Sol. Let E1, E2, E3, E4 and A be event defined as
E1 = the lost card is a spade card, E2 = the lost card is a non spade card
and A = drawing three spade cards from the remaining cards.
13 1 39 3
Now, P (E1) = = , P (E2) = =
52 4 52 4
12 C 13 C
A A
P e E o = 51 P e E o = 51
3 220 3 286
= ; =
1 C 3
20825 2 C 3
20825
E1
Here, required probability = P e o
A
A
P (E1) .P e E o 1
×
220
1 4 20825

= =
A A 1 220 3 286
e
P (E1) .P E o + e
P (E2) .P E o × + ×
4 20825 4 20825
1 2
220 220 10
= = =
220 + 3×286 1078 49

Probability 511
@Cbsebookshub - Join Us on Telegram
3. Bag I contains 3 red and 4 black balls and bag II contains 4 red and 5 black balls. Two balls
are transferred at random from bag I to bag II and then a ball is drawn from bag II. The ball
so drawn is found to be red in colour. Find the probability that the transferred balls were both
black. [CBSE Delhi 2012]
Sol. Let E1, E2, E3 and A be events such that
E1 = both transferred balls from bag I to bag II are red.
E2 = both transferred balls from bag I to bag II are black.
E3 = out of two transferred balls one is red and other is black.
A = drawing a red ball from bag II.
E2
Here, P d n is required.
A
3C 4C
3! 2!×5! 1 4! 2!×5! 2
Now, P (E1) = 7 2 = × = ; P (E2) = 7C
2
= × =
C2 2!1! 7! 7 2 2!2! 7! 7
3 C ×4 C
1 1 3×4 2!5! 4
P (E3) = 7C = × =
2
7! 1 7

Pd E n = , Pd E n = , Pd E n =
A 6 A 4 A 5

1 11 2 11 3 11

P (E2) .P d E n
A
E
∴ P d 2 n = 2

P (E1) .P d E n + P (E2) .P d E n + P (E3) .P d E n


A A A A
1 2 3

2 4 8
× 8 77 4
= 7 11 = 77 = × =
1 6 2 4 4 5 6 8 20 77 34 17
× + × + × + +
7 11 7 11 7 11 77 77 77
4. There are three coins. One is two headed coin (having head on both faces), another is a biased
coin that comes up head 75% of the times and third is an unbiased coin. One of the three coins
is chosen at random and tossed, it shows heads, what is the probability that it was the two
headed coin? [CBSE Delhi 2009, (F) 2011]
Sol. Let E1, E2, E3 and A be event defined as:
E1 = selection of a two headed coin; E2 = selection of a biased coin
E3 = selection of an unbiased coin; A = coin shows head after tossing
1
Now, P (E1) = P (E2) = P (E3) =
3

P d E n = 1, Pd E n = Pd E n =
A A 75 3 A 1
= ,
1 2 100 4 3 2
E
Here, required probability = P d 1 n
A
By using Baye’s theorem,

P (E1) .P d E n
A 1
E ×1
Pd 1 n = 3
1
=
P (E1) .P d E n + P (E2) .P d E n + P (E3) .P d E n
A A A A 1 1 3 1 1
×1 + × + ×
1 2 3
3 3 4 3 2
1
3 1 4
= = =
c1 + + m
1 3 1 4 +3+2 9
3 4 2 4

512 Xam idea Mathematics–XII

@Cbsebookshub - Join Us on Telegram


5. An urn contains 4 white and 3 red balls. Let X be the number of red balls in a random draw of
three balls. Find the mean and variance of X. [CBSE (F) 2010]
Sol. Let X be the number of red balls in a random draw of three balls.
As there are 3 red balls, possible values of X are 0, 1, 2, 3.
3 C ×4 C 3C × 4C
0 3 4×3×2 4 1 2 3 × 6 × 6 18
P (0) = 7C
= = P (1 ) = 7C
= =
3 7 × 6 × 5 35 3 7 × 6 × 5 35

3 C ×4 C 3 C ×4 C
2 1 3 × 4 × 6 12 3 0 1×1× 6 1
P (2) = 7C
= = P (3) = 7C
= =
3 7 × 6 × 5 35 3 7 × 6 × 5 35
For calculation of Mean & Variance
X P(X) XP(X) X2P(X)
0 4/35 0 0
1 18/35 18/35 18/35
2 12/35 24/35 48/35
3 1/35 3/35 9/35
Total 1 9/7 15/7

9
Mean = / XP (X) =
7
15 81 24
Variance = / X 2 .P (X) – (/ X. P (X)) 2 = – =
7 49 49
6. A beg contains 5 red and 3 black balls and another bag contains 2 red and 6 black balls. Two
balls are drawn at random (without replacement) from one of the bags and both are found to
be red. Find the probability that balls are drawn from first bag. [CBSE 2019 (65/5/3)]
Sol.

Let E be the event of drawing two red balls.


Let B1 be the event that first bag is chosen and B2 be the event that second bag is chosen.
1 5 4 20
P (B1) = P (B2) = , P (E/B1) = × =
2 8 7 56
2 1 2
P (E/B2) = × =
8 7 56
P (B1) ×P (E/B1)
` P (B1 /E) =
P (B1) ×P (E/B1) + P (B2) P (E/B2)
1 20
×
= 2 56
1 20 1 2
× + ×
2 56 2 56
20 20 10
= = =
20 + 2 22 11

Probability 513
@Cbsebookshub - Join Us on Telegram
7. In a certain college, 4% of boys and 1% of girls are taller than 1.75 metres. Furthermore, 60%
of the students in the college are girls. A student is selected at random from the college and is
found to be taller than 1.75 metres. Find the probability that the selected student is a girl.
[CBSE (F) 2012]
Sol. Let E1, E2, A be events such that
E1 = student selected is girl; E2 = student selected is boy
A = student selected is taller than 1.75 metres.
E
Here P d 1 n is required.
A

Pd E n = , Pd E n =
60 3 40 2 A 1 A 4
Now, P (E1) = = , P (E2) = =
100 5 100 5 1 100 2 100

P (E1) .P d E n
A
E1
Pd n=
1

P (E1) .P d E n + P (E2) .P d E n
A A A
1 2

3 1 3
× 3 500 3
   = 5 100 = 500 = × =
3 1 2 4 3 8 500 11 11
× + × +
5 100 5 100 500 500
8. A factory has two machines A and B. Past record shows that machine A produced 60% of the
items of output and machine B produced 40% of the items. Further, 2% of the items produced
by machine A and 1% produced by machine B were defective. All the items are put into one
stockpile and then one item is chosen at random from this and is found to be defective. What
is the probability that it was produced by machine B? [CBSE (F) 2011]
Sol. Let E1, E2 and A be event such that
E1 = Production of items by machine A
E2 = Production of items by machine B
A = Selection of defective items.

= , Pd E n = , Pd E n =
60 3 40 2 A 2 1 A 1
P (E1) = = , P (E2) = =
100 5 100 5 1 100 50 2 100
E
P d 2 n is required
A
By Baye’s theorem
P (E2) .P d E n
A
E
P d 2 n =
2

P (E1) .P d E n + P (E2) .P d E n
A A A
1 2

2 1 2
× 2 500 1
= 5 100 = 500 = × =
3 1 2 1 3 2 500 6 + 2 4
× + × +
5 50 5 100 250 500

9. A bag I contains 5 red and 4 white balls and a bag II contains 3 red and 3 white balls. Two balls
are transferred from the bag I to the bag II and then one ball is drawn from the bag II. If the
ball drawn from the bag II is red, then find the probability that one red and one white ball are
transferred from the bag I to the bag II. [CBSE Sample Paper 2016]

514 Xam idea Mathematics–XII

@Cbsebookshub - Join Us on Telegram


Sol. Let E1, E2, E3 and A be event such that
E1 = Both transferred balls from bag I to bag II are red.
E2 = Both transferred balls from bag I to bag II are white.
E3 = Out of two transferred balls one is red and other is white.
A = Drawing a red ball from bag II
5C I II
2 5×4 20 5
P (E1) = 9C = = =
2 9×8 72 18
5 Red 3 Red
4C
2 4 × 3 12 3
P (E2) = 9C = = =
2 9 × 8 72 18 4 White 3 White
5 C ×4 C
1 1 5 × 4 × 2 40 10
P (E3) = 9C = = =
2 9×8 72 18

P d E n = ; P d E n = ; P d E n =
A 5 A 3 A 4
1 8 2 8 3 8
E3
We require P d n.
A
Now, by Baye’s theorem

P (E3) .P d E n
A
E
Pd 3 n =
3

P (E1) .P d E n + P (E2) .P d E n + P (E3) .P d E n
A A A A
1 2 3
10 4 40
× 40 144 20
= 18 8 = 144 = × =
5 5 3 3 10 4 25 9 40 144 74 37
× + × + × + +
18 8 18 8 18 8 144 144 144

PROFICIENCY EXERCISE
QQ Objective Type Questions: [1 mark each]
1.
Choose and write the correct option in the following questions.
3 4
(i) If A and B are two independent events with P (A) = and P (B) = , then P (Al + Bl ) equals
5 9
4 8 1 2
(a) (b) (c) (d)
15 45 3 9
(ii) If A and B are two events and A ! z, B ! z, then
P (A + B)
(a) P (A | B) = P (A) . P (B) (b) P (A | B) =
P (B)
(c) P (A | B) . P (B | A) = 1 (d) P (A | B) = P (A) | P (B)
(iii) If two events are independent then
(a) they must be mutually exclusive
(b) the sum of their probabilities must be equal to 1
(c) both (a) and (b) are correct.
(d) None of the above
(iv) If the events A and B are independent, then P (A + B) equals
(a) P(A) + P(B) (b) P(A) – P(B) (c) P(A) . P(B) (d) P(A)/P(B)

Probability 515
@Cbsebookshub - Join Us on Telegram
(v) For the following probability distribution:

X 1 2 3 4

P(X) 1 1 3 2
10 5 10 5
E(X2) is equal to

(a) 3 (b) 5 (c) 7 (d) 10
(vi) Eight coins are tossed together. The probability of getting exactly 3 heads is

1 7 5 3
(a) (b) (c) (d)
256 32 32 32
(vii) A card is picked at random from a pack of 52 playing cards. Given that the picked card is a
queen, the probability of this card to be a card of spade is [CBSE 2020 (65/5/1)]

1 4 1 1
(a) (b) (c) (d)
3 13 4 2
(viii) A die is thrown once. Let A be the event that the number obtained is greater than 3. Let B be
the event that the number obtained is less than 5. Then P (A , B) is [CBSE 2020 (65/5/1)]

2 3
(a) (b) (c) 0 (d) 1
5 5
2.
Fill in the blanks.
(i) If P (A) = 0.8, P (B) = 0.5, P (B/A) = 0.4 then the value of P(A/B) is equal to ____________.
(ii) The probability distribution of a random variable X, is given below:

X 0 1 2

P(X) 0.2 0.5 k

then the value of k is ____________.


(iii) If the mean and variance of a binomial variate X are 2 and 1 respectively, then P (X $ 1) =
____________.

QQ Very Short Answer Questions: [1 mark each]


3. Two coins are tossed. What is the probability of coming up two heads if it is known that at least
one head comes up?
4. Four cards are drawn from 52 cards with replacement. Find the probability of getting at least
3 aces.
5. A bag contains 5 white and 4 red balls. 2 balls are drawn from the bag. Find the probability that
both balls are white.
1
6. The probability that Hari hits a target is . He fires 64 times. Find the expected number (µ) of
4
times he will hit the target and also the variance ( v 2 ).
7. If P(A) = 0.4, P(B) = p, P(A , B) = 0.6 and A and B are given to be independent events, find the
value of ‘p’.  [CBSE (F) 2017]

QQ Short Answer Questions–I: [2 marks each]


8. Find the binomial distribution of which the mean is 4 and variance is 3.

516 Xam idea Mathematics–XII

@Cbsebookshub - Join Us on Telegram


1 7 1
9. Events A and B are such that P(A) = , P(B) = and P (not A or not B) = . State whether A
2 12 4
and B are independent.

10. Find the mean of the binomial distribution B c 4, m.


1
3

and P (A , B) = then find P c m + P c m .


3 2 3 B A
11. If P (A) = , P (B) =
10 5 5 A B
12. In a girl’s hostel’s, 60% of the students read Hindi newspaper, 40% read English newspaper and
20% read both Hindi and English newspaper. A student is selected at random. If she reads English
newspaper, then find the probability that she reads Hindi newspaper also.
13. The probability of simultaneous occurrence of atleast one of two events A and B is p. If the
probability that exactly one of A, B occurs is q, then prove that P (A) + P (B) = 2 – 2p + q .

QQ Short Answer Questions–II: [3 marks each]


14. A and B throw a pair of dice alternately. A wins the game if he gets a total of 7 and B wins the game
if he gets a total of 10. If A starts the game, then find the probability that B wins. [CBSE Delhi 2016]
15. A random variable X has the following probability distribution:
X 0 1 2 3 4 5 6
P(X) C 2C 2C 3C C2 2C2 7C2 + C

Find the value of C and also calculate mean of the distribution. [CBSE (East) 2016]
16. A and B throw a pair of dice alternately, till one of them gets a total of 10 and wins the game. Find
their respective probabilities of winning, if A starts first. [CBSE (North) 2016]
17. A box has 20 pens of which 2 are defective. Calculate the probability that out of 5 pens drawn one
by one with replacement, at most 2 are defective. [CBSE (F) 2016]
18. Let, X denote the number of colleges where you will apply after your results and P(X = x) denotes
your probability of getting admission in x number of colleges. It is given that
]Z] kx, if x = 0 or 1
]]
] 2kx , if x = 2
P (X = x) = ][
]] k (5 – x), if x = 3 or 4
]]
0 if x 2 4
\
where k is a positive constant. Find the value of k. Also, find the probability that you will get
admission in (i) exactly one college (ii) at most 2 colleges (iii) at least 2 colleges. [CBSE (F) 2016]
19. There are 4 cards numbered 1, 3, 5 and 7, one number on one card. Two cards are drawn at
random without replacement. Let X denote the sum of the numbers on the two drawn cards. Find
the mean and variance of X.[CBSE AI 2017]
20. P speaks truth in 70% of the cases and Q in 80% of the cases. In what percent of cases are they
likely to agree in stating the same fact?  [CBSE Delhi 2013]
21. In a group of 50 scouts in a camp, 30 are well trained in first aid techniques while the remaining
are well trained in hospitality but not in first aid. Two scouts are selected at random from the
group. Find the probability distribution of number of selected scouts who are well trained in first
aid. Find the mean of the distribution also. [CBSE (F) 2013]
22. Of the students in a school; it is known that 30% have 100% attendance and 70% students are
irregular. Previous year results report that 70% of all students who have 100% attendance attain
A grade and 10% irregular students attain A grade in their annual examination. At the end of the
year, one student is chosen at random from the school and he has A grade. What is the probability
that the student has 100% attendance? [CBSE (AI) 2017]

Probability 517
@Cbsebookshub - Join Us on Telegram
23. How many times should a man toss a fair coin so that the probability of having at least one head
is more than 90%?
24. Three numbers are selected at random (without replacement) from first six positive integers. Let
X denote the largest of the three numbers obtained. Find the probability distribution of X. Also,
find the mean and variance of the distribution. [CBSE (North) 2016]
25. Suppose 10000 tickets are sold in a lottery each for `1. First prize is of `3000 and the second prize
is of `2000. There are three third prizes of `500 each. If you buy one ticket, then what is your
expectation? [NCERT Exemplar]
1 2
26. The probability that A hits a target is and the probability that B hits it is . If each one of A
3 5
and B shoots at the target, what is the probability that
(i) the target is hit? (ii) exactly one of them hits the target? [CBSE (F) 2009]
27. A bag X contains 4 white balls and 2 black balls, while another bag Y contains 3 white balls and
3 black balls. Two balls are drawn (without replacement) at random from one of the bags and
were found to be one white and one black. Find the probability that the balls were drawn from
bag Y. [CBSE (North) 2016]
3 5
28. The probabilities of two students A and B coming to the school in time are and respectively.
7 7
Assuming that the events, ‘A coming in time’ and ‘B coming in time’ are independent, find the
probability of only one of them coming to the school in time. [CBSE (AI) 2013]

QQ Long Answer Questions: [5 marks each]


29. Find the probability distribution of the random variable X, which denotes the number of doublets
in four throws of a pair of dice.
Hence, find the mean of the number of doublets (X). [CBSE 20120 (65/2/1)]
30. An insurance company insured 3,000 scooters, 4,000 cars and 5,000 trucks. The probabilities
of the accident involving a scooter, a car and a truck are 0·02, 0·03 and 0·04 respectively. One
of the insured vehicles meet with an accident. Find the probability that it is a (a) scooter
(b) car (c) truck. [CBSE (AI) 2008]
31. In a bolt factory, three machines A, B and C manufacture 25, 35 and 40 per cent of the total bolts
manufactured. Of their output, 5, 4 and 2 per cent are defective respectively. A bolt is drawn at
random and is found to be defective. Find the probability that it was manufactured by either
machine A or C. [CBSE Allahabad 2015]
32. In a factory which manufactures bolts, machines A, B and C manufacture respectively 30%, 50%
and 20% of the bolts. Of their outputs 3, 4, 1 per cent respectively are defective bolts. A bolts is
drawn at random from the product and is found to be defective. Find the probability that this is
not manufactured by machine B. [CBSE Allahabad 2015]
33. Find the mean, the variance and the standard deviation of the number of doublets in three throws
of a pair of dice. [CBSE Sample Paper 2016]
34. A, B and C throw a pair of dice in that order alternately till one of them gets a total of 9 and wins
the game. Find their respective probabilities of winning, if A starts first. [CBSE (East) 2016]
35. Bag A contains 3 red and 5 black balls, while bag B contains 4 red and 4 black balls. Two balls are
transferred at random from bag A to bag B and then a ball is drawn from bag B at random. If the
ball drawn from bag B is found to be red, find the probability that two red balls were transferred
from A to B. [CBSE (F) 2016]
36. An urn contains 3 white and 6 red balls. Four balls are drawn one by one with replacement from
the urn. Find the probability distribution of the number of red balls drawn. Also find mean and
variance of the distribution. [CBSE Delhi2016]

518 Xam idea Mathematics–XII

@Cbsebookshub - Join Us on Telegram


37. Five bad oranges are accidently mixed with 20 good ones. If four oranges are drawn one by one
successively with replacement, then find the probability distribution of number of bad oranges
drawn. Hence find the mean and variance of the distribution. [CBSE (Central) 2016]
2 1
38. If A and B are two independent events such that P ( A + B) = and P (A + B ) = , then find
15 6
P(A) and P(B). [CBSE Delhi 2015]
39. An urn contains 5 red and 2 black balls. Two balls are randomly drawn, without replacement. Let
X represent the number of black balls drawn. What are the possible values of X? Is X a random
variable? If yes, find the mean and variance of X. [CBSE Ajmer 2015]
40. A man is known to speak truth 3 out of 5 times. He throws a die and reports that it is 4. Find the
probability that it is actually a 4. [CBSE Bhubaneswar 2015]
41. There are two boxes I and II. Box I contains 3 red and 6 black balls. Box II contains 6 red and ‘n’ black
balls. One of the two boxes, box I and box II is selected at random and a ball is drawn at random.
3
The ball drawn is found to be red. If the probability that this red ball comes out from box II is ,
5
find the value of ‘n’.  [CBSE 2019 (65/3/1)]
42. A girl throws a die. If she gets a 5 or 6, she tosses a coin three times and notes the number of
heads. If she gets 1, 2, 3 or 4, she tosses a coin two times and notes the number of heads obtained.
If she obtained exactly two heads, what is the probability that she threw 1, 2, 3 or 4 with the die?
 [CBSE Delhi 2012]
43. A bag contains 4 balls. Two balls are drawn at random and are found to be white. What is the
probability that all balls are white? [CBSE (AI) 2010]
44. Two groups are competing for the position on the Board of Directors of a corporation. The
probabilities that the first and the second group will win are 0. 6 and 0 .4 respectively. Further, if
the first group wins, the probability of introducing a new product is 0.7 and the corresponding
probability is 0.3, if the second group wins. Find the probability that the new product was
introduced by the second group. [CBSE Delhi 2009]

Answers
1. (i) (d) (ii) (b) (iii) (d) (iv) (c) (v) (d) (vi) (b)
(vii) (c) (viii) (d)
15 1 49 5
2. (i) 0.64 (ii) 0.3 (iii) 3. 4. 5.
16 3 28561 18
, q = , n = 16; B c16, m
1 1 3 1
6. 16, 12 7. 8. p = 9. No
3 4 4 4
4 7 1 5 1
10. 11. 12. 14. 15. C = ; 2.66
3 12 2 17 10

12 11 99144 1 1 5 7
16. , 17. 18. k = ; (i) 8 (ii) 8 (iii) 8
23 23 100000 8

20 31 X 0 1 2 294
19. 8; 20. or 62% 21. ; mean =
3 50 245
P(X) 38 120 87
245 245 245
3
22. 23. n $ 4
4

Probability 519
@Cbsebookshub - Join Us on Telegram
24.
X 3 4 5 6

P(X) 1 3 6 10
20 20 20 20

Mean = 5.25, variance = 0.79


3 7 9 26
25. 0.65 26. (i) (ii) 27. 28.
5 15 17 49
29.
X 0 1 2 3 4
2
625 500 150 20 1 ; Mean =
P(X) 3
1296 1296 1296 1296 1296
3 6 10
30. (a) (b) (c)
19 19 19
41 11 1 5 15
31. 32. 33. Mean = ; Var = ; SD =
69 31 2 12 6

81 72 64 18
34. , , 35.
217 217 217 33
8 8 X or x 0 1 2 3 4
36. Mean = Var =
3 9
P(X) or 1 8 24 32 16
p(x) 81 81 81 81 81

X 0 1 2 3 4
4 16
37. Mean = , Var =
5 25 256 256 96 16 1
P(X)
625 625 625 625 625

1 1
38. P (A) =
, P (B) =
5 6
Hint: Apply P (A ∩ B) = P (A).P(B)

P (A ∩ B) = P (A). P (B)
P (A ∩ B) = P (A) . P (B)
39. X is random variable and have values 0, 1, 2

4 50
Mean = and variance =
7 147
5C 2 C ×5 C 2C
2 1 1 2
Hint: P (X = 0) =
7C , P (X = 1) = 7C , P (X = 2) = 7C
2 2 2
3
40.
13
Hint: Let E1 = 4 occurs; E2 = 4 does not occur; A = man report that 4 occurs.

1 5 3 2
P (E1) = , P (E2) = , P (A/E1) = , P (A/E2) =
6 6 5 5
P (E1 / A) = required
41. n = 5
4 3 2
42. 43. 44.
7 5 9

520 Xam idea Mathematics–XII

@Cbsebookshub - Join Us on Telegram


SELF-ASSESSMENT TEST
Time allowed: 1 hour Max. marks: 30

1. Choose and write the correct option in the following questions. (4 × 1 = 4)


(i) If two events are independent, then
(a) they must be mutually exclusive
(b) the sum of their probabilities must be equal to 1
(c) both (a) and (b) are correct
(d) none of the above is correct
(ii) Three person A, B and C, fire at a target in turn, starting with A. Their probability of hitting

the target are 0.4, 0.3 and 0.2, respectively. The probability of two hits is

(a) 0.025 (b) 0.188 (c) 0.339 (d) 0.475


(iii) In a college, 30% students fail in Physics, 25% fail in Mathematics and 10% fail in both. One

student is chosen at random. The probability that she fails in Physics, if she has failed in
Mathematics is
3 2 7 1
(a) (b) (c) (d)
10 5 90 3

(iv) In a box containing 100 bulbs, 10 are defective. The probability that out of a sample of 5 bulbs,

none is defective is
1 5 9 5
(a) 10–1 (b) c m (c) c m (d)
9
2 10 10

2.
Fill in the blanks. (2 × 1 = 2)
(i) Two dice are thrown. The probability of getting an odd number on first and a multiple of 3 on the
other die, is ____________.
(ii) If A and Bl are independent events then P (Al , B) = 1 – ____________.

QQ Solve the following questions. (2 × 1 = 2)


3. Given that E and F are events such that P(E) = 0.6, P(F) = 0.3 and P(E ∩ F) = 0.2, find P(E/F) and
P(F/E).
4. Compute P(A/B) if P(B) = 0.5 and P(A ∩ B) = 0.32.

QQ Solve the following questions. (4 × 2 = 8)


A
5. Given that P ^ A h = 0.4, P ^B h = 0.2 and P c m = 0.5 . Find P(A ∪ B).
B
6. 10% of the bulbs produced in a factory are of red colour and 2% are red and defective. If one bulb
is picked up at random, determine the probability of its being defective if it is red.
1 7 1
7. Events A and B are such that P(A) = , P(B) = and P (not A or not B) = . State whether A
2 12 4
and B are independent.
8. The probability of simultaneous occurrence of atleast one of two events A and B is p. If the
probability that exactly one of A, B occurs is q, then prove that P (A) + P (B) = 2 – 2p + q .

Probability 521
@Cbsebookshub - Join Us on Telegram
QQ Solve the following questions. (3 × 3 = 9)
9. A and B throw a pair of dice alternately, till one of them gets a total of 10 and wins the game. Find
their respective probabilities of winning, if A starts first.
10. There are 4 cards numbered 1, 3, 5 and 7, one number on one card. Two cards are drawn at
random without replacement. Let X denote the sum of the numbers on the two drawn cards. Find
the mean and variance of X.
11. Of the students in a school; it is known that 30% have 100% attendance and 70% students are
irregular. Previous year results report that 70% of all students who have 100% attendance attain
A grade and 10% irregular students attain A grade in their annual examination. At the end of the
year, one student is chosen at random from the school and he has A grade. What is the probability
that the student has 100% attendance?
QQ Solve the following question. (1 × 5 = 5)

12. Five bad oranges are accidently mixed with 20 good ones. If four oranges are drawn one by one
successively with replacement, then find the probability distribution of number of bad oranges
drawn. Hence find the mean and variance of the distribution.

Answers
1. (i) (d) (ii) (b) (iii) (b) (iv) (c)
1
2. (i) (ii) P (A) .P (Bl )
6
2 1 16 1 12 11
3. , 4. 5. 0.7 6. 7. No 9. ,
3 3 25 5 23 23
20 3 4 16
10. Mean = 8, Variance = 11. 12. Mean = , Var = ,
3 4 5 25

X 0 1 2 3 4
256 256 96 16 1
P(X)
625 625 625 625 625

zzz

522 Xam idea Mathematics–XII

@Cbsebookshub - Join Us on Telegram


PART–B

COMPETENCY-BASED
QUESTIONS
(CASE STUDY BASED QUESTIONS)

Mathematics
@Cbsebookshub - Join Us on Telegram
@Cbsebookshub - Join Us on Telegram
Case Study Based
Questions

Chapter-1: Relations and Functions


1. Read the following and answer any four questions from (i) to (v).
A general election of Lok Sabha is a gigantic exercise. About 911 million people were eligible to

vote and voter turnout was about 67%, the highest ever

ONE – NATION
ONE – ELECTION
FESTIVAL OF
DEMOCRACY
GENERAL ELECTION– 2019
MY VOTE
MY VOICE

Let I be the set of all citizens of India who were eligible to exercise their voting right in general
election held in 2019. A relation ‘R’ is defined on I as follows:
R = {(V1, V2) : V1, V2 ∈ I and both use their voting right in general election – 2019}

[CBSE Question Bank]

Answer the questions given below.
(i) Two neighbours X and Y ∈ I. X exercised his voting right while Y did not cast her vote in
general election – 2019. Which of the following is true?
(a) (X, Y) ∈ R (b) (Y, X) ∈ R
(c) (X, X) ∉ R (d) (X, Y) ∉ R
(ii) Mr.‘X’ and his wife ‘W’ both exercised their voting right in general election -2019, Which
of the following is true?
(a) both (X,W) and (W,X) ∈ R (b) (X,W) ∈ R but (W,X) ∉ R
(c) both (X,W) and (W,X) ∉ R (d) (W,X) ∈ R but (X,W) ∉ R
(iii) Three friends F1, F2 and F3 exercised their voting right in general election- 2019, then
which of the following is true?
(a) (F1, F2 ) ∈ R, (F2, F3) ∈ R and (F1, F3) ∈ R
(b) (F1, F2 ) ∈ R, (F2, F3) ∈ R and (F1, F3) ∉ R
(c) (F1, F2 ) ∈ R, (F2, F2) ∈R but (F3, F3) ∉ R
(d) (F1, F2 ) ∉ R, (F2, F3) ∉ R and (F1, F3) ∉ R

Case Study Based Questions 525


@Cbsebookshub - Join Us on Telegram

(iv) The above defined relation R is
(a) Symmetric and transitive but not reflexive
(b) Universal relation
(c) Equivalence relation
(d) Reflexive but not symmetric and transitive
(v) Mr. Shyam exercised his voting right in General Election – 2019, then Mr. Shyam is
related to which of the following?
(a) All those eligible voters who cast their votes
(b) Family members of Mr.Shyam
(c) All citizens of India
(d) Eligible voters of India
Sol. We have a relation 'R' is defined on I as follows:
R = {V1, V2} : V1, V2 ∈ I and both use their voting right in general election – 2019}

(i) Two neighbors X and Y ∈ I. Since X exercised his voting right while Y did not cast her vote
in general election – 2019
Therefore, (X, Y) ∉ R

∴ Option (d) is correct.

(ii) Since Mr. 'X' and his wife 'W' both exercised their voting right in general election – 2019.
∴ Both (X, W) and (W, X) ∈ R.

∴ Option (a) is correct.

(iii) Since three friends F1, F2 and F3 exercised their voting right in general election – 2019,
therefore
(F1, F2) ∈ R, (F2, F3) ∈ R and (F1, F3) ∈ R

∴ Option (a) is correct.

(iv) This relation is an equivalence relation.
∴ Option (c) is correct.

(v) Mr. Shyam exercised his voting right in General election – 2019, then Mr. Shyam is related
to all those eligible votes who cast their votes.
∴ Option (a) is correct.

2. Read the following and answer any four questions from (i) to (v).
Sherlin and Danju are playing Ludo at home during Covid-19. While rolling the dice, Sherlin’s

sister Raji observed and noted the possible outcomes of the throw every time belongs to set
{1,2,3,4,5,6}. Let A be the set of players while B be the set of all possible outcomes.

A = {S, D}, B = {1,2,3,4,5,6}


[CBSE Question Bank]
526 Xam idea Mathematics–XII

@Cbsebookshub - Join Us on Telegram


Answer the questions given below.
(i) Let R : B → B be defined by R = {(x, y): y is divisible by x} is
(a) Reflexive and transitive but not symmetric
(b) Reflexive and symmetric and not transitive
(c) Not reflexive but symmetric and transitive
(d) Equivalence
(ii) Raji wants to know the number of functions from A to B. How many number of functions
are possible?
(a) 62 (b) 26 (c) 6! (d) 212

(iii) Let R be a relation on B defined by R = {(1,2), (2,2), (1,3), (3,4), (3,1), (4,3), (5,5)}. Then R is
(a) Symmetric (b) Reflexive (c) Transitive (d) None of these three

(iv) Raji wants to know the number of relations possible from A to B. How many numbers of
relations are possible?
(a) 62 (b) 26 (c) 6! (d) 212
(v) Let R : B → B be defined by R={(1,1),(1,2), (2,2), (3,3), (4,4), (5,5),(6,6)}, then R is
(a) Symmetric (b) Reflexive and Transitive
(c) Transitive and symmetric (d) Equivalence
Sol. (i) Given R : B → B be defined by
R = {(x, y) : y is divisible by x}

Reflexive : Let x ∈ B, since x always divide x itself.
∴ (x, x) ∈ R
It is reflexive.
Symmetric : Let x, y ∈ B and let (x, y) ∈ R.
⇒ y is divisible by x.
y
⇒ x = k1 , where k1 is an integer.
x 1
⇒ y = ! integer.
k1
∴ (y, x) ∉ R
It is not symmetric.
Transitive : Let x, y, z ∈ B and
y
let (x, y) ∈ R ⇒ x = k1 , where k1 is an integer.
z
and, (y, z) ∈ R ⇒ y = k2 , where k2 is an integer.
y z
∴ x × y = k1 .k2 = k (integer)

z
⇒ x =k ⇒ (x, z) ∈ R
It is transitive.
Hence, relation is reflexive and transitive but not symmetric.
∴ Option (a) is correct.

(ii) We have,
A = { S, D} ⇒ n(A) = 2
and, B = {1, 2, 3, 4, 5, 6} ⇒ n(B) = 6

Case Study Based Questions 527


@Cbsebookshub - Join Us on Telegram
∴ Number of functions from A to B is 62.


Option (a) is correct.
(iii) Given,
R be a relation on B defined by
R = {(1, 2), (2, 2), (1, 3), (3, 4), (3, 1), (4, 3), (5, 5)}
R is not reflexive since (1, 1), (3, 3), (4, 4) ∉ R

R is not symmetric as (1, 2) ∈ R but (2, 1) ∉ R

and, R is not transitive as (1, 3) ∈ R and (3, 1) ∈ R but (1, 1) ∉ R

∴ R is neither reflexive nor symmetric nor transitive.

∴ Option (d) is correct.

(iv) Total number of possible relations from A to B = 212
∴ Option (d) is correct.

(v) Given R : B → B be defined by R = {(1, 1), (1, 2), (2, 2), (3, 3), (4, 4), (5, 5), (6, 6)}
∴ R is reflexive as each elements of B is related to itself and R is also transitive as (1, 2) ∈ R
and (2, 2) ∈ R ⇒ (1, 2) ∈ R.
∴ R is reflexive and transitive.

∴ Option (b) is correct.

3. Read the following and answer any four questions from (i) to (v).
An organization conducted bike race under 2 different categories-boys and girls. In all, there

were 250 participants. Among all of them finally three from Category 1 and two from Category 2
were selected for the final race. Ravi forms two sets B and G with these participants for his college
project.
Let B = {b1, b2, b3} G={g1, g2} where B represents the set of boys selected and G the set of girls who
were selected for the final race. [CBSE Question Bank]

Ravi decides to explore these sets for various types of relations and functions

Answer the questions given below.


(i) Ravi wishes to form all the relations possible from B to G. How many such relations are
possible?
(a) 26 (b) 25 (c) 0 (d) 23

528 Xam idea Mathematics–XII

@Cbsebookshub - Join Us on Telegram


(ii) Let R : B → B be defined by R = {(x, y) : x and y are students of same sex}, Then this relation
R is
(a) Equivalence (b) Reflexive only
(c) Reflexive and symmetric but not transitive
(d) Reflexive and transitive but not symmetric

(iii) Ravi wants to know among those relations, how many functions can be formed from
B to G?
(a) 22 (b) 212 (c) 32 (d) 23
(iv) Let R : B → G be defined by R = { (b1, g1), (b2, g2), (b3,g1)}, then R is

(a) Injective (b) Surjective
(c) Neither Surjective nor Injective (d) Surjective and Injective
(v) Ravi wants to find the number of injective functions from B to G. How many numbers of
injective functions are possible?
(a) 0 (b) 2! (c) 3! (d) 0!
Sol. We have sets
B = {b1, b2, b3}, G = {g1, g2}
⇒ n(B) = 3 and n(G) = 2

(i) Number of all possible relations from B to G = 23 × 2 = 26
∴ Option (a) is correct.

(ii) Given relation R = {(x, y) : x and y are student of same sex}
On the set B.
Since the set is B = {b1, b2, b3} = all boys
∴ It is an equivalence relation.

∴ Option (a) is correct.

(iii) We have,
B = {b1, b2, b3} ⇒ n(B) = 3
G = {g1, g2} ⇒ n(G) = 2
∴ Total no. of possible functions from B to G = 23

∴ Option (d) is correct.

(iv) We have,
R : B → G be defined by
R = {(b1, g1), (b2, g2), (b3, g1)}
It is not injective because (b1, g1) ∈ R and (b3, g1) ∈ R
So b1 ≠ b3 ⇒ same image g1.

It is surjective because its Co-domain = Range.
∴ R is surjective.

∴ Option (b) is correct.

(v) Since R is not injective therefore number of injective functions = 0.
∴ Option (a) is correct.

4. Read the following and answer any four questions from (i) to (v).
Students of Grade 9, planned to plant saplings along straight lines, parallel to each other to one
side of the playground ensuring that they had enough play area. Let us assume that they planted

Case Study Based Questions 529


@Cbsebookshub - Join Us on Telegram
one of the rows of the saplings along the line y = x − 4. Let L be the set of all lines which are
parallel on the ground and R be a relation on L. [CBSE Question Bank]

Answer the questions given below.


(i) Let relation R be defined by R = {(L1, L2): L1 || L2 where L1, L2 dL} then R is______ relation
(a) Equivalence (b) Only reflexive
(c) Not reflexive (d) Symmetric but not transitive
(ii) Let R = {(L1, L2): L1 ⊥ L2 where L1, L2 dL} which of the following is true?
(a) R is Symmetric but neither reflexive nor transitive.
(b) R is Reflexive and transitive but not symmetric.
(c) R is Reflexive but neither Symmetric nor transitive.
(d) R is an Equivalence relation.
(iii) The function f : R → R defined by f(x) = x − 4 is

(a) Bijective (b) Surjective but not injective
(c) Injective but not Surjective (d) Neither Surjective nor Injective
(iv) Let f : R → R be defined by f(x) = x − 4. Then the range of f(x) is

(a) R (b) Z (c) W (d) Q
(v) Let R = {(L1 , L2 ) : L1 is parallel to L2 and L1 : y = x – 4} then which of the following can be
taken as L2 ?
(a) 2x – 2y + 5 = 0 (b) 2x + y = 5 (c) 2x + 2y + 7 = 0 (d) x + y = 7
Sol. (i) Given relation R defined by
R = {(L1, L2) : L1 || L2 where L1, L2 ∈ L}
Reflexive : Let L1 ∈ L ⇒ L1 || L1 ⇒ (L1, L1) ∈ R.
It is reflexive.
Symmetric : Let L1, L2 ∈ L and let (L1, L2) ∈ R.
⇒ L1 || L2 ⇒ L2 || L1
⇒ (L2, L1 ) ∈ R
It is symmetric.
Transitive : Let L1, L2, L3 ∈ L.

and, let (L1, L2) ∈ R and (L2, L3) ∈ R

∴ L1 || L2 and L2 || L3

530 Xam idea Mathematics–XII

@Cbsebookshub - Join Us on Telegram


⇒ L1 || L3 ⇒ ( L1, L3) ∈ R
∴ It is transitive.
Hence R is an equivalence relation.

∴ Option (a) is correct.

(ii) Given relation R defined by R = {(L1, L2) : L1 ⊥ L2 where L1, L2 ∈ L}
Reflexive : Since every line is not perpendicular to itself.
∴ (L1, L1) ∉ R
It is not reflexive.
Symmetric : Let L1, L2 ∈ L and let (L1, L2) ∈ R
⇒ L1 ⊥ L2 ⇒ L2 ⊥ L1
⇒ (L2, L1 ) ∈ R
It is symmetric.
Transitive : Let L1, L2, L3 ∈ L
and, let (L1, L2) ∈ R and (L2, L3) ∈ R
∴ L1 ⊥ L2 and L2 ⊥ L3
⇒ L1 || L3
⇒ ( L1, L3) ∉ R
∴ It is not transitive.
Hence relation R is symmetric but neither reflexive nor transitive.

∴ Option (a) is correct.

(iii) Given function f : R → R defined by f(x) = x – 4
Injective : Let x1, x2 ∈ R such that x1 ≠ x2.

⇒ x1 – 4 ≠ x2 – 4 f(x1) ≠ f(x2)
It is injective.
Surjective : Let y = x – 4 ⇒ x = y + 4
for every y ∈ R there exists x ∈ R.
i.e, Co-domain = Range
∴ It is surjective.
Hence given function is bijective.
∴ Option (a) is correct.

(iv) Given function f : R → R defined by f(x) = x – 4

Let y = f(x) ⇒ y=x–4 ⇒ x=y+4
⇒ x∈R ⇒ y∈R
∴ Range of f(x) is R (Set of real numbers).

∴ Option (a) is correct.

(v) Option (a) is correct choice, because the equation of line 2x – 2y + 5 = 0 is parallel to y = x – 4.
∴ Option (a) is correct.

zzz

Case Study Based Questions 531


@Cbsebookshub - Join Us on Telegram
Chapter-2: Inverse Trigonometric Functions
1. Read the following and answer any four questions from (i) to (v).

Two men on either side of a temple 30 meters high observe its top at the angles of elevation α and
β respectively. (as shown in the figure above). The distance between the two men is 40 3 metres
and the distance between the first person A and the temple is 30 3 metres. Based on the above
information answer the following: [CBSE Question Bank]
Answer the questions given below.
(i) ∠CAB = α =
2 1 3
(a) sin –1 e o (b) sin –1 c m (c) sin–1(2) (d) sin –1 d n
3 2 2
(ii) ∠CAB = α =
1 2 3 4
(a) cos –1 c m (b) cos –1 c m (c) cos –1 d n (d) cos –1 c m
5 5 2 5
(iii) ∠BCA = β =

1 1
(a) tan –1 c m (b) tan–1(2) (c) tan –1 e o (d) tan –1 ^ 3 h
2 3
(iv) ∠ABC =

r r r r
(a) (b) (c) (d)
4 6 2 3
(v) Domain and Range of cos–1 x are respectively
(a) (–1, 1), (0, π) (b) [–1, 1], (0, π) (c) [–1, 1], [0, π] (d) ^ –1, 1h, ;– , E
r r
2 2
Sol. We have,
B

30 m

α β

A D C
30 √3m 10 √3m

(i) Now in ∆ABD(right angled)


BD 30 1 1
tan a =
= = ⇒ tan a = = tan 30°
AD 30 3 3 3

532 Xam idea Mathematics–XII

@Cbsebookshub - Join Us on Telegram


⇒ α = 30°
1
⇒ sin α = sin 30° =
2
1
⇒ a = sin –1 c m
2
∴ Option (b) is correct.
3
(ii) We have from (i) α = 30° ⇒ cos a = cos 30° =
2
3
⇒ a = cos –1 d n
2
∴ Option (c) is correct.

(iii) In right ∆BCD, we have
BD 30 3
tan b = ⇒ tan b = = = 3
DC 10 3 3
⇒ b = tan –1 ^ 3 h
∴ Option (d) is correct.

(iv) In ∆ABC, we have,
∠ABC + ∠BAC + ∠ACB = 180°
⇒ ∠ABC + α + β = 180°
⇒ ∠ABC + 30° + 60° = 180° [From (iii) b = tan –1 ( 3 ) = 60° ]
⇒ ∠ABC = 90°
r
⇒ +ABC =
2

Option (c) is correct.
(v) Let cos–1 x = y ⇒ x = cos y
–1 ≤ cos y ≤ 1 ⇒
–1 ≤ x ≤ 1 ⇒ Domain = [–1, 1]
0≤y≤π
⇒ Range = [0, π].
∴ Option (c) is correct.

2. Read the following and answer any four questions from (i) to (v).
The Government of India is planning to fix a hoarding board at the face of a building on the road

of a busy market for awareness on COVID-19 protocol. Ram, Robert and Rahim are the three
engineers who are working on this project. “A” is considered to be a person viewing the hoarding
board 20 metres away from the building, standing at the edge of a pathway nearby. Ram, Robert
and Rahim suggested to the firm to place the hoarding board at three different locations namely
C, D and E. “C” is at the height of 10 metres from the ground level. For the viewer A, the angle of
elevation of “D” is double the angle of elevation of “C” The angle of elevation of “E” is triple the
angle of elevation of “C” for the same viewer. Look at the figure given and based on the above
information answer the following: [CBSE Question Bank]

10m

A' 5m A 20m B

Case Study Based Questions 533


@Cbsebookshub - Join Us on Telegram
Answer the questions given below.
(i) Measure of ∠CAB =
1
(a) tan–1(2) (b) tan –1 c m (c) tan–1(1) (d) tan–1(3)
2
(ii) Measure of ∠DAB =
3 4
(a) tan –1 c m (b) tan–1(3) (c) tan –1 c m (d) tan–1(4)
4 3
(iii) Measure of ∠EAB =
2 11
(a) tan–1(11) (b) tan–1(3) (c) tan –1 c m (d) tan –1 c m
11 2
(iv) A' Is another viewer standing on the same line of observation across the road. If the
width of the road is 5 metres, then the difference between ∠CAB and ∠CA'B is
1 1 2 11
(a) tan –1 c m (b) tan –1 c m (c) tan –1 c m (d) tan –1 c m
12 8 5 21
(v) Domain and Range of tan–1 x are respectively
(a) R , c – , m (b) R –, c – , m (c) R, c – , m (d) R, c 0, m
+ r r r r r r r
2 2 2 2 2 2 2

Sol. Let ∠CAB = α, therefore ∠DAB = 2α and ∠EAB = 3α. C


(i) In right ∆ABC, we have
BC 10 1
tan a = = =
AB 20 2 10 m
1
⇒ a = tan –1 c m
2
α
1
+CAB = tan –1 c m
A
⇒ 20 m B
2
∴ Option (b) is correct.
1
(ii) We have, tan a = (from (i)) D
2
1

2 tan a 2
∴ tan 2a = =
1 – tan 2 a 1 – 1 10 m
4
1 4
tan 2a = = 2α
3 3 A
20 m B
4
4
⇒ 2a = tan –1 c m
3
4
⇒ +DAB = tan –1 c m
3
∴ Option (c) is correct.
1
(iii) We have, tan a =
2
3 tan a – tan 3 a
∴ tan 3a =
1 – 3 tan 2 a E
1 3
3× –c m
1
2 2
=
1 2 10 m
1– 3×c m
2

A
20 m B

534 Xam idea Mathematics–XII

@Cbsebookshub - Join Us on Telegram


3 1 11

2 8 8 11 11
tan 3a = = = ×4=
3 1 8 2
1–
4 4
11 11
⇒ 3a = tan –1 c m ⇒ +EAB = tan –1 c m .
2 2
∴ Option (d) is correct.

(iv) Let ∠CA'B = β C

We have, In A'CB
BC 10
tan b = =
A'B 20 + 5
10 m
10 2
⇒ tan b = =
25 5
2
b = tan –1 c m
β α

5 A' 5m A 20 m B
2 1
⇒ +CAl B = tan –1 c m and we already know +CAB = tan –1 c m
5 2
1 2
∴ +CAB – +CAl B = tan –1 c m – tan –1 c m
2 5
KJK 1 – 2 ONO JK 5 – 4 NO
K O KK OO
–1 K 2 5 O –1 K 10 O
= tan KK O = tan KK O
KK 1 + 1 × 2 OOO KK 12 OOO
K 2 5P O K O
L L 10 P
1
= tan –1 c m
12
∴ Option (a) is correct.

(v) Domain and Range of tan–1 x are respectively (–∞, ∞) and c – , m i.e R, c – , m
r r r r
2 2 2 2
∴ Option (c) is correct.

zzz

Chapter-3: Matrices
1. Read the following and answer any four questions from (i) to (v).
A manufacturer produces three stationery products Pencil, Eraser and Sharpener which he sells

in two markets. Annual sales are indicated below

Case Study Based Questions 535


@Cbsebookshub - Join Us on Telegram
Products (in numbers)
Market
Pencil Eraser Sharpener
A 10,000 2,000 18,000
B 6,000 20,000 8,000

If the unit sale price of Pencil, Eraser and Sharpener are `2.50, `1.50 and `1.00 respectively, and
unit cost of the above three commodities are `2.00, `1.00 and `0.50 respectively, then,
[CBSE Question Bank]
Answer the questions given below.
(i) Total revenue of market A is
(a) `64,000 (b) `60,400 (c) `46,000 (d) `40,600
(ii) Total revenue of market B is
(a) `35,000 (b) `53,000 (c) `50,300 (d) `30,500

(iii) Cost incurred in market A is
(a) `13,000 (b) `30,100 (c) `10,300 (d) `31,000

(iv) Profit in market A and B respectively are
(a) (`15,000, `17,000) (b) (`17,000, `15,000)
(c) (`51,000, `71,000) (d) ( `10,000, `20,000)
(v) Gross profit in both market is
(a) `23,000 (b) `20,300 (c) `32,000 (d) `30,200
RS VWSR2.50WV
SS WWSS WW
Sol. (i) Total revenue for market A = SS10, 000 2000 18000WWSS1.50WW
S WSS W
S WS1.00WW
T XT X
= 10,000 × 2.50 + 2000 × 1.50 + 18000 × 1.00
= `46000
∴ Option (c) is correct. RS VWSR2.50WV
SS WWSS WW
(ii) Total revenue for market B = SS6, 000 20, 000 8, 000WWSS1.50WW
SS WWSS W
S1.00WW
T XT X
= 6,000 × 2.50 + 20,000 × 1.50 + 8,000 × 1.00
= `53000
∴ Option (b) is correct. RS VWSR2.00WV
SS WWSS WW
(iii) Cost incurred in market A = SS10, 000 2000 18000WWSS1.00WW
SS WWSS W
S0.50WW
T XT X
= 10,000 × 2.00 + 2000 × 1.00 + 18000 × 0.50
= `31000
∴ Option (d) is correct.

(iv) We have, RS VWRS2.00VW
SS WWSS WW
Cost incurred in market B = SS6, 000 20, 000 8, 000WWSS1.00WW
SS WWSS W
S0.50WW
T XT X
= 6,000 × 2.00 + 20,000 × 1.00 + 8,000 × 0.50
= 12,000 + 20,000 + 4000
= `36,000
∴ Profit in market A = 46,000 – 31,000 = 15,000

536 Xam idea Mathematics–XII

@Cbsebookshub - Join Us on Telegram


and profit in market B = 53,000 – 36,000 = 17,000
∴ Option (a) is correct.
(v) Gross profit = Total SP – Total CP for both market A and B.
= (46,000 + 53,000) – (31,000 + 36,000)
= 32,000
∴ Option (c) is correct.

2. Read the following and answer any four questions from (i) to (v).

Amit, Biraj and Chirag were given the task of creating a square matrix of order 2. Below are the
matrices created by them. A, B , C are the matrices created by Amit, Biraj and Chirag respectively.
1 2 4 0 2 0
A== GB== GC = = G
–1 3 1 5 1 –2

If a = 4 and b = –2, based on the above information answer the following: [CBSE Question Bank]


Answer the questions given below.
(i) Sum of the matrices A, B and C , A + (B + C) is
1 6 6 1 7 2 2 1
(a) = G (b) = G (c) = G (d) = G
2 7 7 2 1 6 7 6

(ii) (AT)T is equal to


1 2 2 1 1 –1 2 3
(a) = G (b) = G (c) = G (d) = G
–1 3 3 –1 2 3 –1 1
(iii) (bA)T is equal to

–2 –4 –2 2 –2 2 –6 –2
(a) = G (b) = G (c) = G (d) = G
2 –6 –4 –6 –6 –4 2 4

(iv) AC− BC is equal to
–4 –6 –4 –4 –4 –4 –6 4
(a) = G (b) = G (c) = G (d) = G
–4 4 4 –6 –6 4 –4 –4
(v) (a + b)B is equal to
0 8 2 10 8 0 2 0
(a) = G (b) = G (c) = G (d) = G
10 2 8 0 2 10 8 10
Sol. (i) We have,
1 2 4 0 2 0
A + (B + C) = = G + )= G+= G3
–1 3 1 5 1 –2
1 2 6 0
== G+= G
–1 3 2 3
7 2
⇒ A + (B + C) = = G

1 6
∴ Option (c) is correct.

(ii) We have,
T
1 2 1 –1
AT = = G == G
–1 3 2 3
T
1 –1 1 2
^ AT h = = G == G
T


2 3 –1 3
∴ Option (a) is correct.

Case Study Based Questions 537


@Cbsebookshub - Join Us on Telegram
(iii) We have,
1 2 –2 –4
bA = –2 = G== G
–1 3 2 –6
T
–2 –4 –2 2

]bAgT = = G == G
2 –6 –4 –6
∴ Option (b) is correct.

(iv) We have,
1 2 2 0 4 0 2 0
AC – BC = = G= G–= G= G
–1 3 1 –2 1 5 1 –2
4 –4 8 0 –4 –4
== G–= G== G
1 –6 7 –10 –6 4
∴ Option (c) is correct.

(v) We have
4 0 4 0
]a + bg B = ]4 –2g = G= 2= G
1 5 1 5
8 0
== G
2 10
∴ Option (c) is correct.

3. Read the following and answer any four questions from (i) to (v).
Three schools DPS, CVC and KVS decided to organize a fair for collecting money for helping the

flood victims. They sold handmade fans, mats and plates from recycled material at a cost of `25,
`100 and `50 each respectively. The numbers of articles sold are given as

School/Article DPS CVC KVS


Handmade fans 40 25 35
Mats 50 40 50
Plates 20 30 40
[CBSE Question Bank]


Answer the questions given below.
(i) What is the total money (in `) collected by the school DPS?
(a) 700 (b) 7,000 (c) 6,125 (d) 7875

538 Xam idea Mathematics–XII

@Cbsebookshub - Join Us on Telegram


(ii) What is the total amount of money (in `) collected by schools CVC and KVS?
(a) 14,000 (b) 15,725 (c) 21,000 (d) 13,125

(iii) What is the total amount of money collected by all three schools DPS, CVC and KVS?
(a) `15,775 (b) `14,000 (c) `21,000 (d) `17,125

(iv) If the number of handmade fans and plates are interchanged for all the schools, then
what is the total money collected by all schools?
(a) `18,000 (b) `6,750 (c) `5,000 (d) `21,250
(v) How many articles (in total) are sold by three schools?
(a) 230 (b) 130 (c) 430 (d) 330
(i) Total money (in Rupees) collected by the school DPS
Sol.
RS VW
25
Handmade fans Mats Plates SSS WWW
= = G 100
40 50 20 SSS WWW
S 50 W
T X
= 40 × 25 + 50 × 100 + 20 × 50 = 1000 + 5000 + 1000
= `7000
∴ Option (b) is correct.
(ii) Amount of money collected by CVC and KVS
RS VW
25
25 40 30 SSS WWW
= = G 100
35 50 40 SSS WWW
S 50 W
T X
25 × 25 + 40 × 100 + 30 × 50 6125
= = G== G
35 × 25 + 50 × 100 + 40 × 50 7875
∴ Total amount of money collected by CVC and KVS

= 6125 + 7875 = `14,000
∴ Option (a) is correct.

(iii) Total amount of money collected by all three schools
= 7000 + 6125 + 7875 = `21,000
∴ Option (c) is correct.

(iv) After interchanging the number of handmade fans and plates,

We have
School/Article DPS CVC KVS
Handmade fans 20 30 40
Mats 50 40 50
Plates 40 25 35

Amount of money collected by all schools


RS VR V R V
SS20 50 40WWW SSS 25 WWW SSS20 × 25 + 50 × 100 + 40 × 50WWW
= SS30 40 25WW SS100WW = SS30 × 25 + 40 × 100 + 25 × 50WW
SS WS W S W
S40 50 35WW SS 50 WW SS40 × 25 + 50 × 100 + 35 × 50WW
T XT X T X
RS V
SS7500WWW
= SS6000WW
SS W
S7750WW
T X
∴ Total amount = 7500 + 6000 + 7750 = `21,250
∴ Option (d) is correct.

Case Study Based Questions 539


@Cbsebookshub - Join Us on Telegram
(v) Total number of articles sold by three schools
= (40 + 25 + 35) + (50 + 40 + 50) + (20 + 30 + 40)
= 100 + 140 + 90 = 330
∴ Option (d) is correct.

4. Read the following and answer any four questions from (i) to (v).
On her birth day, Seema decided to donate some money to children of an orphanage home.

If there were 8 children less, everyone would have got `10 more. However, if there were 16
children more, everyone would have got `10 less. Let the number of children be x and the amount
distributed by Seema for one child be y (in `). [CBSE Question Bank]


Answer the questions given below.
(i) The equations in terms of x and y are
(a) 5x – 4y = 40 (b) 5x – 4y = 40 (c) 5x – 4y = 40 (d) 5x + 4y = 40

5x – 8y = –80 5x – 8y = 80
5x + 8y = –80 5x – 8y = –80
(ii) Which of the following matrix equations represent the information given above?
5 4 x 40 5 –4 x 40
(a) = G= G = = G (b) = G= G = = G
5 8 y –80 5 –8 y 80

5 –4 x 40 5 4 x 40
(c) = G= G = = G (d) = G= G = = G
5 –8 y –80 5 –8 y –80

(iii) The number of children who were given some money by Seema, is
(a) 30 (b) 40 (c) 23 (d) 32

(iv) How much amount is given to each child by Seema?
(a) `32 (b) `30 (c) `62 (d) `26
(v) How much amount Seema spends in distributing the money to all the students of the
orphanage?
(a) `609 (b) `960 (c) `906 (d) `690
Sol. (i) We have number of children be x and amount for one child be `y.
∴ Total amount distributed by Seema = ` xy


Now, according to question
(x – 8) (y + 10) = xy ⇒ 10x – 8y = 80
⇒ 5x – 4y = 40 ...(i)

Also,
(x + 16) (y – 10) = xy ⇒ xy – 10x + 16y – 160 = xy

–10x + 16y = 160 ⇒ 5x – 8y = –80 ...(ii)

540 Xam idea Mathematics–XII

@Cbsebookshub - Join Us on Telegram


∴ Equations be

5x – 4y = 40
5x – 8y = –80
∴ Option (a) is correct.

(ii) Given equation can be written in matrix form as follows:
5 –4 x 40
= = G= G = = G
5 –8 y –80
∴ Option (c) is correct.

(iii) We have,
5 –4 x 40
= = G= G = = G
5 –8 y –80
. . .
A X B
⇒ AX = B
⇒ X = A–1.B ...(iii)

A == G⇒
5 –4 5 –4
A = = – 40 + 20 = –20 ≠ 0
5 –8 5 –8
∴ Inverse of Matrix A exists.

∴ Co-factors of Matrix A are

C11 = –8, C21 = 4
C12 = –5, C22 = 5
–8 4 adj A
∴ adj A = =
G ⇒ A –1 =
–5 5 A

= G
–1 –8 4
⇒ A –1 =

20 –5 5
Putting in (iii), we get
x 1 –8 4 40 1 –320 –320
= G = – = G= G = – = G
y 20 –5 5 –80 20 –200 –400
x 32
⇒ = G== G
⇒ x = 32, y = 30
y 30
∴ Number of children = 32

∴ Option (d) is correct.

(iv) Amount given to each child = y
= `30
∴ Option (b) is correct.

(v) Total amount be distributed by Seema = xy
= 32 × 30
= ` 960
∴ Option (b) is correct.

zzz

Case Study Based Questions 541


@Cbsebookshub - Join Us on Telegram
Chapter-6: Continuity and Differentiability
1. Read the following and answer any four questions from (i) to (v).

A potter made a mud vessel, where the shape of the pot is


based on f(x) = |x – 3| + | x – 2|, where f(x) represents the
height of the pot.

[CBSE Question Bank]



Answer the questions given below.
(i) When x > 4 What will be the height in terms of x ?
(a) x – 2 (b) x – 3 (c) 2x – 5 (d) 5 – 2x
(ii) Will the slope vary with x value?
(a) Yes (b) No
(c) may or may not vary (d) none of these
dy

(iii) What is at x = 3 ?
dx
(a) 2 (b) –2
(c) Function is not differentiable (d) 1

(iv) When the x value lies between (2, 3) then the function is
(a) 2x – 5 (b) 5 – 2x (c) 1 (d) 5
(v) If the potter is trying to make a pot using the function f(x) = [x] , will he get a pot or not?
Why?
(a) Yes, because it is a continuous function
(b) Yes, because it is not continuous
(c) No , because it is a continuous function
(d) No , because it is not continuous

Sol. (i) We have, f(x) = |x – 3| + |x – 2|


When x > 4

∴ f(x) = (x – 3) + (x – 2) = 2x – 5

∴ Option (c) is correct.

(ii) Yes, because when 2 < x < 3, we have
f(x) = – (x – 3) + (x – 2) = 1
⇒ Slope = f'(x) = 0

but when x > 3, we have

f(x) = x – 3 + x – 2 = 2x – 5
then slope = f'(x) = 2
∴ Option (a) is correct.
(iii) At x = 3 f ]3 – mg – f ]3g – ]3 – m – 3g + ]3 – m – 2g – 1
L.H.D = lim = lim
m"0 –m m"0 –m
+
m 1– m –1 0
= lim = lim =0
m"0 –m m " 0 –m

542 Xam idea Mathematics–XII

@Cbsebookshub - Join Us on Telegram


f ^ 3 + m h – f ]3 g

R.H.D = lim
m"0 m
]3 + m – 3g + ]3 + m – 2g – 1
= lim
m"0 m
m +1+ m –1
= lim =2
m"0 m
L.H.D ≠ R.H.D at x = 3

∴ f(x) is not differentiable at x = 3

∴ Option (c) is correct.

(iv) When 2 < x < 3, we have
f(x) = –(x – 3) + (x – 2) = 1
∴ Option (c) is correct.
(v) We have the function f(x) = [x] ≤ x, where x is an integer.
Y

X X

It is not a continuous function, so the potter can not make a pot using the function f(x) = [x].
∴ Option (d) is correct.

zzz

Chapter-10: Differential Equations


1. Read the following and answer any four questions from (i) to (v).
A Veterinary doctor was examining a sick cat brought by a pet lover. When it was brought to

the hospital, it was already dead. The pet lover wanted to find its time of death. He took the
temperature of the cat at 11.30 pm which was 94.6°F. He took the temperature again after one
hour; the temperature was lower than the first observation. It was 93.4°F. The room in which the
cat was put is always at 70°F. The normal temperature of the cat is taken as 98.6°F when it was
alive. The doctor estimated the time of death using Newton law of cooling which is governed
a ]T – 70g , where 70°F is the room temperature and T is the
dT
by the differential equation:
dt
temperature of the object at time t.

Substituting the two different observations of T and t made, in the solution of the differential
= k ]T – 70g where k is a constant of proportion, time of death is calculated.
dT
equation
dt
[CBSE Question Bank]
Answer the questions given below.
(i) State the degree of the above given differential equation.
(a) 0 (b) 1 (c) 2 (d) Not defined
(ii) Which method of solving a differential equation helped in calculation of the time of
death?

Case Study Based Questions 543


@Cbsebookshub - Join Us on Telegram
(a) Variable separable method (b) Solving Homogeneous differential equation
(c) Solving Linear differential equation (d) all of the above

(iii) If the temperature was measured 2 hours after 11.30 pm, will the time of death change?
(Yes/No)
(a) Yes (b) No
(c) Cannot be determined (d) None of these
dT

(iv) The solution of the differential equation = k ]T – 70g is given by,
dt
(a) log |T – 70| = kt + C (b) log |T – 70| = log |kt|+ C
(c) T – 70 = kt + C (d) T – 70 = kt C
(v) If t = 0 when T is 72, then the value of C is
(a) –2 (b) 0 (c) 2 (d) Log 2
Sol. (i) We have differential equation
= k ]T – 70g with degree 1
dT

dt
∴ Option (b) is correct.
(ii) To solve the differential equation
= k ]T – 70g
dT

dt
Variable- Separable method is useful. (Helpful)
∴ Option (a) is correct.

(iii) No, the time of death would not change.
∴ Option (b) is correct.

(iv) We have,
= k ]T – 70g
dT

dt
dT
⇒ y = y kdt
T – 70
⇒ log |T – 70| = kt + C


Option (a) is correct.
(v) Given t = 0 when T = 72
Now, log|T – 70| = kt + C

⇒ log|72 – 70| = k × 0 + C

⇒ log 2 = C

∴ Option (d) is correct.

2. Read the following and answer any four questions from (i) to (v).
Polio drops are delivered to 50K children in a district. The rate at which polio drops are given is

directly proportional to the number of children who have not been administered the drops. By
the end of 2nd week half the children have been given the polio drops. How many will have been
given the drops by the end of 3rd week can be estimated using the solution to the differential
dy
equation = k ^50 – y h where x denotes the number of weeks and y the number of children who
dx
have been given the drops. [CBSE Question Bank]
Answer the questions given below.
(i) The order of the above given differential equation is
(a) 1 (b) 2 (c) 3 (d) none of these

544 Xam idea Mathematics–XII

@Cbsebookshub - Join Us on Telegram


dy
(ii) Which method of solving a differential equation can be used to solve = k ^50 – y h ?
dx
(a) Variable separable method (b) Solving Homogeneous differential equation
(c) Solving Linear differential equation (d) all of the above
dy

(iii) The solution of the differential equation = k ^50 – y h is given by,
dx
(a) log|50 – y|= kx + C (b) –log|50 – y| = kx + C
(c) log|50 – y| = log|kx|+ C (d) 50 – y = kx + C

(iv) The value of c in the particular solution given that y(0)=0 and k = 0.049 is.
1
(a) log 50 (b) log (c) 50 (d) –50
50
(v) Which of the following solutions may be used to find the number of children who have
been given the polio drops?
(a) y = 50 – ekx (b) y = 50 – ekx (c) y = 50 (1 – e–kx) (d) y = 50 (ekx – 1)
Sol. (i) Given differential equation
dy
= k ^50 – y h
dx
has order 1.
∴ Option (a) is correct.

dy
(ii) Variable Separable method can be used to solve differential equation = k ^50 – y h
dx
∴ Option (a) is correct.

(iii) We have,
dy
= k ^50 – y h
dx
dy
⇒ y = y kdx ⇒ –log|50 – y| = kx + C
50 – y
∴ Option (b) is correct.
(iv) Given y(0) = 0 and k = 0.049
∴ –log|50 – y| = kx + C

⇒ –log|50 – 0| = 0.049 × 0 + C

1
–log 50 = C ⇒ C = log
50

Option (b) is correct.
(v) We have,
1
– log 50 – y = kx + log (from (iv)
50
1

–kx = log 50 – y + log
50
50 – y

–kx = log
50
50 – y y

e –kx = =1–
50 50
y –kx

=1– e ⇒ y = 50(1 – e–kx)
50
This is the required solution to find the number of children who have been given the polio
drops.
∴ Option (c) is correct.

zzz
Case Study Based Questions 545
@Cbsebookshub - Join Us on Telegram
Chapter-11: Vector Algebra
1. Read the following and answer any four questions from (i) to (v).
Solar Panels have to be installed carefully so that the tilt of the roof, and the direction to the sun,

produce the largest possible electrical power in the solar panels.
A surveyor uses his instrument to determine the coordinates of the four corners of a roof where
solar panels are to be mounted. In the picture , suppose the points are labelled counter clockwise
from the roof corner nearest to the camera in units of meters P1 (6,8,4) , P2 (21,8,4), P3 (21,16,10)
and P4 (6,16,10) [CBSE Question Bank]


Answer the questions given below.
(i) What are the components to the two edge vectors defined by A = PV of P2 – PV of P1 and
B = PV of P4 – PV of P1? (where PV stands for position vector)
(a) (15, 8, 4), (0, 8, 6) (b) (15, 0, 0), (6, 8, 0)
(c) (15, 0, 0), (0, 8, 6) (d) (15, 8, 4), (6, 8, 0)
(ii) The vector in standard notation with it, jt and kt , (where it, jt and kt are the unit vectors along
the three axes) are
(a) _15it + 8jt + 4kti, _0it + 8jt + 6kti (b) _15it + 0jt + 0kti, _6it + 8jt + 6kti
(c) _15it + 0jt + 0kti, _0it + 8jt + 6kti (d) _15it + 8jt + 4kti, _6it + 8jt + 6kti

(iii) What are the magnitudes of the vectors A and B ?
(a) 325 units, 10 units (b) 15 units, 136 units
(c) 15 units, 10 units (d) 325 units, 136 units

(iv) What are the components to the vector N , perpendicular to A and B and the surface of
the roof?
(a) 0, –90, 120 (b) 0, –90, –120
(c) 0, 90, 120 (d) 0, 90, –120
1 6 1
(v) The sun is located along the unit vector S = it – jt + kt. If the flow of solar energy is
2 7 7
given by the vector F = 910 S units of watts/meter2, what is the dot product of vectors F
with N , and the units for this quantity?
(a) 84,800 watts (b) 85,800 watts (c) 54600 watts (d) 86255 watts
Sol. Given points are P1 (6,8,4) , P2 (21,8,4), P3 (21,16,10) and P4 (6,16,10).
(i) We have, A = P.V of P – P.V of P = _ 21it + 8jt + 4kti – _6it + 8jt + 4kti
2 1
A = 15it + 0jt + 0kt
∴ Components of vector A are 15, 0, 0

546 Xam idea Mathematics–XII

@Cbsebookshub - Join Us on Telegram


and B = P.V of P4 – P.V of P1 = _6it + 16jt + 10kti – _6it + 8jt + 4kti
= 0it + 8jt + 6kt
∴ Components of vector B are 0, 8, 6

∴ Option (c) is correct.
(ii) A = 15it + 0jt + 0kt and B = 0it + 8jt + 6kt

Option (c) is correct.
(iii) We have,
A = ]15g + ]0g + ]0g = 15 units
2 2 2

B = ]0g + ]8g + ]6g = 64 + 36 = 100 = 10 units


2 2 2

∴ Option (c) is correct.



(iv) We have, it jt kt
N = A × B = 15 0 0
0 8 6
= it]0 – 0g – jt ]90 – 0g + kt]120 – 0g
= 0it – 90jt + 120kt
Its components are 0, –90, 120.
∴ Option (a) is correct.

1 6 1
(v) F = 910 c it – jt + kt m = 455it – 780jt + 130kt
2 7 7
The dot product is F .N = 455 (0) – 780(–90) + 130(120) = 85,800 watts.
∴ Option (b) is correct.

2. Read the following and answer any four questions from (i) to (v).
A class XII student appearing for a competitive examination was asked to attempt the following

questions.
Let a , b and c be three non zero vectors. [CBSE Question Bank]
Answer the questions given below.
(i) If a and b are such that a + b = a – b then
(a) a = b (b) a < b (c) a = b (d) None of these
(ii) If a = it – 2jt, b = 2it + jt + 3kt then evaluate ^2a + b h . 7^a + b h × ^a – 2b hA
(a) 0 (b) 4 (c) 3 (d) 2
(iii) If a and b are unit vectors and θ be the angle between them then a – b is

i i i i
(a) sin (b) 2 sin (c) 2 cos (d) cos
2 2 2 2

(iv) Let a , b and c be unit vectors such that a . b = a . c = 0 and angle between b and c is

r
is then a =
6
(a) 2 ^b × c h (b) –2 ^b × c h (c) ! 2 ^b × c h (d) �^b ! c h
(v) The area of the parallelogram formed by a and b as diagonals is
70
(a) 70 (b) 35 (c) (d) 70
2
Sol. (i) Given, a + b = a – b
2 2

a +b = a –b

Case Study Based Questions 547


@Cbsebookshub - Join Us on Telegram
2 2 2 2

a + b + 2 a b cos i = a + b –2 a b cos i
(Where θ is the angle between a and b .)
r
⇒ 4 a b cos i = 0 ⇒ cos i = 0 = cos
2
r
⇒ i=
2

a =b
∴ Option (a) is correct.

(ii) We have,
a = it – 2jt and b = 2it + jt + 3kt

2a + b = 2it – 4jt + 2it + jt + 3kt = 4it – 3jt + 3kt
a + b = 3it – jt + 3kt and a – 2b = _it – 2jti – _ 4it + 2jt + 6kti = –3it – 4jt – 6kt
it jt kt

^a + b h × ^a – 2b h = 3 –1 3 = it]6 + 12g – jt]–18 + 9g + kt]–12 – 3g
–3 – 4 – 6
= 18it + 9jt – 15kt

^2a + b h . 7^a + b h × ^a – 2b hA = _ 4it – 3jt + 3kti . _18it + 9jt – 15kti
= 4 × 18 – 3 × 9 + 3 × –15
= 72 – 27 – 45 = 72 – 72 = 0
∴ Option (a) is correct.
(iii) Given a = b = 1
2 2 2
∴ a – b
= a + b –2 a b cos i
2
⇒ a – b
= 1 + 1 – 2 × 1 × 1 × cos i = 2 – 2 cos i

= 2 ]1 – cos ig = 2 × 2 sin 2 = 4 sin 2


i i

2 2
i
⇒ a – b = 2 sin

2
∴ Option (b) is correct.

(iv) We have,
a .b = a . c = 0

a .b = 0 ⇒ a = b
a .c = 0 ⇒ a = c
and
r
Since angle between b and c is
6
∴ a = ! 2 ^b × c h

∴ Option (c) is correct.

(v) The area of the parallelogram formed by a and b as
1
diagonals is a ×b
2
1
∴ A = a × b ....(i)
2
it jt kt
Now, a × b = 1 –2 0 = it]–6 – 0g –jt]3 – 0g + kt]1 + 4g
2 1 3
= –6it – 3jt + 5kt
a × b = ]–6g + ]–3g + ]5g = 36 + 9 + 25
2 2 2

548 Xam idea Mathematics–XII

@Cbsebookshub - Join Us on Telegram


= 70
1

Area = 70 sq. units
2
∴ Option (c) is correct.

3. Read the following and answer any four questions from (i) to (v).
A cricket match is organized between two Clubs A and B for which a team from each club is

chosen. Remaining players of Club A and Club B are respectively sitting on the plane represented
by the equation r . _2 i – j + k i = 3 and r . _ i + 3 j + 2k i = 8 to cheer the team of their own clubs.
[CBSE Question Bank]


Answer the questions given below.
(i) The Cartesian equation of the plane on which players of Club A are seated is
(a) 2x − y + z = 3 (b) 2x − y + 2z = 3 (c) 2x − y + z = –3 (d) x − y + z = 3
(ii) The magnitude of the normal to the plane on which players of club B are seated, is
(a) 15 (b) 14 (c) 17 (d) 20

(iii) The intercept form of the equation of the plane on which players of Club B are seated is
x y z x y z x y z x y z
(a) + + = 1 (b) + + = 1 (c) + + = 1 (d) + + = 1
8 8 2 5 8 3 8 8 4 8 7 2
3 3

(iv) Which of the following is a player of Club B?
(a) Player sitting at (1, 2, 1) (b) Player sitting at (0, 1, 2)
(c) Player sitting at (1, 4, 1) (d) Player sitting at (1, 1, 2)
(v) The distance of the plane, on which players of Club B are seated, from the origin is
8 6 7 9
(a) units (b) units (c) units (d) units
14 14 14 14

Sol. (i) The players of club A are seated is
r . (2it – jt + kt) = 3
⇒ (xit + yjt + zkt) . _ 2it – jt + kti = 3

⇒ 2x – y + z = 3

Which is the Cartesian equation.
∴ Option (a) is correct.

(ii) Given equation of the plane in players of club B are sitting be
r . _it + 3jt + 2kti = 8
∴ Normal vector to the plane N = it + 3jt + 2kt

Case Study Based Questions 549


@Cbsebookshub - Join Us on Telegram
N = ]1 g + ]3g + ]2g
2 2 2


= 1 + 9 + 4 = 14
∴ Option (b) is correct.

(iii) Equation of the plane for the players of club B
r . _it + 3jt + 2kti = 8

x + 3y + 2z = 8
x 3y 2z x 3y z

+ + =1 ⇒ + + =1
8 8 8 8 8 4
3
which is intercept form
∴ Option (a) is correct.

(iv) We have Cartesian equation of the player of club B is
x + 3y + 2z = 8 .... (i)
and the point (1, 1, 2) satisfy the equation. ....(i)
i.e. 1 + 3 × 1 + 2 × 2 = 8

⇒ 8 = 8

∴ Option (d) is correct.

(v) The distance of the plane x + 3y + 2z – 8 = 0 from the point (0, 0, 0) i.e. origin is given by
0+ 3×0+ 2×0 – 8 8
D = = units
]1 g2 + ]3g2 + ]2g2 14
∴ Option (a) is correct.

4. Read the following and answer any four questions from (i) to (v).
The Indian coast guard, while patrolling, saw a suspicious boat with people. They were nowhere

looking like fishermen. The coast guard were closely observing the movement of the boat for an
opportunity to seize the boat. They observed that the boat is moving along a planar surface. At
an instant of time, the coordinates of the position of the coast guard helicopter and the boat is
(1, 3, 5) and (2, 5, 3) respectively. [CBSE Question Bank]

550 Xam idea Mathematics–XII

@Cbsebookshub - Join Us on Telegram



Answer the questions given below.
(i) If the line joining the positions of the helicopter and the boat is perpendicular to the
plane in which the boat moves, then the equation of the plane is
(a) –x + 2y – 2z = 6 (b) x + 2y + 2z = 6 (c) x + 2y – 2z = 6 (d) x – 2y – 2z = 6
(ii) If the coast guard decide to shoot the boat at that given instant of time, then what is the
distance (in meters) that the bullet has to travel?
(a) 5m (b) 3m (c) 6m (d) 4m

(iii) If the coast guard decides to shoot the boat at that given instant of time, when the speed
of bullet is 36m/sec, then what is the time taken for the bullet to travel and hit the boat?
1 1 1 1
(a) seconds (b) seconds (c) seconds (d) seconds
8 14 10 12


(iv) At that given instant of time, the equation of line passing through the positions of the
helicopter and boat is
x–1 y–3 z–5 x–1 y+3 z–5
(a) = = (b) = =
1 2 –2 2 1 –2
x+1 y–3 z–5 x–1 y+3 z+5
(c) = = (d) = =
–2 –1 –2 2 –1 2
(v) At a different instant of time, the boat moves to a different position along the planar
surface. What should be the coordinates of the location of the boat if the coast guard
x y–1 z–2
shoots the bullet along the line whose equation is = = for the bullet to hit
1 2 1
the boat?
–8 19 –14 8 –19 –14 8 –19 14
(a) c , , m (b) c , , m (c) c , , m (d) none of the above
3 3 3 3 3 3 3 3 3
Sol. We have positions of helicopter and the boat are respectively (1, 3, 5) and (2, 5, 3).
(i) Let a = 2it + 5jt + 3kt, n = (2 – 1) it + (5 – 3) jt + (3 – 5) kt
n = it + 2jt – 2kt
it + 2jt – 2kt
⇒ nt =

3
∴ The equation of the plane in which boat moves is given by

^ r – a h . n = 0
it + 2jt – 2kt

# r – _ 2it + 5jt + 3kti- . =0
3

r . _it + 2jt – 2kti – ]2 + 10 – 6g = 0
⇒ x + 2y – 2z – 6 = 0 ⇒ x + 2y – 2z = 6
∴ Option (c) is correct.

(ii) Distance that bullet has to travel = ]2 – 1g + ]5 – 3g + ]3 – 5g
2 2 2

= 1 + 4 + 4 = 3 m.
∴ Option (b) is correct.

3 1
(iii) Time taken for the bullets to travel and hit the boat = = seconds.
36 12
∴ Option (d) is correct.

(iv) We have position of helicopter be(1, 3, 5) and the boat be (2, 5, 3)
∴ Equation of line passing through the positions of the helicopter and boat is given by
x–1 y–3 z–5
= =
2–1 5–3 3–5

Case Study Based Questions 551


@Cbsebookshub - Join Us on Telegram
x–1 y–3 z–5

= =
1 2 –2

Option (a) is correct.
(v) Given line of the bullet be
x–1 y–3 z–5
⇒ = = = k (let)
1 2 –2
∴ Any point on the line be (k + 1, 2k + 1, k + 2)

Since the points lies on the plane
x + 2y – 2z = 6

∴ k + 2(2k + 1) – 2(k + 2) = 6

8
⇒ 3k = 8
⇒ k=
3
∴ Co-ordinate of the location be

8 19 14
c , , m
3 3 3
∴ Option (d) is correct.

5. Read the following and answer any four questions from (i) to (v).
The equation of motion of a missile are x = 3t, y = – 4t, z = t, where the time ‘t’ is given in seconds,

and the distance is measured in kilometres. [CBSE Question Bank]


Answer the questions given below.
(i) What is the path of the missile?
(a) Straight line (b) Parabola (c) Circle (d) Ellipse
(ii) Which of the following points lie on the path of the missile?
(a) (6, 8, 2) (b) (6, –8, –2) (c) (6, –8, 2) (d) (-6, –8, 2)

(iii) At what distance will the rocket be from the starting point (0, 0, 0) in 5 seconds?
(a) 550 kms (b) 650 kms (c) 450 kms (d) 750 kms

(iv) If the position of rocket at a certain instant of time is (5, –8, 10), then what will be the
height of the rocket from the ground? (The ground is considered as the xy – plane).
(a) 12 km (b) 11 km (c) 20 km (d) 10 km

552 Xam idea Mathematics–XII

@Cbsebookshub - Join Us on Telegram


(v) At a certain instant of time, if the missile is above the sea level, where the equation of the
surface of sea is given by 2x + y + 3z = 1 and the position of the missile at that instant of
time is (1, 1, 2), then the image of the position of the rocket in the sea is
–9 –1 –10 9 –1 –10 –9 1 –10 –9 –1 10
(a) c , , m (b) c , , m (c) c , , m (d) c , , m
7 7 7 7 7 7 7 7 7 7 7 7

Sol. (i) Given equation of motion of a missile be


x = 3t, y = –4t, z = t
x y z
⇒ = = which is a straight line.
3 –4 1
Hence, the path of the missile is a straight line.
∴ Option (a) is correct.

(ii) We have equation of the path of the missile as
x y z
= =
3 –4 1

and the point (6, –8, 2) satisfy the equation
∴ Point is (6, –8, 2) lie on the path of missile.

∴ Option (c) is correct.

(iii) After 5 seconds position of the rocket be
x = 3t = 3 × 5 = 15
y = –4t = – 4 × 5 = –20
z = t = 5
∴ Point is (15, –20, 5).

Its distance from origin (0, 0, 0) is ]15 – 0g + ]–20 –0g + ]5 – 0g
2 2 2

= 225 + 400 + 25 = 650 km


∴ Option (b) is correct.

(iv) Given position of the rocket at a time is (5, –8, 10)
∴ Height of the rocket from the ground

= Distance between the points (5, –8, 10) and (5, –8, 0).
(Since ground is considered as the XY-Plane)
= ]5 – 5g + ]–8 + 8g + ]10 – 0g = 10 Km
2 2 2
P (1,1,2)
∴ Option (d) is correct.

(v) Let R(x, y, z) be the image of the point P(1, 1, 2).
Let Q(α, β, γ) be the mid point of PR lie on the plane.

∴ 2α + β + 3g = 1
...(i)
Direction ratio of the line PQ be
1
z=

α – 1, β – 1, γ – 2
Q (α,β,γ)
y+3

a–1 b–1 c–2


∴ = = =k
2x+

2 1 3
⇒ α = 2k + 1, β = k + 1, g = 3k + 2


from (i), we have
2(2k + 1) + (k + 1) + 3(3k + 2) = 1 ⇒ 14k = – 8
–4 R (x,y,z)
⇒ k=
7

Case Study Based Questions 553


@Cbsebookshub - Join Us on Telegram
–8 –1

a = 2k + 1 = +1 =
7 7
–4 3 –12 2
b = k + 1 = +1 = and g = 3k + 2 = +2=
7 7 7 7
Since Q(α, β, g) be the mid point of PR
x+1 x + 1 –1 –2 –9
∴ =a ⇒ = ⇒ x= –1=
2 2 7 7 7
y+1 y+1 3 6 1
=b ⇒ = ⇒ y= –1=–
2 2 7 7 7
z+2 z+2 2 4 4
and, =c ⇒ = ⇒ z+2 = ⇒ z= –2
2 2 7 7 7
–10
⇒ z=
–9 1 –10 7
∴ Co-ordinates of image be c , – , m
7 7 7
∴ Option (a) is correct.
6. Read the following and answer any four questions from (i) to (v).
Suppose the floor of a hotel is made up of mirror polished Salvatore stone. There is a large crystal

chandelier attached to the ceiling of the hotel room. Consider the floor of the hotel room as a plane
having the equation x – y + z = 4 and the crystal chandelier is suspended at the point (1, 0, 1).
[CBSE Question Bank]

Answer the questions given below.


(i) The direction ratios of the perpendicular from the point (1, 0, 1) to the plane
x – y + z = 4 is
(a) (–1, –1, 1) (b) (1, –1, –1) (c) (–1, –1, –1) (d) (1, –1, 1)
(ii) The length of the perpendicular from the point (1, 0, 1) to the plane x – y + z = 4 is
2 4 6 8
(a) units (b) units (c) units (d) units
3 3 3 3

(iii) The equation of the perpendicular from the point (1, 0, 1) to the plane x – y + z = 4 is
x–1 y–3 z+5 x–1 y+3 z–5
(a) = = (b) = =
2 –1 2 –2 –1 2
x–1 y z–1 x–1 y z–1
(c) = = (d) = =
1 –1 1 2 –2 1

(iv) The equation of the plane parallel to the plane x – y + z = 4, which is at a unit distance
from the point (1, 0, 1) is
(a) x – y + z + ^2 – 3 h (b) x – y + z – ^2 + 3 h
(c) x – y + z + ^2 + 3 h (d) Both (a) and (c)

554 Xam idea Mathematics–XII

@Cbsebookshub - Join Us on Telegram


(v) The direction cosine of the normal to the plane x – y + z = 4 is
1 –1 –1 1 –1 1 –1 –1 1 –1 –1 –1
(a) e , , o (b) e , , o (c) e , , o (d) e , , o
3 3 3 3 3 3 3 3 3 3 3 3

Sol. (i) Direction ratios of the perpendicular from the point (1, 0, 1) to the plane x – y + z = 4 is
(1, –1, 1)
∴ Option (d) is correct.
P (1,0,1)

(ii) The required length of perpendicular from point P


to the plane is given by
1–0+1– 4
PM =
]1 g2 + ]–1g2 + ]1 g2
2 M
units =
3 x-y+z = 4
∴ Option (a) is correct.

(iii) Required equation be
x–1 y–0 z–1
= =
1 –1 1
x–1 y z–1
⇒ = =
1 –1 1
∴ Option (c) is correct.
(iv) Let the equation of the plane parallel to the plane
x – y + z = 4 is x – y + z = k ...(i)
∴ Distance of the plane (i) from the point (1, 0, 1) is

1–0+1+k

=1 ⇒ 2 + k =! 3
3
⇒ k =! 3 – 2
from (i) equation of the plane be
x – y + z = ! 3 – 2 ⇒ x – y + z + 2 – 3 = 0
or x – y + z + ^ 3 + 2 h = 0

∴ Option (d) is correct.



(v) Direction cosines of the normal to the plane
x – y + z = 4 is given by
1 –1 1
, ,
]1 g + ]–1g + ]1 g
2 2 2
]1 g + ]–1g2 + ]1 g2
2
]1 g + ]–1g2 + ]1 g2
2

1 1 1
i.e.
,– ,
3 3 3
∴ Option (b) is correct.
7. Read the following and answer any four questions from (i) to (v).
A mobile tower stands at the top of a hill. Consider the surface on which the tower stands as a

plane having points A(1, 0, 2), B(3, -1, 1) and C(1, 2, 1) on it. The mobile tower is tied with 3 cables
from the point A, B and C such that it stands vertically on the ground. The top of the tower is at
the point (2, 3, 1) as shown in the figure. [CBSE Question Bank]

Case Study Based Questions 555


@Cbsebookshub - Join Us on Telegram
(2, 3, 1)

B (3, –1, 1)

A (1, 0, 2)
C (1, 2, 1)


Answer the questions given below.
(i) The equation of the plane passing through the points A, B and C is
(a) 3x – 2y + 4z = –11 (b) 3x + 2y + 4z = 11
(c) 3x – 2y – 4z = 11 (d) –3x + 2y + 4z = –11
(ii) The height of the tower from the ground is
5 7 6 8
(a) units (b) units (c) units (d) units
29 29 29 29

(iii) The equation of the perpendicular line drawn from the top of the tower to the ground is
x–1 y+3 z–5 y–3 z–1
(a) = = (b) x – 2 = =
2 1 –2 –3 2 –4
x–2 y–3 z–1 x+1 y+3 z–5
(c) = = (d) = =
3 2 4 –2 –1 2

(iv) The coordinates of the foot of the perpendicular drawn from the top of the tower to the
ground are
43 –77 –9 9 –1 –10 –43 77 –9 43 77 9
(a) c , , m (b) c , , m (c) c , , m (d) c , , m
29 29 29 7 7 7 29 29 29 29 29 29
(v) The area of ΔABC is
29 29 39 39
(a) sq. units (b) sq. units (c) sq. units (d) sq. units
4 2 2 4
Sol. (i) Given Co-ordinates of the points in a plane are
A ^1, 0, 2h, B ^3, –1, 1h and C ^1, 2, 1h
x1 y1 z1 x2 y2 z2 x3 y3 z3

∴ Equation of plane containing three non-collinear points is given by



x–x1 y–y1 z–z1
2 –x1 y2 –y1 z2 –z1 = 0
x
x3 –x1 y3 –y1 z3 –z1
x–1 y–0 z–2 x–1 y z–2

3–1 –1–0 1–2 = 0 ⇒ 2 –1 –1 = 0
1–1 2–0 1–2 0 2 –1
⇒ (x – 1)(1 + 2) –y(–2) + (z – 2) (4) = 0

⇒ 3x – 3 + 2y + 4z – 8 = 0
⇒ 3x + 2y + 4z = 11

Option (b) is correct.

556 Xam idea Mathematics–XII

@Cbsebookshub - Join Us on Telegram


(ii) We have Co-ordinates of the top of the tower is (2, 3, 1)

3 × 2 + 2 × 3 + 4 × 1 –11
∴ Required distance = P (2,3,1)
9 + 4 + 16
5
= units
29
∴ Option (a) is correct.
(iii) The equation of the perpendicular line drawn from the top
of the tower to the ground is given by
x–2 y–3 z–1 3x+2y+4z = 11
= =
3 2 4
∴ Option (c) is correct.

(iv) Let Q(α, β, γ)be the foot of the tower.
P (2,3,1)
∴ 3α + 2β + 4γ = 11
...(i)
Direction ratios of PQ be
α – 2, β – 3, γ – 1
a–2 b–3 c–1

= = = k (let)
3 2 4
⇒ α = 3k + 2, β = 2k + 3 and g = 4k + 1

Q (α,β,γ) 3x+2y+4z = 11

From (i), we have
3(3k + 2) + 2(2k + 3) + 4(4k + 1) = 11
–5
⇒ 29k + 16 = 11 ⇒ k =
29
–5 –15 + 58 43
∴ a= 3× +2= =
29 29 29
–5 77
b = 2 × +3=
29 29
–5 9
γ = 4k + 1 = 4 × +1=
29 29
43 77 9
∴ Coordinates of foot of perpendicular are c
, , m.
29 29 29
∴ Option (d) is correct.

(v) We have
AB = _ 3it – jt + k i – _it + 0jt + 2kti = 2it – jt – kt
AC = _it + 2jt + kti – _it + 0jt + 2kti = 0it + 2jt – kt
1
∴ Area of TABC = AB × AC ...(i)
2
it jt kt
Now, AB × AC = 2 –1 –1 = it]1 + 2g – jt]–2g + kt]4g = 3it + 2jt + 4kt
0 2 –1
∴ AB × AC = 9 + 4 + 16 = 29
From (i)
29
ar ]TABCg = × 29 =
1
sq. units
2 2
∴ Option (b) is correct.

zzz

Case Study Based Questions 557


@Cbsebookshub - Join Us on Telegram
Chapter-14: Probability
1. Read the following and answer any four questions from (i) to (v).
A coach is training 3 players. He observes that the player A can hit a target 4 times in 5 shots,

player B can hit 3 times in 4 shots and the player C can hit 2 times in 3 shots [CBSE Question Bank]

Answer the questions given below.


(i) Let the target is hit by A, B: the target is hit by B and, C: the target is hit by A and C. Then,
the probability that A, B and, C all will hit, is
4 3 2 1
(a) (b) (c) (d)
5 5 5 5
(ii) Referring to (i), what is the probability that B, C will hit and A will lose?
1 3 7 4
(a) (b) (c) (d)
10 10 10 10


(iii) With reference to the events mentioned in (i), what is the probability that 'any two' of A,
B and C will hit?
1 11 17 13
(a) (b) (c) (d)
30 30 30 30


(iv) What is the probability that ‘none of them will hit the target’?
1 1 1 2
(a) (b) (c) (d)
30 60 15 15
(v) What is the probability that at least one of A, B or C will hit the target?
59 2 3 1
(a) (b) (c) (d)
60 5 5 60
Sol. We have,
4
P(A) = Probability of hitting the target by Player A =
5
3
P(B) = Probability of hitting the target by Player B =
4
2
P(C) = Probability of hitting the target by Player C =
3
(i) Probability that A, B and C all will hit the
target = P(A) × P(B) × P(C)
4 3 2 2
= × × =
5 4 3 5
∴ Option (c) is correct.

558 Xam idea Mathematics–XII

@Cbsebookshub - Join Us on Telegram


(ii) Probability that B, C will hit and A will lose
= P(A') × P(B) × P(C)
4
= c1 – m × × = × =
3 2 1 1 1

5 4 3 5 2 10
∴ Option (a) is correct.

(iii) P(any two will hit) = P(A).P(B).P(C′) + P(A).P(C).P(B') + P(A').P(B).P(C)
4 3 1 4 2 1 1 3 2 1 2 1
= × × + × × + × × = + +
5 4 3 5 3 4 5 4 3 5 15 10
6 + 4 + 3 13
= =
30 30
∴ Option (d) is correct.

(iv) P(None of them will hit the target)
= P(A′).P(B').P(C′)
1 1 1 1
= × × =
5 4 3 60
∴ Option (b) is correct.

(v) P(at least one of A, B or C will hit the target)
= 1 – P(none will hit the target)
1 59
= 1 – =
60 60
∴ Option (a) is correct.

2. Read the following and answer any four questions from (i) to (v).
In answering a question on a multiple choice test for class XII, a student either knows the answer

or guesses. Let 3/5 be the probability that he knows the answer and 2/5 be the probability that
he guesses. Assume that a student who guesses at the answer will be correct with probability 1/3.
Let E1, E2, E be the events that the student knows the answer, guesses the answer and answers
correctly respectively. [CBSE Question Bank]

Answer the questions given below.


(i) What is the value of P(E1)?
2 1 3
(a) (b) (c) 1 (d)
5 3 5
(ii) Value of P(E|E1) is
1 2 4
(a) (b) 1 (c) (d)
3 3 15

Case Study Based Questions 559


@Cbsebookshub - Join Us on Telegram
(iii) / P _ E | Ek i P _ Ek i equals
k=2

k=1

11 4 1
(a) (b) (c) (d) 1
15 15 5
k=2
(iv) Value of / P (Ek) is

k=1
1 1 3
(a) (b) (c) 1 (d)
3 5 5
(v) What is the probability that the student knows the answer given that he answered it
correctly?
2 5 9
(a) (b) (c) (d) 3
11 3 11
Sol. It is given that E1, E2 and E be the events that the student Knows the answer, guesses the answer
and give answer correctly respectively.
3
(i) We have, probability that student knows answer = .
5
3
∴ _ E
P 1 = i
5
∴ Option (d) is correct.
(ii) We have,
P _E + E1 i P _E1 i
P _E/E1 i = = =1
P _E1 i P _E1 i
∴ Option (b) is correct.

(iii) We have,
1
P _E/E1 i = 1, P _E/E2 i =
3
3 2
P _E1 i = and P _E2 i =
5 5
/ P _E/Ek i .P _Ek i = P _E/E1 i . P _E1 i + P _E/E2 i .P _E2 i
k=2


k=1
3 1 2
+ × = 1×
5 3 5
3 2 11
= + =
5 15 15
∴ Option (a) is correct.
(iv) / P _Ek i = P _E1 i + P _E2 i
k=2

k =1
3 2 5
= + = =1
5 5 5
∴ Option (c) is correct.

P _E1 i .P _E/E1 i
(v) We have,
P _E1 /E i =
P _E1 i .P _E/E1 i + P _E2 i .P _E/E2 i
3
×1
5 3 9
= = =
3 2 1 2 11
×1+ × 3+
5 5 3 3
∴ Option (c) is correct.

zzz

560 Xam idea Mathematics–XII

@Cbsebookshub - Join Us on Telegram

You might also like